Você está na página 1de 479

Curso de Análise vol.

de Elon Lages Lima:

soluções dos exercícios

1
Rubens Rodrigues Onishi
onrubens@gmail.com

junho de 2 022
(1a versão)

1 Licenciado e mestre em Matemática, ambos pelo IME-USP.


ii
Sumário

Primeiras Palavras xv

1 Conjuntos e Funções 1
1.1 Exercício 1 . . . . . . . . . . . . . . . . . . . . . . . . . . . . . . . . . . . . . . . 1
1.2 Exercício 2 . . . . . . . . . . . . . . . . . . . . . . . . . . . . . . . . . . . . . . . 2
1.3 Exercício 3 . . . . . . . . . . . . . . . . . . . . . . . . . . . . . . . . . . . . . . . 2
1.4 Exercício 4 . . . . . . . . . . . . . . . . . . . . . . . . . . . . . . . . . . . . . . . 4
1.5 Exercício 5 . . . . . . . . . . . . . . . . . . . . . . . . . . . . . . . . . . . . . . . 5
1.6 Exercício 6 . . . . . . . . . . . . . . . . . . . . . . . . . . . . . . . . . . . . . . . 6
1.7 Exercício 7 . . . . . . . . . . . . . . . . . . . . . . . . . . . . . . . . . . . . . . . 6
1.8 Exercício 8 . . . . . . . . . . . . . . . . . . . . . . . . . . . . . . . . . . . . . . . 7
1.9 Exercício 9 . . . . . . . . . . . . . . . . . . . . . . . . . . . . . . . . . . . . . . . 7
1.10 Exercício 10 . . . . . . . . . . . . . . . . . . . . . . . . . . . . . . . . . . . . . . 8
1.11 Exercício 11 . . . . . . . . . . . . . . . . . . . . . . . . . . . . . . . . . . . . . . 10
1.12 Exercício 12 . . . . . . . . . . . . . . . . . . . . . . . . . . . . . . . . . . . . . . 11
1.13 Exercício 13 . . . . . . . . . . . . . . . . . . . . . . . . . . . . . . . . . . . . . . 11
1.14 Exercício 14 . . . . . . . . . . . . . . . . . . . . . . . . . . . . . . . . . . . . . . 12
1.15 Exercício 15 . . . . . . . . . . . . . . . . . . . . . . . . . . . . . . . . . . . . . . 12
1.16 Exercício 16 . . . . . . . . . . . . . . . . . . . . . . . . . . . . . . . . . . . . . . 13
1.17 Exercício 17 . . . . . . . . . . . . . . . . . . . . . . . . . . . . . . . . . . . . . . 13
1.18 Exercício 18 . . . . . . . . . . . . . . . . . . . . . . . . . . . . . . . . . . . . . . 13
1.19 Exercício 19 . . . . . . . . . . . . . . . . . . . . . . . . . . . . . . . . . . . . . . 18
1.20 Exercício 20 . . . . . . . . . . . . . . . . . . . . . . . . . . . . . . . . . . . . . . 20
1.21 Exercício 21 . . . . . . . . . . . . . . . . . . . . . . . . . . . . . . . . . . . . . . 21

2 Conjuntos Finitos, Enumeráveis e Não-Enumeráveis 23


2.1 Exercício 1 . . . . . . . . . . . . . . . . . . . . . . . . . . . . . . . . . . . . . . . 23
2.2 Exercício 2 . . . . . . . . . . . . . . . . . . . . . . . . . . . . . . . . . . . . . . . 24
2.3 Exercício 3 . . . . . . . . . . . . . . . . . . . . . . . . . . . . . . . . . . . . . . . 24
2.4 Exercício 4 . . . . . . . . . . . . . . . . . . . . . . . . . . . . . . . . . . . . . . . 25
2.5 Exercício 5 . . . . . . . . . . . . . . . . . . . . . . . . . . . . . . . . . . . . . . . 32
2.6 Exercício 6 . . . . . . . . . . . . . . . . . . . . . . . . . . . . . . . . . . . . . . . 32
2.7 Exercício 7 . . . . . . . . . . . . . . . . . . . . . . . . . . . . . . . . . . . . . . . 33
2.8 Exercício 8 . . . . . . . . . . . . . . . . . . . . . . . . . . . . . . . . . . . . . . . 36
2.9 Exercício 9 . . . . . . . . . . . . . . . . . . . . . . . . . . . . . . . . . . . . . . . 39
2.10 Exercício 10 . . . . . . . . . . . . . . . . . . . . . . . . . . . . . . . . . . . . . . 42
2.11 Exercício 11 . . . . . . . . . . . . . . . . . . . . . . . . . . . . . . . . . . . . . . 42
2.12 Exercício 12 . . . . . . . . . . . . . . . . . . . . . . . . . . . . . . . . . . . . . . 43

iii
iv SUMÁRIO

2.13 Exercício 13 . . . . . . . . . . . . . . . . . . . . . . . . . . . . . . . . . . . . . . 45
2.14 Exercício 14 . . . . . . . . . . . . . . . . . . . . . . . . . . . . . . . . . . . . . . 47
2.15 Exercício 15 . . . . . . . . . . . . . . . . . . . . . . . . . . . . . . . . . . . . . . 49
2.16 Exercício 16 . . . . . . . . . . . . . . . . . . . . . . . . . . . . . . . . . . . . . . 49
2.17 Exercício 17 . . . . . . . . . . . . . . . . . . . . . . . . . . . . . . . . . . . . . . 50
2.18 Exercício 18 . . . . . . . . . . . . . . . . . . . . . . . . . . . . . . . . . . . . . . 51
2.19 Exercício 19 . . . . . . . . . . . . . . . . . . . . . . . . . . . . . . . . . . . . . . 52
2.20 Exercício 20 . . . . . . . . . . . . . . . . . . . . . . . . . . . . . . . . . . . . . . 52
2.21 Exercício 21 . . . . . . . . . . . . . . . . . . . . . . . . . . . . . . . . . . . . . . 53
2.22 Exercício 22 . . . . . . . . . . . . . . . . . . . . . . . . . . . . . . . . . . . . . . 54
2.23 Exercício 23 . . . . . . . . . . . . . . . . . . . . . . . . . . . . . . . . . . . . . . 54
2.24 Exercício 24 . . . . . . . . . . . . . . . . . . . . . . . . . . . . . . . . . . . . . . 56
2.25 Exercício 25 . . . . . . . . . . . . . . . . . . . . . . . . . . . . . . . . . . . . . . 56
2.26 Exercício 26 . . . . . . . . . . . . . . . . . . . . . . . . . . . . . . . . . . . . . . 57
2.27 Exercício 27 . . . . . . . . . . . . . . . . . . . . . . . . . . . . . . . . . . . . . . 59
2.28 Exercício 28 . . . . . . . . . . . . . . . . . . . . . . . . . . . . . . . . . . . . . . 60
2.29 Exercício 29. Teorema de Cantor-Bernstein-Schröder . . . . . . . . . . . . . . . 62

3 Números Reais 65
3.1 Exercício 1 . . . . . . . . . . . . . . . . . . . . . . . . . . . . . . . . . . . . . . . 65
3.2 Exercício 2 . . . . . . . . . . . . . . . . . . . . . . . . . . . . . . . . . . . . . . . 66
3.3 Exercício 3 . . . . . . . . . . . . . . . . . . . . . . . . . . . . . . . . . . . . . . . 68
3.4 Exercício 4 . . . . . . . . . . . . . . . . . . . . . . . . . . . . . . . . . . . . . . . 68
3.5 Exercício 5 . . . . . . . . . . . . . . . . . . . . . . . . . . . . . . . . . . . . . . . 69
3.6 Exercício 6 . . . . . . . . . . . . . . . . . . . . . . . . . . . . . . . . . . . . . . . 69
3.7 Exercício 7 . . . . . . . . . . . . . . . . . . . . . . . . . . . . . . . . . . . . . . . 70
3.8 Exercício 8 . . . . . . . . . . . . . . . . . . . . . . . . . . . . . . . . . . . . . . . 70
3.9 Exercício 9 . . . . . . . . . . . . . . . . . . . . . . . . . . . . . . . . . . . . . . . 71
3.10 Exercício 10 . . . . . . . . . . . . . . . . . . . . . . . . . . . . . . . . . . . . . . 71
3.11 Exercício 11 . . . . . . . . . . . . . . . . . . . . . . . . . . . . . . . . . . . . . . 72
3.12 Exercício 12 . . . . . . . . . . . . . . . . . . . . . . . . . . . . . . . . . . . . . . 72
3.13 Exercício 13 . . . . . . . . . . . . . . . . . . . . . . . . . . . . . . . . . . . . . . 73
3.14 Exercício 14 . . . . . . . . . . . . . . . . . . . . . . . . . . . . . . . . . . . . . . 74
3.15 Exercício 15 . . . . . . . . . . . . . . . . . . . . . . . . . . . . . . . . . . . . . . 74
3.16 Exercício 16 . . . . . . . . . . . . . . . . . . . . . . . . . . . . . . . . . . . . . . 75
3.17 Exercício 17 . . . . . . . . . . . . . . . . . . . . . . . . . . . . . . . . . . . . . . 75
3.18 Exercício 18 . . . . . . . . . . . . . . . . . . . . . . . . . . . . . . . . . . . . . . 76
3.19 Exercício 19 . . . . . . . . . . . . . . . . . . . . . . . . . . . . . . . . . . . . . . 76
3.20 Exercício 20 . . . . . . . . . . . . . . . . . . . . . . . . . . . . . . . . . . . . . . 77
3.21 Exercício 21 . . . . . . . . . . . . . . . . . . . . . . . . . . . . . . . . . . . . . . 77
3.22 Exercício 22 . . . . . . . . . . . . . . . . . . . . . . . . . . . . . . . . . . . . . . 78
3.23 Exercício 23 . . . . . . . . . . . . . . . . . . . . . . . . . . . . . . . . . . . . . . 79
3.24 Exercício 24 . . . . . . . . . . . . . . . . . . . . . . . . . . . . . . . . . . . . . . 79
3.25 Exercício 25 . . . . . . . . . . . . . . . . . . . . . . . . . . . . . . . . . . . . . . 80
3.26 Exercício 26 . . . . . . . . . . . . . . . . . . . . . . . . . . . . . . . . . . . . . . 81
3.27 Exercício 27 . . . . . . . . . . . . . . . . . . . . . . . . . . . . . . . . . . . . . . 81
3.28 Exercício 28 . . . . . . . . . . . . . . . . . . . . . . . . . . . . . . . . . . . . . . 81
3.29 Exercício 29 . . . . . . . . . . . . . . . . . . . . . . . . . . . . . . . . . . . . . . 82
SUMÁRIO v

3.30 Exercício 30 . . . . . . . . . . . . . . . . . . . . . . . . . . . . . . . . . . . . . . 82
3.31 Exercício 31 . . . . . . . . . . . . . . . . . . . . . . . . . . . . . . . . . . . . . . 83
3.32 Exercício 32 . . . . . . . . . . . . . . . . . . . . . . . . . . . . . . . . . . . . . . 83
3.33 Exercício 33 . . . . . . . . . . . . . . . . . . . . . . . . . . . . . . . . . . . . . . 84
3.34 Exercício 34 . . . . . . . . . . . . . . . . . . . . . . . . . . . . . . . . . . . . . . 84
3.35 Exercício 35 . . . . . . . . . . . . . . . . . . . . . . . . . . . . . . . . . . . . . . 85
3.36 Exercício 36 . . . . . . . . . . . . . . . . . . . . . . . . . . . . . . . . . . . . . . 85
3.37 Exercício 37 . . . . . . . . . . . . . . . . . . . . . . . . . . . . . . . . . . . . . . 86
3.38 Exercício 38 . . . . . . . . . . . . . . . . . . . . . . . . . . . . . . . . . . . . . . 87
3.39 Exercício 39 . . . . . . . . . . . . . . . . . . . . . . . . . . . . . . . . . . . . . . 88
3.40 Exercício 40 . . . . . . . . . . . . . . . . . . . . . . . . . . . . . . . . . . . . . . 89
3.41 Exercício 41 . . . . . . . . . . . . . . . . . . . . . . . . . . . . . . . . . . . . . . 90
3.42 Exercício 42 . . . . . . . . . . . . . . . . . . . . . . . . . . . . . . . . . . . . . . 91
3.43 Exercício 43 . . . . . . . . . . . . . . . . . . . . . . . . . . . . . . . . . . . . . . 92
3.44 Exercício 44 . . . . . . . . . . . . . . . . . . . . . . . . . . . . . . . . . . . . . . 93
3.45 Exercício 45 . . . . . . . . . . . . . . . . . . . . . . . . . . . . . . . . . . . . . . 93
3.46 Exercício 46 . . . . . . . . . . . . . . . . . . . . . . . . . . . . . . . . . . . . . . 93
3.47 Exercício 47 . . . . . . . . . . . . . . . . . . . . . . . . . . . . . . . . . . . . . . 94
3.48 Exercício 48 . . . . . . . . . . . . . . . . . . . . . . . . . . . . . . . . . . . . . . 95
3.49 Exercício 49 . . . . . . . . . . . . . . . . . . . . . . . . . . . . . . . . . . . . . . 95
3.50 Exercício 50 . . . . . . . . . . . . . . . . . . . . . . . . . . . . . . . . . . . . . . 96
3.51 Exercício 51 . . . . . . . . . . . . . . . . . . . . . . . . . . . . . . . . . . . . . . 97
3.52 Exercício 52 . . . . . . . . . . . . . . . . . . . . . . . . . . . . . . . . . . . . . . 98
3.53 Exercício 53 . . . . . . . . . . . . . . . . . . . . . . . . . . . . . . . . . . . . . . 100
3.54 Exercício 54 . . . . . . . . . . . . . . . . . . . . . . . . . . . . . . . . . . . . . . 100
3.55 Exercício 55 . . . . . . . . . . . . . . . . . . . . . . . . . . . . . . . . . . . . . . 101
3.56 Exercício 56 . . . . . . . . . . . . . . . . . . . . . . . . . . . . . . . . . . . . . . 107
3.57 Exercício 57 . . . . . . . . . . . . . . . . . . . . . . . . . . . . . . . . . . . . . . 109
3.58 Exercício 58 . . . . . . . . . . . . . . . . . . . . . . . . . . . . . . . . . . . . . . 110
3.59 Exercício 59 . . . . . . . . . . . . . . . . . . . . . . . . . . . . . . . . . . . . . . 114
3.60 Exercício 60 . . . . . . . . . . . . . . . . . . . . . . . . . . . . . . . . . . . . . . 114

4 Sequências e Séries de Números Reais 117


4.1 Exercício 1 . . . . . . . . . . . . . . . . . . . . . . . . . . . . . . . . . . . . . . . 117
4.2 Exercício 2 . . . . . . . . . . . . . . . . . . . . . . . . . . . . . . . . . . . . . . . 117
4.3 Exercício 3 . . . . . . . . . . . . . . . . . . . . . . . . . . . . . . . . . . . . . . . 117
4.4 Exercício 4 . . . . . . . . . . . . . . . . . . . . . . . . . . . . . . . . . . . . . . . 118
4.5 Exercício 5 . . . . . . . . . . . . . . . . . . . . . . . . . . . . . . . . . . . . . . . 118
4.6 Exercício 6 . . . . . . . . . . . . . . . . . . . . . . . . . . . . . . . . . . . . . . . 122
4.7 Exercício 7 . . . . . . . . . . . . . . . . . . . . . . . . . . . . . . . . . . . . . . . 122
4.8 Exercício 8 . . . . . . . . . . . . . . . . . . . . . . . . . . . . . . . . . . . . . . . 122
4.9 Exercício 9 . . . . . . . . . . . . . . . . . . . . . . . . . . . . . . . . . . . . . . . 122
4.10 Exercício 10 . . . . . . . . . . . . . . . . . . . . . . . . . . . . . . . . . . . . . . 122
4.11 Exercício 11 . . . . . . . . . . . . . . . . . . . . . . . . . . . . . . . . . . . . . . 123
4.11.1 Exercício 11a . . . . . . . . . . . . . . . . . . . . . . . . . . . . . . . . . 124
4.12 Exercício 12 . . . . . . . . . . . . . . . . . . . . . . . . . . . . . . . . . . . . . . 124
4.13 Exercício 13 . . . . . . . . . . . . . . . . . . . . . . . . . . . . . . . . . . . . . . 126
4.14 Exercício 14 . . . . . . . . . . . . . . . . . . . . . . . . . . . . . . . . . . . . . . 127
vi SUMÁRIO

4.15 Exercício 15 . . . . . . . . . . . . . . . . . . . . . . . . . . . . . . . . . . . . . . 128


4.16 Exercício 16 . . . . . . . . . . . . . . . . . . . . . . . . . . . . . . . . . . . . . . 128
4.17 Exercício 17 . . . . . . . . . . . . . . . . . . . . . . . . . . . . . . . . . . . . . . 128
4.18 Exercício 18 . . . . . . . . . . . . . . . . . . . . . . . . . . . . . . . . . . . . . . 131
4.19 Exercício 19 . . . . . . . . . . . . . . . . . . . . . . . . . . . . . . . . . . . . . . 131
4.20 Exercício 20 . . . . . . . . . . . . . . . . . . . . . . . . . . . . . . . . . . . . . . 136
4.21 Exercício 21 . . . . . . . . . . . . . . . . . . . . . . . . . . . . . . . . . . . . . . 136
4.22 Exercício 22 . . . . . . . . . . . . . . . . . . . . . . . . . . . . . . . . . . . . . . 138
4.23 Exercício 23 . . . . . . . . . . . . . . . . . . . . . . . . . . . . . . . . . . . . . . 139
4.24 Exercício 24 . . . . . . . . . . . . . . . . . . . . . . . . . . . . . . . . . . . . . . 139
4.25 Exercício 25 . . . . . . . . . . . . . . . . . . . . . . . . . . . . . . . . . . . . . . 140
4.26 Exercício 26 . . . . . . . . . . . . . . . . . . . . . . . . . . . . . . . . . . . . . . 141
4.27 Exercício 27 . . . . . . . . . . . . . . . . . . . . . . . . . . . . . . . . . . . . . . 142
4.28 Exercício 28 . . . . . . . . . . . . . . . . . . . . . . . . . . . . . . . . . . . . . . 142
4.29 Exercício 29 . . . . . . . . . . . . . . . . . . . . . . . . . . . . . . . . . . . . . . 144
4.30 Exercício 30 . . . . . . . . . . . . . . . . . . . . . . . . . . . . . . . . . . . . . . 145
4.31 Exercício 31 . . . . . . . . . . . . . . . . . . . . . . . . . . . . . . . . . . . . . . 145
4.32 Exercício 32 . . . . . . . . . . . . . . . . . . . . . . . . . . . . . . . . . . . . . . 146
4.33 Exercício 33 . . . . . . . . . . . . . . . . . . . . . . . . . . . . . . . . . . . . . . 147
4.34 Exercício 34 . . . . . . . . . . . . . . . . . . . . . . . . . . . . . . . . . . . . . . 148
4.35 Exercício 35 . . . . . . . . . . . . . . . . . . . . . . . . . . . . . . . . . . . . . . 148
4.36 Exercício 36 . . . . . . . . . . . . . . . . . . . . . . . . . . . . . . . . . . . . . . 149
4.37 Exercício 37 . . . . . . . . . . . . . . . . . . . . . . . . . . . . . . . . . . . . . . 149
4.38 Exercício 38 . . . . . . . . . . . . . . . . . . . . . . . . . . . . . . . . . . . . . . 150
4.39 Exercício 39 . . . . . . . . . . . . . . . . . . . . . . . . . . . . . . . . . . . . . . 151
4.40 Exercício 40 . . . . . . . . . . . . . . . . . . . . . . . . . . . . . . . . . . . . . . 151
4.41 Exercício 41 . . . . . . . . . . . . . . . . . . . . . . . . . . . . . . . . . . . . . . 151
4.42 Exercício 42 . . . . . . . . . . . . . . . . . . . . . . . . . . . . . . . . . . . . . . 152
4.43 Exercício 43 . . . . . . . . . . . . . . . . . . . . . . . . . . . . . . . . . . . . . . 152
4.44 Exercício 44 . . . . . . . . . . . . . . . . . . . . . . . . . . . . . . . . . . . . . . 153
4.45 Exercício 45 . . . . . . . . . . . . . . . . . . . . . . . . . . . . . . . . . . . . . . 153
4.46 Exercício 46 . . . . . . . . . . . . . . . . . . . . . . . . . . . . . . . . . . . . . . 155
4.47 Exercício 47 . . . . . . . . . . . . . . . . . . . . . . . . . . . . . . . . . . . . . . 156
4.48 Exercício 48 . . . . . . . . . . . . . . . . . . . . . . . . . . . . . . . . . . . . . . 156
4.49 Exercício 49 . . . . . . . . . . . . . . . . . . . . . . . . . . . . . . . . . . . . . . 157

5 Topologia da Reta 159


5.1 Exercício 1 . . . . . . . . . . . . . . . . . . . . . . . . . . . . . . . . . . . . . . . 159
5.2 Exercício 2 . . . . . . . . . . . . . . . . . . . . . . . . . . . . . . . . . . . . . . . 159
5.3 Exercício 3 . . . . . . . . . . . . . . . . . . . . . . . . . . . . . . . . . . . . . . . 160
5.4 Exercício 4 . . . . . . . . . . . . . . . . . . . . . . . . . . . . . . . . . . . . . . . 160
5.5 Exercício 5 . . . . . . . . . . . . . . . . . . . . . . . . . . . . . . . . . . . . . . . 161
5.6 Exercício 6 . . . . . . . . . . . . . . . . . . . . . . . . . . . . . . . . . . . . . . . 162
5.7 Exercício 7 . . . . . . . . . . . . . . . . . . . . . . . . . . . . . . . . . . . . . . . 162
5.8 Exercício 8 . . . . . . . . . . . . . . . . . . . . . . . . . . . . . . . . . . . . . . . 162
5.9 Exercício 9 . . . . . . . . . . . . . . . . . . . . . . . . . . . . . . . . . . . . . . . 163
5.10 Exercício 10 . . . . . . . . . . . . . . . . . . . . . . . . . . . . . . . . . . . . . . 163
5.11 Exercício 11 . . . . . . . . . . . . . . . . . . . . . . . . . . . . . . . . . . . . . . 164
SUMÁRIO vii

5.12 Exercício 12 . . . . . . . . . . . . . . . . . . . . . . . . . . . . . . . . . . . . . . 164


5.13 Exercício 13 . . . . . . . . . . . . . . . . . . . . . . . . . . . . . . . . . . . . . . 164
5.14 Exercício 14 . . . . . . . . . . . . . . . . . . . . . . . . . . . . . . . . . . . . . . 166
5.15 Exercício 15 . . . . . . . . . . . . . . . . . . . . . . . . . . . . . . . . . . . . . . 167
5.16 Exercício 16 . . . . . . . . . . . . . . . . . . . . . . . . . . . . . . . . . . . . . . 167
5.17 Exercício 17 . . . . . . . . . . . . . . . . . . . . . . . . . . . . . . . . . . . . . . 168
5.18 Exercício 18 . . . . . . . . . . . . . . . . . . . . . . . . . . . . . . . . . . . . . . 170
5.19 Exercício 19 . . . . . . . . . . . . . . . . . . . . . . . . . . . . . . . . . . . . . . 172
5.20 Exercício 20 . . . . . . . . . . . . . . . . . . . . . . . . . . . . . . . . . . . . . . 173
5.21 Exercício 21 . . . . . . . . . . . . . . . . . . . . . . . . . . . . . . . . . . . . . . 173
5.22 Exercício 22 . . . . . . . . . . . . . . . . . . . . . . . . . . . . . . . . . . . . . . 174
5.23 Exercício 23 . . . . . . . . . . . . . . . . . . . . . . . . . . . . . . . . . . . . . . 174
5.24 Exercício 24 . . . . . . . . . . . . . . . . . . . . . . . . . . . . . . . . . . . . . . 175
5.25 Exercício 25 . . . . . . . . . . . . . . . . . . . . . . . . . . . . . . . . . . . . . . 175
5.26 Exercício 26 . . . . . . . . . . . . . . . . . . . . . . . . . . . . . . . . . . . . . . 176
5.27 Exercício 27 . . . . . . . . . . . . . . . . . . . . . . . . . . . . . . . . . . . . . . 176
5.28 Exercício 28 . . . . . . . . . . . . . . . . . . . . . . . . . . . . . . . . . . . . . . 176
5.29 Exercício 29 . . . . . . . . . . . . . . . . . . . . . . . . . . . . . . . . . . . . . . 177
5.30 Exercício 30 . . . . . . . . . . . . . . . . . . . . . . . . . . . . . . . . . . . . . . 178
5.31 Exercício 31 . . . . . . . . . . . . . . . . . . . . . . . . . . . . . . . . . . . . . . 179
5.32 Exercício 32 . . . . . . . . . . . . . . . . . . . . . . . . . . . . . . . . . . . . . . 179
5.33 Exercício 33 . . . . . . . . . . . . . . . . . . . . . . . . . . . . . . . . . . . . . . 179
5.34 Exercício 34 . . . . . . . . . . . . . . . . . . . . . . . . . . . . . . . . . . . . . . 180
5.35 Exercício 35 . . . . . . . . . . . . . . . . . . . . . . . . . . . . . . . . . . . . . . 180
5.36 Exercício 36 . . . . . . . . . . . . . . . . . . . . . . . . . . . . . . . . . . . . . . 180
5.37 Exercício 37 . . . . . . . . . . . . . . . . . . . . . . . . . . . . . . . . . . . . . . 181
5.38 Exercício 38 . . . . . . . . . . . . . . . . . . . . . . . . . . . . . . . . . . . . . . 181
5.39 Exercício 39 . . . . . . . . . . . . . . . . . . . . . . . . . . . . . . . . . . . . . . 182
5.40 Exercício 40 . . . . . . . . . . . . . . . . . . . . . . . . . . . . . . . . . . . . . . 182
5.41 Exercício 41 . . . . . . . . . . . . . . . . . . . . . . . . . . . . . . . . . . . . . . 182
5.42 Exercício 42 . . . . . . . . . . . . . . . . . . . . . . . . . . . . . . . . . . . . . . 183
5.43 Exercício 43 . . . . . . . . . . . . . . . . . . . . . . . . . . . . . . . . . . . . . . 184
5.44 Exercício 44. Teorema de Lindelöf . . . . . . . . . . . . . . . . . . . . . . . . . . 185
5.45 Exercício 45 . . . . . . . . . . . . . . . . . . . . . . . . . . . . . . . . . . . . . . 187
5.46 Exercício 46 . . . . . . . . . . . . . . . . . . . . . . . . . . . . . . . . . . . . . . 188
5.47 Exercício 47 . . . . . . . . . . . . . . . . . . . . . . . . . . . . . . . . . . . . . . 188
5.48 Exercício 48 . . . . . . . . . . . . . . . . . . . . . . . . . . . . . . . . . . . . . . 189
5.49 Exercício 49 . . . . . . . . . . . . . . . . . . . . . . . . . . . . . . . . . . . . . . 190
5.50 Exercício 50 . . . . . . . . . . . . . . . . . . . . . . . . . . . . . . . . . . . . . . 191
5.51 Exercício 51 . . . . . . . . . . . . . . . . . . . . . . . . . . . . . . . . . . . . . . 192
5.52 Exercício 52 . . . . . . . . . . . . . . . . . . . . . . . . . . . . . . . . . . . . . . 192
5.53 Exercício 53 . . . . . . . . . . . . . . . . . . . . . . . . . . . . . . . . . . . . . . 193
5.54 Exercício 54. Teorema de Baire . . . . . . . . . . . . . . . . . . . . . . . . . . . 194
5.55 Exercício 55. R−Q não é Fσ e Q não é Gδ . . . . . . . . . . . . . . . . . . . . . 195
5.56 Exercício 56 . . . . . . . . . . . . . . . . . . . . . . . . . . . . . . . . . . . . . . 196
5.57 Exercício 57 . . . . . . . . . . . . . . . . . . . . . . . . . . . . . . . . . . . . . . 198
5.58 Exercício 58 . . . . . . . . . . . . . . . . . . . . . . . . . . . . . . . . . . . . . . 198
5.59 Exercício 59 . . . . . . . . . . . . . . . . . . . . . . . . . . . . . . . . . . . . . . 198
viii SUMÁRIO

5.60 Exercício 60 . . . . . . . . . . . . . . . . . . . . . . . . . . . . . . . . . . . . . . 199


5.61 Exercício 61 . . . . . . . . . . . . . . . . . . . . . . . . . . . . . . . . . . . . . . 200
5.62 Exercício 62 . . . . . . . . . . . . . . . . . . . . . . . . . . . . . . . . . . . . . . 202
5.63 Exercício 63 . . . . . . . . . . . . . . . . . . . . . . . . . . . . . . . . . . . . . . 202
5.64 Exercício 64. Teorema de Bendixson . . . . . . . . . . . . . . . . . . . . . . . . 203

6 Limites de Funções 205


6.1 Exercício 1 . . . . . . . . . . . . . . . . . . . . . . . . . . . . . . . . . . . . . . . 205
6.2 Exercício 2 . . . . . . . . . . . . . . . . . . . . . . . . . . . . . . . . . . . . . . . 205
6.3 Exercício 3 . . . . . . . . . . . . . . . . . . . . . . . . . . . . . . . . . . . . . . . 206
6.4 Exercício 4 . . . . . . . . . . . . . . . . . . . . . . . . . . . . . . . . . . . . . . . 206
6.5 Exercício 5 . . . . . . . . . . . . . . . . . . . . . . . . . . . . . . . . . . . . . . . 208
6.6 Exercício 6 . . . . . . . . . . . . . . . . . . . . . . . . . . . . . . . . . . . . . . . 208
6.7 Exercício 7 . . . . . . . . . . . . . . . . . . . . . . . . . . . . . . . . . . . . . . . 209
6.8 Exercício 8 . . . . . . . . . . . . . . . . . . . . . . . . . . . . . . . . . . . . . . . 210
6.9 Exercício 9 . . . . . . . . . . . . . . . . . . . . . . . . . . . . . . . . . . . . . . . 211
6.10 Exercício 10 . . . . . . . . . . . . . . . . . . . . . . . . . . . . . . . . . . . . . . 212
6.11 Exercício 11 . . . . . . . . . . . . . . . . . . . . . . . . . . . . . . . . . . . . . . 214
6.12 Exercício 12 . . . . . . . . . . . . . . . . . . . . . . . . . . . . . . . . . . . . . . 216
6.13 Exercício 13 . . . . . . . . . . . . . . . . . . . . . . . . . . . . . . . . . . . . . . 217
6.14 Exercício 14 . . . . . . . . . . . . . . . . . . . . . . . . . . . . . . . . . . . . . . 218
6.15 Exercício 15 . . . . . . . . . . . . . . . . . . . . . . . . . . . . . . . . . . . . . . 219
6.16 Exercício 16 . . . . . . . . . . . . . . . . . . . . . . . . . . . . . . . . . . . . . . 220
6.17 Exercício 17 . . . . . . . . . . . . . . . . . . . . . . . . . . . . . . . . . . . . . . 221
6.18 Exercício 18 . . . . . . . . . . . . . . . . . . . . . . . . . . . . . . . . . . . . . . 221
6.19 Exercício 19 . . . . . . . . . . . . . . . . . . . . . . . . . . . . . . . . . . . . . . 222
6.20 Exercício 20 . . . . . . . . . . . . . . . . . . . . . . . . . . . . . . . . . . . . . . 224
6.21 Exercício 21 . . . . . . . . . . . . . . . . . . . . . . . . . . . . . . . . . . . . . . 224
6.22 Exercício 22 . . . . . . . . . . . . . . . . . . . . . . . . . . . . . . . . . . . . . . 225
6.23 Exercício 23 . . . . . . . . . . . . . . . . . . . . . . . . . . . . . . . . . . . . . . 229
6.24 Exercício 24 . . . . . . . . . . . . . . . . . . . . . . . . . . . . . . . . . . . . . . 230

7 Funções Contínuas 231


7.1 Exercício 1 . . . . . . . . . . . . . . . . . . . . . . . . . . . . . . . . . . . . . . . 231
7.2 Exercício 2 . . . . . . . . . . . . . . . . . . . . . . . . . . . . . . . . . . . . . . . 231
7.3 Exercício 3 . . . . . . . . . . . . . . . . . . . . . . . . . . . . . . . . . . . . . . . 232
7.4 Exercício 4 . . . . . . . . . . . . . . . . . . . . . . . . . . . . . . . . . . . . . . . 233
7.5 Exercício 5 . . . . . . . . . . . . . . . . . . . . . . . . . . . . . . . . . . . . . . . 233
7.6 Exercício 6 . . . . . . . . . . . . . . . . . . . . . . . . . . . . . . . . . . . . . . . 233
7.7 Exercício 7 . . . . . . . . . . . . . . . . . . . . . . . . . . . . . . . . . . . . . . . 234
7.8 Exercício 8 . . . . . . . . . . . . . . . . . . . . . . . . . . . . . . . . . . . . . . . 234
7.9 Exercício 9 . . . . . . . . . . . . . . . . . . . . . . . . . . . . . . . . . . . . . . . 235
7.10 Exercício 10 . . . . . . . . . . . . . . . . . . . . . . . . . . . . . . . . . . . . . . 235
7.11 Exercício 11 . . . . . . . . . . . . . . . . . . . . . . . . . . . . . . . . . . . . . . 237
7.12 Exercício 12 . . . . . . . . . . . . . . . . . . . . . . . . . . . . . . . . . . . . . . 237
7.13 Exercício 13 . . . . . . . . . . . . . . . . . . . . . . . . . . . . . . . . . . . . . . 237
7.14 Exercício 14 . . . . . . . . . . . . . . . . . . . . . . . . . . . . . . . . . . . . . . 239
7.15 Exercício 15 . . . . . . . . . . . . . . . . . . . . . . . . . . . . . . . . . . . . . . 240
SUMÁRIO ix

7.16 Exercício 16 . . . . . . . . . . . . . . . . . . . . . . . . . . . . . . . . . . . . . . 240


7.17 Exercício 17 . . . . . . . . . . . . . . . . . . . . . . . . . . . . . . . . . . . . . . 242
7.18 Exercício 18 . . . . . . . . . . . . . . . . . . . . . . . . . . . . . . . . . . . . . . 243
7.19 Exercício 19 . . . . . . . . . . . . . . . . . . . . . . . . . . . . . . . . . . . . . . 244
7.20 Exercício 20 . . . . . . . . . . . . . . . . . . . . . . . . . . . . . . . . . . . . . . 245
7.21 Exercício 21 . . . . . . . . . . . . . . . . . . . . . . . . . . . . . . . . . . . . . . 247
7.22 Exercício 22. Teorema de Brouwer em dimensão 1 . . . . . . . . . . . . . . . . . 249
7.23 Exercício 23 . . . . . . . . . . . . . . . . . . . . . . . . . . . . . . . . . . . . . . 249
7.24 Exercício 24 . . . . . . . . . . . . . . . . . . . . . . . . . . . . . . . . . . . . . . 250
7.25 Exercício 25 . . . . . . . . . . . . . . . . . . . . . . . . . . . . . . . . . . . . . . 251
7.26 Exercício 26 . . . . . . . . . . . . . . . . . . . . . . . . . . . . . . . . . . . . . . 251
7.27 Exercício 27 . . . . . . . . . . . . . . . . . . . . . . . . . . . . . . . . . . . . . . 252
7.28 Exercício 28 . . . . . . . . . . . . . . . . . . . . . . . . . . . . . . . . . . . . . . 253
7.29 Exercício 29 . . . . . . . . . . . . . . . . . . . . . . . . . . . . . . . . . . . . . . 253
7.30 Exercício 30 . . . . . . . . . . . . . . . . . . . . . . . . . . . . . . . . . . . . . . 254
7.31 Exercício 31 . . . . . . . . . . . . . . . . . . . . . . . . . . . . . . . . . . . . . . 256
7.32 Exercício 32 . . . . . . . . . . . . . . . . . . . . . . . . . . . . . . . . . . . . . . 259
7.33 Exercício 33 . . . . . . . . . . . . . . . . . . . . . . . . . . . . . . . . . . . . . . 260
7.34 Exercício 34 . . . . . . . . . . . . . . . . . . . . . . . . . . . . . . . . . . . . . . 262
7.35 Exercício 35 . . . . . . . . . . . . . . . . . . . . . . . . . . . . . . . . . . . . . . 263
7.36 Exercício 36 . . . . . . . . . . . . . . . . . . . . . . . . . . . . . . . . . . . . . . 263
7.37 Exercício 37 . . . . . . . . . . . . . . . . . . . . . . . . . . . . . . . . . . . . . . 264
7.38 Exercício 38 . . . . . . . . . . . . . . . . . . . . . . . . . . . . . . . . . . . . . . 265
7.39 Exercício 39 . . . . . . . . . . . . . . . . . . . . . . . . . . . . . . . . . . . . . . 266
7.40 Exercício 40 . . . . . . . . . . . . . . . . . . . . . . . . . . . . . . . . . . . . . . 267
7.41 Exercício 41 . . . . . . . . . . . . . . . . . . . . . . . . . . . . . . . . . . . . . . 267
7.42 Exercício 42 . . . . . . . . . . . . . . . . . . . . . . . . . . . . . . . . . . . . . . 268
7.43 Exercício 43 . . . . . . . . . . . . . . . . . . . . . . . . . . . . . . . . . . . . . . 269
7.44 Exercício 44 . . . . . . . . . . . . . . . . . . . . . . . . . . . . . . . . . . . . . . 270
7.45 Exercício 45 . . . . . . . . . . . . . . . . . . . . . . . . . . . . . . . . . . . . . . 270
7.46 Exercício 46 . . . . . . . . . . . . . . . . . . . . . . . . . . . . . . . . . . . . . . 271
7.47 Exercício 47 . . . . . . . . . . . . . . . . . . . . . . . . . . . . . . . . . . . . . . 272

8 Derivadas 283
8.1 Exercício 1 . . . . . . . . . . . . . . . . . . . . . . . . . . . . . . . . . . . . . . . 283
8.2 Exercício 2 . . . . . . . . . . . . . . . . . . . . . . . . . . . . . . . . . . . . . . . 283
8.3 Exercício 3 . . . . . . . . . . . . . . . . . . . . . . . . . . . . . . . . . . . . . . . 284
8.4 Exercício 4 . . . . . . . . . . . . . . . . . . . . . . . . . . . . . . . . . . . . . . . 284
8.5 Exercício 5 . . . . . . . . . . . . . . . . . . . . . . . . . . . . . . . . . . . . . . . 285
8.6 Exercício 6 . . . . . . . . . . . . . . . . . . . . . . . . . . . . . . . . . . . . . . . 285
8.7 Exercício 7 . . . . . . . . . . . . . . . . . . . . . . . . . . . . . . . . . . . . . . . 287
8.8 Exercício 8 . . . . . . . . . . . . . . . . . . . . . . . . . . . . . . . . . . . . . . . 288
8.9 Exercício 9 . . . . . . . . . . . . . . . . . . . . . . . . . . . . . . . . . . . . . . . 288
8.10 Exercício 10 . . . . . . . . . . . . . . . . . . . . . . . . . . . . . . . . . . . . . . 290
8.11 Exercício 11 . . . . . . . . . . . . . . . . . . . . . . . . . . . . . . . . . . . . . . 290
8.12 Exercício 12 . . . . . . . . . . . . . . . . . . . . . . . . . . . . . . . . . . . . . . 294
8.13 Exercício 13 . . . . . . . . . . . . . . . . . . . . . . . . . . . . . . . . . . . . . . 294
8.14 Exercício 14 . . . . . . . . . . . . . . . . . . . . . . . . . . . . . . . . . . . . . . 295
x SUMÁRIO

8.15 Exercício 15 . . . . . . . . . . . . . . . . . . . . . . . . . . . . . . . . . . . . . . 296


8.16 Exercício 16 . . . . . . . . . . . . . . . . . . . . . . . . . . . . . . . . . . . . . . 296
8.17 Exercício 17 . . . . . . . . . . . . . . . . . . . . . . . . . . . . . . . . . . . . . . 297
8.18 Exercício 18 . . . . . . . . . . . . . . . . . . . . . . . . . . . . . . . . . . . . . . 297
8.19 Exercício 19 . . . . . . . . . . . . . . . . . . . . . . . . . . . . . . . . . . . . . . 298
8.20 Exercício 20 . . . . . . . . . . . . . . . . . . . . . . . . . . . . . . . . . . . . . . 298
8.21 Exercício 21 . . . . . . . . . . . . . . . . . . . . . . . . . . . . . . . . . . . . . . 299
8.22 Exercício 22 . . . . . . . . . . . . . . . . . . . . . . . . . . . . . . . . . . . . . . 300
8.23 Exercício 23 . . . . . . . . . . . . . . . . . . . . . . . . . . . . . . . . . . . . . . 302
8.24 Exercício 24 . . . . . . . . . . . . . . . . . . . . . . . . . . . . . . . . . . . . . . 303
8.25 Exercício 25. Teorema do Ponto Fixo de Banach . . . . . . . . . . . . . . . . . . 304
8.26 Exercício 26 . . . . . . . . . . . . . . . . . . . . . . . . . . . . . . . . . . . . . . 305
8.27 Exercício 27 . . . . . . . . . . . . . . . . . . . . . . . . . . . . . . . . . . . . . . 305
8.28 Exercício 28 . . . . . . . . . . . . . . . . . . . . . . . . . . . . . . . . . . . . . . 306
8.29 Exercício 29 . . . . . . . . . . . . . . . . . . . . . . . . . . . . . . . . . . . . . . 308
8.30 Exercício 30 . . . . . . . . . . . . . . . . . . . . . . . . . . . . . . . . . . . . . . 308
8.31 Exercício 31 . . . . . . . . . . . . . . . . . . . . . . . . . . . . . . . . . . . . . . 309
8.32 Exercício 32 . . . . . . . . . . . . . . . . . . . . . . . . . . . . . . . . . . . . . . 309
8.33 Exercício 33 . . . . . . . . . . . . . . . . . . . . . . . . . . . . . . . . . . . . . . 311
8.34 Exercício 34 . . . . . . . . . . . . . . . . . . . . . . . . . . . . . . . . . . . . . . 313
8.35 Exercício 35 . . . . . . . . . . . . . . . . . . . . . . . . . . . . . . . . . . . . . . 314
8.36 Exercício 36 . . . . . . . . . . . . . . . . . . . . . . . . . . . . . . . . . . . . . . 316
8.37 Exercício 37 . . . . . . . . . . . . . . . . . . . . . . . . . . . . . . . . . . . . . . 318
8.38 Exercício 38 . . . . . . . . . . . . . . . . . . . . . . . . . . . . . . . . . . . . . . 319
8.39 Exercício 39 . . . . . . . . . . . . . . . . . . . . . . . . . . . . . . . . . . . . . . 319
8.40 Exercício 40 . . . . . . . . . . . . . . . . . . . . . . . . . . . . . . . . . . . . . . 320
8.41 Exercício 41 . . . . . . . . . . . . . . . . . . . . . . . . . . . . . . . . . . . . . . 321
8.42 Exercício 42 . . . . . . . . . . . . . . . . . . . . . . . . . . . . . . . . . . . . . . 322
8.43 Exercício 43 . . . . . . . . . . . . . . . . . . . . . . . . . . . . . . . . . . . . . . 323
8.44 Exercício 44 . . . . . . . . . . . . . . . . . . . . . . . . . . . . . . . . . . . . . . 326
8.45 Exercício 45 . . . . . . . . . . . . . . . . . . . . . . . . . . . . . . . . . . . . . . 327
8.46 Exercício 46 . . . . . . . . . . . . . . . . . . . . . . . . . . . . . . . . . . . . . . 329
8.47 Exercício 47 . . . . . . . . . . . . . . . . . . . . . . . . . . . . . . . . . . . . . . 330
8.48 Exercício 48 . . . . . . . . . . . . . . . . . . . . . . . . . . . . . . . . . . . . . . 332
8.49 Exercício 49 . . . . . . . . . . . . . . . . . . . . . . . . . . . . . . . . . . . . . . 333
8.50 Exercício 50 . . . . . . . . . . . . . . . . . . . . . . . . . . . . . . . . . . . . . . 334
8.51 Exercício 51 . . . . . . . . . . . . . . . . . . . . . . . . . . . . . . . . . . . . . . 334
8.52 Exercício 52 . . . . . . . . . . . . . . . . . . . . . . . . . . . . . . . . . . . . . . 336
8.53 Exercício 53 . . . . . . . . . . . . . . . . . . . . . . . . . . . . . . . . . . . . . . 336
8.54 Exercício 54 . . . . . . . . . . . . . . . . . . . . . . . . . . . . . . . . . . . . . . 337
8.55 Exercício 55 . . . . . . . . . . . . . . . . . . . . . . . . . . . . . . . . . . . . . . 337

9 Integral de Riemann 341


9.1 Exercício 1 . . . . . . . . . . . . . . . . . . . . . . . . . . . . . . . . . . . . . . . 341
9.2 Exercício 2 . . . . . . . . . . . . . . . . . . . . . . . . . . . . . . . . . . . . . . . 342
9.3 Exercício 3 . . . . . . . . . . . . . . . . . . . . . . . . . . . . . . . . . . . . . . . 343
9.4 Exercício 4 . . . . . . . . . . . . . . . . . . . . . . . . . . . . . . . . . . . . . . . 344
9.5 Exercício 5 . . . . . . . . . . . . . . . . . . . . . . . . . . . . . . . . . . . . . . . 344
SUMÁRIO xi

9.6 Exercício 6 . . . . . . . . . . . . . . . . . . . . . . . . . . . . . . . . . . . . . . . 345


9.7 Exercício 7 . . . . . . . . . . . . . . . . . . . . . . . . . . . . . . . . . . . . . . . 347
9.8 Exercício 8 . . . . . . . . . . . . . . . . . . . . . . . . . . . . . . . . . . . . . . . 348
9.9 Exercício 9 . . . . . . . . . . . . . . . . . . . . . . . . . . . . . . . . . . . . . . . 349
9.10 Exercício 10 . . . . . . . . . . . . . . . . . . . . . . . . . . . . . . . . . . . . . . 351
9.11 Exercício 11. Mudança de variável . . . . . . . . . . . . . . . . . . . . . . . . . . 353
9.12 Exercício 12 . . . . . . . . . . . . . . . . . . . . . . . . . . . . . . . . . . . . . . 358
9.13 Exercício 13 . . . . . . . . . . . . . . . . . . . . . . . . . . . . . . . . . . . . . . 358
9.14 Exercício 14 . . . . . . . . . . . . . . . . . . . . . . . . . . . . . . . . . . . . . . 360
9.14.1 Exercício 14a . . . . . . . . . . . . . . . . . . . . . . . . . . . . . . . . . 360
9.15 Exercício 15 . . . . . . . . . . . . . . . . . . . . . . . . . . . . . . . . . . . . . . 365
9.16 Exercício 16 . . . . . . . . . . . . . . . . . . . . . . . . . . . . . . . . . . . . . . 369
9.17 Exercício 17 . . . . . . . . . . . . . . . . . . . . . . . . . . . . . . . . . . . . . . 371
9.18 Exercício 18 . . . . . . . . . . . . . . . . . . . . . . . . . . . . . . . . . . . . . . 372
9.19 Exercício 19 . . . . . . . . . . . . . . . . . . . . . . . . . . . . . . . . . . . . . . 374
9.20 Exercício 20 . . . . . . . . . . . . . . . . . . . . . . . . . . . . . . . . . . . . . . 375
9.21 Exercício 21 . . . . . . . . . . . . . . . . . . . . . . . . . . . . . . . . . . . . . . 377
9.22 Exercício 22 . . . . . . . . . . . . . . . . . . . . . . . . . . . . . . . . . . . . . . 377
9.23 Exercício 23 . . . . . . . . . . . . . . . . . . . . . . . . . . . . . . . . . . . . . . 378
9.24 Exercício 24 . . . . . . . . . . . . . . . . . . . . . . . . . . . . . . . . . . . . . . 378
9.25 Exercício 25 . . . . . . . . . . . . . . . . . . . . . . . . . . . . . . . . . . . . . . 379
9.26 Exercício 26 . . . . . . . . . . . . . . . . . . . . . . . . . . . . . . . . . . . . . . 381
9.27 Exercício 27 . . . . . . . . . . . . . . . . . . . . . . . . . . . . . . . . . . . . . . 382
9.28 Exercício 28 . . . . . . . . . . . . . . . . . . . . . . . . . . . . . . . . . . . . . . 384
9.29 Exercício 29 . . . . . . . . . . . . . . . . . . . . . . . . . . . . . . . . . . . . . . 385
9.30 Exercício 30 . . . . . . . . . . . . . . . . . . . . . . . . . . . . . . . . . . . . . . 385
9.31 Exercício 31 . . . . . . . . . . . . . . . . . . . . . . . . . . . . . . . . . . . . . . 389
9.32 Exercício 32 . . . . . . . . . . . . . . . . . . . . . . . . . . . . . . . . . . . . . . 389
9.33 Exercício 33 . . . . . . . . . . . . . . . . . . . . . . . . . . . . . . . . . . . . . . 393
9.34 Exercício 34 . . . . . . . . . . . . . . . . . . . . . . . . . . . . . . . . . . . . . . 394
9.35 Exercício 35 . . . . . . . . . . . . . . . . . . . . . . . . . . . . . . . . . . . . . . 396
9.36 Exercício 36 . . . . . . . . . . . . . . . . . . . . . . . . . . . . . . . . . . . . . . 397
9.37 Exercício 37 . . . . . . . . . . . . . . . . . . . . . . . . . . . . . . . . . . . . . . 398
9.38 Exercício 38 . . . . . . . . . . . . . . . . . . . . . . . . . . . . . . . . . . . . . . 398
9.39 Exercício 39 . . . . . . . . . . . . . . . . . . . . . . . . . . . . . . . . . . . . . . 400
9.40 Exercício 40 . . . . . . . . . . . . . . . . . . . . . . . . . . . . . . . . . . . . . . 400
9.41 Exercício 41 . . . . . . . . . . . . . . . . . . . . . . . . . . . . . . . . . . . . . . 401
9.42 Exercício 42 . . . . . . . . . . . . . . . . . . . . . . . . . . . . . . . . . . . . . . 402
9.43 Exercício 43 . . . . . . . . . . . . . . . . . . . . . . . . . . . . . . . . . . . . . . 403
9.44 Exercício 44 . . . . . . . . . . . . . . . . . . . . . . . . . . . . . . . . . . . . . . 403
9.45 Exercício 45. Conjunto de Cantor com medida positiva . . . . . . . . . . . . . . 404

10 Sequências e Séries de Funções 407


10.1 Exercício 1 . . . . . . . . . . . . . . . . . . . . . . . . . . . . . . . . . . . . . . . 407
10.2 Exercício 2 . . . . . . . . . . . . . . . . . . . . . . . . . . . . . . . . . . . . . . . 408
10.3 Exercício 3 . . . . . . . . . . . . . . . . . . . . . . . . . . . . . . . . . . . . . . . 409
10.4 Exercício 4 . . . . . . . . . . . . . . . . . . . . . . . . . . . . . . . . . . . . . . . 410
10.5 Exercício 5 . . . . . . . . . . . . . . . . . . . . . . . . . . . . . . . . . . . . . . . 411
xii SUMÁRIO

10.6 Exercício 6 . . . . . . . . . . . . . . . . . . . . . . . . . . . . . . . . . . . . . . . 411


10.7 Exercício 7 . . . . . . . . . . . . . . . . . . . . . . . . . . . . . . . . . . . . . . . 412
10.8 Exercício 8 . . . . . . . . . . . . . . . . . . . . . . . . . . . . . . . . . . . . . . . 413
10.9 Exercício 9 . . . . . . . . . . . . . . . . . . . . . . . . . . . . . . . . . . . . . . . 414
10.10Exercício 10 . . . . . . . . . . . . . . . . . . . . . . . . . . . . . . . . . . . . . . 415
10.11Exercício 11 . . . . . . . . . . . . . . . . . . . . . . . . . . . . . . . . . . . . . . 416
10.12Exercício 12 . . . . . . . . . . . . . . . . . . . . . . . . . . . . . . . . . . . . . . 416
10.13Exercício 13 . . . . . . . . . . . . . . . . . . . . . . . . . . . . . . . . . . . . . . 417
10.14Exercício 14 . . . . . . . . . . . . . . . . . . . . . . . . . . . . . . . . . . . . . . 420
10.15Exercício 15 . . . . . . . . . . . . . . . . . . . . . . . . . . . . . . . . . . . . . . 420
10.16Exercício 16 . . . . . . . . . . . . . . . . . . . . . . . . . . . . . . . . . . . . . . 421
10.17Exercício 17 . . . . . . . . . . . . . . . . . . . . . . . . . . . . . . . . . . . . . . 421
10.18Exercício 18 . . . . . . . . . . . . . . . . . . . . . . . . . . . . . . . . . . . . . . 422
10.19Exercício 19 . . . . . . . . . . . . . . . . . . . . . . . . . . . . . . . . . . . . . . 422
10.20Exercício 20 . . . . . . . . . . . . . . . . . . . . . . . . . . . . . . . . . . . . . . 422
10.21Exercício 21 . . . . . . . . . . . . . . . . . . . . . . . . . . . . . . . . . . . . . . 423
10.22Exercício 22 . . . . . . . . . . . . . . . . . . . . . . . . . . . . . . . . . . . . . . 424
10.23Exercício 23 . . . . . . . . . . . . . . . . . . . . . . . . . . . . . . . . . . . . . . 425
10.24Exercício 24 . . . . . . . . . . . . . . . . . . . . . . . . . . . . . . . . . . . . . . 425
10.25Exercício 25 . . . . . . . . . . . . . . . . . . . . . . . . . . . . . . . . . . . . . . 427
10.26Exercício 26 . . . . . . . . . . . . . . . . . . . . . . . . . . . . . . . . . . . . . . 428
10.27Exercício 27 . . . . . . . . . . . . . . . . . . . . . . . . . . . . . . . . . . . . . . 429
10.28Exercício 28 . . . . . . . . . . . . . . . . . . . . . . . . . . . . . . . . . . . . . . 429
10.29Exercício 29 . . . . . . . . . . . . . . . . . . . . . . . . . . . . . . . . . . . . . . 430
10.30Exercício 30 . . . . . . . . . . . . . . . . . . . . . . . . . . . . . . . . . . . . . . 430
10.31Exercício 31 . . . . . . . . . . . . . . . . . . . . . . . . . . . . . . . . . . . . . . 431
10.32Exercício 32 . . . . . . . . . . . . . . . . . . . . . . . . . . . . . . . . . . . . . . 432
10.33Exercício 33 . . . . . . . . . . . . . . . . . . . . . . . . . . . . . . . . . . . . . . 433
10.34Exercício 34 . . . . . . . . . . . . . . . . . . . . . . . . . . . . . . . . . . . . . . 435
10.35Exercício 35 . . . . . . . . . . . . . . . . . . . . . . . . . . . . . . . . . . . . . . 435
10.36Exercício 36 . . . . . . . . . . . . . . . . . . . . . . . . . . . . . . . . . . . . . . 436
10.37Exercício 37 . . . . . . . . . . . . . . . . . . . . . . . . . . . . . . . . . . . . . . 438
10.38Exercício 38 . . . . . . . . . . . . . . . . . . . . . . . . . . . . . . . . . . . . . . 438
10.39Exercício 39 . . . . . . . . . . . . . . . . . . . . . . . . . . . . . . . . . . . . . . 439
10.40Exercício 40 . . . . . . . . . . . . . . . . . . . . . . . . . . . . . . . . . . . . . . 440
10.41Exercício 41 . . . . . . . . . . . . . . . . . . . . . . . . . . . . . . . . . . . . . . 440
10.42Exercício 42 . . . . . . . . . . . . . . . . . . . . . . . . . . . . . . . . . . . . . . 440
10.43Exercício 43 . . . . . . . . . . . . . . . . . . . . . . . . . . . . . . . . . . . . . . 441
10.44Exercício 44 . . . . . . . . . . . . . . . . . . . . . . . . . . . . . . . . . . . . . . 446
10.45Exercício 45 . . . . . . . . . . . . . . . . . . . . . . . . . . . . . . . . . . . . . . 446
10.46Exercício 46 . . . . . . . . . . . . . . . . . . . . . . . . . . . . . . . . . . . . . . 448
10.47Exercício 47 . . . . . . . . . . . . . . . . . . . . . . . . . . . . . . . . . . . . . . 448
10.48Exercício 48 . . . . . . . . . . . . . . . . . . . . . . . . . . . . . . . . . . . . . . 449
10.49Exercício 49 . . . . . . . . . . . . . . . . . . . . . . . . . . . . . . . . . . . . . . 452
10.50Exercício 50 . . . . . . . . . . . . . . . . . . . . . . . . . . . . . . . . . . . . . . 453
10.51Exercício 51 . . . . . . . . . . . . . . . . . . . . . . . . . . . . . . . . . . . . . . 453
10.52Exercício 52 . . . . . . . . . . . . . . . . . . . . . . . . . . . . . . . . . . . . . . 457
10.53Exercício 53 . . . . . . . . . . . . . . . . . . . . . . . . . . . . . . . . . . . . . . 458
SUMÁRIO xiii

Referências Bibliográcas 461


xiv SUMÁRIO
Primeiras Palavras

a
Neste livro organizei minhas soluções do volume 1 do Curso de Análise (14 edição, Rio de
Janeiro: Associação Instituto Nacional de Matemática Pura e Aplicada, 2012), escrito pelo
ilustre Elon Lages Lima. Essas soluções são não ociais no sentido de que eu não as enviei
para a aprovação daqueles que possuem os direitos autorais do Curso de Análise. Também
transcrevi os enunciados dos exercícios, quase sempre de modo literal; digo quase porque
corrigi um ou outro erro de tipograa e removi boa parte dos tremas (permaneceram apenas os
tremas em nomes próprios). Por outro lado, mantive nos enunciados e nos nomes dos capítulos
todos os hífens, mesmo nos casos em que as atuais regras ociais da gramática dizem para não
1
usá-los. Além disso, quando identiquei que um exercício se referia a um conceito ou resultado
mais conhecido, para marcar esta conexão, o intitulei indicando este conceito ou resultado do
qual ele trata; títulos esses que não aparecem necessariamente no original.

De que modo este livro pode ser útil? Naturalmente, ele pode ser bem aproveitado por
alguém que esteja à procura de uma solução de um determinado exercício do Curso de Análise
após ter feito um esforço considerável para resolvê-lo ou ter passado algum tempo pensando
sobre ele. A respectiva solução aqui encontrada pode ser lida na íntegra ou, ainda, pode
conter alguma ideia que o inspire a criar a sua própria estratégia. Por outro lado, entendo que
existem certos contextos especícos nos quais decidimos ver uma solução sem antes fazermos
algumas tentativas para criá-la por conta própria; nestes contextos espero que a leitura possa
ser produtiva e pelo menos provocar alguma reexão em quem a faz. Uma outra possibilidade
é que o interessado ou a interessada sequer esteja trabalhando com alguma questão do livro do
Elon mas queira vir até aqui consultar algo que possa contribuir com seu trabalho (como, por
exemplo, a demonstração de algum teorema ou o desenvolvimento de um seminário ou de uma
aula). E assim por diante, estas são apenas algumas utilidades deste material dentre tantas
outras possíves. De todo modo, use-o com sabedoria.

Para quem quiser falar comigo (por exemplo, me enviar comentários, sugestões, indicar
alguma inadequação quanto à gramática ocial ou algum erro na matemática do texto), meu
email é onrubens@gmail.com e o endereço do arquivo é
https://www.dropbox.com/s/nb9bnabwv6hs5c1/CursoDeAnalise1ElonSolucoes.pdf ?dl=0
2
(acessando sua conta no Dropbox é possível fazer comentários no arquivo).

Agradeço ao Lísias Cruz Rodrigues e ao Rafael Vieira Bonangelo pela leitura minuciosa e
pelos comentários acerca deste capítulo Primeiras Palavras.

1 Por exemplo, elas dizem para não usar o hífen após o termo não atuando como prexo.
2 Para visualizar o arquivo não é necessário possuir uma conta no Dropbox.

xv
xvi PRIMEIRAS PALAVRAS

A demonstração estruturada
3
Em diversas soluções procurei pelo menos sugerir o caminho que percorri para encontrá-las.
Algumas delas redigi de forma estruturada, na tentativa de seguir a proposta de Uri Leron
em seu artigo Structuring Mathematical Proofs. Esta estruturação é composta por níveis
numerados, cada um (com exceção do Nível 1, o qual apresenta um panorama geral da solução)
contendo a demonstração de alguma armação feita durante a redação, de um tal modo que
o leitor ou a leitora não precise ler o texto de forma linear. Por exemplo, ao ler o Nível 1,
ele ou ela se depara com uma armação seguida de uma frase entre parênteses  algo como
Nível 2 ou Nível 2.1 , que indica em que parágrafo está o porquê desta armação ser
verdadeira; neste momento ele ou ela pode escolher se continua a leitura do Nível 1, deixando
o parágrafo da justicativa desta asserção para depois, ou se segue para o nível que a contém.
Além desses níveis, podem ser encontrados eventuais elevadores nos quais são feitas discussões
que motivam o desenvolvimento do nível subsequente; eles são parágrafos que funcionam como
uma espécie de parênteses no meio da solução.
O principal motivo pelo qual achei esta forma de redigir demonstrações interessante é que
ela nos dá espaço para mostrar um pouco do processo por trás do trabalho realizado, bastante
rico para ser ocultado como é feito na demonstração linear (a forma convencional de se redigir
demonstrações). Com isso, procura-se quebrar a ideia, infelizmente bastante difundida, de que
a matemática é somente para gênios, ou algo no qual as coisas já caem todas prontas do céu.
Algumas outras vantagens de uma demonstração estruturada são as seguintes: (a) nela é
possível destacar melhor em que momento uma certa hipótese está sendo usada, o que pode
facilitar a análise da sua necessidade e, consequentemente, a criação de ramicações para al-
guma generalização ou para novas perguntas; (b) durante um processo de construção de sua
autonomia, o leitor ou a leitora pode querer demonstrar algum passo por conta própria sem
antes ver uma justicativa já pronta deste passo; nestes casos, o espaço que ele ou ela precisa
para tanto é mais respeitado na demonstração estruturada do que na demonstração linear,
na qual o argumento que válida uma certa armação pode vir sem aviso prévio; (c) mesmo as
ideias que não deram certo, mas que, ainda sim, são interessantes, não precisam ser descartadas
completamente, elas encontram lugar para serem registradas nessa forma de redação; (d) ela
separa a parte mais formal da parte informal (que ca toda contida dentro do parágrafo No
elevador), o que pode facilitar a leitura e também pode funcionar como um recurso didático
para quem está desenvolvendo seu entendimento do conceito de formalismo.

A contrapositiva
Um conceito que utilizei bastante ao longo deste livro e que não foi denido no Curso de
Análise foi o da contrapositiva. Muitas vezes ela pode aparecer, por exemplo, após nos per-
guntarmos e se a tese não for verdadeira? (uma pergunta que podemos nos fazer para captar
melhor o signicado de um teorema e também para ter um ponto de partida para investigar
sua demonstração).
Uma vez trabalhando com a negação da tese, teremos demonstrado a contrapositiva do
respectivo teorema quando chegarmos à negação de sua hipótese.
Mais formalmente:

3 Sobre a diferença entre o modo como uma descoberta cientíca é feita e o modo como ela costuma ser orga-
nizada textualmente, ver, por exemplo, o texto História da matemática e ensino, que faz parte da introdução
do livro História da Matemática: uma visão crítica, desfazendo mitos e lendas, de Tatiana Roque, p. 29-33.
xvii

Dado um conjunto E e dada uma propriedade R referente aos elementos de E, denotemos


por ∼R a negação de R; a propriedade ∼R ainda sendo referente aos elementos de E. Dadas
as propriedades P e Q referentes a um dado conjunto E , a contrapositiva da implicação P ⇒ Q
é a implicação ∼ Q ⇒∼ P .
O que nos permite substituir um teorema pela sua contrapositiva é o fato de que eles são
equivalentes. Vejamos uma forma de justicar isso.
Ainda com a notação acima, no livro já foi observado que a implicação P ⇒ Q signica que
o conjunto X denido por X = {x ∈ E; x possui a propriedade P } está contido no conjunto Y
denido por Y = {x ∈ E; x possui a propriedade Q} (páginas 5 e 6). Analogamente, sua con-
trapositiva ∼ Q ⇒∼ P traduz a inclusão E −Y ⊂ E −X , pois {x ∈ E; possui a propriedade ∼
R} = E − {x ∈ E; possui a propriedade R}. Agora podemos concluir que uma implicação e
sua contrapositiva são equivalentes porque X ⊂ Y ⇔ E − Y ⊂ E − X (propriedade C2 da
operação de se tomar complementares, página 10).

Notações e ans
De um modo geral, procurei seguir as mesmas notações e expressões do Curso de Análise.
Abaixo, destaque para algumas exceções:

f −1 [Y ]

• f [X] e para indicar as respectivas imagem (que em alguns momentos também
chamei de imagem direta ) de X pela função f (sendo X um subconjunto do domínio de f )
e imagem inversa de Y (Y ⊂ R) por f , em vez de f (X) e de f −1 (Y ), usei os símbolos f [X]
−1
e f [Y ]. Além disso, mantive o uso do símbolo f −1 [{y}], para y ∈ R. Isto nas soluções.
−1
Nos enunciados dos exercícios foram mantidos os símbolos f (X) e f (Y ) signicando
imagens direta e inversa;

• (:=) o símbolo := signica igual por denição ;


• (spg) esta abreviação signica sem perda de generalidade ;

•  e um quadradinho preto signica que a solução de um exercício chegou ao m. O
quadradinho branco signica que chegaram ao m: a solução de um item, de um nível
ou de um elevador;

• (Desigualdade Triangular) só após ter usado esta expressão diversas vezes é que notei que
ela não foi usada pelo Elon. Chama-se Desigualdade Triangular à desigualdade |x + y| 6
|x| + |y|, válida para quaisquer reais x e y (item (i) do Teorema 2 do Capítulo III);

• na solução de um exercício de um certo Capítulo x, ao falar, por exemplo, pelo Teorema


y (sem explicitar o capítulo), me rero ao Teorema y do próprio Capítulo x.

Rubens Rodrigues Onishi


São Paulo, junho de 2022.
xviii PRIMEIRAS PALAVRAS
Capítulo 1
Conjuntos e Funções

1.1 Exercício 1
Dados os conjuntos A e B, seja X um conjunto com as seguintes propriedades:

a
1 X⊃A e X ⊃ B,
a
2 Se Y ⊃A e Y ⊃B então Y ⊃ X.

Prove que X = A ∪ B.

Solução.

Nível 1. Vamos mostrar que A∪B ⊂ X (Nível 2.1) e X ⊂ A∪B (Nível 2.2). Disto segue
X =A∪B pela antissimetria da relação de inclusão.

Nível 2.1. Seja x ∈ A ∪ B. Queremos mostrar que x ∈ X. Se x ∈ A ∪ B, então x∈A ou


a
x ∈ B. De qualquer forma concluímos que x∈X pela 1 propriedade, como queríamos.

No elevador. Num primeiro momento nosso intuíto pode ser o de seguir o mesmo caminho
do Nível 2.1, o que nos leva à pergunta dado x ∈ X, como concluir que x∈A ou x ∈ B ?.
Sem ter muitas ideias de como respondê-la (pelo menos a princícpio) partimos em busca de
uma outra estratégia. O seguinte raciocínio é bastante comum em situações deste tipo. Não
a
usamos até agora todas as informações dadas (das duas propriedades, usamos apenas a 1 ),
então talvez possamos descobrir algo de novo a partir da investigação daquilo que ainda não foi
a
usado. Este raciocínio nos leva a olhar para a 2 propriedade (a informação que ainda não foi
usada). Ela é semelhante a algo já observado na página 7, a saber, que A ⊂ A∪B e B ⊂ A ∪ B,
ou, o que é o mesmo, A∪B ⊃A e A ∪ B ⊃ B. Assim podemos concluir o resultado desejado
substituindo Y por A ∪ B.
(Voltar para a Solução 1 do Exercício 3, para seu elevador: página 3.)

Nível 2.2. O conjunto A∪B contém ambos os conjuntos A e B, portanto A∪B ⊃ X


a
pela 2 propriedade:
A∪B ⊃A e A ∪ B ⊃ B ⇒ A ∪ B ⊃ X,
como queríamos. 

1
2 CAPÍTULO 1. CONJUNTOS E FUNÇÕES

1.2 Exercício 2
Enuncie e demonstre um resultado análogo ao anterior, caracterizando A ∩ B.

Solução. Dados os conjuntos A, B e X, armamos que se X satisfaz as propriedades

a) X⊂A e X ⊂ B, e;

b) se Y ⊂A e Y ⊂ B, então Y ⊂ X,
então X = A ∩ B.
A ideia é a mesma do exercício anterior, a propriedade a) implica X ⊂ A∩B e a propriedade
b) implica A ∩ B ⊂ X:
X⊂A e X ⊂B ⇒X ⊂A∩B
e
A∩B ⊂A e A ∩ B ⊂ B ⇒ A ∩ B ⊂ X.
Logo, pela propriedade antissimétrica da relação de inclusão, X = A ∩ B, como queríamos.

(Voltar para a Solução do Exercício 17: página 13.)

1.3 Exercício 3
Sejam A, B ⊂ E . Prove que A∩B = ∅ se, e somente se, A ⊂ {B . Prove também que A∪B = E
se, e somente se, {A ⊂ B .
(Voltar para a Solução 2 do Exercício 6: página 6.)

Solução 1. Vamos dividir este exercício em duas partes. Na Primeira Parte vamos mostrar
a primeira equivalência, isto é,
A ∩ B = ∅ ⇔ A ⊂ {B,
e, na Segunda Parte, a segunda equivalência, isto é

A ∪ B = E ⇔ {A ⊂ B.

Primeira Parte.

Nível 1. Temos duas implicações a demonstrar. No Nível 2.1 vamos mostrar que

A ∩ B = ∅ ⇒ A ⊂ {B

e, no Nível 2.2, que

A ⊂ {B ⇒ A ∩ B = ∅.

Nível 2.1. A ∩ B = ∅, queremos mostrar que A ⊂ {B . Assim, seja x um


Supondo
elemento de A. x não pode pertencer a B , pois, se pertencesse, então teríamos x ∈ A ∩ B ,
Este
contrariando a hipótese de que A ∩ B = ∅. Agora x é um elemento de E (se é elemento de A,
então é elemento de E porque A ⊂ E ) e não é um elemento de B , isto é, x ∈ E − B := {B .
Isto mostra a inclusão A ⊂ {B (qualquer elemento de A é também elemento de {B ).
1.3. EXERCÍCIO 3 3

Nível 2.2. Aqui estamos supondo A ⊂ {B e queremos mostrar que A ∩ B = ∅. Por


hipótese, se x ∈ A, então x ∈/ B . Isto é suciente para concluir que A ∩ B = ∅, pois, se existisse
um x ∈ A ∩ B , então teríamos x ∈ A e x ∈ B , e acabamos de ver que não é possível ter essas
duas coisas simultaneamente.

Segunda Parte.

Nível 1. No Nível 2.1 vamos mostrar a implicação

A ∪ B = E ⇒ {A ⊂ B
e, no Nível 2.2, a implicação
{A ⊂ B ⇒ A ∪ B = E.
Destas duas implicações vem a equivalência desejada: A ∪ B = E ⇔ {A ⊂ B .

Nível 2.1. Supondo A ∪ B = E , dado x ∈ {A, queremos mostrar que x ∈ B . Se


x ∈ {A := E − A, então x ∈ E e x ∈/ A. De x ∈ E e de E = A ∪ B (por hipótese) segue x ∈ A
ou x ∈ B . Portanto, como x ∈
/ A, só pode ser x ∈ B , como queríamos.

Nível 2.2. Reciprocamente, supondo {A ⊂ B , queremos mostrar que A ∪ B = E , uma


igualdade entre dois conjuntos. Para tanto, temos duas inclusões a vericar: A ∪ B ⊂ E e
E ⊂ A ∪ B. Vamos vericar a primeira delas no Nível 3.1 e, a segunda delas, no Nível 3.2.
O resultado desejado E = A∪B segue então pela propriedade antissimétrica da relação de
inclusão.

Nível 3.1 A referida inclusão é mais imediata: A⊂E e B⊂E (hipóteses do exercício)
implicam A ∪ B ⊂ E.

No elevador. Dado x ∈ E, queremos concluir que x ∈ A ∪ B. Notemos que até agora


não usamos em nenhum momento a hipótese assumida no Nível 2.2 (ver o comentário feito
no elevador do Nível 2.2 do Exercício 1 (página 1). Segundo ela, um elemento de E que não
pertence a A pertence a B . Isto nos sugere dividir o nível abaixo em dois casos, o caso em que
um elemento pertence a A e o caso em que um elemento não pertence a A.

Nível 3.2. x ∈ E . Com respeito a pertencer ou não a A, existem apenas duas


Seja
possibilidades: ou x ∈ A ou x ∈/ A. Analisemos cada uma delas.
Se x ∈ A, então x ∈ A ∪ B . Se x ∈ / A, então, x ∈ {A e, por hipótese (ainda estamos dentro
das hipóteses do Nível 2.2, ou seja, {A ⊂ B ), x ∈ B , logo, x ∈ A ∪ B . Em qualquer um destes
dois casos temos x ∈ A ∪ B , como queríamos. 

Observação. Existe um outro modo de redigirmos os Níveis 2.1 e 2.2 da Primeira Parte da
Solução 1, evitando as (aparentes) contradições que nelas apareceram. Este modo consiste no
uso da contrapositiva. A ∩ B = ∅ ⇒ A ⊂ {B é equivalente à implicação
Por ela, a implicação
A 6⊂ {B ⇒ A ∩ B 6= ∅ e a implicação A ⊂ {B ⇒ A ∩ B = ∅ é equivalente à implicação
A ∩ B 6= ∅ ⇒ A 6⊂ {B . Assim, demonstrar que A ∩ B = ∅ ⇔ A ⊂ {B é equivalente a
demonstrar que A ∩ B 6= ∅ ⇔ A 6⊂ {B . Isto nos dá, inclusive, uma forma mais sucinta
de demonstrar a primeira equivalência dada, o que nos leva à solução abaixo. Nela também
veremos como resolver a segunda parte deste exercício usando a primeira.
4 CAPÍTULO 1. CONJUNTOS E FUNÇÕES

Solução 2. De forma equivalente (usando a contrapositiva), vamos mostrar que A ∩ B 6= ∅


se, e somente se, A 6⊂ {B . Temos

A ∩ B 6= ∅ ⇔ existe x ∈ A ∩ B
⇔ existe x tal que x ∈ A e x ∈ B
⇔ existe x tal que x ∈ A e x ∈
/ {B
⇔ existe x ∈ A tal que x ∈
/ {B
⇔ A 6⊂ {B,
como queríamos.
Lembrando da observação feita na página 11, a de que a operação de tomar complementa-
res de conjuntos transforma reuniões em interseções, podemos resolver a segunda parte deste
exercício usando a primeira:

A∪B =E ⇔ {(A ∪ B) = ∅
(Propriedade C4) ⇔ {A ∩ {B = ∅

(Primeira parte) ⇔ {A ⊂ { {B
(Propriedade C1) ⇔ {A ⊂ B,
onde a equivalência A ∪ B = E ⇔ {(A ∪ B) = ∅ vale pelas propriedades C3 e C1:

A ∪ B = { {(A ∪ B) = E ⇔ {(A ∪ B) = ∅.
Logo, A ∪ B = E ⇔ {A ⊂ B , como queríamos. 

Observação. Também usando a primeira parte do exercício, de


 {A ∩ {B = ∅ poderíamos ter
tirado {B ⊂ { {A em vez de {A ⊂ {({B) como zemos. Claramente isto deve nos levar ao
mesmo lugar:
 
{B ⊂ { {A = A ⇔ {A ⊂ { {B = B.

1.4 Exercício 4
Dados A, B ⊂ E , prove que A⊂B se, e somente se, A ∩ {B = ∅.

Solução 1.

No elevador. Trabalhando com a forma em que este exercício se apresenta, temos o


seguinte. (Ida) Supondo A ⊂ B , queremos mostrar que A ∩ {B = ∅; se A ∩ {B fosse não
vazio, existiria um x ∈ A ∩ {B , ou seja, x ∈ A e x ∈ / B , contrariando a hipótese de que A é
subconjunto de B , logo A ∩ {B = ∅. (Volta) Reciprocamente, supondo A ∩ {B = ∅, queremos
mostrar que A ⊂ B ; se A não fosse subconjunto de B , então existiria um x ∈ A que não
pertence a B , isto é, x ∈ A e x ∈ {B , o que signica que A ∩ {B é não vazio, logo A ⊂ B .
Na ida desta solução, a hipótese de A ser subconjunto de B não foi verdadeiramente usada
durante o desenvolvimento de seu raciocínio, isto é, no trecho se A∩{B fosse não vazio, existiria
um x ∈ A ∩ {B , ou seja, x ∈ A e x ∈ / B . O mesmo vale para a volta: em nenhum momento
no trecho se A não fosse subconjunto de B , então existiria um x ∈ A que não pertence a B ,
isto é, x ∈ A e x ∈ {B , o desenvolvimento do argumento, foi usada a hipótese de A ∩ {B ser
não vazio. Em situações deste tipo, nas quais temos uma aparente contradição, é a forma
contrapositiva quem nos permite remover os excessos de nosso texto, tudo aquilo que não foi
efetivamente usado.
1.5. EXERCÍCIO 5 5

Nível 1. De forma equivalente, vamos provar a contrapositiva deste exercício, isto é, que

A 6⊂ B ⇔ A ∩ {B 6= ∅.

No elevador. Olhar para onde queremos chegar pode nos dar a dica do que fazer em cada
passo.

Nível 2. Temos:

A 6⊂ B ⇔ existe x ∈ A tal que x ∈


/B
⇔ existe x ∈ A tal que x ∈ {B
⇔ existe x ∈ A ∩ {B
⇔ A ∩ {B 6= ∅,

como queríamos. 

Solução 2.

No elevador. Utilizar algo que já foi demonstrado também é bastante comum em mate-
mática. Notemos a semelhança deste exercício com o exercício anterior, ambos relacionam uma
inclusão com uma interseção. De fato, um é consequência do outro.
Como A, {B ⊂ E , pelo exercício anterior, temos

A ∩ {B = ∅ ⇔ A ⊂ { {B = B,

como queríamos. 

1.5 Exercício 5
Dê exemplo de conjuntos A, B, C tais que

(A ∪ B) ∩ C 6= A ∪ (B ∩ C).

(Voltar para a Observação do Exercício 7: página 7.)

Solução.

No elevador. Ao procurar um exemplo de objetos satisfazendo determinadas proprieda-


des, uma dica é começar investigando os casos mais simples. Aqui fazemos A = {1}, B = {2}
e C = {3} e, após fazer contas, vemos que isso funciona.
Fazendo A = {1}, B = {2} e C = {3}, temos:

(A ∪ B) ∩ C = {1, 2} ∩ {3} = ∅,

enquanto
A ∪ (B ∩ C) = {1} ∪ ∅ = {1}.
Portanto, como ∅=
6 {1}, (A ∪ B) ∩ C 6= A ∪ (B ∩ C). 
(Voltar para a Segunda Parte da Solução do Exercício 10: página 9.)
6 CAPÍTULO 1. CONJUNTOS E FUNÇÕES

1.6 Exercício 6
Se A, X ⊂ E são tais que A∩X =∅ e A ∪ X = E, prove que X = {A.

Solução 1.

Nível 1. Vamos mostrar que X ⊂ {A (Nível 2.1) e {A ⊂ X (Nível 2.2). Assim, podemos
concluir que X = {A pela propriedade antissimétrica da relação de inclusão.

Nível 2.1. x ∈ X , para concluir que x ∈ {A, vamos vericar que x ∈


Dado / A. Isto
acontece porque X∩A = ∅: um x que pertence a X não pode pertencer a A (pois, se pertencesse,
então teríamos X ∩ A 6= ∅). Logo X ⊂ {A.

Nível 2.2. x ∈ {A, ou seja, um x ∈ E que não pertence a A, queremos concluir que
Dado
x ∈ X . Isto acontece porque E = A ∪ X : um elemento de E deve pertencer a A ou a X ; se
não pertence a A, então só pode pertencer a X . Logo {A ⊂ X . 

Solução 2.

No elevador. Notemos a semelhança deste exercício com o Exercício 3 (página 2). De


fato podemos aplicá-lo aqui. Por exemplo, por ele, de A ∩ X = ∅ podemos concluir que A ⊂ {X
ou X ⊂ {A. Olhando para onde queremos chegar (queremos chegar a X = {A), dessas duas
inclusões é mais conveniente usar a segunda. Claro que também podemos usar a primeira,

poisA ⊂ {X ⇒ { {X = X ⊂ {A (propriedades C2 e C1). A mesma discussão vale para
A ∪ X = E.
Pelo Exercício 3, temos
A ∩ X = ∅ ⇒ X ⊂ {A
e
A ∪ X = E ⇒ {A ⊂ X.
Logo, pela propriedade antissimétrica da relação de inclusão, X = {A, como queríamos. 

Observação. O Exercício 3 também nos permite concluir a recíproca do exercício acima


se X e A são subconjuntos de E : X = {A implica X ⊂ {A e {A ⊂ X ; (se A, X ⊂ E )
X ⊂ {A ⇒ X ∩ A = ∅ e {A ⊂ X ⇒ A ∪ X = E .

1.7 Exercício 7
Se A ⊂ B, então, B ∩ (A ∪ C) = (B ∩ C) ∪ A para todo conjunto C. Por outro lado, se existir
C de modo que a igualdade acima seja satisfeita, então A ⊂ B.

Solução. Para a primeira parte deste exercício, um uso das propriedades das operações de
reunião e interseção nos dá o resultado desejado. Assim temos

B ∩ (A ∪ C) = (B ∩ A) ∪ (B ∩ C) = A ∪ (B ∩ C) = (B ∩ C) ∪ A,

onde a primeira igualdade vale pela propriedade ∩7, a segunda, pela propriedade ∩5 (pois
A ⊂ B) e, a terceira, pela ∪3.
1.8. EXERCÍCIO 8 7

Com respeito a segunda parte, dado x ∈ A, queremos mostrar que x ∈ B . Devemos usar a
hipótese de alguma forma. Deste modo, seja C tal que B ∩ (A ∪ C) = (B ∩ C) ∪ A. Usando
esta igualdade escrevemos

x ∈ A ⇒ x ∈ (B ∩ C) ∪ A ⇒ x ∈ B ∩ (A ∪ C) ⇒ x ∈ B e x ∈ A ∪ B.

Em particular (a parte que nos interessa), x ∈ B, como queríamos. 

Observação. O exercício acima dialoga com o Exercício 5 (página 5). Com relação à igual-
dade B ∩ (C ∪ A) = (B ∩ C) ∪ A, um nos diz que ela nem sempre é válida, enquanto que, o
outro, nos diz quando ela o é (uma vez que B ∩ (C ∪ A) = B ∩ (A ∪ C)).

1.8 Exercício 8
Prove que A=B se, e somente se, (A ∩ {B) ∪ ({A ∩ B) = ∅.

Solução 1.

Nível 1. No Nível 2.1 vamos mostrar que A = B ⇒ (A ∩ {B) ∪ ({A ∩ B) = ∅ e no Nível


2.2, que (A ∩ {B) ∪ ({A ∩ B) = ∅ ⇒ A = B .

Nível 2.1. Suponha A = B. Substituindo A por B em A∩{B obtemos A∩{B = B ∩{B =


∅. Analogamente, substituindo B por A em {A ∩ B , obtemos {A ∩ B = {A ∩ A = ∅. Portanto,

(A ∩ {B) ∪ ({A ∩ B) = ∅ ∪ ∅ = ∅,

como queríamos.

No elevador. Para a recíproca, consideremos a questão de mostrar que um dado x∈A


pertence a B. Como a hipótese (A ∩ {B) ∪ ({A ∩ B) = ∅ envolve o complemento de B , ela nos
sugere dividir o problema no caso em que x ∈ B e no caso em que x ∈ / B . No primeiro caso
não há nada a se fazer, o objetivo já está automaticamente atingido. No segundo, lembrando
que o x já foi xado em A, temos x ∈ A ∩ {B , o que faz o conjunto (A ∩ {B) ∪ ({A ∩ B) ser
não vazio. Neste ponto surge o argumento da contrapositiva. Usá-lo desde o início vai tornar
nossa demonstração com menos passos, mais limpa.

Nível 2.2. Vamos mostrar a forma equivalente da implicação dada, sua contrapositiva

A 6= B ⇒ (A ∩ {B) ∪ ({A ∩ B) 6= ∅.

A 6= B surgem os dois casos seguintes: existe x ∈ A que não pertence a B , ou existe x ∈ B


De
que não pertence a A. No primeiro caso existe x ∈ A ∩ {B e, no segundo, existe x ∈ B ∩ {A.
Em qualquer um deles existe x ∈ (A ∩ {B) ∪ ({A ∩ B), portanto, (A ∩ {B) ∪ ({A ∩ B) 6= ∅,
como queríamos. 

1.9 Exercício 9
Prove que (A − B) ∪ (B − A) = (A ∪ B) − (A ∩ B).
8 CAPÍTULO 1. CONJUNTOS E FUNÇÕES

Solução. Dado x ∈ (A − B) ∪ (B − A), queremos concluir que x ∈ A ∪ B e x ∈ / A ∩ B . Se


x ∈ (A − B) ∪ (B − A), então x ∈ A − B ou x ∈ B − A. Se x ∈ A − B , então x ∈ A e x ∈ / B . De
x ∈ A vem x ∈ A ∪ B e de x ∈ / B vem x ∈/ A ∩ B . Portanto x ∈ A − B ⇒ x ∈ (A ∪ B) − (A ∩ B).
O caso em que x ∈ B − A é análogo: x ∈ B ⇒ x ∈ A ∪ B , e; x ∈ /A⇒x∈ / A ∩ B , de modo que
x ∈ B − A ⇒ x ∈ (A ∪ B) − (A ∩ B). Logo, qualquer elemento de (A − B) ∪ (B − A) também
é elemento de (A ∪ B) − (A ∩ B), ou seja, (A − B) ∪ (B − A) ⊂ (A ∪ B) − (A ∩ B).
Reciprocamente, dado x ∈ (A∪B)−(A∩B), queremos concluir que x ∈ A−B ou x ∈ B −A.
Se x ∈ (A ∪ B) − (A ∩ B), então x ∈ A ∪ B e x ∈ / A ∩ B . De x ∈ A ∪ B vem x ∈ A ou x ∈ B .
E de x ∈ / A ∩ B vem x ∈ / A ou x ∈/ B . A combinação de todas estas possiblidades dá origem
aos 4 casos seguintes: (i) x ∈ A e x ∈
/ A; (ii) x ∈ A e x ∈/ B ; (iii) x ∈ B e x ∈
/ A, e; (iv) x ∈ B
e x ∈/ B . Destes, o primeiro e o quarto são absurdos, logo só podemos ter os segundo ou o
terceiro, ou seja, x ∈ A − B ou x ∈ B − A, como queríamos. 

1.10 Exercício 10
Seja A∆B = (A − B) ∪ (B − A). Prove que A∆B = A∆C implica B = C. Examine a validez
de um resultado análogo com ∩, ∪ ou × em vez de ∆.

Solução 1.

Primeira Parte. Nesta Primeira Parte vamos provar que A∆B = A∆C ⇒ B = C .

Nível 1. O objetivo é mostrar que B = C , uma igualdade entre conjuntos. Como sempre,
para tanto, vamos mostrar que B está contido em C (Nível 2.1) e que C está contido em B
(Nível 2.2). A igualdade B=C segue então pela antissimetria da relação de inclusão.

No elevador. Começando com a inclusão B ⊂ C, dado x ∈ B, queremos concluir que


x ∈ C . Para tirarmos esta conclusão, a única coisa que temos para nos auxiliar é a hipótese
A∆B = A∆C ; devemos incluí-la em nossa demonstração de alguma forma. Olhando para ela,
uma vez que x ∈ B , com mais um pouco vamos podemos dizer que x ∈ A∆B e, consequemente,
x ∈ A∆C (pois A∆B = A∆C ), A∆C sendo um conjunto que envolve C (o objetivo a ser
alcançado) em sua composição. Este pouco depende de x pertencer ou não a A. Considerar
então estes casos (o de x pertencer ou não a A) parece promissor para prosseguirmos com a
demonstração. É o que faremos.

Nível 2.1. Seja x ∈ B . Queremos mostrar que x ∈ C . Se x ∈/ A, então x ∈ B − A ⊂


(A − B) ∪ (B − A) := A∆B = A∆C (A∆B = A∆C por hipótese), portanto x ∈ A∆C :=
(A − C) ∪ (C − A), donde x ∈ A − C ou x ∈ C − A. Disto segue x ∈ C − A (x não pertence
a A − C porque x não pertence a A, por hipótese), em particular x ∈ C , como queríamos.
Analisemos agora o caso em que x ∈ A.
Se x ∈ A, então x não pertence a B − A (por denição, uma condição necessária para
pertencer a B − A é não pertencer a A) e nem pertence a A − B (não pertence a A − B porque
x ∈ B , por hipótese), ou seja, x ∈
/ A∆B = A∆C := (A − C) ∪ (C − A), portanto x ∈ / A−C e
x∈/ C − A, onde
x∈/ A−C ⇒x∈ / A ou x ∈ C
e
x∈
/ C −A⇒x∈
/C ou x ∈ A.
1.10. EXERCÍCIO 10 9

Estas duas últimas implicações (lembrando que queremos chegar a x ∈ C ) nos dizem o que
fazer em seguida. De x∈ / A−C e x∈ / C − A segue, em particular, x ∈
/ A − C , o que, por sua
vez, implica x ∈
/ A ou x ∈ C . Como x ∈ A (por hipótese), só pode ser então x ∈ C .
De qualquer forma, x pertencendo ou não a A, não havendo nenhum outro caso a analisar
além deste dois, temos x ∈ C , como queríamos.

Nível 2.2. Esta demonstração é análoga à do nível anterior. Seja x ∈ C. Queremos


mostrar que x ∈ B. Temos

x∈
/A ⇒ x∈C −A
⇒ x ∈ (C − A) ∪ (A − C) = A∆C = A∆B
⇒ x ∈ B − A ou x ∈ A − B
⇒ x∈B−A
⇒ x∈B

(onde x∈B−A ou x∈A−B implica x∈B−A porque x∈


/A⇒x∈
/ A − B ), e

x∈A ⇒ x∈/ (A − C) ∪ (C − A) = A∆C


⇒ x∈/ A∆B = (A − B) ∪ (B − A)
⇒ x∈/ A−B e x∈ / B−A
⇒ x∈/ A−B
⇒ x∈/ A ou x ∈ B
⇒ x ∈ B,

onde: x ∈ A implica x ∈ / (A − C) ∪ (C − A) porque x ∈ C (x ∈ A ⇒ x ∈ / C − A, e


x∈C⇒x∈ / A − C ); x ∈
/ A∆C ⇒ x ∈ / A∆B porque A∆C = A∆B por hipótese, e; x ∈ / A ou
x ∈ B implica x ∈ B porque x ∈ A. Logo, de qualquer forma, x ∈ B , como queríamos.

Segunda Parte (interseção). Queremos vericar se A∩B =A∩C B = C. O


implica
exemplo da Solução do Exercício 5 (página 5) se encaixa aqui também: para A = {1}, B = {2}
e C = {3} temos A ∩ B = ∅ = A ∩ C porém B 6= C . Um outro exemplo é A = {1}, B = {1, 2}
e C = {1, 3}: A ∩ B = {1} = A ∩ C mas B 6= C . Portanto A ∩ B = A ∩ C não implica B = C .
Agora, se ambos B e C forem subconjuntos de A, então o resultado torna-se verdadeiro: se
B ⊂ A, C ⊂ A e A∩B = A∩C , então, lembrando da propriedade ∩5, B = A∩B = A∩C = C .
De um modo um pouco mais geral, o resultado também é verdadeiro se B − A = C − A, pois
B = (B − A) ∪ (A ∩ B) e C = (C − A) ∪ (A ∩ C).

Terceira Parte (reunião). Para a reunião o resultado também não é verdadeiro. Por
exemplo, para A = {1, 2}, B = {1} e C = {2} temos A ∪ B = A = A ∪ C , enquanto B 6= C .
Por outro lado, se A ⊂ B e A ⊂ C , então, aí sim vale B = C se A ∪ B = A ∪ C , uma vez que,
nesse caso, pela propriedade ∪5, A ∪ B = B e A ∪ C = C .

Quarta Parte (produto cartesiano). Supondo A × B = A × C , queremos vericar se


B = C . Tomando, por exemplo, b ∈ B , para usarmos a hipótese, se pudermos considerar um
a ∈ A, então teremos (a, b) ∈ A × B e, portanto, (a, b) ∈ A × C , em particular, b ∈ C . Vamos
poder concluir então que A × B = A × C implica B = C se A 6= ∅. Caso contrário, caso A = ∅,
o resultado é falso. Por exemplo, para B = {1} e C = {2}, ∅ × B = ∅ = ∅ × C . 
10 CAPÍTULO 1. CONJUNTOS E FUNÇÕES

Solução 2 (para a Primeira Parte). B 6= C , então existe x ∈ B−C ou existe x ∈ C −B .


Se
x ∈ B−C . O segundo caso será análogo.
Consideremos o primeiro destes dois casos e xemos um
Se x ∈ A, então x ∈ A − C ⊂ A∆C e x ∈ / A∆B , x ∈ / A − B pois x ∈ B e x ∈ / B − A pois
x ∈ A. Se x ∈/ A, então x ∈ B − A ⊂ A∆B e x ∈
/ A∆C , x ∈ / A − C pois x ∈
/Aex∈ / C −A
pois x ∈
/ C . Portanto A∆B 6= A∆C . Logo B 6= C ⇒ A∆B 6= A∆C e, pela contrapositiva,
A∆B = A∆C ⇒ B = C . 

1.11 Exercício 11
Prove as seguintes armações:

1. (A ∪ B) × C = (A × C) ∪ (B × C);

2. (A ∩ B) × C = (A × C) ∩ (B × C);

3. (A − B) × C = (A × C) − (B × C);

4. A ⊂ A0 , B ⊂ B 0 ⇒ A × B ⊂ A0 × B 0 .

Solução.

item 1. (x, y) ∈ (A∪B)×C . Então x ∈ A∪B e y ∈ C . Se x ∈ A, então (x, y) ∈ A×C


Seja
e, se x ∈ B , então (x, y) ∈ B × C . Portanto (x, y) ∈ (A × C) ∪ (B × C), o que mostra a inclusão
(A ∪ B) × C ⊂ (A × C) ∪ (B × C). Reciprocamente, seja (x, y) ∈ (A × C) ∪ (B × C). Se
(x, y) ∈ A×C , então (x, y) ∈ (A∪B)×C porque y ∈ C e x ∈ A ⇒ x ∈ A∪B . Se (x, y) ∈ B ×C ,
então (x, y) ∈ (A ∪ B) × C porque y ∈ C e x ∈ B ⇒ x ∈ A ∪ B . Portanto, (x, y) ∈ (A ∪ B) × C ,
o que mostra a inclusão (A × C) ∪ (B × C) ⊂ (A ∪ B) × C . Dessas duas inclusões segue a
igualdade (A ∪ B) × C = (A × C) ∪ (B × C).

item 2. (x, y) ∈ (A ∩ B) × C , então x ∈ A ∩ B e y ∈ C . De x ∈ A ∩ B segue


Se
x ∈ A e x ∈ B , portanto (x, y) ∈ A × C e (x, y) ∈ B × C . Se (x, y) ∈ (A × C) ∩ (B × C),
então (x, y) ∈ A × C e (x, y) ∈ B × C , o que implica x ∈ A e x ∈ B e y ∈ C , portanto
(x, y) ∈ (A ∩ B) × C .

item 3. (x, y) ∈ (A − B) × C , então: (x, y) ∈ A × C porque x ∈ A e y ∈ C , e;


Se
(x, y) ∈
/ B × C porque x ∈ / B . Portanto, (A − B) × C ⊂ (A × C) − (B × C). Se (x, y) ∈
(A × C) − (B × C), então (x, y) ∈ A × C e (x, y) ∈ / B × C . Só é possível acontecer estas duas
coisas simultaneamente se for x ∈ A, y ∈ C e x ∈ / B . Isto porque (x, y) ∈
/ B × C implica x ∈
/B
ou y ∈/ C ; se fosse y ∈
/ C , então teríamos (x, y) ∈
/ A × C . Portanto, de x ∈ A, y ∈ C e x ∈ / B,
vem (x, y) ∈ (A − B) × C . Logo (A × C) − (B × C) ⊂ (A − B) × C .

item 4. Aqui temos:

(x, y) ∈ A × B ⇒ x ∈ A e y ∈ B ⇒ x ∈ A0 e y ∈ B 0 ⇒ (x, y) ∈ A0 × B 0 .

A segunda das implicações acima vale por causa das hipóteses A ⊂ A0 e B ⊂ B0. Logo,
A × B ⊂ A0 × B 0 . 
1.12. EXERCÍCIO 12 11

1.12 Exercício 12
Dada da [ sic ] função f : A → B :
1. Prove que se tem f (X − Y ) ⊃ f (X) − f (Y ), sejam quais forem os subconjuntos X e Y
de A;

2. Mostre que se f for injetiva então f (X − Y ) = f (X) − f (Y ) para quaisquer X , Y contidos


em A.

(Voltar para a Solução do Exercício 29 do Capítulo II: página 63.)

Solução. Dado um elemento a f [X] − f [Y ], seja x ∈ X tal que a = f (x).


do conjunto
Este x não pertence a Y , pois, se pertencesse, então a, por ser igual a f (x), pertenceria a
f [Y ], mas a ∈ / f [Y ] por hipótese (por denição, z ∈ Y ⇒ f (z) ∈ f [Y ] ou, equivalentemente,
f (z) ∈/ f [Y ] ⇒ z ∈/ Y ). Portanto x ∈ X − Y , o que faz a ser um elemento de f [X − Y ]. Logo,
f [X] − f [Y ] ⊂ f [X − Y ], como queríamos.
Suponhamos agora f injetiva. Como f [X] − f [Y ] ⊂ f [X − Y ], para concluir que f [X − Y ] =
f [X] − f [Y ], só falta mostrar que f [X − Y ] ⊂ f [X] − f [Y ]. Assim, seja f (x) ∈ f [X − Y ], com
x ∈ X − Y . Queremos mostrar que f (x) ∈ f [X] e que f (x) ∈ / f [Y ]. O fato de x pertencer
a X implica f (x) ∈ f [X]. Agora, se f (x) também pertencesse a f [Y ], então existiria y ∈ Y
tal que f (x) = f (y) e, portanto, pela injetividade de f , teríamos x = y , e isto faria com
que x fosse um elemento de Y . Em outras palavras, com este raciocínio, dado z ∈ A, temos
f (z) ∈ f [Y ] ⇒ z ∈ Y , por causa da injetividade de f . Portanto, pela contrapositiva, como
x∈ / Y , segue f (x) ∈ / f [Y ]. Logo f (x) ∈ f [X] − f [Y ], como queríamos. 

Observação. Podemos ter f (x) ∈ f [Y ] mesmo que x ∈/ Y. Por exemplo: f : R → R denida


por f (z) = z 2 para todo z ∈ R, Y = [0, 1] e x = −1. Por um lado, −1 ∈
/ [0, 1]. Por outro,
f (−1) ∈ f [[0, 1]], pois f (−1) = f (1) e 1 ∈ [0, 1]. Também podemos chegar a este resultado
fazendo contas: f (−1) = 1 ∈ [0, 1] = f [[0, 1]].

1.13 Exercício 13
Mostre que a função f :A→B é injetiva se, e somente se, f (A − X) = f (A) − f (X) para todo
X ⊂ A.

Solução. O exercício anterior nos dá uma parte desta equivalência: se f é injetiva, então
f [A − X] = f [A] − f [X] qualquer que seja o subconjunto X de A. Reciprocamente, supondo
f [A − X] = f [A] − f [X] para todo X ⊂ A, sejam x e y elementos distintos de A. Queremos
mostrar que f (x) 6= f (y). Para usar a hipótese, podemos considerar, por exemplo, X = {x}.
Assim temos

y ∈ A − {x} ⇒ f (y) ∈ f [A − {x}] = f [A] − f [{x}]


⇒ f (y) ∈/ f [{x}]
⇒ f (y) =
6 f (x),

pois f (x) ∈ f [{x}]. Segue a tese. 


12 CAPÍTULO 1. CONJUNTOS E FUNÇÕES

1.14 Exercício 14
Dada a função f : A → B, prove:

1. f −1 (f (X)) ⊃ X para todo X ⊂ A;


2. f é injetiva se, e somente se, f −1 (f (X)) = X para todo X ⊂ A.

(Voltar para a Observação 1 do Exercício 47 do Capítulo V: página 189.)

(Voltar para o Item iii) do Exercício 14 do Capítulo VII: página 239.)

Solução. Sabemos que se x ∈ X, entãof (x) ∈ f [X] e que se f (x) ∈ Y , então x ∈ f −1 [Y ].


Juntando estas duas implicações para Y = f [X] temos
x ∈ X ⇒ f (x) ∈ f [X] ⇒ x ∈ f −1 [f [X]],
isto é, X ⊂ f −1 [f [X]].
Se f é injetiva,
y ∈ f −1 [f [X]] ⇒ f (y) ∈ f [X]
⇒ existe x ∈ X tal que f (y) = f (x)
⇒ existe x ∈ X tal que y=x
⇒ y ∈ X,
de modo que f −1 [f [X]] ⊂ X e, portanto, f −1 [f [X]] = X , pois X ⊂ f −1 [f [X]] conforme vimos
no parágrafo acima.
Reciprocamente, isto é, supondo f −1 [f [X]] = X para todo X ⊂ A, vamos mostrar que f é
injetiva. Se x, y são elementos de A tais que f (x) = f (y), fazendo X = {x} em nossa hipótese,
temos
f (y) ∈ f [{x}] ⇒ y ∈ f −1 [f [{x}]] = {x} ⇒ y ∈ {x} ⇒ y = x,
logo, f é injetiva. 

1.15 Exercício 15
Dada f : A → B, prove:

1. para todo Z ⊂ B, tem-se f (f −1 (Z)) ⊂ Z ;


2. f é sobrejetiva se, e somente se, f (f −1 (Z)) = Z para todo Z ⊂ B.

(Voltar para a Observação 1 do Exercício 47 do Capítulo V: página 189.)

(Voltar para a Solução do Exercício 9 do Capítulo VII: página 235.)

Solução. y ∈ f [f −1 [Z]], então existe x ∈ f −1 [Z] tal que y = f (x), onde x ∈ f −1 [Z] ⇒
Se
f (x) ∈ Z , logo y ∈ Z , o que prova a inclusão f [f −1 [Z]] ⊂ Z .
−1
Se f é sobrejetiva, dado y ∈ Z , seja x ∈ A tal que y = f (x); x este que pertence a f [Z]
−1 −1
porque f (x) ∈ Z ; portanto y ∈ f [f [Z]]. Logo Z ⊂ f [f [Z]]. Isto, junto com o fato acima
−1
visto de que f [f [Z]] ⊂ Z , implica f [f −1 [Z]] = Z .
−1
Reciprocamente, se f [f [Z]] ⊂ Z para todo Z ⊂ B , então, dado y ∈ B , temos {y} =
f [f [{y}]]. Assim, existe x ∈ f −1 [{y}] tal que y = f (x). Logo f é sobrejetiva.
−1

1.16. EXERCÍCIO 16 13

1.16 Exercício 16
Dada uma família de conjuntos (Aλ )λ∈L , seja X um conjunto com as seguintes propriedades:

a
1 ) para todo λ ∈ L, tem-se X ⊃ Aλ ;
a
2 ) Se Y ⊃ Aλ para todo λ ∈ L, entãoY ⊃ X.
[
Prove que, nestas condições, tem-se X= Aλ .
λ∈L

(Voltar para a Solução do Exercício 18: página 14.)

a a
[ [
Solução. Da 1 propriedade é imediato que Aλ ⊂ X . E, da 2 , como Aλ ⊃ Aµ para

[ λ∈L [ λ∈L
todo µ ∈ L, segue Aλ ⊃ X . Logo, por antissimetria, X= Aλ . 
λ∈L λ∈L

1.17 Exercício 17
\
Enuncie e demonstre um resultado análogo ao anterior, caracterizando Aλ .
λ∈L

Solução. Já zemos algo parecido anteriormente, a saber, no Exercício 2 (página 2). Seja
(Aλ )λ∈L uma família de conjuntos. Armamos que se X está contido em Aλ para todo\λ ∈ L
e se X contém qualquer conjunto que também esteja contido em todo Aλ , então X = Aλ .
λ∈L
De fato,
\
X ⊂ Aλ ∀λ ∈ L ⇒ X ⊂ Aλ
λ∈L
\ \ \
e X⊃ Aλ , uma vez que Aλ ⊂ Aλ para todo λ ∈ L. Logo X= Aλ . 
λ∈L λ∈L λ∈L

(Voltar para a Solução do Exercício 18: página 14.)

1.18 Exercício 18
Seja f : P(A) → P(A) uma função tal que X ⊂ Y ⇒ f (Y ) ⊂ f (X) e f (f (X)) = X . Prove
que f (∪Xλ ) = ∩f (Xλ ) e f (∩Xλ ) = ∪f (Xλ ). [Aqui X , Y e cada Xλ são subconjuntos de A].

Solução 1. Seja
! L um conjunto de índices. Este exercício está pedindo
! para mostrarmos
[ \
que f Xλ é a interseção da família (f (Xλ ))λ∈L e que f Xλ é a reunião desta
λ∈L λ∈L
mesma família. Interessante notar que os dois exercícios anteriores nos dão justamente isso:
uma caracterização para cada um dos dois, a reunião e a interseção de uma dada família.
Consideremos então os seguintes itens:
!
\
(a) f Xλ ⊃ f (Xµ ) para todo µ ∈ L;
λ∈L
14 CAPÍTULO 1. CONJUNTOS E FUNÇÕES

!
\
(b) se Y ⊃ f (Xµ ) para todo µ ∈ L, então Y ⊃f Xλ ;
λ∈L

!
[
(c) f Xλ ⊂ f (Xµ ) para todo µ ∈ L, e;
λ∈L

!
[
(d) se Y ⊂ f (Xµ ) para todo µ ∈ L, então Y ⊂f Xλ .
λ∈L

.
Pelo Exercício 16 (página 13), dos itens (a) e (b) segue

!
\ [
f Xλ = f (Xλ )
λ∈L λ∈L

e, pela Solução do Exercício 17 (página 13), dos itens (c) e (d) segue

!
[ \
f Xλ = f (Xλ ).
λ∈L λ∈L

Para concluirmos o resultado desejado, passemos então para a demonstração de cada um


dos itens acima.

!
\ \
item (a) Dado µ ∈ L, a inclusão f (Xµ ) ⊂ f Xλ vem simplesmente de Xλ ⊂ Xµ
λ∈L λ∈L
através da primeira propriedade de f:
!
\ [
Xλ ⊂ Xµ ⇒ f (Xµ ) ⊂ f Xλ .
λ∈L λ∈L

item (b) De acordo com este item, consideremos um conjunto Y que contém cada f (Xµ ).
!
\
Precisamos mostrar que Y ⊃f Xλ .
λ∈L
Até agora usamos apenas a primeira das propriedades de f. Isto pode ser um indício de
que a segunda propriedade pode ser útil aqui. Vamos tentar aplicá-la. Se Y ⊃ f (Xµ ), então,
aplicando as duas propriedades de
\ f , obtemos f (f (Xµ )) = Xµ ⊃ f (Y ). Isto acontece para todo

µ ∈ L, portanto, Xλ ⊃ f (Y ). Aplicando novamente as propriedades de f, nalmente vem


λ∈L !
\
Y = f (f (Y )) ⊃ f Xλ , como queremos.
λ∈L
Porém há uma falha neste raciocínio: só é legítimo aplicar f em Y se Y estiver no domínio
de f, ou seja, se Y for subconjunto de A. Podemos corrigir esta falha considerando Y ∩ A:

f (Xµ ) ⊂ Y ⇒ f (Xµ ) ∩ A ⊂ Y ∩ A ⇒ f (Xµ ) ⊂ Y ∩ A,


1.18. EXERCÍCIO 18 15

pois f (Xµ )∩A = f (Xµ ), uma vez que f (Xµ ) ⊂


! A por hipótese. A partir daí, seguindo o mesmo
\
caminho do parágrafo acima, vem f Xµ ⊂ Y ∩ A, o qual, por sua vez, é subconjunto de
! λ∈L
\
Y , logo, f Xµ ⊂ Y , como queríamos.
λ∈L
Devido ao paralelismo que há entre os dois objetos centrais dessa questão  f de uma
reunião e f de uma interseção  é de se esperar que as soluções dos itens (c) e (d) sejam
análogas às dos itens (a) e (b), respectivamente.

!
[
item (c) Vale f Xλ ⊂ f (Xµ ) para todo µ∈L porque
λ∈L
!
[ [
Xµ ⊂ Xλ ⇒ f Xλ ⊂ f (Xµ )
λ∈L λ∈L

[
de acordo com a primeira propriedade de f, e Xµ ⊂ Xλ para todo µ ∈ L.
λ∈L

item (d) Seguindo os mesmos passos de seu análogo, o item (b), partindo de
! Y ⊂ f (Xµ ),
[
chegamos a Y ∩A ⊂ f Xµ e não conseguimos dar o passo seguinte do mesmo modo como
µ∈L
foi dado lá. O detalhe aqui é que Y ⊂ f (Xµ ) também implica Y ⊂ A, uma vez que f (Xµ ) ⊂ A
da hipótese de f ser uma função de P(A) em P(A). Assim, temos

Y ⊂ f (Xµ ) ∀µ ∈ L ⇒ Y ∈ P(A) e f (f (Xµ )) ⊂ f (Y ) ∀µ ∈ L


⇒ Xµ ⊂ f (Y ) ∀µ ∈ L
[
⇒ Xµ ⊂ f (Y )
µ∈L
!
[
⇒ f (f (Y )) ⊂ f Xµ
µ∈L
!
[
⇒ Y ⊂f Xµ ,
µ∈L

como queríamos. 

Questão. Podemos nos questionar agora como seria resolver este exercício do modo, digamos,
clássico, convencional, isto é, usando a antissimetria da relação de inclusão. Uma parte é
imediata. De \ [
Xλ ⊂ Xµ ⊂ Xλ ∀µ,
λ λ

obtemos ! !
[ \
f Xλ ⊂ f (Xµ ) ⊂ f Xλ ∀µ,
λ λ
16 CAPÍTULO 1. CONJUNTOS E FUNÇÕES

donde, por causa daquele para todo, vem:

! !
[ \ [ \
f Xλ ⊂ f (Xλ ) e f (Xλ ) ⊂ f Xλ .
λ λ λ λ

Agora só faltam as outras duas inclusões.


!Por exemplo, dado um y ∈ f (Xλ ) para todo
[
índice λ, queremos mostrar que y∈f Xλ . Mas o exercício não diz como trabalhar com
λ
este pertence, o que ele diz é como f se comporta com relação a inclusões. Vamos então
passar do pertence para o incluso:

\
y∈ f (Xλ ) ⇔ y ∈ f (Xµ ) ∀µ
λ
⇔ {y} ⊂ f (Xµ ) ∀µ
⇒ {y} ∈ P(A) e f (f (Xµ )) = Xµ ⊂ f ({y}) ∀µ
[
⇒ Xλ ⊂ f ({y})
λ
!
[
⇒ f (f ({y})) = {y} ⊂ f Xλ
λ
!
[
⇔ y∈f Xλ .
λ

!
[ \
Logo, f Xλ = f (Xλ ).
λ λ
Procedendo de forma análoga com respeito a última inclusão que falta, logo no primeiro
\
passo chegamos a Xλ ⊂ f ({y}) e não conseguimos mais imitar o raciocício acima. Devemos
λ
então procurar uma outra saída. É curioso que a igualdade que acabamos de demonstrar pode
ser usada aqui escrevendo Xλ = f (f (Xλ )), conforme veremos mais abaixo.
\
É interessante notar que as implicações feitas acima para mostrar a inclusão f (Xλ ) ⊂
! λ
[
f Xλ podem ser trocadas por equivalências. Isto porque o que temos na realidade a
λ
respeito de f é

X ⊂ Y ⇔ f (Y ) ⊂ f (X).

Substituir aquelas implicações por equivalências, junto com o desenvolvimento do caminho


indicado no parágrafo acima, tornam as contas feitas no início deste texto redundantes. Disto
surge a solução abaixo.

Solução 2. Começamos observando que

X ⊂ Y ⇔ f (Y ) ⊂ f (X),
1.18. EXERCÍCIO 18 17

pois f (Y ) ⊂ f (X) ⇒ f (f (X)) ⊂ f (f (Y )) ⇒ X ⊂ Y . Assim, temos:


\
y∈ f (Xλ ) ⇔ y ∈ f (Xµ ) ∀µ
λ
⇔ {y} ⊂ f (Xµ ) ∀µ
⇔ {y} ∈ P(A) e f (f (Xµ )) = Xµ ⊂ f ({y}) ∀µ
[
⇔ {y} ∈ P(A) e Xλ ⊂ f ({y})
λ
!
[
⇔ {y} ∈ P(A) e f (f ({y})) = {y} ⊂ f Xλ
λ
!
[
⇔ {y} ⊂ f Xλ
λ
!
[
⇔ y∈f Xλ .
λ
!
\ [
Logo, f (Xλ ) = f Xλ .
λ λ !
\ [
Escrevendo Xλ = f (f (Xλ )), como f (f (Xλ )) = f f (Xλ ) , pela igualdade demons-
λ λ
trada no último parágrafo, temos:
! ! !!
\ \ [ [
f Xλ =f f (f (Xλ )) =f f f (Xλ ) = f (Xλ ),
λ λ λ λ

como queríamos. 

Observações. As contas feitas no início do texto que introduz a Solução 2 parecem bastantes
interessantes para serem descartadas. Manipulando-as mais um pouco e aproveitando as ideias
vistas na solução acima, surge a

Solução 3. Primeiramente, temos


! !
\ [ [ \
Xλ ⊂ Xµ ⊂ Xλ ∀µ ⇒ f Xλ ⊂ f (Xµ ) ⊂ f Xλ ∀µ
λ λ λ λ
! !
[ \ [ \
⇒ f Xλ ⊂ f (Xλ ) e f (Xλ ) ⊂ f Xλ .
λ λ λ λ
!
[
Aplicando mais uma vez a primeira propriedade de f, desta vez à inclusão f Xλ ⊂
\ λ
f (Xλ ), obtemos, em conjunção com a segunda propriedade,
λ
!
\ [
f f (Xλ ) ⊂ Xλ .
λ λ
18 CAPÍTULO 1. CONJUNTOS E FUNÇÕES

Disto, e de Xλ = f (f (Xλ )), vem


! !
\ \ [
f Xλ =f f (f (Xλ )) ⊂ f (Xλ ).
λ λ λ

Logo, por antissimetria, !


\ [
f Xλ = f (Xλ ).
λ λ
Quanto à igualdade que falta, usando a igualdade acima, temos
! ! !!
[ [ \ \
f Xλ =f f (f (Xλ )) =f f f (Xλ ) = f (Xλ ),
λ λ λ λ

o que conclui esta demonstração. 

1.19 Exercício 19
Dadas as famílias (Aλ )λ∈L e (Bµ )µ∈M , forme duas famílias com índices em L×M considerando
os conjuntos
(Aλ ∪ Bµ )(λ,µ)∈L×M e (Aλ ∩ Bµ )(λ,µ)∈L×M .
Prove que se tem ! !
(Aλ ∩ Bµ ) [sic ],
[ [ [
Aλ ∩ Bµ =
λ∈L µ∈M (λµ)∈L×M
! !
(Aλ ∪ Bµ ) [sic ].
\ \ \
Aλ ∪ Bµ =
λ∈L µ∈M (λµ)∈L×M

Solução 1. Temos
! !
[ [ [ [
x∈ Aλ ∩ Bµ ⇔ x∈ Aλ e x∈ Bµ
λ∈L µ∈M λ∈L µ∈M
⇔ existem λ ∈ L e µ ∈ M tais que x ∈ Aλ e x ∈ Bµ
⇔ existe (λ, µ) ∈ L × M tal que x ∈ Aλ ∩ Bµ
[
⇔ x∈ (Aλ ∩ Bµ ),
(λ,µ)∈L×M
! !
[ [ [
logo Aλ ∩ Bµ = (Aλ ∩ Bµ ), e;
λ∈L µ∈M (λ,µ)∈L×M
! !
\ \ \ \
x∈
/ Aλ ∪ Bµ ⇔ x∈
/ Aλ e x∈
/ Bµ
λ∈L µ∈M λ∈L µ∈M
⇔ existem λ ∈ L e µ ∈ M tais que x ∈
/ Aλ e x ∈
/ Bµ
⇔ existe (λ, µ) ∈ L × M tal que x ∈
/ Aλ ∪ Bµ
\
⇔ x∈
/ (Aλ ∪ Bµ ),
(λ,µ)∈L×M
1.19. EXERCÍCIO 19 19

portanto, pela contrapositiva,

! !
\ \ \
x∈ (Aλ ∪ Bµ ) ⇔ x ∈ Aλ ∪ Bµ ,
(λ,µ)∈L×M λ∈L µ∈M

! !
\ \ \
ou seja, Aλ ∪ Bµ = (Aλ ∪ Bµ ). 
λ∈L µ∈M (λ,µ)∈L×M

! !
\ \ \
Considerações. Sejam X = Aλ ∪ Bµ e Y = (Aλ ∪ Bµ ). Acima,
λ∈L µ∈M (λ,µ)∈L×M
usamos a forma contrapositiva da equivalência x∈Y ⇔x∈X Y = para mostrar a igualdade
X . Vejamos como seria sua demonstração em sua forma direta. A implicação x ∈ X ⇒ x ∈ Y
não oferece muitas diculdades, mas, com relação à implicação x ∈ Y ⇒ x ∈ X , podemos
colocar a seguinte questão. Um elemento de Y pertence a Aλ ∪ Bµ para todo (λ, µ) ∈ L × M ;
não poderia acontecer de ele car saltando de A para B  o tempo todo? De modo mais preciso,
dado x ∈ Y não poderia acontecer, por exemplo, de existirem λ1 , λ2 ∈ L e µ1 , µ2 ∈ M tais que
x ∈ Aλ1 , x ∈/ Bµ1 , x ∈
/ Aλ2 e x ∈ Bµ2 ? Isto ainda nos daria x ∈ Aλ1 ∪ Bµ1 e x ∈ Aλ2 ∪ Bµ2 ,
porém x ∈ / Aλ1 ∩ Aλ2 e x ∈ / Bµ1 ∩ Bµ2 . No entanto, se tais índices λ1 , λ2 , µ1 , µ2 existissem,
então também teríamos x ∈ / Aλ2 ∪ Bµ1 , contrariando a hipótese de que x ∈
/ Y . A solução acima
evita o surgimento desta questão e também engloba a inclusão X ⊂ Y . Por outro lado, questão
resolvida, podemos desenrolar a demonstração direta da igualdade Y = X na

! !
\ \
Solução 2 (para a segunda igualdade). Se x ∈ Aλ ∪ Bµ , então x ∈ Aλ
λ∈L µ∈M
para todo λ ∈ L ou x ∈ Bµ para todo µ ∈ M . Se x ∈ Aλ , então x ∈ Aλ ∪ Bµ para todo µ ∈ M,
e; se x ∈ Bµ , então x ∈ Aλ ∪ Bµ para todo λ ∈ L. Portanto,

! !
\ \ \
x∈ Aλ ∪ Bµ ⇒x∈ (Aλ ∪ Bµ ).
λ∈L µ∈M (λ,µ)∈L×M

!
\ \
Reciprocamente, xado x ∈ (Aλ ∪ Bµ ), queremos mostrar que x ∈ Aλ
(λ,µ)∈L×M λ∈L
!
\
ou x ∈ Bµ . Consideremos os únicos dois casos possíveis (no sentido que segue), ou
µ∈M
\ \ \
x∈ Aλ ou x∈
/ Aλ . Se x∈ Aλ , então não há nada a se fazer, já chegamos ao nosso
λ∈L \ λ∈L λ∈L
destino. Se x∈
/ Aλ , seja λ ∈ L tal que x ∈
/ Aλ . Para este λ, por hipótese, temos x ∈ Aλ ∪Bµ
λ∈L
para todo µ∈ M . Portanto, como \ x∈
/ Aλ , de x ∈ Aλ ∪ Bµ para todo µ ∈ M, segue x ∈ Bµ
para todo µ ∈ M, isto é, x∈ Bµ .
µ∈M
Segue a tese. 
20 CAPÍTULO 1. CONJUNTOS E FUNÇÕES

1.20 Exercício 20
Seja (Aij )(i,j)∈N × N uma família de conjuntos com índices em N × N. Prove, ou disprove por
contra-exemplo, a igualdade

∞ ∞
! ∞ ∞
!
sic ].
[ \ \ [
Aij = Aij [
j=1 i=1 i=1 i=1

Solução. Vamos analisar a igualdade

∞ ∞
! ∞ ∞
!
[ \ \ [
Aij = Aij .
j=1 i=1 i=1 j=1

De

\ ∞
[
Ain ⊂ Amj ,
i=1 j=1

para todo n ∈ N, vem


∞ ∞
! ∞
[ \ [
Ain ⊂ Amj ,
n=1 i=1 j=1

para todo m ∈ N, portanto,

∞ ∞
! ∞ ∞
!
[ \ \ [
Ain ⊂ Amj ,
n=1 i=1 m=1 j=1

ou seja (trocando variáveis),

∞ ∞
! ∞ ∞
!
[ \ \ [
Aij ⊂ Aij .
j=1 i=1 i=1 j=1

Agora queremos vericar a veracidade da inclusão inversa. Em outras palavras, dado um


elemento x com a propriedade para todo i ∈ N existe j ∈ N tal que x ∈ Aij , queremos
responder à seguinte pergunta: existe j ∈ N tal que x ∈ Aij para todo i ∈ N? Em para todo
i ∈ N existe j ∈ N tal que x ∈ Aij  o j depende de cada i; assim, por exemplo, para i = 1,
existe j1 ∈ N tal que x ∈ A1j1 e, para i = 2, existe j2 ∈ N tal que x ∈ A2j2 , mas nada garante
que j1 e j2 sejam iguais, eles podem muito bem ser diferentes. Já a frase existe j ∈ N tal que
x ∈ Aij para todo j ∈ N arma algo mais forte, a saber, que um mesmo j funciona para todos
os ii; assim, no exemplo anterior, teríamos j1 = j2 . O fato de a segunda frase armar algo mais
forte do que a primeira é um indício de que a inclusão inversa pode ser falsa, então partimos
em busca de um contra-exemplo.
Com algum displicência, tentamos Aij = {i, j}, Aij = {i} e Aij = {i + j}. Mas, após
algumas contas, cada um desses casos logo mostram-se não funcionar para o que queremos.
Isto nos leva a analisar um pouco mais a fundo o problema. Por um lado queremos um x
satisfazendo a condição  ∀i∈ N ∃j ∈ N tal que x ∈ Aij  e, por outro, que ele também satisfaça
a condição  ∀j ∈ N ∃i ∈ N tal que x ∈
/ Aij . Podemos interpretar esta questão lançando mão de
uma tabela do seguinte modo: dispondo cada Aij na i-ésima linha e na j -ésima coluna, o que
queremos é um x que, em cada linha, apareça em alguma coluna e que, em cada coluna, haja
alguma linha na qual ele não apareça. Uma tal interpretação abre o caminho para diversos
exemplos. Vamos mencionar dois
1.21. EXERCÍCIO 21 21

(a) Aii = {1} e Aij = ∅ se i 6= j ;

(b) Aii = ∅ e Aij = {1} se i 6= j .


∞ ∞
! ∞ ∞
!
\ [ [ \
Em qualquer um deles, temos Aij = {1}, enquanto Aij = ∅.
i=1 j=1 j=1 i=1

[
A11 A12 ··· A1j ··· −→ A1j = {1}
j=1
[∞
A21 A22 ··· A2j ··· −→ A2j = {1}
j=1
. . . .
. . . .
. . . .

[
Ai1 Ai2 ··· Aij ··· −→ Aij = {1}
j=1
. . .
. . .
. . .
↓ ↓ ↓

\ ∞
\ ∞
\
Ai1 = ∅ Ai2 = ∅ ··· Aij = ∅
i=1 i=1 i=1
∞ ∞
! ∞ ∞
!
[ \ \ [
Conclusão: a inclusão Aij ⊂ Aij vale sempre, porém a igualdade
j=1 i=1 i=1 j=1
∞ ∞
! ∞ ∞
!
[ \ \ [
Aij = Aij nem sempre é válida. 
j=1 i=1 i=1 j=1

Observação. Pela solução acima também podemos armar que os quanticadores para todo
e existe nem sempre comutam, as frases  ∃j ∈ N ∀i ∈ N x ∈ Aij  e  ∀i ∈ N ∃j ∈ N x ∈ Aij 
não dizem a mesma coisa.

1.21 Exercício 21
Dados os conjuntos A, B , C , estabeleça uma bijeção entre F(A × B; C) e F(A; F(B; C)).

Solução. Antes de dizer como associar, vamos trabalhar com a questão de saber o que é
que estamos querendo associar. Da análise de como são os elementos dos conjuntos a serem
associados vai surgir naturalmente qual é a função a ser considerada.
Um elemento de F(A × B; C)
f : A × B → C . Fixemos uma tal função. A
é uma função
ela queremos associar uma função de A em F(B; C). Para cada a ∈ A, temos f (a, y) ∈ C para
todo y ∈ B . E isto dene uma função de B em C . Subjacente a essas considerações há uma
outra função. Vamos explicitá-la:

Φ : F(A × B; C) → F(A; F(B; C))


f 7→ Φ(f ) : A → F(B; C)
a 7→ Φ(f )(a) : B → C
b 7→ Φ(f )(a)(b) = f (a, b).
22 CAPÍTULO 1. CONJUNTOS E FUNÇÕES

De modo mais sucinto, temos Φ : F(A × B; C) → F(A; F(B; C)) denida por Φ(f )(a)(b) =
f (a, b) ∀f ∈ F(A × B; C) ∀a ∈ A ∀b ∈ B . Ela é a bijeção pedida por este exercício, conforme
demonstraremos abaixo.

Φ é injetiva. Qual é o objetivo? Dadas f, g ∈ F(A × B; C) distintas, queremos mostrar


que Φ(f ) 6= Φ(g), ou seja, que existe a ∈ A tal que Φ(f )(a) 6= Φ(g)(a). Para isto, por sua
vez, precisamos encontrar b ∈ B tal que Φ(f )(a)(b) 6= Φ(g)(a)(b). Mas este último equivale a
f (a, b) 6= g(a, b). Saber qual é o objetivo torna o caminho a ser seguido mais claro. Como f e
g são funções distintas, seja (a, b) ∈ A × B tal que f (a, b) 6= g(a, b). Para este par ordenado,
temos:

f (a, b) 6= g(a, b) ⇒ Φ(f )(a)(b) 6= Φ(g)(a)(b) ⇒ Φ(f )(a) 6= Φ(g)(a) ⇒ Φ(f ) 6= Φ(g),

como queríamos.

Φ é sobrejetiva. g ∈ F(A; F(B; C)), queremos f ∈ F(A × B; C) tal que Φ(f ) = g .


Dada
Precisamos denir esta função f , uma função de A × B em C . E a única coisa que temos
em mão é g . Vamos analisá-la então para saber o que fazer. Esssa g é uma função de A em
F(B; C); assim, g(a) é uma função de B em C ∀a ∈ A e, portanto, g(a)(b) ∈ C ∀a ∈ A e
∀b ∈ B . A f que queremos denir também deve ser tal que f (a, b) ∈ C ∀(a, b) ∈ A × B . Vamos
denir então f : A × B → C fazendo f (a, b) = g(a)(b) ∀(a, b) ∈ A × B . Para uma f assim
denida, como f (a, b) = Φ(f )(a)(b), temos:

Φ(f )(a)(b) = g(a)(b) ∀(a, b) ∈ A × B ⇔ Φ(f )(a) = g(a) ∀a ∈ A ⇔ Φ(f ) = g,

como queríamos. 
Capítulo 2
Conjuntos Finitos, Enumeráveis e
Não-Enumeráveis

2.1 Exercício 1
Prove que, na presença dos axiomas P1 e P2, o axioma (A) abaixo é equivalente a P3. (A)
Para todo subconjunto não-vazio A ⊂ N, tem-se A − s(A) 6= ∅.

Solução.

Nível 1. Utilizando os axiomas P1 e P2, temos duas implicações a demonstrar, a saber:


P3 ⇒ (A) (Nível 2.1) e (A) ⇒ P3 (Nível 2.2).

Nível 2.1: P3 implica (A). Suponha P3. Seja A um subconjunto não vazio de N.
Queremos encontrar um número a∈A tal que a 6= s(b) ∀b ∈ A.
O menor elemento de A possui essa propriedade (a demonstração disto está no Nível 3.1).
A questão agora é: podemos tomar o menor elemento de A? Revisitando a seção que trata
do princípio da boa ordenação, mais especicamente o Teorema 1, nós vemos que sim, que, de
fato, P3 implica a existência de um menor elemento de A. Logo, se a é este menor elemento,
então a ∈ A − s[A], como queríamos.

Nível 2.2: (A) implica P3. A, vamos demonstrar a contrapo-


Supondo a propriedade
sitiva de P3, que sabemos ser equivalente a P3. Assim, seja X um subconjunto próprio de N.
Queremos mostrar que 1 ∈ / X ou encontrar um n ∈ X tal que s(n) ∈ / X.
Se X é um subconjunto próprio de N, então N −X 6= ∅. Surge então o nosso candidato a
ser o conjunto A da propriedade (A). Aplicando esta propriedade, seja y ∈ (N −X) − s[N −X].
Em outras palavras, y ∈ N, y ∈/ X e y 6= s(x) ∀x ∈ N −X . Se y = 1, então temos o resultado
desejado. Se y 6= 1, podemos considerar x0 ∈ N tal que y = s(x0 ). Armamos que x0 ∈ X (cuja
demonstração está no Nível 3.2). Assim temos um x0 ∈ X tal que s(x0 ) ∈ / X , como queríamos.

Nível 3.1: Lema. Sea é o menor elemento de um conjunto A, então a 6= s(b) ∀b ∈ A.


Solução. De fato, se a = s(b) para algum b ∈ A, então b < a  pelo modo como foram
denidas a ordem e a soma  e, portanto, a não pode ser o menor elemento de A. O resultado
desejado agora segue pela contrapositiva.

23
24 CAPÍTULO 2. CONJUNTOS FINITOS, ENUMERÁVEIS E NÃO-ENUMERÁVEIS

Nível 3.2: Armação. Dado X ⊂ N, sejam y ∈ N −X tal que y 6= s(x) ∀x ∈ N −X e


x0 ∈ N . Se y = s(x0 ), então x0 ∈ X .
Solução. Simplesmente porque x0 ∈ N −X ⇒ y 6= s(x0 ). 

2.2 Exercício 2
Dados os números naturais a, b, prove que existe um número natural m tal que m · a > b.

Solução. Negando a tese, isto é, supondo b > m·a para todo m ∈ N, o conjunto M =
{m · a; m ∈ N}, não vazio e de números naturais, seria limitado (por denição) e, portanto,
pelo Teorema 5, M M não é
seria um conjunto nito. Porém, conforme veremos mais abaixo,
um conjunto nito, ele é um conjunto innito. Assim, como b > m · a para todo m ∈ N implica
M nito, pela contrapositiva, M innito implica a existência de um m ∈ N tal que m · a > b,
como queríamos.

Um modo de justicar que M é innito é usando a função f : M → N, denida por


f (m · a) = m para todo m ∈ N. Ela é claramente sobrejetiva, portanto, pela observação feita
no m da página 45 (consequência do Corolário 2 do Teorema 4), a innitude de M vem do
fato de N ser innito. 

Observações. Outro modo imediato de justicar que M é innito é usando a função g:N→
M denida por g(m) = m·a para todo m ∈ N; ela é injetiva pela lei do corte da multiplicação.
Ainda usando as propriedades da multiplicação, poderíamos também ter usado o Teorema 5 de
outra forma, usando o fato de que M não admite um maior elemento, pela monotonicidade da
multiplicação.

2.3 Exercício 3
Seja a um número natural. Se um conjunto X é tal que a ∈ X e, além disso, n ∈ X ⇒ n+1 ∈ X ,
então X contém todos os números naturais > a.

(Voltar para a Solução do Exercício 6: página 33.)

(Voltar para a Solução do Exercício 12, para sua Armação: página 43.)

Solução.

Nível 1. Pela forma contrapositiva, vamos supor que existe um x>a que não pertença
a X. Vamos concluir que a∈
/X ou encontrar um n∈X tal que n+1∈
/ X.

No elevador. Pelo princípio da boa ordem vamos poder tomar o menor x>a que não
pertence a X de modo que seu antecessor n pertence a X , mas x = s(n) ∈
/ X , como queremos.
Mas aqui é sempre aquele cuidado: podemos considerar esse antecessor? Um jeito de arrumar
isso é simplesmente supor a > 1. Para um tal a temos: x > a ⇒ x 6= 1.
2.4. EXERCÍCIO 4 25

Nível 2. Para a = 1 o resultado é verdadeiro, ele é simplesmente o princípio da indução.


Vamos supor a > 1. O conjunto Y = {x > a; x ∈ / X} é não vazio por hipótese. Seja x seu
menor elemento. Como x > a > 1, segue x 6= 1 e, portanto, existe n ∈ N tal que x = s(n).
Como n < x, segue n ∈ / Y . Assim n < a ou n ∈ X . Se n < a, então x = a (a demonstração
disto está no Nível 3) e a ∈
/ X . Se n ∈ X , então temos n ∈ X tal que n + 1 = s(n) = x ∈ / X.
De qualquer forma temos o resultado desejado.

Nível 3: Lema. Se n<a e s(n) > a, então s(n) = a.


Solução. Vamos mostrar a contrapositiva desse lema. Pela tricotomia, vamos supor s(n) < a
ou s(n) > a. Queremos concluir que n > a ou s(n) < a.
Se s(n) < a, então não há nada a fazer; de fato teremos o resultado desejado: n > a ou
s(n) < a. Se s(n) > a, então, por denição, existe um p ∈ N tal que s(n) = a + p. Se p = 1,
então s(n) = a + 1 = s(a) ⇒ n = a. Se p > 1, seja q ∈ N tal que p = s(q), o que nos dá:

s(n) = a + p = a + s(q) = s(a + q) ⇒ n = a + q ⇒ n > a,

como queríamos. 

Questão. Como seria demonstrar o exercício acima pelo princípio da indução?


Para tanto, num primeiro momento precisamos encontrar em qual objeto aplicar a indução.
Na solução acima, vimos que o resultado é verdadeiro para a = 1. Então, que tal aplicar aplicar
a indução em a?
m, ou seja, que qualquer conjunto que possua
Vamos supor que o resultado é verdadeiro para
o m e possua os sucessores de seus elementos possui todos os elementos maiores ou iguais a m.
Fixado um conjunto X tal que m + 1 ∈ X e tal que n ∈ X ⇒ n + 1 ∈ X , queremos mostrar
que todo x > m + 1 pertence a X .
Se m ∈ X , então o resultado é verdadeiro pela nossa hipótese de indução. Se m ∈/ X , então
consideremos o conjunto Y = {m} ∪ X . Temos m ∈ Y e n ∈ Y ⇒ n + 1 ∈ Y . Portanto x ∈ Y
∀x > m. Se x > m + 1, então x 6= m e, assim, como x ∈ Y , só pode ser x ∈ X . Segue a tese.
Isto constitui uma segunda solução para o exercício acima.

2.4 Exercício 4
Tente descobrir, independentemente, algumas das demonstrações omitidas no texto. Caso não
consiga alguma, consulte um dos livros aqui citados, ou outros de sua predileção. [ Sugestão :
Praticamente todas as proposições sobre N se demonstram por indução.]

Solução.

Teorema da Denição por Indução. Fixada a função f : X → X , X 6= ∅, trata-se


n
de associar de modo único cada número natural n a uma função f : X → X de maneira que
f 1 = f e f s(n) = f ◦ f n (página 35 do livro). Mais adiante, já em outro contexto, o autor sugere
uma consulta ao livro Naive Set Theory do Halmos. Neste livro também podemos encontrar
o

Teorema da Recursão. Dados um conjunto não vazio X , um elemento a ∈ X e


uma função f : X → X , existe uma única função ϕ : N → X tal que ϕ(1) = a e
ϕ(s(n)) = f (ϕ(n)) para todo n ∈ N
26 CAPÍTULO 2. CONJUNTOS FINITOS, ENUMERÁVEIS E NÃO-ENUMERÁVEIS

(voltar para as Observações do Exercício 23: página 55);

(voltar para as Observações do Exercício 26: página 58);

(voltar para a Solução do Exercício 27: página 59),

cuja aplicação Halmos também chama de denição por indução. Vejamos então como podemos
utilizar este teorema e o demonstremos em seguida.
Demonstração do Teorema da Denição por Indução usando o Teorema da Recursão. Para
cada x ∈ X , seja a função ϕx : N → X satisfazendo ϕx (1) = x e ϕx (s(n)) = f (ϕx (n)) para
todo n ∈ N. Assim, dado n ∈ N, denimos

fn : X → X
x 7→ f n (x) = ϕx (n + 1).

Esta denição nos dá: f 1 (x) = ϕx (1 + 1) = ϕx (s(1)) = f (ϕx (1)) = f (x), e; f s(n) (x) =
ϕx (s(n) + 1) = ϕx (s(s(n))) = f (ϕx (s(n))) = f (ϕx (n + 1)) = f (f n (x)) = (f ◦ f n )(x), logo,
f 1 = f e f s(n) = f ◦ f n , o que conclui uma parte deste Teorema da Denição por Indução.
n 1
Com relação à outra parte, à unicidade, para cada n ∈ N seja g : X → X satisfazendo g = f
s(n)
e g = f ◦ g n ; queremos mostrar que g n = f n para todo n ∈ N. Vamos fazer isto por indução
n n 1 1
em n (considerando o conjunto {n ∈ N; g = f }): para n = 1, temos g = f = f , e; se
n n s(n) n n s(n) n n
g = f , então g = f ◦ g = f ◦ f = f . Logo, de fato, g = f para todo n ∈ N, como
queríamos. Está demonstrada então a armação da página 35.
Notemos que as contas acima também implicam a unicidade de ϕ: dado x ∈ X , se ψx : N →
n
X é tal que ψx (1) = x e ψx (s(n)) = f (ψx (n)) para todo n ∈ N, então, denindo g : X → X
n
por g (x) = ψx (n + 1) para todo n ∈ N e para todo x ∈ X , pelas mesmas contas do começo
1 s(n)
do último parágrafo, temos g = f e g = f ◦ g n e, portanto (pela unicidade de f n ), g n = f n
n n
para todo n ∈ N, de modo que ψx (n + 1) = g (x) = f (x) = ϕx (n + 1) para todo x ∈ X e para
todo n ∈ N, além do que, ψx (1) = x = ψx (1), ou seja ψx (n) = ϕx (n) para todo n ∈ N, donde
ψx = ϕx .
Demonstração do Teorema da Recursão. Fixado o elemento a ∈ X, a construção que
daremos da função ϕ é essencialmente a mesma encontrada no já referido livro do Halmos.
Vamos denir ϕ a partir de um subconjunto de N ×X . Para tanto, dado A ⊂ N ×X , sejam os
itens

(a) (1, a) ∈ A, e;

(b) (n, y) ∈ A ⇒ (s(n), f (y)) ∈ A

(estes itens referem-se às propriedades que queremos que ϕ possua) e seja (Aλ )λ∈L uma família
P(N ×X) tal que, para cada λ ∈ L, os itens (a) e (b) valham para A\
de elementos de = Aλ e, se
A ⊂ N ×X satisfaz os itens (a) e (b), então existe λ ∈ L tal que A = Aλ . Seja Xa = Aλ (L é
λ∈L
não vazio porque (a) e (b) valem para A = N ×X ). Por indução é imediato que, para todo n ∈ N,
existey ∈ X tal que (n, y) ∈ Xa . Além disto, armamos que, dado n ∈ N, existe um único
y ∈ X tal que (n, y) ∈ Xa . Temos assim tudo que é necessário para que de Xa surja a função
ϕ; para cada n ∈ N, denimos ϕ(n) como aquele único y ∈ X tal que (n, y) ∈ Xx . Deste modo:
ϕ(1) = a, pois (1, a) ∈ Xa , e ϕ(s(n)) = f (ϕ(n)), pois (n, ϕ(n)) ∈ Xa ⇒ (s(n), f (ϕ(n))) ∈ Xa .
Logo, ϕ possui todas as propriedades pretendidas, só restando justicar a armação feita.
Vamos demonstrá-la por indução em n. Assim, seja Y = {n ∈ N; existe um único y ∈
X tal que (n, y) ∈ Xa }. Para mostrar que 1 ∈ Y , denindo Z = {1}×(X −{f (x)}), armamos
2.4. EXERCÍCIO 4 27

\
que existe λ ∈ L tal que Xa − Z = A λ . Em existindo este λ, como Xa = Aµ , teremos
µ∈L
Xa ⊂ Xa − Z , donde seguirá Xa ∩ Z = ∅, ou seja, que não existe y ∈ X distinto de f (x) tal que
(1, y) ∈ Xa , como desejamos. Para chegarmos a existência de um tal λ precisamos mostrar que
os itens (a) e (b) são válidos para A = Xa − Z : (1, a) ∈ Xa , (1, f (x)) ∈
/ Z e, se (n, y) ∈ Xa − Z ,
então (s(n), f (y)) ∈ Xa e (s(n), f (y)) ∈
/ Z , porque s(n) 6= 1. Logo, de fato existe λ ∈ L tal que
Xa − Z = Aλ e podemos concluir que 1 ∈ Y .
O passo de indução é semelhante. Seja n ∈ Y , seja y ∈ X o único elemento de X tal que
(n, y) ∈ Xa e seja W = {s(n)} × (X − {f (y)}). Quanto a este W , (1, f (x)) não pertence a ele
porque 1 6= s(n), e, se (k, z) ∈ Xa − W , vejamos que (s(k), f (z)) ∈ / W : se k = n, então z = y
(pela unicidade de y ) e (s(k), f (z)) = (s(n), f (y)) ∈
/ W , pois f (y) ∈
/ X − {f (y)}, e; se k 6= n,
então s(k) 6= s(n) e, portanto, (s(k), f (z)) ∈ / W . Logo, existe λ ∈ L tal que Xa − W = Aλ .
Disto vem Xa ⊂ Xa − W , donde W ∩ Xa = ∅, ou seja, não existe z ∈ X distinto de f (y) tal
que (s(n), z) ∈ Xa , portanto s(n) ∈ Y . Logo, como 1 ∈ Y e n ∈ Y ⇒ s(n) ∈ Y , Y = N; em
outras palavras, para todo n ∈ N, existe um único y ∈ X tal que (n, y) ∈ Xa , como queríamos.
Com respeito à unicidade de um tal ϕ, já vimos um modo de justicá-la no parágrafo
n
precedente a esta demonstração, passando pela função f . Um outro modo de fazê-lo é por
indução. Sejam então ϕ, ψ : N → X funções tais que ϕ(1) = a = ψ(1), ϕ(s(n)) = f (ϕ(n)) e
ψ(s(n)) = f (ψ(n)), para todo n ∈ N. Sendo S = {n ∈ N; ϕ(n) = ψ(n)}, temos 1 ∈ S , e

n ∈ S ⇒ ϕ(n) = ψ(n) ⇒ f (ϕ(n)) = f (ψ(n)) ⇒ ϕ(s(n)) = ψ(s(n)) ⇒ s(n) ∈ S,

logo, pelo Princípio da Indução, S = N, isto é, ψ = ϕ, como queríamos. 

Propriedades da adição.
Comutatividade. Seja X = {n ∈ N; n + m = m + n ∀m ∈ N}. Vamos mostrar que 1∈X e
que n ∈ X ⇒ s(n) ∈ X . Assim, pelo Princípio da Indução, seguirá X = N, ou seja, que todo
número natural comuta com todo número natural.
Começando por mostrar que 1 ∈ X , queremos mostrar que 1 + m = m + 1 para todo m ∈ N.
Façamos isto por indução. Para m = 1 é imediato (se x = y , então é claro que x + y = y + x).
Supondo agora que 1 + m = m + 1, vejamos que 1 + s(m) = s(m) + 1:

1 + s(m) = s(1 + m) = s(m + 1) = (m + 1) + 1 = s(m) + 1

(onde as primeira, terceira e quarta igualdades valem por denição e a segunda, por hipótese).
Então 1 + m = m + 1 para todo m ∈ N, o que permite o acesso do número 1 ao conjunto X .
Supondo n+m = m+n para todo m ∈ N, o objetivo agora é mostrar que s(n)+m = m+s(n)
para todo m ∈ N. Temos

s(n) + m = (n + 1) + m
(Associatividade) = n + (1 + m)
(1 ∈ X) = n + (m + 1)
(Associatividade) = (n + m) + 1
(Hipótese) = (m + n) + 1
(Associatividade) = m + (n + 1)
(Denição) = m + s(n),

logo, de fato s(n) ∈ X se n ∈ X, o que conclui esta demonstração.


28 CAPÍTULO 2. CONJUNTOS FINITOS, ENUMERÁVEIS E NÃO-ENUMERÁVEIS

Uma brincadeira. Compare as contas acima com a seguinte brincadeira. Você tem três
cartões, o cartão A, o cartão B e o cartão C. Eles estão dispostos em la numa mesa na
seguinte ordem: na esquerda está o A, no meio, o B e na direita, o C. O objetivo é transferir
o cartão C para a primeira posição da la, cando então o C na esquerda, o A no meio e o
B na direita. Para tanto, o único movimento que você pode fazer é o de inverter a ordem de
dois cartões adjacentes. O uso dos parênteses no desenvolvimento das contas acima é como
se primeiro você estivesse indicando qual par de cartões você escolheu antes de invertê-los de
ordem.
Lei do corte. Dados os números naturais n e p, temos

1 + n = 1 + p ⇒ n + 1 = p + 1 ⇒ s(n) = s(p) ⇒ n = p,

primeiro, comutatividade; segundo, denição, e; terceiro, injetividade da função s. Supondo


agora a implicação m + x = m + y ⇒ x = y, na qual x, y ∈ N, dados os naturais n e p temos

s(m) + n = s(m) + p ⇒ (m + 1) + n = (m + 1) + p
(Associatividade) ⇒ m + (1 + n) = m + (1 + p)
(Hipótese) ⇒ 1+n=1+p
⇒ n = p.

Logo, dados n, p ∈ N, m + n = m + p ⇒ n = p vale para todo m ∈ N.


a implicação
Tricotomia. Vamos dizer que um dado número n ∈ N pode ser comparado com um número
m ∈ N quando m = n, ou existir p ∈ N tal que m = n + p, ou existir q ∈ N tal que n = m + q .
Seja X o conjunto dos números naturais que podem ser comparados com todos os números
naturais. Num primeiro momento, por indução, vamos mostrar que X = N. Num segundo
momento, vamos mostrar que pode acontecer apenas um desses três casos.
Primeiro momento. m um número natural. Se m = 1, então o número 1 pode ser
Seja
comparado com m. Se m 6= 1, para k ∈ N tal que m = s(k) (propriedade P2 dos números
naturais), temos m = s(k) = k + 1 = 1 + k (por denição e pela comutatividade), ou seja, se
m 6= 1, então o 1 também pode ser comparado com m. Logo 1 ∈ X .
Se n ∈ X , dado m ∈ N, queremos mostrar que s(n) pode ser comparado com m. Como n
pode ser comparado com todo número natural (por hipótese), ou m = n, ou existe p ∈ N tal
que m = n + p, ou existe q ∈ N tal que n = m + q . Vejamos cada um desses casos:

m = n ⇒ s(m) = s(n) ⇒ s(n) = m + 1;

m = n + p ⇒ s(m) = s(n + p) = s(n) + p ⇒ m + 1 = s(n) + p.


Se p = 1, então, pela lei do corte, m + 1 = s(n) + p ⇒ m = s(n). Se p = s(k) = k + 1, então,
novamente pela lei do corte, m + 1 = s(n) + p ⇒ m = s(n) + k , e;

n = m + q ⇒ s(n) = s(m + q) = m + (q + 1).

De qualquer um desses casos segue s(n) ∈ X , como queríamos (nas contas deste parágrafo
usamos a denição de que s(x) = x+1 para todo x ∈ N, a denição da adição, a comutatividade
e a associatividade, além da já explicitada lei do corte).
Logo, pelo Princípio da Indução, de 1 ∈ X e da implicação n ∈ X ⇒ s(n) ∈ X segue
X = N, o que conclui este primeiro momento.
Segundo momento. Se m = n, armamos que não existe p ∈ N tal que m = n + p e nem
existe q∈N tal que n = m + q . Isto acontece porque sp (n) 6= n para todo p ∈ N e sq (m) 6= m
2.4. EXERCÍCIO 4 29

para todo q ∈ N. Já vimos anteriormente algo semelhante a estas duas últimas armações, a
saber, no primeiro exemplo de uma demonstração por indução do livro, no qual foi mostrado
que x 6= s(x) para todo x ∈ N. De modo semelhante, xado k ∈ N, pela injetividade de s,
x 6= sk (x) ⇒ s(x) 6= s(sk (x)), onde
s(sk (x)) = sk (x) + 1
(Denição da adição) = (x + k) + 1
(Comutatividade) = (k + x) + 1
(Associatividade) = k + (x + 1)
(Denição) = k + s(x)
(Comutatividade) = s(x) + k
(Denição da adição) = sk (s(x)),
ou seja, x 6= sk (x) ⇒ s(x) 6= sk (s(x)) e, por indução, podemos concluir então que, xado k ∈ N,
x 6= s (x) para todo x ∈ N. Portanto, como este k era qualquer, xado x ∈ N, x 6= sk (x) para
k
p q
todo k ∈ N. Em particular, n 6= s (n) para todo p ∈ N e m 6= s (m) para todo q ∈ N, como
queríamos.
Os outros dois casos a serem analisados também vão depender do fato demonstrado acima
de que
x 6= sk (x) ∀x ∈ N ∀k ∈ N .
Se m = n + p, então m 6= n, pois m = sp (n) 6= n, e; n 6= m + q para todo q ∈ N porque
(m + q) + p = m + (q + p) = sq+p (m) 6= m para todo q ∈ N (n = m + q implicaria sq+p (m) = m,
o que é absurdo). Analogamente, se n = m + q , então n 6= m e m 6= n + p para todo p ∈ N.
Segue a tese. 

Propriedades da relação de ordem.


Transitividade. Dados m, n, p ∈ N, se m < n e n < p, então, por denição, existem a ∈ N
e b ∈ N tais que n = m + a e p = n + b. Substituindo n = m + a em p = n + b, obtemos
p = (m + a) + b = m + (a + b), pela associatividade. Portanto, novamente pela denição da
relação de ordem, m < p.
Tricotomia. Isto é consequência da tricotomia da adição vista mais acima. Sejam m, n ∈
N. Se m = n, então não existe p ∈ N tal que m = n + p e nem existe q ∈ N tal que n = m + q ,
ou seja, não vale nem n < m e nem m < n. Se existe p ∈ N tal que m = n + p, então: n < m,
m 6= n e não vale m < n, pois não existe q ∈ N tal que n = m + q . O caso em que existe q ∈ N
tal que n = m + q é análogo. 

Propriedades da multiplicação.
Associatividade. Dados m, n, p ∈ N, queremos mostrar que m · (n · p) = (m · n) · p. Para
p = 1 é imediato: m · (n · 1) = m · n = (m · n) · 1. Supondo agora m · (n · p) = (m · n) · p, vamos
mostrar que m · (n · s(p)) = (m · n) · s(p):

m · (n · s(p)) = m · (n · p + n)
(Distributividade) = m · (n · p) + m · n
(Hipótese) = (m · n) · p + m · n
(Denição) = (m · n) · s(p),
onde a primeira igualdade também vale por denição, uma vez que n · s(p) = n · p + n. Logo,
por indução, dados m, n ∈ N, m · (n · p) = (m · n) · p para todo p ∈ N, como queríamos.
30 CAPÍTULO 2. CONJUNTOS FINITOS, ENUMERÁVEIS E NÃO-ENUMERÁVEIS

Comutatividade. Dado m ∈ N, queremos mostrar que m · n = n · m para todo n ∈ N. Para


tanto, vamos mostrar que m · 1 = 1 · m e que m · n = n · m ⇒ m · s(n) = s(n) · m.
Como m · 1 = m, vamos mostrar que m = 1 · m para todo m ∈ N: para m = 1, 1 · m =
1 · 1 = 1 = m, e; se m = 1 · m, então 1 · s(m) = 1 · m + 1 = m + 1 = s(m). Logo m = 1 · m para
todo m ∈ N, como queríamos.
Se m · n = n · m, então:

m · s(n) = m · n + m = n · m + m = n · m + 1 · m = (n + 1) · m = s(n) · m,

como queríamos. A penúltima igualdade valendo pela distributividade, já demonstrada no livro


(demonstrada sem o uso das propriedades da multiplicação). Segue a tese.

. Lei do corte. Dados m, n, p ∈ N, m·p = n·p ⇒


o objetivo aqui é mostrar a implicação
m = n. Tentando o caminho da indução, para p = 1, temos m · 1 = n · 1 ⇒ m = n, mas, em
m · s(p) = n · s(p) ⇒ m · p + m = n · p + n, como usar a hipótese de que x · p = y · p implica
x = y ? Tentemos então outro caminho.
Pela contrapositiva, supondo m 6= n, vamos mostrar que m · p 6= n · p. Se m 6= n, então,
pela tricotomia, existe q ∈ N tal que m = n + q ou tal que n = m + q . No primeiro caso temos
m · p = (n + q) · p = n · p + q · p. Número este que, novamente pela tricotomia, não é igual a
n · p, portanto, m · p 6= n · p, como queríamos. O outro caso é análogo: n = m + q ⇒ n · p =
(m + q) · p = m · p + q · p 6= m · p.
(Voltar para a Observação do Exercício 8 do Capítulo III: página 71.)

Monotonicidade. Dados m, n, p ∈ N, queremos mostrar que m < n ⇒ m · p < n · p. Notemos


m < n,
que, implicitamente, esta implicação já foi contemplada na Lei do corte logo acima. Se
então, por denição, existe q ∈ N tal que n = m+q e, assim, n·p = (m+q)·p = m·p+q·p > m·p,
como queríamos demonstrar.
Claro que também podemos tentar uma segunda solução, para este item da monotonicidade,
por indução. Vejamos. Parap = 1, não há nada a se fazer. Sejam agora m e n números naturais
com m < n. Supondo m · p < n · p, vamos mostrar que m · s(p) < n · s(p). Pela monotonicidade
da adição temos

m · p < n · p ⇒ m · p + m < n · p + m ⇒ m · s(p) < n · p + m < n · p + n = n · s(p),

novamente pela monotonicidade da adição, pois m < n ⇒ n · p + m < n · p + n. Portanto, pelo


Princípio da Indução, dados m, n ∈ N, m < n ⇒ m · p < n · p para todo p ∈ N. Isto nos
temos
dá uma segunda solução para a monotonicidade da multiplicação. 

Teorema Fundamental da Aritmética (unicidade). Ver o Exercício 7 (página 33).

Método geral de denição por indução.


[ Enunciado. Sejam um conjunto não vazio

X, um elemento a∈X e uma função g: X n → X . Existe uma única função f : N → X tal


n∈N 
que f (1) = a e f (s(n)) = g (f (1), . . . , f (n)) para todo n ∈ N (g é a regra que permite obter
f (n) a partir dos valores de f (m), com m < n).
(Voltar para as Observações do Exercício 23: página 55.)

(Voltar para as Observações do Exercício 26: página 58.)

A armação de que esta tese é válida encontra-se na página 41 do livro e, para ela, o autor
sugere o livro Fundamentals of Abstract Analysis, de Gleason. É deste livro que tiramos a
2.4. EXERCÍCIO 4 31

ideia central da demonstração que daremos abaixo. Felizmente vamos poder aplicar o Teorema
da Recursão já enunciado e demonstrado na primeira parte deste exercício, no Teorema da
Denição por Indução.

Construção/Existência.
[ [
Seja h: Xn → Xn a função denida por
n∈N n∈N
  
h (x1 , . . . , xn ) = x1 , . . . , xn , g (x1 , . . . , xn )
para todo N e toda n-úpla (x1 , . . . , xn ) ∈ X n .
n ∈[ Deste modo, pelo Teorema da Recursão,

seja ϕ:N→ X n uma função tal que ϕ(1) = a e ϕ(s(n)) = h(ϕ(n)). Por um lado, temos,
n∈N
por exemplo,
ϕ(1) = a,
ϕ(2) = h(ϕ(1)) = (a,
 g(a)), 
ϕ(3) = h(ϕ(2)) = a, g(a), g (a, g(a)) ,
e, por outro, queremos f satisfazendo

f (1) = a,
f (2) = g(f (1)) = g(a), 
f (3) = g (f (1), f (2)) = g (a, g(a)) ;
ϕ(n) nos desse um pedaço da função f pretendida. Assim, denimos
quer dizer, é como se cada
f :N→X f (n) = ϕ(n)(n) para todo n ∈ N  por indução é imediato que cada ϕ(n)
fazendo
é de fato uma n-úpla de elementos de X , ou seja, uma função de {1, . . . , n} em X .
Vericação. Vejamos que f possui todas as propriedades desejadas. Primeiro, f (1) =
ϕ(1)(1) = a (estamos identicando o elemento a com a sequência (a)) e, segundo,
f (s(n)) = ϕ(s(n))(s(n)) = h(ϕ(n))(s(n)) = g(ϕ(n)),
só restando vericar que ϕ(n) = (f (1), . . . , f (n)) para todo n ∈ N. Façamos isto por indução
em n: ϕ(1) = a = ϕ(1)(1) = f (1), e ϕ(n) = (f (1), . . . , f (n)) implica
ϕ(s(n)) = h(ϕ(n))
= h((f (1), . . . , f (n)))
 
= f (1), . . . , f (n), g (f (1), . . . , f (n))
 
= f (1), . . . , f (n), g ϕ(n)

= f (1), . . . , f (n), f (s(n)) .
Essas contas signicam que, denindo Y = {n ∈ N; ϕ(n) = (f (1), . . . , f (n))}, 1 ∈ Y e n ∈
Y ⇒ s(n) ∈ Y . Logo, pelo Príncipio da Indução, Y = N e ϕ(n) = (f (1), . . . , f (n)) para todo
n ∈ N, como queríamos. Agora podemos concluir a conta começada mais acima,

f (s(n)) = g(ϕ(n)) = g (f (1), . . . , f (n)) ,
para todo n ∈ N, o que também conclui esta vercação.
Unicidade.

Seja ψ : N → X tal que ψ(1) = a e ψ(s(n)) = g (f (1), . . . , f (n)) para todo
n ∈ N. Assim, ψ(1) = a = f (1) e

ψ(s(n)) = g (f (1), . . . , f (n)) = f (s(n)) ∀n ∈ N
Logo, ψ = f (observemos que não foi usado o Princípio da Indução aqui; o que zemos foi
o seguinte: o objetivo era mostrar que ψ(m) = f (m) para todo m ∈ N, então dividimos o
problema em dois casos, a saber, ou m = 1 ou m = s(n) para algum n ∈ N). 
32 CAPÍTULO 2. CONJUNTOS FINITOS, ENUMERÁVEIS E NÃO-ENUMERÁVEIS

2.5 Exercício 5
Um elemento a ∈ N chama-se antecessor de b ∈ N quando se tem a < b mas não existe c ∈ N
tal que a < c < b. Prove que, exceto 1, todo número natural possui um antecessor.

(Voltar para a Solução do Exercício 23, para a Unicidade: página 55.)

Solução.

Nível 1. Pela nossa experiência, já sabemos quem é um bom candidato a ser o antecessor
de um dado número b, é o número a tal que s(a) = b. Só nos resta agora mostrar que, de fato,
a possui as propriedades que o fazem ser um antecessor de b conforme a denição do exercício.
A primeira delas iremos demonstrar no Nível 2.1 e a segunda, no Nível 2.2.

Nível 2.1. Que a<b é imediato da denição de menor dada no texto:

s(a) = b ⇒ a + 1 = b ⇒ a < b.

Nível 2.2. Vamos mostrar que se a < c, então s(a) 6 c. Vamos fazer isto no Nível 3.
Assim, tendo também c < b, seguirá: s(a) 6 c < b e, portanto, pela transitividade, s(a) < b.
Logo, pela contrapositiva, teremos:

s(a) = b ⇒ s(a) > b ⇒ c 6 a ou b 6 c ∀c ∈ N .

Em outras palavras, se b é o sucessor de a, então não existe um c tal que a < c < b.

Nível 3: Lema. a < c ⇒ s(a) 6 c.


Solução. Se a < c, então, por denição, existe um p ∈ N tal que c = a + p. Se p = 1, então
c = s(a). Se p > 1, então existe um q ∈ N tal que s(q) = p. Para este q , temos:

c = a + p = a + s(q) = s(a + q) > s(a),

por causa da monotonicidade da adição: a < a + q ⇒ s(a) < s(a + q). Logo s(a) 6 c de
qualquer forma, como queríamos. 

2.6 Exercício 6
Use indução para demonstrar os seguintes fatos:

a) 2(1 + 2 + · · · + n) = n(n + 1);


b) 1 + 3 + 5 + · · · + (2n + 1) = (n + 1)2
c) (a − 1)(1 + a + · · · + an ) = an+1 − 1, seja quais forem a, n ∈ N;
d) n > 4 ⇒ n! > 2n .
2.7. EXERCÍCIO 7 33

Solução. Sejam
X1 = {n ∈ N; 2(1 + 2 + · · · + n) = n(n + 1)},
X2 = {n ∈ N; 1 + 3 + 5 + · · · + (2n + 1) = (n + 1)2 },
X3 = {n ∈ N; (a − 1)(1 + a + · · · + an ) = an+1 − 1 ∀a ∈ N}
e
X4 = {n ∈ N; n! > 2n }.
Vamos mostrar que Xi = N para i ∈ {1, 2, 3} e que X4 possui todos os números naturais maiores
do que ou iguais a 4. Temos:

2 · 1 = 1 · (1 + 1) ⇒ 1 ∈ X1 ,
1 + (2 · 1 + 1) = 1 + 3 = (1 + 1)2 ⇒ 1 ∈ X2 ,
(a − 1)(1 + a1 ) = a2 − 1 = a1+1 − 1 ∀a ∈ N ⇒ 1 ∈ X3
e
4! = 24 > 16 = 24 ⇒ 4 ∈ X4 .
Além disto:

n ∈ X1 ⇒ 2(1 + · · · + n + (n + 1)) = 2(1 + · · · + n) + 2(n + 1)


= n(n + 1) + 2(n + 1)
= (n + 1)(n + 2)
= (n + 1)[(n + 1) + 1] ⇒ n + 1 ∈ X1 ;

n ∈ X2 ⇒ 1 + · · · + (2n + 1) + (2(n + 1) + 1) = (n + 1)2 + (2n + 3)


= n2 + 2n + 1 + 2n + 3
= n2 + 4n + 4
= (n + 2)2
= [(n + 1) + 1]2 ⇒ n + 1 ∈ X2 ;

n ∈ X3 ⇒ (a − 1)(1 + · · · + an + an+1 ) = (a − 1)(1 + · · · + an ) + (a − 1)an+1


= an+1 − 1 + an+2 − an+1
= an+2 − 1
= a(n+1)+1 − 1 ⇒ n + 1 ∈ X3 ,
e;
n ∈ X4 ⇒ (n + 1)! = (n + 1) · n! > (n + 1) · 2n ,
onde (n + 1) · 2n > 2n+1 ⇔ n + 1 > 2, pela lei do corte e pela monotonicidade, ambas da
multiplicação. Portanto n + 1 ∈ X4 se n > 4 e se n ∈ X4 , pois n > 4 ⇒ n + 1 > 2.
Logo, Xi = N para i ∈ {1, 2, 3} pelo Princípio da Indução, e X4 possui todos os naturais
> 4 pelo Exercício 3 (página 24). 

2.7 Exercício 7
Use o Segundo Princípio da Indução para demonstrar a unicidade da decomposição de um
número natural em fatores primos.
34 CAPÍTULO 2. CONJUNTOS FINITOS, ENUMERÁVEIS E NÃO-ENUMERÁVEIS

Solução.

Nível 1. Seja n um número natural (maior do que 1) e vamos supor que todo número
natural menor do que n e diferente de 1 possa ser decomposto de modo único como um produto
de fatores primos. Se n é primo, então, por denição, n = p1 · · · pk ⇒ k = 1 e p1 = n,
se p1 , . . . , pk são primos (pois ab > a se a, b ∈ N e b > 1, o que pode ser demonstrado por
indução), ou seja, o modo de se decompor um número primo como um produto de fatores
primos é único. Se n não é primo, sejam a e b números naturais menores do que n tais que
n = ab. Vamos supor que existam números naturais k e l e números primos p1 , . . . , pk , q1 , . . . , ql
tais que
ab = p1 · · · pk = q1 · · · ql .
Para concluirmos que a decomposição de n em fatores primos é única, vamos mostrar que

{p1 , . . . , pk } = {q1 , . . . , ql }

e que
n({i ∈ Ik ; pi = x}) = n({i ∈ Ij ; qj = x}) ∀x ∈ {p1 , . . . , pk },
lembrando da notação Im = {1, . . . , m}, para m ∈ N, e denotando por n(X) o número de
elementos de um dado conjunto nito X (ver as observações feitas no começo da Solução do
Exercício 9: página 39). As duas últimas igualdades centralizadas acima signicam que os
fatores que aparecem nessa decomposição são sempre os mesmos e que o número de vezes que
cada um deles aparece também é sempre o mesmo.
Em particular, de n = ab, com a, b < n, segue que ambos a e b são distintos de 1 (pois, se
um deles fosse igual a 1, então o outro seria igual a n). Armamos que existem conjuntos não
vazios I e J, I contido propriamente em Ik e J contido propriamente em Il , tais que
Y Y
a= pi = qj
i∈I j∈J
Y
(Nível 2), onde o símbolo pi denota o produto dos pi 's conforme i varia em I, por exemplo,

Y i∈I
pi = pi 1 · pi2 · · · · · pim se I = {i1 , i2 , . . . , im }. Portanto, pela hipótese de indução, como a<n
i∈I
e a 6= 1,
{pi ; i ∈ I} = {qj ; j ∈ J}
e
n({i ∈ I; pi = x}) = n({j ∈ J; qj = x}) ∀x ∈ {pi ; i ∈ I}.
Pela lei do corte, denindo I 0 = Ik − I e J 0 = Il − J (I 0 e J0 também não vazios e contidos
propriamente em Ik e Il , respectivamente), temos

Y Y
b= pi = qj
i∈I 0 j∈J 0

e, do mesmo modo como acontecia com a, pela hipótese de indução,

{pi ; i ∈ I 0 } = {qj ; j ∈ J 0 }

e
n({i ∈ I 0 ; pi = x}) = n({j ∈ J 0 ; qj = x}) ∀x ∈ {pi ; i ∈ I 0 }.
2.7. EXERCÍCIO 7 35

Portanto,

{p1 , . . . , pk } = {pi ; i ∈ I} ∪ {pi ; i ∈ I 0 } = {qj ; j ∈ J} ∪ {qj ; j ∈ J 0 } = {q1 , . . . , ql },

o que nos dá uma parte do que queremos. Com relação à outra parte, para cairmos nos casos
anteriores, notemos o seguinte. Fixado x ∈ {pi ; i ∈ Ik }, o número de vezes que ele aparece
Y
no produto p1 · · · pk é igual ao número de vezes que ele aparece no produto pi somado com

Y i∈I
o número de vezes que ele aparece no produto pi (um destes dois números, e apenas um,
i∈I 0
podendo ser zero); mais precisamente,

n({i ∈ Ik ; pi = x}) = n({i ∈ I; pi = x}) + n({i ∈ I 0 ; pi = x})

e, analogamente,

n({j ∈ Il ; qj = x}) = n({j ∈ J; qj = x}) + n({j ∈ J 0 ; qj = x})

(Teorema 6). Assim: se x ∈ {pi ; i ∈ I} e se x ∈ {pi ; i ∈ I 0 }, então

n({i ∈ Ik ; pi = x}) = n({i ∈ I; pi = x}) + n({i ∈ I 0 ; pi = x})


0
= n({j ∈ J; qj = x}) + n({j ∈ J ; qj = x})

= n({j ∈ Il ; qi = x});

se x ∈ {pi ; i ∈ I} e se / {pi ; i ∈ I 0 },
x∈ então

n({i ∈ Ik ; pi = x}) = n({i ∈ I; pi = x}) + n({i ∈ I 0 ; pi = x})


= n({i ∈ I; pi = x}) + 0
0
= n({j ∈ J; qj = x}) + n({j ∈ J ; qj = x})

= n({j ∈ Il ; qi = x}),

pois
x ∈ {pi ; i ∈ I} ⇒ n({i ∈ I; pi = x}) = n({j ∈ J; qj = x}),
/ {pi ; i ∈ I 0 } ⇒ n({i ∈ I 0 ; pi = x}) = 0
x∈
e, como {pi ; i ∈ I 0 } = {qj ; j ∈ J 0 }, x ∈
/ {pi ; i ∈ I 0 } também implica x ∈ / {qj ; j ∈ J 0 }, o que,
0
por sua vez, implica n({j ∈ J ; qj = x}) = 0. O caso em que x ∈ / {pi ; i ∈ I} e x ∈ {pi ; i ∈ I 0 }
é análogo, neste caso:

n({i ∈ Ik ; pi = x}) = n({i ∈ I; pi = x}) + n({i ∈ I 0 ; pi = x})


= 0 + n({i ∈ I 0 ; pi = x})
= n({j ∈ J; qj = x}) + n({j ∈ J 0 ; qj = x})
= n({j ∈ Il ; qi = x}).

Logo, de qualquer forma,

n({i ∈ I; pi = x}) = n({j ∈ J; qj = x}) ∀x ∈ {pi ; i ∈ I},

o que nos dá a parte que faltava para concluirmos o resultado desejado.


36 CAPÍTULO 2. CONJUNTOS FINITOS, ENUMERÁVEIS E NÃO-ENUMERÁVEIS

Nível 2: Lema. Dados os números naturais


Y a e b distintos de 1, se existem m ∈ N
e números primos r1 , . . . , rm tais que ab = ri , então existe um conjunto K não vazio e
i∈Im
Y
contido propriamente em Im tal que a= ri .
i∈K
Solução. Vamos aplicar o Segundo Princípio da Indução em a. Se a é primo, armamos
que existe i ∈ {1, . . . , m} tal que a = ri (Nível 3) e, portanto, para um tal i, o K procurado
é K = {i}. Ele é não vazio e está contido propriamente em Im , porque m > 2 (pois, se fosse
m = 1, então teríamos p1 = ab, onde p1 é primo e ambos a e b são distintos de 1, o que é
absurdo).
Se a não é primo, sejam x e y números naturais menores do queY
a tais que a = xy . Isto
também implica x 6= 1 e y 6= 1. Assim temos ab = (xy)b = x(yb) = 6 1, x < a
ri , com x =
i∈Im
e yb 6= 1. Portanto, pela hipótese de indução, seja
Y K0 não vazio e contido propriamente em
Y
Im tal que x = ri , donde, pela lei do corte, yb = ri e, novamente, pela hipótese
i∈K 0 i∈Im −K 0
Y
00
de indução, seja K não vazio e contido propriamente em Im − K 0 tal que y= ri . Agora
i∈K 00
K = K 0 ∪ K 00 é não vazio, está contido propriamente em Im e é tal que

Y
a = xy = ri ,
i∈K

como queríamos.

Nível 3. Aqui vamos lançar mão do seguinte resultado de Álgebra:

se p é primo e se p divide x · y, sendo x e y números naturais, então p divide x ou


p divide y,
a expressão  α divide γ , com α, γ ∈ N, γ = α · β.
signicando que existe um β ∈N tal que
Por indução (no número de fatores), deste resultado podemos inferir que se p p é primo e se
divide um produto x1 · x2 · · · · · xn , então p divide xi para algum i ∈ In . Assim, se a é primo
Y Y
e se ab = ri , sendo cada ri primo, então a divide o produto ri e, consequentemente, a
i∈Im i∈Im
divide ri i ∈ Im , ou seja, existe x ∈ N tal que ri = a · x. Daí, por um lado, como
para algum
ri é primo, então a > ri ou x > ri . Por outro lado, como a · x > a e a · x > x (o que pode
ser demonstrado por indução), então, na verdade, temos: ri = a · x ⇒ ri = a ou ri = x, onde
ri = a ⇒ x = 1, e ri = x ⇒ a = 1 (lei do corte). Como a 6= 1, segue a = ri , como queríamos.
(Voltar para a Solução do Exercício 4: página 30.)

(Voltar para a Solução do Exercício 42 do Capítulo III, item i: página 91.)

2.8 Exercício 8
Seja X um conjunto com n elementos. Use indução para provar que o conjunto das bijeções
(ou permutações) f : X → X tem n! elementos.
(Voltar para a Solução do Exercício 12: página 43.)

(Voltar para a Solução do Exercício 13, para sua Armação: página 46.)
2.8. EXERCÍCIO 8 37

Solução.

Nível 1. Para n = 1, a armação é prontamente verdadeira: se X = {x1 } é um conjunto


com apenas um elemento, então x1 7→ x1 é a única bijeção de X X , onde 1 = 1!.
em
Vamos supor que a armação seja verdadeira para n  a armação se Y é um
conjunto
com n elementos, então o número de bijeções de Y
Y é n! . Devemos mostrar que ela é
em
verdadeira para n + 1. Para tanto xemos o conjunto X = {x1 , · · · , xn+1 } = {xi ; i ∈ In+1 } com
n + 1 elementos e, portanto, xi 6= xj , se i e j são elementos distintos de In+1 . Como sempre, e
até sendo prolixo, vamos manipular o caso n + 1 para cair no caso n. O que vamos fazer nesta
solução é construir uma bijeção entre o conjunto {f ∈ F(X; X); f é bijeção} e algum conjunto
que sabemos ter (n + 1)! elementos. Uma tal bijeção irá concluir nossa demonstração.

No elevador. X um elemento xi , obtendo o conjunto


Uma primeira tentativa é retirar de
Xi = X −{xi }, e denir uma bijeção g : X → X por g(xi ) = xi e g(x) = f (x) ∀x ∈ Xi , onde f é
alguma bijeção de Xi em Xi . A partir disso de fato obteríamos (deveríamos dizer: no máximo)
(n + 1) · n! funções  para cada i ∈ In+1 , as bijeções de Xi em Xi determinam n! bijeções de X
em X , e existem n + 1 conjuntos da forma Xi . Porém isso nos daria apenas as bijeções que
possuem um ponto xo e sabemos que nem todas as bijeções são assim, por exemplo, a bijeção
denida por x1 7→ x2 e x2 7→ x1 no conjunto {x1 , x2 }.
Um jeito de corrigir isso é não considerar as bijeções de Xi em Xi , mas as bijeções de
Xi em Xj e passando agora a denir, a partir das bijeções de Xi em Xj , uma nova bijeção
colocando g(xi ) = xj . Isso nos leva a armar que o conjunto das bijeções de X em Y tem a
mesma cardinalidade do conjunto das bijeções de X em X caso card(X) = card(Y ). Isso de
fato é verdadeiro e será útil mais adiante. O problema dessa tentativa é que agora obtemos
(n + 1) · (n + 1) · n! bijeções porque, nessa construção, cada par ordenado (Xi , Xj ) e cada bijeção
de Xi em Xj determina uma nova bijeção. Existem (n + 1) · (n + 1) pares ordenados daquela
forma. Então certamente existem funções que foram contadas mais de uma vez nessa contagem,
e encontrá-las pode constituir um problema.
Nesta última tentativa obtemos um indesejado fator n + 1. Ele sumiria se xássemos a
primeira coordenada dos pares ordenados (Xi , Xj ), e é isto que iremos fazer abaixo.

Nível 2. Para cada i ∈ In+1 consideremos o conjunto de todas as bijeções de X1 :=


X − {x1 } emXi := X − {xi }. Armamos que este conjunto tem n! elementos (Nível 3.1, junto
com a hipótese de indução). Sejam m = n! e

Ai = {f ∈ F(X1 ; Xi ); f é bijeção}

n+1
[
o conjunto dessas bijeções para cada i ∈ In+1 . Para cada f∈ Ai vamos denir uma função
i=1
n+1
[
Φ(f ) : X → X por Φ(f )(x) = f (x) ∀x ∈ X1 e Φ(f )(x1 ) = xi , se f ∈ Ai . A reunião Ai tem
i=1
(n + 1) · m elementos (Nível 3.2). Além disso, Φ é uma bijeção entre essa reunião e o conjunto
das bijeções de X em X (Nível 3.3). Logo, este conjunto, o das bijeções de X em X, possui
(n + 1) · m = (n + 1) · n! = (n + 1)! elementos, como desejamos.

Nível 3.1: Lema. Se card(A) = card(B), então os conjuntos {f ∈ F(A; A); f é bijeção}
e {f ∈ F(A; B); f é bijeção} têm a mesma cardinalidade.
38 CAPÍTULO 2. CONJUNTOS FINITOS, ENUMERÁVEIS E NÃO-ENUMERÁVEIS

Solução. Como card(A) = card(B), existe, por denição, uma bijeção ψ : A → B . É essa
bijeção quem vai nos auxiliar a construir a bijeção desejada, basta associar f 7→ ψ ◦ f , para
toda bijeção f ∈ F(A; A) (veja o diagrama abaixo, à esquerda). A inversa dessa associação é
g 7→ ψ −1 ◦ g . Logo, a associação f 7→ ψ ◦ f é uma bijeção e segue o resultado.
g f
A B A B
f ψ ψ φ
A C g D

Observação. Este lema pode ser facilmente generalizado para o caso em que card(A) =
card(C) e card(B) = card(D), conforme sugere o diagrama acima e à direita.

n+1
[
Nível 3.2: Armação. A reunião Ai possui (n + 1) · m elementos, onde cada Ai é o
i=1
conjunto de todas as bijeções de X1 = X − {x1 } em Xi = X − {xi } e card(Ai ) =m (este m é
uma constante conforme já vimos no Nível 3.1).

Solução. Basta mostrar que não há elementos repetidos nessa reunião. De modo mais
preciso, dados f ∈ Ai e g ∈ Aj , com i 6= j , vamos mostrar que f 6= g . A ideia central desta
solução é que xi ∈ Xj mas xi ∈ / Xi e, portanto, f nunca poderá alcançá-lo, enquanto g pode.
Seja a ∈ X1 tal que g(a) = xi . Como xi ∈ / Xi , temos f (x) 6= xi ∀x ∈ X1 , em particular
f (a) 6= xi . Logo f 6= g , como queríamos.

Nível 3.3: Armação. A função

n+1
[
Φ: {f ∈ F(X1 ; Xi ); f é bijeção} −→ {f ∈ F(X; X); f é bijeção}
i=1

denida por:
Φ(f )(x) = f (x) ∀x ∈ X1 e Φ(f )(x1 ) = xi ,
para toda bijeção f : X1 → Xi para todo i ∈ In+1 , é uma bijeção (lembrando que Xi = X −{xi }
∀i ∈ In+1 ).
Solução.

É sobrejetiva. Dada uma bijeção g : X → X , seja g(x1 ) = xi . Para este i, temos que a
função f : X1 → Xi , denida por f = g|X1 , pertence a Ai . É imediato que Φ(f ) = g , como
queríamos.

n+1
[
É injetiva. Sejam f, g ∈ Ai funções distintas. Queremos encontrar um x∈X tal que
i=1
Φ(f )(x) 6= Φ(g)(x).
Sejam Xi e Xj os respectivos contradomínios de f e de g . Se i 6= j , então Φ(f )(x1 ) =
xi 6= xj = Φ(g)(xj ) e teremos concluído. Porém, se i = j , então o x1 não serve para os nossos
própositos, pois, neste caso, Φ(f )(x1 ) = xi = xj = Φ(g)(x). Como f e g são distintas, existe
um x ∈ X1 tal que f (x) 6= g(x). Para este x  que é diferente de x1 , uma vez que x1 ∈
/ X1 , por
denição  temos Φ(f )(x) = f (x) 6= g(x) = Φ(g)(x). Logo Φ é de fato uma bijeção. 
2.9. EXERCÍCIO 9 39

Nos bastidores. Num primeiro momento, para mostrar a injetividade de Φ, eu havia me


esquecido da denição da X1 e comecei a me complicar com a questão do porquê o x ∈ X1
que torna f e g diferentes poder ser tomado distinto de x1 , achando até mesmo que toda a
solução acima poderia vir por água abaixo. Só depois percebi que x 6= x1 simplesmente porque
X1 := X − {x1 }. De qualquer forma, neste pequeno desvio de percurso, surgiu o lema abaixo,
o qual não deixa de ser interessante e também poderia ter sido usado na demonstração acima.

Lema. Seja A um conjunto com pelo menos dois elementos. Se g:A→B f :A→B e
são bijeções distintas, então elas diferem em pelo menos dois pontos, ou seja, existem a1 , a2 ∈ X
distintos tais que f (a1 ) 6= g(a1 ) e f (a2 ) 6= g(a2 ).

Solução. A tabela dos valores assumidos por f e por g, na qual cada um dos elementos
de B aparece uma e uma única vez em cada linha, irá nos auxiliar em nossa demonstração.
Abaixo destacamos só a parte dela que nos interessa

a1 a2
f b1 c 1
g c1 d1

Comof 6= g , f (a1 ) 6= g(a1 ). Sejam f (a1 ) = b1 e g(a1 ) = c1 . Como b1 6= c1 ,


existe a1 tal que
certamente o c1 vai aparecer na linha do f em alguma coluna diferente da primeira. Nesta
coluna, ele não vai aparecer na linha do g .
De um modo mais preciso. Como f é sobrejetiva, existe a2 ∈ X tal que f (a2 ) = c1 . Este
a2 6= a1 simplesmente porque f é uma função; ela não pode assumir dois valores distintos num
mesmo ponto. Para este a2 temos f (a2 ) 6= g(a2 ). Pois:

c1 = f (a2 ) = g(a2 ) ⇒ g(a1 ) = g(a2 ) ⇒ a1 = a2 .

Segue a tese. 

2.9 Exercício 9
Sejam X e Y conjuntos nitos.

a) Prove que card(X ∪ Y ) + card(X ∩ Y ) = card(X) + card(Y ).

b) Qual seria a fórmula correspondente para três conjuntos?

c) Generalize.

Solução. A rigor, ainda não temos um signicado para a expressão card(X) + card(Y ). Não
há nada no livro dizendo que card(X) é um número natural caso X seja nito. O mais próximo
que ele chega disso é dizer: dois conjuntos nitos têm o mesmo número cardinal se, e somente
se, possuem o mesmo número de elementos na página 52, mas não é armado que este número
cardinal é este número de elementos. Para conseguirmos resolver este exercício, vamos assumir
então a

Denição. Dado um conjunto nito X, card(X) é o seu número de elementos.


40 CAPÍTULO 2. CONJUNTOS FINITOS, ENUMERÁVEIS E NÃO-ENUMERÁVEIS

Observação. Com esta denição, dado o conjunto nito X, o n(X) da Solução do Exer-
cício 7 (página 34) torna-se n(X) = card(X).

item a) Pelo Teorema 6 sabemos contar elementos de uma reunião caso os conjuntos dessa
reunião sejam disjuntos. Um modo de tornar os conjuntos X e Y disjuntos é considerar os
conjuntos X e Y − (X ∩ Y ). Neste processo a reunião não muda, quer dizer, ainda temos
X ∪ Y = X ∪ (Y − (X ∩ Y )), além de X ∩ (Y − (X ∩ Y )) = ∅. Portanto, pelo referido teorema,
a reunião X ∪ Y tem card(X) + card(Y − (X ∩ Y )) elementos. Agora só resta mostrar que
card(Y − (X ∩ Y )) = card(Y ) − card(X ∩ Y ). O cuidado que precisamos tomar aqui é que
existe uma denição para o conceito de número de elementos de um conjunto nito. E é esta
denição que precisamos usar. Um outro cuidado é que ainda não temos um signicado para
a subtração de números naturais que acabou de aparecer. Reescrever o que queremos em
termos da soma irá, inclusive, facilitar nosso trabalho. O lema abaixo colocará m na solução
do item a), ele também pode ser considerado mais um dos corolários do Teorema 6.

Lema. Sejam A e B conjuntos nitos. Se B ⊂ A, então card(A) = card(A−B)+ card(B).


Solução. Como A = B ∪ (A − B) e B ∩ (A − B) = ∅, pelo Teorema 6 temos
card(A) = card(B) + card(A − B),
como queríamos.
Assim, somando card(X ∩Y) a ambos os membros da equação

card(X ∪ Y ) = card(X) + card(Y − (X ∩ Y ))


já encontrada acima, obtemos

card(X ∪ Y ) + card(X ∩ Y ) = card(X) + (card(Y − (X ∩ Y )) + card(X ∩ Y ))


= card(X) + card(Y ),

como queríamos! 

item b) A equação do item anterior envolve uma reunião de dois conjuntos. Uma reunião
X ∪ Y ∪ Z de três conjuntos também é uma reunião de dois conjuntos, a saber, dos conjuntos
X e Y ∪ Z . Esta observação nos permite aplicar o item a) sucessivamente. Assim, temos

card(X ∪ (Y ∪ Z)) + card(X ∩ (Y ∪ Z)) = card(X) + card(Y ∪ Z).


Somando card(Y ∩ Z) a ambos os membros, como card(Y ∪ Z) + card(Y ∩ Z) = card(Y ) +
card(Z):

card(X ∪ Y ∪ Z) + card(X ∩ (Y ∪ Z)) + card(Y ∩ Z) = card(X) + card(Y ) + card(Z),


ou seja,

card(X ∪ Y ∪ Z) + card((X ∩ Y ) ∪ (X ∩ Z)) + card(Y ∩ Z) =


= card(X) + card(Y ) + card(Z).
Agora somamos card((X ∩ Y ) ∩ (X ∩ Z)) = card(X ∩ Y ∩ Z) a ambos os membros. E,
substituindo card((X ∩ Y ) ∪ (X ∩ Z)) + card((X ∩ Y ) ∩ (X ∩ Z)) por card(X ∩ Y ) + card(X ∩ Z),
obtemos:

card(X ∪ Y ∪ Z) + card(X ∩ Y ) + card(X ∩ Z) + card(Y ∩ Z) =


= card(X) + card(Y ) + card(Z) + card(X ∩ Y ∩ Z).
Esta é uma fórmula correspondente para três conjuntos. 
2.9. EXERCÍCIO 9 41

item c) O que as equações dos itens a) e b) têm em comum? Do lado esquerdo nós temos
a reunião de todos os conjuntos considerados. Nelas há também todas as interseções tomadas
i a i. Essas interseções caram dispostas do seguinte modo: do lado esquerdo se i é par e do
lado direito se i é ímpar (inclusive o caso em que os conjuntos aparecem sozinhos, caso em que
i = 1). Com essas observações podemos formar a seguinte

Conjectura. n, seja (Ai )i∈In uma familía de conjuntos


Fixado um número natural nitos.
Sejam I = {X ∈ P(In ); card(X) é par} e J = {X ∈ P(In ); card(X) é ímpar}. Então:
! ! !
[ X \ X \
card Ai + card Ai = card Ai .
i∈In X∈I i∈X X∈J i∈X

Uma bonita equação. Ela é verdadeira? Eis a questão.

Solução.
Para n = 2 já vimos que ela é verdadeira. Vamos ver agora se ela é verdadeira
para n + 1 supondo-a verdadeira para n. A experiência obtida na obtenção da fórmula para
n = 3 vai ser útil aqui. Sejam I e J exatamente como no enunciado da conjectura. Aplicando
[n
o item a) aos conjuntos An+1 e Ai , temos:
i=1
  ! !
[ [ [
card  Ai  + card An+1 ∩ Ai = card(An+1 ) + card Ai ,
i∈In+1 i∈In i∈In

ou seja, reescrevendo a segunda parcela do lado esquerdo da igualdade:


  ! !
[ [ [
card  Ai  + card (An+1 ∩ Ai ) = card(An+1 ) + card Ai .
i∈In+1 i∈In i∈In
!
X \
Somando card Ai a ambos os membros dessa igualdade e usando a hipótese de
X∈I i∈X
indução:

  ! !
[ [ X \
card  Ai  + card (An+1 ∩ Ai ) + card Ai =
i∈In+1 i∈In X∈I i∈X
!
X \
= card(An+1 ) + card Ai .
X∈J i∈X
!
X \
Agora somamos card (An+1 ∩ Ai ) a ambos os membros e usamos novamente a
X∈I i∈X
hipótese de indução:

  ! !
[ X \ X \
card  Ai  + card (An+1 ∩ Ai ) + card Ai =
i∈In+1 X∈J i∈X X∈I i∈X
! !
X \ X \
= card(An+1 ) + card Ai + card (An+1 ∩ Ai ) .
X∈J i∈X X∈I i∈X
42 CAPÍTULO 2. CONJUNTOS FINITOS, ENUMERÁVEIS E NÃO-ENUMERÁVEIS

Observemos que já chegamos onde queríamos. Vejamos. Se X é um subconjunto de In+1 de


cardinalidade par, então ou X ⊂ In ou X é da forma X = Y ∪{n+1}, onde Y é um subconjunto
de In de cardinalidade ímpar. Ora, todos os subconjuntos dessa forma estão aparecendo ali
do lado esquerdo da igualdade acima. Algo análogo acontece com subconjuntos de In+1 de
cardinalidade ímpar. Tais subconjuntos estão aparecendo ali do lado direito da equação acima.
Logo a conjectura feita é verdadeira para todo n∈N (!). 

2.10 Exercício 10
Dado um conjunto nito X, prove que uma função f :X →X é injetiva se, e somente se, é
sobrejetiva (e portanto uma bijeção).

(Voltar para a Solução do Exercício 12: página 43.)

(Voltar para a Solução do Exercício 8 do Capítulo III: página 71.)

Solução. Suponha f :X →X f [X] ⊂ X . Em segundo lugar,


injetiva. Em primeiro lugar,
f [X] e X possuem o mesmo número de elementos, pois a função g : X → f [X], denida por
g(x) = f (x) para todo x ∈ X , é uma bijeção, donde card(X) = card(f [X]) (lembrando que
dois conjuntos nitos  f [X] é nito porque é subconjunto de um conjunto nito, Teorema 4 
possuem o mesmo número cardinal se, e somente se, possuem o mesmo número de elementos,
conforme já foi observado na página 52 do livro). Logo, pelo Teorema 4, f [X] = X , o que
signica que f é sobrejetiva. [
Suponha f : X → X sobrejetiva. De X = f −1 [{y}], onde f −1 [{y}] ∩ f −1 [{z}] = ∅
y∈X
−1
se y 6= z , e, dos fatos de X e f [{y}] serem nitos, para cada y ∈ X (Teorema 4, pois
f −1 [{y}] ⊂ X ∀y ∈ X ), denotanto por n(S) o número de elementos de um conjunto nito S,
segue, pelo Corolário 1 do Teorema 6,

X
−1
n(X) = n(f [{y}]).
y∈X

−1
Como f é sobrejetiva, temos n(f [{y}]) > 1 ∀y ∈ X . Assim, se existisse um y ∈ X tal
−1
que n(f [{y}]) > 1, então o somatório acima seria estritamente maior do que n(X)  há n(X)
−1
parcelas nele. Logo, só pode ser n(f [{y}]) = 1 ∀y ∈ X , o que mostra que f é injetiva. 
(Voltar para a Solução do Exercício 15: página 49.)

Curiosidade. É curioso notar que as transformações lineares em espaços vetoriais de di-


mensão nita apresentam esse mesmo comportamento. De um modo preciso, para um espaço
vetorial V de dimensão nita, uma transformação linear T : V → V é injetiva se, e somente
se, é sobrejetiva. Um modo rápido de vericar isso é lembrar que T é injetiva se, e somente se,
{x ∈ V ; T (x) = 0} = {0} e aplicar o teorema do núcleo e da imagem.

2.11 Exercício 11
Formule matematicamente e demonstre o seguinte fato (conhecido como o princípio das gave-
tas). Se m < n, então, de qualquer modo como se guardem n objetos em m gavetas, haverá
sempre uma gaveta, pelo menos, que conterá mais de um objeto.
2.12. EXERCÍCIO 12 43

Solução. Seja f : In → Im uma função. Cada x ∈ In é um objeto. Cada y ∈ Im é uma


gaveta, e a função f está nos dizendo em quê gaveta y cada objeto x será guardado. Assim,
podemos traduzir o princípio das gavetas com a seguinte armação:

se m < n, então nenhuma função f : In → Im pode ser injetiva.

De fato, dados os números naturais m e n, se uma função f : In → Im é injetiva, então, pelo


Corolário 1 do Teorema 4, n > m. O resultado desejado agora segue pela contrapositiva. 

2.12 Exercício 12
Seja X um conjunto com n elementos. Determine o número de funções injetivas f : Ip → X .

Solução. O exercício anterior nos dá a resposta para o caso em que n < p: se n < p,
então o número de funções injetivas de Ip emX é zero, pois, se existisse uma função injetiva
f : Ip → X , então, considerando uma bijeção ϕ : X → In , a função ϕ ◦ f : Ip → In seria
injetiva, mas acabamos de ver que não existem funções injetivas de Ip em In se n < p. Vamos
supor então, daqui em diante, n > p.
Um outro caso para o qual também já temos uma resposta apenas combinando exercícios
anteriores é o caso n = p. Para este caso, pelo Exercício 10 (página 42), temos que f : Ip → X
é injetiva se, e somente se, f : Ip → X é bijetiva. Na verdade, naquele exercício o domínio
e o contradomínio das funções estudadas eram iguais. No entanto, revisitando sua solução,
podemos ver que o resultado é válido mesmo quando o domínio e o contradomínio são distintos,
apenas supondo que suas cardinalidades são iguais. Pelo Exercício 8 (página 36) sabemos contar
essas bijeções. Logo, o número de funções injetivas de Ip em X é n!, se n = p.
Fixemos agora n > p. Podemos aplicar o princípio multiplicativo da análise combinatória
vendo de quantos modos podemos escolher a imagem de cada elemento de Ip : podemos escolher
uma imagem para o 1 de n modos; para o 2, de n−1 = (n−2)+1 modos; para o 3, de (n−3)+1
modos, e assim por diante até chegar ao p; há n − p + 1 modos de escolhermos uma imagem
para ele. Claro que o caso n = p também pode ser abordado desta forma. Daí vem a

Armação. Sen > p, então o número de funções injetivas de Ip em X é n · (n − 1) · (n −


n!
2) · · · · · (n − p + 1) = .
(n − p)!
Solução 1. Vamos usar o Exercício 3 (página 24). O caso n = p já foi vericado mais
acima. Dado agora n > p, vamos supor que o número de funções injetivas de Ip em Y é
n!
qualquer que seja o conjunto Y com n elementos (hipótese de indução). Fixado um
(n − p)!
conjunto X = {x1 , . . . , xn+1 }, queremos mostrar que o número de funções injetivas de Ip em X
(n + 1)!
é .
(n + 1 − p)!
n!
Seja m = . Pela hipótese de indução, o número de funções injetivas de Ip em
(n − p)!
X − {xi } é m para todo i ∈ In+1 . Dada f : Ip → X − {xi }, podemos denir g : Ip → X
simplesmente fazendo g(x) = f (x) para todo x ∈ Ip . Injetiva esta g é. Quem não é injetiva
é esta maneira de associar f a g ; dada uma função injetiva g : Ip → X , para cada i ∈ In+1
tal que xi ∈/ g[Ip ], existe uma, e apenas uma, função injetiva f : Ip → X − {xi } associada a
ela pela regra que acabamos de enunciar, e, se xi ∈ g[Ip ], então não existe uma função injetiva
44 CAPÍTULO 2. CONJUNTOS FINITOS, ENUMERÁVEIS E NÃO-ENUMERÁVEIS

f : Ip → X − {xi } tal que f (x) = g(x) para todo x ∈ Ip (sequer existe uma função assim).
Como g[Ip ] tem p elementos, o conjunto X − g[Ip ] possui n + 1 − p elementos, portanto, existem
exatamente n + 1 − p funções associadas a g do modo descrito no início deste parágrafo; assim
(n + 1) · m, o número de elementos do conjunto das funções injetivas de Ip em X − {xi } para
algum i ∈ In+1 , é igual ao número de funções injetivas de Ip em X dividido por (n + 1 − p).
É desta equação que surge o número a ser alcançado. Vamos formalizar um pouco mais esta
ideia.
Para cada i ∈ In+1 , seja Ai = {f ∈ F(Ip ; X − {xi }); f é injetiva}, seja B = {g ∈
F(Ip ; X); g é injetiva} e consideremos a função

n+1
[
Φ: Ai → B
i=1
f 7→ Φ(f ) : Ip → X
x 7→ Φ(f )(x) = f (x);
n+1
[
de fato Φ(f ) ∈ B para todo f∈ Ai . Agora, de
i=1

n+1
[ [
Ai = Φ−1 [{g}]
i=1 g∈B

(para ser consistente com as convenções feitas no livro, é como se estivéssemos olhando para
B como um conjunto de índices e escrevendo, por exemplo, Φ−1 [{g}] = Xg para todo g ∈ B),
n+1
[
do fato de Ai ter (n + 1) · m elementos (hipótese de indução + Corolário 1 do Teorema
i=1
6, lembrando ainda da observação feita na página 14 do livro, segundo a qual uma condição
necessária para que duas funções sejam iguais é que elas tenham o mesmo contra-domínio) e,
−1
do fato de Φ [{g}] ter n − p + 1 (um número diferente de zero) elementos, qualquer que seja
g ∈B  fato este já observado no parágrafo anterior , segue que B é um conjunto nito e,
sendo α o número de elementos de B,
(n + 1) · m = α · (n − p + 1),
novamente pelo Corolário 1 do Teorema 6, uma vez que Φ−1 [{g}] ∩ Φ−1 [{h}] = ∅ se g e h são
elementos distintos de B (essa disjunção acontece simplesmente porque Φ é uma função), logo,

1
α = (n + 1) · m ·
n+1−p
n! 1
= (n + 1) · ·
(n − p)! (n + 1 − p)
(n + 1)!
= ,
(n + 1 − p)!
como queríamos.
Procendo por indução em p, camos mais próximos do princípio multiplicativo, utilizado
para apoiar a armação feita.

Solução 2. Para p = 1, o número de funções injetivas de Ip = {1} em X = {x1 , . . . , xn }


n!
é n = ; elas são as denidas por fi (1) = xi para cada i ∈ In ; claramente, neste caso,
(n − p)!
n > p para todo n ∈ N.
2.13. EXERCÍCIO 13 45

Dado um natural p, vamos supor que o número de funções injetivas de Ip em qualquer


n!
conjunto com n elementos seja m = , qualquer que seja o natural n > p. Seja X =
(n − p)!
{x1 , . . . , xn } um conjunto com n > p + 1 elementos. Queremos mostrar que o conjunto A =
n!
{f ∈ F(Ip+1 ; X); f é injetiva} tem elementos.
(n − p − 1)!
Pela suposição feita, como n é também maior do que p, o conjunto B = {g ∈ F(Ip ; X); g é injetiva}
tem m elementos. Para cada f ∈ A, seja a função Ψ(f ) : Ip → X denida por Ψ(f )(x) = f (x)
para todo x ∈ Ip ; Ψ(f ) é injetiva e, assim, temos a função Ψ : A → B . Daí, de modo semelhante
ao que zemos na solução anterior, vem a igualdade

[
A= Ψ−1 [{g}],
g∈B

onde cada conjunto da forma Ψ−1 [{g}] n − p elementos: para cada g ∈ B , o conjunto
possui
X − g[Ip ] possui n − p elementos e a associação Ψ−1 [{g}] 3 f 7→ f (p + 1) ∈ X − g[Ip ] é bijetiva;
−1 −1
de fato f (p + 1) ∈ X − g[Ip ] se f ∈ Ψ [{g}], pois f ∈ Ψ [{g}] ⇒ Ψ(f ) = g ⇒ f (x) =
g(x) ∀x ∈ Ip ⇒ f [Ip ] = g[Ip ] ⇒ f (p + 1) ∈
/ g[Ip ], uma vez que f é injetiva. Portanto, como B
possui m ∈ N elementos, pelo Corolário 1 do Teorema 6, A possui

n! n!
m · (n − p) = · (n − p) = ,
(n − p)! (n − p − 1)!

como queríamos demonstrar.

Resposta. O número de funções injetivas de Ip em X é

0, se n < p, e;
n!
, se n > p. 
(n − p)!

Observação. Na Solução 1 acima, para justicar que Ai ∩ Aj = ∅, se i 6= j , usamos que duas


funções f : A → B e g : A → C são distintas se B 6= C , mesmo que se tenha f (x) = g(x) para
todo x ∈ A. Porém existem livros que dizem que duas funções f : A → B e g : C → D são
iguais se, e somente se, A = C e f (x) = g(x) para todo x ∈ A. Neste caso não é necessariamente
verdade que i 6= j implica Ai ∩ Aj = ∅ e poderíamos fazer uma pequena adaptação na referida
solução considerando Ai × {i} no lugar de Ai , quando conveniente. Vamos ver uma situação
semelhante a esta na Solução 2 da Armação feita na Solução abaixo.

2.13 Exercício 13
Quantos subconjuntos com p elementos possui um conjunto X, sabendo-se que X tem n ele-
mentos?

Solução. Claramente a resposta é zero caso p > n. Vamos supor então n > p. Acabamos
de estudar as funções injetivas. Será que há como aproveitar aqui o que acabamos de ver no
exercício anterior? Vejamos
Uma função injetiva f : Ip → X de fato determina um subconjunto de X com p elementos,
a saber, o conjunto f [Ip ]. Porém, duas funções injetivas distintas f e g de Ip em X podem
46 CAPÍTULO 2. CONJUNTOS FINITOS, ENUMERÁVEIS E NÃO-ENUMERÁVEIS

determinar um mesmo subconjunto f [Ip ] = g[Ip ]. Por outro lado, observemos que cada permu-
tação de um subconjunto de X com p elementos determina uma função injetiva distinta. Assim,
podemos concluir que o número de funções injetivas de Ip em X é p! vezes o de subconjuntos
de X com p.

Armação. Seja X um conjunto com n elementos. Se p6n (denindo 06n para todo
n!
n ∈ {0} ∪ N), o número de subconjuntos de X com p elementos é .
p!(n − p)!
Solução 1. Esta solução é uma formalização da ideia apresentada antes dessa armação.
Se p = 0 ou se n = 0, então X p elementos (existe um
admite um único subconjunto com
n! 0!
único conjunto sem elementos), onde 1 = = (admitindo também a implicação
0!(n − 0)! 0! · 0!
p 6 0 ⇒ p = 0, se p ∈ {0} ∪ N). Vamos supor então p, n ∈ N. Sejam
A = {f ∈ F(Ip ; X); f é injetiva},

B = {S ∈ P(X); o número de elementos de S é p}


e a função Φ:A→B denida por Φ(f ) = f [Ip ] para todo f ∈ A. Assim,
[
A= Φ−1 [{S}].
S∈B

n!
Pela solução do exercício anterior, sabemos que A elementos. Além disto,
possui
(n − p)!
−1
armamos que Φ [{S}] possui p! elementos, para todo S ∈ B . Para justicar esta armação,
seja S ∈ B e consideremos o conjunto CS = {g ∈ F(S; S); g é bijeção}, o qual já sabemos
−1
possuir p! elementos (Exercício 8, página 36). Dada f ∈ Φ [{S}], como f : Ip → X é uma
função injetiva tal que f [Ip ] = S , a função g : S → S , denida por g(x) = f (hS (x)), para todo
x ∈ S , onde hS : S → Ip é uma bijeção (uma tal hS existe porque S possui p elementos), é
−1
uma bijeção. Isto nos dá então uma função Ψ : Φ [{S}] → CS , denida por Ψ(f ) = g (f e g
como acima). Sua inversa é a função Ψ
−1
: CS → Ψ−1 [|S}] denida por Ψ−1 (g)(x) = g(h−1
S (x)),
−1
Ψ (g) : Ip → X , para todo g ∈ CS e para todo x ∈ Ip (hS sendo a mesma função usada para
−1
denir Ψ). Portanto, Ψ é uma bijeção e Φ [{S}] possui p! elementos, como havíamos armado.
−1
Assim, da igualdade centralizada acima, B é um conjunto nito, pois Φ [{S}] é não vazio
para todo S ∈ B , e, sendo α o número de elementos de B , temos

n! n!
= α · p! ⇒ α =
(n − p)! p!(n − p)!
(Corolário 1 do Teorema 6), como queríamos.
Podemos fazer também uma segunda solução, motivada pela Solução 1 da Armação da
Solução do exercício anterior.

Solução 2. Fixemos p ∈ {0} ∪ N. Se um conjunto X possui p elementos, então X admite um


único subconjunto com p elementos (Teorema 4). Logo, a armação é verdadeira para n = p:
p!
1= . Suponhamos agora a armação verdadeira para um dado n > p, ou seja, que
p!(p − p)!
n!
qualquer conjunto com n elementos admita m = subconjuntos com p elementos.
p!(n − p)!
Fixado o conjunto X = {x1 , . . . , xn+1 }, com n+1 elementos (denindo 0 + 1 = 1), queremos
(n + 1)!
mostrar que o conjunto A = {S ∈ P(X); S tem p elementos} possui elementos.
p!(n + 1 − p)!
2.14. EXERCÍCIO 14 47

Para cada i ∈ In+1 , seja Ai = {S ∈ P(X − {xi }); S tem p elementos}. Pela hipótese de
indução, Ai tem m elementos, para cada i ∈ In+1 . Seja a função
n+1
[
Φ: Ai × {i} → A
i=1
(S, i) 7→ Φ(S, i) = S.

Para ela, temos


n+1
[ [
Ai × {i} = Φ−1 [{S}],
i=1 S∈A
−1
onde Φ [{S}] possui n + 1 − p elementos, qualquer que seja S ∈ A. Isto porque (T, i) ∈
−1
Φ [{S}] ⇔ T = S e xi ∈ / S , equivalência esta que nos dá uma bijeção entre Φ−1 [{S}] e X − S ,
a associação (S, i) 7→ xi . Portanto, A é nito, e, sendo α seu número de elementos, temos

(n + 1) · m = α · (n + 1 − p),

donde,

1 n! 1 (n + 1)!
α = (n + 1) · m · = (n + 1) · · = ,
n+1−p p!(n − p)! n + 1 − p p! · (n + 1 − p)!

como queríamos.

Resposta. Um conjunto X com n elementos possui

0, se n < p, e
n!
, se n > p,
p!(n − p)!

subconjuntos com p elementos (para poder escrever essa resposta desse modo sucinto, estamos
considerando 0 < x e x > 0 para todo x ∈ N e que não existe x ∈ {0} ∪ N tal que 0>x ou tal
que x < 0). 

Curiosidade. Dados um conjunto X e um p ∈ {0} ∪ N, é bastante utilizado em Teoria dos


p
Conjuntos o símbolo [X] para denotar o conjunto {S ∈ P(X); X tem p elementos}. Com
uma noção mais ampla de números cardinais, a notação [X] também ganha sentido para todo
α
α
número cardinal α: [X] = {S ∈ P(X); card(S) = α}.

(Voltar para a Solução do Exercício 16: página 49.)

2.14 Exercício 14
Prove que se A tem n elementos, então P(A) tem 2n elementos.

Solução 1 (binômio de Newton). Podemos aplicar o exercício anterior aqui. Para cada
p ∈ {0, . . . , n}, seja
Ap = {S ∈ P(A); S tem p elementos},
48 CAPÍTULO 2. CONJUNTOS FINITOS, ENUMERÁVEIS E NÃO-ENUMERÁVEIS

de modo que
n
[
P(A) = Ap ,
p=0

além do que, Ap ∩ Aq = ∅ se p 6= q , pois, se dois conjuntos não têm o mesmo número de


elementos, então eles não podem ser iguais. Portanto, pelo Corolário 1 do Teorema 6, P(A)
n
X n!
possui elementos, onde
p=0
p!(n − p)!
n n
X n! X n!
= · 1p · 1n−p = (1 + 1)n = 2n ,
p=0
p!(n − p)! p=0
p!(n − p)!

pelo binômio de Newton. 

Observação. Denotando por [A]p o conjunto {S ∈ P(A); S tem p elementos},


p ∈ {0} ∪ N
[n
(ver a Curiosidade antes deste exercício), se A tem n ∈ {0}∪N elementos, então P(A) = [A]p .
p=0

Solução 2 (bijeção). Por um lado, vimos no livro a bijeção ξ : P(A) → F(A; {0, 1}). Por
n
outro, o conjunto F(A; {0, 1}) tem 2 elementos (Corolário 4 do Teorema 6). Logo, P(A)
n
também possui 2 elementos: se ϕ : F(A; {0, 1}) → I2n é uma bijeção, então ϕ ◦ ξ : P(A) → I2n
também é uma bijeção 

Solução 3 (indução). Para n = 0, P(A) é igual a {∅} e, portanto, tem apenas um elemento,
0 n
pois existe um único conjunto sem elementos, e 1 = 2 = 2 . Logo, a propriedade vale para
n = 0. Vamos supor agora que P(Y ) tenha 2n elementos, qualquer que seja o conjunto Y
com n elementos. Fixado o conjunto X = {x1 , . . . , xn+1 } (com 1 elemento se n = 0), vamos
mostrar que P(X) possui 2n+1 elementos. Em primeiro lugar, isso vai implicar a validade da
propriedade para n = 1, portanto, pelo Princípio da Indução, teremos o resultado desejado
para todo n ∈ N e, então, como já vericamos que o resultado também vale para n = 0, para
todo n ∈ {0} ∪ N.
De uma forma geral, para fazer demonstrações desse tipo, devemos encontrar um modo de,
a partir do caso n + 1, obter o caso n. A título de exemplo, consideremos o conjunto {a, b, c},
com 3 elementos. Todos os seus subconjuntos são:

∅ {a} {b} {a, b}


{c} {a, c} {b, c} {a, b, c}.
Observemos que os conjuntos da linha de cima são todos os subconjuntos de {a, b, c} − {c} =
{a, b} e que os elementos da linha de baixo são da forma Z ∪ {c}, com Z ⊂ {a, b}. Além disso,
o número de elementos da linha de baixo é igual ao número de elementos da linha de cima.
Seja entãoY = X − {xn+1 } e consideremos o conjunto A = {Z ∪ {xn+1 } ∈ P(X); Z ∈
P(Y )}. Deste modo, temos: P(X) = P(Y ) ∪ A, P(Y ) ∩ A = ∅ e card(P(Y )) = card(A),
porque a associação Z ∪ {xn+1 } 7→ Z , Z ∈ P(Y ), é uma bijeção entre A e P(Y ), portanto,
n
pela hipótese de indução, ambos os conjuntos P(Y ) e A possuem 2 elementos. Assim, pelo
Corolário 1 do Teorema 6, P(X) tem

2n + 2n = 2 · 2n = 2n+1
elementos, como queríamos. 
2.15. EXERCÍCIO 15 49

2.15 Exercício 15
Dena uma função sobrejetiva f :N→N tal que, para todo n ∈ N, o conjunto f −1 (n) seja
innito.

Solução. Uma primeira tentativa pode ser a de denir f −1 [{n}] como o conjunto dos múltiplos
de n. Porém, há um problema aqui. Já tivemos a oportunidade de ver anteriormente  por
exemplo, na Solução do Exercício 10 (página 42)  que imagens inversas de conjuntos unitários
distintos são disjuntas, e não é o que ocorre nessa tentativa, o conjunto dos múltiplos de m
não é disjunto do conjunto dos múltiplos de n (o elemento mn pertence a ambos). Podemos
−1
corrigir isso lançando mão dos números primos, denindo f [{n}] como o conjunto de todas
as potências do n-ésimo número primo.
De um modo mais preciso (só para deixar o conjunto dos primos mais organizado), seja
P o conjunto dos números primos e seja ψ : N → P
uma bijeção (podemos considerar essa
−1
bijeção porque já sabemos que P é um conjunto enumerável). Dado n ∈ N, seja f [{n}] =
m m
{ψ(n) ; m ∈ N}. Temos então f (ψ(n) ) = n ∀m ∈ N. Falta agora denir a f nos outros
m
elementos de N que não são potências de nenhum primo. Para tanto, seja X = {p ; p ∈
P e m ∈ N}. Basta denir f (x) = 1 para todo x ∈ N −X . 

2.16 Exercício 16
Prove que se X é innito enumerável, o conjunto das partes nitas de X também é (innito)
enumerável.

(Voltar para a Solução do Exercício 20, item a): página 52.)

(Voltar para a Solução do Exercício 20, item b): página 53.)

Solução. Seja

[X]n = {Y ∈ P(X); o número de elementos de Y é n} ∀n ∈ {0} ∪ N .


(ver a Curiosidade após o Exercício 13, página 47). Com esta notação, uma parte do que quere-
[∞
mos mostrar é que o conjunto [X]n é enumerável. Tal conjunto, conforme a notação indica,
n=0
é a reunião da família ([X]n )n∈{0}∪N . Armamos que cada [X]n é enumerável (a justicativa
disto está no parágrafo abaixo). Portanto, pelo Corolário 2 do Teorema 10, segue uma parte
do resultado desejado: é enumerável qualquer reunião enumerável de conjuntos enumeráveis.

[ ∞
[
n
A outra parte, a de que [X] é innito, segue do Teorema 4, porque [X] ⊂1
[X]n e
n=0 n=0
1
card([X] ) = card(X)
(a justicativa desta igualdade também está no parágrafo abaixo).
n
Vamos demonstrar a armação acima por indução em n. Para n = 0, [X] = {∅} e, para
n = 1, basta associar {x} 7→ x ∀{x} ∈ [X]1 , associação esta bijetiva. É também por causa dela
1 n+1
que [X] é innito. Dado n ∈ N, para o caso n + 1, se z ∈ [X] , então podemos escrever
n
z = x ∪ {y} para algum x ∈ [X] e algum y ∈ X − x. Disto obtemos a igualdade
[ [
[X]n+1 = {x ∪ {y}}.
x∈[X]n y∈X−x

O resultado agora segue da asserção de que [X]n é enumerável 


50 CAPÍTULO 2. CONJUNTOS FINITOS, ENUMERÁVEIS E NÃO-ENUMERÁVEIS

2.17 Exercício 17
Seja f : X → X uma função. Um subconjunto Y ⊂ X chama-se estável relativamente a f
quando f (Y ) ⊂ Y . Prove que um conjunto X é nito se, e somente se, existe uma função
f : X → X que só admite os subconjuntos estáveis ∅ e X .

Solução.

Nível 1. Temos duas implicações a demonstrar: X nito ⇒ existe uma função f : X → X


que só admite os subconjuntos estáveis ∅eX (vamos demonstrar esta no Nível 2.1) e; se existe
uma função f :X→X que só admite os subconjuntos estáveis ∅ e X, então X é nito (vamos
demonstrar esta no Nível 2.2).

No elevador. Vamos tentar a função sucessor (entre aspas porque a rigor o dominío da
função sucessor é o conjunto dos naturais, mas aqui sequer temos X ⊂ N). Não temos X ⊂ N,
mas o fato de X ser nito vai nos dar uma identicação deX com algum subconjunto de N,
então vamos poder continuar pensando em X como um conjunto de naturais, para guiar as
ideias. Sendo nito, todo subonjunto Y de X admite um maior elemento. Então, de fato a
função sucessor vai tirar esse maior elemento do conjunto Y . O problema é o que fazer com
o maior elemento de X , a função sucessor tiraria ele de X e ela caria sem sentido. Tentamos
levá-lo ao primeiro elemento e isso funciona.

Nível 2.1. Suponha X x : In → X uma bijeção, a qual nos permite escrever


nito e seja
X = {x1 , . . . , xn }. Vamos denir f : X → X por f (xi ) = xi+1 ∀i ∈ In−1 e f (xn ) = x1 (se X
tem apenas um elemento  e portanto n − 1 perderia o sentido  então não há muito o que
fazer).
Agora só falta mostrar que essa função de fato funciona. De modo mais preciso, que se
Y ⊂X f [Y ] ⊂ Y e se Y 6= ∅ (para Y = ∅ não há nada a fazer), então Y = X .
é tal que
Já observamos que xn ∈
/ Y ⇒ Y não é estável: se xn ∈ / Y , tome xm ∈ Y tal que m > i
∀xi ∈ Y ; então f (xm ) = xm+1 ∈/ Y . Portanto f [Y ] ⊂ Y ⇒ xn ∈ Y . Aplicar f aos elementos de
Y não vai tirá-los de Y :

xn ∈ Y ⇒ f (xn ) = x1 ∈ Y ⇒ f (x1 ) = x2 ∈ Y ⇒ · · · ⇒ f (xn−2 ) = xn−1 ∈ Y.

Logo Y = X, como queríamos.

Nível 2.2. Vamos demonstrar a contrapositiva, isto é, vamos supor X innito, xar uma
função f :X→X e construir um conjunto estável relativamente a f distinto de X e não vazio.
Vamos fazer essa construção no Nível 3.

No elevador. Um conjunto claramente estável é o conjunto

Y = {y, f (y), f 2 (y), . . . , f n (y), . . .},

onde denotamos f1 = f e f n+1 = f ◦ f n ∀n ∈ N.


Y 6= X , então teremos acabado. Se
Se
Y = X , então devemos tirar algum elemento conveniente de Y . Tirar o y não o faz perder
n
sua estabilidade, a não ser que junto com o y acabe saindo algum f (y) = y . Daí teremos
n−1 n−1
f (y) ∈ Y mas f (f (y)) ∈ / Y.
2.18. EXERCÍCIO 18 51

Nível 3. y ∈ X . Se Y = {y, f (y), . . . , f n (y), . . .} 6= X , então este é o conjunto


Fixe
n
procurado. Se Y = X e se f (y) 6= y ∀n ∈ N, então Y − {y} é estável e é distinto de X .
n 2 n−1
Se y = f (y) para algum n ∈ N, consideremos o conjunto Z = {y, f (y), f (y), . . . , f } (se
1−1 0
y = f (y), dena f (y) = f (y) = y ). Este Z , sendo nito, é claramente distinto de X , que
estamos supondo innito. Além disso ele também é estável. 

(Voltar para a Solução do Exercício 29: página 63.)

Curiosidades. Novamente aqui podemos fazer um paralelo com a Álgebra Linear: dado um
operador linear T : V → V , sendo V um espaço vetorial, um subespaço W ⊂V com a pro-
priedade T [W ] ⊂ W é chamado de T -invariante, ou invariante sob T. Isso de ir aplicando f
sucessivamente também aparece como ideia central numa demonstração do Teorema do Ponto
Fixo de Banach ; segundo a qual, sob certas condições, não importa qual y você tome, a sequên-
cia dos f n (y)'s sempre vai se aproximar de um mesmo ponto (ver, por exemplo, o Exercício
25 do Capítulo VIII, página 304).

2.18 Exercício 18
Seja f : X → X uma função injetiva tal que f (X) 6= X . Tomando x ∈ X − f (X), prove que
os elementos x, f (x), f (f (x)), . . . são dois a dois distintos.

Solução.

No elevador. Vamos ver o que acontece nos primeiros passos. Isso vai nos dar a ideia
de qual estratégia adotar. O x e o f (x) f (x) ∈ f [X], mas
são distintos simplesmente porque
x ∈/ f [X] por hipótese. Daí, da injetividade de f : x 6= f (x) ⇒ f (x) 6= f 2 (x). Também
f (x) 6= x, porque f 2 (x) ∈ f [X]. Até aqui temos então que os elementos de {x, f (x), f 2 (x)}
2
3
são dois a dois distintos. Ao acrescentar o f (x) ele deve ser distinto de cada um dos elementos
2 2 3 2 3
que já estavam lá, vejamos: f (x) 6= f (x) ⇒ f (x) 6= f (x), f (x) 6= x ⇒ f (x) 6= f (x) e
3 3
f (x) 6= x porque f (x) ∈ f [X]. E assim por diante.

n
Nível 1. No Nível 2 vamos mostrar que os elementos de {x, f (x), . . . , f (x)} são dois a
1 n+1 n
dois distintos, para todo n ∈ N, onde f = f e f = f ◦ f . Assim, dados m, n ∈ N,
m n m n
com m > n, teremos f (x) 6= f (x) porque ambos f (x) e f (x) pertencem ao conjunto
{x, f (x), . . . , f m (x)}.

Nível 2. A ideia central já está contida no elevador. Temos

f n (x) 6= f i (x) ∀i ∈ In−1 ⇒ f n+1 (x) 6= f i+1 (x) ∀i ∈ In−1 , ∀n ∈ N −{1},

f n (x) 6= x ⇒ f n+1 (x) 6= f (x), ∀n ∈ N,

e f n+1 (x) 6= x simplesmente porque f n+1 (x) ∈ f [X], mas x∈


/ f [X]. A tese agora segue por
indução. 
52 CAPÍTULO 2. CONJUNTOS FINITOS, ENUMERÁVEIS E NÃO-ENUMERÁVEIS

2.19 Exercício 19
Sejam X um conjunto innito e Y um conjunto nito. Mostre que existe uma função sobrejetiva
f : X → Y e uma função injetiva g : Y → X .
(Voltar para a Solução do Exercício 26: página 57.)

Solução.

No elevador. A questão aqui é a de encontrar um subconjunto de X com o mesmo


número de elementos de Y. Encontrado um tal conjunto, poderemos denir nossas funções
com o auxílio dele.

Nível 1. Pelo Teorema 7, existe um subconjunto Z ⊂ X enumerável. Seja z : N → Z


uma bijeção. A partir dela escrevemos Z = {z1 , z2 , . . . , zm , . . .}, zi = z(i). Supondo Y 6= ∅,
sejam n ∈ N e y : In → Y uma bijeção. De modo análogo temos Y = {y1 , . . . , yn }, yi = y(i).
Agora denimos f : X → Y fazendo f (zi ) = yi ∀i ∈ In e f (x) = y1 ∀x ∈ X − {z1 , . . . , zn }, e
denimos g : Y → X fazendo g(yi ) = zi ∀i ∈ In .
Essas funções estão bem denidas, porque y e z são bijeções. A sobrejetividade de f vem
da sobrejetividade de y e a injetividade de g vem da sobrejetividade de y e da injetividade de
z (junto com o fato de y ser uma função). 

2.20 Exercício 20
a) Se X é nito e Y é enumerável, então F(X; Y ) é enumerável.

b) Para cada função f : N → N seja Af = {n ∈ N; f (n) 6= 1}. Prove que o conjunto X das
funções f : N → N tais que Af é nito é um conjunto enumerável.

Solução.

item a)

No elevador. Um resultado que pode ajudar aqui é o Corolário 1 do Teorema 8. Por


ele, basta encontrarmos uma função injetiva entre F(X; Y ) e algum conjunto que sabemos ser
enumerável. O conjunto das partes nitas de Y é enumerável, de acordo com o Exercício 16
(página 49). O problema é que duas funções f g
podem ser tais que f [X] = g[X] mesmo se
e
n
elas forem distintas. Um outro conjunto enumerável é Y , o produto cartesiano de n fatores
iguais a Y. Ele é enumerável por causa do Teorema 10. É ele quem iremos usar no parágrafo
abaixo.

Nível 1. Vamos supor X e Y não vazios. Seja n o número de elementos de X , n ∈ N, e


escrevamos X = {x1 , . . . , xn }. Por uma observação feita acerca do Teorema 10 (na página 51
Yn
n
do livro), sabemos que Y = Yi , com Yi = Y ∀i ∈ In , é enumerável. A associação
i=1

F(X; Y ) 3 f 7−→ (f (x1 ), . . . , f (xn )) ∈ Y n


é claramente injetiva. Logo F(X; Y ) é enumerável pelo Corolário 1 do Teorema 8. 
2.21. EXERCÍCIO 21 53

item b)

Nível 1. Y enumerável e uma


Assim como no item anterior, vamos construir um conjunto
função injetiva Φ : X − X1 → Y , sendo X1 um subconjunto enumerável de X . Feito isso, nosso
teorema estará demonstrado pelo Corolário 1 do Teorema 8 e pelo Corolário 2 do Teorema 10,
pois X = (X − X1 ) ∪ X1 será uma reunião enumerável de enumeráveis.

No elevador. Veja como Af f . Mas, novamente, precisamos tomar cuidado


identica
com essa identicação, pois podemos ter Af = Ag mesmo que f 6= g . Precisamos encontrar
uma associação injetiva. O que podemos fazer é associar f a uma função Φ(f ) : Af → N
denida por Φ(f )(n) = f (n) ∀n ∈ Af . Essa sim é injetiva. Mas qual é o contradomínio de
Φ? Temos Φ(f ) ∈ F(Af ; N), onde Af é uma parte nita de N, e o conjunto das partes nitas
de N é enumerável pelo Exercício 16 (página 49). Um outro cuidado que iremos tomar é com
o conjunto vazio, ∅ é um conjunto nito por denição, além do que ∅ = Ah , para h : N → N
denida por h(n) = 1 para todo n ∈ N. Felizmente, colocar um elemento (a h tal que Ah = ∅
é única) num conjunto enumerável não afeta sua enumerabilidade. Agora sim temos o terreno
preparado para a nossa construção.

Nível 2. L o conjunto de todas as partes nitas não vazias de N; L é enumerável


Seja
pelo Exercício 16 (L é a diferença entre um conjunto enumerável, o conjunto de todas as partes
[
nitas de N, e um conjunto unitário, o conjunto {∅}), de modo que o conjunto Y := F(A; N)
A∈L
também é enumerável: ele é uma reunião enumerável de enumeráveis (Corolário 2 do Teorema
10), uma vez que F(A; N) é enumerável para cada A ∈ L, pelo item anterior.
Seja h : N → N a função denida por h(n) = 1 para todo n ∈ N. Vamos denir Φ :
X − {h} → Y como a função Φ(f ) : Af → N denida por Φ(f )(n) = f (n) ∀n ∈ Af , para cada
f ∈ X.
Ela é injetiva. Dados f, g ∈ X − {h} distintos, existe um x ∈ N tal que f (x) 6= g(x). Não
é possível que este x esteja fora de Af e fora de Ag ao mesmo tempo, pois x ∈/ Af ⇒ f (x) = 1
e, portanto, x ∈ / Af e x ∈ / Ag implicaria f (x) = g(x). Portanto x ∈ Af ou x ∈ Ag . Se
Af = Ag , então a presença desse x em Af = Ag é o que torna Φ(f ) 6= Φ(g). Se Af 6= Ag , então
Φ(f ) 6= Φ(g) porque seus domínios são distintos.
Logo, pelo Corolário 1 do Teorema 8, X−{h} é enumerável, e, portanto, X = (X−{h})∪{h}
é também enúmerável (Corolário 2 do Teorema 10). 

2.21 Exercício 21
Obtenha uma decomposição N = X1 ∪X2 ∪· · ·∪Xn ∪· · · tal que os conjuntos X 1 , X2 , . . . , X n , . . .
são innitos e dois a dois disjuntos.

(Voltar para a Solução do Exercício 24: 56.)

(Voltar para a Solução do Exercício 28, item f ): 62.)

(Voltar para a Primeira Parte da Solução do Exercício 15 do Capítulo V: 167.)

Solução. Vamos usar o Teorema Fundamental da Aritmética. Esse teorema vai nos dar as
duas partes desse exercício  tanto o N como uma reunião, quanto o fato de essa reunião ser de
54 CAPÍTULO 2. CONJUNTOS FINITOS, ENUMERÁVEIS E NÃO-ENUMERÁVEIS

elementos dois a dois disjuntos . Seja P = {pi ; i ∈ N} o conjunto dos números primos. Vamos
m m n
denir X1 = {p1 ; m ∈ N}, X2 = {p1 · p2 ; m ∈ N e n ∈ N} e, de um modo geral:

mi
Xi = {pm m2
1 · p2 · pi ; m1 , . . . , mi ∈ N} ∀i ∈ N .
1


[
Temos N= Xi , porque todo número natural pode ser decomposto em fatores primos, e
i=1
Xi ∩ Xj = ∅ se i 6= j , porque essa decomposição é única. 

2.22 Exercício 22
Dena f : N × N → N, pondo f (1, n) = 2n − 1 e f (m + 1, n) = 2m · (2n − 1). Prove que f é
uma bijeção.

(Voltar para a Solução 2 do Exercício 5 do Capítulo IV: página 121.)

Solução.

É injetiva. Aqui é fazer contas considerando todos os casos possíveis

• Sejam (1, x) e (1, y), com x 6= y . Assim: f (1, x) = 2x − 1 6= 2y − 1 = f (1, y);

• Sejam (1, x) e (y + 1, z). O número f (1, x) é ímpar, enquanto f (y + 1, z) é par. Portanto


eles não podem ser iguais;

• Sejam (x + 1, y) e (z + 1, w) distintos. Se x = z , então y 6= w e 2y − 1 6= 2w − 1 ⇒


2 (2y − 1) 6= 2z (2w − 1). Se y = w, então x 6= z e 2x 6= 2z ⇒ 2x (2y − 1) 6= 2z (2w − 1).
x

É sobrejetiva. y ∈ N ou é par ou é ímpar. Se é ímpar, seja n ∈ N tal que


Um dado
y = 2n − 1. Para este n temos y = f (1, n). Se é par, podemos escrevê-lo como y = 2m (2n − 1),
o divindo o máximo que pudermos por 2, de modo que o fator que sobre é ímpar. Para este m
e este n temos y = f (1 + m, n). 

2.23 Exercício 23
Seja X⊂N um subconjunto innito. Prove que existe uma única bijeção crescente f :N→X

Solução. A Demonstração do Teorema 8 já constroi uma função assim e, depois, seu Corolário
2 rearma a sua existência. Aqui vamos dar uma solução diferente da do livro. Sendo min Y o
menor elemento de um dado Y ⊂ N não vazio (o que dá sentido a essa notação é o Princípio da
Boa Ordenação, junto com o fato de que esse menor elemento, quando existe, é único), vamos
denir f :N→X fazendo
f (1) = min X
e
f (n + 1) = min{x ∈ X; x > f (n)} ∀n ∈ N .
Essa f está bem denida porque {x ∈ X; x > f (n)} é sempre não vazio, uma vez que X é
innito (e, portanto, não vazio e não limitado, Teorema 5).
2.23. EXERCÍCIO 23 55

f é injetiva e crescente. Basta mostrar que ela é crescente. Da construção de f é


imediato que f (n + 1) > f (n). Se m < n, seja n = m + p. Vamos mostrar que f (m) < f (n)
por indução em p. Isso segue de f (m + p + 1) > f (m + p). Assim, se f (m + p) > f (m), então,
por transitividade, f (m + p + 1) > f (m). Logo, dado m ∈ N, f (m + p) > f (m), para todo
p ∈ N.

f é sobrejetiva. De modo informal, a nossa f está ordenando o conjunto X; se um


certo y∈X é o  x-ésimo menor elemento de X, então y = f (x). Nós só precisamos escrever
isso de modo mais formal.
Fixemos um m ∈ X. Se m = 1, então m = f (1). Se m 6= 1, seja n ∈ N seu antecessor e
consideremos o conjunto F = In ∩ X . Se F = ∅, então m é o menor elemento de X e, portanto,
m = f (1). Se F tem p elementos, então m = f (p + 1). Vamos demonstrar essas armações
abaixo.
Se m não é o menor elemento de X (lembrando que m = s(n)), então existe um x ∈ X
estritamente menor do que m; este x, portanto, pertencente a In , isto é, In ∩ X 6= ∅. Logo,
pela contrapositiva, In ∩ X = ∅ ⇒ m = min X , o que conclui uma das armações feitas. Com
relação à outra, vamos proceder por indução em p. Assim, vamos supor que In ∩ X tenha
p+1 elementos e ques(x) = f (p + 1), se s(x) ∈ X
e se Ix ∩ X tem p elementos. Queremos
mostrar que m = f (p + 2). Como In ∩ X é um subconjunto nito não vazio de N, ele admite
um maior elemento (Teorema 5). Seja ele a. Se a = 1, então só pode ser n = 1, p = 1 e m = 2,
portanto, a = f (1) e m = f (2). Se a 6= 1, seja b tal que s(b) = a. Agora o conjunto Ib ∩X tem p
elementos, de modo que a = f (p + 1) pela hipótese de indução. Para concluir que m = f (p + 2),
vamos demonstrar que m possui as propriedades que denem o f (p + 2). Por um lado, m > a,
pois a 6 n < m. Por outro, se existisse um x ∈ X menor do que m e maior do que a, então a
não seria o máximo do conjunto In ∩ X . Logo m = f (p + 2), como queríamos.

Unicidade. Seja g:N→X uma função bijetiva e crescente. Vamos mostrar que g(n) =
f (n) n ∈ N. Vamos fazê-lo por indução.
para todo
Como 1 6 n ∀n ∈ N, temos g(1) 6 g(n) ∀n ∈ N. Do fato de g ser sobrejetiva, segue que
todo elemento de X é da forma g(n) para algum n. Assim, podemos armar que g(1) 6 x
∀x ∈ X . Portanto g(1) = min X = f (1).
Suponha f (n) = g(n), de modo que f (n + 1) = min{x ∈ X; x > g(n)}. Do fato de g ser
crescente, temos g(n + 1) > g(n). Agora, para concluir que g(n + 1) = f (n + 1), precisamos
mostrar que g(n + 1) é o menor elemento de X maior do que g(n). Se existisse um x ∈ X
tal que g(n) < x < g(n + 1), seja x = g(m) (por causa da sobrejetividade), então teríamos
g(n) < g(m) < g(n + 1) e, portanto, (por g ser crescente) existiria um m tal que n < m < n + 1.
Mas já vimos (no Exercício 5, página 32) que um tal m não existe. Logo g(n + 1) = f (n + 1),
como queríamos. 

Observações. A rigor, o que justica a existência da função construída na Solução acima


é o Teorema da Recursão (visto na Solução do Exercício 4, página 25) e o que justica a
existência da função construída na Demonstração do Teorema 8 é o método geral de denição
[
por indução (ver página 30). Sejam as funções g : X → X e h : X n → X denidas por
n∈N
g(m) = min{x ∈ X; x > m}, para cada m ∈ N, e h (x1 , . . . , xm ) = min X − {x1 , . . . , xm },
m
para cada m ∈ N e cada (x1 , . . . , xm ) ∈ X . Seja também a = min X . Então, pelo Teorema da
Recursão, existe ϕ : N → X tal que ϕ(1) = a e ϕ(n + 1) = g(ϕ(n)) = min{x ∈ X; x > ϕ(n)},
para todo n ∈ N, e, pelo método geral de denição por indução, existe ψ : N → X tal que
56 CAPÍTULO 2. CONJUNTOS FINITOS, ENUMERÁVEIS E NÃO-ENUMERÁVEIS


ψ(1) = a ψ(n + 1) = h (ψ(1), . . . , ψ(n)) = min X − {ψ(1), . . . , ψ(n)}, para todo n ∈ N. A
e
função da Solução acima é a função ϕ e a função da Demonstração do Teorema 8 é a função ψ .

2.24 Exercício 24
Prove que todo conjunto innito se decompõe como reunião de uma innidade enumerável de
conjuntos innitos, dois a dois disjuntos.

Solução. No Exercício 21 (página 53) já resolvemos isso para o caso em que esse conjunto

[
innito é N. Lá escrevemos N= Xi , onde cada Xi é innito e Xi ∩ Xj = ∅ se i 6= j . Vamos
i=1
usar isso para demonstrar o caso geral.
Seja X um conjunto innito. Pelo Teorema 7, seja Y ⊂X um conjunto enumerável. Seja

[
f :N→Y uma bijeção. Assim, considerando os Xi 's do parágrafo acima, temos Y = f [Xi ],
i=1
onde cada f [Xi ] continua innito por causa da injetividade de f e f [Xi ] ∩ f [Xj ] = ∅ se i 6= j ,
pois f [Xi ] ∩ f [Xj ] 6= ∅ ⇒ Xi ∩ Xj 6= ∅. Mas ainda falta tratarmos um pedaço do X , a saber,
o pedaço X − Y . Se ele for innito, então teremos acabado. Se ele for nito, então podemos
incorporá-lo em algum pedaço do Y , no primeiro, por exemplo. De modo mais preciso façamos
o seguinte.

• Se X −Y é innito, dena Y1 = X − Y e Yi+1 = f [Xi ] ∀i ∈ N;

• Se X −Y é nito, dena Y1 = f [X1 ] ∪ (X − Y ) e Yi+1 = f [Xi ] ∀i ∈ N.


De qualquer modo temos X escrito como uma reunião da família (Yi )i∈N , família essa enu-
merável, de elementos dois a dois disjuntos e com cada Yi innito. 

2.25 Exercício 25
Seja A um conjunto. Dadas duas funções f, g : A → N, dena a soma f + g : A → N, o produto
f · g : A → N, e dê o signicado da armação f 6 g . Indicando com ξX a função característica
de um subconjunto X ⊂ A, prove :

a) ξX∩Y = ξX · ξY ;

b) ξX∪Y = ξX + ξY − ξX∩Y . Em particular,

ξX∪Y = ξX + ξY ⇔ X ∩ Y = ∅;

c) X ⊂ Y ⇔ ξX 6 ξY ;

d) ξA−X = 1 − ξX .

Solução. Vamos denir

(f · g)(x) = f (x) · g(x), (f + g)(x) = f (x) + g(x) ∀x ∈ A

e
f 6 g ⇔ f (x) 6 g(x) ∀x ∈ A.
2.26. EXERCÍCIO 26 57

item a) Dado x ∈ A, ou x pertence a ambos X e Y ou x não pertence a algum dos dois,


X ou Y . Se x ∈ X ∩ Y , então ξX∩Y (x) = 1 e ξX (x) = ξY (x) = 1. Se x ∈
/ X ou se x ∈/ Y , então
0 ∈ {ξX (x), ξY (x)}, de modo que ξX (x) · ξY (x) = 0 e ξX∩Y (x) = 0. De qualquer forma temos
ξX∩Y (x) = ξX (x) · ξY (x), para todo x ∈ A. 

item b) x ∈ A, temos três casos a considerar: ou x ∈


Dado / X ∪Y , ou x pertence a apenas
um dos dois X ou Y  e, portanto, x ∈ X ∪ Y , mas x ∈ / X ∩ Y , ou x ∈ X ∩ Y . Fixemos
então x ∈ X ∪ Y tal que x ∈
/ X ∩ Y , um y ∈ X ∩ Y e um z ∈ / X ∪ Y (z ∈ A). Temos:

ξX (z) + ξY (z) − ξX∩Y (z) = 0 = ξX∪Y (z),

(ξX (x) + ξY (x)) − ξX∩Y (x) = 1 − 0 = 1 = ξX∪Y (x),

ξX (y) + ξY (y) − ξX∩Y (y) = 1 + 1 − 1 = 1 = ξX∪Y (y).


Logo, ξX∪Y = ξX + ξY − ξX∩Y .
Se X ∩ Y = ∅, então ξX∩Y (x) = 0 ∀x ∈ A. Assim:

ξX∪Y (x) = ξX (x) + ξY (x) − ξX∩Y (x) = ξX (x) + ξY (x) ∀x ∈ A.

X ∩ Y 6= ∅, seja x ∈ X ∩ Y . Por um lado, ξX∪Y (x) = 1. Por outro, ξX (x) + ξY (x) = 2.


Se
Portanto, ξX∪Y 6= ξX + ξY . Logo, pela contrapositiva, ξX∪Y = ξX + ξY ⇒ X ∩ Y 6= ∅. 

item c) Vamos mostrar a contrapositiva da equivalência em questão. Se x ∈ X −Y , então


ξX (x) = 1 > 0 = ξY (x). Um tal x existe se X não está contido em Y . Reciprocamente, se
existe x ∈ A tal que ξX (x) > ξY (x), então só pode ser ξX (x) = 1 e ξY (x) = 0, donde x ∈ X e
x∈/ Y . A existência deste x implica que X não está contido em Y . Segue a tese. 

item d) Podemos aplicar o item b) aqui. Escrevendo A = X∪(X−A), como X∩(A−X) =


∅, temos

ξA = ξX + ξA−X ,
ou seja, podemos dizer que ξA−X = 1 − ξX , já que ξA (x) = 1 para todo x ∈ A. 

2.26 Exercício 26
Prove que o conjunto das sequências crescentes (n1 < n2 < n3 < · · · ) de números naturais não
é enumerável.

Solução.

Nível 1. X o conjunto dessas sequências, isto é, o conjunto das funções x : N → N


Seja
tais que i < j ⇒ x(i) < x(j). Fixada a função f : N → X , inspirados pelo Argumento da
Diagonal de Cantor, armamos que f não é sobrejetiva (Nível 2). Assim, pelo Exercício 19
(página 52), não existindo uma função sobrejetiva de N em X , como N é innito, X não é
nito. E, não existindo uma função sobrejetiva de N em X , também não existe uma bijeção de
N em X . Logo, X não é enumerável.
58 CAPÍTULO 2. CONJUNTOS FINITOS, ENUMERÁVEIS E NÃO-ENUMERÁVEIS

No elevador. Queremos construir uma sequência x:N→N crescente tal que x 6= f (n)
∀n ∈ N, ou seja, tal que para cada n∈N exista um in satisfazendox(in ) 6= f (n)(in ). Cada in ,
assim como no Argumento da Diagonal de Cantor, poderá ser simplesmente in = n e teremos
x(n) 6= f (n)(n) ∀n ∈ N.
Dado o f (1)(1) ∈ N, podemos denir x(1) como um natural diferente de f (1)(1). Dado o
f (2)(2), podemos denir o x(2) como um natural diferente de f (2)(2) e maior do que x(1). E
assim por diante.

Nível 2. x : N → N uma sequência tal que x(1) 6= f (1)(1), x(n + 1) > x(n) e
Seja
x(n + 1) 6= f (n + 1)(n + 1), para todo n ∈ N (denido o x(n), o conjunto N −{f (n + 1)(n + 1)}
é não vazio e não limitado; em particular, x(n) não o limita e podemos denir o x(n + 1) do
modo como acabamos de escrever). Uma tal sequência é crescente e não pertence a imagem de
f, pois x 6= f (n) para todo n ∈ N. Logo, f não é sobrejetiva, como queríamos. 

Observações. Para um certo grau de rigor, podemos admitir a existência de uma sequência
x com as propriedades mencionadas no Nível 2. Mas, para um grau de rigor um pouco maior,
cabe a pergunta: ela existe? Para responder essa pergunta, pelo fato de x(n + 1) depender de
x(n), podemos começar aplicando o Teorema da Recursão (página 25), por exemplo, à função
h : N → N denida por h(n) = min{y ∈ N −{f (n + 1)(n + 1)}; y > n}, o que nos leva a uma
função ϕ : N → N tal que ϕ(n+1) = h(ϕ(n)) = min{y ∈ N −{f (ϕ(n)+1)(ϕ(n)+1)}; y > ϕ(n)},
o que de fato dá ϕ(n + 1) > ϕ(n), mas não necessariamente ϕ(n + 1) 6= f (n + 1)(n + 1), uma
vez que ϕ(n + 1) 6= f (ϕ(n) + 1)(ϕ(n) + 1).
geral de denição (página 30), considerando a g1 :
[ Podemos corrigir isso usando o método
n

N → N denida por g1 (y1 , . . . , yn ) = min N −{y1 , . . . , yn , f (n + 1)(n + 1)}. Agora temos
n∈N 
ψ : N → N tal que ψ(1) = min N −{f (1)(1)} e tal que ψ(n + 1) = g1 (ψ(1), . . . , ψ(n)) =
min N −{ψ(1), . . . , ψ(n), f (n + 1)(n + 1)} e, de fato, ψ(n + 1) 6= f (n + 1)(n + 1). Porém, o
problema agora é com a propriedade de ψ ser crescente. Por exemplo, se for f (1)(1) = 1, então
ψ(1) = 2 e ψ(2) = min N −{ψ(1), f (2)(2)} = 1, se f (2)(2) = ψ(1), ou seja, ψ(2) < ψ(1).
Continuando com essas denições através de mínimos, com relação à monotonicidade, no-
tamos que se X ⊂ Y, então min X > min Y , poismin X ∈[Y . Agora sim temos um modo

Nn → N

de contemplar as duas propriedades desejadas, denindo g : por g (y1 , . . . , yn ) =
n∈N
min N −{y1 , . . . , yn , f (1)(1), . . . , f (n + 1)(n + 1)}. Para esta g , pelo método geral de denição
por indução, existe uma função x : N → N tal que x(1) = min N −{f (1)(1)} e tal que x(n+1) =

g (x(1), . . . , x(n)) = min N −{x(1), . . . , x(n), f (1)(1), . . . , f (n + 1)(n + 1)}, para todo n ∈ N.
Denindo B1 = N −{f (1)(1)} e Bn+1 = N −{x(1), . . . , x(n), f (1)(1), . . . , f (n + 1)(n + 1)}, para
todo n ∈ N, como x(n) = min Bn e

B1 ⊃ B2 ⊃ · · · ⊃ Bn ⊃ Bn+1 ⊃ · · · ,

temos x(n + 1) > x(n), para todo n ∈ N; além do que, x(n + 1) 6= x(n), porque x(n) ∈ / Bn+1 .
Logo, x(n + 1) > x(n), para todo n ∈ N, e x é uma função crescente.
Resumindo, o que garante a existência de uma função x : N → N crescente e tal que
x(n)[6= f (n)(n), para todo n ∈ N, é o método geral de denição por indução aplicado à função
Nn → N, denida por g (y1 , . . . , yn ) = min N −{y1 , . . . , yn , f (1)(1), . . . , f (n+1)(n+1)},

g:
n∈N
para cada n∈N e cada (y1 , . . . , yn ) ∈ Nn , e ao número a = min N −{f (1)(1)}. Esta aplicação
2.27. EXERCÍCIO 27 59


arma a existência de uma função x ∈ N → N tal que x(1) = a e x(n+1) = g (x(1), . . . , x(n)) ,
função esta que se encaixa em nossos propósitos.

2.27 Exercício 27
Sejam (N, s) e (N0 , s0 ) dois pares formados, cada um, por um conjunto e uma função. Suponha-
0
mos que ambos cumpram os axiomas de Peano. Prove que existe uma única bijeção f : N → N
0 0
tal que f (1) = 1 , f (s(n)) = s (f (n)). Conclua que

a) m < n ⇔ f (m) < f (n);


b) f (m + n) = f (m) + f (n) e

c) f (m · n) = f (m) · f (n).

Solução. Aqui, o 10 denota o único elemento do conjunto N0 −s0 [N0 ]. Seja f : N → N0 uma
função tal que f (1) = 10 e f (s(n)) = s0 (f (n)). Suas existência e unicidade são garantidas
pelo Teorema da Recursão, já visto (e demonstrado) na Solução do Exercício 4 (página 25).
Considerando o conjunto

X = {x ∈ N0 ; f −1 [{x}] possui 1 elemento},


0 0 0
vamos mostrar que X = N pelo Princícipio da Indução (cumprido pelo par (N , s )). Ter
X = N0 signica que f é uma bijeção; sobrejetiva porque f −1 [{x}] 6= ∅, para todo x ∈ N0 , e
injetiva porque o número de elementos de f
−1
[{x}] não é maior do que 1, para todo x ∈ N0 .
0 0 −1
O elemento 1 pertence a X , porque f (1) = 1 ⇒ 1 ∈ f [{10 }] e porque
/ s0 [N0 ] ⇒
10 ∈ 10 6= s0 (x) ∀x ∈ N0
⇒ 10 6= s0 (f (n)) ∀n ∈ N
⇒ 10 6= f (s(n)) ∀n ∈ N
⇒ 10 6= f (m) ∀m ∈ N −{1},
−1
ou seja, nenhum número natural diferente de 1 pertence a f [{10 }]. Logo, f −1 [{10 }] = {1}.
−1
Dado x ∈ X , seja f [{x}] = {n}. Queremos mostrar que s0 (x) ∈ X . Em primeiro lugar,
temos

n ∈ f −1 [{x}] ⇒ f (n) = x
⇒ s0 (f (n)) = s0 (x)
⇒ f (s(n)) = s0 (x)
⇒ s(n) ∈ f −1 [{s0 (x)}].
Além disso, se m ∈ f −1 [{s0 (x)}], então f (m) = s0 (x). Em particular, m 6= 1, pois f (m) ∈ s0 [N0 ],
enquanto / s0 [N0 ]. Seja então k ∈ N tal que m = s(k), de modo que
f (1) ∈
s0 (x) = f (m) = f (s(k)) = s0 (f (k)) ⇒ x = f (k) ⇒ k ∈ f −1 [{x}] ⇒ k = n ⇒ m = s(n),
(cuja primeira destas implicações vale pela injetividade de s0 ). Logo, f −1 [{s0 (x)}] = {s(n)} e,
0
de fato, s (x) ∈ X , como queríamos.
0
Segue a tese, isto é, X = N .
Com respeito aos itens a), b) e c), vamos entender a adição, a relação de ordem e a multi-
0
plicação em N denidas de modo inteiramente análogo aos seus correspondentes em N. O item
b) vais nos auxiliar com os itens a) e c), o que faz sentido porque as denições da relação de
ordem e da multiplicação dependem da adição. Vamos começar então por ele.
60 CAPÍTULO 2. CONJUNTOS FINITOS, ENUMERÁVEIS E NÃO-ENUMERÁVEIS

item b) Fixado m ∈ N, vamos demonstrar por indução que f (m + n) = f (m) + f (n)


∀n ∈ N. Temos:

f (m + 1) = f (s(m)) = s0 (f (m)) = f (m) + 10 = f (m) + f (1)

e, se f (m + n) = f (m) + f (n), então

f (m + s(n)) = f (s(m + n))


= s0 (f (m + n))
= s0 (f (m) + f (n))
= f (m) + s0 (f (n))
= f (m) + f (s(n)),

igualdades estas nas quais m + s(n) = s(m + n) e s0 (f (m) + f (n)) = f (m) + s0 (f (n)) pelas
0
denições das somas em N e em N . Logo, f (m + n) = f (m) + f (n), para todo n ∈ N. E, como
o m era qualquer, f (m + n) = f (m) + f (n), para todo m ∈ N e todo n ∈ N.

item a) Se m < n, p ∈ N tal que n = m + p e, portanto, f (n) = f (m + p) =


então existe
f (m) + f (p), f (m) < f (n).
pelo item b), o que implica
0
Reciprocamente, se f (m) < f (n), então existe q ∈ N tal que f (n) = f (m) + q . Sendo f
sobrejetiva, seja p ∈ N tal que q = f (p). Portanto, f (n) = f (m) + q = f (m) + f (p) = f (m + p),
pelo item b). Assim, pela injetividade de f , temos n = m + p, logo m < n.

item c) Fixado m ∈ N, vamos demonstrar por indução que f (m·n) = f (m)·f (n) ∀n ∈ N.
Temos:
f (m · 1) = f (m) = f (m) · 10
e, supondo f (m · n) = f (m) · f (n),

f (m · s(n)) = f (m · n + m)
= f (m · n) + f (m)
= f (m) · f (n) + f (m)
= f (m) · s0 (f (n))
= f (m) · f (s(n)),

cuja segunda destas igualdades vale pelo item b). Logo, f (m · n) = f (m) · f (n), para todo
n ∈ N. E, como o m era qualquer, f (m · n) = f (m) · f (n), para todo m ∈ N e todo n ∈ N. 

2.28 Exercício 28
Dada uma !sequência de conjuntos A1!
, A2 , . . . , An , . . ., considere os conjuntos lim sup An =

\ [∞ ∞
[ ∞
\
Ai e lim inf An = Ai .
n=1 i=n n=1 i=n

a) Prove que lim sup An é o conjunto dos elementos que pertencem a An para uma innidade
de valores de n e que lim inf An é o conjunto dos elementos que pertencem a todo An
salvo para um número nito de valores de n.
2.28. EXERCÍCIO 28 61

b) Conclua que lim inf An ⊂ lim sup An .



[
c) Mostre que se An ⊂ An+1 para todo n então lim inf An = lim sup An = An .
n=1


\
d) Por outro lado, se An ⊃ An+1 para todo n então lim inf An = lim sup An = An .
n=1

e) Dê exemplo de uma sequência (An ) tal que lim sup An 6= lim inf An .

f ) Dê exemplo de uma sequência para a qual os dois limites coincidem mas Am 6⊂ An


quaisquer que sejam m e n.

Solução.

item a) Seja x ∈ lim sup An . Pela denição de lim sup An , para todo n ∈ N, existe um
natural i > n tal que x ∈ Ai . Isto signica que o conjunto {i ∈ N; x ∈ Ai } é ilimitado (dado
n ∈ N existe i ∈ N, i > n + 1 tal que x ∈ Ai ; i > n), portanto, innito, de acordo com o
Teorema 5. Reciprocamente, seja x um elemento tal que {i ∈ N; x ∈ Ai } é innito. Sendo
innito, este conjunto também é ilimitado (Teorema 5), isto é, para todo n ∈ N, existe um
natural i > n tal que x ∈ Ai , o que signica que x ∈ lim sup An .
Seja x ∈ lim inf An . Pela denição de lim inf An , existe n ∈ N tal que x ∈ Ai para todo
natural i > n e, então, o conjunto F ⊂ N tal que x ∈ An para todo n ∈ N −F é nito, por
ser subconjunto de {1, . . . , n}, um conjunto nito (Teorema 4). Reciprocamente, seja x um
elemento tal que x ∈ An para todo n ∈ N −F , para algum conjunto F ⊂ N nito. Novamente
pelo Teorema 5, F sendo nito, existe n ∈ N tal que n > i para todo i ∈ F . Para este n, temos

\
x∈ Ai , pois, se i > n, então i ∈
/ F , ou seja, x ∈ Ai . Portanto, x ∈ lim inf An . 
i=n

item b) Se x ∈ lim An , então, pelo item anterior, existe um subconjunto F ⊂ N nito


inf
tal que x ∈ An para todo n ∈ N −F . Como N −F é innito, x pertence a An para uma
innidade de valores n, condição suciente para x estar em lim sup An , conforme já vimos no
item interior. Logo, lim inf An ⊂ lim sup An . 

∞ ∞
" ∞
#
\ \ \
item c) Nessas condições, temos Ai = An ∀n ∈ N, pois Ai = An ∩ Ai , e
i=n i=n i=n+1

\
An ⊂ An+p , para todo p ∈ N, implica An ⊂ Ai (o resultado agora segue da propriedade ∩5,
i=n+1
vista no Capítulo I), valendo a inclusão An ⊂ An+p , para todo p ∈ N, por indução. Portanto:

∞ ∞
! ∞
[ \ [
lim inf An = Ai = An .
n=1 i=n n=1

Agora, pelo item anterior, só falta mostrar que lim sup An ⊂ lim inf An . Se x ∈ lim sup An ,
então o conjunto {i ∈ N; x ∈ Ai } é innito (item a)). Em particular, ele é não vazio e existe
n ∈ N tal que x ∈ An . Portanto, para um tal n, x ∈ Am ∀m > n, já que An ⊂ Am para todo
m > n. Assim, existe F ⊂ N nito tal que x ∈ An para todo n ∈ N −F , F ⊂ {i ∈ N; i < n},
62 CAPÍTULO 2. CONJUNTOS FINITOS, ENUMERÁVEIS E NÃO-ENUMERÁVEIS

ou seja, x ∈ lim inf An , pelo item a). Logo, lim sup An ⊂ lim inf An . E, como já tínhamos a
inclusão lim inf An ⊂ lim sup An , segue lim inf An = lim sup An . 

∞ ∞
" ∞
#
[ [ [
item d) Nessas condições, temos Ai = An ∀n ∈ N, pois Ai = An ∪ Ai , e
i=n i=n i=n+1

[
An+p ⊂ An , para todo p ∈ N, implica Ai ⊂ An (o resultado agora segue da propriedade ∪5,
i=n+1
vista no Capítulo I), valendo a inclusão An+p ⊂ An , para todo p ∈ N, por indução. Portanto:

∞ ∞
! ∞
\ [ \
lim sup An = Ai = An .
n=1 i=n n=1

De modo análogo ao item anterior, pelo item b), só falta mostrar que lim sup An ⊂ lim inf An .
Se x ∈ lim sup An , então, pela igualdade acima, temos x ∈ An ∀n ∈ N. Logo, x ∈ lim inf An ,
pelo item a) (o conjunto F de índices tal que x ∈
/ Ai , para i ∈ F , é nito porque vazio).
Portanto lim sup An ⊂ lim inf An . 

item e) Podemos simplesmente tomar dois conjuntos disjuntos X e Y e denir A2n = X


e A2n−1 = Y ∀n ∈ N. Assim, para cada x ∈ X ∪ Y , ambos os conjuntos {i ∈ N; x ∈ Ai } e
{i ∈ N; x ∈
/ Ai } são innitos. Por exemplo, denindo X = {2n ∈ N; n ∈ N} e Y = {2n − 1 ∈
N; n ∈ N}, temos: lim sup An = N e lim inf An = ∅. 

item f ) Vamos tomar uma família de conjuntos dois a dois disjuntos, de modo que, para
todo x, o conjunto de índices L tal que n ∈ L ⇔ x ∈ An não pode ser innito. Por exem-
plo, a família dos Xi 's construída no Exercício 21 (página 53). Para uma tal família temos:
lim sup An = lim inf An = ∅. 

2.29 Exercício 29. Teorema de Cantor-Bernstein-Schröder


Dados os conjuntos A e B suponha que existam funções injetivas f :A→B e g : B → A.
Prove que existe uma bijeção h : A → B. (Teorema de Cantor-Bernstein-Schröder.)

Solução.

No elevador. No livro Naive Set Theory do Halmos, livro sugerido pelo Elon ao men-
cionar este fato no texto, logo após a demonstração deste teorema, é deixado um exercício cujo
resultado pode ser usado para uma segunda demonstração, diferente da apresentada lá. É este
resultado que iremos aproveitar nesta solução.

Nível 1. Armamos que existe um subconjunto X de A tal que g[B−f [X]] = A−X (Nível
2). Seja X um tal conjunto. Dado x ∈ A, temos apenas duas possibilidades: ou x ∈ X , ou
x ∈ A−X . Se x ∈ A−X , então existe b ∈ B−f [X] tal que x = g(b), já que A−X = g[B−f [X]].
Existe e é único, por causa da injetividade de g . Denimos então h : A → B fazendo

h(x) = f (x) ∀x ∈ X e h(g(b)) = b ∀b ∈ B − f [X].


2.29. EXERCÍCIO 29. TEOREMA DE CANTOR-BERNSTEIN-SCHRÖDER 63

Ela é injetiva porque: f é injetiva; g é uma função, e; h(x) ∈ f [X] se x ∈ X , enquanto


h(x) ∈/ f [X] se x ∈ A − X . E é sobrejetiva porque, dado b ∈ B : se b ∈
/ f [X], então b = h(g(b)),
e; se b ∈ f [X], então b = h(x) para x ∈ X tal que f (x) = b.

No elevador. Como g é injetiva, temos g[B − f [X]] = g[B] − g[f [X]] (Exercício 12 do
Capítulo I, página 11). Na Solução do Exercício 17 (página 50) vimos um conjunto do tipo
2
X = {y, ϕ(y), ϕ (y), . . . , ϕn (y), . . .}, no qual ϕ = g ◦ f ; ele é tal que X = {y} ∪ ϕ[X] e, então,
vale a inclusão g[B] − ϕ[X] ⊂ A − X , caso y ∈ / g[B]. Para valer a inclusão inversa, dado
a ∈ A − X , uma primeira condição é que a pertença à imagem de g . Este a é distinto de y , e
y∈/ g[B], mas isto não signica necessariamente que a esteja em g[B]. Em vez então de formar
X a partir de sucessivas aplicações de ϕ a um ponto fora de g[B], vamos formá-lo a partir de
sucessivas aplicações de ϕ a todo o conjunto A − g[B].

Nível 2. Seja ϕ=g◦f e consideremos o conjunto

!
[
ϕn A − g[B]
  
X = A − g[B] ∪
n∈N

(se A − g[B] = ∅, então não há nada a se fazer, g já será uma bijeção). De

X = (A − g[B]) ∪ ϕ[X]
[
ϕn A −

(igualdade análoga à que observamos no elevador acima; ela vale porque ϕ[X] =
 n∈N
g[B] , por uma propriedade enunciada na página 26 do livro), temos

 
A−X = A − (A − g[B]) ∩ A − ϕ[X]
= g[B] ∩ (A − ϕ[X])
= g[B] − ϕ[X]
= g[B] − g[f [X]]
= g[B − f [X]].

Com respeito a estas 5 últimas igualdades, a primeira, a segunda e a terceira são aplicações de
propriedades enunciadas na página 10 do livro, a quarta vale pela denição de ϕ e, a quinta,
pela injetividade de g (Exercício 12 do Capítulo I, item 2). Logo, um tal conjunto X é quem
pode nos auxiliar a construir a bijeção h do Nível 1, pelo fato de ele satisfazer a igualdade

A − X = g[B − f [X]]. 
64 CAPÍTULO 2. CONJUNTOS FINITOS, ENUMERÁVEIS E NÃO-ENUMERÁVEIS
Capítulo 3
Números Reais

3.1 Exercício 1
Dados a, b, c, d num corpo K, sendo b e d diferentes de zero, prove:

o a c ad + bc
1 ) + = ;
b d bd
o a c a·c
2 ) · = .
b d b·d

(Voltar para a Solução do Exercício 13: página 74.)

(Voltar para a Solução do Exercício 55, Nível 1.1: página 102.)

Solução. Começando com a propriedade do elemento neutro, temos

a c a c
+ = ·1+ ·1
b d b d
−1 −1
(Inverso Multiplicativo) = (a · b ) · (d · d) + (c · d−1 ) · (b−1 · b)
−1
(Associatividade) = a · (b · (d−1 · d)) + c · (d−1 · (b−1 · b))
−1
(Associatividade) = a · ((b · d−1 ) · d) + c · ((d−1 · b−1 ) · b)
−1
(Comutatividade) = a · (d · (b · d−1 )) + c · (b · (d−1 · b−1 ))
−1
(Associatividade) = (a · d) · (b · d−1 ) + (c · b) · (d−1 · b−1 )
−1
(Comutatividade) = (a · d) · (b · d−1 ) + (c · b) · (b−1 · d−1 )
−1
(Distributividade) = ((a · d) + (c · b)) · (b · d−1 ).
o
Para nalizar o item 1 ), vejamos que b−1 · d−1 = (b · d)−1 ,
b−1 · d−1 é o inverso
isto é, que
multiplicativo de b · d (de fato b · d 6= 0, conforme já observado no livro, pois b 6= 0 e d 6= 0).
−1
Para tanto, pela unicidade do inverso multiplicativo, basta vericar que (b · d−1 ) · (b · d) = 1:

(b−1 · d−1 ) · (b · d) = (d−1 · b−1 ) · (b · d)


(Associatividade) = d−1 · (b−1 · (b · d))
(Associatividade) = d−1 · ((b−1 · b) · d)
(Inverso Multiplicativo) = d−1 · (1 · d)
(Elemento Neutro) = d−1 · d
(Inverso Multiplicativo) = 1.

65
66 CAPÍTULO 3. NÚMEROS REAIS

a c a·d+c·b
Logo, + = (a · d + c · b) · (b−1 · d−1 ) = (a · d + c · b) · (b · d)−1 = .
b d b·d
Além disso,

a c
· = (a · b−1 ) · (c · d−1 )
b d
(Associatividade) = a · (b−1 · (c · d−1 ))
(Associatividade) = a · ((b−1 · c) · d−1 )
(Comutatividade) = a · ((c · b−1 ) · d−1 )
(Associatividade) = a · (c · (b−1 · d−1 ))
(Associatividade) = (a · c) · (b−1 · d−1 )
(b · d−1 = (b · d)−1 ) =
−1
(a · c) · (b · d)−1
a·c
= ,
b·d
o
como queríamos para o 2 ) item. 

3.2 Exercício 2
1
Dado a 6= 0 num corpo K, põe-se, por denição, a0 = 1 e, se n ∈ N, a−n = , ou seja,
an
a−n = (an )−1 . Prove:

o
1 ) am · an = am+n ;

o
2 ) (am )n = amn sejam quais forem m, n ∈ Z.

Solução.

item 1o ) Para n = 0, temos

am · an = am · a0 = am · 1 = am = am+0 = am+n .

n ∈ N ∪{0}, vamos supor am ·an = am+n (vamos chamar isto de hipótese de indução).
Fixado
n+1
Respectivamente pela denição de a , pela associatividade da multiplicação, pela hipótese
m+n m+n+1
de indução e porque a ·a=a , temos:

am · an+1 = am · (an · a)
= (am · an ) · a
= am+n · a
= am+n+1 .

Logo, am · an = am+n ∀n ∈ N ∪{0}.


3.2. EXERCÍCIO 2 67

Se n∈Z e n < 0, então −n ∈ N, de modo que, pelo que acabamos de demonstrar, temos

1 1
am · an = ·
a−m a−n
1
=
a−m · a−n
1
= −m−n
a
1
= −(m+n)
a
= am+n .
1
Faltando apenas justicar por que an = qualquer que seja o n ∈ Z. Para n = 0 é imediato.
a−n
Se n < 0, então −n ∈ N; daí, pela denição

1
an = a−(−n) = .
a−n
Se n ∈ N, temos:
1 1
= 1 = an .
a−n an
Isto conclui este item. 

item 2o ) Para n = 0:

(am )n = (am )0 = 1 = a0 = am·0 = amn .

Para n ∈ N ∪{0}, supondo (am )n = amn e usando o item anterior (bem como a denição de
potências com expoentes naturais), temos

(am )n+1 = (am )n · am


= amn · am
= amn+m
= am(n+1) .

Portanto (am )n = amn ∀n ∈ N ∪{0}. Se n < 0, então −n ∈ N, de modo que podemos usar o
resultado que acabamos de demonstrar:

(am )n = (am )−(−n)


1
=
(a )−n
m

1
= −mn
a
= amn .

Logo (am )n = amn para todos m, n inteiros, como queríamos. 


o
(Voltar para o item 1 ) do Exercício 60: página 115.)
68 CAPÍTULO 3. NÚMEROS REAIS

3.3 Exercício 3
x1 x2 xn
Se = = ··· = num corpo K , prove que, dados a1 , . . . , a n ∈ K tais que a1 y 1 + · · · +
y1 y2 yn
a1 x 1 + · · · + an x n x1
an yn 6= 0, tem-se = .
a1 y 1 + · · · + an y n y1

Solução. Temos:

xi
= α ∀i ∈ {1, . . . , n} ⇒ xi = αyi ∀i ∈ {1, . . . , n}
yi
⇒ ai xi = αai yi ∀i ∈ {1, . . . , n}
⇒ a1 x1 + · · · + an xn = α(a1 y1 + · · · + an yn )
a1 x1 + · · · + an xn
⇒ =α
a1 y 1 + · · · + an y n
a1 x 1 + · · · + an x n x1
⇒ = . 
a1 y 1 + · · · + an y n y1

3.4 Exercício 4
Sejam K , L corpos. Uma função f : K → L chama-se um homomorsmo quando se tem
f (x + y) = f (x) + f (y) e f (x · y) = f (x) · f (y), quaisquer que sejam x, y ∈ K .

i) Dado um homomorsmo f : K → L, prove que f (0) = 0.

ii) Prove também que, ou f (x) = 0 para todo x ∈ K, ou então f (1) = 1 e f é injetivo.

(Voltar para a Solução do Exercício 55, Nível 1.4: página 104.)

(Voltar para a Solução do Exercício 56, Nível 4: página 108.)

Solução. Temos:

f (0) = f (0 + 0) = f (0) + f (0) ⇒ f (0) = f (0) − f (0) = 0.

Se f não é identicamente nula, seja x∈K tal que f (x) 6= 0. Assim:

f (x)
f (x) = f (1 · x) = f (1) · f (x) ⇒ f (1) = = 1.
f (x)

Com relação a injetividade, se x, y ∈ K são tais que f (x) = f (y), então 0 = f (x) − f (y) =
f (x − y). Assim, basta mostrar que f (x) = 0 ⇒ x = 0 (é claro que isso não é verdade
caso f (x) = 0 ∀x ∈ K , que também é um homomorsmo), o que nos dará: f (x − y) =
0 ⇒ x − y = 0 ⇒ x = y . Se x 6= 0, então podemos considerar seu inverso multiplicativo:
1 = f (1) = f (x · x−1 ) = f (x) · f (x−1 ). Em particular f (x) 6= 0. Logo, f (x) = 0 ⇒ x = 0
(contrapositiva). 

Observações. De 1 = f (x) · f (x−1 ), obtemos também f (x−1 ) = [f (x)]−1 . O mesmo acontece


com os inversos aditivos:

0 = f (0) = f (x − x) = f (x) + f (−x) ⇒ f (−x) = −f (x) ∀x ∈ K.


3.5. EXERCÍCIO 5 69

3.5 Exercício 5
Seja f : Q → Q um homomorsmo. Prove que, ou f (x) = 0 para todo x ∈ Q ou então f (x) = x
para todo x ∈ Q.
(Voltar para a Solução do Exercício 56, Primeria Parte: página 107.)

Solução. Como Q é um corpo, podemos aplicar sucessivamente o exercício anterior aqui:

f (1) = 1 ⇒ f (2) = f (1 + 1) = f (1) + f (1) = 1 + 1 = 2,

e assim por diante: f (n + 1) = f (n) + 1 = n + 1, se n ∈ N e se f (n) = n. Além disso, como


já havíamos observado, f (−n) = −f (n) = −n se n ∈ N, o que nos dá f (x) = x ∀x ∈ Z (se
f (1) = 1). Agora, se m, n ∈ Z, com n 6= 0, como f (x−1 ) = (f (x))−1 (para x 6= 0, se f não é
identicamente nula), temos
m f (m) m
f = = .
n f (n) n
Logo f (x) = x ∀x ∈ Q, se f não é identicamente nula. 

3.6 Exercício 6
Verique as associatividades da adição e da multiplicação em Z2 . (Nota. Há dois modos de se
proceder. Um requer a vericação de 16 igualdades. Outro consiste em observar que, denindo-
se f : Z → Z2 f (n) = 0 se n é par, e f (n) = 1 se n é ímpar, f é sobrejetiva e, para m, n ∈ Z
por
quaisquer, valem f (m + n) = f (m) + f (n), f (m · n) = f (m) · f (n). As associatividades em Z
implicam nas de Z2 .)

Solução. A sobrejetividade de f é imediata. Com respeito à preservação das operações,


temos: se m e n são pares, então m + n e m · n são pares e, portanto, f (m + n) = 0 = 0 + 0 =
f (m) + f (n) e f (m · n) = 0 = 0 · 0 = f (m) · f (n); se apenas um dos dois, m ou n, é ímpar,
então m + n é ímpar e m · n é par, de modo que, f (m + n) = 1 = 1 + 0 = f (m) + f (n) e
f (m · n) = 0 = 1 · 0 = f (m) · f (n), e; se ambos m e n são ímpares, então m + n é par e m · n é
ímpar e, por conseguinte, f (m + n) = 0 = 1 + 1 = f (m) + f (n) (0 = 1 + 1 na adição em Z2 ) e
f (m · n) = 1 = 1 · 1 = f (m) · f (n).
Dados a, b, c ∈ Z2 , sejam x, y, z ∈ Z tais que f (x) = a, f (y) = b e f (z) = c. Assim, temos:

(a + b) + c = (f (x) + f (y)) + f (z)


= f (x + y) + f (z)
= f ((x + y) + z)
= f (x + (y + z))
= f (x) + f (y + z)
= f (x) + (f (y) + f (z))
= a + (b + c),

como queríamos! Para a multiplicação, as contas são inteiramente análogas. 


70 CAPÍTULO 3. NÚMEROS REAIS

3.7 Exercício 7
Sejap um número natural primo. Para cada inteiro m, indiquemos com m o resto da divisão
de m por p. No conjunto Zp = {0, 1, · · · , p − 1} denamos duas operações: uma adição ⊕ e
uma multiplicação , pondo m ⊕ n = m + n e m n = m · n. Prove que a função f : Z → Zp ,
denida por f (n) = n, cumpre f (m + n) = f (m) ⊕ f (n) e f (m · n) = f (m) f (n). Conclua
que ⊕ e são comutativas, associativas, vale a distributividade, existem 0 e 1. Observe que
dados m, n ∈ Zp , m n = 0 ⇒ m = 0 ou n = 0. Conclua que Zp é um corpo.

Solução. Na divisão euclidiana, escrevamos m = ap + q


n = bp + r, de modo que f (m) = q ,
e
f (n) = r e m + n = (a + b)p + (q + r). Escrevamos também q + r = cp + s, com 0 6 s < p.
Assim: m + n = (a + b + c)p + s. Portanto: f (m + n) = s = q + r = q ⊕ r = f (m) ⊕ f (n).
Com relação ao produto, temos: m · n = (ap + q)(bp + r) = (abp + ar + qb)p + qr . Em sua
divisão euclidiana, seja qr = cp + s (não são necessariamente o mesmo c nem o mesmo s de
antes). Assim: f (m · n) = s = q · r = q r = f (m) f (n).
Do mesmo modo como aconteceu no exercício anterior, as comutatividades, associatividades
e a distributividade de ⊕ e de seguem da sobrejetividade de f, do fato de ela preservar as
operações e das respectivas propriedades em Z. Os elementos neutros da adição e da multipli-
cação são o 0 e o 1, respectivamente: 0 ⊕ x = 0 + x = x = x e 1 x = 1 · x = x = x, se x ∈ Zp .
O simétrico de um x ∈ Zp é p − x se x 6= 0 e é 0 se x = 0: x ∈ Zp , x 6= 0 ⇒ 0 < x < p ⇒ 0 <

p − x < p ⇒ p − x ∈ Zp , e x ⊕ (p − x) = x + (p − x) = p = 0.
Só falta mostrar que, dado um a ∈ Zp não nulo, existe um b ∈ Zp tal que b a = 1. A ideia
é considerar todos os produtos x a com x ∈ Zp e vericar que o conjunto {x a ∈ Zp ; x ∈ Zp }
tem p elementos, donde: {x a ∈ Zp ; x ∈ Zp } = Zp (Teorema 4 do Capítulo II) e x a = 1 para
algum x ∈ Zp (como queremos). É daí que vem a sugestão de mostrar que m = 0 ou n = 0 caso
m n = 0; dados x, y ∈ Zp : x a = y a ⇒ (x − y) a = 0 ⇒ x − y = 0 ou a = 0 ⇒ x − y =
0 ⇒ x = y , se a 6= 0 e se a sugestão for verdadeira. Essas contas signicam que a associação
x 7→ x a, para todo x ∈ Zp , é injetiva e, portanto, de fato o conjunto {x a ∈ Zp ; x ∈ Zp }
terá p elementos (Teorema 4 e seu Corolário 1, Capítulo II).
Vejamos então que, dados m, n ∈ Zp , m n = 0 ⇒ m = 0 ou n = 0. Podemos aproveitar
a f novamente. Sejam i e j números inteiros tais que f (i) = m e f (j) = n, de modo que:
m n = f (i) f (j) = f (ij) = 0. Isso signica que ij é um múltiplo de p, ou seja que p divide
ij . É um resultado de Álgebra que nos permite concluir agora que p divide i ou p divide j , por
p ser primo. Se p divide i, então m = f (i) = 0. E, se p divide j , então n = f (j) = 0. Seguem
os resultados desejados. 
(Voltar para a Solução do Exercício 42 do Capítulo III, item i: página 91.)

3.8 Exercício 8
Seja K um conjunto onde são válidos todos os axiomas de corpo, salvo a existência de inverso
multiplicativo.

i) Dado a 6= 0 em K , prove que a função f : K → K , denida por f (x) = ax, é uma bijeção
se, e somente se, a possui inverso.
ii) Mostre que f é injetiva se, e somente, vale a lei do corte para a.
iii) Conclua que, se K é nito, a lei do corte é equivalente à existência de inverso para cada
elemento não-nulo de K.
3.9. EXERCÍCIO 9 71

Solução. Se f é uma bijeção, então existe x ∈ K tal que f (x) = 1. Este x é justamente o
inverso do a. Se a possui inverso a−1 , então a função g : K → K denida por g(x) = a−1 x é a
inversa de f (à direita e à esquerda). Logo f é uma bijeção.
Se f é injetiva, então f (x) = f (y) ⇒ x = y . Em outras palavras ax = ay ⇒ x = y .
Reciprocamente, ax = ay ⇒ x = y signica que f é injetiva.
Fixe um a ∈ K não nulo. Se vale a lei do corte para a, então, pelo item ii), a f do item
i) é injetiva. Injetiva num conjunto nito, portanto sobrejetiva (Exercício 10 do Capítulo II
(página 42). Sendo bijeção, segue do item i) que a admite um inverso. Reciprocamente, se a
admite um inverso, então, pelo item i), f é uma bijeção. Em particular, injetiva. Portanto,
pelo item ii), vale a lei do corte para a. 

Interessante... Compare este exercício com o que foi feito na demonstração de que cada
elemento não nulo do Zp admite um inverso, do exercício anterior.

Observação. Num corpo, a lei do corte decorre da existência do inverso multiplicativo.


Agora, podemos ter um conjunto (innito) com uma operação para a qual valha a lei do corte,
mas na qual nem todo elemento admita um inverso. Por exemplo, o conjunto Z com a adição
e a multiplicação usuais. Para ele podemos demonstrar sua lei do corte a partir da lei do corte
dos números naturais (esta já foi demonstrada no Exercício 4 do Capítulo II; ver página 30).

3.9 Exercício 9
Explique por que as operações usuais não tornam corpos o conjunto Z dos inteiros nem o
conjunto Q[t] dos polinômios de coecientes racionais.

Solução. Em ambos existem elementos que não admitem inversos multiplicativos (esta é a
única propriedade que os impedem de ser corpos). Por exemplo, o elemento 2, que pertence ao
mesmo tempo a Ze a Q[t]: o 1 é o elemento neutro da multiplicação de ambos, 2x 6= 1 ∀x ∈ Z
e 2x 6= 1 ∀x ∈ Q[t]. 

3.10 Exercício 10
Num corpo ordenado K, prove que a2 + b2 = 0 ⇔ a = b = 0.

Solução. Uma implicação é mais imediata: a = b = 0 ⇒ a2 + b2 = 0 + 0 = 0. Vamos mostrar


agora a forma contrapositiva de sua recíproca. Sendo P ⊂ K o conjunto dos elementos positivos
2
de K , lembremos que x ∈ P qualquer que seja x ∈ K −{0}. Sem perda de generalidade, vamos
supor a 6= 0 e consideremos as duas únicas possibilidades para b: ou b = 0 ou b 6= 0. Se b = 0,
2 2 2 2 2 2 2
então a + b = a ∈ P . E, se b 6= 0, então a + b ∈ P , pois ambos a e b pertencem a P
2 2
(propriedade P1). Logo, de qualquer forma, a 6= 0 implica a + b ∈ P , o que, por sua vez,
2 2
implica a + b 6= 0 (propriedade P2), como queríamos. 

Observação. Num corpo não ordenado, a implicação a2 + b2 = 0 ⇒ a = b = 0 pode ser falsa.


2 2
Por exemplo, o Z2 : nele, para a = b = 1, temos a + b = 0, mas a 6= 0.
72 CAPÍTULO 3. NÚMEROS REAIS

3.11 Exercício 11
Seja P o conjunto dos elementos positivos de um corpo ordenado K.

i) Dado um número natural n, prove que a função f : P → P , denida por f (x) = xn , é


monótona crescente (isto é, x < y ⇒ f (x) < f (y)).

ii) Dê um exemplo em que f não é sobrejetiva.

iii) Prove que f (P ) não é um subconjunto limitado superiormente de K.

(Voltar para o Lema da Solução do Exercício 54: página 101.)

o
(Voltar para a Solução do Exercício 60, item 2 ): página 116.)

(Voltar para a Solução do Exercício 11 do Capítulo IV: página 123.)

(Voltar para o item (a) do Exercício 13 do Capítulo VI: página 217.)

(Voltar para a Solução do Exercício 23 do Capítulo VII: página 249.)

Solução. Sejam x, y ∈ P e suponha x < y, ou seja, y − x > 0. Queremos mostrar que


y n − xn > 0. Lembremos que

y n − xn = (y − x)(y n−1 + xy n−2 + x2 y n−3 + · · · + xn−2 y + xn−1 ).


i n−1−i
Cada uma das parcelas da forma x y pertence a P já que ambos xey também pertencem.
n n
Portanto, se y − x ∈ P , então y − x ∈ P , como queríamos.

Um exemplo em que f
não é sobrejetiva vem do nosso velho conhecido 2. Para K = Q e
2
n = 2, f não é sobrejetiva; não existe x ∈ P tal que f (x) = x = 2, onde 2 ∈ P.
Dado b ∈ K, queremos encontrar um x ∈ P tal que f (x) > b. Se b = 0, então não há
quase nada a fazer, qualquer x∈P serve. Vamos supor então b > 0. A f associa cada x a sua
enésima potência. A desigualdade de Bernoulli também envolve potências. Nessa desigualdade
temos a enésima potência da soma de 1 com algum elemento. Vejamos:

f (1 + b) = (1 + b)n > 1 + nb,

este, por sua vez, é maior do que b. f (1 + b) > b qualquer que


Logo seja b > 0, ou seja, f [P ]
não é um conjunto limitado superiormente em K , como queríamos. 

3.12 Exercício 12
Sejam X um conjunto qualquer e K um corpo. Indiquemos com F(X; K) o conjunto de todas
as funções f : X → K . Denamos em F(X; K) as operações de adição e de multiplicação de
modo natural: dadas f, g : X → K , as funções f + g : X → K e f · g : X → K são dadas por
(f + g)(x) = f (x) + g(x) e (f · g)(x) = f (x) · g(x). Verique quais dos axiomas de corpo são
válidos e quais não são válidos no conjunto F(X; K), relativamente a estas operações.

Solução.
3.13. EXERCÍCIO 13 73

Adição.
Associatividade. Dados f, g, h ∈ F(X; K) e x ∈ X, temos

(f + (g + h))(x) = f (x) + (g + h)(x)


= f (x) + (g(x) + h(x))
= (f (x) + g(x)) + h(x)
= (f + g)(x) + h(x)
= ((f + g) + h)(x).
Logo, f + (g + h) = (f + g) + h; a associatividade vale.
Comutatividade. Temos

(f + g)(x) = f (x) + g(x) = g(x) + f (x) = (g + f )(x),


para todo f, g ∈ F(X; K) e para todo x ∈ X. Logo a comutatividade vale.
Elemento neutro. O elemento neutro da adição de F(X; K) é a função h:X→K denida
por h(x) = 0, para todo x ∈ K; a função identicamente nula. Para ela, temos

(f + h)(x) = f (x) + h(x) = f (x),


para todo x ∈ X, ou seja f + h = f , para todo f ∈ F(X; K).
Simétrico. Dado f ∈ F(X; K), seu simétrico é a função g : X → K denida por g(x) =
−f (x), para todo x ∈ X . De fato:
(f + g)(x) = f (x) + g(x) = f (x) − f (x) = 0,
isto é, f +g é igual á função h do parágrafo anterior.

Multiplicação. Aqui, as contas que mostram as validades da associatividade, da comu-


tatividade e a existência do elemento neutro são semelhantes às de seus análogos da adição.
O elemento neutro sendo, desta vez, a função h : X → K h(x) = 1 para todo
denida por
x ∈ X . A única diferença surge no inverso multiplicativo. Dada uma função f : X → K não
nula, queremos uma função g : X → K tal que f (x) · g(x) = 1 para todo x ∈ X . Não podemos
−1
denir g(x) = f (x) para todo x ∈ X , pois pode existir x ∈ X tal que f (x) = 0 (f ser não
nula signica que existe x ∈ X tal que f (x) 6= 0, não que f (x) 6= 0 para todo x ∈ X ), o máximo
−1
que podemos fazer é denir g(x) = f (x) para todo x ∈ X tal que f (x) 6= 0.

Conclusão. De todos os axiomas de corpo, o único que não é necessariamente válido em


F(X; K) é o da existência do inverso multiplicativo (este axioma só é válido se X possuir apenas
um elemento). 

3.13 Exercício 13
Sejam x, y
elementos positivos de um corpo ordenado K. Tem-se x < y ⇔ x−1 > y −1 . Prove
−1
também que x > 0 ⇔ x > 0.
(Voltar para a Solução do Exercício 19: página 77.)

(Voltar para a Solução do Exercício 32: página 84.)

(Voltar para a Solução do Exercício 55, Nível 2.5: página 107.)


o
(Voltar para a Solução do Exercício 60, item 3 ): página 116.)
74 CAPÍTULO 3. NÚMEROS REAIS

Solução. Estes fatos já foram mencionados na página 68. Seja P o conjunto dos elementos
−1
positivos de K . Se x > 0, então x ∈ P e, de x · x = 1 ∈ P , em primeiro lugar, temos x−1 6= 0
−1
(pois x = 0 implicaria x · x−1 = 0) e, em segundo lugar, se fosse x−1 ∈ −P , então teríamos
−x ∈ P e x · (−x−1 ) = −(x · x−1 ) = −1 ∈
−1
/ P , o que é um absurdo (uma multiplicação de
−1
dois elementos de P cujo produto não pertence a P ). Logo, x ∈ P (propriedade P2). Deste
−1
modo, temos a implicação x > 0 ⇒ x > 0. Ela signica que o inverso multiplicativo de um
−1 −1
número positivo é positivo e, como o inverso multiplicativo de x é x (se x 6= 0), esta mesma
implicação nos dá:

x−1 > 0 ⇒ (x−1 )−1 > 0 ⇒ x > 0.

Logo, x > 0 ⇔ x−1 > 0.


Por este mesmo argumento (para f , denida por f (x) = x−1 ∀x ∈ K − {0}, f ◦ f = id),
−1
para x, y ∈ P , a equivalência x < y ⇔ x > y −1 vem da implicação x < y ⇒ x−1 > y −1 .
Vamos mostrá-la (o argumento central é o mesmo da página 68 do livro). Se x, y ∈ P e se
x < y , então xy ∈ P e y − x ∈ P . Também pertence a P , conforme vimos no parágrafo acima,
−1
o elemento (xy) . Portanto, pertence a P :

y−x y + (−1) · x 1 −1
(y − x) · (xy)−1 = = = + = x−1 + (−1) · y −1 = x−1 − y −1
xy xy x y

o
(a terceira igualdade vale pelo item 1 do Exercício 1, página 65), o que signica que x−1 > y −1 ,
como queríamos. 

3.14 Exercício 14
Seja a um elemento positivo de um corpo ordenado K . Denamos f : Z → K pondo f (n) = an .
(Veja o Exercício 2.) Prove que f é crescente se a > 1, decrescente se a < 1 e constante se
a = 1.

(Voltar para a Solução do Exercício 25: página 80.)

o
(Voltar para a Solução do Exercício 60, item 3 ): página 116.)

Solução. a > 1, vamos mostrar que an+1 > an . Este é o ponto central para concluirmos,
Se
n+p
por indução, que a > an para todo p ∈ N. Se a é positivo, então an também é positivo,
n n
de modo que, pela monotonicidade da multiplicação, a > 1 ⇒ a · a > a , ou seja, a > 1 ⇒
an+1 > an , como queríamos.
n+1
Suponha a < 1. Queremos mostrar que a < an . O que zemos no parágrafo anterior
n n
se aplica aqui também, pois a > 0 já que a é positivo por hipótese (caso a fosse negativo, a
n
seria negativo para n ímpar e multiplicar ambos os lados de a < 1 por a mudaria o sinal).
n
Se a = 1, então f (n) = 1 = 1 ∀n ∈ N. 

3.15 Exercício 15
Dados x 6= 0 num corpo ordenado K e n∈N qualquer, prove que (1 + x)2n > 1 + 2n · x.
3.16. EXERCÍCIO 16 75

Solução. Esta desigualdade se assemelha à de Bernoulli. Lá tínhamos (1 + x)n > 1 + nx


caso x 6= 0, x > −1 e n > 1. Um jeito interessante de aplicar Bernoulli é passar aquele 2 do
expoente para dentro:

(1 + x)2n = (1 + (x2 + 2x))n .


2
Agora, precisamos analisar quando x + 2x possui as propriedades necessárias para aplicarmos
2 2 2
Bernoulli: x + 2x > −1 para todo x ∈ K , uma vez que x + 2x + 1 = (x + 1) > 0; para x 6= 1,
2
esta desigualdade é estrita e, para x = −2 ou x = 0, apenas eles, x + 2x = x(x + 2) é igual a
zero. Portanto, se x 6= −1, x 6= −2, x 6= 0 e n > 1, então:

(1 + (x2 + 2x))n > 1 + n(x2 + 2x) = 1 + 2nx + nx2 > 1 + 2nx,

2 2
uma vez que nx > 0. Se n = 1, então queremos ver que 1 + x + 2x > 1 + 2x. Isto é equivalente
2
a x > 0, o que é o caso, já que x 6= 0. Portanto, o resultado é verdadeiro para n = 1 qualquer
2n
que seja o x não nulo. Se x = −1, então (1 + x) = 0, enquanto 1 + 2nx = 1 − 2n < 0. Se
2n
x = −2, então (1 + x) = 1, enquanto 1 + 2nx = 1 − 4n < 0. Segue a tese. 

3.16 Exercício 16
Se n∈N e x<1 num corpo ordenado K, prove que (1 − x)n > 1 − nx.

Solução. Temos:

x < 1 ⇒ −x > −1 ⇒ (1 − x)n = (1 + (−x))n > 1 + n(−x) = 1 − nx ∀n ∈ N . 

3.17 Exercício 17
Num corpo ordenado, se a e a+x são positivos, prove que (a + x)n > an + n · an−1 · x. Enuncie
e demonstre desigualdades análogas às dos Exercícios 15 e 16, com a em vez de 1.

Solução. Temos

  x n  x n  x
(a + x)n = a 1 + = an 1 + > an 1 + n · = an + nan−1 x,
a a a
x
pois an > 0 e a + x > 0 ⇒ x > −a ⇒ > −1.
a
x
Com relação aos Exercícios 15 e 16, seja a > 0. Para x 6= 0, como 6= 0, pelo Exercício 15,
a
temos

2n

2nx 2n 2n
 x
(a + x) =a 1+ >a 1 + 2n · = a2n + 2na2n−1 x ∀n ∈ N .
a a
x
Para fazer algo análogo ao 16, vamos supor < 1, ou seja, x < a. Para x < a, pelo
a
Exercício 16, temos:

x n x
 
(a − x)n = an 1 − a
> an 1 − n · a
= an − nan−1 x ∀n ∈ N . 
76 CAPÍTULO 3. NÚMEROS REAIS

Observação. A desigualdade de Bernoulli nos permite comparar uma curva de grau n com
uma reta de inclinação n:
n=2 n=3 n=4

3.18 Exercício 18
a+c
Sejam a, b , c , d elementos de um corpo ordenado K, onde b e d são positivos. Prove que
b+d
a c
está compreendido entre o menor e o maior dos elementos e . Generalize: mostre que
b d
a1 + · · · + an a1 an
está compreendido entre o menor e o maior dos elementos ,··· , , desde que
b1 + · · · + bn b1 bn
b1 , . . . , bn sejam todos positivos.

Solução. Temos

a a+c c b+d b+d


6 6 ⇔ a· 6a+c6c·
b b+d d b  d 
d b
⇔ a· 1+ 6a+c6c· 1+
b d
d b
⇔ a+a· 6a+c6c+c·
b d
d b
⇔ a· 6c e a6c·
b d
a c
⇔ 6 .
b d
 
a1 an
Genericamente, sejam α e β o menor e o maior elemento do conjunto ,..., , res-
b1 bn
pectivamente. Sendo In = {1, . . . , n}, temos

ak
α6 6β ∀k ∈ In ⇒ α · bk 6 ak 6 β · bk ∀k ∈ In
bk
⇒ α(b1 + · · · + bn ) 6 a1 + · · · + an 6 β(b1 + · · · + bn )
a1 + · · · + an
⇒ α6 6 β,
b1 + · · · + bn
como queríamos. 

3.19 Exercício 19

−1
x |x| −1
6 0, prove que |x·y | = |x|·|y| , ou seja =
Dados x, y num corpo ordenado K , com y = .
y |y|
3.20. EXERCÍCIO 20 77

−1
Solução Pelo item (ii) do Teorema 2, temos |x · y | = |x| · |y −1 |, só faltando mostrar que
−1 −1
|y | = |y| . Como y 6= 0, então y > 0 ou y < 0. Pelo Exercício 13 (página 73), y > 0 ⇒
y −1 > 0 e y < 0 ⇒ y −1 < 0. Assim, se y > 0, então |y −1 | = y −1 = |y|−1 e, se y < 0, então
|y −1 | = −(y −1 ) = (−y)−1 = |y|−1 . 

3.20 Exercício 20
Prove por indução que, dados x1 , . . . , xn num corpo ordenado K, tem-se |x1 + · · · + xn | 6
|x1 | + · · · + |xn | e |x1 · x2 · · · xn | = |x1 | · |x2 | · · · |xn |.

Solução Além de |x1 | 6 |x1 |, temos

|x1 + · · · + xn | 6 |x1 | + · · · + |xn | ⇒


⇒ |x1 + · · · + xn + xn+1 | 6 |x1 + · · · + xn | + |xn+1 | 6 |x1 | + · · · + |xn | + |xn+1 |,

pelo item (i) do Teorema 2, e

|x1 · · · · · xn | = |x1 | · · · · · |xn | ⇒


⇒ |x1 · · · · · xn · xn+1 | = |x1 · · · · · xn | · |xn+1 | = |x1 | · · · · · |xn | · |xn+1 |,

pelo item (ii) do Teorema 2. 

3.21 Exercício 21
Seja K um corpo ordenado. Exprima cada um dos conjuntos abaixo como reunião de intervalos:

a) o conjunto dos x∈K tais que |x − 3| + |x + 3| < 8;


b) idem |x2 − 2| 6 1;
c) |2x + 1| 6 1;
d) |x − 5| < |x + 1|;
e) (2x + 3)6 (x − 2) > 0.

Solução

item a) Temos:

x > 3 ⇒ |x − 3| + |x + 3| = (x − 3) + (x + 3) = 2x,

−3 6 x < 3 ⇒ |x − 3| + |x + 3| = −(x − 3) + (x + 3) = 6
e
x < −3 ⇒ |x − 3| + |x + 3| = −(x − 3) − (x + 3) = −2x.
Como 6 < 8, 2x < 8 ⇔ x < 4 e −2x < 8 ⇔ x > −4:

|x − 3| + |x + 3| < 8 ⇔ −4 < x < 4.

Logo, {x ∈ K; |x − 3| + |x + 3| < 8} = (−4, 4). 


78 CAPÍTULO 3. NÚMEROS REAIS

item b) Começando com o Teorema 1,


√ √
|x2 − 2| 6 1 ⇔ −1 6 x2 − 2 6 1 ⇔ 1 6 x2 6 3 ⇔ − 3 6 x 6 −1 ou 16x6 3.
√ √
Logo, {x ∈ K; |x2 − 2| 6 1} = [− 3, −1] ∪ [1, 3]. 

item c) |2x + 1| 6 1 ⇔ −1 6 2x + 1 6 1 ⇔ −2 6 2x 6 0 ⇔ −1 6 x 6 0, ou seja,


{x ∈ K; |2x + 1| 6 1} = [−1, 0]. 

item d) Geometricamente, um ponto x ∈ K pertence ao conjunto em questão se, e


somente se, x está mais próximo do 5 do que do -1. Tais pontos são aqueles que estão à direita
5−1
do = 2, o ponto médio dos segmentos de extremos -1 e 5. Assim, armamos que
2
{x ∈ K; |x − 5| < |x + 1|} = (2, +∞).

Isto pode ser demonstrado de modo semelhante ao que zemos no item a), considerando os
casos x > 5, −1 6 x < 5 e x < −1. 

item e) (2x + 3)6 é um número ao quadrado,


Como a saber, o quadrado de (2x + 3)3 , e,
6
portanto, (2x + 3) > 0, para todo x ∈ K , temos

(2x + 3)6 (x − 2) > 0 ⇔ x − 2 > 0 ⇔ x > 2.

Logo, {x ∈ K; (2x + 3)6 (x − 2) > 0} = [2, +∞). 

3.22 Exercício 22
Prove que, para todo x num corpo ordenado K, tem-se

|x − 1| + |x − 2| > 1.

|x − 1| + |x − 2| + |x − 3| > 2.

Solução

No elevador. Geometricamente, dado um ponto x ∈ K , a primeira desigualdade signica


que a soma de sua distância até o 1 com sua distância até o 2 é sempre maior do que ou igual
a 1. A distância do 1 até o 2 já é 1, então podemos desconar que o resultado desejado venha
disto, o que se traduz como |2 − 1| = 1. E de fato vem.
Usando o item (i) do Teorema 2 (a Desigualdade Triangular), temos

1 = |2 − 1|
= |2 + (x − x) − 1|
= |(2 − x) + (x − 1)|
6 |2 − x| + |x − 1|
= |x − 2| + |x − 1|,

para todo x ∈ K, o que prova a primeira desigualdade dada.


3.23. EXERCÍCIO 23 79

No elevador. Inuídos pela primeira desigualdade, podemos ser levados a provar a se-
gunda fazendo |x − 1| + |x − 2| + |x − 3| > 1 + |x − 3| > 1, o que é verdade, mas ainda é pouco.
O que nos leva a um caminho mais adequado é o resgate, não da primeira desigualdade, mas
do argumento por trás dela; dos três pontos, 1, 2 e 3, são o 1 e o 3 cuja distância um do outro
é 2.

Começando de modo semelhante ao que zemos na primeira desigualdade, temos

2 = |3 − 1|
6 |3 − x| + |x − 1|
6 |3 − x| + |x − 1| + |x − 2|
= |x − 1| + |x − 3| + |x − 2|,

para todo x ∈ K, pois |3 − 1| = |(3 − x) + (x − 1)| 6 |3 − x| + |x − 1|, pela Desigualdade


Triangular, e porque |x − 2| > 0. 

3.23 Exercício 23
Dados a, b , ε num corpo ordenado K, prove que

|a − b| < ε ⇒ |b| − ε < |a| < |b| + ε .

Conclua que |a − b| < ε ⇒ a < |b| + ε.

Solução. Pelo item (iii) do Teorema 2, ||a| − |b|| 6 |a − b|, de modo que

|a − b| < ε ⇒ ||a| − |b|| < ε,

tese esta equivalente a − ε < |a| − |b| < ε, de acordo com o Teorema 1. Assim,

||a| − |b|| < ε ⇔ |b| − ε < |a| < |b| + ε .

Portanto, |a − b| < ε ⇒ |b| − ε < |a| < |b| + ε. Em particular, |a − b| < ε ⇒ |a| < |b| + ε, o que
implica a < |b| + ε, uma vez que |a| > a. 

3.24 Exercício 24
Prove que, num corpo ordenado K, as seguintes armações são equivalentes:

(i) K é arquimediano;

(ii) Z é ilimitado superior e inferiormente;

(iii) Q é ilimitado superior e inferiormente.


80 CAPÍTULO 3. NÚMEROS REAIS

Solução. Uma aplicação do Teorema 3, junto com o fato de N estar contido e ser ilimitado
superiormente em cada um dos Z e Q nos dá:

K é arquimediano ⇔ N é ilimitado superiormente

⇔ Z é ilimitado superiormente

⇔ Q é ilimitado superiormente.

N é ilimitado superiormente em K é equivalente a dizer que o conjunto − N =


Dizer que
{−n ∈ K; n ∈ N} é ilimitado inferiormente em K . Assim, a parte que falta é análoga ao que
zemos acima: − N ⊂ Z, − N ⊂ Q e − N é ilimitado inferiormente em Z e em Q:

K é arquimediano ⇔ − N é ilimitado inferiormente


⇔ Z é ilimitado inferiormente
⇔ Q é ilimitado inferiormente. 

3.25 Exercício 25
Prove que um corpo ordenado K é arquimediano se, e somente se, para todo ε > 0 em K , existe
1
n∈N tal que < ε.
2n

Solução. Notemos a semelhança deste exercício com o item iii) do Teorema 3, o qual diz:

1
K é arquimediano ⇔ ∀a ∈ K, a > 0, ∃n ∈ N tal que 0< < a.
n

Nível 1.1 (ida). Suponha K arquimediano e seja ε ∈ K positivo. Seja n ∈ N tal que
1 1 1
0 < < ε. Armamos que existe m∈N tal que 0 < m < (a demonstração disto está no
n 2 n
1
Nível 2). Para um tal m temos < ε, como queremos.
2m

1
Nível 1.2 (volta). Dado ε∈K positivo, seja n∈N tal que < ε. O número 2n é um
2n
número natural, portanto K é arquimediano.

1 1
No elevador. A desigualdade
m
< é equivalente à desigualdade 2m > n. Assim, dado
m
2 n
o n, queremos um m tal que 2 > n. Observamos que não podemos aplicar o Exercício 14
n
(página 74) aqui. Tal exercício nos diz apenas que a função denida por f (n) = 2 é crescente;
isto não signica que ela seja ilimitada. Mas já vimos que Bernoulli é uma boa ferramenta para
mostrar que certas coisas não são limitadas.

Nível 2. Por Bernoulli, xado m, temos 2m = (1 + 1)m > 1 + m > m. Portanto, xado
n
n, temos 2 > n. Logo, basta tomarmos m = n. 
3.26. EXERCÍCIO 26 81

3.26 Exercício 26
Seja a > 1 num corpo arquimediano K . Considere a função f : Z → K , denida por f (n) = an .
Prove as seguintes armações:

(i) f (Z) não é limitado superiormente;

(ii) inf f (Z) = 0.

(Voltar para a Solução do Exercício 43: página 92.)

(Voltar para a Solução do Exercício 12 do Capítulo VI: página 216.)

Solução. Dado b ∈ K, queremos encontrar um n∈N tal que an > b. Por Bernoulli, como
a − 1 > 0 > −1, temos

an = (1 + (a − 1))n > 1 + n(a − 1) = na + 1 − n > na.


Como K é arquimediano, sabemos que existe tal quen∈N na > b. Para um tal n temos,
n
pela desigualdade acima, o resultado desejado do item (i), a > b.
1
Se an > b e b é positivo, então 0 < a−n <
. Isto vai nos ajudar a concluir que inf f [Z] = 0.
b
1
Dado ε ∈ K positivo, seja n ∈ N tal que 0 < < ε. Como f [Z] não é limitado, seja m ∈ Z tal
n
1
que a
m
> n. Portanto: 0 < a−m < < ε e, em particular, 0 < a−m < ε, sendo a−m ∈ f [Z].
n
Disto, e de 0 < x, para todo x ∈ f [Z], segue inf f [Z] = 0. 

3.27 Exercício 27
Sejam a racional diferente de zero, x irracional. Prove que ax e a + x são irracionais. Dê
exemplos de dois números irracionais x, y tais que x + y e x · y são racionais.

(Voltar para a Solução do Exercício 30: página 83.)

(Voltar para a Solução do Exercício 55: página 102.)

(Voltar para a Solução do Exercício 58, Nível 3.2 do item iii): página 112.)

(Voltar para a Solução do Exercício 16 do Capítulo V: página 167.)

Solução. Este exercício segue do fato de Q ser um corpo. Temos apenas duas possibilidades
para ax: ou ax é racional, ou ax é irracional. Se ax fosse racional, escrevamos ax = r, com
r ∈ Q. Assim, a−1 r ∈ Q também, de modo que x ∈ Q, pois x = a−1 r. Logo, como x é
irracional, segue ax ∈ R − Q.
Para o a + x é análogo: a + x ∈ Q ⇒ x = (a + x) − a ∈ Q. Logo, como x é irracional, segue
a + x ∈ R − Q.
Os exemplos mais simples de números irracionais x e y tais que x + y e xy são racionais
√ √
ocorrem para x = 2 e y = − 2. Para eles temos x + y = 0 e xy = −2. 

3.28 Exercício 28
√ √
Sejam a, b , c e d números racionais. Prove que a+b 2=c+d 2⇔a=c e b = d.
(Voltar para Observações do item iii) do Exercício 58: página 113.)
82 CAPÍTULO 3. NÚMEROS REAIS

√ √
Solução. Se a=c e b = d, então é imediato que a + b 2 = c + d 2. Para a ida, temos:
√ √ √
a + b 2 = c + d 2 ⇒ (a − c) + (b − d) 2 = 0.

Assim, para concluir que a−c=0 e b − d = 0, basta mostrar que x+y 2=0⇒x=0 e
y = 0, x e y são racionais.
se
√ √ x √
Se y 6= 0, então x + y 2 = 0 ⇒ 2 = − e, então, 2 seria um número racional. Portanto
√ y
y = 0. Para y = 0: x + y 2 = 0 ⇒ x = 0. Segue a tese. 

3.29 Exercício 29

Prove que o conjunto K dos números reais da forma a + b 2, com a e b racionais, é um corpo
relativamente às operações de adição e multiplicação de números reais. Examine se o mesmo

ocorre com os números da forma a + b 3 2, com a, b ∈ Q.

Solução. O 0 é um número da forma a + b 2 para a e b racionais, basta tomar a = b = 0. As
associatividades e as comutatividades das adição e multiplicação, assim como a distributividade,
√ √
são imediatas. O simétrico de um número
√ a+b 2 é (−a) + (−b) 2 e a unidade é a mesma de
K: 1 = 1 + 0 · 2. Só falta vermos se o inverso está lá.

√ 1
Conversando. O inverso de a+b 2 é √ (o inverso dele nesse corpo só pode ser
a+b 2 √
seu mesmo inverso em K ). Agora, tal número é da forma x+y 2 para x e y racionais? Aqui
sim a técnica da racionalização será útil. De modo geral não há problema algum em termos
uma raiz no denominador, mas aqui, para os nossos própositos, tiremos ela de lá.


Voltando à solução. Dado um a+b 2 6= 0, multiplicando o numerador e o denominador
1 √ √
de √ por a − b 2 (a − b 2 6= 0 pelo exercício anterior), temos:
a+b 2
√ √
1 1 a−b 2 a−b 2 a b √
√ = √ · √ = 2 = + · 2,
a+b 2 a+b 2 a−b 2 a − 2b2 a2 − 2b2 2b2 − a2
a b √
sendo os números
2 2
e
2 2
· 2 ambos racionais. Logo, K é um corpo com as
a − 2b 2b − a
operações de adição e multiplicação usuais.
√3
O mesmo já não acontece com o conjunto dos números da forma a+b 2, com a e b racionais.

3

Por exemplo, o inverso multiplicativo do número 2=0+1· 32 é
√ √
2 √
3 3
1 1 22 3
√3
= √3
· √
3
= · 2,
2 2 22 2

3
que não é um número da forma a + b 2, com a e b racionais, pelo mesmo motivo do Exercício

3
√ 3
anterior: dados a, b, c, d ∈ Q, vale a + b 2 = c + d 2 se, e somente se, a = c e b = d. 

3.30 Exercício 30
√ √ √
Sejam
√ a, b números racionais positivos. Prove que a+√ b é√racional se, e somente se, a e
b forem ambos racionais. ( Sugestão: multiplique por a − b.)
3.31. EXERCÍCIO 31 83

√ √ √ √
Solução. A volta é imediata: se a e b são racionais, então sua soma a+ b também
é racional.
Para a ida vamos ver o que acontece se aplicarmos a sugestão dada:

√ √ √ √
( a + b) · ( a − b) = a − b.
√ √
Assim, tal produto é um número racional. Então, se
√ √ a+ b é racional, pelo Exercício 27
(página 81), podemos concluir que a− b é racional também. Agora, a soma e a diferença
desses números também são racionais:

√ √ √ √ √
( a + b) + ( a − b) = 2 a,
√ √ √ √ √
( a + b) − ( a − b) = 2 b.
√ √
Outra aplicação do Exercício 27 nos permite concluir o resultado desejado, a e b são racio-
nais, pois 2 é racional. 

3.31 Exercício 31
Sejam X ⊂R não-vazio, limitado superiormente, e c um número real. Tem-se c 6 sup X se,
e somente se, para cada ε>0 real dado pode-se achar x∈X tal que c − ε < x. Enuncie e
demonstre um resultado análogo com inf em vez de sup.
(Voltar para a Solução do Exercício 20 do Capítulo IX: página 375.)

Solução 1. Dado ε > 0, se c 6 sup X , então c − ε < sup X , de modo que, pela condição S2',
existe x ∈ X tal que c − ε < x. Reciprocamente, se c > sup X (considerando a contrapositiva),
para ε = c − sup X , não existe x ∈ X tal que x > c − ε, pois, neste caso, c − ε = sup X > x
para todo x ∈ X .
Com respeito ao ínmo, armamos o seguinte: dados Y ⊂ R não vazio, limitado inferior-
mente, e c ∈ R, vale c > inf Y se, e somente se, para cada real positivo ε existe y ∈ Y tal que
y < c + ε. Isto pode ser demonstrado de modo análogo ao que zemos no parágrafo acima.
Dado ε > 0, se c > inf Y , então existe y ∈ Y tal que y < c + ε, porque c + ε > inf Y (condição
I2'). E, se c < inf Y , então, para ε = inf Y − c, não existe y ∈ Y tal que y < c + ε, porque
c + ε = inf Y 6 y para todo y ∈ Y . 

Solução 2 (para a volta). ε > 0 existe x ∈ X tal que c − ε < x, então, de


Se para todo
sup X > x para todo x ∈ X , segue sup X > c − ε para todo ε > 0. Isto implica sup X > c, pela
condição S2, pois c = sup{c − ε ∈ R; ε > 0}. Analogamente, se Y é um conjunto de números
reais não vazio e limitado inferiormente e, se para todo ε > 0, existe y ∈ Y tal que y < c + ε,
então inf Y < c + ε para todo ε > 0 e, portanto, inf Y 6 c = inf{c + ε ∈ R; ε > 0} (condição
I2). 

3.32 Exercício 32
 
1
Seja X= ;n∈N . Prove que inf X = 0.
n
84 CAPÍTULO 3. NÚMEROS REAIS

Solução. Isto é verdade apenas em corpos arquimedianos. Temos n>0 para todo n ∈ N, o
que implica 0 < 1/n (Exercício 13, página 73). E, se K é arquimediano, dado x∈K positivo,
seja n ∈ N tal que n > 1/x, de modo que 1/n < x (Exercício 13). Logo, inf X = 0. 

Observação. Em corpos não arquimedianos este exercício é falso: se K não é arquimediano,


pelo Teorema 3, seja b ∈ K tal que b > n para todo n ∈ N 1/b 6 1/n para todo
e, então,
n ∈ N, onde 1/b > 0, uma vez que b é necessariamente positivo. Concluímos que X não admite
1
ínmo em K se K não é arquimediano (seria inf X = se sup N ∈ K , mas, no Exemplo
sup N
14, vimos que, se K não é arquimediano, então N não admite supremo em K ).

3.33 Exercício 33
Sejam A ⊂ B conjuntos não-vazios limitados de números reais. Prove que inf B 6 inf A 6
sup A 6 sup B .
(Voltar para a Solução do Exercício 38: página 87.)

(Voltar para a Solução do Exercício 40, item iii): página 90.)

(Voltar para a Solução do Exercício 49: página 95.)

(Voltar para a Solução do Exercício 55, Nível 2.3: página 106.)

(Voltar para a Solução do Exercício 21 do Capítulo VI: página 224.)

(Voltar para a Solução do Exercício 20 do Capítulo IX: página 375.)

(Voltar para a Solução do Exercício 32 do Capítulo IX, item 1: página 390.)

Solução. Pela condição I1, temos inf B 6 x, para todo x ∈ B , donde, por ser A ⊂ B , inf B 6
x, para todo x ∈ A e, portanto, pela condição I2, inf B 6 inf A. A segunda desigualdade,
inf A 6 sup A, vale porque existe x ∈ A tal que inf A 6 x 6 sup A (qualquer x ∈ A satisfaz
estas desigualdades, pelas condições I1 e S1). A última desigualdade vale por motivo análogo
ao da primeira:

sup B > x ∀x ∈ B ⇒ sup B > x ∀x ∈ A ⇒ sup B > sup A,

respectivamente, pela condição S1, pela inclusão A⊂B e pela condição S2. 

3.34 Exercício 34
Sejam A, B conjuntos não-vazios de números reais, tais que x ∈ A, y ∈ B ⇒ x 6 y . Prove que
sup A 6 inf B . Prove que sup A = inf B se, e somente se, para todo ε > 0 dado, podem-se
obter x ∈ A e y ∈ B tais que y − x < ε.

Solução. Fixado um y0 ∈ B (B é não vazio), temos

x 6 y0 ∀x ∈ A ⇒ sup A 6 y0 .

Como o y0 de B era qualquer, podemos concluir que sup A 6 y ∀y ∈ B . Portanto, sup A 6


inf B , o que conclui a primeira parte do exercício.
3.35. EXERCÍCIO 35 85

ε
Suponha sup A = inf B := a. Dado ε > 0, existem x∈A e y ∈B tais que a− <x e
2
ε
y <a+ . y − x < ε.
Portanto
2
Para a recíproca, se inf A < sup B , seja ε = sup B − inf A > 0. Como x 6 inf A e y > sup B
para todo x ∈ A e para todo y ∈ B , segue y − x > sup B − inf A = ε quaisquer que sejam o x
em A e o y em B (contrapositiva). Segue a tese. 

3.35 Exercício 35
Dado A ⊂ R não-vazio, limitado inferiormente, seja −A = {−x; x ∈ A}. Prove que −A é
limitado superiormente e que sup(−A) = − inf A.
(Voltar para a Solução do Exercício 21 do Capítulo VI: página 224.)

Solução. Sendo A⊂R não vazio e limitado inferiormente, seja a seu ínmo. Temos

a 6 x ∀x ∈ A ⇒ −a > −x ∀x ∈ A ⇒ −a > x ∀x ∈ −A,

portanto, −a é uma cota superior para −A, o que mostra também que −a satisfaz a condição
S1 para ser o supremo de −A. Só talta então mostrar que −a também satisfaz a condição S2;
dado c ∈ R:

c > x ∀x ∈ −A ⇒ −c 6 −x ∀x ∈ −A
⇒ −c 6 x ∀x ∈ A
⇒ −c 6 a
⇒ c > −a,

ou seja, −a também satisfaz a condição S2. Logo, − inf A = sup(−A). 

3.36 Exercício 36
Seja A⊂R não-vazio, limitado. Dado c > 0, seja c · A = {c · x; x ∈ A}. Prove que c · A é
limitado e que sup(c · A) = c · sup A, inf(c · A) = c · inf A. Enuncie e demonstre o que ocorre
quando c < 0.

Solução. Sejam a = inf A e b = sup B . Temos

a 6 x 6 b ∀x ∈ A ⇒ ca 6 cx 6 cb ∀x ∈ A ⇒ ca 6 x 6 cb ∀x ∈ c · A,

ou seja, ca e cb limitam c·A por baixo e por cima, respectivamente. Além disso, dado d ∈ R,
temos

d 6 x ∀x ∈ c · A ⇒ d 6 cx ∀x ∈ A
d
⇒ 6 x ∀x ∈ A
c
d
⇒ 6a
c
⇒ d 6 ca
86 CAPÍTULO 3. NÚMEROS REAIS

e, analogamente,

d > x ∀x ∈ c · A ⇒ d > cx ∀x ∈ A
d
⇒ > x ∀x ∈ A
c
d
⇒ >b
c
⇒ d > cb.
Estas implicações, junto com as desigualdades ca 6 x 6 cb, para todo x ∈ c · A, signicam que
ca = inf(c · A) e cb = sup(c · A), isto é,

inf(c · A) = c · inf(A) e sup(c · A) = c · sup(A),


como queríamos para a primeira parte deste exercício.
Para a segunda parte, sejam c < 0 e c · A = {cx; x ∈ A}. Retomando o exercício anterior,
lá tínhamos c = −1 e −A = −1 · A. Assim, se c < 0, armamos que sup(c · A) = c · inf A e
inf(c · A) = c · sup A. Vamos demonstrar isso usando o resultado que acabamos de provar e o
exercício anterior, observando que −c > 0 e c · A = −(−c · A). Mais precisamente, é apenas
na primeira igualdade dessa armação que poderemos proceder desse modo. Para a segunda, o
que precisaremos, na verdade, é de um resultado análogo ao do exercício anterior, a saber: se X
é um subconjunto não vazio de R limitado superiormente, então −X é limitado inferiormente
e inf(−X) = − sup X ; de fato:

sup X > x ∀x ∈ X ⇒ − sup X 6 −x ∀x ∈ X ⇒ − sup X 6 x ∀x ∈ −X


e

d 6 x ∀x ∈ −X ⇒ d 6 −x ∀x ∈ X ⇒ −d > x ∀x ∈ X ⇒ −d > sup X ⇒ d 6 − sup X,


portanto, − sup X possui tudo o que é preciso para ser o ínmo do conjunto −X .
Pelo Exercício 35 e por tudo que zemos acima, temos: c·A limitado superior e inferiormente,
porque −c · A é limitado inferior e superiormente (respectivamente);

sup(c · A) = sup(−(−c · A)) = − inf(−c · A) = −(−c) · inf A = c · inf A,


e;
inf(c · A) = inf(−(−c · A)) = − sup(−c · A) = −(−c) · sup A = c · sup A,
como queríamos. 

3.37 Exercício 37
Dados A, B ⊂ R não-vazios e limitados, seja A + B = {x + y; x ∈ A, y ∈ B}. Prove:

i) A+B é limitado;

ii) sup(A + B) = sup A + sup B ;


iii) inf(A + B) = inf A + inf B ;
iv) Enuncie e demonstre resultados análogos supondo apenas AeB limitados superiormente
(ou A e B limitados inferiormente).

(Voltar para a Solução do Exercício 55, Nível 1.1: página 102.)


3.38. EXERCÍCIO 38 87

Solução.

itens i), ii) e iii) inf A 6 x 6 sup A, para todo x ∈ A, e inf B 6 y 6 sup B , para
De
todo y ∈ B , vem inf A + inf B 6 x + y 6 sup A + sup B , para todo x ∈ A e para todo y ∈ B .
Portanto, A + B é limitado: por baixo, por inf A + inf B e, por cima, por sup A + sup B . Além
disso,

c 6 x + y 6 d ∀x ∈ A ∀y ∈ B ⇒ c − y 6 x 6 d − y ∀x ∈ A ∀y ∈ B
⇒ c − y 6 inf A e sup A 6 d − y ∀y ∈ B
⇒ c − inf A 6 y 6 d − sup A ∀y ∈ B
⇒ c − inf A 6 inf B e sup B 6 d − sup A
⇒ c 6 inf A + inf B e sup A + sup B 6 d.

Logo, inf A + inf B = inf(A + B) e sup A + sup B = sup(A + B).

item iv) A e B são não vazios e limitados superiormente, então A + B é limitado


Se
superiormente e sup(A+B) = sup A+sup B . Se A e B são não vazios e limitados inferiormente,
então A + B é limitado inferiormente e inf(A + B) = inf A + inf B . As demonstrações destas
duas implicações já estão contidas no parágrafo acima. 

3.38 Exercício 38
Seja X ⊂ R. Uma função f :X→R chama-se limitada quando sua imagem f (X) ⊂ R é um
conjunto limitado. Neste caso dene-se o sup f como o supremo do conjunto f (X). (Às vezes
se escreve sup f (x) ou sup f )
x∈X X

i) Prove que a soma de duas funções limitadas f, g : X → R é uma função limitada f +g :


X → R.

ii) Mostre que (f + g)(X) ⊂ f (X) + g(X), na notação do Exercício 37.

iii) Conclua que sup(f + g) 6 sup f + sup g e inf(f + g) > inf f + inf g .

iv) Considerando as funções f, g : [−1, +1] → R, denidas por f (x) = x e g(x) = −x, mostre
que se pode ter sup(f + g) < sup f + sup g e inf(f + g) > inf + inf g .

(Voltar para a Solução do Exercício 21 do Capítulo VI: página 224.)

(Voltar para a Observação do Exercício 20 do Capítulo VII: página 245.)

Solução.

itens i), ii) e iii) (f + g)[X] ⊂ f [X] + g[X]. Os itens i)


Primeiramente vejamos que
e iii) decorrem disso. Se y ∈ (f + g)[X], então, para x ∈ X tal que y = (f + g)(x), temos
y = f (x) + g(x) ∈ f [X] + g[X]. Logo, (f + g)[X] ⊂ f [X] + g[X]. Daí, se f e g são limitadas,
então (f +g)[X] também é limitado; f [X]+g[X] é limitada, pelo Exercício anterior, e (f +g)[X]
88 CAPÍTULO 3. NÚMEROS REAIS

é, portanto, subconjunto de um conjunto limitado. Claramente (f + g)[X] é não vazio e, então,


da inclusão (f + g)[X] ⊂ f [X] + g[X], temos também

inf(f [X] + g[X]) 6 inf((f + g)[X]) 6 sup((f + g)[X]) 6 sup(f [X] + g[X]),

pelo Exercício 33 (página 84), onde, pelo exercício anterior,

inf(f [X] + g[X]) = inf(f [X]) + inf(g[X])

e
sup(f [X] + g[X]) = sup(f [X]) + sup(g[X]).
Logo,
inf f + inf g 6 inf(f + g) 6 sup(f + g) 6 sup f + sup g.

item iv) Por um lado, temos(f + g)(x) = 0, para todo x ∈ [−1, 1] e, então, sup(f +
g) = 0 = inf(f + g). Por outro, temos sup f = 1 = sup g e inf f = −1 = inf g . Logo,
inf f + inf g < inf(f + g) e sup(f + g) < sup f + sup g , porque −2 < 0 < 2. 

3.39 Exercício 39
Sejam A, B conjuntos de números reais positivos. Denamos A · B = {x · y; x ∈ A e y ∈ B}.
Prove que se A e B forem limitados então A·B é limitado, sendo sup(A · B) = sup A · sup B
e inf(A · B) = inf A · inf B .
(Voltar para a Solução do Exercício 40, item iii): página 90.)

(Voltar para a Solução do Exercício 55: página 103.)

(Voltar para a Solução do Exercício 21 do Capítulo VI: página 224.)

Solução. Considerando os supremos de A e de B , 0 < x 6 sup A, para todo x ∈ A, e


0 < y 6 sup B , para todo y ∈ B , implicam 0 < xy 6 sup A · sup B , para todo x ∈ A e todo
y ∈ B . Portanto, 0 e sup A · sup B são, respectivamente, uma cota inferior e uma cota superior
de A · B . Assim, inf(A · B) > 0 e sup(A · B) 6 sup A · sup B . Por outro lado, dado c ∈ R, como
os elementos de A e de B são todos positivos (bem como seus supremos),

c
c > xy ∀x ∈ A ∀y ∈ B ⇒ > x ∀x ∈ A ∀y ∈ B
y
c
⇒ > sup A ∀y ∈ B
y
c
⇒ > y ∀y ∈ B
sup A
c
⇒ > sup B
sup A
⇒ c > sup A · sup B.

Logo, sup(A · B) = sup A · sup B . Ao fazer, contas análogas para o ínmo, o cuidado que
devemos tomar é que agora pode ser inf A = 0;
c c
c 6 xy ∀x ∈ A ∀y ∈ B ⇒ 6 x ∀x ∈ A ∀y ∈ B ⇒ 6 inf A ∀y ∈ B,
y y
3.40. EXERCÍCIO 40 89

c c
daí, seinf A > 0, então 6 y , para todo y ∈ B e, portanto, 6 inf B e c 6 inf A·inf B .
inf A inf A
c
Se inf A = 0, então 6 inf A = 0, para todo y ∈ B , implica c 6 0 = inf A · inf B (lembrando
y
que y > 0). De qualquer forma (inf A > 0, porque 0 é uma cota inferior para A), c 6 z , para
todo z ∈ A · B , implica c 6 inf A · B . Além do que, inf A · inf B é uma cota inferior de A · B .
Logo, inf(A · B) = inf A · inf B . 

3.40 Exercício 40
i) Prove que o produto de duas funções limitadas f, g : X → R é uma função limitada
f · g : X → R.

ii) Mostre que (f · g)(X) ⊂ f (X) · g(X).

iii) Conclua que, se f e g forem ambas positivas, tem-se sup(f ·g) 6 sup f · sup g e inf(f ·g) >
inf f · inf g .

iv) Dê exemplo em que valham as desigualdades estritas.

2
v) Mostre porém que para toda f positiva tem-se sup(f ) = [sup f ]2 .

Solução.

item i) Sejam a, b, c, d reais tais que a 6 f (x) 6 b e c 6 g(x) 6 d ∀x ∈ X . Vamos


querer multiplicar essas desigualdades. Como o sinal delas pode mudar dependendo do número
multiplicado ser positivo ou não, temos três casos a considerar, a saber, quando ambos f (x) e
g(x) forem positivos, ambos forem negativos e quando um deles for positivo e outro negativo:


0 6 f (x) 6 b
⇒ 0 6 f (x)g(x) 6 bd;
0 6 g(x) 6 d
 
a 6 f (x) 6 0 0 6 −f (x) 6 −a
⇒ ⇒ 0 6 f (x)g(x) 6 ac;
c 6 g(x) 6 0 0 6 −g(x) 6 −c

 
a 6 f (x) 6 0 0 6 −f (x) 6 −a
⇒ ⇒ 0 6 −f (x)g(x) 6 −ad ⇒ ad 6 f (x)g(x) 6 0;
0 6 g(x) 6 d 0 6 g(x) 6 d

 
0 6 f (x) 6 b 0 6 f (x) 6 b
⇒ ⇒ 0 6 −f (x)g(x) 6 −bc ⇒ bc 6 f (x)g(x) 6 0.
c 6 g(x) 6 0 0 6 −g(x) 6 −c

Portanto: min{0, ad, bc} 6 f (x)g(x) 6 max{0, ac, bd}, para todo x ∈ X. 

item ii) Se y ∈ (f · g)[X], para x∈X tal que y = (f · g)(x), temos y = f (x) · g(x) ∈
f [X] · g[X]. 
90 CAPÍTULO 3. NÚMEROS REAIS

item iii) Combinando a inclusão do item anterior com os Exercícios 39 (página 88) e 33
(página 84), temos

   
sup(f · g) = sup (f · g)[X] 6 sup f [X] · g[X] = sup f [X] · sup g[X] = sup f · sup g

e    
inf(f · g) = inf (f · g)[X] > inf f [X] · g[X] = inf f [X] · inf g[X] = inf f · inf g.

Logo, sup(f · g) 6 sup f · sup g e inf(f · g) > inf f · inf g , como queríamos. 

item iv) Para as funções f, g : [−1, 1] → R, denidas por f (x) = x + 2 e g(x) = −x + 2


(ambas positivas), temos: inf f = inf g = 1, sup f = sup g =3 e(f · g)(x) = (2 + x)(2 − x) =
4 − x2 , cujo ínmo é 3 e cujo supremo é 4. Portanto:

inf f · inf g = 1 < 3 = inf(f · g) < 4 = sup(f · g) < 9 = sup f · sup g,

ou seja, inf f · inf g < inf(f · g) < sup(f · g) < sup f · sup g . 

2
item v) Pelo item iii) temos: sup(f ) = sup(f · f ) 6 sup f · sup f = [sup f ]2 . Vamos
2 2
mostrar então que [sup f ] 6 sup(f ). Como f (x) > 0 ∀x ∈ X , sup f > 0 também, e podemos
fazer as contas abaixo:

p
f 2 (x) 6 sup f 2 ⇒ f (x) 6 sup f 2 ∀x ∈ X
p
⇒ sup f 6 sup f 2
⇒ [sup f ]2 6 sup f 2 ,

como queríamos. 

3.41 Exercício 41
Analise os Exercícios 39 e 40 sem as hipóteses de positividade neles feitas.

Solução. Consideremos, por exemplo, o conjunto A = {−1, 0}. Para ele, temos A · A = {0, 1}
e, então,

inf(A · A) = 0 6= inf A · inf A = 1 e sup(A · A) = 1 6= sup A · sup A = 0,

ou seja, não vale a conclusão do Exercício 39. Esse conjunto também pode ser aproveitado para
analisarmos o Exercício 40, considerando a função f : {0, 1} → R denida por f (0) = −1 e
f (1) = 0, o que nos dá: (f · f )(0) = 1, (f · f )(1) = 0,

inf(f · f ) = 0 < inf f · inf f = 1 e sup(f · f ) = 1 > sup f · sup f = 0.

Portanto não vale nenhuma das desigualdades do item iii) do Exercício 40, e nem a igualdade
de seu item v). 
3.42. EXERCÍCIO 42 91

3.42 Exercício 42
Seja f (x) = a0 + a1 x + · · · + an xn um polinômio com coecientes inteiros

 
p p
i) Se um número racional (com p e q primos entre si) é tal que f = 0, prove que p
q q
divide a0 e q divide an .

ii) Conclua que, quando an = 1, as raízes reais de f são inteiras ou irracionais. Em particular,
n
examinando x − a = 0, conclua que, se um número inteiro a > 0 não possui n-ésima raiz

inteira, então
n
a é irracional.

√ √
3
iii) Use o resultado geral para provar que 2+ 2 é irracional.

Solução.

item i) Multiplicando ambos os membros de

p pn−1 pn
a0 + a1 · + · · · + an−1 · n−1 + an · n = 0
q q q

por qn, obtemos

a0 q n + a1 pq n−1 + · · · + an−1 pn−1 q + an pn = 0,

de onde vem

an pn = −q(a0 q n−1 + a1 pq n−2 + · · · + an−1 pn−1 )

a0 q n = −p(a1 q n−1 + · · · + an−1 pn−2 q + an pn−1 ).

Como a0 q n−1 +a1 pq n−2 +· · ·+an−1 pn−1 e a1 q n−1 +· · ·+an−1 pn−2 q +an pn−1 são números inteiros,
n n
estas duas últimas igualdades signicam que q divide an p e p divide a0 q . Com respeito a isso,
como p e q são primos entre si, há um resultado de Álgebra que diz: dados os números inteiros
a, b e c, se a divide bc e se a e b são primos entre si, então a divide c.1 É por este resultado que
podemos concluir que q divide an e p divide a0 . 

item ii) Seja x uma raiz real de f. Então x ∈ Q ou x ∈ R − Q. Se x ∈ Q, seja x = p/q ,


com peq primos entre si. Então, pelo item anterior, q divide an = 1 e, portanto, q = ±1. Logo
x ∈ Z.

Supondo
√ a > 0, o número n
a é uma raiz do polinômio xn − a. Portanto, pelo√que acabamos
de ver,
n
a é inteiro ou irracional. Se a não possui n-ésima raiz inteira, então
n
a só pode ser
irracional. 
1 Ainda sendo a, b, c ∈ Z, deste resultado decorre que se a é primo e se a divide bc, então a divide b ou a
divide c  o que foi utilizado no Exercício 7 do Capítulo II (página 36) e no Exercício 7 do Capítulo III (página
70) , pois se a é primo e se a não divide b, então mdc(a, b) = 1.
92 CAPÍTULO 3. NÚMEROS REAIS

item iii) Precisamos encontrar um polinômio de coecientes inteiros e com coeciente


√ √
3
dominante igual um do qual o número 2+ 2 seja raiz. Vamos realizar operações que
eliminem a raiz quadrada e a raiz cúbica do número dado. Temos

√ √ √
(X − 2)3 = 2
3
X= 2+ 2 ⇒
√ √
⇒ X 3 − 3X 2 2 + 6X − 2 2 − 2 = 0

⇒ X 3 − 2(3X 2 + 2) + 6X − 2 = 0

⇒ [ 2(3X 2 + 2)]2 = (X 3 + 6X − 2)2
⇒ 18X 4 + 24X 2 + 8 = X 6 + 36X 2 − 4 + 12X 4 − 4X 3 − 24X
⇒ X 6 − 6X 4 − 4X 3 + 12X 2 − 24X − 12 = 0.
√ √
Essas contas signicam que o número 2+ 3 2 é raiz do polinômio x6 − 6x4√
− 4x3 √
+ 12x2 −
24x − 12. Todos seus coecientes são números inteiros,
√ √ an = 1 (com n = 6) e 2 + 3 2 não é
3
um inteiro. Portanto, pelo item ii), 2 + 2 ∈ R − Q. 

3.43 Exercício 43
m
Dado um número natural p > 1, prove que os números racionais da forma , onde m∈Z e
pn
n∈N constituem um subconjunto denso em R.

(Voltar para o Nível 1 da Solução do Exercício 17 do Capítulo V: página 168.)

m
Solução. Dados a e b reais com a < b, queremos encontrar m ∈ Z e n ∈ N tais que a < < b.
pn
A ideia aqui é a mesma usada na demonstração do Teorema 4.

Pelo Exercício 26 (página 81), como p > 1, o conjunto {pn ∈ R; n ∈ N} não é limitado
1
superiormente. Podemos tomar então um n ∈ N tal que pn > . Para este n temos
 b−a
1 m
0 < n < b − a. Consideramos agora o conjunto ∈ R; m ∈ Z . Este conjunto não é
p pn
limitado nem superiormente nem inferiormente.
m m
Se a > 0, seja m o menor inteiro tal que > a. Armamos que é menor do que b.
pn pn
m 1 1
Pois, se fosse
n
> b, então, uma vez que n < b − a ⇒ − n > a − b, somando, teríamos
p p p
m 1 m−1 m
n
− n = n
> b + (a − b) = a, contrariando a minimalidade de m. Portanto a < n < b.
p p p p
m m
Se b < 0, seja m o maior inteiro tal que < b. Armamos que n é maior do que a.
pn p
m 1 m 1 m+1
Pois, se fosse
n
6 a, então, uma vez que n < b − a, somando teríamos n + n = <
p p p p pn
m
a + (b − a) = b, contrariando a maximalidade de m. Portanto a < n < b.
p
Se a = 0, então m = 1 serve. Se b = 0, então m = −1 serve. De qualquer forma
demonstramos que tais números são densos na reta. 

(Voltar para a Solução do Exercício 58, item i): página 110.)


3.44. EXERCÍCIO 44 93

3.44 Exercício 44
Um número real r chama-se algébrico quando existe um polinômio f (x) = a0 + a1 x + · · · + an xn ,
não identicamente nulo, com coecientes inteiros, tal que f (r) = 0.
i) Prove que o conjunto dos polinômios de coecientes inteiros é enumerável.

ii) Dada uma enumeração {f1 , f2 , . . . , fn , . . .} desses polinômios não identicamente nulos,
seja, para cada n ∈ N, An o conjunto das raízes reais de fn . Cada An é um conjunto nito
[
(podendo ser vazio). O conjunto A dos números algébricos reais escreve-se A= An .
n∈N
Conclua que A é enumerável. Mostre que A é denso em R.
(Voltar para a Solução do Exercício 28 do Capítulo V: página 177.)

(Voltar para a Solução do Exercício 39 do Capítulo V: página 182.)

(Voltar para o item (d) da Solução do Exercício 21 do Capítulo VII: página 248.)

Solução.

item i) Podemos identicar cada polinômio com seus coecientes respeitando a ordem
n−1
destes. Assim, a um dado polinômio a1 +a2 x+· · ·+an x associamos a sequência (a1 , . . . , an ) ∈
n
Z , se n > 1, e, se n = 1, associamos o polinômio a ao número a ∈ Z. Esta associação é sobre
[
Zn , um conjunto enumerável (Teorema 10 e seu Corolário 2, Capítulo II). Logo, o conjunto
n∈N
dos polinômios de coecientes inteiros é enumerável (Teorema 9 do Capítulo II). 

item ii) O conjunto A é enumerável porque é uma reunião enumerável de conjuntos enu-
meráveis (Corolário 2 do Teorema 10 do Capítulo II). Ele é denso em R porque todo número
racional é algébrico, uma vez que o número p/q , com p e q inteiros e q 6= 0, é raiz do polinômio
qx − p, de coecientes inteiros 

3.45 Exercício 45
Seja X o complementar de um conjunto enumerável de números reais. Mostre que, para cada
intervalo aberto (a, b), a interseção (a, b) ∩ X é não-enumerável. Em particular, X é denso.

(Voltar para a Solução do Exercício 39 do Capítulo V: página 182.)

Solução. Seja N = R −X o conjunto enumerável do qual X é complementar. Escrevamos


I = (a, b). I = (I ∩ N ) ∪ (I ∩ X). Por um lado, sabemos que I é não
Vale a igualdade
enumerável. Por outro, I ∩ N é enumerável. Assim, se I ∩ X fosse enumerável, então I também
seria enumerável. Logo, I ∩ X é não enumerável. Como isso vale para qualquer intervalo I ,
podemos concluir que X é denso em R. 

3.46 Exercício 46
Um número real chama-se transcendente quando não é algébrico. Prove que o conjunto dos
números transcendentes é não-enumerável e denso em R.
(Voltar para o item (d) da Solução do Exercício 21 do Capítulo VII: página 248.)
94 CAPÍTULO 3. NÚMEROS REAIS

Solução. Por denição, o conjunto dos números transcendentes é o conjunto de números reais
complementar do conjunto dos números algébricos. Este é enumerável (item ii) do Exercício
44). Logo, pelo exercício anterior, o conjunto dos números transcendentes é não enumerável e
denso em R. 

3.47 Exercício 47
Prove que o conjunto dos números algébricos é um corpo. (Este exercício requer conhecimentos
de Álgebra muito acima do que estamos admitindo até aqui.)

(Voltar para o item (b) da Solução do Exercício 21 do Capítulo VII: página 247.)

(Voltar para o item (c) da Solução do Exercício 21 do Capítulo VII: página 247.)

Solução. Os números 0 e 1 são algébricos, eles são, respectivamente, as raízes dos polinô-
mios x e x − 1,
ambos não identicamente nulos e com coecientes inteiros. Para completar,
−1
xados os números algébricos a e b, com a 6= 0, é suciente mostrar que −a, a , ab e a + b
também são algébricos. Todas as demais propriedades  associatividades, comutatividades e a
distributividade  seguirão porque elas valem para todos os números reais.
Notemos que um número é algébrico se, e somente se, existe um polinômio não identicamente
nulo com coecientes racionais do qual ele é uma raiz. De um modo geral, dado um corpo K , se
F ⊂K é um corpo com as mesmas operações de K (ou seja, F é um subcorpo de K ), vamos dizer
que um elemento de K é algébrico sobre F quando existe um polinômio não identicamente nulo
com coecientes em F do qual ele é uma raiz. Nesse caso em que F é um subcorpo de K vamos
dizer que K é uma extensão de F e que esta extensão é nita quando K for um espaço vetorial
de dimensão nita sobre F . Além disso, dados α ∈ K e F ⊂ K , seja F (α) o menor subcorpo
\
de K que contém F ∪ {α} (F (α) = {X ∈ P(K); X é subcorpo de K e X ⊃ F ∪ {α}},
interseção de uma coleção de conjuntos, não interseção de uma família). Consideremos os
seguintes resultados (ver, por exemplo, Topics in Algebra, de Herstein).

Resultado 1. Dado o corpo K e um subcorpo F de K, α de K é


um elemento
algébrico sobre F se, e somente se, F (α) é uma extensão nita de F (se F (α) é uma
extensão nita de F , seja n a dimensão do espaço vetorial F (α) sobre F , então o
n
conjunto {1, α, . . . , α }, com n + 1 elementos, é linearmente dependente, portanto
n
existem a0 , . . . , an em F tais que a0 + a1 · α + · · · + an · α = 0).

Resultado 2. Se L K e se K é uma extensão nita


é uma extensão nita de
de F, então L F (se {ui ; i ∈ Im } e {vj ; j ∈ In } são
é uma extensão nita de
bases para os espaços vetoriais L sobre K e K sobre F , respectivamente, então
{ui vj ; i ∈ Im e j ∈ In } é uma base para o espaço vetorial L sobre F ).

Resultado 3. Se L é uma extensão nita de F e se K é um subcorpo de L que


contém F , então K é uma extensão nita de F (porque K é um subespaço vetorial
do espaço vetorial L sobre F ).

Pelo Resultado 1, paraF = Q, K = F (a) é uma extensão nita de F (porque o número


real a é algébrico sobre Q) e L = K(b) é uma extensão nita de K , porque o polinômio não
identicamente nulo com coecientes racionais do qual b é uma raiz também é um polinômio
com coecientes em K (pois Q ⊂ K por construção), ou seja, b também é algébrico sobre
3.48. EXERCÍCIO 48 95

K . Portanto, L é uma extensão nita de F (Resultado 2). Sendo um corpo, L contém F ∪


{−a, a−1 , a + b, ab} e, portanto, pela minimalidade, também contém os corpos F (−a), F (a−1 ),
F (a + b) e F (ab). Então, pelo Resultado 3, todos estes corpos são extensões nitas de F , logo,
−1
pelo Resultado 1, −a, a , a + b e ab são algébricos sobre F , portanto números algébricos, como
queríamos. 

3.48 Exercício 48
Dê exemplo de uma sequência decrescente de intervalos fechados (ilimitados) cuja interseção
seja vazia e de uma sequência decrescente de intervalos (abertos) limitados cuja interseção seja
vazia.
(Voltar para o Exercício 22 do Capítulo V: página 174.)

Solução.

A4 B4

A3 B3

A2 B2

A1 B1
 
1
Vamos denir An = [n, +∞) e Bn = 0, para todo n ∈ N. Cada An é um intervalo
n
fechado ilimitado e cada Bn é um intervalo aberto limitado. Além disso, An+1 ⊂ An e Bn+1 ⊂ Bn
para todo n ∈ N. Dadox ∈ R, existe n ∈ N tal que n > x e portanto x ∈ / An , o que faz
\ 1
x∈/ An . Isto vale para todo x ∈ R. Também, caso x 6= 0, existe n ∈ N tal que n > ; daí
n∈N
x
1 \
<xex∈ / Bn . Se x = 0, então x ∈/ B1 . Portanto nenhum x ∈ R pertence a Bn . Logo
n n∈N
\ \
ambos os conjuntos An e Bn são vazios. 
n∈N n∈N

Observação. Este exercício mostra a necessidade de os intervalos do Teorema 5  o teorema


dos intervalos encaixados  serem limitados e fechados.

3.49 Exercício 49
Sejam B⊂A conjuntos não-vazios de números reais. Suponha que A seja limitado superior-
mente e que, para cada x ∈ A, exista um y∈B tal que x 6 y. Prove que, nestas condições,
tem-se sup B = sup A. Enuncie e demonstre um resultado análogo para inf.

Solução. B ⊂ A, A 6= ∅ e B 6= ∅ temos sup B 6 sup A (Exercício 33, página 84). Dado


De
x ∈ A, para y ∈ B tal que y > x, temos sup B > y > x. Portanto, sup B > x ∀x ∈ A, donde
segue sup B > sup A. Logo, de sup B 6 sup A e sup B > sup A, vem sup A = sup B .
96 CAPÍTULO 3. NÚMEROS REAIS

Podemos armar que inf B = inf A, se A é limitado inferiormente e, para cada x ∈ A, existe
um y ∈ B tal que y 6 x. De fato: B ⊂ A ⇒ inf B > inf A. Dado x ∈ A, a existência do y ∈ B
tal que y 6 x, implica inf B 6 x, pois inf B 6 y 6 x. Portanto, inf B 6 x ∀x ∈ A, o que, por
sua vez, implica inf B 6 inf A. Logo inf B = inf A, como queríamos. 

3.50 Exercício 50
Um corte de Dedekind é um par ordenado (A, B) onde A
B são subconjuntos não-vazios de
e
números racionais, tais que A não possui elemento máximo, A ∪ B = Q e, dados x ∈ A e y ∈ B
quaisquer, tem-se x < y .

a) Prove que, num corte de Dedekind (A, B), vale sup A = inf B .
b) Seja D o conjunto dos cortes de Dedekind. Prove que existe uma bijeção f : D → R.

Solução.

A B A B
p∈
/Q p∈Q

item a) Primeiro podemos concluir que sup A 6 inf B repetindo uma conta que já feita
mais de uma vez anteriormente: dado x ∈ A, temos:

x<y ∀y ∈ B ⇒ x 6 inf B.

Assim:
inf B > x ∀x ∈ A ⇒ inf B > sup A.
Se valesse a desigualdade estrita, então esse corte deixaria um buraco muito grande nos
racionais. De modo mais preciso, sesup A < inf B , tome um x ∈ Q tal que sup A < x < inf B .
Portantox∈ / Aex∈ / B , o que implicaria x ∈
/ A ∪ B . E então teríamos A ∪ B 6= Q. Como
A ∪ B = Q, só pode ser sup A = inf B . 

item b) Vamos denir essa função f do modo mais natural possível  com as únicas coisas
que temos até agora , associando (A, B) 7→ sup A. Vamos mostrar que a f assim denida é
uma bijeção.

É injetiva. Dados dois cortes distintos (A1 , B1 ) e (A2 , B2 ), vamos mostrar que sup A1 6=
sup A2 . Se (A1 , B1 ) 6= (A2 , B2 ), então A1 6= A2 ou B1 6= B2 . Vamos começar supondo A1 6= A2 .
Assim, existe x ∈ A1 tal que x ∈ / A2 ou existe x ∈ A2 tal que x ∈ / A1 . Sem perda de
generalidade, vamos supor x ∈ A1 e x ∈ / A2 . Como x ∈ / A2 e a reunião de A2 com B2 é igual
a Q, temos x ∈ B2 (lembrando que A1 é um conjunto de racionais). Agora x ∈ A1 e x ∈ B2 .
Portanto sup A1 > x e inf B2 6 x. O supremo de A1 é estritamente maior do que x porque A1
não tem um elemento máximo. Como inf B2 = sup A2 , temos sup A2 6 x < sup A1 . Portanto a
existência de um x ∈ A1 − A2 torna o supremo de A1 estritamente maior do que o de A2 . Logo
f (A1 , B1 ) 6= f (A2 , B2 ).
Se B1 6= B2 , então, sem perda de generalidade, podemos considerar um x ∈ B1 − B2 , onde
x∈ / B2 ⇒ x ∈ A2 , e; x ∈ A2 e x ∈ B1 implicam f (A1 , B1 ) = sup A1 = inf B1 6 x < sup A2 =
f (A2 , B2 ). De qualquer forma segue a injetividade de f .
3.51. EXERCÍCIO 51 97

É sobrejetiva. A sobrejetividade está sugerida no desenho que ilustra este exercício. Dado
p ∈ R, sejam A = (−∞, p) ∩ Q e B = [p, ∞) ∩ Q. O par (A, B) é um corte de Dedekind tal
que sup A = p. Logo f é sobrejetiva. 

3.51 Exercício 51
Sejam X , Y conjuntos não-vazios e f : X × Y → R uma função limitada. Para cada x0 ∈ R e
cada y0 ∈ R, ponhamos s1 (x0 ) = sup{f (x0 , y); y ∈ Y } e s2 (y0 ) = sup{f (x, y0 ); x ∈ X}. Isto
dene funções s1 : X → R e s2 : Y → R. Prove que se tem sup s1 (x) = sup s2 (y). Em outras
x∈X y∈Y
palavras
sup[sup f (x, y)] = sup[sup f (x, y)].
x y y x

Solução.
R R

f (x, y0 )
f (x0 , y)
s1 (x0 )
s2 (y0 )
y0
Y Y
x0

X X

Vamos mostrar que sup s1 (x) > sup s2 (y) e que sup s2 (y) > sup s1 (x). Destas desigualdades
x∈X y∈Y y∈Y x∈X
o resultado desejado, sup s1 (x) = sup s2 (y), segue pela antissimetria. De sup s1 (x) > s1 (x),
x∈X y∈Y x∈X
para todo x ∈ X, e de s1 (x) > f (x, y), para todo x∈X e todo y ∈Y, vem

sup s1 (x) > f (x, y) ∀x ∈ X ∀y ∈ Y,


x∈X

o que implica (condição S2 da denição de supremo)

sup s1 (x) > sup{f (x, y); x ∈ X} ∀y ∈ Y,


x∈X

ou seja (pela denição de s2 ),


sup s1 (x) > s2 (y) ∀y ∈ Y,
x∈X

donde (novamente pela condição S2),

sup s1 (x) > sup s2 (y).


x∈X y∈Y
98 CAPÍTULO 3. NÚMEROS REAIS

E, analogamente,

sup s2 (y) > f (x, y) ∀x ∈ X ∀y ∈ Y ⇒ sup s2 (y) > sup{f (x, y); y ∈ Y } ∀x ∈ X
y∈Y y∈Y
⇒ sup s2 (y) > s1 (x) ∀x ∈ X
y∈Y
⇒ sup s2 (y) > sup s1 (x).
y∈Y x∈X

Segue a tese. 

3.52 Exercício 52
Enuncie e demonstre um resultado análogo ao anterior com inf em vez de sup. Considere, em
seguida, o caso misto e prove que

sup[inf f (x, y)] 6 inf [sup f (x, y)].


y x x y

Dê um exemplo onde se tem < na desigualdade acima.

Solução.

Primeira Parte. Dados os conjuntos não vazios X eY e a função limitada f : X ×Y → R,


vamos denir as funções t1 : X → R e t2 : Y → R colocando:

t1 (x) = inf{f (x, y) ∈ R; y ∈ Y } ∀x ∈ X,

e
t2 (y) = inf{f (x, y) ∈ R; x ∈ X} ∀y ∈ Y.
Armamos que inf t1 (x) = inf t2 (y), ou, em outras palavras:
x∈X y∈Y

inf [inf f (x, y)] = inf [inf f (x, y)].


x y y x

A demonstração disto é análoga ao do exercício anterior. Fixado x0 ∈ X , temos:

inf f (x, y) 6 f (x0 , y) ∀y ∈ Y.


x

Assim, tomando o ínmo (na variável y) dos dois lados, vem

inf inf f (x, y) 6 inf f (x0 , y),


y x y

onde inf f (x0 , y) 6 inf inf f (x, y). Portanto, por transitividade, obtemos
y x y

inf inf f (x, y) 6 inf inf f (x, y).


y x x y

Do mesmo modo, começando por xar o y0 na segunda variável, obtemos inf inf f (x, y) 6
x y
inf inf f (x, y), donde segue o resultado dessa primeira parte.
y x
3.52. EXERCÍCIO 52 99

Segunda Parte. Vamos ver que não pode ocorrer o contrário.


Por um lado, se inf [sup f (x, y)] < sup[inf f (x, y)] (uma coisa menor do que um supremo),
x y y x
então existe y0 ∈ Y tal que inf f (x, y0 ) > inf sup f (x, y) (uma coisa maior do que um ínmo).
x x y
Assim existe x0 ∈ X tal que

sup f (x0 , y) < inf f (x, y0 ).


y x

Por outro lado:

inf f (x, y0 ) 6 f (x0 , y0 ) 6 sup f (x0 , y),


x y

ou seja

inf f (x, y0 ) 6 sup f (x0 , y),


x y

quaisquer que sejam o x0 ∈ X e o y0 ∈ Y . Logo, só pode ser sup[inf f (x, y)] 6 inf [sup f (x, y)].
y x x y

Claramente esta Segunda Parte pode ser parafraseada para a sua forma direta, a partir da
desigualdade

inf f (x, y0 ) 6 sup f (x0 , y) ∀(x0 , y0 ) ∈ X × Y,


x y

primeiro tomando o supremo em y do lado esquerdo e, em seguida, tomando o ínmo em x do


lado direto.

Terceira Parte. Lembrando das nossas notações (as deste exercício e as anterior), que-
remos agora uma função f tal que

sup t2 [Y ] < inf s1 [X].

Construção. As guras da solução do exercício anterior ajudam a xar as ideias. Se, por
exemplo, conseguirmos sempre t2 (y) = 0 e s1 (x) = 1 constantes, então teremos inf s1 = 1 e
sup t2 = 0. Zeros e uns nos lembram as funções características. Vamos começar então com
a função característica do conjunto Q × Q, ou seja, a função de R × R em R denida por
(x, y) 7→ 1 se (x, y) ∈ Q × Q e (x, y) 7→ 0, se algum dos dois, x ou y , for irracional.

(a) x∈Q (b) x∈


/Q (c) y∈Q (d) y∈
/Q

y y
x
x

Porém há um problema na gura (b) acima: para x ∈ R − Q, temos s1 (x) = 0. Apenas


transladar aquela linha uma unidade para cima pode fazer surgir outros problemas. Após
algumas outras tentativas e erros chegamos à resposta abaixo.
100 CAPÍTULO 3. NÚMEROS REAIS

Resposta. Vamos associar um par ordenado ao 0 se ambas as coordenadas forem racionais


ou se ambas as coordenadas forem irracionais; se apenas uma das coordenadas for racional, então
vamos associá-lo ao 1. De modo mais preciso, a função f : R × R → R dada por
    
0 se (x, y) ∈  Q × Q ∪ (R − Q) × (R − Q)
f (x, y) =   
1 se (x, y) ∈ Q ×(R − Q) ∪ (R − Q) × Q

(a) x∈Q (b) x∈


/Q (c) y∈Q (d) y∈
/Q

y y

x x

Se x ∈ / Q, seja y1 ∈ Q e y2 ∈
/ Q e teremos f (x, y1 ) = 1 e f (x, y2 ) = 0, o que torna s1 (x) = 1
e t2 (x) = 0 (aqui Y = X = R). Se x ∈ Q, sejam y1 ∈ / Q e y2 ∈ Q e teremos f (x, y1 ) = 1
e f (x, y2 ) = 0, o que torna s1 (x) = 1 e t2 (x) = 0. Portanto s1 (x) = 1 e t2 (x) = 0 para todo
x ∈ R. Logo
sup t2 [R] = 0 < 1 = inf s1 [R],
como queríamos. 

3.53 Exercício 53
Sejam x, y números reais positivos. Prove que se tem

√ x+y
xy 6 .
2

Solução. Como todos os números aí envolvidos são positivos, podemos fazer as equivalências
abaixo:
√ x+y
xy 6 ⇔ 4xy 6 (x + y)2 ⇔ 0 6 x2 − 2xy + y 2 = (x − y)2 ,
2
onde (x − y)2 > 0, porque números ao quadrado nunca são negativos. Segue a tese. 

3.54 Exercício 54
A desigualdade entre a média aritmética e a média geométrica, vista no exercício anterior,
√ x1 + · · · + xn
vale para n números reais positivos x1 , . . . , xn . Sejam G = n x1 x2 . . . xn e A = .
n
Tem-se G 6 A. Isto é evidente quando x1 = · · · = xn . Para provar a desigualdade no caso
geral, considere a operação que consiste em substituir o menor dos números dados, digamos
xi · xj
xi e o maior deles, digamos xj respectivamente por x0i = 0
e xj = G. Isto não altera
G
a média geométrica e, quanto à aritmética, ela não aumenta, pois, como é fácil de se ver,
x0i + x0j 6 xi + xj . Prove que, repetida esta operação no máximo n vezes, obtemos n números
3.55. EXERCÍCIO 55 101

todos iguais a G e, portanto, sua média aritmética é G. Como em cada operação a média
√ x1 + · · · + xn
aritmética não aumentou, conclua que G 6 A, ou seja n x x ...x 6
1 2 n .
n
(Voltar para a Solução do Exercício 11 do Capítulo IV: página 123.)

(Voltar para a Solução do Exercício 13 do Capítulo IV, Primeira Parte: página 126.)

Solução. Vamos chamar a lista x1 , . . . , x n de lista 1 e de lista k+1 à lista obtida a partir de
uma dada lista k , k ∈ N,através da operação descrita no enunciado. Assim, como a lista n + 1
é a que é alcançada após n repetições desse processo, queremos encontrar l ∈ {1, . . . , n + 1} tal
que todos os elementos da lista l são iguais a G.
A diferença do número de vezes que G aparece na lista k + 1 com o número de vezes que ele
aparece na lista k ou é 0 ou é 1. E só é zero se G for o maior elemento da lista k . Neste caso,
armamos que todos os elementos da lista k já são iguais a G (ver lema abaixo). Portanto,
sendo x o número de vezes que G aparece na lista 1, ou G aparece x + (n − x) = n vezes na
lista n − x + 1 ou G é o maior elemento de alguma lista k , com k ∈ {1, . . . , n − x}. No primeiro
caso, como cada lista tem n elementos, todos os elementos da lista n − x + 1 são iguais a G,
onde n − x + 1 6 n + 1. No segundo caso, idem, todos os elementos da lista k são iguais a G,
onde k 6 n + 1. De qualquer forma, existe l ∈ {1, . . . , n + 1} tal que todos os elementos da
lista l são iguais a G, como queríamos.
Sendo Ak a média aritmética da lista k , para o número l tal que todos os elementos da lista
l são iguais a G, temos Al = G e Al 6 A1 = A. Isto por causa da armação feita no enunciado,
da qual decorre A1 > A2 > A3 > · · · . Ela é válida porque

x i xj G2 − G(xi + xj ) + xi xj (G − xi )(G − xj )
x0i + x0j − xi − xj = + G − xi − xj = =
G G G
é menor do que ou igual a 0 se, e somente, (G − xi )(G − xj ) 6 0. Desigualdade esta que vem
das desigualdades xi 6 G e xj > G, as quais também estão contempladas no lema abaixo.


Lema. Sejam os números positivos y1 6 y2 6 · · · 6 yn e seja G = n y1 y2 · · · yn . Então
y1 6 G 6 yn . Além disso, se G > yn , então y1 = y2 = · · · = yn .
Solução. De y1 6 yk 6 yn , para todo k ∈ {1, . . . , n}, vem (multiplicando) y1n 6 y1 y2 · · · yn 6
ynn , ou seja, y1n 6 Gn 6 ynn e, daí, y1 6 G 6 yn (contrapositiva do Exercício 11, página 72),
como queríamos para a primeira parte do lema.
Se G > yn , então G = yn (pois G 6 yn , conforme acabamos de demonstrar), de modo que
yn = Gn = y1 · · · yn−1 yn , o que implica, pela lei do corte, ynn−1 = y1 · · · yn−1 . Esta igualdade
n

signica que yn é a média geométrica dos números y1 , . . . , yn−1 e, portanto, pelo que vimos
acima, yn 6 yn−1 . Disto, e da hipótese yn > yn−1 , segue yn = yn−1 . E assim, repetindo este
argumento, chegamos a y1 = · · · = yn−1 = yn . 

Observação. Neste lema, analogamente, G 6 y1 também implica y1 = · · · = yn .

3.55 Exercício 55
Seja K um corpo ordenado completo. Indique com 00 e 10 o zero e a unidade de K . Para
cada n ∈ N, sejam n = n · 1 + [sic ] · · · + 1 (n vezes) e (−n) = −n . Denamos uma
0 0 0 0 0
0
 
p p p
função f : R → K pondo f = 0 para todo ∈ Q e, para x irracional, seja f (x) =
q q q
102 CAPÍTULO 3. NÚMEROS REAIS

p0
 
p
sup ∈ K; < x . Prove que f é um homomorsmo sobrejetivo e conclua que f é uma
q0 q
bijeção, ou seja um isomorsmo de R sobre K.
(Voltar para a Solução do Exercício 56, Segunda Parte: página 109.)

Solução.

No elevador. f (x+y) = f (x)+f (y) e f (xy) = f (x)f (y) pela denição,


Para mostrar que
como ela depende de o ponto no qual f é aplicada ser racional ou não, devemos considerar os
casos em que x e y são racionais, x e y são irracionais ou um deles é racional e o outro é
irracional, sendo que o caso em que x e y são irracionais deve ser ainda dividido nos casos
x + y ∈ Q, xy ∈ Q, x + y ∈ R − Q e xy ∈ R − Q (Exercício 27, página 81). Para não termos que
car dividindo a questão em tantos casos, seria mais conveniente toda a denição de f estar
escrita numa única igualdade. E é dessa forma que começaremos a solução abaixo.

Nível 1. Armamos que

p0 p
 
f (x) = sup ; <x ∀x ∈ R
q0 q

(Nível 2.1). Além disso, vamos convencionar que o xeoy que aparecerem numa notação como
x
sempre serão números inteiros, com y 6= 0. A mesma convenção valendo para uma notação
y
x0
do tipo , para representar o análogo de um número racional em K. Nos Níveis 1.1, 1.2, 1.3
y0
e 1.4 vamos mostrar, respectivamente, que f (x + y) = f (x) + f (y), f (xy) = f (x)f (y), f é
sobrejetiva e f é injetiva.

Nível 1.1 (adição). Pelo Exercício 37 (página 86), temos

p0 p
 0  0
r0 p
   
r r p r
f (x) + f (y) = sup ; < x + sup ; < y = sup + ; <x e <y .
q0 q s0 s q 0 s0 q s

p0 r 0 p
   0 
r t t
Assim, sendo X1 = + ; < x e < y e X2 = ; < x + y , vamos mostrar que
q 0 s0 q s u0 u
X1 = X2 . Disto segue sup X1 = sup X2 , isto é, f (x) + f (y) = f (x + y).
p0 r 0 p r p r (ps + qr)0
Seja + ∈ X 1 . De < x e < y , vem + < x + y e então ∈ X2 , pois
q0 s0 q s q s (qs)0
p r ps + qr p0 r 0
+ = . Agora, para concluir que + ∈ X2 , armamos que f restrita à Z é um
q s qs q 0 s0
 a  a0
homomorsmo (Nível 2.2). Deste modo, notando que f (a) = f = 0 = a0 se a ∈ Z, temos
1 1

(ps + qr)0 f (ps + qr) f (p)f (s) + f (q)f (r) p0 s0 + q 0 r 0 p0 r 0


= = = = +
(qs)0 f (qs) f (q)f (s) q 0 s0 q 0 s0

(Exercício 1, página 65). Portanto, X1 ⊂ X2 .


3.55. EXERCÍCIO 55 103

t0 t p t p
Reciprocamente, seja
0
∈ X2 ; ele é tal que < x+y . Para tal que −y < < x (o qual
u u q  u q
t t p t t p p qt − pu p
existe pelo Teorema 4, porque −y < x), temos − < y e = − + = + .
u u q u u q q qu q
(qt − pu)0 p0
Então + 0 ∈ X1 , onde, analogamente às contas do último parágrafo,
(qu)0 q
(qt − pu)0 p0 q 0 t0 − p0 u0 p0
 0
p0 p0 t0

t
+ = + = − + = .
(qu)0 q0 q 0 u0 q0 u0 q 0 q0 u0
Logo, X2 ⊂ X1 . Segue a tese: X1 ⊂ X2 e X2 ⊂ X1 implicam X1 = X2 .

No elevador. Imitar o nível anterior para mostrar que f (xy) = f (x)f (y) envolve escrever
o produto de dois supremos como o supremo de um conjunto só. O Exercício 39 (página 88) nos
permite fazer isso, mas só se os elementos dos conjuntos envolvidos forem todos positivos. No
nível abaixo vamos começar supondo então ambos x e y positivos e, em seguida, quando um dos
dois for negativo, vamos cair no caso em que ambos são positivos, usando que f (−x) = f (x).

Nível 1.2 (multiplicação). x e y números


Sejam
 reais positivos. De modo geral, arma-
 0
p p
mos que, se z > 0, então f (z) = sup ; 0 < < z (Nível 2.3). Esta armação e o Exercício
q 0 q
39 nos permitem traduzir a igualdade f (x)f (y) = f (xy) como
 0 0   0 
p r p r t t
sup 0 · 0 ; 0 < < x e 0 < < y = sup ; 0 < < xy .
q s q s u0 u
É ela quem iremos demonstrar. De modo semelhante ao que zemos no item anterior, temos

(pr)0 t0
 
p r p r t
0< <x e 0 < < y ⇒ 0 < · < xy ⇒ ∈ ; 0 < < xy
q s q s (qs)0 u 0 u
t0 t t 1 p p t q
e, dado tal que 0< < xy , seja · < < x. Isto implica · < y e,
tal que 0<
u0 u u y q q u p
portanto,
p0 (qt)0
 0 0 
p r p r
· ∈ · ;0< <xe0< <y ,
q 0 (pu)0 q 0 s0 q s
0 0 0 0 0 0 0 0 0 0 0
(pr) pr p r p (qt) p qt t
onde:
0
= 0 0 = 0· 0 e 0· 0
= 0 · 0 0 = 0 , pelo fato de f | Z ser um homomorsmo
(qs) qs q s q (pu) q pu u
(Nível 2.2). Portanto, os dois conjuntos, cujos supremos queremos mostrar serem iguais, são
iguais. Logo, f (xy) = f (x)f (y) se x e y são positivos.
Se x e y não são positivos, então um dos dois é igual a zero, um é negativo e o outro é
positivo, ou ambos são negativos. Se um dos dois é igual a zero, temos

f (0 · x) = f (0) = 00 = 00 · f (x) = f (0) · f (x),


para todo x ∈ R. Nos outros casos, notemos que f (−z) = −f (z), para todo z real. Isto segue
0
do nível acima, f (−z) é o simétrico de f (z), pois f (−z) + f (z) = f (−z + z) = f (0) = 0 , ou
seja, f (−z) = −f (z). Assim, se x é negativo e y é positivo, então −x e y são positivos e

f (xy) = f (−((−x) · y)) = −f ((−x) · y) = −(f (−x) · f (y)) = −(−f (x)) · f (y) = f (x)f (y);
e, se x e y são negativos, então −x e −y são positivos e

f (xy) = f ((−x)(−y)) = f (−x)f (−y) = (−f (x))(−f (y)) = f (xy).


Logo, f (xy) = f (x)f (y), para todo x∈R e todo y ∈ R.
104 CAPÍTULO 3. NÚMEROS REAIS

Nível 1.3 (sobrejetividade). Vamos exibir uma inversa à direita para f (f é sobrejetiva
se, e somente se, ela admite uma inversa à direita, conforme já foi visto na página 22 do livro);
0
 
p p
armamos que ela é a função g:K→R denida por g(x) = sup ∈ R; 0 < x , para todo
q q
x∈K (no Nível 2.4 está a justicativa do porquê ser lícito denir g desta forma).
Queremos mostrar que f (g(x)) = x, ou seja, que x é o supremo do conjunto

p0 p
 
Y = ; < g(x) .
q0 q
p0
Vamos vericar que x possui as duas propriedades que o fazem ser esse supremo. Dado ∈Y,
q0
p r p r r0
temos < g(x) e, então, como g(x) é um supremo, existe tal que < < g(x) e < x.
q s q s s0
p r p0 r0 p0
Armamos que < implica < (Nível 2.5). Portanto, x> , e x satisfaz a condição
q s q0 s0 q0
S1.
r0 r0
Dado c ∈ K , c < x, seja tal que c< <x (ele existe por causa do Teorema 4 e da
s0 s0
observação feita na página 80, este teorema
 também0 se aplica a K porque este é um corpo
0

r r p p r
ordenado completo). Daí
0
< x ⇒ ∈ ∈ R; 0 < x ⇒ g(x) > . Podemos garantir
s s q q s
r r0 p0 r p p r0
g(x) > ; para ver isto basta tomar < < x , o que faz < e g(x) > . Logo, ∈ Y,
s s0 q0 s q q s0
r0
com c < 0 , o que signica que x também satisfaz a condição S2'. Segue a tese, x = f (g(x)).
s

Nível 1.4 (injetividade). A injetividade de f agora é consequência do Exercício 4 (pá-


gina 68): f é injetiva porque é um homomorsmo e f (1) = 10 .

p0 p
 
Nível 2.1. Queremos mostrar que f (x) = sup ; <x , para todo x ∈ R. Por deni-
q0 q
ção, esta igualdade é verdadeira se x é irracional. Vamos mostrar então que

p0 p a0
 
a
sup 0
; < = .
q q b b0
a p a0 p0 a0
Em primeiro lugar, temos > ⇒ 0 > 0 (Nível 2.5) e, portanto, satisfaz a condição
b q b q b0
S1 da denição de supremo.
a0 p0 p0
Em segundo lugar, dado c ∈ K, c < , queremos satisfazendo as condições c< e
b0 q0 q0
p a
< . Como K é um corpo ordenado completo, o Teorema 4 se aplica a ele também (ver a
q b
p0 p0 a0
observação feita após o Axioma da página 80). Assim, existe tal que c< < . Para
q0 q0 b0
p0 p a a0
um tal , armamos que < , donde podemos concluir que também satisfaz a condição
q0 q b b0
p0 a0 p a
S2' da denição de supremo. De fato < implica < , pois o Nível 2.5 nos dá a sua
q0 b0 q b
a p a0 p0
contrapositiva: 6 ⇒ 0 6 0.
b q b q
3.55. EXERCÍCIO 55 105

a0 p0 p
 
a
Logo, é o supremo do conjunto ; < , como queríamos.
b0 0
q q b

Nível 2.2 (f | Z é homomorsmo). Vamos dividir este nível nos itens (a), (b), (c) e (d).
No item (a), dado m ∈ Z, vamos demonstrar por indução que f (m + n) = f (m) + f (n) para
todo n ∈ N. Isto será suciente para concluirmos que f (−x) = −f (x), para todo x ∈ Z, donde
seguirá f (x + y) = f (x) + f (y) para todo x ∈ Z e todo y ∈ Z, o que será feito no item (b). No
item (c), usando o item (b), dado m ∈ Z, demonstraremos por indução que f (mn) = f (m)f (n)
para todo n ∈ N e o item (d) nos dará f (xy) = f (x)f (y), para todo x ∈ Z e todo y ∈ Z,
também usando o fato de que f (−x) = −f (x).

(a) m ∈ Z. Para n = 1, se m > 0, então f (m + 1)  que é igual a (m + 1)0  é uma


Seja
0
soma com m + 1 parcelas iguais a 1'. Soma esta que, pela associatividade, é do tipo x + 1 ,
0
na qual x é uma soma com m parcelas iguais a 1', ou seja, x = m , por denição. Portanto,
f (m + 1) = m0 + 10 = f (m) + f (1) se m > 0.
Suponhamos agora m < 0. Para dar um argumento semelhante ao do último parágrafo,
0 0 0 0 0 0
notemos que m é uma soma com −m parcelas iguais a (−1) : (−1) +· · ·+(−1) = −1 −· · ·−1 =
−(10 + · · · + 10 ) = −(−m)0 = (−(−m))0 = m0 , pois −m ∈ N. Assim, se m = −1, então
f (m) + f (1) = m0 + 10 = (−1)0 + 10 = −10 + 10 = 00 = f (0) = f (m + 1). E, se m < −1, então
f (m+1) é uma soma com −m−1 parcelas iguais a (−1)0 . Como −m > 1, por hipótese, esta soma
0 0 0
é igual a x − (−1) , na qual x é uma soma com −m parcelas iguais a (−1) , ou seja, x = m , pela
0 0 0 0
observação do início deste parágrafo. Portanto, f (m + 1) = m − (−1 ) = m + 1 = f (m) + f (1).
0
Se m = 0, então f (m + 1) = f (1) = 0 + f (1) = f (0) + f (1) = f (m) + f (1). Concluímos
então que f (m + 1) = f (m) + f (1), para todo m ∈ Z. Este fato será usado no passo de indução.

Fixado m ∈ Z, dado n ∈ N, suponhamos que f (m + n) = f (m) + f (n). Assim, temos,

f (m + (n + 1)) = f ((m + n) + 1)
= f (m + n) + f (1)
= (f (m) + f (n)) + f (1)
= f (m) + (f (n) + f (1))
= f (m) + f (n + 1).

Nesta sucessão de igualdades, a segunda e a quinta valem pelo que vimos mais acima; m+n
e n são números inteiros; e a terceira vale por hipótese. Portanto, por indução, dado m ∈ Z,
temos f (m + n) = f (m) + f (n), para todo n ∈ N. Isto é, f (m + n) = f (m) + f (n), para todo
m∈Z e todo n ∈ N.

(b) Assim, f (−x) = −f (x), para todo x ∈ Z. x ∈ Z: se x = 0, então ambos


De fato, dado
f (−x) e −f (x) x 6= 0, então
são iguais a 0', e; se −x é inteiro e o outro é um
um dentre x e
0
número natural, de modo que, pelo item anterior, podemos fazer 0 = f (0) = f (x + (−x)) =
f (x) + f (−x), o que signica que f (−x) = −f (x). Dados os inteiros x e y , para concluir que
f (x + y) = f (x) + f (y), só falta considerar o caso em que ambos são negativos. Neste caso, −x
106 CAPÍTULO 3. NÚMEROS REAIS

e −y são números naturais e caímos no item (a):

f (x + y) = f (−(−x + (−y)))
= −f (−x + (−y))
= −(f (−x) + f (−y))
= −(−f (x) − f (y))
= f (x) + f (y),

como queríamos.

(c) Seja m ∈ Z. Temos f (m·1) = f (m) = f (m)·10 = f (m)·f (1). E, se f (mn) = f (m)f (n),
para n ∈ N, então:

f (m(n + 1)) = f (mn + m)


= f (mn) + f (m)
= f (m)f (n) + f (m)
= f (m)(f (n) + 10 )
= f (m)(f (n) + f (1))
= f (m)f (n + 1)

(a segunda e a sexta igualdades valem pelos itens anteriores). Logo, por indução, f (mn) =
f (m)f (n), para todo m∈Z e todo n ∈ N.

(d) Se x e y são inteiros negativos, então −x e −y são números naturais. Portanto, pelo
item anterior, usando novamente que f (−m) = −f (m), para todo m inteiro, temos

f (xy) = f ((−x)(−y)) = f (−x)f (−y) = (−f (x))(−f (y)) = f (x)f (y).

No caso em que um dos dois for igual a zero: f (0 · x) = f (0) = 00 = 00 · f (x) = f (0) · f (x).
Logo, f (xy) = f (x)f (y), para todo x ∈ Z e todo y ∈ Z.
Todos estes itens nos dizem que a restrição f | Z é um homomorsmo.

Nível 2.3 (f (z) quando z > 0). Seja z um número real positivo e sejam

p0 p0 p
   
p
A= 0
;0< <z e B= ; <z .
q q q0 q

Queremos mostrar que sup A = sup B . De A ⊂ B , vem sup A 6 sup B (Exercício 33, página 84).
Agora, para concluir que sup A > sup B (estas duas últimas desigualdades nos darão sup A =
sup B ), vejamos que sup A > x, para todo x ∈ B (condição S2 da denição desupremo). Dado
p0 p r p r
0
∈ B , pela densidade dos racionais, como < z , seja tal que max , 0 < < z . Para
q q s q s
r p r0 p0 r0 p0 r0 p0
ele, temos > ⇒ 0 > 0 (Nível 2.5) e 0 ∈ A. Portanto, sup A > 0 porque sup A > 0 > 0 .
s q s q s q s q
Logo, sup A > x, para todo x ∈ B , como queríamos.
3.56. EXERCÍCIO 56 107

p0
 
p
Nível 2.4 (denição de g ). Seja x ∈ K e consideremos o conjunto X = ∈ R; 0 < x .
q q
Para ser lícito tomarmos seu supremo, devemos vericar que X é não vazio e limitado superior-
mente. Estas duas coisas são consequências do Teorema 4 (seu análogo vale para K por causa
da observação feita na página 80 do livro: de R usou-se apenas que ele é um corpo ordenado
completo) e do Teorema 3 (este também vale por uma observação feita na página 80 do livro:
0
p
corpos ordenados completos são arquimedianos). Portanto, existe < x (donde X 6= ∅) e,
q0
p p
para n0 > x, temos n> , para todo ∈ X, pois
q q

p p0 p0 p
∈ X ⇒ 0 < x ⇒ 0 < n0 ⇒ < n
q q q q

p p0
(pela contrapositiva do Nível 2.5: n6 ⇒ n0 6 0 ). Isto é, o número natural n tal que n0 > x
q q
é uma cota superior para X. Logo, X é não vazio e limitado superiormente e está tudo certo
com a denição de g.

r p r p qr − ps
Nível 2.5. > , então o número racional − =
Se é positivo. Armamos
  s q s q qs
x x x
agora que f > 0 se > 0, pois, se > 0, então ambos x e y são positivos ou ambos são
y y y
negativos (observação da página 68 e Exercício 13, página 73); se ambos são positivos, então
x0
 
0 0 x
x e y são positivos (eles são somas de números positivos, por denição) e f = 0 >0
y y
0 0
(Exercício 13) e se ambos x e y são negativos, então x e y também são negativos (pois,
0 0 0 0
sendo x inteiro, x < 0 ⇒ −x ∈ N ⇒ 0 < (−x) = f (−x) = −f (x) ⇒ f (x) = x < 0 ;
0
x
f (−x) = −f (x) pelo item (b) do Nível 2.2) e 0 > 0 (Exercício 13, junto com as implicações:
y
a < 0 e b < 0 ⇒ −a > 0  e − b > 0 ⇒ (−a)(−b) > 0 ⇒ ab > 0). Logo, lembrando que
r0 p0

qr − ps qr − ps
> 0, temos 00 < f = 0 − 0 (contas semelhantes às feitas no Nível 1.1,
qs qs s q
r0 p0
usando que f | Z é um homomormo), isto é, > , como queríamos. 
s0 q0

3.56 Exercício 56
Seja f : R → R um isomorsmo de R em si mesmo. Prove que f = identidade. Conclua que se
K e L são corpos ordenados completos, existe um único isomorsmo de K sobre L.

Solução.

Primeira Parte: f é a identidade.

No elevador. Já zemos algo parecido anteriormente, a saber, no Exercício 5 (página


69). Por este exercício, temosf (x) = x ∀x ∈ Q. Só falta concluir que f (x) = x ∀x ∈ R − Q.
Se fosse f (x) < x, seja a um número racional f (x) < a < x. Como a = f (a), temos então
f (x) < f (a) < x. Agora, se f preserva a ordem, então x < a < x, o que é um absurdo.
108 CAPÍTULO 3. NÚMEROS REAIS

Mas eu não estava conseguindo mostrar que f preserva a ordem, o que me fez buscar outros
caminhos. Porém as mais diversas tentativas sempre recaíam na necessidade de mostrar que f
tem essa propriedade. Isso pareceu ser algo essencial e meio que inevitável, portanto. Até que
resolvi dar uma pesquisada nos livros. A grande sacada eu encontrei no livro do Jacy: escrever
a = b2 , se a é positivo. Algo assim tão simples fez toda a diferença. De fato o livro do Jacy
está nas referências do livro Curso de Análise.

Nível 1. Como a restrição f |Q é um homomorsmo não identicamente nulo, temos f (x) =


x ∀x ∈ Q, pelo Exercício 5. Dado x ∈ R, para concluir que f (x) = x, vamos mostrar que
x 6 f (x) e x > f (x) (a demonstração disto está no Nível 2).

Nível 2. Armamos que f preserva a ordem (a demonstração disto está no Nível 3).
Assim, dado um racional a maior do que f (x), temos (lembrando que f (a) = a):
f (x) < a = f (a) ⇒ x < a.
Esta conta signica que todo número racional maior do que f (x) é também maior do que x,
ou seja, (f (x), x) ∩ Q = ∅ e, portanto, x 6 f (x) (pois Q é denso em R). De modo análogo,
para todo racional a < f (x), temos a < x. Assim, também não há números racionais no
intervalo (x, f (x)). Portanto x > f (x). Logo x 6 f (x) e x > f (x) qualquer que seja o x, como
queríamos. .

Nível 3: Lema. Seja K um corpo ordenado completo e L um corpo ordenado. Se


f : K → L é um homomorsmo não identicamente nulo, então, dados x, y ∈ K , vale a
implicação:
x < y ⇒ f (x) < f (y).
Solução. x < y , então, por denição, y − x > 0. Armamos que isto implica f (y − x) > 0
Se
(a demonstração disto está no Nível 4). Daí, por f ser um homomorsmo, temos f (y)−f (x) > 0,
ou seja, f (x) < f (y), pela denição de ordem em L.

No elevador. O que vamos fazer para mostrar que f (x) é um número positivo, caso x o
seja, é escrever f (x) como um número não nulo ao quadrado. Feito isto, o resultado seguirá,
porque todo número não nulo ao quadrado é positivo.

Nível 4: Lema. O homomorsmo não identicamente nulo f : K → L, onde K é um


corpo ordenado completo e L é um corpo ordenado é tal que f (x) > 0, se x > 0.
Solução. Dado um x > 0 em K , como K é completo, existe um y não nulo tal que x = y 2 .
Assim, como f é um homomorsmo, temos:
2
f (x) = f (y 2 ) = f (y) · f (y) = f (y) .
Agora, este y é tal que f (y) 6= 0. Para ver isto basta lembrarmos que 0 é o único elemento
associado ao 0 num homomorsmo não identicamente nulo (Exercício 4, página 68). Logo
f (x) > 0, como queríamos.

Pergunta. Assim como nos lemas acima, seja f : K → L um homomorsmo não identi-
camente nulo, onde K e L são corpos ordenados e, além disso, K é completo. O fato de f
preservar a ordem torna L completo também? Existe, por exemplo, um homomorsmo não
identicamente nulo de R em Q?
3.57. EXERCÍCIO 57 109

Segunda Parte: unicidade. Dados dois corpos ordenados completos K eL, para com-
pletar este exercício, queremos concluir que existe um único homomorsmo de K sobre L. O
Exercício 55 (página 101) mostra a existência. Só falta agora a unicidade. Como tudo indica,
devemos dar um jeito de utilizar o resultado que acabamos de demonstrar. Se f, g : K → L são
isomorsmos, queremos concluir que f = g . O diagrama abaixo mostra como sair de e chegar
a R passando por K e por L através da f e da g .

f
K L
h g −1
R K
h−1

h : R → K um isomorsmo; assim, a composição h−1 ◦ g −1 ◦ f ◦ h é um isomorsmo


Seja
−1
de R em R. Portanto, como acabamos de ver acima, h ◦ g −1 ◦ f ◦ h = id, onde id denota a
identidade de R em R. Deste modo:

h−1 ◦ g −1 ◦ f ◦ h = id ⇒ g −1 ◦ f ◦ h = h ⇒ g −1 ◦ f = idK .
−1
A primeira igualdade acima está dizendo que a função g ◦ f ◦ h é a inversa à direita de
h−1 , que não pode ser outra coisa senão h, por isso vale a primeira implicação. A segunda
−1
implicação vale por causa da sobrejetividade de h, até que chegamos a g ◦ f . Esta função
−1
ser igual a identidade em K signica que f é a inversa à direita de g , portanto só pode ser
f = g, como queríamos. 

Observação. Podemos também desenvolver as contas acima do seguinte modo:

id = h−1 ◦ g −1 ◦ f ◦ h = (g ◦ h)−1 ◦ f ◦ h ⇒ f ◦ h = g ◦ h ⇒ f = g.

3.57 Exercício 57
x
Verique que f : R → (−1, 1), denida por f (x) = √ , é uma bijeção de R sobre o
1 + x2
intervalo (−1, 1).

x
Solução. Vamos ver que a função g : (−1, 1) → R denida por g(x) = √ é a inversa
1 − x2
de f:
x x x
√ √ √ √
1−x 2 1−x 2 1 − x2 x
f (g(x)) = s = r = r = √ · 1 − x2 = x,
2 1 1 − x 2

x
2 x
1+ √ 1+
1 − x2 1 − x2 1 − x2

se x ∈ (−1, 1) f é sobrejetiva) e
(portanto

x x x
√ √ √ √
1 + x2 1 + x2 1 + x2 x
g(f (x)) = s = r = r = √ · 1 + x2 = x,
2 1 1 + x 2

x
2 x
1− √ 1−
1 + x2 1 + x2 1 + x2

se x∈R (portanto f é injetiva). Logo, f é uma bijeção. 


110 CAPÍTULO 3. NÚMEROS REAIS

3.58 Exercício 58
Um conjunto G de números reais chama-se um grupo aditivo quando 0 ∈ G e x, y ∈ G ⇒ x−y ∈
G. Então, x ∈ G ⇒ −x ∈ G e x, y ∈ G ⇒ x + y ∈ G. Seja então G ⊂ R um grupo aditivo de
+
números reais. Indiquemos com G o conjunto dos números reais positivos pertencentes a G.
+
Excetuando o caso trivial G = {0}, G é não-vazio. Suponhamos pois G 6= {0}. Prove que:

i) Se inf G+ = 0, então G é denso em R;

ii) Se inf G+ = a > 0, então G = {0, ±a, ±2a, . . .}. [Sugestão: para provar (ii)
a ∈ G+ e
a
note primeiro que se fosse a∈/ G+ +
existiriam g, h ∈ G com a < h < g < a + , donde
2
a +
> g − h ∈ G , uma contradição. Em seguida, observe que todo g ∈ G se escreve sob
2
a forma g = a · q + r , com q ∈ Z, sendo 0 6 r < a. Veja que r = g − a · q ∈ G, pois q é
inteiro.]

iii) Conclua que, se α∈R é irracional, os números reais da forma m + nα, com m, n ∈ Z,
constituem um subconjunto denso em R.

(Voltar para a Solução do Exercício 47 do Capítulo IV: página 156.)

(Voltar para a Solução do Exercício 16 do Capítulo V: página 168.)

Solução.

item i)Novamente aqui, assim como na Solução do Exercício 43 (página 92), a ideia da
+
demonstração do Teorema 4 rende seus frutos. Dados os reais a < b, seja x ∈ G tal que
+
0 < x < b − a (este x existe porque b − a > inf G ). Seja A = {n ∈ Z; nx > b}. Este conjunto
é não vazio por causa da propriedade arquimediana, e é limitado inferiormente pelo elemento
b/x. Seja então m = min A, de modo que (m − 1)x < b. Além disto: mx > b e −x > a − b
implicam (somando membro a membro) mx − x = (m − 1)x > a. Portanto a < (m − 1)x < b,
onde (m − 1)x ∈ G. Logo G é denso em R. 

item ii) Vamos supora > 0 e a ∈/ G+ e produzir uma contradição. Como estamos
+
tratando de um ínmo, tomando ε > 0, podemos considerar um x ∈ G , x < a + ε e x 6= a,
uma vez que a ∈ / G+ , isto é, x ∈ G+ e a < x < a + ε. Olhando agora para a < x, podemos
+
repetir esse processo e encontrar um y ∈ G tal que a < y < x < a + ε. Esses x e y estão bem
+ +
próximos, para eles temos x − y < ε, onde x − y ∈ G , uma vez que G é um grupo aditivo.
+
Assim, se 0 < ε < a, temos 0 < x − y < a e, portanto, a é e não é um ínmo de G . Logo
+ +
só podemos ter a ∈ G (junto com inf G = a > 0, temos uma das duas possibilidades: ou
+ +
a ∈ G ou a ∈ / G ; como não pode ocorrer a ∈ / G+ , só pode ser a ∈ G+ ).
Do parágrafo anterior temos a ∈ G; assim, somando sucessivamente a consigo mesmo,
chegamos à inclusão {0, ±a, ±2a, . . .} ⊂ G. A questão que se coloca agora é: existem outros
elementos em G além dos elementos da forma ma para m ∈ Z? Vamos supor que sim, e seja
b ∈ G tal que b 6= ma ∀m ∈ Z. O que podemos fazer com essa informação? Podemos, por
exemplo, cercar o b. De modo mais preciso, seja um inteiro n tal que (n − 1)a < b < na (ele
existe: tome n o menor dos inteiros m tais que ma > b, o antecessor de n não é b, porque b não
é da forma (n − 1)a). Dessa desigualdade, somando (1 − n)a, chegamos a 0 < b + (1 − n)a < a,
+
onde b + (1 − n)a ∈ G e, então, a não seria o ínmo de G . Logo, pela contrapositiva, um tal
elemento b não existe. 
3.58. EXERCÍCIO 58 111

Observação. O que zemos na solução da segunda parte deste item foi o seguinte: nx <
+
b < (n + 1)x ⇒ 0 < b − nx < x, o que, por sua vez, implica que x não é ínmo de G , caso
nx, b ∈ G. Para x = a, isto é a forma contrapositiva (e, portanto, equivalente) do resultado
+ +
desejado: inf G ∈ G ⇒ G = {m · inf G ; m ∈ Z}.

item iii)

No elevador. Num primeiro momento este resultado pode causar uma certa estranheza:
o conjunto dos nα deixa muitos espaços vazios; agora, basta somar cada um deles com algum
número inteiro que eles se tornam densos na reta? A resposta é sim, a ideia é que essas
translações por números inteiros diminuem cada vez mais a distância que existiam entre eles.
A forma como este exercício está disposto nos leva a observar que o conjunto G desses
+
números é um grupo aditivo e nos sugere mostrar que inf G = 0. Uma condição para que isso
+ +
ocorra é que, para todo x ∈ G , exista y ∈ G estritamente menor do que x. Propriedades
+
de x podem auxiliar na busca deste y , mas não sabemos se todo elemento de G possuem
tais propriedades. Percebemos então que não é necessário encontrar y para todo x, basta que o
+
encontremos para cada x de um certo conjunto conveniente de G . É só após todo esse processo
investigativo é que nascem o conjunto X e a função f considerados na solução abaixo, os quais
desempenharão papéis centrais em nossa solução.

Nível 1. Seja G = {m + nα ∈ R; m, n ∈ Z}. Armamos que existe um subconjunto não


+
vazio X⊂G e uma função f : X → X com a seguinte propriedade:

0 < f (x) < x ∀x ∈ X

(se f (x) ∈ X ⊂ G+ , é claro que f (x) > 0; escrevemos da forma acima só para enfatizar que
todo f (x) é positivo, o que é algo importante conforme veremos durante a solução). Essa f
está exibida no Nível 2.1. Ela implica a densidade de G (a demonstração desta armação está
no Nível 2.2).

No elevador. Nessa construção tentamos aplicar uma ideia já vista pelo menos outras três
vezes anteriormente, a saber, a do Teorema 4. Então tomamos nosso x, tomamos um a ∈ G+
e consideramos o menor dos n nx > a. A questão agora é: o antecessor de
naturais tais que
n (já que n é mínimo) é tal que (n − 1)x < a? Se sim, a diferença nx − a será estritamente
menor do que a diferença nx − (n − 1)x = x, uma vez que o (n − 1)x está mais à esquerda do
a. Porém existe o risco de esse (n − 1)x ser igual a a. Mas ainda podemos controlar que a
tomar. E, assim, por tentativa e erro, colocando certas perguntas e as respondendo, chegamos,
enm, a nossa f.

Nível 2.1.

d1
(n0 − 1)x

0 x+1 n0 x
d2 = x
Se (n0 − 1)x < x + 1, estamos feitos: dena f (x) = d1 .
112 CAPÍTULO 3. NÚMEROS REAIS

Tomemosx ∈ G+ irracional (eles existem, por exemplo, para m = 0). Seja n0 = min{n ∈
N; nx > x + 1} (podemos assim proceder por causa do princípio da boa ordem e da propriedade
arquimediana). Armamos que (n0 − 1)x < x + 1 (Nível 3.1), de modo que (n0 − 1)x − 1 < x,
além de (n0 − 1)x − 1 = n0 x − (x + 1) > 0, pois n0 x > x + 1. Assim, denimos:

f : X −→ X
x 7−→ f (x) = (n0 − 1)x − 1,

onde X = G+ ∩ (R − Q) e n0 = min{n ∈ N; nx > x + 1}. No Nível 3.2, vamos mostrar que


f tem todas as propriedades desejadas, além de, claro, estar bem denida (quer dizer, de fato
f (x) ∈ X ∀x ∈ X ).

No elevador. Conforme vamos aplicando sucessivamente f a si mesma vamos obtendo


elementos
 estritamente
 menores do que os anteriores, ou seja, vamos obtendo a sequência ··· <
 
f f f (x) < f f (x) < f (x) < x, qualquer que seja x ∈ X. Se essa sequência está se

aproximando n
cada vez mais do zero, tudo bem. Mas, pode ocorrer também de esses f (x)'s 

n+1
vamos denir f (x) = f f n (x)  carem sempre acima de um certo número positivo, e é
este caso que devemos tratar.

Nível 2.2: Lema. Se f :X →X é uma função tal que 0 < f (x) < x ∀x ∈ X ⊂ G+ ,
então inf G+ = 0.
Solução.

f n+1 (x) = f f n (x) ∀n ∈ N e f 1 (x) = f (x). Podem ocorrer apenas
Denotemos
duas coisas: ou, dado ε > 0, existe x ∈ X tal que x − f (x) < ε, ou, caso contrário, existe ε > 0
tal que x − f (x) > ε para todo x ∈ X .

>ε >ε
···
0 f 2 (x) f (x) x

ε > 0 tal que x − f (x) > ε para todo x ∈ X , então, para cada x ∈ X ,
Porém, se existe
existe n∈N f n (x) < 0 (Nível 3.3). Como f nunca assume valores negativos, podemos
tal que
concluir que, dado ε > 0, existe x ∈ X tal que 0 < x − f (x) < ε, onde x − f (x) ∈ G, porque G
+
é um grupo aditivo. Logo inf G = 0, como queríamos.

Nível 3.1. n0 = min{n ∈ N; nx > x+1}, para um dado x irracional, queremos mostrar
Se
que (n0 − 1)x < x + 1. Pela minimalidade de n0 , temos (n0 − 1)x 6 x + 1. Falta mostrar que
não pode ser (n0 − 1)x = x + 1. De fato (n0 − 1)x = x + 1 ⇒ (n0 − 2)x = 1 ⇒ x ∈ Q. Logo
(n0 − 1)x < x + 1.

Nível 3.2. Vamos mostrar que a f do Nível 2.1 possui todas as propriedades desejadas.
Lá já vimos o porquê de valer 0 < f (x) < x ∀x ∈ X . A única coisa que falta agora é ver que
f (x) ∈ X ∀x ∈ X . Mas isto é uma consequência imediata do Exercício 27 (página 81), uma
vez que x é irracional. Segue a tese.
3.58. EXERCÍCIO 58 113

Nível 3.3: Lema. Seja f : X → X uma função, onde X é um conjunto de números reais,
e suponha que existe ε > 0 tal que x − f (x) > ε para todo x. Então, para cada x ∈ X , existe
n ∈ N tal que f n (x) < 0.
Solução. Fixado x ∈ X, temos

x − f (x) > ε
f (x) − f 2 (x) > ε
f 2 (x) − f 3 (x) > ε
.
.
.

ou seja,
f (x) 6 x−ε
f 2 (x) 6 f (x) − ε 6 (x − ε) − ε = x − 2 ε
f 3 (x 6 f 2 (x) − ε = (x − 2 ε) − ε = x − 3 ε
.
.
.

Assim, de um modo geral, se f n (x) 6 x − n ε, então:

f n (x) − f (f n (x)) > ε ⇒ f n+1 (x) 6 f n (x) − ε 6 (x − n ε) − ε = x − (n + 1) ε,

f n (x) 6 x − n ε para todo natural n maior do que 1. Tomando então


o que, por indução, nos dá
n
um natural n maior do que 1 tal que n ε > x (propriedade arquimediana), temos: f (x) 6
x − n ε < 0, como queríamos. 

Observações. No Nível 3.1, existe um outro modo de mostrar que (n0 − 1)x 6= x + 1, veri-
cando que x + 1 6= nx ∀n ∈ N. De fato, seja x = a + bα, a, b ∈ Z, b 6= 0, já que x é irracional.
Desse modo:

x + 1 = nx ⇒ (a + 1) + bx = na + nbx ⇒ nb = b e a + 1 = na ⇒ n = 1 e a+1=a

(acontece essa injetividade na segunda implicação, pelo mesmo motivo que ela acontecia
no Exercício 28 página 81; veja a importância de ser b 6= 0, quer dizer, provavelmente essa
construção não funcionaria caso x não fosse irracional). Ter a + 1 = a é um absurdo. Logo
x + 1 6= nx ∀n ∈ N, em particular: x + 1 6= (n0 − 1)x (se n = 1 com maior razão, o que mostra
que isso não funciona para x = 1).
Também existe um outro modo de provarmos o Nível 2.2, ou seja, que f : X → X tal que
+
0 < f (x) < x ∀x ∈ X implica inf G = 0, mas agora usando que X é um grupo aditivo e
aplicando a contrapositiva do item ii) acima. Primeiro:

X ⊂ G+ ⇒ inf X > inf G+ .

Segundo: a f nos mostra que X não tem mínimo, ou seja, inf X ∈


/ X. Se X é um grupo aditivo,
então, pelo item ii):
inf X ∈
/ X ⇒ inf X 6 0.
Como X é de números positivos, segue inf X = 0. Assim:

inf X = 0 > inf G+ > 0 ⇒ inf G+ = 0,

como queríamos.
114 CAPÍTULO 3. NÚMEROS REAIS

Curiosidade. Uma função f tal que f (x) < x para todo x de seu domínio é chamada de
função regressiva.

3.59 Exercício 59
Sejam f, g ∈ R × R → R e ϕ, ψ : R × R × R → R as funções denidas por f (x, y) = 3x − y ,
g(x, y) = (x − 1)2 + (y + 1)2 − 9, ϕ(x, y, z) = 3z , ψ(x, y, z) = x2 + y 2 − z . Interpretando (x, y)
2
como as coordenadas cartesianas de um ponto do plano R e (x, y, z) como coordenadas de um
3 −1
ponto do espaço R , descreva geometricamente os conjuntos f (0), g −1 (0), ϕ−1 (0), ψ −1 (0).

Solução.
z
f −1
[{0}] ψ −1 [{0}]

g −1 [{0}]

y
ϕ−1 [{0}]
x
Geometricamente: f −1 [{0}]
é a reta determinada pelos pontos (0, 0) e (1, 3); g
−1
[{0}] é a
−1 −1
circunferência de raio 3 e com centro em (1, −1); ϕ [{0}] é o plano z = 0, e; ψ [{0}] é um
2
parabolóide, a rotação da parábola z = y em torno do eixo z . 

3.60 Exercício 60
Seja a
um número real positivo. Dado um número racional p/q (onde
√ p∈Z e q ∈ N), dena a
p/q
potência de base a e expoente racional p/q como a = q ap . Prove:
o
1 ) Para quaisquer r, s ∈ Q tem-se ar · as = ar+s e (ar )s = ar·s ;
o
2 ) Para todo r ∈ Q+ , a função f : (0, +∞) → (0, +∞), dada por f (x) = xr , é uma bijeção
crescente;

o
3 ) A função g : Q → R denida por g(r) = ar , (onde a é um número real positivo xado) é
crescente se a > 1, e decrescente se 0 < a < 1.

(Voltar para a Solução do Exercício 10 do Capítulo IV: página 123.)

(Voltar para o item (a) do Exercício 11 do Capítulo VI: página 214.)

(Voltar para o item (d) do Exercício 11 do Capítulo VI: página 216.)

Solução.
3.60. EXERCÍCIO 60 115

p1 p2
item 1o ) Sejam r= e s= . Temos:
q1 q2
p1 p p1 q2 +p2 q1
+ q2
ar+s = a q1 2 =a q1 q2
.
O número an é positivo qualquer que seja o inteiro n (an é um produto de números positivos
sen > 0 e inversos multiplicativos de números positivos são positivos). Portanto, pela denição
de raiz n-ésima (página 81 do livro)  para n = q1 q2 , basta mostrarmos que
 p1 p2 q1 q2
a q1 · a q2 = ap1 q2 +p2 q1 .
O expoente pode entrar em cada um dos fatores, isto é, (xy)n = xn y n para x, y ∈ R e
n ∈ Z. Deste modo, temos:
 p1 p2 q1 q2  p1 q1 q2  p2 q1 q2
a q1 · a q2 = a q1 · a q2
= ap1 q2 · ap2 q1
= ap1 q2 +p2 q1 .
Na segunda igualdade acima usamos que (am )n = amn se m é racional e n é natural. A mesma
demonstração do segundo item do Exercício 2 (página 66) se aplica a essa igualdade. Por ela
m n mn
podemos concluir mais, a saber, que (a ) = a se m é racional e n é inteiro, porque, naquele
1
item, a única hipótese que usamos do fato de m ser inteiro foi a de que a−x = qualquer que
ax
seja x ∈ Z. Para racionais acontece a mesma coisa:
p −p
a− q = aq

q
= qa−p
(ap )−1
q
=
1
= pq
(ap )
 p −1
= aq .
A demonstração de que (xy)n = xn y n para todo n ∈ Z e x, y reais também é semelhente às
demonstrações do Exercício 2 (página 66). Logo a · as = ar+s .
r

Dados agora p1 , p2 ∈ Z e q1 , q2 ∈ N, queremos mostrar que:


 p1  pq2 p1 p2
a q1 2 = a q1 q2 ,
ou seja, que
  p2 q1 q2
p1
q
a q1 2 = ap 1 p 2 .

Conforme já observamos, (am )n = amn para todo n ∈ Z, mesmo que m seja racional. Deste
modo, temos:

p1  pq2 q1 q2  p1  pq2 ·q1 q2
2 2
a q1
= a q1
 p1 p2 q1
= a q1

p1
·p q
= a q1 2 1
= ap1 ·p2 ,
como queríamos. 
116 CAPÍTULO 3. NÚMEROS REAIS

p
item 2o ) Seja r= , com p e q naturais. Pelo Exercício 11 (página 72), porque (0, +∞) é
q
o conjunto dos números positivos deR, temos 0 < x < y ⇒ 0 < xp < xp . Dados x e y positivos,
1 1
o Exercício 11 também nos permite concluir que x < y ⇒ x q < y q , pois esta implicação é a
1 1 1 1 1 1
q q
contrapositiva da implicação y q 6 x q ⇒ y = (y q ) 6 (x q ) = x (y q e x q são positivos por
denição). Portanto
p p
0 < x < y ⇒ 0 < xp < y p ⇒ 0 < x q < y q ,
logof é crescente e, em particular, é uma função injetiva. Ela é uma sobrejeção porque, dado
y > 0, o número x > 0 tal que f (x) = y é a raíz p-ésima de y q :
1 1 1
f (y r ) = (y r )r = y r ·r = y,

para todo y > 0, pelo item anterior, pois 1/r ∈ Q. 

item 3o ) Vamos começar supondo a > 1. Dados r, s ∈ Q, queremos mostrar que

r < s ⇒ ar < as .

Isto é, pela denição de desigualdade em R, queremos mostrar que o número

as − ar = ar · [as−r − 1]

é positivo, onde s − r é racional positivo. Por denição (as da página 81 e a deste exercício), o
ar é positivo (se r = p/q , com q ∈ N, então ar é a raiz q -ésima de ap , onde ap > 0 conforme já
x
observamos). Portanto, é sucente mostrar que a > 1 se x é um racional positivo.
x q p
Seja x = p/q , com p, q ∈ N. Por denição a é o único número positivo b tal que b = a .
q 1
Queremos concluir que b > 1, se a > 1. Por um lado, se b 6 1, então b 6 b = b 6 1 (pelo
p 1
Exercício 14, página 74). Por outro, se a > 1, então a > a = a > 1 (também pelo Exercício
14). Logo, se a > 1, só pode ser b > 1, como queríamos.
Se 0 < a < 1, então a−1 > 1. Agora, usando o resultado que acabamos de demonstrar, como
−1 x
(a ) = a−x = (ax )−1 ∀x ∈ Q (pelo primeiro item deste exercício), podemos fazer as seguintes
contas (para r, s ∈ Q)

r s
r < s ⇒ a−1 < a−1
⇔ (ar )−1 < (as )−1
⇒ ar > as ,

(Exercício 13, página 73). Segue a tese 


Capítulo 4
Sequências e Séries de Números Reais

4.1 Exercício 1
Se lim xn = a, então lim |xn | = |a|. Dê um contra-exemplo mostrando que a recíproca é falsa,
salvo quando a = 0.

(Voltar para a Solução do Exercício 37: página 150.)

(Voltar para a Primeira Parte da Solução do Exercício 15 do Capítulo VI: página 219.)

(Voltar para a Solução do Exercício 47 do Capítulo VIII, Nível 4.2: página 332.)

(Voltar para a Solução do Exercício 13 do Capítulo X, Nível 4.3: página 419.)

(Voltar para a observação 5 do Exercício 51 do Capítulo X: página 456.)

Solução. Seja ε > 0. Queremos um n0 ∈ N tal que

n > n0 ⇒ ||xn | − |a|| < ε .


Pelo item (iii) do Teorema 2 do Capítulo 3, temos ||xn | − |a|| 6 |xn − a|, para todo n ∈ N.
Portanto, um n0 que satisfaz a implicação acima é o mesmo que satisfaz n > n0 ⇒ |xn − a| < ε,
o qual existe porque lim xn = a,
por hipótese.
n
Quanto à recíproca, para a sequência ((−1) )n∈N  que não é convergente , temos lim |(−1)n | =
lim 1 = 1. Porém, se a = 0, então |xn − a| = |xn | = ||xn || = ||xn | − |a||, logo (xn )n∈N converge
para a se lim |xn | = |a|. 

4.2 Exercício 2
Seja lim xn = 0. Para cada n, ponha yn = min{|x1 |, |x2 |, . . . , |xn |}. Prove que yn → 0.

Solução. Temos 0 6 yn 6 |xn |, para todo n ∈ N, e lim |xn | = 0 (pelo exercício anterior),
logo, lim yn = 0, pelo Teorema 8. 

4.3 Exercício 3
Se lim x2n = a e lim x2n−1 = a, prove que lim xn = a.
(Voltar para a Solução do Exercício 17, Desigualdades estritas: página 130.)

(Voltar para a Solução do Exercício 44: página 153.)

117
118 CAPÍTULO 4. SEQUÊNCIAS E SÉRIES DE NÚMEROS REAIS

Solução. Seja ε > 0. Queremos encontrar um natural n0 tal que

n > n0 ⇒ |xn − a| < ε .

Por hipótese, sejam os naturais n1 e n2 tais que n > n1 ⇒ |x2n − a| < ε, e n > n2 ⇒
|x2n−1 − a| < ε. Equivalentemente,

2n > 2n1 ⇒ |x2n − a| < ε

e
2n − 1 > 2n2 − 1 ⇒ |x2n−1 − a| < ε .
Vamos vericar que o n0 procurado é n0 = max{2n1 , 2n2 − 1}. Dado n > n0 , temos dois
casos a considerar, ou n é par, ou n é ímpar. Se n = 2k , então

n > n0 ⇒ 2k > n0 ⇒ 2k > 2n1 ⇒ |x2k − a| < ε ⇒ |xn − a| < ε

e, se n = 2k − 1, então

n > n0 ⇒ 2k − 1 > n0 ⇒ 2k − 1 > 2n2 − 1 ⇒ |x2k−1 − a| < ε ⇒ |xn − a| < ε .

De qualquer forma, n > n0 ⇒ |xn − a| < ε, como queríamos. 

4.4 Exercício 4
Se N = N1 ∪ N2 ∪ · · · ∪ Nk e lim xn = lim xn = · · · = lim xn = a, então, lim xn = a. (Não
n∈N1 n∈N2 n∈Nk n∈N
reproduzi aqui o erro de tipograa do enunciado original.)

(Voltar para a Solução do Exercício 18: página 131.)

(Voltar para a Solução do Exercício 13 do Capítulo VII: página 237.)

(Voltar para a Solução do Exercício 27 do Capítulo VIII: página 306.)

Solução. Este exercício é uma generalização do anterior. Dado ε > 0, para cada i ∈
{1, . . . , k} := Ik , seja ni ∈ N tal que

n ∈ Ni , n > ni ⇒ |xn − a| < ε .

n0 = max{n1 , . . . , nk }. Dado um natural n maior do que n0 , como N = N1 ∪ · · · ∪ Nk ,


Seja
seja i ∈ Ik tal que n ∈ Ni . Em particular, n > n0 ⇒ n > ni . Portanto |xn − a| < ε, pois n ∈ Ni
e n > ni . Logo, n > n0 ⇒ |xn − a| < ε, ou seja, lim xn = a. 

4.5 Exercício 5
Dê exemplo de uma sequência (xn ) N = N1 ∪ N2 ∪ · · · ∪ Nk ∪ · · · de N
e uma decomposição
como reunião de uma innidade de subconjuntos innitos tais que, para todo k , a subsequência
(xn )n∈Nk tenha limite a, mas não se tem lim xn = a. [Sugestão. Para cada k ∈ N seja Nk o
k−1
conjunto dos números naturais da forma n = 2 · m, onde m é ímpar. Dado n ∈ Nk , ponha
1
xn = 1, se n for o menor elemento de Nk e xn = , nos demais casos.]
n
4.5. EXERCÍCIO 5 119

Solução 1.

No elevador. Veja como esse exercício se assemelha ao anterior. Mas, se são tão parecidos,
onde está a diferença entre eles que torna seus resultados tão distintos  e, de fato, opostos ?
No exercício anterior, a decomposição era nita, de modo que foi possível considerar o elemento
máximo de um outro conjunto nito. Aqui a decomposição é innita e agora não conseguiremos
tomar o máximo do modo como zemos no anterior. Aí está a diferença.
Minha ideia foi preencher a tabela abaixo com os elementos da forma 1/n do seguinte
modo: na primeira posição colocamos o 1; deixamos um espaço vazio e colocamos o 1/2, depois
deixamos dois espaços vazios e colocamos o 1/3, deixamos três espaços vazios e colocamos o
1/4 e assim por diante. Depois voltamos, o segundo espaço cou vazio, então preenchemos ele
com o 1, damos um pulo e colocamos o 1/2, dois e colocamos o 1/3 e assim por diante. Agora
é o quinto espaço que cou vazio, nele colocamos o 1 e agora damos dois pulos (e não 1) para
colocar o 1/2, depois três pulos e colocamos o 1/3 e assim vamos. A primeira diculdade foi
escrever isso em linguagem matemática. Com algum trabalho, surgiram as expressões desta
solução.
1 1 1 1 1 1 1 1 1 1 1 1 1
1 1 1 1 1
2 2 3 3 2 4 4 3 2 5 5 4 3

Nível 1. Com base no que foi exposto acima, surge a decomposição denida por

n2 + n n2 + n
   
N1 = ;n∈N e Ni+1 = + i; n ∈ N, n > i ∀i ∈ N
2 2
e as sequências denidas por
1
x n2 +n = ∀n ∈ N
2 n
e, xado i ∈ N,
1
x n2 +n +i = ∀n > i, n ∈ N .
2 n−i+1
Para cada k ∈ N, a subsequência (xn )n∈Nk converge a zero. Armamos que: essa sequência
está bem denida (demonstração no Nível 2.1) e que essas subsequências denem uma sequência
(xn ) (Nível 2.2) que não converge a zero (Nível 2.3).

No elevador. Para completar esta demonstração precisamos responder as perguntas abaixo.


Pergunta 1. Há ambiguidade nessa denição? De modo mais preciso, existem m e n
distintos (sem e n são iguais, então a resposta negativa para a última pergunta é mais imediata)
2
m +m n2 + n m2 + m n2 + n m2 + m
tais que = = a, +i = = b (para algum i 6 m) ou +i =
2 2 2 2 2
2
n +n
+ j = c (para algum i 6 m e algum j 6 n)? Nestes casos correríamos o risco de ter uma
2
1 1
contradição na denição de algum dos números xa , xb ou xc , pois teríamos xa = e xa = ,
m n
1 1 1 1
xb = e xb = , ou xc = e xc = . Por exemplo, se permitíssemos
m−i+1 n m−i+1 n−j+1
12 + 1 (−2)2 + (−2)
que o n se estendesse a todo número inteiro, como = = 1, o x1 seria igual
2 2
1 1
a 1 ou igual a =− ?
−2 2
120 CAPÍTULO 4. SEQUÊNCIAS E SÉRIES DE NÚMEROS REAIS

Pergunta 2. Temos todos os xn 's denidos para poder considerar a sequência (xn )n∈N toda,
ou cou faltando denir algum xn ?
Pergunta 3. Sendo a pergunta acima armativa, teremos o resultado desejado, ou seja,
que (xn ) não converge a zero?
Geometricamente, uma parte da Pergunta 1 signica o seguinte. Fixados j ∈ N ∪{0} e
2
n +n
n > j , n ∈ N, consideramos a reta horizontal y = + j e olhamos para as suas interseções
2
x2 + x
com as parábolas da forma + i, com i ∈ N ∪{0} distinto de j . Queremos ver que essas
2
interseções não ocorrem em pontos cujas abscissas são números naturais.

Acima da reta horizontal

n2 + n
+j
2

i j n n+1

Nível 2.1: Lema. Dados i, j ∈ N ∪{0} e os números naturais m>i e n > j, se m 6= n,


então
m2 + m n2 + n
+ i 6= + j.
2 2
Solução. Armamos que:

(n + 1)2 + (n + 1) n2 + n
> +j
2 2
(a demonstração disso está no Nível 3). Desse modo, temos (sempre usando que a função
2
x +x
denida por f (x) = para todo x∈N é crescente, o que também decorre do Nível 3 por
2
indução, para j = 0):

m2 + m (n + 1)2 + (n + 1) (n + 1)2 + (n + 1) n2 + n
m > n ⇒ m > n+1 ⇒ +i > +i > > +j.
2 2 2 2
Além disso, se i 6 j:
m2 + m m2 + m n2 + n
m<n⇒ +i6 +j < + j.
2 2 2
Se é j 6 i, a demonstração é análoga, bastando trocar as letras, no lugar do n colocamos o
m. Logo, de qualquer forma, segue o resultado desejado:

m2 + m n2 + n
+ i 6= +j
2 2
4.5. EXERCÍCIO 5 121

se m e n são números naturais distintos tais que m>i e n > j, com i, j ∈ N ∪{0}.
2 2
m +m n +n
m = n e se
Se a= +i = + j, então i=j e também não há ambiguidade na
2 2
denição de xa .

n2 + n
Nível 2.2: Lema. Dado um natural m, então existe um natural n tal que m=
2
n2 + n
ou existem naturais n e i, com n>i tais que m= + i.
2
Solução. Podemos proceder por indução em m. Para m = 1 o resultado é verdadeiro, o n
procurado é n = 1, pois:
12 + 1
1= .
2
n2 + n n2 + n
Dado m > 1, se m= para algum natural n, então m+1 = + 1, onde n > 1.
2 2
n2 + n n2 + n
Se m= + i, com n > i, então m+1= + i + 1. Se i = n, então:
2 2
n2 + n n2 + n (n + 1)2 + (n + 1)
m+1= +i+1= +n+1= .
2 2 2
Se i < n, então n > i + 1. De qualquer forma temos o resultado desejado.

Nível 2.3: Armação. A sequência construída no Nível 1 não converge para zero (pode
ser que ela não convirja para nenhum número ou convirja para algum outro número).
 
Solução. Basta considerar a subsequência x i2 +i +i
. Ela é constante e todos seus termos
i∈N 2
são iguais a 1. Se a sequência convergisse para zero, então todas suas subsequências também
deveriam convergir para zero.

Nível 3: Lema. Dados j ∈ N ∪{0} e um natural n > j, vale a desigualdade:

(n + 1)2 + (n + 1) n2 + n
> +j
2 2
Solução. Temos:

(n + 1)2 + (n + 1) n2 + n (n2 + n) + (2n + 2) n2 + n


> +j ⇔ > +j
2 2 2 2
n2 + n n2 + n
⇔ +n+1> +j
2 2
⇔ n + 1 > j.

Onde, justamente, n > j ⇒ n + 1 > j. Segue a tese. 

Solução 2. Consideremos a sugestão do exercício. A igualdade N = N1 ∪ N2 ∪ · · · ∪ Nk ∪ · · ·


vale pelo Exercício 22 do Capítulo II (página 54): a função f daquele exercício é sobrejetiva
e é tal que N1 = f [{1} × N] e Nk = f [{k} × N], para k ∈ N −{1}. Para cada k ∈ N, a
subsequência (xn )n∈Nk converge para zero, e o que justica a armação de que a sequência
(xn )n∈N não converge para zero é o Teorema 2, todos os termos da subsequência (x2k−1 )k∈N são
iguais a 1. 
122 CAPÍTULO 4. SEQUÊNCIAS E SÉRIES DE NÚMEROS REAIS

4.6 Exercício 6
Se lim xn = a e lim(xn − yn ) = 0 então lim yn é igual a a.
(Voltar para a Solução do Exercício 9 do Capítulo VIII: página 289.)

(Voltar para a Solução do Exercício 25 do Capítulo VIII: página 304.)

Solução. Pelo Teorema 6, como yn = (yn − xn ) + xn , e ambos os limites lim(yn − xn ) e lim xn


existem, sendo lim(yn − xn ) = − lim(xn − yn ) = 0 (Teorema 6 também), podemos somar:

a = 0 + a = lim(yn − xn ) + lim xn = lim yn . 

4.7 Exercício 7
yn
Seja a 6= 0. Se lim =1 então lim yn é igual a a.
a

yn  y 
n
Solução. Pelo Teorema 6: a = a · 1 = a · lim = lim a · = lim yn . 
a a

4.8 Exercício 8
xn a
Seja b 6= 0. Se lim xn = a e lim = b, então, lim yn = .
yn b

Solução. Pelo Teorema 6:


a xn
= lim xn = lim yn
b yn

(ver também a observação feita no parágrafo que precede o Teorema 6). 

4.9 Exercício 9
b
Se lim xn = a 6= 0 e lim xn yn = b então lim yn = .
a
(Voltar para a Observação do Exercício 12: página 125.)

Solução. Pelo Teorema 6:


b xn y n
= lim = lim yn .
a xn
(ver também a observação feita no parágrafo que precede o Teorema 6). 

4.10 Exercício 10

Sejam k∈N e a > 0. Se a 6 xn 6 n k para todo n, então lim n x = 1.
n
4.11. EXERCÍCIO 11 123

√ √ √
n √
Solução. Essas desigualdades implicam as desigualdades
n
a6 n x 6
n nk = ( n n)k (Exer-
o
cício 60 do Capítulo III, 2 item, página 114), onde, pelos Exemplos 13 e 14, sabemos que ambos
√ √ √
os limites lim
n
a e lim( n n)k são iguais a 1 (na verdade, para concluir que lim( n n)k = 1 pre-

cisamos da ajuda do Teorema 6 também). Portanto, pelo Teorema 8, segue lim n xn = 1. 

4.11 Exercício 11
Use a desigualdade entre as médias aritmética e geométrica dos n + 1 números 1 − 1/n, . . . , 1 −
1/n, 1 e prove que a sequência (1 − 1/n)n é crescente. Conclua que (1 − 1/n)n > 1/4 para todo
n > 1.
(Voltar para a Solução do Exercício 13, Primeira Parte: página 126.)

Solução. A média geométrica é menor do que ou é igual a média aritmética (Exercício 54 do


Capítulo III, página 100). Assim, temos
s n
n+1 1 n · (1 − n1 ) + 1 n
1− 6 = ,
n n+1 n+1
donde, elevando ambos os membros a n + 1, vem
 n  n+1
1 n
1− 6
n n+1
(Exercício 11 do Capítulo III, página 72), cujo lado direito é justamente o termo seguinte da
sequência:
1 (n + 1) − 1 n
1− = = .
n+1 n+1 n+1
A partir disso, por indução, podemos concluir que a sequência dada é nãodecrescente
 . n
n−1
Para concluir que ela é crescente, vejamos agora que não existe n∈N tal que =
n
 n+1
n
. Esta igualdade é equivalente a (n − 1)n · (n + 1)n+1 = nn · nn+1 , o que signica
n+1
que os conjuntos (  n  n+1 )
n−1 n
X= n ∈ N; =
n n+1
e
Y = n ∈ N; (n − 1)n · (n + 1)n+1 = nn · nn+1


n n+1
são iguais. Por outro lado, este é vazio, a paridade do número (n − 1) · (n + 1) é diferente
n n+1
da do número n · n ; se n é par, então n − 1 e n + 1 são ímpares e, se n é ímpar, então n − 1
e n + 1 são pares, além do que, produto de número ímpar por número ímpar é ímpar e produto
de número par por número par é par. Portanto, como X
 n  n+1 = Y e Y = ∅, temos que X = ∅, isto
n−1 n
é, não existe n ∈ N tal que = , como queríamos. Logo, a sequência é
n n+1
crescente.  n
1 1
Sendo crescente, para mostrar que 1− > para todo n > 1, basta vericar que esta
n 4
 2  2
1 1 1
desigualdade é válida para n = 2: 1− = = . 
2 2 4
124 CAPÍTULO 4. SEQUÊNCIAS E SÉRIES DE NÚMEROS REAIS

4.11.1 Exercício 11a

Sejam xn = (1 + 1/n)n e yn = (1 − 1/(n + 1))n+1 . Mostre que lim xn yn = 1 e deduza daí que
lim(1 − 1/n)n = e−1 .

Solução. Temos:

 n  n+1
1 1
xn y n = 1+ · 1−
n n+1
   n  
1 1 1
= 1+ · 1− · 1−
n n+1 n+1
   n  
n+1 n 1
= · · 1−
n n+1 n+1
1
= 1− .
n+1
Portanto:  
1
lim xn yn = lim 1 − = 1.
n+1
Sabemos que lim xn = e. Assim, pelo Teorema 6:

1 xn y n
= lim = lim yn ,
e xn
onde:  n
1
lim yn = lim 1 − .
n
 n
1
Logo, lim 1 − = e−1 . 
n
(Voltar para a Solução do Exercício 13, Segunda Parte: página 126.)

4.12 Exercício 12
k−1
X
Fazendo y=x 1/k
e b=a 1/k
na identidade
k k
y −b = (y −b)· y i bk−i−1 obtenha x−a = (x1/k −
i=0
k−1
X √ √
a1/k ) · xi/k · a1−(i+1)/k e use isto para provar que se lim xn = a > 0, então lim k
x n = k a.
n→∞
i=0
r r
Conclua, daí, que lim (xn ) = a para todo racional r.
n→∞

Solução.

Primeira Parte.

Nível 1. Fixado ε > 0, queremos encontrar um natural n0 tal que:

1/k
|xn − a1/k | < ε ∀n > n0 .
4.12. EXERCÍCIO 12 125

No elevador. Pela identidade mencionada no exercício, temos:

|xn − a| 1
|x1/k
n −a
1/k
| = k−1 = |xn − a| ·
k−1
.
X X
i/k 1−(i+1)/k i/k 1−(i+1)/k
x · a x · a

n n

i=0 i=0


Xk−1
Assim, se existir um certo M >0 tal que |xn − a| < M · ε e xi/k · a 1−(i+1)/k
> M, então

n

i=0
teremos o resultado desejado multiplicando membro a membro.

Nível 2. Para cada natural n∈N vamos denir

k−1
X
Sn = xi/k
n ·a
1−(i+1)/k
.
i=0

Como a sequência (xn ) converge para a > 0, existe n1 ∈ N tal que xn > a/2 para todo n > n1 .
A partir daí conseguimos limitar Sn por baixo também. Isto é, conseguimos obter um M
positivo tal que Sn > M para todo n > n1 . Para este M, existe um natural n2 tal que

|xn − a| < M · ε ∀n > n2 .

Para n0 = max{n1 , n2 }, temos

1 1
n > n0 ⇒ n > n1 ⇒ <
Sn M
e

n > n0 ⇒ n > n2 ⇒ |xn − a| < M · ε .


Portanto,
|xn − a|
n > n0 ⇒ < ε,
Sn
onde
|xn − a|
= |x1/k
n −a
1/k
|.
Sn
1/k
Logo, n > n0 ⇒ |xn − a1/k | < ε, como queríamos.

Segunda Parte. Se x n → a, já sabemos que xpn → ap , qualquer que seja o inteiro p


(aplicação do Teorema 6). E, com o resultado que acabamos de mostrar, se q ∈ N, então
p/q
(xpn )1/q → (a ) , ou seja, xn → ap/q . Segue a tese.
p 1/q


1/k 
Observação. 0 = lim(xn − a) = lim (xn − a1/k ) · Sn . O problema é
Até podemos escrever
1/k 1/k
 1/k  
que ainda não podemos escrever lim (xn − a ) · Sn = lim(xn − a1/k ) · lim Sn , porque
ainda não sabemos se cada um desses dois fatores existe; sem saber se a sequência (Sn ) converge
também não podemos aplicar Exercício 9 (página 122).
126 CAPÍTULO 4. SEQUÊNCIAS E SÉRIES DE NÚMEROS REAIS

4.13 Exercício 13
 r n
Prove que, para todo r ∈ Q, tem-se lim 1+ = er . [Sugestão : Pelo Exercício 11, basta
n→∞ n
p
considerar o caso em que r = é >0. Examine a subsequência onde n = p · m. Para esses
q
 qm p/q
 r n 1
valores de n, tem-se 1+ = 1+ . Use o Exercício 12.]
n qm

Solução.

Primeira Parte (racionais positivos). Vamos começar supondo r > 0 e escrevamos


 qm !
p 1
r= , com p e q naturais. Como q ∈ N, a sequência 1+ é uma subsequência
q qm
m∈N
 n !
1
da sequência 1+ . Portanto, pelo Exemplo 16 e pelo Teorema 2,
n
n∈N
 qm
1
lim 1 + = e.
m→∞ qm
 pm  qm r
r 1
Assim, pelo exercício anterior (pois e > 0) e pela igualdade 1+ = 1+ ,
pm qm
 pm
r
lim 1 + = er ,
m→∞ pm
 n !
r
e temos uma subsequência de 1+ convergindo para er . Tendo em mente agora
n
n∈N
o Teorema 4 e a Observação após o corolário do !Teorema 2, podemos concluir o resultado
 n
1
desejado vericando que a sequência 1+ é monótona. Podemos fazer isso do
n
n∈N
mesmo modo como no Exercício 11 (página 123), por indução e usando a desigualdade entre
as médias aritmética e geométrica (Exercício 54 do Capítulo III, página 100), uma vez que
1 + r/n > 0 n ∈ N:
para todo

 r
r
r n
 n 1 + +1 r  r n

r
n+1
n+1
1+ 6 n =1+ ⇒ 1+ 6 1+ .
n n+1 n+1 n n+1
Logo,
 r n
lim 1+ = er ,
n→∞ n
se r > 0.
p
Segunda Parte (racionais negativos). Continuando com o número racional r =
q
positivo, com p e q naturais, armamos que
 r n
lim 1− = e−r .
n→∞ n
4.14. EXERCÍCIO 14 127

Usando esta igualdade, se s é um número racional negativo, virá


 n
 s n −s
lim 1+ = lim 1 − = e−(−s) = es ,
n→∞ n n→∞ n
porque −s é um número racional positivo, e poderemos concluir este exercício.
Temos
 n  qn 1/q
 r n p p
1− = 1− = 1− ,
n qn qn
 qn !  n !
p p
onde 1− é uma subsequência de 1− . Deste modo, pelo Exercício
qn n
n∈N n∈N  p n
anterior e pelo Teorema 2, o problema se reduz agora a mostrar que lim 1− = e−p . Para
n→∞ n
isso, podemos proceder do mesmo modo como no Exercício 11a (página 124). Lá foi feita uma
n
translação à sequência dos (1 − 1/n) , de modo que o produto entre colchetes da segunda
igualdade de sua solução fosse igual a 1. Para que o mesmo aconteça aqui, a translação x que
devemos aplicar à nossa sequência deve ser tal que
 
 p p n+p n+x−p
1+ · 1− = · = 1;
n n+x n n+x
x=p satisfaz esta condição.
 n+p !
p
Consideramos então a sequência 1−  se seu limite existe, então o limite
n+p
n∈N
 n !
p
da sequência 1− também existe e é igual ao dela  e fazemos as contas abaixo:
n
n∈N
 n+p  p
 p n p p
lim 1+ · 1− = lim 1 − = 1.
n→∞ n n+p n→∞ n+p
Portanto,
 n+p
p
lim 1 − = e−p ,
n→∞ n+p
 p n
pois lim 1+ = ep , de acordo com o que já vimos na Primeira Parte desta solução. Logo,
n→∞ n
 p n
lim 1 − = e−p
n→∞ n
(observação após o Corolário do Teorema 2), como queríamos. Segue a tese. 

4.14 Exercício 14

n
Seja a > 0, b > 0. Prove que lim an + bn = max{a, b}.
n→∞

Solução. Esse resultado segue do Teorema 8. Suponha, sem perda de generalidade, que esse
máximo seja b. Assim:

n

n

n
√n
bn = b 6 an + bn 6 bn + bn = 2 · b.

n

n

Como 2 → 1, temos 2 · b → b. Portanto, pelo Teorema 8, n an + bn → b. 
128 CAPÍTULO 4. SEQUÊNCIAS E SÉRIES DE NÚMEROS REAIS

4.15 Exercício 15
Dada uma sequência (xn ), um termo xp chama-se um termo destacado quando xp > xn
para todo n > p. Seja P = {p ∈ N; xp é destacado}. Se P = {p1 < p2 < . . .} for innito,
(xp )p∈P é uma subsequência não-crescente de (xn ). Se P for nito (em particular, vazio),
mostre que existe uma subsequência crescente de (xn ). Conclua que toda sequência possui uma
subsequência monótona.

Solução. Como P n0 tal que xn ∈


é nito, existe um / P para todo n > n0 . O termo xn0 não é
destacado, então existe um n1 > n0 tal que xn1 > xn0 . O xn1 também não é destacado porque
n1 > n0 . Tendo denido o xnk , com nk > n0 , existe um nk+1 > nk tal que xnk+1 > xnk . Isso
nos dá o conjunto X = {nk ; k ∈ N} (se preferir, dena nk+1 = min{n ∈ N; n > nk , xn > xnk })
e a subsequência crescente (xn )n∈X .
Podemos agora concluir que toda sequência admite uma subsequência monótona porque
podem acontecer apenas duas coisas: ou o conjunto P é innito, ou é nito. 

4.16 Exercício 16
Seja (xn ) uma sequência limitada. Se lim an = a e cada an é um valor de aderência de (xn ),
então a é um valor de aderência de (xn ).

Solução. Vamos aplicar o Teorema 9. Em primeiro lugar vejamos que a é um número real.
Como (xn ) é limitada, sejam os reais α e α 6 xn 6 β , para todo n ∈ N. Isto implica
β tais que
α 6 an 6 β , para todo n ∈ N, o que, por sua vez, implica α 6 a 6 β (estas duas implicações
valem pelo Corolário 1 do Teorema 7). Logo, de fato, a ∈ R.
Dado ε > 0, vejamos agora que existem innitos termos de (xn ) no intervalo (a − ε, a + ε).
Pela convergência an → a, seja an ∈ (a − ε, a + ε). Como an é um valor de aderência da
sequência (xn ), pelo Teorema 9, existem innitos termos dela no intervalo (a − ε, a + ε), como
queríamos. Logo, pelo Teorema 9, a é um valor de aderência de (xn ). 
(Voltar para a Solução 1 do Exercício 10 do Capítulo V: página 163.)

Exemplo (necessidade da limitação da sequência para o resultado acima). Sejam


N1 e Nk = 2k−1 · N1 , para todo k ∈ N, (xn ) denida por xn = k se n ∈ Nk .
e seja a sequência
Então todo número natural é um valor de aderência de (xn ), mas lim n = +∞ não é sequer um
número real e, por denição, apenas números reais podem ser candidatos a valores de aderência.

4.17 Exercício 17
Sejam (xn ) e (yn ) sequências limitadas. Ponhamos a = lim inf xn , A = lim sup xn , b = lim inf yn
e B = lim sup yn . Prove que

a) lim sup(xn + yn ) 6 A + B, lim inf(xn + yn ) > a + b;


b) lim sup(−xn ) 6 −a, lim inf(−xn ) > −A;
c) lim sup(xn · yn ) 6 A · B e lim inf(xn · yn ) > ab;
valendo as duas últimas desigualdades sob a hipótese de xn > 0 e yn > 0. Dê exemplos em que
se tenham desigualdades estritas nas relações acima.

(Voltar para a Solução do Exercício 21 do Capítulo VI: página 224.)


4.17. EXERCÍCIO 17 129

Solução. Em todos os itens queremos mostrar que um certo número é maior do que ou igual
a um limite superior, ou menor do que ou igual a um limite inferior. A caracterização do
Teorema 11 vem a calhar, portanto. Tal teorema diz o seguinte: dada uma sequência limitada
(zn ), se C é um número com a propriedade  ∀ ε > 0 ∃n0 , n > n0 ⇒ zn < C + ε, então
C > lim sup zn . Algo análogo para o limite inferior: se c é um número com a propriedade
 ∀ ε > 0 ∃n0 , n > n0 ⇒ c − ε < zn , então c 6 lim inf zn . Podemos então mostrar que: no item
a), o A + B tem a propriedade do C e o a + b tem a propriedade do c para zn = xn + yn ; no
item b), o −a a do C e o −A a do c para zn = −xn , e; no item c), o AB a do C e o ab a do c
para zn = xn yn .

 ε  ε  ε
item a) Dado ε > 0, A+B+ε= A+
escreva + B+ e a+b−ε = a− +
2 2 2
 ε 
b− . Como a = lim inf xn , A = lim sup xn , b = lim inf yn e B = lim sup yn , existem naturais
2
n1 e n2 tais que
ε ε
n > n1 ⇒ a − < xn < A + ,
2 2
ε ε
n > n2 ⇒ b − < yn < B + .
2 2
Assim, tomando n0 = max{n1 , n2 }, temos:

n > n0 ⇒ a + b − ε < xn + yn < A + B + ε .

Portanto, pelo Teorema 11:

lim sup(xn + yn ) 6 A + B e lim inf(xn + yn ) > a + b. 

item b) Existe n0 tal que:

n > n0 ⇒ a − ε < xn < A + ε,

ou seja,
n > n0 ⇒ −A − ε < −xn < −a + ε .
Portanto: lim sup(−xn ) 6 −a e lim inf(−xn ) > −A. 

item c) Dado ε > 0, queremos um n0 tal que

n > n0 ⇒ xn yn 6 AB + ε .

A questão agora é como alcançar esse ε do mesmo modo como zemos nos itens anteriores.
0
Para um ε > 0, temos naturais n1 e n2 tais que

n > n1 ⇒ xn < A + ε0 e n > n 2 ⇒ y n < B + ε0 .

Assim, tomando n0 como o maior dentre n1 e n2 , temos

n > n0 ⇒ xn yn < (A + ε0 )(B + ε0 ) = AB + (A + B) ε0 + ε02 ,

pois A + ε0 e B + ε0
são ambos positivos (eles são maiores do que números não negativos).
0 0 02
Portanto, se conseguirmos encontrar ε > 0 tal que ε = (A + B) ε + ε , então o resultado
0
seguirá pelas contas acima. Por outro lado, sempre é possível encontrar um tal ε por causa da
130 CAPÍTULO 4. SEQUÊNCIAS E SÉRIES DE NÚMEROS REAIS

sobrejetividade da função dada por f (x) = (A + B)x + x2 para todo x > x0 sobre os números
2
positivos, onde x0 é a maior das raízes de (A + B)x + x = 0 (nem precisa usar que A e B são
não negativos!).
ε > 0, caso ambos a e b sejam positivos, consideramos x > 0
Para o limite inferior, dado
2 0 0
tal que f (x) = ε, onde agora f (x) = −(a + b)x + x e tomamos um 0 < ε < x tal que a − ε e
b − ε0 sejam positivos. Para um tal ε0 , encontramos n0 tal que
n > n0 ⇒ xn yn > (a − ε0 )(b − ε0 ) = ab + f (ε0 ) < ab + f (x) = ab + ε,
porque a f é crescente no intervalo (x0 , +∞), onde x0 é a maior de suas raízes. Se a for igual
a zero, então existe n0 tal que xn = 0 para todo n > n0 . 

Desigualdades estritas. O Exemplo 18 ajuda no item a). Dena:

1 1
x2n−1 = − , x2n = 1 + ,
n n
1 1
y2n−1 = 1 + , y2n = − ,
n n
Portanto xn + yn = 1 para todo n, enquanto 1 = lim sup xn = lim sup yn e 0 = lim inf xn =
lim inf yn , ou seja, A + B = 2 e a + b = 0.
No item b), após diversas tentativas frustradas, começamos a desconar que é impossível
termos as desigualdades estritas. E é o que acontece de fato. Qualquer que seja a sequência
limitada (xn ) já vimos que

lim sup(−xn ) 6 − lim inf(xn ) (4.1)

e
lim inf(−xn ) > − lim sup(xn ). (4.2)

Aplicando a desigualdade (2) à sequência (−xn ), temos

lim inf(xn ) = lim inf(−(−xn )) > − lim sup(−xn ),


ou seja (multiplicando ambos os membros por -1):

lim sup(−xn ) > − lim inf(xn ). (4.3)

Portanto, pelas desigualdades (1) e (3) segue a igualdade

lim sup(−xn ) = − lim inf(xn ).


E, de modo análago, temos também:

lim inf(−xn ) = − lim sup(xn ).


Alguma modicação no exemplo do item a) origina um exemplo para o item c). Dena:

1 1
x2n−1 = 3 + , x2n = 2 −,
n n
1 1
y2n−1 =2− , y2n = 3 + ,
n n
onde xn yn → 6, uma vez que ambas as subsequências x2n−1 y2n−1 e x2n y2n convergem para 6
(Exercício 3, página 117). Então, temos:

lim inf(xn )·lim inf(yn ) = 2·2 < 6 = lim inf(xn yn ) = lim sup(xn yn ) < 3·3 = lim sup(xn )·lim sup(yn ).
Este exemplo também serve para o item a).
4.18. EXERCÍCIO 18 131

Em azul, os pontos de xn ,
em vermelho os de yn


4.18 Exercício 18
Para cada n ∈ N, seja 0 6 tn 6 1. Se lim xn = lim yn = a, prove que lim[tn xn + (1 − tn )yn ] = a.
(Voltar para a Solução do Exercício 8 do Capítulo VIII: página 288.)

Solução. Vamos chamar de zn esse número tn xn + (1 − tn )yn . Um tal número aparece em


Geometria Analítica por exemplo, quando estudamos a forma de um ponto que pertence a um
segmento de reta. O zn sempre está entre xn e yn e, então, podemos aplicar o Teorema 8. Esta
é a ideia dessa solução.
Temos:
zn = tn xn + (1 − tn )yn = tn (xn − yn ) + yn .
Se xn − yn > 0, então:

0 6 tn 6 1 ⇒ 0 6 tn (xn − yn ) 6 xn − yn ⇒ yn 6 zn 6 xn .

Se xn − yn < 0, então:

0 6 tn 6 1 ⇒ xn − yn 6 tn (xn − yn ) 6 0 ⇒ xn 6 zn 6 yn .

Sejam então N1 = {n ∈ N; yn 6 zn 6 xn } e N2 = {n ∈ N; xn 6 zn 6 yn }. Se um desses dois


conjuntos for nito, então apenas uma aplicação do Teorema 8 conclui este exercício. Se ambos
são innitos, então consideramos as subsequências (xn )n∈N1 e (yn )n∈N2 . Ambas convergem para
a. Então, como yn 6 zn 6 xn ∀n ∈ N1 e xn 6 zn 6 yn ∀n ∈ N2 , ambas as subsequências
(zn )n∈N1 e (zn )n∈N2 convergem para a, pelo Teorema 8. A sequência (zn ) converge para a por
causa do Exercício 4 (página 118). 
(Voltar para a Solução do Exercício 51 do Capítulo VIII: página 335.)

4.19 Exercício 19
Diz-se que uma sequência (xn ) tem variação limitada quando a sequência (vn ) dada por vn =
Xn
|xi+1 − xi | é limitada. Prove que, nesse caso, (vn ) converge. Prove também:
i=1

a) Se (xn ) tem variação limitada, então existe lim xn ;

b) Se |xn+2 − xn+1 | 6 c|xn+1 − xn | para todo n∈N com 0 6 c < 1, então (xn ) tem variação
limitada;
132 CAPÍTULO 4. SEQUÊNCIAS E SÉRIES DE NÚMEROS REAIS

c) (xn ) tem variação limitada se, e somente se, xn = yn − zn onde (yn ) e (zn ) são sequências
não-decrescentes limitadas;

d) Dê exemplo de uma sequência convergente que não seja de variação limitada.

Solução. A sequência (vn ) é monótona:

n
X
vn+1 = |xi+1 − xi | + |xn+2 − xn+1 | = vn + |xn+2 − xn+1 | > vn .
i=1

Se ela também é limitada, então é convergente. 

item a)

No elevador. Como o conceito de variação limitada envolve módulos de diferenças dos


termos da sequência, talvez seja uma boa mostrar que (xn ) é uma sequência de Cauchy. Um
outro indício de que esse pode ser um bom caminho é a igualdade

|xn+2 − xn+1 | = vn+1 − vn ∀n ∈ N,

obtida do que foi feito acima.

Nível 1. Armamos que

|xm − xn | 6 vm−1 − vn−1 ∀n ∈ N ∀m > n

(a demonstração disso está no Nível 2). Se (xn ) tem variação limitada, então, como vimos, (vn )
converge, portanto é de Cauchy e a desigualdade acima implica que (xn ) é de Cauchy também,
portanto convergente.

Nível 2. Se m > n, escrevamos m = n + p. Vamos demonstrar a armação acima por


indução em p. Para p = 1, já vimos que:

|xn+1 − xn | = vn − vn−1 ,

para n > 2. Vamos supor agora que

|xn+p − xn | 6 vn+p−1 − vn−1 .

Então temos, com a ajuda da Desigualdade Triangular (item (i) do Teorema 2, Capítulo III):

|xn+p+1 − xn | = |xn+p+1 − xn+p + xn+p − xn | 6 |xn+p+1 − xn+p | + |xn+p − xn |


6 (vn+p − vn+p−1 ) + (vn+p−1 − vn−1 )
6 vn+p − vn−1 ,

como queríamos. 
4.19. EXERCÍCIO 19 133

item b) Este se assemelha ao Exemplo 19. Temos:

|xn+2 − xn+1 | 6 cn |x2 − x1 | ∀n ∈ N .

Portanto:

1
vn+1 6 (1 + c + c2 + · · · + cn ) · |x2 − x1 | < · |x2 − x1 | ∀n ∈ N,
1−c
esta última desigualdade valendo desde que c 6= 0. Se c = 0, então xn+2 = xn+1 ∀n ∈ N e
vn = |x2 − x1 | para todo n. Portanto, (vn ) é limitada de qualquer forma. 

item c)

Nível 1. Temos duas implicações a demonstrar: a volta (Nível 2.1) e a ida (Nível 2.2).

No elevador. Por exemplo:

v2 = |x3 − x2 | + |x2 − x1 | = |(y3 − y2 ) − (z3 − z2 )| + |(y2 − y1 ) − (z2 − z1 )|


6 |y3 − y2 | + |z3 − z2 | + |y2 − y1 | + |z2 − z1 |
= (y3 − y2 ) + (z3 − z2 ) + (y2 − y1 ) + (z2 − z1 )
= y3 + z3 − (y1 + z1 ).

É aí que aparece a hipótese de (yn ) e (zn ) serem não decrescentes, essa hipótese faz todas as
parcelas da segunda linha serem positivas. Isso que nos motiva a enunciar a armação do Nível
2.1.

Nível 2.1 (a volta). Armamos que:

vn 6 (yn+1 − y1 ) + (zn+1 − z1 ) ∀n ∈ N

(a demonstração disso está no Nível 3.1). Desse modo, sendo Y e Z tais que Y > |yn | e Z > |zn |
para todo n, temos:

|vn | = vn 6 (yn+1 −y1 )+(zn+1 −z1 ) = |yn+1 −y1 |+|zn+1 −z1 | 6 |yn |+|y1 |+|zn |+|z1 | 6 2Y +2Z,

para todo n. Portanto (vn ) é limitada.

No elevador. Lembrando que vn+1 − vn = |xn+2 − xn+1 |, temos

xn+2 = xn+1 + |xn+2 − xn+1 |


= (xn+1 + vn+1 ) − vn ,

se xn+2 > xn+1 e

xn+2 = xn+1 − |xn+2 − xn+1 |


= (xn+1 + vn ) − vn+1 ,

se xn+2 < xn+1 . É dessa observação que surge a denição abaixo.


134 CAPÍTULO 4. SEQUÊNCIAS E SÉRIES DE NÚMEROS REAIS

Nível 2.2 (a ida). Denimos:

yn+2 = xn+1 + vn+1 e zn+2 = vn ,

se xn+2 > xn+1 e

yn+2 = xn+1 + vn e zn+2 = vn+1 ,


se xn+2 < xn+1 .
Vamos demonstrar no Nível 3.2 que essas sequências (yn ) e (zn ) têm todas as propriedades
desejadas.

Nível 3.1: Lema. Se xn = yn − zn , onde (yn ) e (zn ) são sequências não decrescentes,
então

vn 6 (yn+1 − y1 ) + (zn+1 − z1 ) ∀n ∈ N .

Solução. Para n=1 temos:

v1 = |x2 − x1 |
= |(y2 − z2 ) − (y1 − z1 )|
= |(y2 − y1 ) − (z2 − z1 )|
6 |y2 − y1 | + |z2 − z1 |
= (y2 − y1 ) + (z2 − z1 ).

Agora, o passo seguinte:

vn+1 = vn + |xn+2 − xn+1 |


= vn + |(yn+2 − yn+1 ) − (zn+2 − zn+1 )|

(hipótese de indução) 6 (yn+1 − y1 ) + (zn+1 − z1 ) + |yn+2 − yn+1 | + |zn+2 − zn+1 |
= (yn+1 − y1 ) + (zn+1 − z1 ) + (yn+2 − yn+1 ) + (zn+2 − zn+1 )
= (yn+2 − y1 ) + (zn+2 − z1 ).

Portanto a armação é verdadeira para n + 1, se ela for verdadeira para n. Logo ela vale para
todo n.

Nível 3.2: Armação. As sequências denidas no Nível 2.2 são limitadas e não decres-
centes, sob a hipótese de (vn ) ser limitada.

Solução. Vale xn+2 = yn+2 − zn+2 pelas contas já feitas no elevador para chegar ao Nível
2.2. Que (zn ) é limitada segue da limitação de (vn ). Que (yn ) é limitada segue da limitação
de (vn ) e da limitação de (xn ) (pois variação limitada implica convergência, pelo item a), e
convergência implica limitação). Só falta vericar que yn+1 > yn e zn+1 > zn . Temos 4 casos a
considerar.

i) Se xn+2 > xn+1 e xn+1 > xn , então:

yn+2 = xn+1 + vn+1 , yn+1 = xn + vn ,

zn+2 = vn e zn+1 = vn−1 .


4.19. EXERCÍCIO 19 135

ii) Se xn+2 > xn+1 e xn+1 < xn , então:

yn+2 = xn+1 + vn+1 , yn+1 = xn + vn−1 ,

zn+2 = vn e zn+1 = vn .

iii) Se xn+2 < xn+1 e xn+1 > xn , então:

yn+2 = xn+1 + vn , yn+1 = xn + vn ,

zn+2 = vn+1 e zn+1 = vn−1 .

iv) Se xn+2 < xn+1 e xn+1 < xn , então:

yn+2 = xn+1 + vn , yn+1 = xn + vn−1 ,

zn+2 = vn+1 e zn+1 = vn .

De imediato dá para perceber que yn+2 > yn+1 e zn+2 > zn+1 em qualquer um desses
casos, com exceção talvez da sequência (yn ) no caso ii). Temos:

xn+1 +vn+1 > xn +vn−1 ⇔ vn+1 −vn−1 > xn −xn+1 ⇔ |xn+2 −xn+1 |+|xn+1 −xn | > xn −xn+1 .

Portanto, de fato, yn+2 > yn+1 se xn+2 > xn+1 e se xn+1 < xn . Segue a tese. 

item d)

−1/2
−1/4

x1 x3 x5 x4 x2

+1/3
+1

Vamos denir uma sequência de modo que a distância entre o n-ésimo e o (n + 1)-ésimo
termos seja 1/n:

1 1
x1 = 0, x2n = x2n−1 + e x2n+1 = x2n − ∀n ∈ N .
2n − 1 2n
1
Ela é de Cauchy, pois, dado , teremos xm , xn ∈ [a, b] se m, n > n0 , onde a = min{xn0 , xn0 +1 }
n0
e b = max{xn0 , xn0 +1 }. Além disso:

1 1 1
vn = |xn+1 − xn | + |xn − xn−1 | + · · · + |x3 − x2 | + |x2 − x1 | = + + ··· + + 1
n n−1 2
não dene uma sequência limitada conforme já vimos no Exemplo 23. 
136 CAPÍTULO 4. SEQUÊNCIAS E SÉRIES DE NÚMEROS REAIS

4.20 Exercício 20
1 1
Seja x1 = 1 e ponha xn+1 = 1 + . Verique que |xn+2 − xn+1 | 6 |xn+1 − xn |. Conclua que
xn 2
existe a = lim xn e determine a.

Solução.

Nível 1. Temos:

1 1 |xn − xn+1 |
|xn+2 − xn+1 | =
− = .
xn+1 xn |xn xn+1 |

Armamos agora que |xn xn+1 | > 2 para todo n (Nível 2). Assim sendo:

|xn − xn+1 | 1
|xn+2 − xn+1 | = 6 · |xn − xn+1 |,
|xn xn+1 | 2
como queríamos.

Nível 2. Temos:  
1
xn xn+1 = xn 1 + = 1 + xn .
xn
Armamos que xn > 1 para todo n (Nível 3). Assim sendo:

xn xn+1 = 1 + xn > 2,

como queríamos.

Nível 3. Vamos demonstrar que xn > 1 n. Para n = 1, xn = 1. Supondo


por indução em
1
xn > 1, em particular xn é positivo e, portanto, xn+1 = 1 + > 1, como queríamos.
xn
1
Portanto, pelo Exemplo 19, o limite a = lim xn existe. Ele é tal que a = 1 + . Além de
a
ser também a > 1, uma vez que xn > 1 para todo n, conforme já vimos. Da equação que a
2
satisfaz obtemos a − a − 1 = 0. E, daí, como a > 1, vem:

1+ 5
a= . 
2

Observação/Curiosidade. O número a obtido acima é conhecido como número de ouro,


razão de ouro ou razão áurea. Este exercício nos dá então uma forma de obtermos aproximações
para um tal número.

4.21 Exercício 21

Ponha x1 = 1 e dena xn+1 = 1 + xn . Mostre que a sequência (xn ), assim obtida, é limitada.
Determine a = lim xn .

Solução.
4.21. EXERCÍCIO 21 137

No elevador. Num primeiro momento, conforme o exercício pede, queremos mostrar que
a sequência é limitada. Para isso, podemos fazer um chute, tentar mostrar, por exemplo, que
todos seus termos são menores do que 3, o que de fato funciona. Porém, num segundo momento,
queremos mostrar a existência do limite. Como temos sua limitação, uma tentativa natural é
mostrar que a sequência é monótona e, mais precisamente, não decrescente, após vermos quem
são seus primeiros termos. Num terceiro momento vemos que, ao mostrar a monotonicidade,
a limitação também sai como subproduto. Naturalmente somos levados a trabalhar com a
2
expressão x − 3x + 1, pois, se os envolvidos são todos positivos:


1+ xn > xn ⇒ xn > (xn − 1)2 = x2n − 2xn + 1 ⇒ x2n − 3xn + 1 6 0.

Nível 1. Armamos que

√ √
3− 5 3+ 5
6 xn 6 ∀n ∈ N
2 2
(a demonstração disso está no Nível 2). Essas desigualdades implicam:


0 > x2n − 3xn + 1 = (xn − 1)2 − xn ⇒ (xn − 1)2 6 xn ⇒ xn 6 1 + xn = xn+1 .

A última implicação vale porque xn > 0 e xn − 1 > 0 para todo n. Portanto essa sequência é
monótona. É também limitada pela primeira armação feita.

Nível 2. Vamos demonstrar a armação acima por indução em n. Para n = 1, temos:


√ √ 3− 5
5>1⇒3− 5<2⇒ < 1,
2
e √
3+ 5 2
> = 1.
2 2
Portanto a armação é verdadeira para n = 1. Supondo que vale para n, extraindo a raiz
quadrada e somando 1, vem:

s √ s √
3− 5 3+ 5
1+ 6 xn+1 6 1 + .
2 2

Armamos que está tudo bem, quer dizer, que valem:

√ s √
3− 5 3− 5
61+
2 2
e s √ √
3+ 5 3+ 5
1+ 6
2 2
(as demonstrações dessas desigualdades estão nos níveis 3.1 e 3.2 respectivamente). Dessas
desigualdades seguem o resultado desejado.
138 CAPÍTULO 4. SEQUÊNCIAS E SÉRIES DE NÚMEROS REAIS


3− 5
Nível 3.1. Escrevamos c = . Temos:
2
√ √
0 = c2 − 3c + 1 = (c − 1)2 − c ⇒ (c − 1)2 = c ⇒ c − 1 = − c ⇒ c = − c + 1,
onde o sinal de menos na segunda implicação aparece porque o número
√ c−1 é negativo.
Portanto, somando 2 c a ambos os membros da última igualdade:
√ √
c + 2 c = c + 1,
√ √
onde c + 2 c > c. Logo, de fato, c < c + 1 como queríamos para a primeira desigualdade.


3+ 5
Nível 3.2. Escrevamos d = . Do mesmo modo como zemos no nível acima, como
2
2 2
d é raiz da equação x − 3x + 1 = 0, temos
√ (d − 1) = d. Porém, √ agora, o número d − 1 é um
número positivo, o que nos dá d − 1 = d, ou seja, d = 1 + d. Em outras palavras, vale a
igualdade:
s √ √
3+ 5 3+ 5
1+ = .
2 2
Segue a tese.

Para concluir, precisamos encontrar o limite da sequência. Veja como obtemos c+2 c 6
xn 6 d para todo n já a partir do segundo termo. Quer
√ dizer, vimos que a sequência se afasta
do c. Porém vimos também que d é tal que d = d + 1. O√a deve satisfazer a igualdade
√ 3+ 5
a = a + 1 e apenas um número a satisfaz. Logo a = d = . 
2
x2 − 3x + 1

c xn d


3− 5 √
Perguntas. Vimos que o número c= é tal que c = − c + 1. Se a sequência fosse
√ 2
então denida por xn+1 = 1 − xn , seu limite seria c? Outra pergunta, alterar o valor inicial,
traria muita perturbação à sequência? Ela poderia se aproximar mais rapidamente ou mais
demoradamente para d, ou mesmo deixar de convergir, principalmente se o x1 estiver fora do
intervalo [c, d]?
Em vista desse último exercício, podemos ver que a sequência do exercício 20 poderia ser
2
denida de outra forma. Por exemplo, vimos que o a (do exercício 20) é tal que a − a − 1 = 0.

Porém, esse
√ a também satisfaz a = a + 1. Assim poderíamos (talvez) ter denido xn+1 =
1 + xn lá. Existe alguma vantagem em denir uma sequência de uma ou outra maneira?
Alguma delas vai mais rapidamente para a? Como medir essa velocidade?

4.22 Exercício 22
A m de que a sequência (xn ) não possua subsequência convergente é necessário e suciente
que lim |xn | = +∞.
(Voltar para o item (b) da Solução do Exercício 48 do Capítulo V: página 190.)
4.23. EXERCÍCIO 23 139

Solução. Vamos mostrar a contrapositiva disso, ou seja, que(xn ) possui subsequência conver-
gente se, e somente se, (|xn |)n∈N não tende para o innito; (|xn |)n∈N não tender para o innito
signica que existe um M > 0 tal que, para todo n ∈ N, existe um natural m > n tal que
|xm | < M .
Se (xn ) (xni )i∈N . Sendo convergente, ela é
possui uma subsequência convergente, seja ela
limitada, portanto, existe um M > 0 tal que |xni | < M para todo i ∈ N. Assim, dado n ∈ N,
tome um i ∈ N tal que ni > n e teremos |xm | < M para m = ni , como queríamos.
Se |xn | não tende para o innito, seja M > 0 tal que, para todo n ∈ N, exista m > n tal
que |xm | < M . Esta propriedade nos dá uma subsequência de (xn )n∈N limitada pelo M . De
fato, existe n1 > 1 tal que |xn1 | < M e, se ni é tal que |xni | < M , tome ni+1 > ni tal que
|xni+1 | < M . Toda sequência limitada admite uma subsequência convergente ( Apêndice ao Ÿ4
[do Capítulo IV], página 125 do livro), portanto (xni ) admite uma subsequência convergente.
Esta subsequência convergente de (xni ) é também uma subsequência de (xn ). Logo, (xn ) possui
uma subsequência convergente. 

4.23 Exercício 23
Seja ϕ:N→N uma sequência de números naturais. Prove que as seguintes armações são
equivalentes:

a) lim ϕ(n) = +∞;


n→∞

b) Para todo k ∈ N, ϕ−1 (k) é um subconjunto nito de N;

c) Para todo subconjunto nito F ⊂ N, ϕ−1 (F ) é nito.

Em particular, se ϕ:N→N for injetiva, então lim ϕ(n) = +∞.


n→∞

Solução. O a) é equivalente ao b) como consequência do exercício anterior, pois uma sequência


(xn ) de números naturais converge se, e somente se, existem k ∈ N e n0 tal que xn = k para
todo n > n0 . Se ϕ−1 [{k}] é nito para todo k ∈ N, então nenhuma subsequência de ϕ tem esta
propriedade que a faz convergir e, reciprocamente, se lim ϕ(n) = +∞, então vale b).
O b) implica c), pois:
[
ϕ−1 [F ] = ϕ−1 [{x}],
x∈F

uma reunião nita de conjuntos nitos se F é nito e se vale b). O c) implica b) porque, dado
k ∈ N, o conjunto F = {k} é nito.
Se ϕ é injetiva, entãoϕ−1 [{n}] é um conjunto unitário para todo n ∈ N, em particular,
nito. Logo, lim ϕ(n) = +∞. 
n→∞

4.24 Exercício 24
Seja (xn ) uma sequência de números reais e suponha queϕ : N → N cumpre uma das (e
portanto todas as) condições do exercício anterior. Prove que se lim xn = a e yn = xϕ(n) , então
lim yn = a. Dê exemplo de ϕ : N → N sobrejetiva, tal que lim xn = a, mas não vale lim yn = a,
onde yn = xϕ(n) .

(Voltar para o Exercício 24 do Capítulo X, para a Observação de seu Nível 2.1: página 426.)
140 CAPÍTULO 4. SEQUÊNCIAS E SÉRIES DE NÚMEROS REAIS

Solução.

Primeira Parte. Dado ε > 0, queremos n0 tal que

n > n0 ⇒ |xϕ(n) − a| < ε .


Como x n → a, existe n1 tal que

n > n1 ⇒ |xn − a| < ε .


Como ϕ(n) → +∞ por hipótese, existe n2 tal que ϕ(n) > n1 para todo n > n2 . Deste
modo:
n > n2 ⇒ ϕ(n) > n1 ⇒ |xϕ(n) − a| < ε,
como queríamos (o n2 é o n0 procurado).

Segunda Parte. Vamos ferir alguma das hipóteses do exercício anterior, por exemplo,
vamos obter ϕ tal que ϕ−1 [{1}] seja um conjunto innito, sem perder de vista a sobrejetividade.
1
Então façamos ϕ(2n − 1) = 1 e ϕ(2n) = n para todo n ∈ N. E xn = , de modo que:
n
1
y2n−1 = x1 = 1 e y2n = xn =
n
para todo n, com xn → 0; esta yn sequer converge.

4.25 Exercício 25

xn+1
Seja xn 6= 0 para todo n ∈ N. Se existirem n0 ∈ N e c ∈ R tais que 0 < 6 c < 1
xn
xn+1
para todo n > n0 , então lim xn = 0. Se porém
xn > c > 1
para todo n > n0 , então

n!
lim |xn | = +∞. Como aplicação, reobtenha os Exemplos 21 e 22 e mostre que lim = 0.
nn
(Voltar para a Solução do Exercício 38: página 150.)


xn+1
Solução. Se 0<
6c para todo n > n0 então 0 < |xn0 +p | 6 cp · |xn0 | para todo p ∈ N.
xn
p
Como c |xn0 | tende a zero conforme p cresce, o resultado segue do Teorema 8. De modo análogo,
xn+1 p p
xn > c para n > n0 implica |xn0 +p | > c · |xn0 | para todo p ∈ N; assim c |xn0 | → +∞ faz

|xn0 +p | → +∞ também.

an
Exemplo 21. Para a > 1, queremos ver que lim= +∞. Temos:
n→∞ n

an+1 n n
· n = · a.
n+1 a n+1
Este número tende para a, que é um número maior do que um. Fixemos então um real c, com
n
1 < c < a. Existe n0 tal que ·a>c para todo n > n0 . Segue a tese. 
n+1
4.26. EXERCÍCIO 26 141

n!
Exemplo 22. Para a > 0, queremos ver que lim = +∞. Como
an
(n + 1)! an n+1
· = → +∞,
an+1 n a
n+1
qualquer c>1 serve. Por exemplo, para c=2 existe n0 tal que >2 para todo n > n0 e
a
segue a tese. 

Última Parte. Por m, temos

n
(n + 1)! nn

n 1 1 1
· = = n+1 n
 =  → .
1 n
(n + 1)n+1 n! n+1 n
1+ n e
 n
n
Como 1/e < 1, seja um real c tal que 1/e < c < 1. Existe n0 tal que <c para
n+1
todo n > n0 . Segue a tese. 

4.26 Exercício 26
Seja T um arranjo triangular de números não-negativos,

t11
t21 t22
t31 t32 t33
.. .. .. ... ..
tn1 tn2 tn3 · · · tnn
.. .. .. ... ..

Faça duas hipóteses sobre o arranjo T . Primeira: cada linha tem soma igual a 1. Segunda:
cada coluna tem limite zero: lim tni = 0 para todo i ∈ N. Dada uma sequência convergente
n→∞
(xn ), com lim xn = a, use o arranjo T para transformá-la numa sequência (yn ), com

yn = tn1 x1 + tn2 x2 + · · · + tnn xn .

Prove que lim yn = a.


[ Sugestão : Considere inicialmente o casoa = 0. Dado ε > 0, existe p ∈ N tal que n >
p ⇒ |xn | < ε /2. Existe também A > 0, tal que |xn | < A para todo n. Em seguida, obtenha
ε
q ∈ N, tal que n > q ⇒ |tn1 | < δ, |tn2 | < δ, . . . , |tnp | < δ , onde δ = . Tome n0 = max{p, q}.
2pA
Observe que n > n0 ⇒ |yn | 6 tn1 |x1 | + · · · + tnp |xp | + · · · + tnn |xn |, onde a soma das p primeiras
ε
parcelas não excede ε /2 e a soma das n − p parcelas restantes não supera (tn,p+1 + · · · + tnn ) .
2
Logo, |yn | < ε. O caso geral reduz-se imediatamente a este.]

(Voltar para a Solução do Exercício 29: página 144.)

Solução. Essa sugestão já é basicamente a solução toda. Começamos supondo a = 0. Fixado


ε > 0, seja p tal que:
ε
n > p ⇒ |xn | < .
2
142 CAPÍTULO 4. SEQUÊNCIAS E SÉRIES DE NÚMEROS REAIS

Seja A tal que |xn | < A para todo n e seja q tal que

ε
n > q ⇒ |tni | < ,
2pA
para i ∈ {1, 2, . . . , p} (este q existe). Tomando n0 = max{p, q}, se n > n0 , então:
 
|yn | 6 tn1 |x1 | + · · · + tnp |xp | + tn,p+1 |xp+1 | + · · · + tnn |xn |
ε  ε
6 · A · p + tn,p+1 + · · · + tnn ·
2pA 2
ε ε
6 +
2 2
= ε,

pois tn,p+1 + · · · + tnn 6 1. Portanto, lim yn = 0, como queríamos.


No caso geral, se lim xn = 0, então lim(xn − a) = 0, de modo que, pelo que acabamos de
ver, a sequência (zn ), denida por zn = tn1 (x1 − a) + tn2 (x2 − a) + · · · + tnn (xn − a), tende para
zero, onde:

zn = tn1 x1 + · · · + tnn xn − a(tn1 + · · · + tnn ) = yn − a,
yn como acima, lembrando que a soma em cada linha é igual a um. Assim, 0 = lim zn =
lim(yn − a). Logo, lim yn = a, como queríamos. 

4.27 Exercício 27
x1 + · · · + xn
Se lim xn = a, pondo yn = , tem-se ainda lim yn = a. (Sugestão : Use o exercício
n
anterior.)

(Voltar para a Solução do Exercício 39: página 151.)

Solução. Isso é imediato do exercício anterior. Essa sequência (yn ) é obtida a partir do
1
arranjo denido por tni = para todo n ∈ N e para todo i ∈ {1, . . . , n}. Para um tal arranjo
n
de fato temos tn1 + · · · + tnn = 1 para todo n e lim tni = 0 para todo i. 
n→∞

4.28 Exercício 28

Se lim xn = a, e os xn são todos positivos, então lim x1 x2 · · · xn = a. [Sugestão :
n
Tome
an+1
logaritmos e reduza ao problema anterior.] Conclua que se an > 0 e lim = a então
√ an
lim n an = a.
(Voltar para a Solução do Exercício 33: página 147.)

(Voltar para a Solução do Exercício 34: página 148.)

Solução.


Nível 1.1. yn = n x1 x2 · · · xn , o que faz log yn = log x1 +···+log
Seguindo a sugestão, seja
n
xn
.
Armamos agora que lim log xn = log a, se a 6= 0 (Nível 2.1). Trataremos o caso a = 0 no Nível
1.2. Deste modo, se a > 0, então, pelo exerício anterior, temos lim log yn = log a. Armamos
que isso implica lim yn = a (Nível 2.2), como queríamos.
4.28. EXERCÍCIO 28 143

Nìvel 1.2. Supondo a = 0, xado ε > 0, queremos n0 tal que


n > n0 ⇒ n x1 · · · xn < ε .

Vamos obter um tal n0 no Nível 2.3.

Nível 2.1: Lema. lim xn = a > 0, com xn > 0 para todo n, então lim log xn = log a.
Se

Solução. Queremos obter n0 tal que | log xn −log a| < ε para todo n > n0 , o que é equivalente
a
xn
− ε < log <ε ∀n > n0 .
a
A B
A função log é sobrejetiva, então existem A e B positivos tais que log = − ε e log = ε,
a a
onde A < a e B > a. Como xn tende para a, existe n0 tal que A < xn < B para todo n > n0 .
Assim, como log também é uma função crescente, se n > n0 , então:

A xn B A xn B xn
A < xn < B ⇒ < < ⇒ log < log < log ⇒ − ε < log < ε,
a a a a a a a
como queríamos.

Nìvel 2.2: Lema. Se lim log yn = log a (com yn > 0 e a > 0), então lim yn = a.
Solução.
De modo análogo ao que zemos no lema acima, demonstramos que o mesmo vale
x a
para função exponencial, ou seja, que lim xn = a ⇒ lim e n = e . Deste modo, temos:

lim log yn = log a ⇒ lim elog yn = elog a ⇒ lim yn = a,

como queríamos.

No elevador. Como xn
tende a zero, sabemos que existe um n1 tal que xn < ε para todo
√ p
n> n1 . Assim, temos: xn1 +1 · · · xn1 +p < εp e n1 +p xn1 +1 · · · xn1 +p < ε n1 +p para todo p natural,
p p
onde ε n1 +p tende a ε conforme p cresce, pois tende a 1. Assim, existe um p1 tal que
n1 + p
p 3ε √
n1 +p x · · · x
ε n1 +p < para todo p > p1 . O número 1 n1 tende a 1 conforme p cresce, como já
2
sabemos. Porém, apareceu aí o número 3 ε /2 quando queríamos na verdade que aparecesse o
ε. Podemos corrigir isso fazendo cada xn car menor do que ε /2, por exemplo (ou menor do
que qualquer número menor do que ε).

ε
Nível 2.3. Supondo a = 0, seja n1 tal que xn < para todo n > n1 . Portanto:
2
√  ε  n p+p
1
n1 +pxn1 +1 · · · xn1 +p < .
2
 ε  n p+p
1
para todo p natural. Seja p1 tal que <ε para todo p > p1 . Então, para este p1 temos:
2

p > p1 ⇒ n1 +p xn1 +1 · · · xn1 +p < ε .

Seja também p2 tal que:



p > p2 ⇒ n1 +p
x1 · · · xn1 < 1.
144 CAPÍTULO 4. SEQUÊNCIAS E SÉRIES DE NÚMEROS REAIS

Assim, se p é, ao mesmo tempo, maior do que p1 e maior do que p2 , então:



n1 +p x · · · x
√ √
1 n1 +p =
n1 +p
x1 · · · xn1 · n1 +p xn1 +1 · · · xn1 +p < ε .
Seja então p0 = max{p1 , p2 } e n0 = n1 +p0 . Se n > n0 , então existe p tal que n = n1 +p0 +p,
onde p + p0 é maior do que ambos p1 e p2 . Deste modo:

n > n0 ⇒ n x1 · · · xn < ε,
como queríamos.

Observação . O caso geral não se reduz de forma tão imediata ao casoa = 0 do mesmo
modo como acontecia no Exercício 26, pois: lim xn = a ⇒ lim(xn − a) = 0, e isso implica
p p √
lim n (x1 − a) · · · (xn − a) = 0, onde, de n (x1 − a) · · · (xn − a), queremos extrair o n x1 · · · xn .

Segunda Parte. É só escrever:


r
an an−1 a2 a1
n
an = n
· ··· ·
an−1 an−2 a1 1
e aplicar a primeira parte desse exercício. 

4.29 Exercício 29
P xn x1 + x2 + · · · + xn
Seja yn > 0 para todo n ∈ N, com yn = +∞. Se lim = a então lim = a.
yn y1 + y2 + · · · + yn

Solução. Algo parecido com o que foi feito no Exercício 26 (página 141) pode ser feito aqui.
Xn
Vamos começar supondo a = 0 e, para simplicar a notação, vamos escrever Yn = yi .
i=1
Temos:    
x1 + · · · + xn |x 1 | |x p | |x p+1 | |x n |
6 + ··· + + + ··· + .
Yn Yn Yn Yn Yn
xi
Conseguimos fazer aparecer os números , os quais sabemos como se comportam, na segunda
yi
parcela acima, e também conseguimos controlar o tamanho da primeira parcela, pelo fato de
Yn crescer acima de qualquer limite. Assim, escrevemos:
   
|x1 | |xp | |xp+1 | |xn |
+ ··· + + + ··· + =
Yn Yn Yn Yn
   
|x1 | |xp | yp+1 |xp+1 | yn |xn |
= + ··· + + · + ··· + ·
Yn Yn Yn yp+1 Yn yn
Pp
i=1 |xi | yp+1 |xp+1 | yn |xn |
= + · + ··· + · .
Yn Yn yp+1 Yn y n

Escrever desse modo prepara o terreno para o que devemos fazer em seguida. Fixado um
|xi | ε
ε > 0, seja p tal que < para todo i > p. Seja n0 tal que
yi 2
p
2 X
n > n0 ⇒ Yn > · |xi |,
ε i=1
4.30. EXERCÍCIO 30 145

ou seja, Pp
i=1 |xi | ε
n > n0 ⇒ < .
Yn 2
Assim, se n > max{n0 , p}, então
     
|x1 | |xp | |xp+1 | |xn | ε yp+1 yn ε
+ ··· + + + ··· + 6 + + ··· + · 6 ε,
Yn Yn Yn Yn 2 Yn Yn 2
pois
yp+1 yn yp+1 + · · · + yn
+ ··· + = 6 1.
Yn Yn y1 + · · · yp + yp+1 + · · · + yn
x1 + · · · + xn xn xn xn − ayn
Logo lim = 0, se lim = 0. Agora, se lim = a, então lim = 0.
y1 + · · · + yn yn yn yn
Assim, pelo que acabamos de ver, temos:
 
(x1 − ay1 ) + · · · + (xn − ayn ) x1 + · · · + xn
0 = lim = lim −a .
y1 + · · · + yn y1 + · · · yn
x1 + · · · + xn
Logo, lim = a, como queríamos. 
y1 + · · · yn

4.30 Exercício 30
xn+1 − xn xn
Se (yn ) é crescente e lim yn = +∞, então lim = a ⇒ lim = a. (Use o Exercício
yn+1 − yn yn
29.)

Solução. Essa implicação vale como consequência do exercício anterior porque

xn (xn − xn1 ) + (xn−1 − xn−2 ) + · · · + (x2 − x1 ) + (x1 − 0)


= ,
yn (yn − yn1 ) + (yn−1 − yn−2 ) + · · · + (y2 − y1 ) + (y1 − 0)
Pn
onde o somatório i=1 (yi − yi−1 ) = yn , com y0 = 0, tende ao innito por hipótese, e é de
parcelas positivas (pois yn é crescente). 

4.31 Exercício 31
1p + 2p + · · · + np 1
lim p+1
= . (Use o exercício anterior.)
n→∞ n p+1

Solução. Pelo exercício anterior, identicamos: xn = 1p + · · · np e yn = np+1 , onde yn é


crescente e tende para o innito. Assim temos

np + p−1 p
1 + p−1 p
P  i P  i−p
xn+1 − xn (n + 1)p i=0 n n
= = p+1 Pp−1 i p+1 i = p+1
 Pi=0 i
p−1 p+1 i−p .
yn+1 − yn (n + 1)p+1 − np+1

p
n + i=0 i n + i=0 i n
p p

Conforme n cresce este número tende ao número

1 1
p+1
= .
p
p+1

Agora o resultado segue pelo exercício anterior. 


146 CAPÍTULO 4. SEQUÊNCIAS E SÉRIES DE NÚMEROS REAIS

4.32 Exercício 32
 
1 1 1 1
Para todo n ∈ N, tem-se 0 < e − 1 + + + ··· + < . Conclua daí que o número
1! 2! n! n!n
e é irracional.

Solução.

1 1
Nível 1. Vamos usar a mesma notação do Exemplo 8: an = 1 + +···+ . Fixado um
1! n!
natural n, vamos mostrar no Nível 2.1 que:

1
an + > an+p ∀p ∈ N .
n!n
1
Como e = lim an , a desigualdade acima implica an + > e. A desigualdade estrita vai seguir
n!n
1 1
do fato de que an+1 + < an + (a demonstração disso está no Nível 2.2),
(n + 1)!(n + 1) n!n
1
pois também temos an+1 + > e. A desigualdade e − an > 0 vale porque e > an
(n + 1)!(n + 1)
para todo n, desigualdade esta que segue do Exemplo 8.

No elevador. Escrevemos algumas desigualdades equivalentes àquela que queremos de-


monstrar e resolvemos escolher a desigualdade

 
(n + 1) · · · (n + p) > n (n + 2) · · · (n + p) + (n + 3) · · · (n + p) + · · · + (n + p) + 1 .

Queremos que ela valha para todo p, então pode ser uma boa fazê-lo por indução em p. Supondo
que vale para p, ao tentar fazer para p + 1, obtemos:

 
(n + 1) · · · (n + p)(n + p + 1) > n(n + p + 1) (n + 2) · · · (n + p) + · · · + 1
 
= n (n + 2) · · · (n + p + 1) + · · · + (n + p + 1)

e cou faltando aparecer uma última parcela igual a 1 dentro destes colchetes. A ideia é
que essa desigualdade vale porque a expressão do lado direito aparece no desenvolvimento da
expressão do lado esquerdo como uma de suas parcelas, além de outras parcelas (é claro), as
quais, pelo visto, não podem ser tão desprezadas assim. Então vamos chamar essas parcelas
excedentes de q e a coisa se desenrola. Dependendo do interesse, talvez até dê para aproveitar
as contas abaixo e dizer explicitamente quem é q.

Nível 2.1. A desigualdade que queremos demonstrar é equivalente a cada uma das desi-
gualdades abaixo
1 1 1 1
> + + ··· + ,
n!n (n + 1)! (n + 2)! (n + p)!
1 1 1 1
> + + ··· + ,
n (n + 1) (n + 1)(n + 2) (n + 1)(n + 2) · · · (n + p)
 
(n + 1)(n + 2) · · · (n + p) > n (n + 2)(n + 3) · · · (n + p) + (n + 3) · · · (n + p) + · · · + (n + p) + 1 .
4.33. EXERCÍCIO 33 147

Vamos demonstrar esta última desigualdade por indução em p. Para p = 1 ca melhor, na
verdade, olhar para a segunda das desigualdades acima: 1/n > 1/(n + 1). Supondo que vale
para p, existe q∈N tal que

(n + 1) · · · (n + p) = n[(n + 2) · · · (n + p) + · · · + (n + p) + 1] + q.

Daí:
 
(n + 1) · · · (n + p + 1) = n(n + p + 1) (n + 2) · · · (n + p) + · · · + (n + p) + 1 + (n + p + 1)q
 
= n (n + 2) · · · (n + p + 1) + · · · + (n + p + 1) + q(n + p + 1)
 
> n (n + 2) · · · (n + p + 1) + · · · + (n + p + 1) + n
 
= n (n + 2) · · · (n + p + 1) + · · · + (n + p + 1) + 1 ,

como queríamos.

Nível 2.2. Temos:

1 1 1 1 1
an+1 + < an + ⇔ + <
(n + 1)!(n + 1) n!n (n + 1)! (n + 1)!(n + 1) n!n
1 1 1
⇔ + <
n + 1 (n + 1)2 n
n(n + 1) + n
⇔ <1
(n + 1)2
n2 + 2n
⇔ <1
n2 + 2n + 1
⇔ n2 + 2n < n2 + 2n + 1.

O resultado desejado segue.

Segunda Parte. Se e fosse um número racional, seja e = m/n, com n ∈ N. Então temos:

m 1 1
an < < an + ⇔ n!an < m(n − 1)! < n!an + < n!an + 1
n n!n n
e teríamos um número inteiro entre dois números inteiros consecutivos, o que é um absurdo.
Portanto e não pode ser um número racional, ou seja, e é irracional. 

4.33 Exercício 33
1p 4
lim n
(n + 1)(n + 2) · · · 2n = [sic ]. (Use o nal do Exercício 28.)
n→∞ 2 e

Solução. Este resultado é falso. Esta sequência tende ao innito. O resultado seria verdadeiro
se tivéssemos um n no lugar daquele 2 no denominador, isto é:

1pn 4
lim (n + 1) · · · (2n) = ,
n e
onde r
1pn n (2n)!
(n + 1) · · · (2n) = .
n n! · nn
148 CAPÍTULO 4. SEQUÊNCIAS E SÉRIES DE NÚMEROS REAIS

Agora sim o Exercício 28 (página 142) se aplica:

n
n! · nn

(2n + 2)! 2n + 1 n
n+1
· =2· ·
(n + 1)! · (n + 1) (2n)! n+1 n+1

4
tende a . 
e

4.34 Exercício 34

Prove que se denirmos an pela igualdade n! = nn · e−n · an , teremos lim n an = 1.

n
Solução. Essa é pelo Exemplo 36, o qual diz: lim √
n
= e. Então temos:
n!

n
√ n! 1
n
an = e · → e · = 1.
n e
Ou, podemos fazer também pela razão:

n
(n + 1)! · en+1 nn

an+1 n 1
= · =e =e· →1
an (n + 1)n+1 n! · en n+1 (1 + n1 )n

e aplicar o Exercício 28 (página 142). 

4.35 Exercício 35
sic ]
P P P
Seja [ an e bn séries de termos positivos. Se bn = +∞ e existe n0 ∈ N tal que
an+1 bn+1 P
> para todo n > n0 então an = +∞.
an bn

Solução. A ideia dessa solução já foi vista no texto. Supondo que essa desigualdade vale para
todo n > n0 (se n > n0 , então n > n0 + 1), então temos:

an0 +1 bn +1 an +2 bn +2 an +p−1 bn +p−1 an0 +p bn0 +p


> 0 , 0 > 0 , ··· , 0 > 0 , > .
an0 bn0 an0 +1 bn0 +1 an0 +p−2 bn0 +p−2 an0 +p−1 bn0 +p−1

Assim, multiplicando membro a membro (uma vez que todos esses números são positivos),

an0 +p bn +p
> 0 ,
an0 bn0

ou seja,
an0
an0 +p > · bn0 +p ∀p ∈ N .
bn 0
Deste modo:
an0
an0 +1 + an0 +2 + · · · + an0 +p > (bn +1 + · · · + bn0 +p )
bn0 0
e
an0
a1 + · · · + an0 +p > a1 + · · · + an0 + (bn +1 + · · · + bn0 +p ).
bn0 0
4.36. EXERCÍCIO 36 149

P X
Se bn = +∞, então bn = +∞ também. Assim, dado M > 0, existe q∈N tal que
n>n0

bn0
bn0 +1 + · · · + bn0 +p > (M − a1 − · · · − an0 ).
an0

para todo p > q. Para este q, temos

p > q ⇒ a1 + · · · + an0 +p > M.


P
Logo an = +∞, como queríamos. 

4.36 Exercício 36
P P an P P
Sejam an e bn séries de termos positivos. Se lim = 0 e bn converge então an
bn
an P P
converge. Se lim = c 6= 0 então an converge se, e somente se, bn converge.
bn

Solução.

Primeira Parte. Temos:


an
an = · bn .
bn
Este número é menor do que bn se an /bn for pequeno o suciente, por exemplo, menor do que
1. Como lim an /bn = 0, tome n0 tal que an /bn < 1 para todo n > n0 . Para este n0 temos
P
an < bn para todo n > n0 . Agora a convergência da série an segue do Corolário do Teorema
16, fazendo c = 1 naquela notação.

Segunda Parte. A ideia aqui é a mesma da primeira parte. Se lim an /bn = c, então existe
n0 tal que an /bn < 3c/2 para todo n > n0 . Assim, se n > n0 , então

an 3c
an = · bn < · bn .
bn 2

P P bn 1
Daí, se bn converge, então an converge. Se lim an /bn = c e c 6= 0, então lim = 6= 0 e
an c
a recíproca se reduz ao caso anterior. Segue a tese. 

4.37 Exercício 37
P 1
Para todo polinômio p(x) de grau superior a 1, a série converge.
p(n)
(Voltar para a Solução do Exercício 41: página 151.)
150 CAPÍTULO 4. SEQUÊNCIAS E SÉRIES DE NÚMEROS REAIS

Solução. p(n) =
Esse raciocínio eu aprendi num dos livros de Análise do Geraldo Ávila. Seja
ar nr + · · · + a1 n + a0 , ar 6= 0. Vamos mostrar que a série dada é absolutamente
com r>1 e
P 1
convergente comparando-a com a série do Exemplo 29.
nr
1 r 1 r 1
Seja xn = . Temos n xn → e (pelo Exercício 1, página 117) n |xn | → . Assim,
p(n) ar |ar |
1 r
tomando um a > , seja n0 tal que n |xn | < a para todo n > n0 . Portanto a > 0 e n0 são
|ar |
tais que
1
|xn | < a · ∀n > n0 .
nr
P 1
Portanto, como converge, pelo Corolário do Teorema 16, a série dada é absolutamente
nr
convergente, logo convergente. 

4.38 Exercício 38
   
m m(m − 1) · · · (m − n + 1) m n
Se −1 < x < 1 e = então lim x = 0 para quaisquer
n n! n→∞ n
m∈R e n ∈ N.
(Voltar para a Solução do Exercício 17 do Capítulo IX, Nível 2: página 372.)

Solução.

No elevador. Diferentemente do que estamos acostumados, aqui o m pode ser um número


real também, e este n pode crescer mesmo acima de m. A princípio eu estava achando então
que esse resultado poderia ser falso e que os termos de 
umatal sequência pudessem ser cada
m
vez mais negativos. Porém, para m natural, os números são iguais a zero se n>m e, no
n
caso em que m não é natural, após alguma investigação, o Exercício 25 (página 140) apareceu
como possibilidade de ser aplicado, mas para mostrar que havia uma subsequência tendendo
ao innito. Eu estava enganado, e o mesmo Exercício 25 mostra isso.

Se m é um número natural, então:


 
m m · · · (m − m + 1)(m − m) · · · (m − p + 1)
= = 0.
m+p (m + p)!
 
m n
Então x = 0 para todo n > m.
n
Se x = 0, então o resultado segue trivialmente. Suponhamos então que x 6= 0 e que m
não seja um número natural, de modo que nenhum termo da sequência da tese é igual a zero.
Poderemos então aplicar o Exercício 25;

m

xn+1


n+1
n! m · · · (m − n + 1)(m − n) |m − n|
= |x| · · = |x| · .

m

xn m · · · (m − n + 1) (n + 1)! n+1

n

Pela Desigualdade Triangular (o item (i) do Teorema 2 do Capítulo III), este número é menor
 
|m| n
do que ou igual ao número |x| · + , o qual tende para |x| conforme n cresce.
n+1 n+1
4.39. EXERCÍCIO 39 151

Assim, xando um c no intervalo aberto (|x|, 1), existe n0 tal que



m xn+1
0 < n+1 6c

m

x n
n

para todo n > n0 , como queríamos. 

4.39 Exercício 39
a1 + · · · + an
Se a sequência (an ) é não-crescente e lim an = 0, o mesmo ocorre com bn = .
n
1 1
Conclua que, neste caso, a série a1 − (a1 + a2 ) + (a1 + a2 + a3 ) − · · · é convergente.
2 3

Solução. Que bn converge para 0 é consequência do Exercício 27 (página 142). Queremos ver
agora que bn+1 6 bn . Cada uma das inequações abaixo são equivalentes entre si:
a1 + · · · + an+1 a1 + · · · + an
6 ,
n+1 n
n(a1 + · · · + an ) + nan+1 6 n(a1 + · · · + an ) + (a1 + · · · + an ),
nan+1 6 a1 + · · · + an .
Esta última desigualdade vale porque, pelo fato de an ser não crescente, temos: an+1 6 a1 ,
an+1 6 a2 , . . . , an+1 6 an . Assim, somando membro a membro, vem nan+1 6 a1 + · · · + an .
Portanto bn+1 6 bn como queríamos. Pelo Corolário 2 do Teorema 21, podemos concluir que a
1 1
(−1)n+1 bn = a1 − (a1 + a2 ) + (a1 + a2 + a3 ) − · · · é convergente.
P
série 
2 3

4.40 Exercício 40
a2 a2
Prove que, para todo a ∈ R, a série a2 + + + ··· é convergente e calcule sua
1 + a2 (1 + a2 )2
soma.

Solução. Se a 6= 0, as parcelas dessa série são termos de uma progressão geométrica de razão
1
<1 e, nesse caso, sua soma é (Exemplos 24 e 25):
1 + a2
1 1
a2 · 1 = a2 · a2
= 1 + a2 .
1− 1+a2 1+a2

Se a = 0, então todos os termos dessa série são iguais a zero e sua soma é 0. 

4.41 Exercício 41
X 1
Para todo p∈N xado, a série converge.
n
n(n + 1) · · · (n + p)

Solução. Basta fazer p(n) = n(n + 1) · · · (n + p) no Exercício 37 (página 149) que o resultado
segue. 
152 CAPÍTULO 4. SEQUÊNCIAS E SÉRIES DE NÚMEROS REAIS

4.42 Exercício 42
X an
(an )2
P P
Se an converge e an > 0 então e convergem.
1 + an
P
Solução. Ambas as séries da tese podem ser comparadas com a série dada. Se an converge,
então lim an = 0 n0 tal que an < 1 para todo n > n0 . Assim, multiplicando
e, portanto, existe
2
ambos os membros por an , como an > 0, temos (an ) < an para todo n > n0 . Além disso, como
1 an an
an > 0, temos também 1 + an > 1 ⇒ <1⇒ < an . Como (an )2 e são
1 + an 1 + an 1 + an
números não negativos, a conclusão segue do Corolário do Teorema 16. 

4.43 Exercício 43
X an
(an )2
P
Se converge então converge.
n

Solução.

No elevador. Começamos tentando aplicar os diversos critérios de convergência apresen-


tados no texto, porém sempre nos esbarramos na necessidade de a sequência dos an 's possuir
ou não certas propriedades: um certo critério funciona se an tem certa propriedade, mas deixa
de funcionar caso an não tenha essa propriedade, e nada nos garante que todos os termos te-
nham essa mesma propriedade. Alguns podem ter, outros não. Dentre essas tentativas está
|an | 1 1 1
a comparação de com a2n e com , as quais dependem de ser |an | > ou |an | <
n n2 n n
respectivamente. Mas, espera, essas duas propriedades distintas podem ser consideradas si-
multaneamente, à guisa de como foi feito após o Exemplo 37, quando o texto trata da parte
positiva e negativa de uma série!

Sejam:    
1 1
X= n ∈ N; |an | > e Y = n ∈ N; |an | < ,
n n
de modo que:
1 |an |
n∈X⇒ 6 |an | ⇒ 6 a2n
n n
e
1 |an | 1
n ∈ Y ⇒ |an | < ⇒ < 2
n n n
2
= a2n , por isso vale
(|an | a primeira implicação). Claramente temos X ∩ Y = ∅ e X ∪ Y = N.
an an
Sejam xn = se n ∈ X e xn = 0 se n ∈
/ X e yn = se n ∈ Y e yn = 0 se n ∈
/ Y . Agora:
n n
X an
X X X an
xn = e yn = ,
n∈N n∈X
n n∈N n∈Y
n
P P
onde ambas as séries xn e yn são (absolutamente) convergentes pelo critério da comparação
pelas implicações acima. Portanto (pelo Exemplo 25) é também convergente sua soma:
X  X  X X an
xn + yn = (xn + yn ) = ,
n
como queríamos. 
4.44. EXERCÍCIO 44 153

4.44 Exercício 44
P
Se (an ) é decrescente e an converge então lim n · an = 0.
(Voltar para a Solução do Exercício 46: página 155.)

Solução.

No elevador. Por (an ) ser decrescente, obtemos nan < a1 + · · · + an := sn , para todo n.
Esta desigualdade nos lembra o Teorema 8, mas sn não converge necessariamente para zero. Ao
invés de olharmos para os termos que estão para trás de an , podemos olhar para os que estão
à sua frente, n termos à sua frente, e obter s2n − sn < nan . Como (sn ) converge, o limite de
s2n − sn , sim, é zero, o que também nos lembra a demonstração do Teorema 15. Porém, agora,
a desigualdade está ao contrário do que nos seria útil para aplicar o Teorema 8. Por outro lado,
podemos comparar s2n − sn não com o an , mas com o a2n e obter desigualdades mais adequadas
para os nossos objetivos.

Seja s n = a1 + · · · + an , para todo n ∈ N. Como (an ) é descrescente, temos

na2n < s2n − sn ∀n ∈ N,

em que lim s2n = lim sn (Teorema 2) implica lim(s2n − sn ) = 0 (Teorema 6). Portanto, como
também 0 < na2n para todo n (an > 0 para todo n pois (an ) é decrescente e lim an = 0),
lim na2n = 0 (Teorema 8). Logo,
lim 2na2n = 0
(Teorema 6 novamente).
Para concluir que lim nan = 0 vamos agora utilizar o Exercício 3 (página 117) e mostrar
que

lim(2n − 1)a2n−1 = 0.
Como (2n − 1)a2n−1 = 2na2n−1 − a2n−1 e lim a2n−1 = 0 (teoremas 15 e 2), basta mostrar que
lim 2na2n−1 = 0. Podemos proceder de modo análago ao que zemos mais acima:

0 < na2n−1 < s2n−1 − sn−1 ⇒ lim na2n−1 = 0 ⇒ lim 2na2n−1 = 0,

como queríamos. Logo, lim nan = 0. 

4.45 Exercício 45
P
Se (an ) é decrescente e an = +∞, então,

a1 + a3 + · · · + a2n−1
lim = 1.
n→∞ a2 + a4 + · · · + a2n

(Voltar para a Observação 2 do Exercício 49: página 158.)

Solução.
154 CAPÍTULO 4. SEQUÊNCIAS E SÉRIES DE NÚMEROS REAIS

Nível 1. Vamos mostrar que o número

a1 + a3 + · · · + a2n−1 a1 − a2 + a3 − a4 + · · · + a2n−1 − a2n


−1=
a2 + a4 + · · · + a2n a2 + a4 + · · · + a2n
tende a zero. Armamos que a parte de cima dessa fração é limitada (Nível 2.1) e que a parte
de baixo tende a innito (Nível 2.2). Assim essa sequência tende a zero porque o produto de
uma limitada por uma que tende para zero é zero.

Nível 2.1. Vamos vericar que a1 − a2 6 a1 − a2 + · · · + a2n−1 − a2n 6 a1 para todo n.


Para ver que essa desigualdade vale para n = 1, armamos que an > 0 para todo n ∈ N (Nível
3), de modo que a1 − a2 6 a1 porque a2 > 0. Além disso, como a2n+1 − a2n+2 > 0, temos

a1 − a2 + · · · + a2n−1 − a2n < a1 − a2 + · · · + a2n−1 − a2n + (a2n+1 − a2n+2 ).

Este número é menor do que a1 porque

(−a2 + a3 ) + · · · + (−a2n + a2n+1 ) − a2n+2

é uma soma de números negativos (a2n+2 é positivo, pelo Nível 3). Assim, se o resultado é
verdadeiro para n, então é verdadeiro para n+1 também. Segue a tese.

No elevador. Sabemos como a sequência das somas parciais se comporta, crescendo acima
de qualquer limite. Então vamos comparar a soma dos termos de índices pares a2 +a4 +· · ·+a2n
com a soma a1 + a2 + · · · + a2n+1 + a2n . Mas precisamos fazer isso de um modo que a primeira
soma que maior do que a segunda.

P
Nível 2.2. Queremos mostrar que a2n = +∞. Como a3 < a2 , ..., a2n+1 < a2n , temos

a1 + a2 + a3 + · · · + a2n + a2n+1 < a1 + a2 + a2 + · · · + a2n + a2n .

Portanto:
1 
(∗) a2 + · · · + a2n > (a1 + a2 · · · + a2n+1 ) − a1 .
2
P
Dado M > 0, pelo fato de an = +∞, existe um número da forma 2n0 + 1 tal que:

2n + 1 > 2n0 + 1 ⇒ a1 + a2 + · · · + a2n+1 > 2M + a1

Assim, se n > n0 , então 2n + 1 > 2n0 + 1 e, pela implicação acima e pela desigualdade (∗):

a2 + · · · + a2n > M,

como queríamos.

Nível 3. Queremos mostrar que an > 0 para todo n ∈ N. Se isso não vale, então existe
k ∈ N ak 6P0. Com a contrapositiva em mente, queremos mostrar que (an ) não é
tal que
decrescente ou que an não tende para +∞. Se (an ) não é decrescente (o que inclui o caso
em que (an ) sequer é monótona), então não há nada a se fazer. Supondo (an ) decrescente,
vamos encontrar um M > 0 tal que, dado m ∈ N, exista um natural n maior do que m tal que
Xn
ai 6 M .
i=1
4.46. EXERCÍCIO 46 155

Para k ∈ N tal que ak 6 0, como (an ) é decrescente, temos an 6 0 para todo n > k
n
X Xk Xk
e, portanto, ai 6 ai , para todo n > k . O M ainda não pode ser o número ai
i=1 i=1 i=1
porque não sabemos se este é positivo. Então, pelo Princípio da Boa Ordenação, vamos supor
k−1
X
k = min{n ∈ N; an 6 0}, de modo que ai > 0 se k > 1. Agora denimos M =1 se k=1
i=1
k−1
X
e M = ai se k > 1. Para M assim denido, dado m ∈ N, tomando n > max{k, m}, temos
i=1
n
X k
X k−1
X n
X
ai 6 M , como queríamos, pois ai 6 ai se k>1 e ai 6 0 < 1 para todo n∈N se
i=1 i=1 i=1 i=1
k = 1. 
(Voltar para a Solução do Exercício 49: página 157.)

(Voltar para a Observação 2 do Exercício 49: página 158.)

4.46 Exercício 46
P
Seja
P(ann) uma sequência não-crescente, com lim an = 0. A série an converge se, e somente
se, 2 · a2n converge.

Solução. Já vimos no Exercício 44 (página 153) que os fatos de (an ) ser não crescente e
tender a zero implicam an > 0 para todo n, pois se algum an fosse negativo, então os termos
desta sequência a partir deste n estariam muito longe do zero. Vamos poder então comparar
as duas séries desse exercício e aplicar o Corolário do Teorema 16.

Ida. O a2 é maior do que ou igual ao a2 ; o a3 e o a4 são maiores do que ou iguais ao a4 ;


os a5 , a6 , a7 e a8 são maiores do que ou iguais ao a8 ; e, de modo geral: a > a para 2n−1 +i 2n
i ∈ {1, 2, . . . , 2n−1 }, para todo n ∈ N. Assim, somando estas 2n−1 desigualdades membro a
membro, obtemos:
2n−1 · a2n 6 a2n−1 +1 + a2n−1 +2 + · · · + a2n .
P
Denindo xi = a2i−1 +1 + a2i−1 +2 + · · · + a2i para todo i ∈ N, claramente a série xn converge,
Xn X2n
pois xi = ai (subsequência de uma sequência convergente). Portanto, pelo Corolário
i=1 i=1 P n−1
do Teorema 16, a série
P n−1 P n2 a2n é convergente, e, pelo Exemplo 25, é convergente também a
série 2 · 2 a2n = 2 a2n como queríamos.

Volta. Temos a2 > a2 , a2 > a3 , a4 > a4 , a4 > a5 , a4 > a6 , a4 > a7 , e, de modo geral,
n
a2n > a2n +(i−1) para todo i ∈ {1, 2, . . . , 2 }, para todo n ∈ N, o que implica

a2n + a2n +1 + · · · + a2n+1 −1 6 2n a2n .

2n a2n
P
Assim, se a série converge, do mesmo modo como zemos no parágrafo anterior, pode-
2n+1
X−1
!
mos concluir que a (sub-)sequência ai converge (primeiro com a série começando
i=1 n∈N
156 CAPÍTULO 4. SEQUÊNCIAS E SÉRIES DE NÚMEROS REAIS

n
!
X
em i=2 e, depois, colocando o a1 de volta). Mas, então, a sequência toda ai con-
i=1 n∈N
verge também, pois ela é monótona e limitada pelo mesmo número que limita sua subsequência.
2n+1
X−1
De fato, seja M tal que M > ai para todo n. Dado n, tome m tal que 2m+1 − 1 > n e
i=1
n
X 2m+1
X−1
teremos: ai 6 ai < M . 
i=1 i=1

(Voltar para a Observação 2 do Exercício 49: página 158.)

4.47 Exercício 47
Prove que o conjunto dos valores de aderência da sequência xn = cos(n) é o intervalo fechado
[−1, 1].

Solução. Dado x ∈ [−1, 1] e dado ε > 0, vamos mostrar que o conjunto

N = {n ∈ N; x − ε < cos n < x + ε}

é innito. Assim, pelo Teorema 9, o intervalo [−1, 1] estará contido no conjunto dos valores de
aderência de (xn ). Este conjunto será então o intervalo [−1, 1] porque −1 6 cos n 6 1 para
todo n ∈ N, ou seja, porque ele está contido em [−1, 1].
Sejama, b ∈ [0, π] tais que a < b, x − ε < cos b e cos a < x + ε, de modo que

a < n + 2πm < b ⇒ cos b < cos(n + 2πm) < cos a ⇒ x − ε < cos n < x + ε .

Pelo Exercício 58 do Capítulo III (página 110), sendo G = {n + 2πm ∈ R; m, n ∈ Z},


podemos concluir não só que G ∩ (a, b) é não vazio, como também que G ∩ (a, b) é innito. É
innito também o conjunto

X = {n ∈ Z; ∃m ∈ Z, a < n + 2πm < b},

pois, dado n ∈ N, {m ∈ Z; a < n + 2πm < b} é nito porque é um conjunto limitado (inferior
e superiormente) de números inteiros, logo, X nito implicaria G ∩ (a, b) nito.
Sejam X+ = {n ∈ X; n > 0} e X− = {n ∈ X; n < 0}. Como X+ ∪ X− = X − {0} é
innito, um dos dois, X+ ou X− , é innito. Além disso, X+ ⊂ N e −n ∈ N para todo n ∈ X−
(lembrando que cos α = cos(−α) para todo α ∈ R). Logo, N é innito, como queríamos. 

4.48 Exercício 48
Sejam a, b números reais positivos. Dena indutivamente as sequências (xn ), (yn ) pondo x1 =
√ √
ab, y1 = (a + b)/2 e xn+1 = xn yn , yn+1 = (xn + yn )/2. Prove que xn e yn convergem para o
mesmo limite, chamado a média aritmético-geométrica entre a e b.

(Voltar para a Observação 2 do Exercício 49: página 158.)

Solução.
4.49. EXERCÍCIO 49 157

Nível 1. Pela desigualdade entre a média geométrica e a média aritmética, de imediato


temos xn 6 yn para todo n ∈ N. Assim, pelo Corolário 1 do Teorema 7, sendo X = lim xn e
Y = lim yn , temos X 6 Y . Armamos que não pode acontecer X < Y . Logo só poderá ser
X =Y.

No elevador. Ainda pela desigualdade entre as médias geométrica e aritmética, podemos


ver que a sequência dos xn 's é crescente e a sequência dos yn 's é decrescente (na verdade, a dos
xn 's não decresce e a dos yn 's não cresce). A ideia então é escolher um certo ε que faça o yn+1
ultrapassar o Y .

xn X Y yn Y + ε

Nível 2. X < Y . Seja ε = (Y − X)/2 e tome yn tal que Y 6 yn < Y + ε.


Suponha
Armamos que a sequência (yn ) é não crescente (Nível 3.1) e a sequência (xn ) é não decrescente
(Nível 3.2). Então, de fato, um tal yn existe (por ser não crescente, todos os yn 's estão acima do
ou são iguais ao Y ). Para este n temos o xn 6 X (pelo mesmo motivo: por ser não decrescente,
todos os xn 's estão abaixo do ou são iguais ao X ). Deste modo temos:

y n + xn (Y + ε) + X Y + X + (Y − X)/2 3Y + X 3Y + Y
yn+1 = < = = < = Y.
2 2 2 4 4
Isto é uma contradição, porque usamos o fato de Y ser limite da sequência(yn ) para encon-
trar um tal yn e, ao mesmo tempo, por (yn ) ser não crescente e termos yn+1 < Y , concluímos
que Y não pode ser limite dessa sequência. Logo X >Y, como queríamos.

Nível 3.1. Temos:

y n + xn yn + yn
yn+1 = 6 = yn .
2 2

Nível 3.2. Temos:

√ √
xn+1 = xn · yn > xn · xn = |xn | = xn . 

4.49 Exercício 49
Sejam a1 > a2 > · · · > 0 e sn = a1 − a2 + · · · + (−1)n−1 an . Prove que a sequência (sn ) é limitada
e que lim sup sn − lim inf sn = lim an .

Solução.

Primeira Parte: é limitada. O que acontece aqui é semelhante ao que acontecia na


Solução do Exercício 45 (página 153). Temos a1 − a2 6 sn 6 a1 para todo n: sn 6 a1 porque
sn é a soma do a1 com números não positivos e sn > a1 − a2 porque sn é a soma do a1 − a2
com números não negativos.
158 CAPÍTULO 4. SEQUÊNCIAS E SÉRIES DE NÚMEROS REAIS

Segunda Parte: a igualdade. Continuando com a ideia que mostrou a limitação da


sequência dos sn 's, chegamos a:

s2n 6 s2n+2 6 s2n+3 6 s2n+1 ∀n ∈ N,

pois:
s2n+2 = s2n + (a2n+1 − a2n+2 ),
s2n+1 = s2n + a2n+1 ,
s2n+3 = s2n+1 + (−a2n+2 + a2n+3 ),
onde a2n+1 − a2n+2 > 0, a2n+1 > 0 e −a2n+2 + a2n+3 6 0.
Um caso sai imediatamente do Teorema de Leibniz: lim an = 0, então (por Leibniz)
se
a sequência (sn ) é convergente, portanto: lim sup sn = lim inf sn , o que implica lim sup sn −
lim inf sn = 0 = lim an , como queríamos. Vamos supor então lim an = A 6= 0 (pelo Teorema 4,
(an ) é de fato convergente). Sejam também lim sup sn = L e lim inf sn = l. A ideia aqui vai ser
parecida com a do Exemplo 18.

s2n s2n+2 · · · l L · · · s2n+3 s2n+1 s2n−1

Para cada n natural seja Xn = {sn , sn+1 , sn+2 , . . .}. Temos então s2n ∈ X2n−1 e s2n+1 ∈ X2n ,
o que faz:
inf X2n = s2n = inf X2n−1 ,
e
sup X2n+1 = s2n+1 = sup X2n .
Portanto:
l := lim inf sn := sup inf Xn = sup s2n
n n
e
L := lim sup sn := inf sup Xn = inf s2n−1 .
n n

Desse modo, podemos concluir que L = lim s2n−1 e l = lim s2n . Além disso, temos a2n+1 =
s2n+1 − s2n . Logo:
A = lim a2n+1 = lim(s2n+1 − s2n ) = L − l,
pelo Teorema 6, como queríamos. 

Observação 1. De fato a hipótese A 6= 0 não foi usada na solução acima, portanto podemos
concluir o Teorema de Leibniz a partir deste exercício.

Observação 2. Notemos as semelhanças entre o exercício acima e os Exercícios 45 e 48 (pá-


ginas 153 e 156): em todos eles lidamos com sequências que possuíam subsequências crescentes
e decrescentes. Notemos também as semelhanças entre as soluções dos Exercícios 45 e 46 (pá-
ginas 153 e 155), em ambos trocamos termos convenientes da sequência para podermos fazer
comparações.
Capítulo 5
Topologia da Reta

5.1 Exercício 1
Um conjunto A⊂R é aberto se, e somente se, cumpre a seguinte condição: se uma sequência
(xn ) converge para um ponto a∈A então xn ∈ A para todo n sucientemente grande.

Solução.

Ida. A ⊂ R aberto e sejam a um ponto de A e (xn ) uma sequência que converge para
Seja
a. Como A é aberto e a ∈ A, existe ε > 0 tal que (a − ε, a + ε) ⊂ A. Para este ε, como xn → a,
existe n0 tal que xn ∈ (a − ε, a + ε) para todo n > n0 e, consequentemente, xn ∈ A para todo
n > n0 .

Volta. Seja a um ponto em R e vamos supor que nenhum intervalo aberto ao qual a
pertença esteja contido em A. Vamos concluir que a∈
/ A. Pela
 contrapositiva,
 isto é equivalente
1 1
a dizer que A é aberto. Assim, para cada n ∈ N, seja xn ∈ a− ,a + um ponto que não
n n
pertence
 a A. Isto 
nos dá uma sequência que converge para a, pois, dado ε > 0, seja n∈N tal
1 1
que a − ,a + ⊂ (a − ε, a + ε) e teremos:
n n
 
1 1
a − ,a + ⊂ (a − ε, a + ε) ∀m > n.
m m

Portanto, xm ∈ (a − ε, a + ε) para todo m > n. Por outro lado, se A satisfaz a condição dada,
a existência de uma tal sequência nos permite concluir que a∈
/ A, como queríamos. 

5.2 Exercício 2
Tem-se lim xn = a se, e somente se, para todo aberto A contendo o ponto a, existe n0 ∈ N tal
que n > n0 implica xn ∈ A.

Solução.

159
160 CAPÍTULO 5. TOPOLOGIA DA RETA

Ida. Sejam a e (xn ), com lim xn = a. Seja A um aberto ao qual a pertence. Então existe
ε > 0 tal que (a − ε, a + ε) ⊂ A. Como lim xn = a, existe n0 tal que xn ∈ (a − ε, a + ε) para
todo n > n0 , isto é, xn ∈ A para todo n > n0 , por causa da inclusão (a − ε, a + ε) ⊂ A.

Volta. Seja (xn ) uma sequência e suponha que para todo aberto A ao qual a pertence
exista n0 tal que n > n0 ⇒ xn ∈ A. Em particular, o mesmo acontece para todo intervalo da
forma (a − ε, a + ε) para ε > 0. Logo lim xn = a, como queríamos. 

Observação. Para a ida deste exercício também podemos aplicar o exercício anterior: se
lim xn = a e A é um conjunto aberto possuindo o ponto a, então, pelo exercício anterior, xn ∈ A
para todo n sucientemente grande. Em outras palavras, existe n0 ∈ N tal que n > n0 implica
xn ∈ A.

5.3 Exercício 3
Seja B ⊂R aberto. Então, para todo x ∈ R, o conjunto x + B = {x + y; y ∈ B} é aberto.
Analogamente, se x 6= 0, então o conjunto x · B = {x · y; y ∈ B} é aberto.

(Voltar para a Solução do Exercício 7: página 162.)

Solução. Dado um ponto y ∈ B , sejam a e b reais, com a < b tais que y ∈ (a, b) ⊂ B .
Então o conjunto x + (a, b) = {x + z; z ∈ (a, b)} é um subconjunto do conjunto x + B . Mas
x + (a, b) = (x + a, x + b) (a demonstração disto está no último parágrafo desta solução). Logo,
x + y pertence ao interior de x + B e, como x + y era um ponto qualquer, segue que x + B é
aberto.
A demonstração da segunda parte é a mesma, com o único detalhe que x · (a, b) = (xa, xb)
se x > 0 e x · (a, b) = (xb, xa) se x < a.
Se a < z < b, então x + a < x + z < x + b, então x + z ∈ (x + a, x + b) se z ∈ (a, b).
Reciprocamente, se x + a < z < x + b, então x + a < x + (z − x) < x + b, então a < z − x < b,
então z = x + (z − x) ∈ x + (a, b). A demonstração das armações feitas no parágrafo acima
são análogas (dado z , escreva z = x · (z/x) usando que x 6= 0). 

5.4 Exercício 4
Sejam A, B abertos. Então os conjuntos A + B = {x + y; x ∈ A, y ∈ B} e A · B = {x · y; x ∈
A, y ∈ B} são abertos.
(Voltar para a Solução do Exercício 47: página 188.)

Solução. Como consequência do exercício anterior, x+B é[aberto para todo x ∈ A. Portanto,

pelo item b) do Teorema 1, é também aberto o conjunto (x + B). Logo, A + B é aberto


x∈A
porque
[
(x + B) = A + B.
x∈A

Assim, podemos concluir que, para A+B ser aberto, basta que um dos dois, A ou B, seja
aberto.
5.5. EXERCÍCIO 5 161

[
Analogamente, denindo L = A − {0}, é aberto o conjunto (x · B) = (A − {0}) · B , só
x∈L
faltando vericar que o número 0 é ponto interior de A·B caso 0 ∈ A. ε>0
Se 0 ∈ A, seja
tal que (− ε, ε) ⊂ A. Seja y ∈ B um ponto distinto de 0 ({0} não é aberto, então, como B
é aberto, certamente B possui um número não nulo). Armamos que (−y ε, y ε) ⊂ A · B , se
y > 0, e (y ε, −y ε) ⊂ A · B , se y < 0. De fato, para y > 0:
z z z
z ∈ (−y ε, y ε) ⇒ −y ε < z < y ε ⇒ − ε < <ε⇒ ∈A⇒z = ·y ∈A·B
y y y
e, para y < 0:
z z z
z ∈ (y ε, −y ε) ⇒ y ε < z < −y ε ⇒ − ε < < ε ⇒ ∈ A ⇒ z = · y ∈ A · B.
y y y

Deste modo, sejam Ax = x · B para todo x ∈ A não nulo. Se 0∈


/ A, então

[
A·B = Ax .
x∈A

Se 0 ∈ A, seja ε > 0 tal que (− ε, ε) ⊂ A e, para y ∈ B não nulo, sejam a = min{−y ε, y ε} e


b = max{−y ε, y ε}. Denindo A0 = (a, b), temos
[
A·B = Ax .
x∈A

Logo, A·B é aberto porque podemos escrevê-lo como uma reunião de conjuntos abertos; Ax é
aberto para todo x ∈ A. 

5.5 Exercício 5
Para quaisquer X, Y ⊂ R, tem-se int(X ∩ Y ) = int(X) ∩ int(Y ) e int(X ∪ Y ) ⊃ int(X) ∪ int(Y ).
Dê um exemplo em que a inclusão não se reduza a uma igualdade.

(Voltar para a Solução do Exercício 13 do Capítulo X, para seu Nível 4.3: página 419.)

Solução. x ∈ int(X ∩ Y ), seja x ∈ (a, b) ⊂ X ∩ Y . Então (a, b) ⊂ X e (a, b) ⊂ Y . Portanto


Se
x ∈ int(X) e x ∈ int(Y ), ou seja, x ∈ int(X) ∩ int(Y ). Reciprocamente, se x ∈ int(X) ∩ int(Y ),
sejam x ∈ (a, b) ⊂ X e x ∈ (c, d) ⊂ Y . Então x ∈ (a, b) ∩ (c, d), onde (a, b) ∩ (c, d) é um
intervalo aberto não vazio (pelo lema do Teorema 2) contido na interseção X ∩ Y , portanto,
x ∈ int(X ∩ Y ).
Se x ∈ int(X) ∪ int(Y ), então x ∈ int(X) ou x ∈ int(Y ). Se x ∈ int(X), seja x ∈ (a, b) ⊂ X .
Então x ∈ (a, b) ⊂ X ∪ Y e, portanto, x ∈ int(X ∪ Y ). Analogamente, x ∈ int(Y ) ⇒ x ∈
int(X ∪ Y ).
Sejam X = [0, 1] e Y = [1, 2], de modo que X ∪ Y = [0, 2], e:

int(X) ∪ int(Y ) = (0, 1) ∪ (1, 2)

e
int(X ∪ Y ) = (0, 2).
Portanto, 1 ∈ int(X ∪ Y ) mas 1∈
/ int(X) ∪ int(Y ). 
162 CAPÍTULO 5. TOPOLOGIA DA RETA

5.6 Exercício 6
Se A⊂R é aberto e a∈A então A − {a} é aberto.

Solução. x ∈ A, x 6= a. Então existe um intervalo (α, β) ao qual x pertence que é


Seja
subconjunto de A. Se a ∈ / (α, β), então (α, β) ⊂ A − {a}. Se a ∈ (α, β) e se a > x, então
x ∈ (α, a) ⊂ A − {a}. Se a < x, então x ∈ (a, β) ⊂ A − {a}. Logo, todo ponto de A − {a} é
seu ponto interior. Segue a tese. 

5.7 Exercício 7
Considere as funções f, g, h : R → R, dadas por f (x) = ax + b (a 6= 0), g(x) = x2 e h(x) = x3 .
−1
Mostre que, para cada A ⊂ R aberto, f (A), g −1 (A) e h−1 (A) são abertos.

(Voltar para a Solução do Exercício 47: página 188.)

Solução. A imagem inversa de A por f é igual à imagem direta de A pela função f −1 , onde
1 b
esta é dada por f −1 (x) = · x − . Deste modo:
a a
   
−1 1 b 1 b
f [A] = ·x− ; x∈A = ·A+ − ,
a a a a

na notação do Exercício 3 (página 160). Portanto f −1 [A] é aberto se A é aberto, pelo mesmo
Exercício 3 (página 160).
−1
Se x ∈ g [A], então x2 ∈ A√ 2
. Seja x ∈ (a, b) ⊂ A. Além disso, podemos supor 0 6 a < b,
√ √ √
−1 −1
se x 6= 0. Se x > 0, então x ∈ ( a, b) ⊂ g [A] . Se x < 0, então x ∈ (− b, − a) ⊂ g [A].
2
√ √ √ √ √ √
De fato: a < x < b ⇒ a < |x| < b, e y ∈ (− b, −√ a)√∪ ( a, b) ⇒ a < y < b ⇒ y 2 ∈
2

(a, b) ⊂ A. Se x = 0, seja 0 ∈ (a, b) ⊂ A. Então 0 ∈ (− √b, b) ⊂ g −1 [A].


−1

3
Se x ∈ h [A], seja x ∈ (a, b) ⊂ A. Então x ∈ ( a,
3 3
b) ⊂ h−1 [A]. 

5.8 Exercício 8
No exercício anterior, mostre que, para cada A ⊂ R aberto, f (A) e h(A) são abertos. Dê
exemplo de A aberto tal que g(A) não seja aberto.

(Voltar para a Observação 1 do Exercício 47: página 189.)

(Voltar para a Observação 2 do Exercício 47: página 189.)

Solução. f [A] é aberto o argumento é o mesmo: f [A] = a · A + b. Seja x3 ∈ h[A], com


Que
x ∈ A. Seja x ∈ (a, b) ⊂ A. Então x3 ∈ (a3 , b3) ⊂ h[A], pois a < x < b ⇒ a3 < x3 < b3 , e
√ √ √
a3 < y < b3 ⇒ a < 3 y < b, portanto, y = h 3 y , com 3 y ∈ (a, b) ⊂ A.
Para A = (−1, 1), temos g[A] = [0, 1), conjunto este que não é aberto. 
5.9. EXERCÍCIO 9 163

5.9 Exercício 9
Toda coleção de abertos não-vazios, dois a dois disjuntos é enumerável.

(Voltar para a Solução 1 do Exercício 40: página 182.)

(Voltar para a Nível 2.1 do Exercício 43: página 185.)

(Voltar para a Solução 1 do Exercício 63: página 202.)

(Voltar para a Solução do Exercício 9 do Capítulo VI: página 211.)

Solução 1. A uma coleção de abertos não vazios, dois a dois disjuntos. Pelo Teorema 2,
Seja
X
para cada X ∈ A, seja (Ii )i∈N uma família de intervalos abertos dois a dois disjuntos tais que
[ [
IiX = X . O conjunto A = X (embora o livro não tenha denido o que seja a reunião
i∈N X∈A
de uma coleção, podemos considerar a família (X)X∈A ) é um conjunto aberto (pelo item b) do
X
Teorema 1) e a coleção de todos os intervalos Ii é de intervalos abertos dois a dois disjuntos
cuja reunião é igual a A. Pela demonstração do Teorema 2, esta coleção (de intervalos abertos
A) é única e é enumerável. Além disto, sua cardinalidade é a
dois a dois disjuntos cuja reunião é
mesma do conjunto N ×A. Portanto A só pode ser enumerável (pois se A não fosse enumerável,
então o conjunto N ×A não seria enumerável). 

Solução 2. Na solução acima podemos isolar o trecho da demonstração do Teorema 2 que


foi usado e aproveitar sua ideia para criar uma solução mais sucinta. Seja A uma coleção de
abertos não vazios, dois a dois disjuntos e seja r:A→Q r(X) ∈ X para
uma função tal que
todo X ∈ A, usando a densidade de Q em R e o fato de que todo elemento de A é não vazio.
Esta função é injetiva porque se X e Y são elementos distintos de A, então X ∩ Y 6= ∅ e,
portanto, r(X) 6= r(Y ). Logo, A é enumerável (Corolário 1 do Teorema 8 do Capítulo II). 

Curiosidade. Essa propriedade  de qualquer coleção de abertos dois a dois disjuntos ser
enumerável  é conhecida como propriedade de Suslin. Também podemos dizer que um espaço
topológico com esta propriedade tem celularidade enumerável.

5.10 Exercício 10
O conjunto dos valores de aderência de uma sequência é um conjunto fechado.

Solução 1. Seja (xn ) uma sequência e seja X o conjunto de seus valores de aderência. Qual-
quer conjunto está contido em seu fecho. Reciprocamente, seja a∈X (se X = ∅, então X = ∅
e X é fechado) e seja (an ) uma sequência de pontos em X que converge para a. Esta sequência
existe pela denição de um ponto aderente a um conjunto. O segundo parágrafo da solução do
Exercicio 16 do Capítulo IV (página 128) também mostra que a ∈ X. Em comparação com
aquele exercício, o motivo pelo qual agora a é um número real é outro, a∈R porque X ⊂ R.
Segue X = X. 

Solução 2. Se a (xn ), então, pelo Teorema


não é um valor de aderência de uma sequência
9 do Capítulo IV, existe ε > 0 tal que o número de termos de (xn ) pertencentes ao intervalo
(a − ε, a + ε) é nito. Para um tal ε > 0, o mesmo vale para cada b ∈ (a − ε, a + ε): para
δ > 0 tal que (b − δ, b + δ) ⊂ (a − ε, a + ε), o número de termos da sequência (xn ) pertencentes
164 CAPÍTULO 5. TOPOLOGIA DA RETA

a (b − δ, b + δ) também é nito. Portanto, novamente pelo Teorema 9 do Capítulo IV, nenhum


elemento de (a − ε, a + ε) pode ser um valor de aderência de (xn ), ou seja, (a − ε, a + ε) ⊂ R −X ,
sendo X o conjunto dos valores de aderência de (xn ). Logo, R −X é aberto e, portanto, X é
fechado (Teorema 4), como queríamos. 

5.11 Exercício 11
Se X⊂F e F é fechado então X ⊂ F.
(Voltar para a Segunda Parte da Solução do Exercício 15: página 167.)

(Voltar para o Nível 1 da Solução do Exercício 17: página 168.)

Solução. Conforme já vimos no texto: se X ⊂ F, então X ⊂ F, onde F = F. 

5.12 Exercício 12
Se lim xn = a e X = {x1 , x2 , . . . , xn , . . .} então X = X ∪ {a}.
(Voltar para a Solução do Exercício 16: página 168.)

Solução. X ∪ {a} ⊂ X vale porque X ⊂ X e a ∈ X . Vejamos agora que


A inclusão
X ⊂ X ∪ {a}. Dado x ∈ X , pelo Teorema 8, x ∈ X ou x ∈ X 0 . Se x ∈ X , então não há nada a
0
se fazer. Se x ∈ X , podemos concluir que existe uma subsequência de (xn ) convergindo para
x através do Teorema 9 do Capítulo IV e do item 3. do Teorema 7: para todo ε > 0, existem
innitos índices n ∈ N tais que xn ∈ (x − ε, x + ε). Portanto, x = a (Teorema 2 do Capítulo
IV). Logo, X ⊂ X ∪ {a}, como queríamos. 
(Voltar para o item (c) da Solução do Exercício 48: página 190.)

5.13 Exercício 13
O número 1/4 pertence ao conjunto de Cantor.

Solução.

No elevador. Uma primeira tentativa pode ser a de mostrar que o 1/4 não foi retirado em
nenhuma das etapas da construção dese conjunto. Isto pode ser feito por indução no número
das etapas: se 1/4 não foi retirado na n-ésima etapa, então não será retirado na (n + 1)-ésima
etapa. Porém podem surgir algumas complicações nesse processo, o que nos leva a partir em
busca de outra estratégia. Uma segunda tentativa então surge: escrever o número 1/4 como
o limite de uma sequência de pontos que sabemos pertencerem ao conjunto de Cantor. Em
consequência, se tal for possível, seguirá o resultado desejado uma vez que esse conjunto é
fechado.

Nível 1. Assim como no texto, seja K o conjunto de Cantor. Existe uma sequência (xn )n∈N
de pontos xn ∈ K para todo n ∈ N que converge para 1/4 (vamos denir essa sequência no
Nível 2). Como K é fechado, segue que 1/4 ∈ K .
5.13. EXERCÍCIO 13 165

No elevador. Após alguma investigação, descobrimos que: 1/4 < 1/3, 1/4 > 2/9, 1/4 <
7/27, 1/4 > 20/81, onde: 2/9 = 1/3−1/9 é extremo de um intervalo retirado na segunda etapa,
7/27 = 2/9 + 1/27 é extremo de um intervalo retirado na terceira etapa e 20/81 = 7/27 − 1/81
é extremo de um intervalo retirado na quarta etapa.

7
27
1
+ 27
2
9
− 19
1
3

Nível 2. Para cada n ∈ N, vamos denir:

1 1 1 1 1
xn = − + − + · · · + (−1)n+1 · n .
3 9 27 81 3
1
Armamos que: lim xn = (Nível 3.1) e que xn ∈ K para todo n∈N (Nível 3.2).
4

Nível 3.1. Temos:


X 1 1 1 1 9 3
= · 1 = · =
32n−1 3 1− 9
3 8 8
e
X 1 1 1 1 9 1
= · 1 = · = .
32n 9 1− 9
9 8 8
X 1
Portanto, o lim xn , que é também o somatório (−1)n+1 n , é igual a:
3
3 1 2 1
− = = ,
8 8 8 4
como queríamos.

Nível 3.2. Já sabemos que todos os extremos de intervalos retirados pertencem a K.


Vamos mostrar que cada xn é extremo de um intervalo retirado na etapa n, convencionando
chamar de etapa 1 àquela etapa em que foi retirado o terço médio de [0, 1], de etapa 2 àquela
em que foram retirados os terços médios de [0, 1/3] e de [2/3, 1], e assim por diante. Vamos
mostrar mais, a saber, que dado n, x2n−1 é extremo à esquerda de um intervalo retirado na
etapa 2n − 1 ex2n é extremo à direita de um intervalo retirado na etapa 2n. Vamos proceder
por indução em n.
Por vericação, o x1 é claramente extremo à esquerda de um terço médio retirado na primeira
etapa e o x2 é extremo à direita de um terço médio retirado na segunda etapa.
Se x2n é extremo à direita de um intervalo
 retirado na etapa
 2n, então um intervalo retirado
1
na etapa 2n + 1 é terço médio do intervalo x2n , x2n + 2n , pois já sabemos que os intervalos
3
166 CAPÍTULO 5. TOPOLOGIA DA RETA

1
que permanecem logo após uma certa etapa m têm tamanho . Este terço médio é o intervalo
  3m
1 2
x2n + , x2n + , onde:
32n+1 32n+1

1
x2n + = x2n+1 ,
32n+1
ou seja, x2n+1 é extremo à esquerda de um terço médio retirado na etapa 2n + 1. De modo
análogo concluímos que x2n é extremo à direita de um intervalo retirado na etapa 2n caso o
x2n−1 seja extremo à esquerda de um intervalo retirado na etapa 2n − 1. Logo, a armação
feita vale para todo n ∈ N. 

5.14 Exercício 14
Sejam F, G conjuntos fechados disjuntos tais que F ∪G seja um intervalo fechado (limitado
ou não). Então F =∅ ou G = ∅.

Solução.

No elevador. A semelhança deste exercício com o Corolário do Teorema 2 é gritante.


A princípio, porém, pode não ser tão imediato assim qual transformação devemos fazer nele
para podermos aplicar aquele corolário, o que nos leva a procurar outra solução. Numa delas,
consideramos todos os casos possíveis e trabalhamos com cada um deles. Só que, neste trabalho,
um mesmo raciocínio persiste, o de tomar um ponto x que pertence a um desses conjuntos F ou
G, digamos ao F, e acabar concluindo que existe todo um intervalo ao qual x pertence contido
em F, pelo fato de x não estar em G e, portanto, não ser aderente à G. Só foi após um tempo
que minha cha caiu: F e G se comportam quase como conjuntos abertos! É esta observação
que nos indica o que devemos fazer para cair no referido corolário.

Nível 1. Armamos que todo ponto de F F ∪G é


que não seja extremo do intervalo
interior a F . O mesmo vale para G (Nível 2.1). Assim, se F ∪ G = R, então ambos F e G
são abertos. Portanto, pelo Corolário do Teorema 2, F = ∅ ou G = ∅. Se F ∪ G = [a, +∞),
 
então F − {a} ∪ G − {a} = (a, +∞), onde F − {a} e G − {a} são abertos e disjuntos. Se
F − {a} = ∅, então F = ∅ ou F = {a}. O mesmo para G. De igual modo, se F ∪ G = [a, b],
então F − {a, b} = ∅ ou G − {a, b} = ∅. Se ocorre o primeiro caso, então F = ∅, F = {a},
F = {b} ou F = {a, b}. Também existe a possibilidade de F ∪ G ter apenas um ponto, mas,
neste caso, não há nada a fazer. O resultado desejado segue então da seguinte armação: se
F ∪G é um intervalo com mais de um ponto, então possui innitos pontos aquele conjunto
dentre F e G que possuir um ponto (Nível 2.2). Deste modo, se, por exemplo, for F −{a, b} = ∅,
então só pode ser F = ∅. Análogo para todos os demais casos.

Nível 2.1. Vamos supor que F ∪G tenha mais de um ponto. Sem perda de generalidade,
seja x∈F e suponha que x não seja um extremo do intervalo F ∪G (este ponto que não é
extremo tem que estar em algum lugar, estamos supondo que ele está em F ). Se x ∈ F , então
x∈/ G, porque F e G são disjuntos. Como G é fechado, x não pertencer a G signica que existe
um intervalo aberto I ao qual x pertence tal que I ∩ G = ∅. Como x não é extremo, existe um
intervalo aberto J ao qual x pertence e tal que J ⊂ F ∪ G. Como I ∩ J ainda é um aberto não
5.15. EXERCÍCIO 15 167

vazio, existe um intervalo aberto L ⊂ I ∩ J tal que x ∈ L. Agora L ⊂ F ∪G e L ∩ G = ∅.


Portanto L ⊂ F, isto é, x ∈ int(F ), como queríamos

Nível 2.2. Se x∈F e x não é extremo, então a existência do L como acima mostra que
F tem innitos pontos. Se x é de extremo, suponhamos que ele seja um extremo à esquerda.
Assim como zemos no Nível 2.1, podemos concluir que existe um ε > 0 tal que [x, x + ε) ⊂ F .
Se x é um extremo à direita, então o raciocínio é análogo. Portanto, se um conjunto tem um
ponto, então ele tem innitos pontos. 

5.15 Exercício 15
Seja E ⊂ R enumerável. Consiga uma sequência cujo conjunto dos valores de aderência é
E. Use este fato para mostrar que todo conjunto fechado F ⊂R é o conjunto dos valores de
aderência de alguma sequência.
[ Sugestão : Escreva N como reunião enumerável de conjuntos innitos disjuntos Ni . Para
cada n ∈ Ni faça xn = i-ésimo elemento do conjunto E . Para a segunda parte, use o Teorema
6.]

Solução.

Primeira Parte. A sequência sugerida é a sequência procurada. É possível escrever o N


como uma reunião de uma família enumerável (Ni )i∈N , onde cada Ni é innito e Ni ∩ Nj = ∅ se
i 6= j , por exemplo, pelo Exercício 21 do Capítulo II (página 53).
Seja X o conjunto dos pontos de aderência da sequência (xn ). Queremos mostrar que
X = E. Se x ∈ X, (xn ) que converge para x. Todos os
então existe uma subsequência de
termos dessa subsequência pertencem a E , portanto, x ∈ E . Reciprocamente, seja x ∈ E . Pelo
Teorema 9 do Capítulo IV, para termos x ∈ X , vamos vericar que, dado ε > 0, o conjunto
N = {n ∈ N; xn ∈ (x − ε, x + ε)} é innito. De x ∈ E , temos x ∈ E ou x ∈ E 0 (Teorema 8).
Se x ∈ E e se x é o i-ésimo termo de E , então N é innito porque contém o conjunto innito
Ni , pois xn = x para todo n ∈ Ni . Se x ∈ E 0 , então (x − ε, x + ε) ∩ E é innito (item 3. do
Teorema 7). Consideremos a associação

(x − ε, x + ε) ∩ E 3 y 7→ min{n ∈ N; xn = y} ∈ N.
Ela é injetiva, pois se y e z são elementos distintos de E, e se y
i-ésimo e o j -ésimo
e z são o
elementos de E (respectivamente), então: i 6= j , y = xm se, e somente se, m ∈ Ni e z = xn se,
e somente se, n ∈ Nj , com m 6= n uma vez que Ni ∩ Nj = ∅ por hipótese. Logo, N é innito.

Segunda Parte. Pelo Teorema 6, existe um subconjunto enumerável E contido emF


tal queF ⊂ E . Pelo Exercício 11 (página 164), como E ⊂ F, temos também que E ⊂ F.
Portanto, E é um conjunto enumerável tal que F = E. Agora F é também o conjunto dos
pontos de aderência de uma sequência como a obtida na primeira parte. 

5.16 Exercício 16
Com a notação do Exercício 4, se α é irracional, os conjuntos F = Z e G = α Z são fechados
porém F +G não é fechado. Também H = {0, 1, 1/2, . . . , 1/n, . . .} é fechado mas F · H não é
fechado.
168 CAPÍTULO 5. TOPOLOGIA DA RETA

[ Sugestão : Exercício 58 do Capítulo III.]

Solução. O conjunto G é fechado porque seu complementar é uma reunião de intervalos


abertos. Pelo Exercício 58 do Capítulo III (página 110), temos F + G = R, uma vez que, para
todo x ∈ R, a interseção (x − ε, x + ε) ∩ (F + G) é não vazia. Por outro lado, F + G 6= R. Por
exemplo, nenhum número racional não inteiro pertence a F +G (Exercício 27 do Capítulo III,
página 81). Portanto,F + G 6= F + G, ou seja, F + G não é fechado.
H é fechado, pois, pelo Exercício 12 (página 164), temos: H = H ∪ {0} = H ,
O onde
0 = lim 1/n. O F · H não é fechado porque F · H = Q. 

5.17 Exercício 17
Seja K o conjunto de Cantor. Mostre que {|x − y|; x ∈ K, y ∈ K} = [0, 1].
[ Sugestão : Observe que o conjunto dos |x − y|, com x, y ∈ K , é compacto e convença-se de
que ele contém todas as frações próprias cujos denominadores são potências de 3.]

Solução.

Nível 1. Vamos chamar de X esse conjunto {|x−y|; x ∈ K, y ∈ K},no conjunto de todasoas


m
distâncias entre dois pontos de K . Seguindo a sugestão dada, seja T = ; m ∈ Z, n ∈ N ∩
3n
(0, 1) o conjunto de todas as frações próprias cujos denominadores são potências de 3. Por
um lado armamos que T ⊂ X (Nível 2.1). Por outro, armamos que X é compacto (Nível
2.2). X é fechado, portanto, T ⊂ X (Exercício 11, página 164). Além
Sendo compacto,
disso, pelo Exercício 43 do Capítulo III (página 92), podemos concluir também que [0, 1] ⊂ T .
De [0, 1] ⊂ T ⊂ X , segue [0, 1] ⊂ X . Que X ⊂ [0, 1] é imediato da construção de K :
K ⊂ [0, 1] ⇒ 0 6 |x − y| 6 1 para todos x e y em K . Logo X = [0, 1] como queríamos.

No elevador. Vamos demonstrar algo um pouco mais forte do que o sugerido, a saber, que
n
todas as frações próprias de denominadores 3 (para um dado n) são obtidas como distâncias
de extremos de intervalos que permaneceram logo após a etapa n da construção do conjunto
de Cantor. Na demonstração por indução, o que vai nos ajudar a cair na hipótese de indução
n+1
é considerar a divisão euclidiana do numerador da fração m/3 por 3. A ideia vai ser a de,
n+1
a partir de pontos da etapa anterior, podermos nos afastar r/3 , onde r é o resto da divisão
n+1
de m por 3, indo r/3 para a direita ou para a esquerda, mas isso só será possível se houver
espaço para tal.

Nível 2.1. Seja Xn o conjunto dos pontos da forma |x − y|, onde x e y são extremos
de intervalos remanescentes logo após a etapa n. Por indução em n, vamos demonstrar que
nm o
Xn = ∈ R; m ∈ {0, 1, . . . , 3n } .
3n
Para n = 1, a armação segue porque um terço, dois terços, três terços e o zero pertencem
a K.
m
Vamos supor agora que a armação seja verdadeira para n. Dado , com m natural
3n+1
n+1
entre 0 e 3 , escrevamos m = 3k + r, na divisão euclidiana de m por 3, onde r = 0, r = 1 ou
n
r=2 e k é um natural entre 0 e 3 . Assim:

m k r
= + n+1 .
3n+1 3n 3
5.17. EXERCÍCIO 17 169

Pela hipótese de indução, existem x, y ∈ K , extremos de intervalos que permaneceram logo


k
após a etapa n, tais que |x − y| = . Claramente |x − y| ∈ Xn+1 , o que encerra o caso em que
3n
r = 0. Vamos supor, spg, x < y . Se x é um extremo à direita (de um intervalo que permaneceu
1
logo após a etapa n), então podemos considerar a distância entre y e x − ou entre y e
3n+1
2 1 2
x − n+1 , onde ambos x − n+1 e x − n+1 são extremos de intervalos que permaneceuram logo
3 3 3
após a etapa n + 1, o que nos dá:
 r  r
y − x − n+1 = |y − x| + n+1 .

3 3
r
Análogo se y é um extremo à esquerda, considerando agora a distância entre x e y + .
3n+1
Se x é um extremo à esquerda e y é um extremo à direita de intervalos que permaneceram
logo após a etapa n, fazemos o seguinte. Olhamos para o extremo mais próximo à esquerda de
x e para o extremo mais próximo à direita de y . Sejam eles a e b respectivamente; resulta que
a e b são respectivos extremos à direita e à esquerda de intervalos que permaneceram na etapa
1 1
n. A distância entre x e a é da forma p e a distância entre y e b é da forma q , onde p e q
3 3
1 1
são menores do que ou iguais a n. Vamos supor 6 q , ou seja, q 6 p 6 n. Indo para a
3p 3
etapa p, podemos concluir que existe c, x 6 c < y tal que c e y são extremos de um intervalo
1 1
de comprimento . Assim, subtraindo a ambos x e y , chegamos aos pontos a e c, onde
3p 3p
|a − c| = |x − y|, além de a e c satisfazerem as hipóteses do parágrafo anterior. Algo análogo
1 1
pode ser feito se for
q
< p . Segue a tese.
3 3

No elevador. No Nível 1, do fato de X ser compacto, o que importou mesmo foi X ser
fechado. Podemos então tentar demonstrar apenas isso: que X é fechado. Mas isso pode nos
levar a algumas diculdades. Por exemplo: se a sequência (|xn − yn |) é convergente, então são
convergentes cada uma das sequências (xn ) e (yn )? Porque, se forem, então poderemos armar
que lim |xn − yn | = | lim xn − lim yn |. Porém não temos muitas garantias de que a resposta da
pergunta feita seja positiva. De fato, ela pode ser negativa. Acontece que existe um modo de
mostrar que X é compacto sem passar por essas diculdades.

Nível 2.2 Armamos que uma dada sequência (|xn − yn |)n∈N , com xn , yn ∈ K para todo
n, admite uma subsequência que converge para um ponto de X (Nível 3). Assim, pelo item 4
do Teorema 11, segue que X é compacto.

No elevador. Como K é compacto, de imediato encontramos duas sequências (xn )n∈I e


(yn )n∈J convergentes, com I e J innitos. Mas o cuidado que devemos tomar é que, casos esses
conjuntos de índices sejam, digamos, desencontrados, não podemos armar que a sequência
dos |xn − yn | com, por exemplo, n variando em I é convergente.

Nível 3. A sequência do Nível 2.2 nos dá uma sequência (xn )n∈N de pontos xn em K .
Como K é compacto, existe um conjunto innito N1 de naturais tal que (xn )n∈N1 converge
para um ponto de K. Olhando agora para a sequência (yn )n∈N1 , do mesmo modo encontramos
um subconjunto innito N2 de N1 tal que a sequência (yn )n∈N2 converge para um ponto de
K. A subsequência (xn )n∈N2 ainda converge para o mesmo ponto para o qual convergia a
170 CAPÍTULO 5. TOPOLOGIA DA RETA

sequência (xn )n∈N1 . Sejam x = lim xn e y = lim yn ; ambos x, y ∈ K . Armamos agora que
n∈N2 n∈N2
a subsequência (|xn − yn |)n∈N2 converge para |x − y| (Nível 4), ponto este que pertence a X,
como queríamos.

Nível 4: Lema. Sejam (xn ) e (yn ) sequências convergentes. Sejam lim xn = x e lim yn =
y. Então a sequência (|xn − yn |) é convergente e lim |xn − yn | = |x − y|.
Solução. Se x = y, temos

0 6 |xn − yn | 6 |xn − x| + |x − y| + |y − yn | = |xn − x| + |y − yn |,

onde |xn − x| e |yn − y| convergem para zero e, portanto, pelo Teorema 8 do Capítulo IV,
lim |xn − yn | = 0 = |x − y|.
Se x 6= y , suponhamos, sem perda de generalidade, x < y . Existe n0 tal que xn < yn para
todo n > n0 (por exemplo, a partir do momento em que xn ca menor do que (x + y)/2 e yn
ca maior do que (x + y)/2). Assim, |xn − yn | = yn − xn , se n > n0 . Portanto

lim |xn − yn | = lim(yn − xn ) = y − x = |x − y|,

como queríamos. 

5.18 Exercício 18
Dado qualquer número real a > 0, existem x1 , x2 , . . . , xn no conjunto de Cantor tais que x1 +
· · · + x n = a.
[ Sugestão : O exercício anterior.]

Solução.

No elevador. O exercício anterior nos ensina que é possível escrever um número entre 0 e
1 como uma uma diferença entre dois números do conjunto de Cantor. O 1 é um número desse
conjunto, então podemos nos aproximar o máximo possível do a somando o número 1 consigo
mesmo quantas vezes forem necessárias, o que nos dará a = n + x, com x entre zero e um. Para
esse x teremos x = x2 − x1 , com x2 , x1 ∈ K e x2 > x1 . Assim, a = n + (x2 − x1 ). Porém agora
temos um problema com esse sinal de menos, mas ele pode sumir, escrevendo, por exemplo,
a = (n − 1) + x2 + (1 − x1 ), onde 1 − x1 é um número positivo. A questão agora é: esse 1 − x1
pertence a K ?

Nível 1.1. a > 1, seja n o maior número natural menor do que ou igual a a, n > 1.
Se
Para x = a − n, temos a = n + x, onde n é uma soma x1 + x2 + · · · + xn , com cada xi ∈ K ;
basta fazer xi = 1 para todo i. Além disso, x ∈ [0, 1] pela maximalidade de n. Assim, pelo
exercício anterior, existem x1 , x2 ∈ K , com x1 6 x2 tais que x = x2 − x1 . Agora temos:
a = n + x = (n − 1) + x2 + (1 − x1 ), observando que n > 1 (e, portanto, n − 1 > 0; o 0 pertence
ao conjunto de Cantor), pois estamos supondo a > 1. Armamos que 1 − y ∈ K , se y ∈ K (a
justicativa disso está no Nível 2.1). Portanto, temos o a escrito como uma soma de números
que pertencem à K (o n − 1 continua sendo uma tal soma), como queríamos.
5.18. EXERCÍCIO 18 171

No elevador. a < 1. No Nível 1.1, o que nos ajudou


Falta agora resolver o caso em que
foi um outro número de K , o 1. Podemos tentar introduzir o 1 novamente em nossas contas e
escrever a = x2 − x1 = (x2 − 1) + (1 − x1 ). Sabemos que 1 − x1 ∈ K , mas o x2 − 1 ∈
/ K , pois ele
é negativo. Isso pode ser sanado introduzindo agora, não o 1, mas um c ∈ K entre o x2 e o x1 e
escrever a = x2 − x1 = (x2 − c) + (x1 − c). Agora ambas essas parcelas são positivas, porém não
sabemos se elas pertencem a K . Tentamos demonstrar que sim do mesmo modo como no Nível
2.1, mas isso parece car cada vez mais complicado. Porém, conforme vamos trabalhando nessas
tentativas, observamos que os números de K que pertencem ao primeiro terço de cada etapa
se comportam do mesmo modo como os números de K que pertencem a todo o intervalo [0, 1]
com relação a pertinência 1 − x1 ∈ K caso x1 ∈ K . Então extrapolamos esse comportamento e,
em vez do 1, nos aproximamos do a o máximo possível através de múltiplos de alguma fração
1
do tipo . Essa ideia, além de funcionar, inclusive engloba o caso a 6 1. Porém mantemos o
3p
Nível 1.1 porque é ele quem nos ajudou a chegar a esta ideia.

Nível 1.2. Sejam


 
1 n m o
p = min m ∈ N ∪{0}; m 6 a e q = max m ∈ N; p 6 a .
3 3
m
O q
vale pelo menos 1, pois certamente pelo menos um número da forma 1/3 menor ou
m
igual a a existe (este 1/3 podendo inclusive ser igual a 1). Além disso, pela maximalidade
q 1
de q, o número x denido por x = a − p é não negativo e estritamente menor do que p .
3  3
1
Armamos agora que existem x2 , x1 ∈ 0, p ∩ K , com x2 > x1 tais que x = x2 − x1 (Nível
3
2.2). Assim escrevemos:
 
q 1 1
a = p + x = (q − 1) · p + x2 + − x1 .
3 3 3p
1
Armamos também que − x1 ∈ K (Nível 2.3), o que conclui a demonstração (o número 1/3p
3p
pertence a K ).

Nível 2.1. Para qualquer x ∈ K , o número 1 − x pertence a K . Isso por causa da simetria
que existe nesse conjunto com relação ao ponto 1/2:

1 1
− x = (1 − x) − .
2 2
(se x > 1/2, então 1 − x < 1/2).
Essa simetria já começa na primeira etapa da construção desse conjunto: o intervalo [0, 1/3]
é simétrico ao intervalo [2/3, 1] com relação ao ponto médio do segmento [0, 1]. Se essa simetria
existe numa etapa n da construção do conjunto de Cantor, então ela também existe na etapa
n + 1. Isto pode ser visualizado do seguinte modo. Sejam I e J intervalos simétricos com
relação ao 1/2, remanescentes na etapa n. O terceiro terço do intervalo I é simétrico, com
relação ao meio, ao primeiro terço do segmento J , e o primeiro terço do segmento I , ao terceiro
do segmento J. Isto mostra que a referida simetria existe em cada uma das etapas da construção
de K.
Se x K , para cada n, x não foi retirado na etapa n. Por causa da simetria com
pertence a
relação ao meio, se x não foi retirado na etapa n, então o 1 − x também não foi. Portanto o
1 − x também não foi retirado em nenhuma das etapas, logo, 1 − x ∈ K , como queríamos.
172 CAPÍTULO 5. TOPOLOGIA DA RETA

Nível 2.2. Assim como no exercício anterior, armamos que:

     
1 1
X = |x − y|; x, y ∈ 0, p ∩ K = 0, p ,
3 3

qualquer que seja o número natural p. Podemos justicar isto simplesmente repetindo aquela
mesma demonstração, sendo o conjunto T das frações menores do que ou iguais a 1/3p cujos
denominadores são potências de 3:

  n
1 m n m n−p
o
T = · ; n ∈ N, m ∈ {0, 1, . . . , 3 } = ; n ∈ N, n > p, m ∈ {0, 1, . . . , 3 } .
3p 3n 3n

Há dois modos de mostrarmos que T ⊂ X. Um é por indução, do mesmo modo como feito
lá. O outro é partir do resultado daquele Nível 2.1 e observar que se x e y são extremos de
x y
intervalos da etapa n, então e são extremos de intervalos da etapa n + p.
3p 3p

 
1 1
Nível 2.3. Assim como no Nível 2.1 deste exercício, armamos que − x ∈ 0, p ∩ K
  3p 3
1
se x ∈ 0, ∩ K. A demonstração é a mesma do referido Nível, onde agora a simetria é com
3p
1
relação ao ponto . 
2 · 3p

5.19 Exercício 19
Seja K o conjunto de Cantor. Dado ε > 0 arbitrário, existem intervalos abertos J1 =
X n
(a1 , b1 ), . . . , Jn = (an , bn ) tais que K ⊂ J1 ∪ J2 ∪ · · · ∪ Jn e (bi − ai ) < ε.
i=1

Solução. Logo após a etapa m da construção do conjunto de Cantor, sobram 2m intervalos


1
fechados, cada um com comprimento igual a , portanto a soma de seus comprimentos é igual
 m 3m
2
a . Além disto, o conjunto de Cantor está contido nesses intervalos, os quais podem ser
3  m
2
cobertos com intervalos abertos cuja soma de seus comprimentos é tão próxima de
3
quanto se queira.
 m  m
2 2
Assim, dado ε > 0, seja m ∈ N tal que < ε, seja 0 < δ < ε − e, para
3 3
n = 2m , sejam os intervalos abertos (a1 , b1 ), . . . , (an , bn ) que cobrem os 2m intervalos fechados
Xn
que sobram após a m-ésima etapa da construção do conjunto de Cantor e tais que (bi −ai ) 6
 m i=1
2
+ δ < ε. Por exemplo, para cada i ∈ {1, . . . , 2m }, dado o intervalo [xi , yi ], denindo
3
δ δ
ai = x i − m
e bi = yi + . 
2·2 2 · 2m
5.20. EXERCÍCIO 20 173

5.20 Exercício 20
Para X, Y ⊂ R quaisquer, tem-se X ∪Y = X ∪Y e X ∩Y ⊂ X ∩Y. Dê exemplo no qual a
inclusão não se reduz a uma igualdade.

(Voltar para a Solução 1 do Exercício 41 do Capítulo IX: página 401.)

Solução. Temos: X ⊂ X ∪ Y ⇒ X ⊂ X ∪ Y e, de modo análogo, Y ⊂ X ∪ Y , portanto,


X ∪ Y ⊂ X ∪ Y . Reciprocamente, se x ∈ / X ∪ Y , então x ∈
/X ex∈ / Y e existem a e b positivos
tais que (x − a, x + a) ∩ X = ∅ e (x − b, x + b) ∩ Y = ∅. Assim, tomando ε = min{a, b}, temos
(x − ε, x + ε) ∩ (X ∪ Y ) = ∅, isto é, x ∈
/ X ∪ Y . Isto mostra que X ∪ Y ⊂ X ∪ Y (lembre que
R −S ⊂ R −T ⇒ T ⊂ S quaisquer que sejam os subconjuntos S, T ⊂ R, propriedade C2) da
página 10 do livro).
Se x ∈ X ∩ Y , dado um intervalo aberto I ao qual x pertence, temos I ∩ (X ∩ Y ) 6= ∅;
assim, como I ∩ (X ∩ Y ) = (I ∩ X) ∩ (I ∩ Y ), ambos os conjuntos I ∩ X e I ∩ Y são não vazios.
Portanto x ∈ X ∩ Y .
Sejam X = (0, 1) e Y = (1, 2). Então: X ∩ Y = ∅ = ∅, enquanto X ∩Y = [0, 1]∩[1, 2] = {1},
ou seja, X ∩ Y 6⊂ X ∩ Y . 

5.21 Exercício 21
Um conjunto A⊂R é aberto se, e somente se, A∩X ⊂A∩X para todo X ⊂ R.

Solução. Suponha A aberto e seja x ∈ A∩X . Dado um intervalo aberto I ao qual x pertence,
queremos ver que I ∩ (A ∩ X) 6= ∅. Seja J um intervalo aberto tal que x ∈ J e J ⊂ I ∩ A (o
qual existe porque x ∈ I ∩ A e I ∩ A é aberto). Este J também satisfaz J ∩ X 6= ∅, pois x ∈ X .
Assim, para y ∈ J ∩ X , temos y ∈ J ⊂ I ∩ A e y ∈ X , ou seja, y ∈ (I ∩ A) ∩ X = I ∩ (A ∩ X)
e, portanto, I ∩ (A ∩ X) 6= ∅, como queríamos.
Reciprocamente, suponha que A não seja aberto; assim existe x ∈ A tal que (x − ε, x +
ε) ∩ (R −A) 6= ∅ para todo ε > 0. Em outras palavras isso signica que x ∈ R −A. Por um
lado temos então x ∈ A ∩ R −A, mas, por outro, A ∩ (R −A) = ∅ = ∅. Portanto, se A não é
aberto, então existe um conjunto X (a saber, X = R −A) tal que A ∩ X 6⊂ A ∩ X . De modo
equivalente (contrapositiva): A ∩ X ⊂ A ∩ X para todo conjunto X ⊂ R implica A aberto,
como queríamos. 

Observação. A ideia do segundo parágrafo desta solução nos permite concluir que

R− int(S) = R −S,

qualquer que seja o subconjunto S ⊂ R.


(Voltar para a Solução 2 do Exercício 25: página 176.)

(Voltar para a Solução do item 1 do Exercício 42: página 183.)

(Voltar para a Solução do item 8 do Exercício 42: página 184.)


174 CAPÍTULO 5. TOPOLOGIA DA RETA

Usando esta igualdade, podemos resolver uma parte do Exercício 20 a partir do Exercício
5, ou o Exercício 5 a partir do 20: dados os subconjuntos S e T de R,

int(S ∩ T ) = int(S) ∩ int(T ) ⇔ R − int(S ∩ T ) = R − int(S) ∩ int(T )
 
⇔ R− int(S ∩ T ) = R − int(S) ∪ R − int(T )

⇔ R −(S ∩ T ) = R −S ∪ R −T
⇔ (R −S) ∪ (R −T ) = R −S ∪ R −T ,

e;

int(S ∪ T ) ⊃ int(S) ∪ int(T ) ⇔ R − int(S ∪ T ) ⊂ R − int(S) ∪ int(T )
 
⇔ R− int(S ∪ T ) ⊂ R − int(S) ∩ R − int(T )

⇔ R −(S ∪ T ) ⊂ R −S ∩ R −T
⇔ (R −S) ∩ (R −T ) ⊂ R −S ∩ R −T .

Assim, dados X, Y ⊂ R, para S = R −X e T = R −Y , temos X ∪ Y = X ∪Y e X ∩ Y ⊂ X ∩


Y. Além disto, S = R −(0, 1) e T = R −(1, 2) são tais que a inclusão int(S)∪ int(T ) ⊂ int(S ∪T )
é própria.

5.22 Exercício 22
\
Sejam F1 ⊃ F2 ⊃ · · · ⊃ Fn ⊃ . . . não-vazios. Dê exemplos mostrando que Fn pode ser vazio
n∈N
se os Fn são apenas fechados ou apenas limitados.

Solução. No Exercício 48 do Capítulo III (página 95) já vimos exemplos assim. 

5.23 Exercício 23
Um conjunto não-vazio X ⊂R é um intervalo se, e somente se, satisfaz a condição seguinte:
 a, b ∈ X, a < x < b ⇒ x ∈ X .

(Voltar para a Solução do Exercício 20 do Capítulo VII: página 245.)

(Voltar para a Solução do Exercício 51 do Capítulo VIII: página 335.)

(Voltar para a Solução 1 do Exercício 22 do Capítulo X: página 424.)

Solução. Claramente um intervalo possui essa propriedade. Reciprocamente, seja X um


conjunto não vazio com essa propriedade.
X não é limitado nem inferiormente, nem superiormente, dado um número real x, sejam
Se
a, b ∈ X tais que a < x < b. Então, por hipótese, x ∈ X . Portanto R ⊂ X e R = X .
Se X é limitado inferiormente e não é limitado superiormente, então X ⊂ [inf X, +∞). Dado
x > inf X , existem a, b ∈ X tais que a < x < b. O a existe porque x é maior do que o ínmo de X
e o b existe porque X não é limitado superiormente. Então x ∈ X . Portanto (inf X, +∞) ⊂ X .
Se inf X ∈ X , então X = [inf X, +∞). Se inf X ∈ / X , então X = (inf X, +∞).
De modo análogo demonstramos o caso em que X é limitado superiormente e não é limitado
inferiormente e o caso em que X é limitado superiormente e inferiormente. De qualquer modo
X é um intervalo. 
5.24. EXERCÍCIO 24 175

5.24 Exercício 24
Mostre que a interseção de uma sequência descendente I1 ⊃ I2 ⊃ · · · ⊃ In ⊃ . . . de intervalos é
um intervalo ou o conjunto vazio.

Solução 1. Seja I essa interseção. Vamos supor I 6= ∅. Se I tem apenas um elemento, então
I é um intervalo (degenerado). Se I possui pelo menos dois elementos, sejam a, b ∈ I e seja
a < x < b. Pelo exercício anterior, vamos mostrar que x ∈ I . Fixado n ∈ N, temos a, b ∈ In ,
porque a, b ∈ Im para todo m, e x ∈ In porque a < x < b, a, b ∈ In e In é um intervalo.
Portanto x ∈ In para todo n, ou seja x ∈ I , como queríamos. 

Observação. Nesta solução não foi usada a hipótese de a sequência dos In 's ser descendente.
Por ela, podemos concluir que o resultado continua válido para qualquer sequência de intervalos.
Um modo de usar uma hipótese semelhante, ainda através do Exercício 23, seria no caso da
reunião de uma sequência ascendente de intervalos (a reunião de uma descendente se resumiria
ao primeiro intervalo), sem usar o Lema do Teorema 2 claramente. Seja I essa reunião. Dados
a, b ∈ I , existem m e n tais que a ∈ Im e b ∈ In . Se m < n, então a, b ∈ In . Portanto,
a < x < b ⇒ x ∈ In ⇒ x ∈ I .
\
Solução 2. Seja I= In . Se In não é limitado inferiormente nem superiormente para todo
n∈N
n ∈ N, então In = R para todon ∈ N e I = R. Se In não é limitado inferiormente para todo
n ∈ N e existe m ∈ N tal que Im é limitado superiormente, então In é limitado superiormente
para todo n ∈ M := N ∩[m, +∞). Neste caso, para cada n ∈ M , seja bn = sup In e a sequência
(bn )n∈M é não crescente. Se ela não é limitada inferiormente, então I = ∅. Se ela é limitada
inferiormente, sendo b = lim bn , teremos I = (−∞, b) ou I = (−∞, b]. O caso em que nenhum
In é limitado superiormente e o caso em que existem m ∈ N e n ∈ N tais que Im é limitado
superiormente e In é limitado inferiormente (então Ik é limitado superior e inferiormente para
todo k > max{m, n}) são semelhantes. 

Observação. Dada uma sequência (In )n∈N qualquer de intervalos, seja a sequência (Jn )n∈N
denida por Jn = I1 ∩ · · · ∩ In para todo n ∈ N. Esta nova sequência é descendente e é tal que
\ \
Jn = In . Além do que todo Jn é um intervalo ou algum Jn é o conjunto vazio. Portanto,
n∈N n∈N
se o resultado deste exercício é válido para sequências descendentes de intervalos, então ele é
válido para qualquer sequência de intervalos.

5.25 Exercício 25
Um conjunto é denso em R se, e somente se, seu complementar tem interior vazio.

Solução 1. De modo equivalente, dado um conjunto S ⊂ R, vamos mostrar que

int(R −S) 6= ∅ ⇔ S 6= R .
Se x ∈ int(R −S), seja o intervalo aberto I tal que x ∈ I ⊂ R −S , portanto, I ∩ S = ∅, ou
seja x ∈
/ S . Logo R 6= S .
Se R 6= S , seja x ∈ R, x ∈/ S (S ⊂ R). Assim, existe um intervalo aberto I ao qual x
pertence tal que I ∩ S = ∅ e, então, I ⊂ R −S . Portanto x ∈ int(R −S). 
176 CAPÍTULO 5. TOPOLOGIA DA RETA

Solução 2. Pelo início da observação após o Exercício 21 (página 173), junto com o fato de
que X = Y ⇔ R −X = R −Y , dado S ⊂ R, temos

∅ = int(R −S) ⇔ R = R − int(R −S) = R −(R −S) = S. 

5.26 Exercício 26
Se F é fechado e A é aberto então F −A é fechado.

(Voltar para a Solução do Exercício 2 do Capítulo VII: página 232.)

Solução 1. O conjunto F −A é fechado porque ele é uma interseção de fechados:

F − A = F ∩ (R −A),

sendo R −A fechado de acordo com o Teorema 4. 

Solução 2. Também podemos concluir que F −A é fechado mostrando que seu complementar
é aberto:

R −(F − A) = (R −F ) ∪ A,
uma reunião de abertos. 

Observação. O conjunto A−F é aberto. Ele é uma interseção nita de abertos: A−F =
A ∩ (R −F ). Seu complementar é fechado, ele é uma reunião nita de fechados: R −(A − F ) =
(R −A) ∪ F .

5.27 Exercício 27
Dê exemplo de um aberto A tal que A⊃Q mas R −A seja não-enumerável.

Solução. Podemos aproveitar o Exemplo A aqui. Sejam A1 = (−∞, 0) e A2 = (1, +∞). Só


falta agora cobrir os números racionais do intervalo [0, 1]. Fazemos como no referido exemplo:
sejam {r1 , r2 , . . . , rn , . . .} uma enumeração para os racionais deste intervalo e, para cada natural
1
n, (an , bn ) um intervalo aberto centrado em rn e com comprimento n+1 , de modo que, pelo
[ 2[
Exemplo B, [0, 1] − (an , bn ) é não enumerável. Seja A = A1 ∪ A2 ∪ (an , bn ). Claramente
n∈N n∈N
A é aberto e Q ⊂ A. O R −A é não enumerável porque

[
[0, 1] − (an , bn ) ⊂ R −A. 
n∈N

5.28 Exercício 28
Dê exemplo de um conjunto fechado, não-enumerável, formado apenas por números transcen-
dentes.
5.29. EXERCÍCIO 29 177

Solução. Podemos simplesmente repetir o que foi feito nos Exemplos A e B, apenas trocando
racionais por algébricos, usando que o conjunto dos números algébricos também é enumerável
(Exercício 44 do Capítulo III, página 93). Isto, aliás, também dá um outro exemplo para o
exercício acima, uma vez que o conjunto dos números racionais está contido no conjunto dos
números algébricos. 

5.29 Exercício 29
Dena a distância de um ponto a ∈ R a um conjunto não-vazio X ⊂ R como d(a, X) =
inf{|x − a|; x ∈ X}.
Prove:

1) d(a, X) = 0 ⇔ a ∈ X ;
2) Se F ⊂R é fechado, então para todo a∈R existe b∈F tal que d(a, F ) = |b − a|.

(Voltar para a Solução do Exercício 24 do Capítulo VII: página 250.)

Solução.

item 1) Para a ∈ R, como 0 6 |x − a| para todo x, temos:

a ∈ X ⇔ ∀ ε > 0 ∃x ∈ X, |a − x| < ε ⇔ d(a, X) = 0,


sendo a validade da primeira equivalência garantida pelo Teorema 3. 

item 2) Pelo Corolário 2 do Teorema 3, seja a sequência (|xn − a|)n∈N , com xn ∈ F para
todo n, tal que lim |xn − a| = d(a, F ). Sejam I = {n ∈ N; xn > a} e J = {n ∈ N; xn < a}.
Como I ∪ J = N, um dos dois, I ou J , é innito (neste caso, ser innito é equivalente a ser
ilimitado). Vamos supor que seja I , de modo que

d(a, F ) = lim |xn − a| = lim |xn − a| = lim(xn − a)


n∈I n∈I

(Teorema 2 do Capítulo IV), o que implica:

lim xn = a + d(a, F )
n∈I

(item 1. do Teorema 6 do Capítulo IV, pois xn = a + (xn − a)). Como F é fechado e (xn )n∈I
é uma sequência de pontos em F que converge para a + d(a, F ), segue a + d(a, F ) ∈ F . Para
b = a + d(a, F ), temos

|b − a| = |(a + d(a, F )) − a| = |d(a, F )| = d(a, F ).


Portanto, a + d(a, F ) é o ponto b∈F procurado, tal que d(a, F ) = |b − a|, caso I seja innito.
De modo análogo, se J é innito, então

d(a, F ) = lim a − xn ⇒ lim xn = a − d(a, F ) ∈ F,


n∈J n∈J

onde:
|(a − d(a, F )) − a| = | − d(a, F )| = d(a, F ).
Portanto a − d(a, F ) é o ponto procurado desta vez. Segue a tese. 
178 CAPÍTULO 5. TOPOLOGIA DA RETA

Observação. Na solução acima usamos particularidades de R; por exemplo, ao considerar


os pontos a + d(a, F ) e a − d(a, F ). Isto nos deu explicitamente candidatos para serem o b
procurado. Porém, o resultado deste exercício também é válido em casos mais gerais. Casos em
que os elementos do espaço d(a, F ) continuará
em questão podem não ser números reais. Neles,
sendo um número real, mas a poderá estar fora de R, o que tornará a expressão a + d(a, F ) sem
sentido. Alguma adaptação na solução acima nos dá soluções que podem ser generalizadas.
Seja (xn )n∈N ∈ F(N; F ) lim |xn − a| = d(a, F ) (Corolário 2 do Teorema 3). Vamos
tal que
ver que alguma subsequência de (xn ) converge; ela vai então convergir para um ponto b ∈ F
porque F é fechado; daí veremos que b é tal que |b − a| = d(a, F ). Pelo Teorema 2 do Capítulo
III (página 73), temos

|a| − |a − xn | 6 |a − (a − xn )| = |xn | = |a + (xn − a)| 6 |a| + |xn − a|.

Sejam A 6 B números reais tais que A 6 xn 6 B para todo n ∈ N. Eles existem porque
sequências convergentes são limitadas (Teorema 3 junto com o item 1. do Teorema 6, ambos,
do Capítulo IV). Portanto, xn pertence ao compacto [A, B] ∩ F para todo n ∈ N e, então,
alguma subsequência de (xn ) converge para um ponto b ∈ [A, B] ∩ F (Teorema 11). Seja (yn )
essa subsequência. Então temos

d(a, F ) = lim |xn − a| = lim |yn − a| = |b − a|

(Teoremas 2 e 8, ambos, do Capítulo IV; |b − a| − |b − yn | 6 |yn − a| 6 |yn − b| + |b − a| e


lim |yn − b| = 0), como queríamos.
Outra opção é desde o início já substituir F por um compacto K ⊂ F tal que d(a, K) =
d(a, F ) e tomar (xn )n∈N ∈ F(N; K) tal que lim |xn − a| = d(a, K). Da compacidade de K
obtemos a subsequência (yn )n∈N de (xn )n∈N tal que lim yn = b ∈ K e, então, b ∈ F . Esta
subsequência é tal que lim |yn − a| = d(a, K) e tal que lim |yn − a| = |b − a|, portanto, pela
unicidade do limite, |b − a| = d(a, K), ou seja, |b − a| = d(a, F ).
O compacto auxiliar (não vazio) K do parágrafo acima pode ser, por exemplo, K = F ∩
[a − (d(a, F ) + 1), a + (d(a, F ) + 1)]: d(a, K) > d(a, F ) pois K ⊂ F , e; ∀ ε > 0 ∃x ∈ F, |x − a| <
d(a, F ) + min{ε, 1}, o que implica x ∈ K e, portanto, d(a, K) 6 |x − a| < d(a, F ) + ε, isto é,
d(a, K) < d(a, F ) + ε ∀ ε > 0, logo d(a, K) 6 d(a, F ).

5.30 Exercício 30
Se X é limitado superiormente, seu fecho X também é. Além disso, sup X = sup X . Enuncie
e prove um resultado análogo para inf .

Solução.

Primeira Parte. X é limitado superiormente, seja b tal que X ⊂ (−∞, b). Disto
Se
segue X ⊂ (−∞, b) = (−∞, b], portanto X é limitado superiormente. O mesmo ocorre se X é
limitado inferiormente: se X é limitado inferiormente, então X é limitado inferiormente, pois
X ⊂ (a, +∞) ⇒ X ⊂ [a, +∞).

Segunda Parte. Como X ⊂ X,


X é limitado superiormente, então sup X 6 sup X
se
e, se X é limitado inferiormente, então inf X > inf X . Se X é limitado superiormente e se
sup X < sup X , então existe x ∈ X tal que x > sup X . Tomando ε = x − sup X , no intervalo
5.31. EXERCÍCIO 31 179

aberto (x − ε, x + ε) não existe nenhum elemento de X , pois x − ε > sup X , o que contraria
a hipótese de x ser aderente a X . Se X é limitado inferiormente e se inf X > inf X , então
existe x ∈ X tal que x < inf X . Tomando ε = inf X − x, não existe nenhum elemento de X no
intervalo aberto (x − ε, x + ε), pois x + ε 6 inf X , o que contraria a hipótese de x ser aderente
a X . Portanto sup X = sup X se X é limitado superiormente e inf X = inf X se X é limitado
inferiormente. 

5.31 Exercício 31
Para todo X⊂R limitado superiormente, sup X é aderente a X. Resultado análogo para inf .

Solução. Isto já está demonstrado no Corolário 2 do Teorema 3. 

5.32 Exercício 32
Para todo X ⊂ R, X 0 é fechado.

(Voltar para o item (a) da Primeira Parte da Solução do Exercício 64: página 203.)

Solução. Fixado x ∈ X 0, queremos concluir que x ∈ X 0. Para tanto, sejaI um intervalo


aberto ao qual x pertence. Vamos mostrar que existe um elemento em I ∩ X distinto de x.
0 0
Como x ∈ X 0 , temos I ∩ X 6= ∅. Seja então y ∈ I ∩ X . Assim, pelo item 3 do Teorema 7,
0
existem innitos pontos de X no intervalo I , porque I é um intervalo aberto possuindo y ∈ X .
Em particular algum deles é diferente de x, como queríamos. 

5.33 Exercício 33
Um número a é ponto de acumulação de X se, e somente se, é ponto de acumulação de X.

Solução. Sejaa ∈ X 0 e seja I um intervalo aberto ao qual a pertence. Existe b ∈ I ∩ X


0
distinto de a. Este b ∈ X já que X ⊂ X . Portanto a ∈ X .
0
Reciprocamente, seja a ∈ X e seja I um intervalo aberto ao qual a pertence. Existe
b ∈ I ∩ X distinto de a. Como b pertence a I e b é ponto aderente de X , existe c ∈ X ∩ I . Mas,
com o que temos até aqui, não há como garantir que este c é distinto de a, então estreitamos
nosso intervalo. Seja então J ⊂ I um intervalo aberto tal que b ∈ J e a ∈ / J (por exemplo, se
I = (x, y) e b > a, seja J = (a, y)). Agora existe c ∈ J ∩ X (J ∩ X 6= ∅ porque b ∈ X e J é
/ J , e c ∈ I , porque J ⊂ I . Portanto a ∈ X 0 , como
um aberto que possui b), c 6= a, porque a ∈
queríamos. 

Observação 1. Conforme o primeiro parágrafo da solução acima indica, algo semelhante ao


que acontece com o fecho acontece com os pontos de acumulação de um conjunto, a saber,

A ⊂ B ⇒ A0 ⊂ B 0 ,

se A, B ⊂ R, pois o ponto que uma vizinhança aberta de um elemento x ∈ A0 encontra em A


também pertence a B.
(Voltar para a Solução do Exercício 62: página 202.)
180 CAPÍTULO 5. TOPOLOGIA DA RETA

Observação 2. Com estes dois exercícios anteriores, temos o seguinte:

0
X0 = X0 = X .

A primeira igualdade pelo Exercício 32 e a segunda, pelo 33. Em outras palavras, podemos
dizer que os símbolos de barra e de linha comutam.

5.34 Exercício 34
(X ∪ Y )0 = X 0 ∪ Y 0 .

Solução 1. Conforme já observamos no exercício anterior (na Observação 1), imediatamente


temos:

X 0 ∪ Y 0 ⊂ (X ∪ Y )0 ,
pois ambos X0 e Y0 são subconjuntos de (X ∪ Y )0 , uma vez que X e Y são subconjuntos de
X ∪Y.
Reciprocamente, se a ∈ (X ∪ Y )0 , então existe uma sequência (an )n∈N de pontos dois a
dois distintos pertencentes a X ∪ Y que converge para a. Sejam I = {n ∈ N; an ∈ X} e
J = {n ∈ N; an ∈ Y }. Como I ∪ J = N, um dos dois, I ou J , é innito. Se I é innito, então
temos a sequência (an )n∈I de pontos dois a dois distintos pertencentes a X que converge para a
0 0 0 0 0
e, portanto, a ∈ X . De modo análogo, se J é innito, então a ∈ Y . Logo, (X ∪ Y ) ⊂ X ∪ Y ,
a inclusão que estava faltando para concluirmos a igualdade desejada. 

Solução 2 (para a inclusão (X ∪ Y )0 ⊂ X 0 ∪ Y 0 ). Se x ∈/ X0 e x ∈ / Y 0 , então existem


intervalos abertos I e J , ambos possuindo x, tais que I ∩ X ⊂ {x} e J ∩ Y ⊂ {x}. A interseção
L = I ∩ J continua sendo um intervalo aberto possuindo x; ela é tal que L ∩ X ⊂ {x} e
L ∩ Y ⊂ {x}. Portanto (L ∩ X) ∪ (L ∩ Y ), que é igual a L ∩ (X ∪ Y ), está contido em {x}.
Logo x ∈/ (X ∪ Y )0 . Isto mostra que R −(X 0 ∪ Y 0 ) ⊂ R −(X ∪ Y )0 , ou seja, (X ∪ Y )0 ⊂ X 0 ∪ Y 0
(propriedade C2) da página 10 do livro), como queríamos. 

5.35 Exercício 35
Todo ponto de um conjunto aberto A é ponto de acumulação de A.

Solução. a ∈ A e I um intervalo aberto tal que a ∈ I . Como A é aberto, I ∩ A contém


Sejam
algum intervalo J tal que a ∈ J . Este intervalo J é innito e todos seus elementos são também
0
elementos de A, portanto I possui uma innidade de elementos de A e, então, a ∈ A (item 3.
do Teorema 7), como queríamos. 

5.36 Exercício 36
Sejam F fechado e x ∈ F. Então x é um ponto isolado de F se, e somente se, F − {x} é ainda
fechado.

(Voltar para a Solução do Exercício 38: página 181.)


5.37. EXERCÍCIO 37 181

Solução. Suponhax isolado em F e seja y um real que não pertence a F − {x}. Queremos
mostrar que y∈/ F − {x}, ou seja, queremos encontrar um intervalo aberto I tal que y ∈ I e tal
que I ∩ (F − {x}) = ∅. Se y ∈/ F − {x}, então y ∈ / F ou y = x. Se y = x, então I é o intervalo
tal que I ∩ F = {x}. Se y ∈/ F , então o I é aquele intervalo aberto tal que y ∈ I e I ∩ F = ∅, o
qual existe porque R −F é aberto (Teorema 4): I ∩ (F − {x}) = ∅ pois I ∩ (F − {x}) ⊂ I ∩ F .

Reciprocamente, suponha F − {x} fechado. Em particular, x ∈ / F − {x} implica que x


não pertence ao fecho de F − {x}. Portanto existe um intervalo aberto I tal que x ∈ I e
I ∩ (F − {x}) = ∅. Este I é então tal que I ∩ F = {x}, ou seja, x é um ponto isolado de F . 

5.37 Exercício 37
Seja X ⊂ R tal que X 0 ∩ X = ∅. Mostre que existe, para cada x ∈ X, um intervalo aberto Ix ,
de centro x, tal que x 6= y ⇒ Ix ∩ Iy = ∅.

(Voltar para a Solução 1 do Exercício 40: página 182.)

(Voltar para a Solução do Exercício 43: página 184.)

Solução. Dado x ∈ X , como X ∩ X 0 = ∅, x ∈ / X 0 ; assim seja δ(x) > 0 tal que (x −


δ(x), x + δ(x)) ∩ X = {x}. Isto signica que, se x e y são elementos distintos de X , então
y∈/ (x − δ(x), x + δ(x)) e x ∈/ (y − δ(y), y + δ(y)). Mas ainda podemos ter (x − δ(x), x + δ(x)) ∩
(y − δ(y), y + δ(y)) 6= ∅, portanto esses intervalos precisam ser estreitados. Armamos que
δ(x) δ(x)
Ix = x − ,x + tem a propriedade desejada.
2 2
Dados x, y ∈ X distintos, podemos supor, spg, y < x. Assim basta vermos que y +
δ(y) δ(x) δ(x) δ(y)
6 x− . Isto é equivalente a x − y > + que, por sua vez, é equivalente a
2 2 2 2
2(x − y) > δ(x) + δ(y). E de fato é isto que ocorre, pois: y < x, y ∈ / (x − δ(x), x + δ(x)) e
x∈/ (y − δ(y), y + δ(y)) implicam y 6 x − δ(x) e x > y + δ(y), isto é, x − y > δ(x) e x − y > δ(y).
Logo, somando, vem 2(x − y) > δ(x) + δ(y), como queríamos. 

(Voltar para a Solução 1 do Exercício 63: página 202.)

5.38 Exercício 38
Seja F ⊂ R fechado, innito enumerável. Mostre que F possui uma innidade de pontos
isolados.

Solução. De modo equivalente, vamos demonstrar a contrapositiva deste exercício. Sejam


então x1 , x2 , . . . , xn todos os pontos isolados de um dado conjunto F (podendo também acon-
tecer de F não ter nenhum ponto isolado). Se F não é fechado, acabamos. Então supo-
nhamos F fechado. Assim como no Exercício 36 (página 180), armamos que o conjunto
X = F − {x1 , . . . , xn } é fechado (lema abaixo). Agora temos X fechado e sem pontos isolados.
Se X = ∅, então F = {x1 , . . . , xn } é um conjunto nito. Se X não é vazio, então aplicamos
o Teorema 9 para concluir que X não é enumerável, portanto, F = X ∪ {x1 , . . . , xn } não é
enumerável. Concluímos então que: F não é fechado ou F é nito ou F não é enumerável.
182 CAPÍTULO 5. TOPOLOGIA DA RETA

Lema. Se F é fechado, para um dado n, se {x1 , . . . , xn } é um conjunto de pontos isolados


de F, entãoF − {x1 , . . . , xn } é fechado.
Solução. Pelo já referido Exercício 36, sabemos que todos os conjuntos F − {x1 }, F − {x2 },
. . ., F − {xn } são fechados. Portanto, a interseção (nita)
  
F − {x1 } ∩ · · · ∩ F − {xn } = F − {x1 } ∪ · · · ∪ {xn } = F − {x1 , . . . , xn }

é também um conjunto fechado, como queríamos. 

5.39 Exercício 39
Mostre que todo número real x é limite de uma sequência de números transcendentes dois a
dois distintos.

Solução. Isto é consequência da combinação do Exercício 45 do Capítulo III (página 93) com
a equivalência entre os itens 2. e 3. do Teorema 7: o conjunto X dos números transcendentes
é o complementar do conjunto enumerável dos números algébricos (Exercício 44 do Capítulo
III, página 93), portanto, dado x ∈ R, se (a, b) é um intervalo aberto possuindo x, então, pelo
Exercício 45 do Capítulo III, (a, b) ∩ X é innito. 

5.40 Exercício 40
Mostre que se X⊂R não é enumerável, então X ∩ X 0 6= ∅.

Solução 1. X ∩ X 0 = ∅, para cada x ∈ X , seja Ix


Vamos demonstrar a contrapositiva. Se
um intervalo centrado em x e tal que Ix ∩ Iy = ∅ para x, y ∈ X se x 6= y (os intervalos do
Exercício 37, página 181). A coleção C de todos esses Ix é então de intervalos abertos não vazios
e dois a dois disjuntos, portanto C é enumerável (página 163). Segue então que X também é
enumerável, porque a associação x 7→ Ix , para cada x ∈ X , é uma bijeção. 

Solução 2. Se X ∩ X 0 = ∅, então, dado x ∈ X, temos / X 0,


x∈ ou seja, qualquer ponto de X
é isolado. Então, pela contrapositiva do Corolário 2 do Teorema 8, X é enumerável. 

5.41 Exercício 41
Se A e A ∪ {a} são abertos então a é ponto de acumulação de A à direita e à esquerda.

Solução. Se A ∪ {a} é aberto, como a ∈ A ∪ {a}, existe δ > 0 tal que (a − δ, a + δ) ⊂ A ∪ {a}.
Isto signica que se x ∈ (a − δ, a + δ) e x 6= a, então x ∈ A, ou, ainda, que:

(a − δ, a) ∪ (a, a + δ) ⊂ A.

É desta inclusão que segue o resultado desejado: no intervalo (a, a + δ) A) encon-


(contido em
tramos uma sequência decrescente de pontos dois a dois distintos pertencentes a A que converge
para a, donde concluímos que a é um ponto de acumulação à direita de A, e, em (a − δ, a)
encontramos uma sequência crescente que, de modo análogo, testemunha o fato de a também
ser um ponto de acumulação à esquerda de A. 
5.42. EXERCÍCIO 42 183

Observação. Na solução acima o que importou mesmo foi termos a ∈ int(A ∪ {a}); nem
mesmo a hipótese de A ser aberto foi usada.

5.42 Exercício 42
Dê explicitamente o signicado de cada uma das seguintes armações. Em suas explicações,
você está proibido de usar qualquer das palavras grifadas abaixo:

1) a ∈ X não é ponto interior de X;

2) a ∈ R não é aderente a X;

3) X ⊂ R não é um conjunto aberto ;


4) O conjunto Y ⊂ R não é fechado ;
5) a ∈ R não é ponto de acumulação do conjunto X ⊂ R;

6) X 0 = ∅;

7) X⊂Y mas X não é denso em Y;

8) int(X) = ∅;

9) X ∩ X 0 = ∅;

10) X não é compacto.

Solução.

item 1) Signica que a é um ponto aderente ao conjunto R −X , o complementar de X


em relação àR (ver o início da observação feita após o Exercício 21, na página 173), ou seja,
a ∈ X ∩ (R −X). 

item 2) Signica que a pertence ao conjunto R −X . Ou, ainda, que a é um ponto interior
do conjunto R −X . 

item 3) Existe um ponto a∈X satisfazendo o item 1) acima, ou seja, existe um ponto
no conjunto a ∈ X ∩ (R −X). 

item 4) Existe um ponto no conjunto Y − Y, ou ainda, um ponto aderente à Y que


pertence ao complementar de Y em relação à R. 

item 5) Se a ∈ X, então a é isolado em X. Se a∈


/ X (ou, para evitar o uso do não
encapsulado no símbolo do não pertence, podemos dizer que a pertence ao complementar
de X em relação à R, ou ainda, a ∈ R −X ), então a satisfaz o item 2) acima. Portanto,
a ∈ (X − X 0 ) ∪ (R −X). Podemos dizer também que existe um intervalo aberto I ao qual a
pertence tal que I ∩ X ⊂ {a}. 
184 CAPÍTULO 5. TOPOLOGIA DA RETA

item 6) Signica que X não tem pontos de acumulação (aqui a palavra não pode ser
usada). Assim, pelo item anterior, para todo número real a existe um intervalo aberto I tal
que a∈I e I ∩ X ⊂ {a}. 

Observação. Um cuidado que devemos tomar é que X0 ser vazio não signica que todos seus
pontos são isolados; um conjunto X pode ter apenas pontos isolados e ainda sim ser tal que
X 0 = ∅, por exemplo, o conjunto X = {1/n; n ∈ N}.

item 7) Signica que existe um ponto y no conjunto Y − X, o complementar do fecho de


X em relação à Y. 

item 8) Signica que X não contém nenhum intervalo aberto, isto é, I ∩ (R −X) 6= ∅
qualquer que seja o intervalo aberto I, ou seja, R −X é denso em R. O conjunto R −X é
também igual ao conjunto int(R −X). Isto é consequência do início da observação do Exercício
21, página 173:

R− int(S) = R −S ⇔ int(S) = R −R −S,

então, para S = R −X ,
int(R −X) = R −X,

como queríamos. O interior do fecho é vazio se, e somente se, o complementar do fecho é denso
se, e somente se, o interior do complementar do conjunto é denso. 

item 9) Signica que todo ponto de X é isolado. 

item 10) Signica que existe uma cobertura C por meio de abertos para X tal que toda
subcobertura de C é innita. 

5.43 Exercício 43
Se todo ponto de acumulação de X é unilateral, X é enumerável.

Solução.

Nível 1. Essa hipótese signica que, dado x ∈ X 0, existe ε(x) > 0 tal que (x, x + ε(x)) ∩
X =∅ ou (x − ε(x), x) ∩ X = ∅.
Já lidamos com algo semelhante anteriormente, a saber, na
0
solução do Exercício 37 (página 181). Vamos mostrar que o conjunto X ∩ X é enumerável
(Nível 2.1). É por isto que escrevemos:

X = X ∩ X 0 ∪ X ∩ (R −X 0 ) ,
 

onde todo ponto de Y := X ∩ (R −X 0 ) é isolado de Y (Nível 2.2), portanto Y é enumerável


(Corolário 2 do Teorema 8). Logo X , sendo uma reunião de dois conjuntos enumeráveis, é
enumerável, como queríamos.
5.44. EXERCÍCIO 44. TEOREMA DE LINDELÖF 185

 
0ε(x)
Nível 2.1. Assim como zemos no Exercício 37, dado x ∈ X ∩X , seja Ix = x − ,x
  2
ε(x)
ou Ix = x, x + dependendo se x for um ponto de acumulação à direita ou à esquerda,
2
respectivamente (onde ε(x) tem a propriedade já mencionada no Nível 1). Se x e y são pontos
0
distintos de X ∩ X , então Ix ∩ Iy = ∅ (pelo mesmo motivo do já referido Exercício 37). Assim
temos uma coleção de intervalos abertos não vazios e dois a dois disjuntos cuja cardinalidade é
0
a mesma de X ∩ X . Portanto, pelo Exercício 9 (página 163), essa coleção e, em consequência,
0
o conjunto X ∩ X só podem ser enumeráveis, como queríamos.

Nível 2.2. Dado x ∈ Y, temos que x é um ponto isolado de X porque x ∈ X e x ∈/ X 0.


Portanto existe um intervalo aberto I possuindo x tal que I ∩ X = {x}. A interseção I ∩ Y
também é igual a {x}, pois x ∈ I ∩ Y ⊂ I ∩ X = {x}. 

Observação. Na solução acima, não precisou todo ponto de X0 ser unilateral, bastou todo
ponto de X∩ X 0 o ser.

5.44 Exercício 44. Teorema de Lindelöf


Seja X ⊂R um conjunto arbitrário. Toda cobertura de X por meio de abertos possui uma
subcobertura enumerável (Teorema de Lindelöf ).

(Voltar para o Nível 2.2 da Solução do Exercício 61: página 201.)

(Voltar para a Solução 2 do Exercício 63: página 202.)

(Voltar para o item (b) da Primeira Parte da Solução do Exercício 64: página 203.)

(Voltar para o item (c) da Primeira Parte da Solução do Exercício 64: página 204.)

Solução.

No elevador. A propriedade mais importante possuída por R que resulta neste exercício é
a de qualquer um de seus subconjuntos abertos poder ser escrito como uma reunião de intervalos
abertos cujos extremos são números racionais. Isto junto com o fato de a coleção de todos estes
intervalos ser enumerável.

[
Nível 1. Seja (Aλ )λ∈L uma família de abertos tal que X⊂ Aλ . Seja {In ∈ P(R); n ∈
λ∈L
N} uma enumeração para todos os intervalos abertos cujos extremos são números racionais
(esta coleção é enumerável porque podemos identicá-la com um subconjunto de Q × Q).
Para cada λ ∈ L, armamos que existe um conjunto Mλ de números naturais tal que

[
Aλ = In
n∈Mλ

(Nível 2.1). Deste modo:


[
X⊂ In ,
n∈M
186 CAPÍTULO 5. TOPOLOGIA DA RETA

[
onde M= Mλ ⊂ N. Pelo fato de M ser enumerável (qualquer subconjunto de um conjunto
λ∈L
enumerável é enumerável, Corolário 1 do Teorema 8 do Capítulo II), armamos que existe um
[
subconjunto J de L enumerável tal que M = Mλ (Nível 2.2). Para este conjunto J temos

[ λ∈J
X⊂ Aλ (Nível 2.3), como queríamos.
λ∈J

Nível 2.1: Lema. Cada conjunto aberto não vazio de números reais pode ser escrito
como uma reunião enumerável de intervalos abertos cujos extremos são números racionais.

Solução. A um conjunto aberto não vazio de números reais. Dado x ∈ A, como A é


Seja
aberto, seja ε > 0 tal que (x − ε, x + ε) ⊂ A. Pela densidade dos números racionais em R,
existem a, b ∈ Q tais que x − ε 6 a < x < b 6 x + ε. Para tais números racionais temos
x ∈ (a, b) ⊂ A. Deste modo, podemos concluir que cada elemento de A pertence a um intervalo
aberto cujos extremos são números racionais, o qual, por sua vez, está contido em A.
Para cada x ∈ A, seja então Jx o intervalo contido em A cujos extremos são números
racionais e ao qual x pertence. Por um lado, armamos que

[
A= Jx
x∈A

(Nível 3). Por outro, a coleção{Jx ∈ P(R); x ∈ A} é enumerável, por ser subcoleção da coleção
enumerável {In ∈ P(R); n ∈ N} de todos os intervalos abertos cujos extremos são números
racionais. Logo A é uma reunião enumerável de intervalos abertos de extremos racionais, como
queríamos (talvez para enxergarmos um pouco melhor essa conclusão, seja M ⊂ N tal que
[
{Jx ; x ∈ A} = {In ; n ∈ M }; para este M , enumerável, temos A = In ). .
n∈M

[
Nível 2.2: Lema. Dada a família (Mλ )λ∈L , se M = Mλ é enumerável, então existe

[ λ∈L
J ⊂L enumerável tal que M= Mλ .
λ∈J
Solução. n ∈ M , como n ∈ Mλ para algum λ ∈ L, xemos λ(n) ∈ L tal que n ∈ Mλ(n) .
Dado
O conjunto de índices {λ(n) ∈ L; n ∈ M } é o conjunto J procurado. Ele é enumerável porque
o domínio da função λ é enumerável. Além disso, temos

[ [
Mλ ⊂ Mλ
λ∈J λ∈L

porque J ⊂L e
[ [
Mλ ⊂ Mλ
λ∈L λ∈J
[ [
por construção: n∈ Mλ ⇒ n ∈ Mλ(n) ⇒ n ∈ Mλ , pois λ(n) ∈ J .
λ∈L λ∈J

Nível 2.3. Referente ao Nível 1, dado x ∈ X, queremos encontrar um λ ∈ J tal que


x ∈ Aλ . Temos [
x ∈ X ⇒ x ∈ In para algum n∈M = Mλ ,
λ∈J
5.45. EXERCÍCIO 45 187

donde n ∈ Mλ para algum λ ∈ J . Lembrando da construção de cada Aλ feita no Nível 1 (como


uma reunião de intervalos In 's cujos índices variam em Mλ ), temos:

n ∈ Mλ ⇒ In ⊂ Aλ .

Portanto x ∈ Aλ , com λ ∈ J, como queríamos.

Nível 3. Se A ⊂ R é um aberto não vazio e se Jx é um intervalo cujos extremos são números


racionais tal que x ∈ Jx ⊂ A para todo x ∈ A, vamos mostrar que

[
A= Jx .
x∈A

[
De fato, por um lado Jx ⊂ A para todo x∈A implica Jx ⊂ A. Por outro, se a ∈ A, então

[ x∈A [
a ∈ Ja por construção, portanto, a∈ Jx , portanto A⊂ Jx . Destas duas inclusões segue
x∈A x∈A
a igualdade desejada. 

Observação/Curiosidade. Em outras palavras, a propriedade de R que nos permitiu fazer


as contas acima do modo como zemos é a seguinte: existe uma coleção enumerável B de abertos
tal que, se A é um aberto não vazio e se x ∈ A, então existe B∈B tal que x ∈ B ⊂ A. Se um
espaço tem essa propriedade dizemos que ele satisfaz o segundo axioma de enumerabilidade.

5.45 Exercício 45
Com a notação do Exercício 4, prove:

a) Se A é compacto e B é fechado então A+B é fechado;

b) se A e B são compactos, então A+B e A·B são compactos;

c) se A é fechado e B é compacto, A·B pode não ser fechado.

(Voltar para a Solução do Exercício 47: página 188.)

Solução.

item a) Sejam as sequências (xn )n∈N e (yn )n∈N , com xn ∈ A e yn ∈ B para todo n ∈ N.
Vamos supor que a sequência (xn + yn )n∈N converge para um número real a. Vamos mostrar
que a ∈ A + B.
Por A ser compacto, existe um conjunto innito M de números naturais tal que a sub-
sequência (xn )n∈M é convergente e converge para um ponto x pertencente a A. A subsequência
(xn + yn )n∈M continua convergindo para a. Como yn = (xn + yn ) − xn , podemos concluir que a
subsequência (yn )n∈M converge para a − x e que a − x ∈ B porque B é fechado. Agora temos
a = x + (a − x), com x ∈ A e a − x ∈ B . Logo a ∈ A + B , como queríamos. 
188 CAPÍTULO 5. TOPOLOGIA DA RETA

item b) Isto acontece por causa do item 4 do Teorema 11  para um conjunto K ser
compacto basta que qualquer sequência de pontos de K admita uma subsequência que converge
para um ponto de K.
(xn )n∈N e (yn )n∈N sequências de pontos em A e B respectivamente. Existe um con-
Sejam
junto innito M de números naturais tal que a sequência (xn )n∈M converge para um ponto
x ∈ A. Considerando agora a sequência (yn )n∈M , o mesmo acontece: existe um subconjunto
innito N de M tal que (yn )n∈N converge para um ponto y ∈ B . A subsequência (xn )n∈N não
deixou de convergir para x. Portanto (xn + yn )n∈N converge para x + y ∈ A + B e (xn · yn )n∈N
converge para xy ∈ A · B , como queríamos. 

item c) Por exemplo, A=N e B = {1}; A é fechado, B é compacto, mas A·B =N não
é compacto; N não é limitado. 

5.46 Exercício 46
Obtenha coberturas abertas de Q e de [0, +∞) que não admitam subcoberturas nitas.

Solução. Uma mesma cobertura serve para ambos os conjuntos dados: ((−n, n))n∈N . Dada
uma subfamília nita ((−n1 , n1 ), (−n2 , n2 ), . . . , (−nm , nm )) sua, seja n0 = max ni . Para este
16i6m
n0 temos (−n1 , n1 ) ∪ · · · ∪ (−nm , nm ) = (−n0 , n0 ). Existe um número racional x ∈ Q ∩[0, +∞)
maior do que n0 . 

5.47 Exercício 47
Considere as funções f, g, h do Exercício 7. Mostre que para K e L compactos arbitrários,
f (K), g(K), h(K), f −1 (L), g −1 (L) e h−1 (L) são compactos.

Solução. Do mesmo modo como zemos na solução do Exercício 7 (página 162), com a
notação do Exercício 4 (página 160), temos:

   
−1 1 b
f [K] = {a} · K + {b} e f [K] = ·K + − ,
a a

onde os 4 conjuntos unitários que aí aparecem são todos compactos. Portanto f [K] e f −1 [K]
são compactos pelo item b) do Exercício 45 acima (página 187).
2
Para vermos que g[K] e h[K] são compactos, sejam (xn )n∈N e (yn3 )n∈N sequências nesses
conjuntos, com xn e yn emn ∈ N. Como K é compacto, existem conjuntos
K para todo
innitos M e N de números naturais tais que (xn )n∈M e (yn )n∈N convergem para x ∈ K e
y ∈ K respectivamente. As subsequências (x2n )n∈M e (yn3 )n∈N convergem, respectivamente,
2 3
para x ∈ g[K] e y ∈ h[K] (item 2 do Teorema 6 do Capítulo IV). Logo g[K] e h[K] são
compactos, pelo item 4 do Teorema 11.
−1
Por m, vamos mostrar que g [L] e h−1 [L] são limitados e fechados.
Sejam a e b elementos
−1
que limitam L por baixo e por cima respectivamente. A imagem inversa h [L] de L por h é
√ √
limitada por baixo por
3
a e por cima por 3 b. √ L√por g , podemos
No caso da imagem inversa de
supor b > 0; esta imagem inversa é limitada por baixo por − b e por cima por b. Além disso,
temos:
/ g −1 [L] ⇒ g(x) ∈
x∈ / L ⇒ g(x) ∈ R −L ⇒ x ∈ g −1 [R −L],
5.48. EXERCÍCIO 48 189

onde g −1 [R −L] é um conjunto aberto pelo Exercício 7 (página 162) já que L − R é aberto. Ele
é tal que
g −1 [R −L] ∩ g −1 [L] = g[(R −L) ∩ L] = ∅
pelo item Inv2) do Exemplo 17 do Capítulo I. Portanto, x ∈/ g −1 [L], donde concluímos que
g −1 [L] é fechado. O h−1 [L] é fechado exatamente pelo mesmo motivo. Logo ambos g −1 [L] e
h−1 [L] são compactos, como queríamos. 

Observação 1. Uma outra demonstração de que h−1 [L] é compacto também pode ser obtida
através de uma combinação de exercícios anteriores. Armamos que se (Aλ )λ∈L é uma cobertura
−1
aberta para h [L], então (h[Aλ ])λ∈L é uma cobertura aberta para L. Deste modo, como L é
compacto, existem λ1 , · · · , λm ∈ L tais que

L ⊂ h[Aλ1 ] ∪ · · · ∪ h[Aλm ] = h[Aλ1 ∪ · · · ∪ Aλm ]

(esta igualdade vale pelo item I1) da página 17 do livro), o que implica:

h−1 [L] ⊂ Aλ1 ∪ · · · ∪ Aλm ,

poisX ⊂ Y ⇒ h−1 [X] ⊂ h−1 [Y ] (qualquer que seja a função h e os conjuntos X e Y no


−1
domínio de h) e h [h[X]] = X (qualquer que seja o conjunto X no domínio de h) pelo item 2
−1
do Exercício 14 do Capítulo I (página 12) uma vez que h é injetiva. Logo h [L] é compacto.
Para concluir só falta demonstrar a armação feita acima. Temos:
[ [
h−1 [L] ⊂ Aµ ⇒ L ⊂ h[Aλ ],
λ∈L λ∈L

porque: L = h[h−1 [L]] pelo item b) do Exercício 15 do Capítulo I (página 12), uma vez que
h é sobrejetiva; X ⊂ Y "⇒ h[X] # ⊂ h[Y ] (qualquer que seja a função h e os conjuntos X e
[ [
Y no domínio de h) e h Aλ = h[Aλ ] (qualquer que seja a função h, como se verica
λ∈L λ∈L
facilmente). Além do que cada h[Aλ ] é aberto, conforme arma o Exercício 8 (página 162).
Segue o resultado desejado.

Observação 2. Note que a solução acima, a da Observação 1, pode não funcionar para g −1 [L].
Isto por causa do já referenciado Exercício 8 (página 162), o qual nos ensina que g[A] não é
necessariamente aberto se A o for.

5.48 Exercício 48
As seguintes armações a respeito de um conjunto X⊂R são equivalentes:

(1) X é limitado;

(2) Todo subconjunto innito de X possui ponto de acumulação (que pode não pertencer a
X );

(3) Toda sequência de pontos de X possui uma subsequência convergente.

Solução. Vamos mostrar que (1) ⇔ (3) e (2) ⇔ (3).


190 CAPÍTULO 5. TOPOLOGIA DA RETA

(a) [(1) ⇒ (3)] Se X é limitado, então qualquer sequência de pontos de X também é


limitada, e qualquer sequência limitada de números reais possui uma subsequência convergente
( Apêndice ao Ÿ4 [do Capítulo IV], página 125 do livro).

⇒ (1)] Vamos mostrar a contrapositiva. Se X não é limitado, então, para todo


(b) [(3)
número natural n podemos encontrar um xn cujo módulo é maior do que n. Assim obtemos
uma sequência (xn ) de pontos em X tal que lim |xn | = +∞. Pelo Exercício 22 do Capítulo IV
(página 138) isto signica que nenhuma subsequência de (xn ) é convergente.

(c) [(2) ⇒ (3)] Seja (xn )n∈N uma sequência de pontos de X. SejaY = {xn ∈ X; n ∈ N}.
Temos dois casos a considerar; se Y é innito, então poderemos aplicar nossa hipótese, mas Y
pode também ser nito.
Se Y Y = {a1 , . . . , am }. Para cada i ∈ {1, . . . , m}, seja Mi = {n ∈ N; xn = ai }.
é nito, seja
Deste modo temos N = M1 ∪ · · · ∪ Mm , portanto algum Mi é innito. Para um tal i, a
subsequência (xn )n∈Mi converge para ai .
0
Se Y é innito, então, por hipótese, seja a ∈ Y e temos uma situação pela qual já passamos
na solução do Exercício 12 (página 164):

a ∈ Y 0 ⇒ ∀ ε ∈ R ∩(0, +∞), (a − ε, a + ε) ∩ Y é innito


⇒ ∀ ε ∈ R ∩(0, +∞), {n ∈ N; xn ∈ (a − ε, a + ε)} é innito
⇒ existe subsequência de (xn )n∈N convergindo para a,

como queríamos, sendo a primeira implicação válida pelo Teorema 7 e a última, pelo Teorema
9 do Capítulo IV.

(d) [[3] ⇒ (2)] Seja A um subconjunto innito de X. De A conseguimos extrair uma


sequência de pontos (xn )n∈N dois a dois distintos (obtidos x1 , . . . , xn , existe xn+1 ∈ A −
{x1 , . . . , xn }). Por hipótese, esta sequência admite uma subsequência que converge para um
ponto a. Esta subsequência é de elementos dois a dois distintos pertencentes a A, portanto
a ∈ A0 , como queríamos. 

5.49 Exercício 49
Seja X⊂R um conjunto compacto cujos pontos, com exceção de a = inf X e b = sup X , são
pontos de acumulação à direita e à esquerda. Então X = [a, b] ou X = {a, b}.

Solução.

Nível 1. Como X é fechado e pelas denições de ínmo e de supremo, temos

{a, b} ⊂ X ⊂ [a, b].

Há dois casos a considerar: ou [a, b] − X 6= ∅ ou [a, b] − X = ∅. Este segundo caso signica


X = [a, b]. Armamos que, se existe x ∈ [a, b] − X , então X = {a, b} (Nível 2).
5.50. EXERCÍCIO 50 191

No elevador. x ∈ [a, b] − X e y ∈ X distinto de ambos a e


Se, ao mesmo tempo, existem
b, então ou a<y<x ou a < y < x, com y ∈ X , então o supremo dos elementos
x < y < b. Se
de X menores do que x, além de pertencer a X , é estritamente maior do que a. Só que um
tal supremo não é um ponto de acumulação à direita de X . Se x < y < b, então acontece algo
análogo: o ínmo dos elementos de X maiores do que x pertence a X , é estritamente menor
do que b e não é um ponto de acumulação à esquerda de X .

Nível 2. [a, b] − X 6= ∅, seja x ∈ [a, b] − X . Por x não pertencer a X , de imediato


Se
podemos armar duas coisas: x ∈ (a, b) (porque a e b pertencem a X ) e existe ε > 0 tal que
(x − ε, x + ε) ∩ X = ∅ (porque X é fechado). Sejam

A = {y ∈ X; y < x} e B = {y ∈ X; y > x}.

AmbosA e B são limitados, por serem subconjuntos de um conjunto limitado, e não vazios,
porque a ∈ A e b ∈ B . Podemos considerar então seus respectivos supremo e ínmo. Para eles,
como X é fechado, temos sup A ∈ X e inf B ∈ X . Além disto, por causa do ε, temos também
sup A < x < inf B . Pela denição dos nossos conjuntos A e B (além de, claro, das denições
de supremo e de ínmo), o sup A não é um ponto de acumulação à direita de X , assim como
o inf B não é um ponto de acumulação à esquerda de X . Portanto só pode ser sup A = a e
inf B = b. Disto já podemos concluir a igualdade X = {a, b}, porque, se não fosse, conforme já
observamos no elevador, teríamos sup A > a ou inf B < b. 

(Ver também a página 268 do livro.)

5.50 Exercício 50
\
Se (Kλ )λ∈L é uma família qualquer de compactos, então Kλ é compacto. Se K1 , . . . , K n são
λ∈L
compactos então K1 ∪ · · · ∪ Kn é compacto. Se K é compacto e F é fechado, então K∩F é
compacto.

(Voltar para o Nível 1 da Solução do Exercício 61: página 200.)

Solução. Esta interseção é limitada e é fechada: é fechada porque uma interseção qualquer
de fechados é fechada; é limitada porque ela está contida em algum Kλ (está em qualquer um
deles) que, por sua vez, é um conjunto limitado (este argumento já foi visto na demonstração
do Teorema 12).

C = (Aµ )µ∈M uma cobertura por meio de abertos para K1 ∪ · · · ∪ Kn . Em particular C


Seja
cobre cada um dos Ki 's. Então, para cada i ∈ {1, . . . , n}, existe um subconjunto nito Fi ⊂ M
tal que (Aλ )λ∈Fi é uma subcobertura para Ki . O conjunto F = F1 ∪ · · · ∪ Fn é um subconjunto
nito de M e é tal que (Aλ )λ∈F é uma cobertura para K1 ∪ · · · ∪ Kn . Logo, qualquer reunião
nita de compactos é compacta, como queríamos.

Seja (xn )n∈N K ∩ F . Por K ser compacto, e porque


uma sequência qualquer de pontos de
xn ∈ K para todo n, existe M ⊂ N innito e um x ∈ K tal que (xn )n∈M converge para x
(Teorema 11). Este x também pertence a F porque F é fechado. Logo, K ∩ F é compacto pelo
Teorema 11 novamente. 
192 CAPÍTULO 5. TOPOLOGIA DA RETA

5.51 Exercício 51
Seja X ⊂ R. Uma função f : X → R diz-se não-decrescente no ponto a ∈ X quando existe
δ > 0 tal que a − δ < x 6 a 6 y < a + δ ⇒ f (x) 6 f (a) 6 f (y). (Bem entendido:
x, y ∈ X .) Mostre que se f é não-decrescente em todos os pontos de um intervalo [a, b] então
f é não-decrescente em [a, b] (isto é, x, y ∈ [a, b], x 6 y ⇒ f (x) 6 f (y)).

Solução. Vamos demonstrar a contrapositiva deste resultado. Isto é, negando a tese, sejam
x, y ∈ [a, b] tais que x < y e f (x) > f (y) (a negação da tese é x 6 y e f (x) > f (y), mas,
claramente, por f ser função, não podemos ter x = y e 6 f (y)) e
f (x) =
vamos encontrar um
ponto em [a, b] tal que f não é não decrescente neste ponto.
Seja A = {z ∈ [a, b]; z > x e f (z) < f (x)}. Por ser limitado e não vazio (y ∈ A por
hipótese), seja y0 seu ínmo. Se y0 = x, por ser ínmo, dado δ > 0, existe z ∈ [a, b], x < z < x+δ
tal que f (z) < f (x). Se y0 > x, dado δ > 0, se x < y1 < y0 , então f (y1 ) > f (x) (y1 não pertence
a A porque ele é menor do que seu ínmo) e existe y2 em A tal que y0 6 y2 < y0 + δ , ou seja,
tal que f (y2 ) < f (x). Então f (y1 ) > f (y2 ), com y0 − δ < y1 < y0 6 y2 < y0 + δ . Logo, de
qualquer forma, f não é não decrescente no ponto y0 . 

5.52 Exercício 52
[
Seja [a, b] ⊂ Aλ onde cada Aλ é aberto. Mostre que é possível decompor [a, b] em um número
λ
nito de intervalos justapostos de modo que cada um deles esteja contido em algum Aλ .

Solução 1. Veja como este exercício se assemelha ao Teorema 10. Basicamente uma cópia
da demonstração dele também resolve este caso.
Seja X o conjunto de todo ponto x ∈ [a, b] tal que o intervalo [a, x] pode ser decomposto
num número nito de intervalos justapostos de modo que cada um deles esteja contido em
algum Aλ . Claramente X é não vazio (pelo menos o elemento a ∈ X , de modo trivial) e é
limitado superiormente pelo b. Seja então c seu supremo. Como c ∈ [a, b] (pois a ∈ X e
b é uma cota superior para X ), seja λ0 um elemento do conjunto de índices considerado tal
que c ∈ Aλ0 e seja (α, β) c ∈ (α, β) ⊂ Aλ0 (intervalo este que
um intervalo aberto tal que
existe porque Aλ0 é aberto). Do fato de c ser o supremo de X e de α ser menor do que c,
existe x ∈ X tal que α < x 6 c. Assim escrevamos [a, x] = I1 ∪ · · · ∪ In , onde, para cada
i ∈ {1, . . . , n − 1}, o extremo à direita do intervalo Ii coincide com o extremo à esquerda do
intervalo Ii+1 e cada Ii está contido em algum Aλ . Além do que o extremo à esquerda do I1 é a
e o extremo à direita do In é x. O intervalo [x, c] está contido em Aλ0 . Portanto a decomposição
[a, c] = I1 ∪ · · · ∪ In ∪ [x, c] mostra que c tem todas as propriedades para pertencer a X . Se c
fosse menor do que b, poderíamos tomar um elemento y maior do que c e menor do que ambos
os elementos b e β . Um tal y também pertenceria a X pelo mesmo motivo que c pertence, o
que é um absurdo. Logo c = b ∈ X e o resultado está demonstrado. 

Observação. Esta solução não lança muita luz sobre o que devemos fazer para resolver o
exercício seguinte, o que nos força a procurar uma outra solução.

Solução 2. Também podemos resolver este exercício usando, não a demonstração do Teorema
10, mas a sua extensão. De acordo com ela, escrevamos [a, b] ⊂ I1 ∪ · · · ∪ In , onde cada Ii é um
5.53. EXERCÍCIO 53 193

intervalo aberto contido em algum Aλ . Temos o a, o b e, entre eles, extremos dos intervalos Ii .
Se não houver entre a e b nenhum desses extremos é porque o [a, b] já está inteiramente contido
em algum Aλ . Caso contrário, do mesmo modo como feito na demonstração da Proposição
1, armamos que podemos fazer o seguinte. Considerar todos os extremos dos intervalos Ii 's
que estão entre a e b e colocá-los em ordem crescente; dois pontos extremos consecutivos
(no sentido dessa ordem) irão determinar um intervalo contido em algum Aλ . De modo mais
preciso, sejam a < α1 < α2 < . . . < αm < b todos os extremos dos intervalos Ii 's que estão entre
a e b, então cada um dos intervalos [a, α1 ), [α1 , α2 ), · · · , [αm−1 , αm ), [αm , b), {b} ({b} também
é um intervalo, o intervalo degenerado da página 71 do livro) está contido em algum dos Aλ .
Além de claramente serem justapostos.
Por comodidade, sejam a = α0 e b = αm+1 . Dado i ∈ {0, 1, 2, . . . , m}, seja j ∈ {1, 2, . . . , n}
tal que αi ∈ Ij . Seja Ij = (aj , bj ). Deste modo, αi+1 6 bj , donde [αi , αi+1 ) ⊂ Ij , isto é, [αi , αi+1 )
está contido em algum dos Aλ , como queríamos. 

5.53 Exercício 53
No exercício anterior, mostre que os intervalos nos quais se decompôs [a, b] podem ser tomados
com o mesmo comprimento.

Solução.

Nível 1. Consideremos a decomposição da Solução 2 do exercício anterior. Como cada


Aλ é aberto, para cada i seja δi > 0 tal que (αi − δi , αi+1 ) esteja contido num mesmo Aλ . Como
acontecia no Teorema 4 do Capítulo III, fazemos o seguinte. Consideramos todos os δi 's (onde
cada δi é o comprimento do espaço entre αi e αi − δi ) e denimos δ = min{δ1 , δ2 , . . . , δm+1 }.
b−a
Seja n um número natural tal que < δ (Propriedade Arquimediana). Armamos que,
n
b−a
para cada i ∈ {1, . . . , m + 1}, existe um natural ki tal que a + ki ∈ (αi − δi , αi ) (Nível 2.1).
 n 
b−a b−a
Deste modo, para cada k ∈ {0, 1, . . . , n − 1}, o intervalo a + k · , a + (k + 1) · está
n n
contido em algum aberto Aλ (Nível 2.2).

b−a
Nível 2.1. Dado i ∈ {1, . . . , m + 1}, existe k tal que a+k · > αi (pelo menos o k=n
n
tem essa propriedade). Tomamos então o menor k com essa propriedade, isto é, consideramos:
 
b−a
k0 = min k ∈ N; a + k · > αi .
n
O ki que possui a propriedade desejada é ki = k0 − 1. De fato, pela minimalidade do k0 , temos:
b−a
a + (k0 − 1) · < αi
n
e
b−a b−a
a + k0 · > αi com < δ 6 δi
n n
implicam
b−a b−a b−a
a + k0 · − = a + (k0 − 1) · > αi − δi .
n n n
194 CAPÍTULO 5. TOPOLOGIA DA RETA

Logo
b−a
a + (k0 − 1) · ∈ (αi − δi , αi ),
n
como queríamos.

Nível 2.2. Dado k ∈ {0, 1, . . . , n − 1}, temos

b−a
a+k· ∈ [a, b) = [α0 , α1 ) ∪ [α1 , α2 ) ∪ · · · ∪ [αm , αm+1 = b).
n
Portanto existe i ∈ {0, 1, . . . , m} tal que

b−a
a+k· ∈ [αi , αi+1 ).
n
b−a b−a
Se a+k· 6 αi+1 − δi+1 , então, por causa da existência do ki+1 , temos a + (k + 1) · <
n n
αi+1 . Deste modo
 
b−a b−a
a+k· , a + (k + 1) · ⊂ (αi − δi , αi+1 )
n n
b−a
está contido num mesmo Aλ por construção. Se αi+1 − δi+1 < a + k · < αi+1 e se for
n
b−a
possível considerar o ki+2 , acontece algo análogo: a+(k +1)· < αi+2 (claramente podemos
n
supor αi+2 − δi+2 > αi+1 para garantir) e, então:
 
b−a b−a
a+k· , a + (k + 1) · ⊂ (αi+1 − δi+1 , αi+2 ).
n n
b−a
Se não for possível considerar o i+2 é porque i = m. Neste caso a + (k + 1) · 6b=
n
b−a
a+n· e
n  
b−a b−a
a+k· , a + (k + 1) · ⊂ (b − δm+1 , b],
n n
onde este último também está contido em algum Aλ . De qualquer forma temos o resultado
desejado. 

5.54 Exercício 54. Teorema de Baire


(Teorema de Baire). Se F1 , F2 , F3 , . . . , Fn , . . . são fechados com interior vazio então S = F1 ∪
· · · ∪ Fn ∪ . . . tem interior vazio. (É preciso mostrar que, dado arbitrariamente um intervalo
aberto I , existe algum x ∈ I ∩ (R −S). Imite a demonstração do Teorema 6, Capítulo III, onde
se tem pontos em vez dos fechados Fn .)

(Voltar para a Solução do Exercício 42 do Capítulo IX: página 402.)

(Voltar para a Solução do Exercício 43 do Capítulo IX: página 403.)

(Voltar para a Solução do Exercício 13 do Capítulo X, para o elevador 1: página 418.)

(Voltar para a Solução do Exercício 13 do Capítulo X, para seu Nível 1: página 418.)
5.55. EXERCÍCIO 55. R−Q NÃO É Fσ E Q NÃO É Gδ 195

Solução. Dado o intervalo aberto não vazio I , como o interior de F1 é vazio, o I não está
contido em F1 , portanto existe x1 ∈ I − F1 . Como F1 é fechado, existe ε1 > 0 tal que
(x1 −ε1 , x1 +ε1 )∩F1 = ∅. Diminuindo este ε1 se necessário, podemos supor (x1 −ε1 , x1 +ε2 ) ⊂ I
e [x1 − ε1 , x1 + ε1 ] ∩ F1 = ∅.
Repetimos este processo, considerando agora I1 := (x1 − ε1 , x1 + ε1 ) e F2 : tomamos um
x2 ∈ I1 − F2 (o qual existe porque I1 6⊂ F2 já que o interior de F2 é vazio) e um ε2 > 0 tal
que (x2 − ε2 , x2 + ε2 ) ∩ F2 = ∅ (o qual existe porque x2 ∈ / F2 e F2 é fechado). Como I1 é
aberto e x2 ∈ I1 , podemos diminuir este ε2 de modo a ter I2 := (x2 − ε2 , x2 + ε2 ) ⊂ I1 e
[x2 − ε2 , x2 + ε2 ] ∩ F2 = ∅. Além disto, I2 ⊂ I1 , pois I2 ⊂ I1 .
Vamos supor construídos os intervalos abertos e não vazios I1 , I2 , . . . , In tais que: I1 ⊃ I2 ⊃
· · · ⊃ In e Ii ∩ Fi = ∅ para cada i ∈ {1, . . . , n}. Fazemos com In e com o Fn+1 exatamente
o que já zemos nos dois parágrafos acima: tomamos xn+1 ∈ In − Fn+1 e εn+1 > 0 tal que
(xn+1 − εn+1 , xn+1 + εn+1 ) ⊂ In e [xn+1 − εn+1 , xn+1 + εn+1 ] ∩ Fn+1 = ∅, e denimos In+1 =
(xn+1 − εn+1 , xn+1 + εn+1 ). Este é um intervalo tal que In+1 ⊂ In e In+1 ∩ Fn+1 = ∅.
Temos assim uma sequência decrescente de intervalos limitados e fechados (e não vazios).
\
Portanto, pelo Teorema 5 do Capítulo III, seja x∈ In . Como In ∩ Fn = ∅, de x ∈ In , segue

[ n∈N
x∈
/ Fn , portanto x∈
/ Fn . Além disto, queremos também ter x ∈ I. Com o que temos até
n∈N
aqui, podemos concluir apenas que x ∈ I1 . Mas podemos diminuir um pouquinho mais aquele
ε1 do primeiro parágrafo de modo a termos também [x1 − ε1 , x1 + ε1 ] ⊂ I . Assim x é uma
testemunha para o fato de que o intervalo I dado não está contido em S . Como o intervalo I
é qualquer, o interior de S é vazio, como queríamos. 

5.55 Exercício 55. R − Q não é Fσ e Q não é Gδ


O conjunto R−Q dos números irracionais não pode ser expresso como reunião enumerável de
fechados. Analogamente, Q não é interseção de uma família enumerável de abertos.

(Voltar para a Solução do Exercício 18 do Capítulo VII: página 244.)

(Voltar para a Solução do Exercício 19 do Capítulo IX: página 374.)

(Voltar para a Solução do Exercício 13 do Capítulo X, para o elevador 1: página 418.)

Solução.

Parte 1.

Nível 1. Vamos supor que seja possível escrever

[
R−Q = Fn ,
n∈N

onde cada Fn ⊂ R é um conjunto fechado. Vamos derivar daí algum problema.

No elevador. Acabamos de mexer com o Teorema de Baire, o qual se pronuncia sobre


uma reunião enumerável de conjuntos fechados. Porém, num primeiro momento, ele se mostra
infrutífero aqui: o interior dessa reunião é vazio porque o interior de R−Q é também vazio,
196 CAPÍTULO 5. TOPOLOGIA DA RETA

portanto o interior de cada Fn é vazio, mas isso não ajuda em nada e sequer é o Teorema de
Baire. Ele seria útil se tivéssemos uma reunião de fechados cujo interior seja não vazio, daí
poderíamos aplicar sua forma contrapositiva para concluir que o interior de algum deles é não
vazio. Talvez se passássemos para seu fecho:

[
R = R−Q = Fn
n∈N

e se o fecho de uma reunião fosse a reunião dos fechos... só que isso não rola. Mas existe um
outro modo de recairmos em R: acrescentando um número racional a cada um dos Fn 's, só um,
e isso já faria um tremendo de um estrago!

Nível 2. Seja {xn ∈ R; n ∈ N} uma enumeração para Q. Para cada n ∈ N, vamos denir
o conjunto Gn = Fn ∪ {xn }. Um tal conjunto é fechado, porque é uma reunião de dois fechados,
e tem seu interior vazio (cada Fn tem seu interior vazio, porque se o interior de algum Fn fosse
não vazio, então o interior da reunião de todos os Fn 's também seria não vazio, mas não é o
caso, porque o interior dos irracionais é vazio). Deste modo temos o R, cujo interior é não
vazio, escrito como uma reunião enumerável de fechados:

[
R= Gn .
n∈N

Portanto, pelo Teorema de Baire (pela sua contrapositiva), existe um Gn cujo interior é não
vazio, o que é uma contradição. Logo não é possível escrever os irracionais como uma reunião
enumerável de fechados.

Parte 2. Seja (An )n∈N uma família de abertos. Se Q fosse igual a interseção dessa família,
então R−Q seria a reunião de seus complementares, os quais, por sua vez, são conjuntos
fechados: \ \ [
Q= An ⇒ R − Q = R − An = R −An .
n∈N n∈N n∈N

Mas não é possível escrever os irracionais desta forma conforme já vimos acima. Logo o conjunto
dos números racionais não pode ser escrito como uma interseção de conjuntos abertos. 

Curiosidades. Um conjunto que pode ser expresso como uma reunião enumerável de fechados
é chamado de Fσ . Um conjunto que pode ser expresso como uma interseção enumerável de
abertos é chamado de Gδ . O Fσ vem do Francês: F para fermé, que signica fechado, e σ
para somme, que signica reunião; e o Gδ vem do Alemão: G para Gebiet, que signica
conjunto aberto, e δ para durchschnitt, que signica interseção.
O exercício acima nos diz então que R−Q não é um Fσ e que Q não é um Gδ .

5.56 Exercício 56
n
[ n
X ∞
[
Se [a, b] ⊂ [ai , bi ], então b−a 6 (bi − ai ). Também [a, b] ⊂ [an , bn ] implica b−a 6
i=1 i=1 n=1

X
(bn − an ). Finalmente, resultados análogos valem para (a, b) em vez de [a, b].
n=1
5.56. EXERCÍCIO 56 197

Solução. Quase que uma cópia da demonstração da Proposição 1 também mostra que, se
n
[ Xn
[a, b] ⊂ [ai , bi ], então b−a 6 (bi − ai ), com a diferença de que agora temos c1 6 a e
i=1 i=1
b 6 ck . Esta é a principal diferença, a outra é a de que cada [cj , cj+1 ] está contido em algum
[ai , bi ]. Para imitar aquela demonstração, além de supor que a interseção de cada [ai , bi ] com
[a, b] é não vazia, também irá nos ajudar a suposição adicional de que [ai , bi ] é não degenerado,
ou seja, que os intervalos degenerados (caso existam) podem ser descartados dessa reunião.
Algum intervalo degenerado não poder ser descartado, digamos [an , bn ] = {an }, signica termos
an ∈ [a, b] e an ∈ / [ai , bi ] para todo i ∈ {1, 2, . . . , n − 1}. Vejamos então que, se an ∈ [a, b], então
an ∈ [ai , bi ] para algum i entre 1 e n − 1. Como a reunião destes n − 1 intervalos é fechada,
basta mostrar que an pertence ao seu fecho. Dado qualquer ε > 0, existe y distinto de an
na interseção (an − ε, an + ε) ∩ [a, b]. Como an = bn , y pertence então a [ai , bi ] para algum
i ∈ {1, . . . , n − 1}, o que signica que an pertence ao fecho de sua reunião, como queríamos.
A implicação

[ ∞
X
[a, b] ⊂ [an , bn ] ⇒ b − a 6 (bn − an )
n=1 n=1
ocorre como consequência da Proposição 3, junto com o fato de ser não enumerável a dife-
rença entre um conjunto não enumerável com um conjunto enumerável. De acordo com esta
X∞
proposição, se b−a é estritamente maior do que (bn − an ), então a diferença
n=1

[
[a, b] − (an , bn )
n=1

é não enumerável. Deste modo, é também não enumerável o conjunto

∞ ∞
!
[ [
[a, b] − [an , bn ] = [a, b] − (an , bn ) − {a1 , b1 , . . . , an , bn , . . .},
n=1 n=1

uma vez que {a1 , b1 , . . . , an , bn , . . .} é apenas enumerável. Portanto, em particular, existe x∈


[∞
[a, b] − [an , bn ], x este que testemunha a não inclusão
n=1

[
[a, b] 6⊂ [an , bn ].
n=1

Temos então demonstrada a contrapositiva da implicação desejada. Exatamente com a


mesma demonstração podemos concluir a implicação


[ ∞
X
(a, b) ⊂ [an , bn ] ⇒ b − a 6 (bn − an ),
n=1 n=1

uma vez que o x acima pode ser escolhido de modo a ser distinto de a e de b (pois há innitos
elementos ali).
n
X
Para o caso faltante, suponhamos b−a > (bi − ai ) e vamos encontrar um elemento
i=1
n
[
no intervalo (a, b) que não pertença à reunião [ai , bi ], o que irá mostrar a contrapositiva do
i=1
198 CAPÍTULO 5. TOPOLOGIA DA RETA

n
[
resultado desejado. Conforme já vimos, existe x ∈ [a, b] − [ai , bi ] (pela contrapositiva do
i=1
resultado já demonstrado). Se x ∈ (a, b), então acabamos. Se x = a ou se x = b podemos
proceder como no primeiro parágrafo acima, ao lidar com os intervalos degenerados (na verdade
podemos fazer isso mesmo que x seja distinto de a e de b). Como x não pertence ao conjunto
[n n
[
fechado [ai , bi ], existe ε > 0 tal que (x − ε, x + ε) ∩ [ai , bi ] = ∅. Neste intervalo (x − ε, x + ε)
i=1 i=1
existem innitos elementos pertencentes ao intervalo [a, b]. Em particular podemos tomar um
que seja diferente de a e de b. É um tal elemento que nos permite concluir que (a, b) não está
n
[
incluso na reunião [ai , bi ] como queríamos. 
i=1

5.57 Exercício 57
Seja X ⊂ R. Uma função f : X → R chama-se localmente limitada quando para cada x ∈ X
existe um intervalo aberto Ix , contendo x, tal que f |Ix ∩ X é limitada. Mostre que se X é
compacto, toda função f : X → R localmente limitada é limitada.

Solução. A família (Ix )x∈X nos dá uma cobertura por meio de abertos para X . Como X é
compacto, sejam x1 , . . . , xn ∈ X tais que X ⊂ Ix1 ∪ · · · ∪ Ixn . Se Mi é um número real positivo
tal que |f (x)| < Mi para todo x ∈ Ixi ∩ X , seja M = max Mi . Armamos que |f (x)| < M
16i6n
para todo x ∈ X.
De fato: se x ∈ X , então x ∈ Ixi ∩ X para algum i ∈ {1, . . . , n}, de modo que |f (x)| <
Mi 6 M . 
(Voltar para a Observação 1 do item j) do Exercício 47 do Capítulo VII: página 281.)

5.58 Exercício 58
Dado X ⊂R não-compacto, dena uma função f :X →R que seja localmente limitada mas
não seja limitada.

(Voltar para o Exercício 27 do Capítulo VII: página 252.)

Solução. Fixados a ∈ X − X e uma sequência (xn )n∈N ∈ F(N; X) convergindo para a,


1
denimos f (xn ) = para todo n ∈ N e f (x) = x para todo x ∈ X − {xn ; n ∈ N} (se
xn − a
X − X é vazio, então f se resume à f (x) = x para todo x ∈ X ). Dado x ∈ X , a restrição
|x − a|
f |(x − δx , x + δx ) ∩ X é limitada para δx = ou, se X − X = ∅, para δx = 1. E f não é
2
limitada (lembrando do Teorema 11: se X não é compacto, então X não é fechado ou X não é
limitado). 

5.59 Exercício 59
Sejam C compacto, A aberto e C ⊂ A. Mostre que existe ε>0 tal que x ∈ C , |y − x| < ε ⇒
y ∈ A.
5.60. EXERCÍCIO 60 199

Solução.

Nível 1. No Nível 2, vamos encontrar um ε > 0 tal que (x − ε, x + ε) ⊂ A qualquer que


seja o x ∈ C. O resultado então segue porque |y − x| < ε ⇔ y ∈ (x − ε, x + ε).

No elevador. C ⊂ A, podemos obter um número nito de intervalos contidos em A


Como
que cobrem C , simplesmente considerando os intervalos que testemunham que cada ponto de C é
interior a A. Daí podemos tomar como ε o raio do menor deles. Porém, em seguida, podemos
ter problemas para demonstrar o resultado desejado para este ε, o que nos leva a procurar
outra solução. A ideia sobre qual é o caminho bom a se seguir vem após uma simplicação do
problema, investigando o caso em que A é um intervalo (a, b). Neste caso podemos considerar
os pontos de C mais próximos de seus extremos, a saber, seu ínmo e seu supremo, e tomar
como ε o menor dos números a − inf C e b − sup C . Este ε resolve. Esta ideia é facilmente
estendida para o caso geral, porque de A conseguimos extrair um número nito de intervalos
com propriedades boas para usá-la.

Nível 2. Pelo Teorema 2 (o da estrutura dos abertos da reta), podemos escrever A como
[
uma reunião A= An de intervalos abertos e dois a dois disjuntos. Como C é compacto, e
n∈N
C ⊂ A, existe um número nito deles que cobrem C . Podemos supor que eles são A1 , A2 , . . . , An .
Sejam Ai = (ai , bi ) para cada i ∈ {1, . . . , n}. Se x ∈ C , então x ∈ (ai , bi ) para algum desses
i's. Vamos denir xi = inf{x ∈ C; x > ai } e yi = sup{x ∈ C; x < bi } (claro que podemos
descartar aqueles Ai 's que não têm interseção com C ). Por um lado, como C é fechado, ambos
xi e yi pertencem a C . Por outro, como os Ai 's são dois a dois disjuntos, nenhum ai e nenhum
bi pertencem a A e, em particular, eles também não pertencem a C . Portanto xi > ai e yi < bi .
Sejam δi = xi − ai e εi = bi − yi . Armamos que ε = min{δ1 , . . . , δn , ε1 , . . . , εn } é o ε procurado.
De fato, se x ∈ C , então x ∈ (ai , bi ) para algum i, o que implica xi 6 x 6 yi . Como ε 6 δi
e ε 6 εi , temos: xi − δi = ai 6 x − ε e x + ε 6 yi + εi = bi , logo (x − ε, x + ε) ⊂ (ai , bi ) ⊂ A,
como queríamos. 

5.60 Exercício 60

\
Dada uma sequência (xn ) seja Xn = {xn , xn+1 , . . .} para todo n ∈ N. Mostre que Xn é o
n=1
conjunto dos valores de aderência de (xn ).

Solução. a é um valor de aderência da sequência (xn )n∈N , xado n0 , vamos encontrar uma
Se
sequência de pontos de Xn0 que converge para a. Por hipótese, existe M ⊂ N innito tal que
(xn )n∈M converge para a. Para N = M ∩ [n0 , +∞), a sequência (xn )n∈N é de pontos em Xn0
que converge para a, como queríamos.
\∞
Dado a ∈ Xn , para concluirmos que a é um valor de aderência da sequência (xn )n∈N , dado
n=1
ε > 0, basta vermos que existem innitos números naturais n tais que xn pertence ao intervalo
(a − ε, a + ε) (Teorema 9 do Capítulo IV). E é isto que de fato ocorre, pois (a − ε, a + ε) ∩ Xn 6= ∅
qualquer que seja o n, por termos a ∈ Xn . De modo mais preciso, dado n0 , existe um elemento
y ∈ (a−ε, a+ε)∩Xn0 . Este y = xn para algum n > n0 , pela denição de Xn0 . Logo, o conjunto
200 CAPÍTULO 5. TOPOLOGIA DA RETA

{n ∈ N; xn ∈ (a − ε, a + ε)} é não limitado e, portanto, innito (Teorema 5 do Capítulo II),


como queríamos. 

5.61 Exercício 61
Uma família de conjuntos (Kλ )λ∈L chama-se uma cadeia quando, para quaisquer λ, µ ∈ L tem-
se Kλ ⊂ Kµ Kµ\⊂ Kλ .
ou Prove que se (Kλ )λ∈L é uma cadeia de compactos não-vazios então

a interseção K= Kλ é não-vazia (e compacta).


λ∈L

Solução.

Nível 1. Que é compacto isto já foi visto no Exercício 50 (página 191), pelo mesmo
argumento do início da demonstração do Teorema 12.
L é enumerável, desta cadeia conseguimos obter uma sequência descendente de compactos
Se
não vazios cuja interseção também é K (Nível 2.1). Portanto, pelo Teorema 12, K é não vazio.
Se L não é enumerável, conseguimos cair no caso anterior, extraindo de L um subconjunto
enumerável N tal que
\ \
Kn ⊂ Kλ
n∈N λ∈L

(Nível 2.2; neste nível veremos que vai valer algo mais forte e mais geral, a saber, a igualdade
para conjuntos fechados). Portanto, pelo que acabamos de armar (como o resultado é válido
para um conjunto de índices enumerável), K contém um subconjunto não vazio, logo, é não
vazio.

No elevador. Em vez de tomar uma interseção enumerável de uma só vez como um


todo, podemos tomá-la passo a a passo, considerando suas interseções nitas, acrescentando
um conjunto da família à interseção de cada vez. Isto nos dá uma sequência descendente de
conjuntos, cuja interseção não muda com relação à interseção original. A hipótese de L ser uma
cadeia nos ajudará a mostrar que cada uma dessas interseções nitas é não vazia.

Nível 2.1. SejaN = {λ1 , . . . , λn , . . .} um subconjunto


\enumerável de L. Seguindo o plano
já anunciado no Nível 1, vamos mostrar que a interseção Kλn é não vazia.
n∈N
Para cada n ∈ N, denamos X n = K λ1 ∩ · · · ∩ K λn . Pelo fato de (Kλ )λ∈L ser uma cadeia de
conjuntos não vazios, cada Xn é não vazio (Nível 3), de modo que X1 ⊃ X2 ⊃ Xn ⊃ · · · é uma
sequência descendente de conjuntos campactos não vazios. Portanto, pelo Teorema 12,

\
Xn 6= ∅.
n∈N

O resultado segue porque

\ \
Xn = K λn .
n∈N n∈N
5.61. EXERCÍCIO 61 201

No elevador. Temos resolvido o caso em que L é enumerável. O problema agora aparece


quando L é não enumerável. Por um lado sabemos que de L podemos extrair um conjunto
enumerável N (Teorema 7 do Capítulo II), mas, por outro, não é qualquer N que nos serve,
porque, se N ⊂ L, então, de modo geral, temos
\ \
Kλ ⊂ Kn ,
λ∈L n∈N

quando, na verdade, o que ajudaria seria a inclusão inversa

\ \
Kn ⊂ Kλ
n∈N λ∈L

Para termos uma tal inclusão, seria bom se para todo λ∈L existisse n∈N tal que Kn ⊂ Kλ ,
porém um tal caminho parece deixar as coisas meio embaraçadas...
Até por conta da reunião de complementares ser igual ao complementar das interseções, esse
desejo de querer extrair uma espécie de subcobertura às avessas de uma interseção qualquer
lembra o Teorema de Lindelöf. E é esse caminho que se mostra ser um bom caminho.

Nível 2.2: Corolário do Teorema de Lindelöf. Seja (Fλ )λ∈L uma coleção qualquer de
fechados. Existe um conjunto enumerável N ⊂L tal que

\ \
Fn = Fλ .
n∈N λ∈L

Solução. Seja
\
F = Fλ
λ∈L

Como cada Fλ é fechado, cada R −Fλ é aberto. Assim

\ [
R −F = R − Fλ = R −Fλ
λ∈L λ∈L

é uma reunião de uma família de abertos. Pelo Teorema de Lindelöf (página 185), de L podemos
extrair um conjunto enumerável N tal que (R −Fλ )λ∈N cobre R −F . Mas a reunião desta família
também está contida em R −F (porque N ⊂ L). Portanto
[ [
R −Fλ = R −Fλ ,
λ∈N λ∈L

ou seja,
\ \
R− Fλ = R − Fλ ,
λ∈N λ∈L

donde vem
\ \
Fλ = Fλ ,
λ∈N λ∈L

como queríamos.
202 CAPÍTULO 5. TOPOLOGIA DA RETA

Nível 3: Lema. Se C = (Aλ )λ∈L é\uma cadeia de conjuntos não vazios e se F é um

subconjunto nito não vazio de L, então Aλ é não vazio.


λ∈F
Solução. Por indução na cardinalidade de
\ F , vamos mostrar que existe λ0 ∈ F tal que

Aλ0 ⊂ Aλ para todo λ ∈ F, donde seguirá Aλ = Aλ0 6= ∅.


λ∈F
n = 1 não há nada a fazer. Se F tem n + 1 elementos, seja µ ∈ F , de modo que
Para
G = F − {µ} tem n elementos. Pela hipótese de indução, existe λ0 ∈ G tal que Aλ0 ⊂ Aλ para
todo λ ∈ G. Como C é uma cadeia, temos Aµ ⊂ Aλ0 ou Aλ0 ⊂ Aµ . No primeiro caso temos
Aµ ⊂ Aλ para todo λ ∈ F . No segundo, Aλ0 ⊂ Aλ para todo λ ∈ F . Segue a tese. 

5.62 Exercício 62
Se X⊂R é não-enumerável, então X0 também o é.

Solução. Como no Corolário 2 do Teorema 8, seja E um subconjunto enumerável denso em


X (Teorema 6), isto é, E ⊂ X e X ⊂ E = E ∪ E 0 , onde E ⊂ X ⇒ E 0 ⊂ X 0 (conforme já vimos,
0
por exemplo, na Observação 1 do Exercício 33, página 179). Assim, se X é enumerável, então
0
E é também enumerável, portanto, E , sendo uma reunião de dois enumeráveis, é enumerável.
Logo X, sendo subconjunto de E, é enumerável. Isto mostra a contrapositiva deste exercício.

5.63 Exercício 63
Para todo X ⊂ R, X − X 0 é enumerável.

Solução 1. De X = X ∪ X 0, temos

X − X 0 = X − X 0,

o conjunto dos pontos isolados de X.


Exatamente do mesmo modo como zemos na Solução
0
do Exercício 37 (página 181), para cada x ∈ X − X , podemos encontrar um intervalo aberto
Ix possuindo x tal que Ix ∩ Iy = ∅ se x e y são elementos distintos de X − X 0 . Portanto, pelo
Exercício 9 (página 163), a coleção de todos os Ix é enumerável. A tese segue do fato de esta
0
coleção e o conjunto X − X terem a mesma cardinalidade, uma vez que a associação x 7→ Ix
é bijetiva. 

Solução 2. Para cada


[ x ∈ Y := X − X 0 , seja Ix um intervalo aberto tal queIx ∩ X = {x}, de
modo que Y ⊂ Ix . Pelo Teorema de Lindelöf (página 185), existe um subconjunto N ⊂ Y
x∈Y [
enumerável tal que Y ⊂ Ix . Portanto:
x∈N

[
Y = (Ix ∩ Y )
x∈N

é enumerável porque é uma reunião enumerável de conjuntos nitos. 


5.64. EXERCÍCIO 64. TEOREMA DE BENDIXSON 203

5.64 Exercício 64. Teorema de Bendixson


Um número real a chama-se ponto de condensação de um conjunto X ⊂ R quando todo intervalo
aberto de centro a contém uma innidade não-enumerável de pontos de X . Seja F0 o conjunto
dos pontos de condensação de um fechado F ⊂ R. Prove que F0 é um conjunto perfeito (isto é,
fechado, sem pontos isolados) e que F −F0 é enumerável. Conclua daí o Teorema de Bendixson :
todo fechado da reta é reunião de um conjunto perfeito com um conjunto enumerável.

Solução.

Primeira Parte.

(a) é fechado. A demonstração de que F0 é fechado é a mesma do Exercício 32 (página


F0
179): sejax ∈ F0 ; se I é um intervalo aberto com centro em x, então existe y ∈ I ∩ F0 (porque x
é aderente à F0 por hipótese); agora I é também um intervalo aberto ao qual y ∈ F0 pertence,
o qual contém um intervalo aberto J com centro em y , intervalo este que, pela denição de F0 ,
tem uma interseção não enumerável com F , portanto, como J ⊂ I , a interseção de I com F é
também não enumerável, logo x ∈ F0 .

(b) F0 não tem pontos isolados. Para mostrar que nenhum ponto de F0 é isolado,
dados x ∈ F0 e um intervalo aberto I possuindo x, queremos mostrar que em I existe y ∈ F0
distinto de x. Vamos trabalhar com a contrapositiva disto, isto é, vamos demonstrar o seguinte.
Dados um intervalo aberto I e um ponto x ∈ I , se y ∈/ F0 para todo y ∈ I distinto de x, então
x∈/ F0 . Vamos fazer isto mostrando que I ∩ F é enumerável (daí, tomamos um intervalo aberto
A centrado em x e contido em I , de modo que A ∩ F ⊂ I ∩ F , portanto A ∩ F é também
enumerável).
Para cada y∈I distinto de x seja Iy um intervalo aberto centrado em y tal que a interseção
de Iy com F é enumerável. Temos assim uma cobertura por meio de abertos para J := I − {x}:
[
I − {x} ⊂ Iy .
y∈J

Pelo Teorema de Lindelöf (página 185), existe um subconjunto enumerável E de J tal que

[
I − {x} ⊂ Iy .
y∈E

Portanto a interseção
!
[ [
Iy ∩F = (Iy ∩ F )
y∈E y∈E

é enumerável. Como I − {x} ∩ F é um subconjunto do conjunto acima, ele também é
enumerável. Acrescentar um ponto a ele não o faz deixar de ser enumerável. Portanto I ∩F é
enumerável, uma vez que
  
I ∩F ⊂ I − {x} ∩ F ∪ {x},

ou seja, I ∩F é um subconjunto de um conjunto enumerável. Logo x∈


/ F0 , como queríamos.
204 CAPÍTULO 5. TOPOLOGIA DA RETA

(c) F − F0 é enumerável. Para cada x ∈ F − F0 xamos um intervalo Ix aberto ao qual


x pertence tal que a interseção Ix ∩ F é enumerável. Denindo L = F − F0 , de

[
F − F0 ⊂ Ix ,
x∈L

pelo Teorema de Lindelöf (página 185), obtemos um subconjunto enumerável E ⊂ F − F0 tal


que
[
F − F0 ⊂ Ix .
x∈E

Desta inclusão, e de F − F0 ⊂ F , segue:

!
[ [
F − F0 ⊂ F ∩ Ix = (F ∩ Ix ).
x∈E x∈E

Este último uma reunião enumerável de enumeráveis. Subconjuntos de enumeráveis são enu-
meráveis. Segue a tese.

Segunda Parte. Então escrevemos

F = (F ∩ F0 ) ∪ (F − F0 ) = F0 ∪ (F − F0 ),

uma reunião de um conjunto perfeito, e um enumerável, F − F0 . A igualdade F ∩ F0 = F0


F0 ,
0 0
vale porque F0 é claramente um subconjunto de F e F ⊂ F , já que F é fechado (Corolário 1
do Teorema 8), logo F fechado implica F0 ⊂ F . 

Curiosidades. Este teorema também é conhecido como o Teorema de Cantor-Bendixson.


0
Um conjunto sem pontos isolados, isto é, tal que X ⊂ X também é chamado de conjunto
denso em si mesmo.

Observação 1. A demonstração de que F − F0 é enumerável também nos dá uma terceira


solução para o Exercício 40, o qual dizia:

X ∩ X0 = ∅ ⇒ X é enumerável,

para X ⊂ R.
Se / X 0 . Seja então Ix um intervalo
x ∈ X , então x ∈ [ aberto possuindo x tal que a interseção
Ix ∩ X é nita. Pelo Teorema de Lindelöf, de X ⊂ Ix , obtemos um subconjunto enumerável
[ [ x∈X
N ⊂ X tal que X ⊂ Ix e, portanto, X = (Ix ∩X), uma reunião enumerável de conjuntos
x∈N x∈N
nitos.

Observação 2. Nos itens (a), (b) e (c) acima não foi usada a hipótese de que F é fechado.
Portanto, não importa quem seja o conjunto X de números reais, o conjunto X0 de seus pontos
de condensação é perfeito e X − X0 é enumerável.
Capítulo 6
Limites de Funções

6.1 Exercício 1
Na denição do limite lim f (x), retire a exigência de ser x 6= a. Mostre que esta nova denição
x→a
coincide com a anterior no caso a ∈/ X mas, para a ∈ X , o novo limite existe se, e somente se,
o antigo existe e é igual a f (a).

Solução. Se a∈
/ X, a nova denição coincide com a anterior porque, neste caso, temos

Vδ = X ∩ (a − δ, a + δ).
Suponhamos a ∈ X. Se o antigo limite existe e é igual a f (a), dado ε > 0, existe δ>0 tal que

x ∈ Vδ ⇒ |f (x) − f (a)| < ε .


A tese desta implicação também esta satisfeita para x = a, pois |f (a) − f (a)| = 0 < ε. Deste
modo, temos:
x ∈ X ∩ (a − δ, a + δ) ⇒ |f (x) − f (a)| < ε .
Portanto o novo limite também é igual a f (a).
Reciprocamente, se o novo limite existe, então ele só pode ser f (a), pois, para L 6= f (a),
tomando ε = |f (a) − L|, temos
f (a) ∈
/ (L − ε, L + ε),
enquanto a ∈ X ∩ (a − δ, a + δ), qualquer que seja o δ > 0. Isto signica que o novo limite não
é igual a nenhum L distinto de f (a), portanto ele só pode ser igual a f (a). Assim, dado ε > 0,
existe δ > 0 tal que
x ∈ X ∩ (a − δ, a + δ) ⇒ |f (x) − f (a)| < ε .
Em particular:
x ∈ Vδ ⇒ |f (x) − f (a)| < ε .
Logo o limite antigo existe e é igual a f (a). 

6.2 Exercício 2
Considere o seguinte erro tipográco na denição de limite:

∀ ε > 0 ∃δ > 0; x ∈ X, 0 < |x − a| < ε ⇒ |f (x) − L| < δ.


Mostre que f cumpre esta condição se, e somente se, é limitada em qualquer intervalo limitado
de centro a. No caso armativo, L pode ser qualquer número real.

205
206 CAPÍTULO 6. LIMITES DE FUNÇÕES

Solução. Lembrando que ||f (x)| − |L|| 6 |f (x) − L| (item (iii) do Teorema 2 do Capítulo
III), temos

|f (x) − L| < δ ⇒ −δ < |f (x)| − |L| < δ ⇒ |L| − δ < |f (x)| < |L| + δ.

Portanto f é limitada em qualquer vizinhança do a, se f cumpre a condição dada.


Reciprocamente, supomos que, dado ε > 0, existe tal que |f (x)| < A se x ∈ Vε . Dado
A
L ∈ R, queremos δ > 0 tal que |f (x) − L| < δ para todo x ∈ Vε . Isto é, temos −A < f (x) < A
e queremos L − δ < f (x) < L + δ . Um δ > 0 que faça simultaneamente L + δ > A e L − δ < −A
serve, ou seja δ > A − L e δ > A + L. Portanto, basta tomar δ > max{0, A − L, A + L}. Para
este δ temos o resultado desejado:

x ∈ Vε ⇒ −A < f (x) < A ⇒ L − δ < f (x) < L + δ ⇒ |f (x) − L| < δ,

pois L − δ < L − (A + L) = −A e L + δ > L + (A − L) = A. 

6.3 Exercício 3
Seja X = Y ∪ Z , com a ∈ Y 0 ∩ Z 0 . Dada f : X → R, tomemos g = f |Y e h = f |Z . Se
lim g(x) = L e lim h(x) = L então lim f (x) = L.
x→a x→a x→a

Solução. Dado ε > 0, queremos δ>0 tal que

x ∈ VX (a; δ) ⇒ |f (x) − L| < ε .

Por hipótese, existem δ1 > 0 e δ2 > 0 tais que

x ∈ VY (a; δ1 ) ⇒ |f (x) − L| < ε

e
x ∈ VZ (a; δ2 ) ⇒ |f (x) − L| < ε .
A tese agora segue da inclusão

VX (a; δ) ⊂ VY (a; δ1 ) ∪ VZ (a; δ2 ),

para δ = min{δ1 , δ2 }. 

6.4 Exercício 4
1
Seja f : R −{0} → R denida por f (x) = . Então lim f (x) = 0 e lim f (x) = 1.
1 + e1/x x→0+ x→0−

Solução. Dado ε > 0, primeiro queremos δ>0 tal que

1
x ∈ (0, δ) ⇒ <ε
1 + e1/x
e, depois, δ>0 tal que
1
x ∈ (−δ, 0) ⇒
− 1 < ε.
1 + e1/x
6.4. EXERCÍCIO 4 207

Estes deltas nascem de trás para frente:

1 1/x 1
< ε ⇒ 1 + e >
1 + e1/x ε
1/x 1
⇒ e > −1
ε 
1 1
⇒ > ln −1
x ε
1
⇒ x< 1

ln ε − 1

1
(na terceira implicação aplicamos o ln, supondo − 1 > 0. Na quarta implicação, supomos
  ε
1
ln − 1 > 0, ou seja, estamos supondo ε pequeno o suciente de modo a termos 1/ε−1 > 1;
ε
este ε é ε < 1/2), e;


1 1 1

1 + e1/x − 1 =1−
1/x
<ε ⇒ >1−ε
1+e 1 + e1/x
1
⇒ 1 + e1/x <
1−ε
1
⇒ e1/x < −1
1− ε 
1 1
⇒ < ln −1
x 1−ε
1
⇒ x> 1
.
ln 1−ε − 1

1
Na segunda implicação: 1−ε positivo. Na quarta implicação, veja que − 1 > 0. Na
1−ε
penúltima linha das implicações acima, veja que os números que aparecem dos dois lados da
desigualdades são negativos se ε < 1/2.
Dado então ε > 0, vamos supó-lo menor do que meio. Todas estas implicações que acabamos
de ver podem ser feitas de baixo para cima, o que nos dá:

1 1
0<x< 1
 ⇒ <ε
ln ε − 1 1 + e1/x
e
1 1
1
 <x<0⇒
− 1 <ε
ln 1−ε
−1 1 + e1/x

Logo, dado 0 < ε < 1/2 o primeiro delta procurado é

1
δ= 1

ln ε
−1

e o segundo é

1
δ=− 1
. 
ln 1−ε
−1
208 CAPÍTULO 6. LIMITES DE FUNÇÕES

6.5 Exercício 5
Seja f (x) = x + 10 · sen x para todo x ∈ R. Então lim f (x) = +∞ e lim f (x) = −∞. Prove
x→+∞ x→−∞
x
o mesmo para a função g(x) = x + sen x.
2

Solução. Para todo x real, uma vez que −1 6 sen x 6 1, temos

x − 10 6 x + 10 sen x 6 x + 10.

As armações para f agora seguem de lim x − 10 = +∞ e lim x + 10 = −∞.


x→+∞ x→−∞
Para g é análogo. Se x > 0:
x x 3x
6 x + sen x 6
2 2 2
e, se x < 0:
3x x x
6 x + sen x 6 .
2 2 2
Portanto lim g(x) = +∞ porque lim x/2 = +∞, e lim g(x) = −∞ porque lim x/2 =
x→+∞ x→+∞ x→−∞ x→−∞
−∞. 

6.6 Exercício 6
Seja f : X → R monótona, com f (X) ⊂ [a, b]. Se f (X) é denso no intervalo [a, b] então, para
cada c ∈ X+0 ∩ X−0 , tem-se lim− f (x) = lim+ f (x). Se c ∈ X então este limite é igual a f (c).
x→c x→c

(Voltar para a Solução do Exercício 16 do Capítulo VII: página 240.)

Solução.

Primeira Parte. Este exercício nos remete ao Teorema 12. Por tal teorema, ambos os
limites laterais existem. Se f é não decrescente, por sua demonstração, temos

L := lim+ f (x) = inf{f (x); x ∈ X, x > c},


x→c

e
M := lim− f (x) = sup{f (x); x ∈ X, x < c}.
x→c

Claramente, L e M pertencem a [a, b]. Se L e M são distintos, então L > M , (pois, para
x1 , x2 ∈ X , x1 < c < x2 ⇒ f (x1 ) 6 f (x2 ) ⇒ M 6 L), portanto a interseção do intervalo aberto
(M, L) com f [X] é vazia, isto é, f [X] não é denso no intervalo [a, b]. Logo, só pode ser L = M .
Se f é não crescente, a demonstração é inteiramente análoga, onde, desta vez

L := lim+ f (x) = sup{f (x); x ∈ X, x > c},


x→c

M := lim− f (x) = inf{f (x); x ∈ X, x < c}


x→c

e (L, M ) ∩ f [X] = ∅ se L 6= M .
(Voltar para o item (c) da Solução do Exercício 9: página 212.)
6.7. EXERCÍCIO 7 209

Segunda Parte. Vamos continuar com os mesmos L e M da Primeira Parte. Se c ∈


X+0 ∩ X−0 , então, pelo que acabamos de ver, lim− f (x) = M = L = lim+ f (x). Portanto, pelo
x→c x→c
Teorema 11, lim f (x) = L = M .
x→c
Se f é não decrescente, então:
M 6 f (c) 6 L,
uma vez que f (c) é uma cota superior para o conjunto {f (x); x ∈ X, x < c} e é uma cota
inferior para o conjunto {f (x); x ∈ X, x > c}. Como M = L, então M = f (c) = L, como
queríamos.
Se f é não crescente, então L 6 f (c) 6 M , o que também implica L = f (c) = M . Segue a
tese. 

6.7 Exercício 7
Demonstre o Teorema 2.

Solução. Este teorema diz o seguinte

Sejam X ⊂ R, f : X → R, a ∈ X 0 . Dado Y ⊂ X tal que a ∈ Y 0 , ponhamos


g = f |Y . Se lim f (x) = L, então lim g(x) = L. Se Y = I ∩ X onde I é um intervalo
x→a x→a
aberto contendo a, então lim g(x) = L ⇒ lim f (x) = L.
x→a x→a

Primeira Parte. Se Y ⊂ X, então

VY (a; δ) ⊂ VX (a; δ)

qualquer que seja o δ > 0. Por hipótese, dado ε > 0, existe δ>0 tal que

x ∈ VX (a; δ) ⇒ |f (x) − L| < ε .

Em particular, pela inclusão acima:

x ∈ VY (a; δ) ⇒ |f (x) − L| < ε,

onde f (x) = g(x) para todo x∈Y por hipótese, isto é

x ∈ VY (a; δ) ⇒ |g(x) − L| < ε .

Logo, lim g(x) = L.


x→a

Segunda Parte. Se Y = I ∩X onde I é um intervalo aberto possuindo a, seja δ0 > 0 tal


que
(a − δ0 , a + δ0 ) ⊂ I.
Ele é tal que, se 0 < δ 6 δ0 , então

VX (a; δ) = VY (a; δ),

pois VX (a; δ) ⊂ (a − δ, a + δ) ⊂ (a − δ0 , a + δ0 ) ⊂ I .
210 CAPÍTULO 6. LIMITES DE FUNÇÕES

Dado ε > 0, por hipótese, seja δ1 > 0 tal que

x ∈ VY (a; δ1 ) ⇒ |f (x) − L| < ε .

Tomando δ = min{δ0 , δ1 }, temos VX (a; δ) = VY (a; δ) ⊂ VY (a; δ1 ). Portanto,

x ∈ VX (a; δ) ⇒ x ∈ VY (a; δ1 ) ⇒ |f (x) − L| < ε .

Segue lim f (x) = L, como queríamos. 


x→a

6.8 Exercício 8
Sejam f : X → R monótona e a ∈ X+0 . Se existir uma sequência de pontos xn ∈ X com
xn > a, lim xn = a e lim f (xn ) = L, então lim+ f (x) = L.
x→a

(Voltar para a Solução do Exercício 11, item (a): página 214.)

(Voltar para a Solução do Exercício 48 do Capítulo X, Nível 3.7: página 452.)

Solução 1. Dado ε > 0, queremos δ>0 tal que

x ∈ (a, a + δ) ∩ X ⇒ f (x) ∈ (L − ε, L + ε).

Por causa da convergência lim f (xn ) = L, existe nε ∈ N tal que f (xn ) ∈ (L − ε, L + ε) para
todo n > nε . Por hipótese, xnε > a. Armamos que

x ∈ (a, xnε ) ∩ X ⇒ f (x) ∈ (L − ε, L + ε).

δ > 0 procurado é δ = xnε − a.


Deste modo, o
x ∈ (a, xnε ) ∩ X , por causa da convergência lim xn = a, seja m ∈ N tal que xm < x
Dado
e m > nε (este m existe porque conjunto de todos os n's tais que xn < x não é limitado), de
modo que ambos f (xm ) e f (xnε ) pertencem ao intervalo (L−ε, L+ε). Agora, de xm < x < xnε ,
temos, por f ser monótona,
f (xm ) 6 f (x) 6 f (xnε )
ou
f (xm ) > f (x) > f (xnε ).
De qualquer forma, segue f (x) ∈ (L − ε, L + ε), como queríamos. 

Solução 2. A existência de uma tal sequência implica a limitação de f no conjunto X∩


(a, a + δ) para algum δ>0 (a justicativa disto está no último parágrafo). Assim, pelo item
10 da página 210, existe o limite à direita de f (x) quando x tende para a. Seja

A = lim+ f (x)
x→a

esse limite. Queremos mostrar que A = L. Pelo Teorema 10, para a função g:Y →R denida
por g(x) = f (x) para todo x ∈ Y := X ∩ (a, +∞), temos

lim g(x) = A.
x→a
6.9. EXERCÍCIO 9 211

Então, para a sequência dada, pelo Teorema 6, temos

A = lim g(xn ) = lim f (xn ) = L.


n→+∞ n→+∞

Logo A = L, como queríamos.


A justicativa da existência do δ do primeiro parágrafo é semelhante a um raciocínio já
visto na Solução 1. Como a sequência dos f (xn )'s converge, sabemos que existe M >0 tal que
|f (xn )| < M para todo n ∈ N (Teorema 3 do Capítulo IV). Fixado xn0 (xn0 > a por hipótese),
se x ∈ (a, xn0 )∩X , então existe n ∈ N tal que xn < x < xn0 (por causa da convergência xn → a).
Portanto, pela monotonicidade de f , temos f (xn ) 6 f (x) 6 f (xn0 ) ou f (xn ) > f (x) > f (xn0 ).
De qualquer forma segue |f (x)| < M , ou seja, f é limitada no conjunto X ∩ (a, a + δ) para
δ = xn0 − a. 
(Voltar para a Primeira Parte da Solução do Exercício 19: página 222.)

Observação. O resultado análogo vale para a ∈ X−0 . Isto é, dado a ∈ X−0 , se existir uma
sequência de pontos xn ∈ X com xn < a, lim xn = a e lim f (xn ) = L, então lim− f (x) = L.
x→a

6.9 Exercício 9
Se f : X → R é monótona então o conjunto dos pontos a ∈ X0 para os quais não se tem
lim− f (x) = lim+ f (x) é enumerável.
x→a x→a

Solução. Para ser lícito usar os símbolos lim f (x) e lim+ f (x), a deve ser um ponto de
x→a− x→a
acumulação à direita e à esquerda de X. Porém, e até para termos uma certa margem de
segurança, vamos incluir também o caso em que não vale a referida igualdade simplesmente
porque não faz sentido falar em um dos limites laterais. Isto é, denindo

A = {a ∈ X+0 ∩ X−0 ; lim+ f (x) 6= lim− f (x)},


x→a x→a

Y+ = X+0 − X−0 e Y− = X−0 − X+0 ,


vamos mostrar que cada um dos conjuntos A, Y+ e Y− é enumerável. Usando ideias já vistas
anteriormente, isto vai ser consequência do Exercício 9 do Capítulo V (página 163), o qual dizia
ser enumerável qualquer coleção de abertos dois a dois disjuntos.

(a) Y+ é enumerável. Cada ponto deY+ é de acumulação à direita de X e não é de


acumulação à esquerda de X . Assim, dado x ∈ Y+ , seja o intervalo Ix = (x − ε(x), x), com
ε(x) > 0, tal que Ix ∩ X = ∅. Armamos que Ix ∩ Iy = ∅, se x e y são pontos distintos de Y+ .
Veja que y ∈/ Ix , porque y ∈ Ix implicaria Ix ∩ X 6= ∅, uma vez que y ∈ X+0 e y < x implicam
a existência de um z ∈ X no intervalo (y, x). Portanto, se y < x, então y < x − ε(x) e se
y > x, então x < y − ε(y). De qualquer modo Ix ∩ Iy = ∅. Logo, a coleção dos intervalos da
forma Ix para x ∈ Y+ é enumerável, coleção esta que tem a mesma cardinalidade de Y+ , como
queríamos.

(b) Y− é enumerável. Esta demonstração é inteiramente análoga a do parágrafo anterior.


212 CAPÍTULO 6. LIMITES DE FUNÇÕES

(c) A é enumerável. Para xar as ideias, vamos supor f não decrescente (para f não
crescente a demonstração é análoga). Conforme já vimos na Primeira Parte da Solução do
Exercício 6 (página 208), para cada a ∈ A, podemos associar o intervalo aberto

 
Ia := lim f (x), lim+ f (x) ,
x→a− x→a

onde (pela demonstração do Teorema 12)

lim f (x) = sup{f (x); x ∈ X e x < a}


x→a−

lim f (x) = inf{f (x); x ∈ X e x > a}.


x→a+

Agora Ia ∩ Ib = ∅, se a e b são pontos distintos de A, porque inf{f (x); x ∈ X e x >


a} 6 sup{f (x); x ∈ X e x < b}, se b > a (pois, tomando c ∈ X tal que a < c < b, temos
inf{f (x); x ∈ X e x > a} 6 f (c) 6 sup{f (x); x ∈ X e x < b}), e inf{f (x); x ∈ X e x > b} 6
sup{f (x); x ∈ X e x < a}, se b < a (como consequência do caso a < b, simplesmente trocando
as letras, a por b e b por a no caso anterior). Logo A é enumerável pelo mesmo motivo já visto
no item (a) acima, como queríamos. 

6.10 Exercício 10
Enuncie e demonstre para funções o análogo do Teorema 14 do Capítulo IV.

Solução.

Primeira Parte: enunciados. Para os itens abaixo, sejam f, g : X → R funções reais


de uma variável real, cujo domínio X é um conjunto ilimitado superiormente.

1. Se lim f (x) = +∞ e g é uma função limitada inferiormente (isto é, existe c ∈ R tal que
x→+∞
g(x) > c para todo x ∈ X ), então lim [f (x) + g(x)] = +∞;
x→+∞

2. Se lim f (x) = +∞ e existe c>0 tal que g(x) > c para todo x ∈ X, então, lim f (x) ·
x→+∞ x→+∞
g(x) = +∞;

1
3. Suponha f (x) > 0 para todo x ∈ X. Então lim f (x) = 0 ⇔ lim = +∞;
x→+∞ x→+∞ f (x)

4. Suponha f (x) > 0 e g(x) > 0 para todo x ∈ X. Então:

(a) se existe c > 0 tal que f (x) > c para todo x ∈ X e se lim g(x) = 0 tem-se
x→+∞
f (x)
lim = +∞;
x→+∞ g(x)

f (x)
(b) se f é limitada e lim g(x) = +∞ então lim = 0.
x→+∞ x→+∞ g(x)
6.10. EXERCÍCIO 10 213

Segunda Parte: demonstrações.

1. Dado M ∈ R, queremos encontrar A>0 tal que

x ∈ X, x > A ⇒ f (x) + g(x) > M.

Pela hipóteselim f (x) = +∞, sabemos que existe um momento a partir do qual f (x)
x→+∞
é sempre maior do que um certo número dado. Assim olhamos para a desigualdade
f (x) > M − g(x), número este que é menor do que M − c para todo x ∈ X . Portanto, o
A > 0 procurado é aquele tal que f (x) > M − c para todo x ∈ X maior do que A. De
fato:
x ∈ X, x > A ⇒ f (x) > M − c > M − g(x) ⇒ f (x) + g(x) > M,
como queríamos.

2. De modo análogo ao que zemos no item anterior, dado M ∈ R, seja A > 0 tal que
M
f (x) > para todo x∈X maior do que A, o qual existe porque lim f (x) = +∞.
c x→+∞
M M
Supondo M > 0, notando que g(x) também é positivo para todo x ∈ X , temos >
c g(x)
para todo x ∈ X, de modo que

M
x ∈ X, x > A ⇒ f (x) > ⇒ f (x)g(x) > M,
g(x)

logo lim f (x)g(x) = +∞.


x→+∞

1
3. Dado M > 0, supondo lim f (x) = 0, seja A>0 tal que f (x) < para todo x ∈ X
x→+∞ M
maior do que A (aqui |f (x)| = f (x) para todo x ∈ X, pois f (x) > 0 para todo x ∈ X ),
portanto
1
x ∈ X, x > A ⇒ > M,
f (x)
1
logo lim = +∞.
x→+∞ f (x)

1 1 1
Reciprocamente, dado ε > 0, supondo lim = +∞, seja A > 0 tal que >
x→+∞ f (x) f (x) ε
para todo x∈X maior do que A. Para este A, como f (x) > 0 para todo x ∈ X , temos

x ∈ X, x > A ⇒ |f (x)| < ε .

Logo, lim f (x) = 0, como queríamos.


x→+∞

4. (a) Dado M, queremos encontrar A>0 tal que

f (x)
x ∈ X, x > A ⇒ > M.
g(x)

f (x) f (x) c
Quanto à desigualdade > M , temos > (pois g(x) > 0 para todo
g(x) g(x) g(x)
c
x ∈ X ), onde, pelo item 3., pode se tornar tão grande quanto desejarmos, pois
g(x)
214 CAPÍTULO 6. LIMITES DE FUNÇÕES

1
c>0 e lim = +∞. De modo mais preciso, tomamos A>0 tal que
x→+∞ g(x)
1 M
x ∈ X, x > A ⇒ > .
g(x) c
f (x)
Portanto > M , se x ∈ X e x > A, como queríamos.
g(x)
(b) Seja L ∈ R tal que |f (x)| < L para todo x ∈ X . Dado ε > 0, queremos A>0 tal
que
f (x)
x ∈ X, x > A ⇒ < ε.
g(x)
Agora tomamos A > 0 tal que

1
x ∈ X, x > A ⇒ < ε,
g(x) L
o qual existe porque lim g(x) = 0 (pelo item 3.) e L 6= 0 (pois 0 6 |f (x)| < L
x→+∞
1 ε L
para todo x ∈ X ). Da implicação acima, da implicação
g(x) < L ⇒ |g(x)| < ε


f (x) L
(pois L > 0) e da
g(x) < |g(x)| vem
desigualdade

f (x)
x ∈ X, x > A ⇒ < ε,
g(x)
como queríamos.

6.11 Exercício 11
p
Dado a > 1, dena f : Q → R pondo, para cada ∈ Q, f (p/q) = ap/q . Prove que lim f (x) = 1.
q x→0
Conclua que para cada b∈R existe lim f (x), sendo este limite igual a f (b) se b ∈ Q. Chame
x→b
b0 b+b0 b0
este limite de ab . Prove que ab · a = a e que b < b 0 ⇒ ab < a .

Solução.

lim f (x) = 1. Já conhecemos um resultado semelhante a este, a saber, que lim a1/n = 1
(a)
x→0
(Exemplo 13 do Capítulo IV). O que nos permite concluir o resultado desejado é este fato em
combinação com o Exercício 8 (página 210), bem como sua observação, com o Exercício 60 do
Capítulo III (página 114) e com o Teorema 11.
0 0
Isto porque f é crescente, 0 ∈ Q+ ∩ Q− , (1/n)n∈N e (−1/n)n∈N são sequências de números
racionais que convergem para 0 tais que 1/n > 0 e −1/n < 0 para todo n ∈ N. Portanto:

lim f (x) = lim f (1/n) = lim a1/n = 1,


x→0+
e
 1/n
1
lim− f (x) = lim f (−1/n) = lim =1
x→0 a
Logo lim f (x) = 1, como queríamos.
x→0
6.11. EXERCÍCIO 11 215

(b) O lim f (x) existe qualquer que seja o b ∈ R.


x→b

No elevador. Uma estratégia na qual o resultado lim f (x) = 1 pode ser aproveitado é o
x→0
Critério de Cauchy, porque se xey são números racionais próximos de b, então a diferença x−y
é um número racional próximo do zero e o limite que acabamos de nos referir diz justamente o
que acontece com f quando ela assume valores próximos do 0.

Nível 1. Seja b ∈ R. Fixado ε > 0, vamos encontrar δ>0 tal que

x, y ∈ VQ (b; δ) ⇒ |ax − ay | < ε .

Portanto, pelo Critério de Cauchy (Teorema 8), existe o lim f (x).


x→b

No elevador. Pelo item (a), sabemos que existe δ>0 tal que

x − y ∈ VQ (0; δ) ⇒ |ax−y − 1| < ε,

onde:
x
x−y
a 1
|a − 1| = y − 1 = y · |ax − ay | < ε ⇒ |ax − ay | < ay · ε .

a a
Quer dizer, lá na ponta aparece um ay · ε , ε sozinho. Se nesta ponta
quando queríamos o
aparecesse um fator menor do que ou igual a 1 multiplicado pelo ε, então este produto seria
menor do que ou igual a ε e teríamos o que queremos. Podemos fazer isto xando p ∈ Q de
modo que se y < p e se |a
x−y
− 1| < a−p · ε, então |ax − ay | < ay−p · ε, com y − p < 0 ⇒ ay−p < 1,
como desejado. Além disto, note que 0 < |x−b| < δ e 0 < |y −b| < δ implicam 0 < |x−y| < 2δ .
São destas considerações que nasce o delta do nível abaixo.

Nível 2. Começamos xando um número racional p estritamente maior do que b e de-


nindo δ1 = p − b (que é maior do que 0). Pelo que já vimos no item (a), existe δ2 > 0 tal
que

(∗) z ∈ VQ (0; δ2 ) ⇒ |az − 1| < a−p · ε .


 
δ2
Seja δ = min δ1 , . Armamos que:
2

x, y ∈ VQ (b; δ) ⇒ |ax − ay | < ε .

De fato, como 2δ 6 δ2 e por (∗), temos

x, y ∈ VQ (b; δ) ⇒ x − y ∈ VQ (0; 2δ) ⊂ VQ (0; δ2 ) ⇒ |ax−y − 1| < a−p ε ⇒ |ax − ay | < ay−p ε < ε,

pois ay−p < 1, uma vez que:

y ∈ VQ (b; δ) ⇒ y < b + δ 6 b + δ1 = b + (p − b) = p ⇒ y − p < 0 ⇒ ay−p < 1.


216 CAPÍTULO 6. LIMITES DE FUNÇÕES

lim f (x) = f (b) se b ∈ Q. Pelo item (b), já sabemos que lim f (x) existe. Agora, pelo
(c)
x→b x→b
Teorema 6,
podemos fazer as contas considerando uma sequência conveniente. Como b ∈ Q, a
1
sequência b+ é de números racionais, todos distintos de b, e converge para b. Além
n n∈N
disto:
 
1
f b+ = ab+1/n = ab · a1/n ,
n
ou seja, pelo Exemplo 13 do Capítulo IV (junto com o Teorema 6 do Capítulo IV), a sequência
b
dos f (b + 1/n) converge para a = f (b). Logo lim f (x) = f (b).
x→b

0 0
ab · ab = ab+b . Como zemos no item anterior, aqui também podemos trabalhar com
(d)
0 0
sequências. Sejam (xn ) e (xn ) sequências de números racionais que convergem para b e b
0 0 0 0
respectivamente, tais que xn 6= b, xn 6= b e xn + xn 6= b + b para todo n ∈ N (para cada n ∈ N,
0 0
há innitos racionais em cada um dos intervalos (b, b + 1/n) e (b , b + 1/n); escolhemos xn no
0 0 0 0
primeiro e xn no segundo tais que xn + xn 6= b + b ), de modo que a sequência (xn + xn )n∈N
0
converge para b + b . Como o resultado análogo ao deste item é válido para números racionais
(Exercício 60 do capítulo III, página 114), temos

0 0 0 0
lim f (xn + x0n ) = lim axn +xn = lim axn · axn = [lim axn ] · [lim axn ] = ab · ab ,

Logo, pelo Teorema 6:


0 0
ab+b := lim 0 f (x) = ab · ab ,
x→b+b

como queríamos.

0
(e) b < b0 ⇒ ab < ab .
Novamente pensando em sequências, podemos fazer o seguinte.
0 0
Tomar um número racional p entre b e b e considerar duas sequências (xn ) e (xn ), a primeira
decrescente, convergindo para b e tal que xn < p para todo n e a segunda, crescente, convergindo
0 0
para b e tal que xn > p para todo n. Então teremos:

0 0
ab = lim axn < ap < lim axn = ab ,
0
donde segue ab < ab , como queríamos. 

6.12 Exercício 12
Dado a > 1, dena g : R → R, pondo g(x) = ax (veja o exercício anterior). Prove lim g(x) =
x→+∞
+∞ e lim g(x) = 0.
x→−∞

Solução. Isto é consequência do Exercício 26 do Capítulo III (página 81) e do exercício


anterior (em particular do item (e) de sua solução).
n x
Dado b real positivo, existe n0 ∈ Z tal que a 0 > b. Para todo x > n0 , temos a > b. Dado
m0 x
ε > 0, existe m0 ∈ Z tal que a < ε. Para todo x 6 m0 , temos a < ε. Seguem as teses. 
6.13. EXERCÍCIO 13 217

6.13 Exercício 13
Seja p : R → R, um polinômio real. Se o coeciente do termo de grau mais elevado de p é
positivo então lim p(x) = +∞ e lim p(x) é igual a +∞ ou a −∞, conforme o grau de p
x→+∞ x→−∞
seja par ou ímpar.

(Voltar para a Solução do Exercício 24: página 230.)

(Voltar para a Solução do Exercício 23 do Capítulo VII: página 249.)

(Voltar para a Solução do Exercício 26 do Capítulo VII: página 252.)

(Voltar para a Solução do Exercício 35 do Capítulo VII: página 263.)

(Voltar para a Solução do Exercício 3 do Capítulo VIII: página 284.)

(Voltar para o Nível 1 da Solução 1 do Exercício 6 do Capítulo VIII: página 286.)

(Voltar para o Nível 3.1 da Solução 1 do Exercício 6 do Capítulo VIII: página 286.)

(Voltar para a Solução 2 do Exercício 6 do Capítulo VIII: página 287.)

Solução. p(x) = an xn + an−1 xn−1 + · · · + a1 x + a0 (n ∈ N) esse polinômio. Escrevemos


Seja

n
 an−1 a1 a0 
p(x) = an x · 1 + + · · · + n−1 + n .
x x x
n n n
Armamos que: (a) lim an x = +∞; (b) lim an x = +∞ se n é par; (c) lim an x =
x→+∞ x→−∞ x→−∞
a a
−∞ se n é ímpar, e; (d) lim m = lim m = 0 para a ∈ R e m ∈ N.
x→+∞ x x→−∞ x
O resultado desejado segue destes fatos pelo item 7 da página 210 (naturalmente, os análogos
dos resultados a que este item se refere valem para x → −∞ ou para p(x) tendendo a −∞).

(a) Dado M > 0, queremos A>0 tal que

x > A ⇒ an xn > M.
n
p
(an x > M ⇒ x >
n
r Olhando para a tese M/an ), armamos que o A procurado é A=
M
n
. De fato, como a associação x 7→ xn para todo x > 0 é crescente (Exercício 11 do Capítulo
an p
n
III, página 72) e M/an > 0, temos:
r
M M
x> n
⇒ xn > ⇒ an xn > M,
an an
como queríamos (o fato da referida associação ser crescente será usado nos itens abaixo tam-
bém).

(b) Dado M > 0, queremos A<0 tal que

x < A ⇒ an xn > M.
r
M
Este A é A=−n :
an
r r
M M M
x<− n
⇒ −x > n
⇒ (−x)n = xn > ⇒ an x n > M
an an an
q
M
(ambos −x e
n
an
da tese da primeira implicação (e hipótese da segunda) são números positi-

vos), como queríamos.


218 CAPÍTULO 6. LIMITES DE FUNÇÕES

(c) Dado M < 0, queremos A<0 tal que

x < A ⇒ an xn < M.
r
−M
O A que serve aqui é A=−n :
an
r r
−M −M −M
x<−n ⇒ −x > n
⇒ (−x)n = −xn > ⇒ an xn < M
an an an
q
−M
(ambos −x e
n
an
da tese da primeira implicação (e hipótese da segunda) são números posi-

tivos), como queríamos.

(d) Dado ε > 0, queremos A>0 tal que

a
|x| > A ⇒ 0 < m < ε,

x
supondo a 6= 0; se a = 0, então lim a/xm = lim a/xm = 0 trivialmente.
x→+∞ x→−∞
Pelos itens anteriores, podemos concluir que, dado M > 0, existe A>0 tal que

|x| > A ⇒ |xm | > M.


m
|a/x
Assim, como | < ε ⇔ |xm | > |a|/ε, seja A tal que |x| > A ⇒ |xm | > |a|/ε, portanto,
a

|x| > A ⇒ 0 < m < ε, como queríamos. 
x

Observação. Se o coeciente do termo de grau mais elevado de p é negativo, então lim p(x) =
x→+∞
−∞ e lim p(x) = −∞ ou lim p(x) = +∞, conforme o grau de p seja par ou ímpar respec-
x→−∞ x→−∞
tivamente.

(Voltar para a Segunda Parte da Solução do Exercício 26 do Capítulo VII: página 252.)

(Voltar para a Solução do Exercício 3 do Capítulo VIII: página 284.)

6.14 Exercício 14
sen(1/x)
Determine o conjunto dos valores de aderência da função f : R −{0} → R, f (x) = ,
1 + e1/x
no ponto x = 0.

Solução.

No elevador. (xn ) que converge para zero, dependendo de como


Dada uma sequência
essa convergência se dá, a sequência dos e1/xn pode convergir para 0 (se todos os xn 's forem
negativos), tender para o innito (se todos os xn 's forem positivos) ou não convergir (se houver
innitos xn 's positivos e innitos xn 's negativos). A sequência dos f (xn )'s vai convergir para
o mesmo número para o qual sen(1/xn ) converge, no primeiro caso; para zero no segundo caso
porque sen(1/xn ) é limitada, ou pode também não convergir. O conjunto V A(sen(1/x); 0) já é
conhecido. Dentro dessas considerações é que surge a armação abaixo.
6.15. EXERCÍCIO 15 219

Nível 1. Armamos que


V A(f ; 0) = [−1, 1].
Nos Níveis 2.1 e 2.2 vamos demonstrar as respectivas inclusões [−1, 1] ⊂ V A(f ; 0) e V A(f ; 0) ⊂
[−1, 1].

Nível 2.1. Como no Exemplo 6, dado c ∈ [−1, 1], tomamos b ∈ R tal que sen b = c, com a
−1
diferença de que agora denimos, para cada n ∈ N, xn = (b − 2πn) . Esta sequência converge
para zero e é tal que a sequência de seus inversos tende para −∞, porque existe n0 ∈ N tal
1/xn
que b − 2πn < 0 para todo n > n0 (pela Propriedade Arquimediana), de modo que e →0
e, então, lim f (xn ) = sen b = c. Portanto [−1, 1] ⊂ V A(f ; 0).

Nível 2.2. e1/x > 0 e −1 6 sen(1/x) 6 1 qualquer que seja x ∈ R −{0}, vamos
De
concluir que −1 6 f (x) 6 1 ∀x ∈ R −{0}. Deste modo, dada uma sequência convergente
(f (xn ))n∈N , seu limite pertence a [−1, 1] porque f (xn ) ∈ [−1, 1] para todo n ∈ N e [−1, 1] é
fechado.
Como e1/x > 0 ∀x ∈ R −{0}, temos

1
0< < 1 ∀x ∈ R −{0}.
1 + e1/x
Assim:
sen(1/x)
0 6 sen(1/x) 6 1 ⇒ 0 6 61
1 + e1/x
e

sen(1/x) sen(1/x)
−1 6 sen(1/x) 6 0 ⇒ 0 6 − sen(1/x) 6 1 ⇒ 0 6 − 6 1 ⇒ −1 6 6 0.
1 + e1/x 1 + e1/x
De qualquer forma, −1 6 f (x) 6 1 ∀x ∈ R −{0}, como queríamos. 

Observação. De modo geral, se a imagem de uma função f está contida num conjunto
fechado F, então V A(f ; a) ⊂ F qualquer que seja o ponto de acumulação a de seu domínio.

(Voltar para o Nível 1 da Solução do Exercício 16: página 220.)

6.15 Exercício 15
Se lim f (x) = L então lim |f (x)| = |L|. Se lim |f (x)| = |L| então o conjunto dos valores de
x→a x→a x→a
aderência de f no ponto a é {L}, {−L} ou {L, −L}.

Solução.

Primeira Parte. O que acontece aqui é o mesmo que acontecia no Exercício 1 do Capítulo
IV (página 117). Dado ε > 0, por hipótese, existe δ>0 tal que

x ∈ Vδ ⇒ |f (x) − L| < ε .
Como ||f (x)| − |L|| 6 |f (x) − L| (Teorema 2 do Capítulo III), segue

x ∈ Vδ ⇒ ||f (x)| − |L|| < ε .


Logo lim |f (x)| = |L|.
x→a
220 CAPÍTULO 6. LIMITES DE FUNÇÕES

Segunda Parte. Vamos supor L > 0. Seja uma sequência (xn ) tal que xn → a. Então,
dado ε > 0, por hipótese, existe n0 ∈ N tal que |f (xn )| ∈ (L − ε, L + ε) para todo n > n0 .
Portanto, f (xn ) ∈ (L − ε, L + ε) ou f (xn ) ∈ (−L − ε, −L + ε). Sejam M = {n ∈ N; f (xn ) ∈
(L−ε, L+ε)} e N = {n ∈ N; f (xn ) ∈ (−L−ε, −L+ε)}. Como M ∪N é um conjunto innito (o
conjunto {n0 , n0 + 1, n0 + 2, . . .} está contido nele), um dos dois, M ou N , é innito (e portanto
não limitado, pois ambos são conjuntos de números naturais). Se, para toda sequência (xn )
que converge para a, M é innito e N é nito, então V A(f ; a) = {L}. Se, para toda sequência
(xn ) que converge para a, N é innito e M é nito, então V A(f ; a) = {−L}. Se existe uma
sequência que converge para a tal que M é innito e se existe uma sequência que converge para
a tal que N é innito, então V A(f ; a) = {L, −L}.
Se L < 0, então o raciocínio é análogo (de fato o mesmo), agora com |f (xn )| ∈ (−L −
ε, −L + ε). Mas, de qualquer forma (L negativo ou não negativo), temos f (xn ) ∈ (L − ε, L + ε)
ou f (xn ) ∈ (−L − ε, −L + ε). 

6.16 Exercício 16
Dados um número real a e um conjunto compacto não-vazio K , obtenha uma função f : R → R
tal que o conjunto dos valores de aderência de f no ponto a seja K .

Solução.

No elevador. Um caso particular deste exercício já é conhecido: quando K = [−1, 1]


1
e a = 0, sabemos que a função denida por sen para todo x 6= a satisfaz as condições
x
dadas (faltando apenas deni-la na origem para que seu domínio seja R, mas isso não é um
grande problema). Esta função pode ser usada para resolver o caso geral, do seguinte modo.
O conjunto K está contido num intervalo da forma [−α, α] para algum α > 0 e a função dada
1
por α sen tem este intervalo como conjunto de valores de aderência no ponto a. Agora
x−a
o cuidado que devemos tomar para denir f a partir dessa função é que K pode estar contido
propriamente nesse intervalo.

Nível 1. Como K é compacto (em particular por ser limitado), seja α>0 tal que K⊂
[−α, α]. Seja g : R −{a} → R denida por
1
g(x) = α sen ∀x ∈ R −{a}.
x−a
Denimos agora f :R→R fazendo:

 g(x), se g(x) ∈ K
f (x) = inf K ∈ K, se g(x) ∈
/K
inf K, se x = a.

f está contida em K , de modo que, conforme já observamos no


Por construção, a imagem de
Exercício 14 (página 219), como K é fechado, V A(f ; a) ⊂ K . Para a inclusão K ⊂ V A(f ; a) o
argumento é semelhante ao do Exemplo 6. Seja b ∈ R um número real qualquer e consideremos
−1
a sequência (a + (b + 2πn) )n∈N , sequência esta que converge para a. Para ela temos:

1
g(a + (b + 2πn)−1 ) = α · sen = α · sen(b + 2πn) = α · sen b.
a + (b + 2πn)−1 − a
6.17. EXERCÍCIO 17 221

Deste modo, dado c ∈ K, se b∈R é tal que sen b = c/α, então

g(a + (b + 2πn)−1 ) = α · sen b = c ∈ K.

Portanto:

f (a + (b + 2πn)−1 ) = c ∀n ∈ N,

ou seja, c é um ponto de aderência de f no ponto a, como queríamos. 

6.17 Exercício 17
 
p p
Seja f : R → R denida por f (x) = x se x é irracional, f = q se é uma fração irredutível
q q
com p > 0, f (0) = 0. Mostre que f é ilimitada em qualquer intervalo não-degenerado.

Solução. Seja I = (a, b) a < b. Podemos supor a > 0 ou b < 0 (se for
um intervalo, com
a<0 e b > 0, 0 < ε < b, consideramos o intervalo (ε, b) e caímos em algum
então tomamos
m
caso anterior). Seja M > 0. Se a > 0, armamos que existem naturais m, n tais que ∈I
n
m
e n > M . Se b < 0, armamos que existem m ∈ N e n ∈ Z tais que ∈ I e n < −M . Em
n
ambos os casos, m e n primos entre si.
Conforme já vimos anteriormente (por exemplo, no Teorema 4 do Capítulo III, e em outros
1
exercícios), para qualquer número natural n tal que < b − a, existe um inteiro m tal que
n
m
∈ I. Porém pode acontecer de m e n não serem primos entre si. Então tomamos um número
n  
1 m
primo n maior do que max M, . Para este n, existe m ∈ Z tal que ∈ I . Agora, a
b−a n
m
única chance de não ser uma fração irredutível é se m for um múltiplo de n. Mas, se m for
n
m
um múltiplo de n, então é um número inteiro. Para garantir que isso não aconteça, podemos
n
supor que não existe um número inteiro no intervalo (a, b) (pois se existir um inteiro em (a, b),
então trabalhamos com o intervalo (k, b), para k = max Z ∩(a, b), no lugar de (a, b)).
m −m
Se a > 0, então m > 0. Se b < 0, então m < 0 e = , agora com −m > 0 e −n < −M .
n −n
Segue a tese. 

6.18 Exercício 18
Sejam X, Y ⊂ R, f : X → R, g : Y → R com f (X) ⊂ Y . Se, para a ∈ X 0 e b ∈ Y 0 tem-se
lim f (x) = b, lim g(y) = c e, além disso, f (x) 6= b para todo x ∈ X −{a}, então lim g(f (x)) = c.
x→a y→b x→a
Mostre que a condição b∈Y0 decorre de ser f (x) 6= b para x 6= a.
(Voltar para a Solução 2 do Exercício 34 do Capítulo IX: página 395.)

Solução.
222 CAPÍTULO 6. LIMITES DE FUNÇÕES

Primeira Parte. Seja ε > 0. Queremos encontrar δ>0 tal que

x ∈ VX (a; δ) ⇒ |g(f (x)) − c| < ε .

O esboço é o mesmo da demonstração do Teorema 9. Como lim g(y) = c, existe η>0 tal
y→b
que:
x ∈ VY (b; η) ⇒ |g(x) − c| < ε .
Para este η, como lim f (x) = b, existe δ>0 tal que
x→a

x ∈ VX (a; δ) ⇒ |f (x) − b| < η.

Como f (x) ∈ Y e f (x) 6= b para x 6= a, se x ∈ VX (a; δ), então f (x) ∈ VY (b; η). Deste modo,
pelas duas implicações acima, temos:

x ∈ VX (a; δ) ⇒ f (x) ∈ VY (b; η) ⇒ |g(f (x)) − c| < ε,

como queríamos.

Segunda Parte. A condição b ∈ Y0 decorre não apenas de ser f (x) 6= b para x 6= a,


mas também de lim f (x) = b. Pela denição de ponto de acumulação, dado ε > 0, queremos
x→a
encontrar y∈Y tal que 0 < |b − y| < ε. Por causa do limite de f (x) ser b quando x tende a a,
existe δ > 0 tal que
x ∈ VX (a; δ) ⇒ |f (x) − b| < ε .
Já sabemos que VX (a; δ) é não vazio por ser a ∈ X 0 . Então xamos x ∈ VX (a; δ); este x
pertence a X e é diferente de a. Portanto f (x) ∈ Y e f (x) 6= b, de modo que o y procurado é
y = f (x). 

6.19 Exercício 19
Para todo número real x, indiquemos com [x] o maior inteiro 6 x. Mostre que se a e b são
positivos então  
x b b b hxi
lim+ = e lim+ = 0.
x→0 a x a x→0 x a
Prove também que, no primeiro caso, o limite à esquerda seria o mesmo mas no segundo caso
o limite é +∞ quando x→0 por valores negativos.

Solução.

 
x b b hxi
Primeira Parte. Sejam f (x) = e g(x) = ∀x ∈ R −{0}. Calculando f em
a x x a
b a
e g em (que são números maiores do que zero), temos:
n n
   
b b/n b b b b
f = · = · [n] = ·n= ∀n ∈ N
n a b/n an an a
e    
a b a/n bn 1 bn
g = · = · = · 0 = 0 ∀n ∈ N −{1}.
n a/n a a n a
6.19. EXERCÍCIO 19 223

Só falta ver o que acontece com f (x) quando x está entre b/(n + 1) e b/n e com g(x) quando
x está entre a/(n + 1) e a/n. Temos:
 
b b 1 x 1 b b
<x6 ⇒ < 6 ⇒n6 <n+1⇒ = n.
n+1 n n+1 b n x x
Portanto:
 
b b b x b n b x b b
<x6 ⇒ < 6 ⇒ · < 6 ∀n ∈ N .
n+1 n (n + 1)a a na n+1 a a x a
n b b b b
Assim, se h é uma função tal que h(x) = · se < x 6 , então h(x) 6 f (x) 6
n+1 a n+1 n a
b
para todo x ∈ (0, b), além do que lim h(x) = . Daí, pelo Teorema 4 (junto com o Teorema
x→0+ a
b
10, do mesmo modo como feito na Solução 2 do Exercício 8, página 210), segue lim f (x) = ,
x→0+ a
como queríamos.
a
Para g temos g(x) = 0 se 0<x6 , portanto lim g(x) = 0.
2 x→0+

Segunda Parte. No caso dos limites à esquerda, podemos fazer contas análogas:

b b 1 −x 1 b b
− <x6− ⇒ 6 < ⇒n< 6 n + 1 ⇒ −(n + 1) 6 < −n
n n+1 n+1 b n −x x
(se −b/n < x 6 −b/(n + 1), então x é negativo e −x é positivo). Portanto:
   
x b n+1 b b
f (x) = =− x, se x ∈ − , − .
a x a n n+1
Assim, como feito na parte anterior, se h é uma função tal que
 
n+1 b b b
h(x) = · ∀x ∈ − , − ∀n ∈ N,
n a n n+1
b b b
então lim h(x) = e 6 f (x) 6 h(x) ∀x ∈ (−b, 0), portanto lim− f (x) = .
x→0− a a x→0 a
x
Se −a < x < 0, então −1 < < 0, de modo que
a
b hxi b
g(x) = =− ∀x ∈ (−a, 0),
x a x
donde podemos ver que lim g(x) = +∞, como queríamos. 
− x→0

f g
· · · b/a
···
224 CAPÍTULO 6. LIMITES DE FUNÇÕES

6.20 Exercício 20
Dadas f, g : X → R dena h = max{f, g} : X → R pondo h(x) = f (x) se f (x) > g(x) e
h(x) = g(x) caso g(x) > f (x). Seja a ∈ X 0 . Prove que se lim f (x) = L e lim g(x) = M , então
x→a x→a
lim h(x) = N , onde N é o maior dos dois números L e M.
x→a

(Voltar para a Solução do Exercício 3 do Capítulo VII: página 232.)

(Voltar para a Solução do item g) do Exercício 47 do Capítulo VII: página 278.)

Solução. Se M > L, pelo Teorema 5, seja δ1 > 0 tal que g(x) > f (x) ∀x ∈ Vδ1 . Dado
ε > 0, seja δ2 > 0 tal que g[Vδ2 ] ⊂ (M − ε, M + ε). Seja δ = min{δ1 , δ2 }, de modo que
h[Vδ ] = g[Vδ ] ⊂ g[Vδ2 ] ⊂ (M −ε, M +ε), pois, se x ∈ Vδ então g(x) > f (x), ou seja, h(x) = g(x);
e Vδ ⊂ Vδ2 . Portanto, lim h(x) = M = N .
x→a
M = L, dado ε > 0, sejam δ1 > 0 e δ2 > 0 tais que f [Vδ1 ] ⊂ (L − ε, L + ε) e g[Vδ2 ] ⊂
Se
(L − ε, L + ε). Assim, tomando δ = min{δ1 , δ2 } temos ambos f [Vδ ] e g[Vδ ] contidos no intervalo
(L−ε, L+ε). Deste modo, como h(x) é igual a f (x) ou é igual a g(x), segue h[Vδ ] ⊂ (L−ε, L+ε)
também, onde L = N , e, portanto, lim h(x) = N .
x→a
O caso M < L é inteiramente análogo ao primeiro páragrafo. 

6.21 Exercício 21
Sejam f, g : X → R limitadas numa vizinhança do ponto a ∈ X 0. Mostre que lim sup(f + g) 6
x→a
lim sup f (x) + lim sup g(x) e que lim sup[−f (x)] = − lim inf[f (x)]. Enuncie e prove resultados
x→a x→a x→a x→a
análogos para lim inf(f + g) e para o produto de duas funções. (Veja o Exercício 17, Capítulo
IV.)

Solução. Podemos considerar um mesmo δ0 > 0 tal que ambos os conjuntos f [Vδ0 ] e g[Vδ0 ]
são limitados (tomando, por exemplo, o menor dos deltas que atestam as limitações de f e de g
nas respectivas vizinhanças de raio delta de a, do mesmo modo como já zemos anteriormente).
Com exceção da parte referente ao produto, todo este exercício é consequência do Exercício 38
(página 87) e do Exercício 35 (página 85), ambos do Capítulo III. Por estes exercícios, para
cada δ ∈ (0, δ0 ], temos:

sup[(f + g)[Vδ ]] 6 sup[f [Vδ ]] + sup[g[Vδ ]],

inf[(f + g)[Vδ ]] > inf[f [Vδ ]] + inf[g[Vδ ]],


e
sup[−f [Vδ ]] = − inf[f [Vδ ]].
Portanto, pelo Corolário 2 do Teorema 5 e pelo Teorema 7, temos:

lim sup[(f + g)[Vδ ]] 6 lim sup[f [Vδ ]] + lim sup[g[Vδ ]],


δ→0 δ→0 δ→0

lim inf[(f + g)[Vδ ]] > lim inf[f [Vδ ]] + lim inf[g[Vδ ]],
δ→0 δ→0 δ→0
e
lim sup[−f [Vδ ]] = − lim inf[f [Vδ ]],
δ→0 δ→0
6.22. EXERCÍCIO 22 225

donde seguem, pelo Teorema 14,

lim sup(f + g) 6 lim sup f (x) + lim sup g(x),


x→a x→a x→a

lim sup[−f (x)] = − lim inf[f (x)]


x→a x→a

(como queríamos), e

lim inf(f + g) > lim inf f (x) + lim inf g(x)


x→a x→a x→a

(como solicitado).
No caso do produto, com a notação do Exercício 39 do Capítulo III (página 88), supondo
f (x) e g(x) positivos para todo x ∈ X (sob essas hipóteses é que foi feito o item (c) do Exercício
17 do Capítulo IV, página 128), temos

(f · g)[Vδ ] ⊂ f [Vδ ] · g[Vδ ] ∀δ ∈ (0, δ0 ].

Portanto, pelo Exercício 33 do Capítulo III (página 84), e ainda pelo referido Exercício 39,
temos

sup[(f · g)[Vδ ]] 6 sup[f [Vδ ]] · sup[g[Vδ ]]

inf[(f · g)[Vδ ]] > inf[f [Vδ ]] · inf[g[Vδ ]],

para todo δ ∈ (0, δ0 ]. Logo (Corolário 2 do Teorema 5, Teorema 7 e Teorema 14), assim como
aconteceu anteriormente, temos

lim sup(f · g) 6 lim sup f · lim sup g


x→a x→a x→a

lim inf(f · g) > lim inf f · lim inf g ,


x→a x→a x→a

se f (x) > 0 e g(x) > 0 para todo x ∈ X. 

6.22 Exercício 22
Seja f : [0, +∞) → R uma função limitada em cada intervalo limitado. Se lim [f (x + 1) −
x→+∞
f (x)
f (x)] = L, então lim = L.
x→+∞ x
(Voltar para a Solução do Exercício 20 do Capítulo VIII: página 298.)

Solução.
226 CAPÍTULO 6. LIMITES DE FUNÇÕES

Nível 1. Fixemos ε > 0. Seja A>0 tal que

L − ε < f (x + 1) − f (x) < L + ε ∀x > A.

Pela hipótese da limitação de f em cada intervalo limitado, sejam m e M tais que

m 6 f (x) 6 M ∀x ∈ [A, A + 1).

Assim, armamos que

m + k(L − ε) 6 f (x) 6 M + k(L + ε) ∀x ∈ [A + k, A + k + 1) ∀k ∈ N

(Nível 2.1) e, dividindo por x, vem

m k f (x) M k
+ (L − ε) 6 6 + (L + ε) ∀x ∈ [A + k, A + k + 1) ∀k ∈ N .
x x x x x
Fazendo x tender ao innito, ainda não podemos falar no limite deg(x) = f (x)/x porque não
sabemos ainda se esse limite existe, mas podemos falar no conjunto V A(g; +∞). A ideia aqui é
que o número k/x, que aparece nas desigualdades acima, tende a 1. Portanto V A(g; +∞) 6= ∅
pelo Corolário 3 do Teorema 13, pois g é limitada numa vizinhança do +∞ (Nível 2.2), e,
tomando y ∈ V A(g; +∞), temos L − ε 6 y 6 L + ε (Nível 2.3). Isto pode ser feito para
qualquer ε > 0, então um tal y só pode ser igual a L, ou seja V A(g; +∞) = {L}, donde, com a
limitação de g numa vizinhança do +∞, segue a tese pelo Corolário do Teorema 15 (bem como
pela sua observação).

Nível 2.1. Dado A > 0 tal que L − ε < f (x + 1) − f (x) < L + ε e sejam os
ε > 0, seja
números reais m e Mm 6 f (x) 6 M para todo x ∈ [A, A + 1). Fixado x ∈ [A, A + 1),
tais que
somando membro a membro as desigualdades L − ε < f (x + 1) − f (x) < L + ε e m 6 f (x) 6 M ,
obtemos

m + L − ε 6 f (x + 1) 6 M + L + ε,
onde x ∈ [A, A + 1) ⇔ x + 1 ∈ [A + 1, A + 2). Portanto

m + L − ε 6 f (x) 6 M + L + ε ∀x ∈ [A + 1, A + 2),

ou seja, a armação é verdadeira para k = 1. Fixados k ∈ N e x ∈ [A + k, A + k + 1), supondo


m + k(L − ε) 6 f (x) 6 M + k(L + ε), como L − ε 6 f (x + 1) − f (x) 6 L + ε, somando obtemos:

m + (k + 1)(L − ε) 6 f (x + 1) 6 M + (k + 1)(L + ε),

onde x ∈ [A + k, A + k + 1) ⇔ x + 1 ∈ [A + k + 1, A + k + 2). Portanto:

m + (k + 1)(L − ε) 6 f (x) 6 M + (k + 1)(L + ε) ∀x ∈ [A + k + 1, A + k + 2).

Logo,

m + k(L − ε) 6 f (x) 6 M + k(L + ε) ∀x ∈ [A + k, A + k + 1) ∀k ∈ N,

por indução em k ∈ N.
6.22. EXERCÍCIO 22 227

Nível 2.2. Sejam a ∈ {m, M } e α ∈ {L − ε, L + ε}. Dado k ∈ N, se x > A + k, então

|a + kα| |a + kα| |a| |kα| |a| |α|


6 6 + = + A 6 |a| + |α|,
x A+k A+k A+k A+k k
+ 1

A
pois A+k >1 e + 1 > 1, lembrando que A é positivo. Portanto,
k

f (x)
−(|m| + |L − ε |) 6 6 (|M | + |L + ε |) ∀x > A + 1.
x

De fato, dado x > A + 1, seja k∈N tal que A+k 6 x < A+k+1 (este k é k = max{j ∈
N; x > A + j}), então

m k f (x) M k
−(|m| + |L − ε |) 6 + (L − ε) 6 6 + (L + ε) 6 |M | + |L + ε |.
x x x x x
Logo, g é limitada numa vizinhança de +∞, como queríamos.

Nível 2.3. O nível acima, em combinação com o Corolário 3 do Teorema 13, implica
V A(g; +∞) 6= ∅. y ∈ V A(g; +∞) e seja (xn )n∈N uma sequência de pontos em [0, +∞) que
Seja
f (xn )
tende para o +∞ tal que lim = y . Como xn → +∞, seja n0 ∈ N tal que xn > A + 1
xn
para todo n ∈ N ∩[n0 , +∞). Assim, para cada n ∈ N ∩[n0 , +∞), seja kn ∈ N tal que xn ∈
[A + kn , A + kn + 1). Então temos

m kn f (xn ) M kn
(∗) + (L − ε) 6 6 + (L + ε) ∀n ∈ N ∩[n0 , +∞).
xn xn xn xn xn

Agora veja que A + kn 6 xn < A + kn + 1 implica

A kn A kn 1
+ 61< + + ,
xn xn x n xn xn

ou seja,
A kn A 1
61− < + .
xn xn xn xn
A A+1
Portanto, como lim = lim = 0, pelo Teorema 8 do Capítulo IV segue
xn xn
 
kn
lim 1 − = 0,
xn

isto é,
kn
lim = 1.
xn
Portanto, fazendo n → +∞ em (∗), obtemos o resultado desejado:

L − ε 6 y 6 L + ε. 
228 CAPÍTULO 6. LIMITES DE FUNÇÕES

Observação. A partir das desigualdades

m + k(L − ε) 6 f (x) 6 M + k(L + ε) ∀x ∈ [A + k, A + k + 1) ∀k ∈ N


f (x)
obtidas acima, podemos concluir que lim
= L não recorrendo ao conjunto dos valores de
x x→+∞
aderência no innito da função denida por f (x)/x, mas sim diretamente à denição de limite,
m k M k
notando que conforme o número natural k cresce, os números + (L − ε) e + (L + ε)
x x x x
se aproximam cada vez mais dos números L − ε e L + ε, respectivamente, não importando
quem seja o x ∈ [A + k, A + k + 1) (o que isto signica de modo mais preciso, bem como sua
demonstração, se encontram no lema mais abaixo). Seguindo esta proposta, a solução ca como
no parágrafo abaixo.
Dado ε > 0, sejam ε1 e ε2 números reais positivos tais que ε = ε1 + ε2 e seja A>0 tal que

x > A ⇒ L − ε1 < f (x + 1) − f (x) < L + ε1 .

Assim, para os números reais m e M tais que m 6 f (x) 6 M para todo x ∈ [A, A + 1), temos

m k f (x) M k
x ∈ [A + k, A + k + 1) ⇒ + (L − ε1 ) 6 6 + (L + ε1 ),
x x x x x
para cada k∈N (ver o Nível 2.1 da Solução acima). Seja k1 ∈ N tal que

m k
x ∈ [A + k, A + k + 1) ⇒ (L − ε1 ) − ε2 < + (L − ε1 )
x x
para cada k ∈ N ∩[k1 , +∞) e seja k2 ∈ N tal que

M k
x ∈ [A + k, A + k + 1) ⇒ + (L + ε1 ) < (L + ε1 ) + ε2
x x
para cada k ∈ N ∩[k2 , +∞) (é o lema abaixo quem garante as existências desses k1 e k2 ).
Portanto, para k0 = max{k1 , k2 }, e lembrando que ε1 + ε2 = ε, temos

f (x)
x ∈ [A + k, A + k + 1) ⇒ L − ε < < L + ε,
x
para cada k ∈ N ∩[k0 , +∞), isto é,

f (x)
x > A + k0 ⇒ L − ε < < L + ε,
x
f (x)
logo, pela denição de limite, lim = L, como queríamos.
x→+∞ x

Lema. Sejam a e α números reais e B > 0. Dado ε > 0, existe k0 ∈ N tal que,

a + kα
x ∈ [B + k, B + k + 1) ⇒ α − ε < < α + ε,
x
para cada k natural maior do que ou igual a k0 .
Solução. Se x ∈ [B + k, B + k + 1), então

1 1 1
< 6 .
B+k+1 x B+k
6.23. EXERCÍCIO 23 229

Se α > 0, seja k1 ∈ N tal que a + k1 α > 0. Assim, multiplicando todos os membros das
desigualdades acima por a + kα, para k ∈ N ∩[k1 , +∞) e para x ∈ [B + k, B + k + 1), temos
a + kα a + kα a + kα
< 6 ,
B+k+1 x B+k
a + kα a + kα
pois k > k1 > −a/α ⇒ a + kα > a + k1 α > 0. Como lim = lim = α,
k→+∞ B + k + 1 k→+∞ B + k
existem naturais k2 e k3 tais que

a + kα a + kα
k ∈ N ∩[k2 , +∞) ⇒ α − ε < , e k ∈ N ∩[k3 , +∞) ⇒ < α + ε.
B+k+1 B+k
Portanto, neste caso, o k0 procurado é k0 = max{k1 , k2 , k3 }.
Se α < 0, fazemos k0 = max{k1 , k2 , k3 }, para k1 ∈ N tal que a + k1 α < 0 e k2 , k3 ∈ N tais
que

a + kα a + kα
k ∈ N ∩[k2 , +∞) ⇒ α − ε < , e k ∈ N ∩[k3 , +∞) ⇒ < α + ε,
B+k B+k+1
pois, neste caso,
a + kα a + kα a + kα
k ∈ N ∩[k1 , +∞) ⇒
6 < ,
B+k x B+k+1
já que k > k1 > −a/α ⇒ a + kα 6 a + k1 α < 0.
|a| |a| |a|
Se α = 0, tomamos k0 tal que k ∈ N ∩[k0 , +∞) ⇒ < ε, pois 0 6 6 para
B+k x B+k
todo k ∈ N, se x ∈ [B + k, B + k + 1).
Segue a tese. 

6.23 Exercício 23
Seja f :R→R denida por f (x) = x + ax · sen x. Mostre que |a| < 1 ⇒ lim f (x) = ±∞.
x→±∞

Solução. Temos −1 6 sen x 6 1 ∀x ∈ R.


Se x>0 e a > 0, então
−ax 6 ax sen x
e
(1 − a)x 6 f (x).
Se x>0 e a < 0, então
ax 6 ax sen x
e
(1 + a)x 6 f (x).
Portanto, destes dois parágrafos anteriores, lim f (x) = +∞ porque lim (1 − a)x = +∞
x→+∞ x→+∞
e lim (1 + a)x = +∞, uma vez que 1−a>0 e 1 + a > 0.
x→+∞
Se x <0 e a > 0, então
ax sen x 6 −ax
e
f (x) 6 (1 − a)x.
230 CAPÍTULO 6. LIMITES DE FUNÇÕES

Se x<0 e a < 0, então


ax sen x 6 ax
e
f (x) 6 (1 + a)x.
Portanto lim f (x) = −∞ porque lim (1 − a)x = −∞ e lim (1 + a)x = −∞, uma vez
x→−∞ x→−∞ x→−∞
que 1−a> 0 e 1+ a > 0. 

6.24 Exercício 24
Seja p : R → R um polinômio não-constante. Dado b ∈ R, suponha que existe uma sequência
(xn ), tal que lim p(xn ) = b ∈ R. Prove que (xn ) é limitada e o conjunto dos seus valores de
−1
aderência é não-vazio, contido em p (b). Em particular, se existe uma sequência (xn ), tal que
lim p(xn ) = 0, então p tem alguma raiz real.

Solução. Pela forma contrapositiva, vejamos que se (xn )n∈N não é limitada, então a sequência
(p(xn ))n∈N não é limitada (se ela não é limitada, então ela não pode ser convergente). Se (xn )n∈N
não é limitada, então dela podemos extrair uma subsequência (xn )n∈M , com M ⊂ N innito,
que tende para +∞ ou para −∞. É consequência do Exercício 13 (página 217, e pelo que
também já foi mencionado em sua observação) que a sequência (p(xn ))n∈M tende para +∞ ou
para −∞. Logo (p(xn ))n∈N não é limitada, como queríamos.
Segue do parágrafo anterior que, se (p(xn ))n∈N converge para b, então (xn )n∈N é limitada.
Assim, desta sequência, podemos extrair uma subsequência (xn )n∈M , com M ⊂ N innito,
convergente ( Apêndice ao Ÿ4 [do Capítulo IV], página 125 do livro), ou seja, o conjunto A
dos valores de aderência da sequência (xn ) é não vazio, como queríamos.
Seja c ∈ A. Seja M ⊂ N innito tal que a subsequência (xn )n∈M convirja para c (porque c
é valor de aderência de (xn )). Assim, aplicando sucessivamente o Teorema 7, podemos concluir
que
p(c) = lim p(xn ),
n∈M

onde lim p(xn ) = b, porque, se p(xn ) → b, então qualquer subsequência de (p(xn ))n∈N também
n∈M
converge para b (Teorema 2 do Capítulo IV). Portanto p(c) = b (unicidade do limite, Teorema
−1
1 do Capítulo IV), ou seja, c ∈ p [{b}], como queríamos. A conclusão de que p tem alguma
raiz real, se existe uma sequência (xn ) tal que p(xn ) → 0, é imediata. 
Capítulo 7
Funções Contínuas

7.1 Exercício 1
Seja f : R → R contínua. Mostre que o conjunto Zf = {x ∈ R; f (x) = 0} é fechado. Conclua
que, se f, g : R → R são contínuas então C = {x ∈ R; f (x) = g(x)} é um conjunto fechado.

Solução 1. Vamos mostrar que seu complementar R −Zf a∈/ Zf , então f (a) 6= 0.
é aberto. Se
Daí, pelo Corolário do Teorema 3 (e as observações que o seguem), seja δ > 0 tal que f (x) 6= 0
para todo x ∈ I = (a − δ, a + δ), isto é, I ⊂ R −Zf , onde I é um intervalo aberto ao qual x
pertence. Logo R −Zf é aberto, como queríamos.
Para o conjunto C , temos:

C = {x ∈ R; (f − g)(x) = 0},

que é um conjunto Zf −g , onde f −g é contínua pelo Teorema 5. O resultado agora segue do


primeiro parágrafo. 

Solução 2. Mostrar que Zf é fechado é equivalente a mostrar que Zf = Zf . A inclusão


Zf ⊂ Zf já temos, pois já sabemos que qualquer conjunto está contido em seu fecho. Para a
outra inclusão, sejam x ∈ Zf e (xn )n∈N uma sequência de pontos em Zf tal que lim xn = x.
Queremos concluir que f (x) = 0, o que segue do Teorema 4:

f (x) = lim f (xn ) = 0,

pois f é contínua (e, em particular, contínua em x) e f (xn ) = 0 para todo n ∈ N (já que
xn ∈ Zf ∀n ∈ N). 

7.2 Exercício 2
Seja f : X → R contínua, denida num subconjunto X ⊂ R. Para todo k ∈ R o conjunto dos
pontos x ∈ X tais que f (x) 6 k tem a forma F ∩ X , onde F é fechado. Resultado análogo
vale para o conjunto dos pontos x ∈ X , tais que f (x) = k e para o conjunto dos pontos x ∈ X ,
nos quais duas funções contínuas f, g : X → R assumem valores iguais. Em particular, se X é
fechado então esses três conjuntos são fechados.

(Voltar para a Solução do Exercício 5: página 233.)

231
232 CAPÍTULO 7. FUNÇÕES CONTÍNUAS

Solução. Já vimos no texto (antes do Teorema 4) que o conjunto A = {x ∈ X; f (x) > k} é


da forma A = U ∩ X , onde U ⊂ R é um conjunto aberto. Seja B = {x ∈ X; f (x) 6 k}. Então,
de X = A ∪ B , como A ∩ B = ∅, temos:

B = X − A = X − (U ∩ X) = X − U = X ∩ (R −U ),

onde F = R −U é o conjunto fechado procurado.


Igual feito na Solução 1 do exercício anterior, para concluirmos a parte faltante deste exer-
cício, basta mostrarmos que o mesmo vale para o conjunto Zf = {x ∈ X; f (x) = 0}, porque
ambas as funçõesf − k e f − g são contínuas.
O conjunto Zf = {x ∈ X; f (x) = 0} é o conjunto dos pontos x ∈ X tais que f (x) 6 0
menos o conjunto dos pontos x ∈ X tais que f (x) < 0. De forma análoga ao que foi feito no
trecho já referenciado do livro (aquele antes do Teorema 4), podemos concluir que este é da
forma U ∩ X, para algum conjunto aberto U . Para aquele, de acordo com o primeiro parágrafo
desta solução, seja F fechado tal que F ∩ X = {x ∈ X; f (x) 6 0}. Deste modo:

Zf = (F ∩ X) − (U ∩ X) = X ∩ (F − U ),

onde F − U é um conjunto fechado pelo Exercício 26 do Capítulo V (página 176). Segue a tese.
Se X é fechado, os três referidos conjuntos são fechados porque interseção de fechados é
fechada. 

7.3 Exercício 3
Dadas f, g : X → R, denamos as funções f ∨ g : X → R e f ∧ g : X → R pondo, para cada
x ∈ X , (f ∨ g)(x) = max{f (x), g(x)} e (f ∧ g)(x) = min{f (x), g(x)}. Mostre que se f e g são
contínuas num ponto a ∈ X o mesmo se dá com f ∨ g e f ∧ g .

(Voltar para a Solução do Exercício 28 do Capítulo IX: página 384.)

(Voltar para a Solução do Exercício 13 do Capítulo X, para seu Nível 4.2: página 419.)

Solução. Pelas continuidades da f e da g, temos:

lim f (x) = f (a) e lim g(x) = g(a).


x→a x→a

Pelo Exercício 20 do Capítulo 6 (página 224), temos

lim (f ∨ g)(x) = max{f (a), g(a)} = (f ∨ g)(a),


x→a

ou seja,f ∨ g é contínua em a, se a ∈ X 0 . Se a ∈
/ X 0 , seja I = (a − δ, a + δ), com δ > 0, tal que
I ∩ X = {a}; para este I temos (f ∨ g)(x) = max{f (x), g(x)} = max{f (a), g(a)} = (f ∨ g)(a)
∀x ∈ I ∩ X e, portanto, dado ε > 0:

x ∈ I ∩ X ⇒ |(f ∨ g)(x) − (f ∨ g)(a)| = 0 < ε,

o que mostra que (f ∨ g) é contínua em a se a é ponto isolado de X. Logo (f ∨ g) é contínua


em a. O caso f ∧ g é análogo. 
7.4. EXERCÍCIO 4 233

7.4 Exercício 4
Uma função f : A → R, denida num aberto A ⊂ R, é contínua se, e somente se, para todo
c∈R os conjuntos E[f < c] = {x ∈ A; f (x) < c} e E[f > c] = {x ∈ A; f (x) > c} são abertos.

(Voltar para a Observação após a Solução do Exercício 10: página 236.)

Solução. Se f é contínua, o conjunto E[f > c] é aberto porque A é aberto (observação feita
antes do Teorema 4). O E[f < c] é aberto por motivo análogo.
Vamos supor agora ambos E[f > c] e E[f < c] abertos qualquer que seja o c ∈ R. Dados
ε>0 e a ∈ A, temos
a ∈ E[f < f (a) + ε] ∩ E[f > f (a) − ε].
Portanto existe δ>0 tal que

(a − δ, a + δ) ⊂ E[f < f (a) + ε] ∩ E[f > f (a) − ε],


isto é, sea − δ < x < a + δ , então: x ∈ A, f (x) < f (a) + ε e f (x) > f (a) − ε, ou seja, x ∈ A
e f (a) − ε < f (x) < f (a) + ε, logo, f é contínua em a (a hipótese de A ser aberto foi usada
apenas na ida). Como a era um ponto qualquer de A, segue a continuidade de f em todo seu
domínio. 

7.5 Exercício 5
Sejaf : F → R denida num conjunto fechado F ⊂ R. A m de que f seja contínua é necessário
e suciente que, para todo c ∈ R, sejam fechados os conjuntos E[f 6 c] = {x ∈ F ; f (x) 6 c} e
E[f > c] = {x ∈ F ; f (x) > c}.
(Voltar para a Solução do Exercício 10: página 235.)

(Voltar para a Observação após a Solução do Exercício 10: página 236.)

(Voltar para a Solução do Exercício 13 do Capítulo X, para seu Nível 4.2: página 419.)

Solução. Se f é contínua, o conjunto E[f 6 c] é fechado pelo Exercício 2 (página 231). De


forma análoga, o conjunto E[f > c] é fechado também.
Reciprocamente, sejam a ∈ F e ε > 0. Se E[f 6 f (a) − ε] e E[f > f (a) + ε] são fechados,
então a interseção de seus complementares é aberta, isto é, é aberto o conjunto
   
Y = R −F ∪ E[f > f (a) − ε] ∩ E[f < f (a) + ε] .

Como a∈Y, e Y é aberto, seja δ>0 tal que I = (a − δ, a + δ) ⊂ Y , portanto

F ∩ I ⊂ E[f > f (a) − ε] ∩ E[f < f (a) + ε],


o que signica que f é contínua em a. Logo f é contínua em F. 

7.6 Exercício 6
Dado um subconjunto não-vazio S ⊂ R, dena f : R → R pondo f (x) = inf{|x − s|; s ∈ S}.
Prove que |f (x) − f (y)| 6 |x − y| para quaisquer x, y ∈ R. Conclua que f é (uniformemente)
contínua.

(Voltar para a Solução do Exercício 24: página 250.)


234 CAPÍTULO 7. FUNÇÕES CONTÍNUAS

Solução. Pela Desigualdade Triangular (item (i) do Teorema 2, Capítulo III), temos:

|x − s| 6 |x − y| + |y − s| ∀s ∈ S

e
|y − s| 6 |y − x| + |x − s| ∀s ∈ S.
Assim, tomando os ínmos dos dois lados de cada uma das inequações acima, obtemos

f (x) 6 |x − y| + f (y)

e
f (y) 6 |x − y| + f (x)
(se fosse, por exemplo, f (x) > |x − y| + f (y), então existiria s ∈ S tal que |x − s| > f (x) >
|y − s| + |x − y|, porque teríamos f (x) − |x − y| > inf |y − s|). Portanto
s∈S

f (x) − f (y) 6 |x − y|

e
f (y) − f (x) 6 |x − y|,
ou seja:
|f (x) − f (y)| = max{f (x) − f (y), f (y) − f (x)} 6 |x − y|,
como queríamos. A f é uniformemente contínua porque ela é lipschitziana pelo que acabamos
de ver (e pelo Exemplo 23). 

7.7 Exercício 7
Sejam f, g : X → R contínuas. Se Y ⊂ X e f (y) = g(y) para todo y ∈ Y , então f |Y = g|Y .
Conclua que se duas funções contínuas f, g : R → R são tais que f (r) = g(r) para todo r ∈ Q,
então f = g.
(Voltar para a Solução do Exercício 36 do Capítulo IX: página 397.)

Solução. Seja y ∈ Y. Seja (yn )n∈N uma sequência de pontos em Y que converge para y.
Então, primeiro pelo Teorema 4, depois por hipótese e novamente pelo Teorema 4, temos:

f (y) = lim f (yn ) = lim g(yn ) = g(y).

Logo f (y) = g(y) ∀y ∈ Y , como queríamos.


Como consequência, se f (r) = g(r) ∀r ∈ Q, então f (x) = g(x) ∀x ∈ R (se R for o domínio
de ambas f e g) porque Q ⊂ R. 

7.8 Exercício 8
Sejam f, g : [0, 1] → R contínuas. Se f (1) = g(0), então é contínua a função h : [0, 1] → R,
denida por h(x) = f (2x) se 0 6 x 6 1/2, e h(x) = g(2x − 1) se 1/2 6 x 6 1. É também
∗ ∗
contínua a função f : [0, 1] → R, denida por f (x) = f (1 − x).
7.9. EXERCÍCIO 9 235

Solução. As restrições de h a conjuntos fechados são composições de funções contínuas. Serão


úteis então o Teorema 6 e o Corolário do Teorema 7.
Sejam os conjuntos fechados F1 = [0, 1/2] e F2 = [1/2, 1]. Sejam as funções f1 : F1 → R
e g1 : F2 → R denidas por f1 (x) = 2x ∀x ∈ F1 e g1 (x) = 2x − 1 ∀x ∈ F2 . Temos f1 e g1
contínuas e ambas as imagens f1 [F1 ] e g1 [F2 ] contidas em [0, 1]. Além disso, como h|F1 = f ◦ f1
e h|F2 = g ◦ g1 são compostas de funções contínuas, segue que ambas as restrições h|F1 e h|F2
são contínuas. Logo h é contínua pelo Corolário do Teorema 7, porque seu domínio é a reunião
F1 ∪ F2 .
∗ ∗
Para a função f é análogo; temos f = f ◦f2 , sendo f2 : [0, 1] → R denida por f2 (x) = 1−x
∀x ∈ [0, 1], cuja imagem está contida no domínio de f ∗ (e portanto faz sentido falar nesta
composição). Como ambas f e f2 são contínuas, o resultado segue pelo Teorema 6. 

7.9 Exercício 9
Seja A⊂R aberto. A m de que f : A → R seja contínua é necessário e suciente que f −1 (B)
seja aberto, qualquer que seja B ⊂ R aberto.

Solução. Vamos supor f contínua eB aberto. Se a ∈ f −1 [B], então f (a) ∈ B . Como B é


aberto, existe ε > 0 tal que (f (a) − ε, f (a) + ε) ⊂ B . Como f é contínua em a, existe δ > 0 tal
−1
que f [A ∩ (a − δ, a + δ)] ⊂ (f (a) − ε, f (a) + ε), ou seja, A ∩ (a − δ, a + δ) ⊂ f [B]. Como A é
0 0 0
aberto, seja δ > 0 tal que (a − δ , a + δ ) ⊂ A ∩ (a − δ, a + δ). Este intervalo de centro em a e
0 −1
raio δ é que atesta ser a um ponto interior de f [B]. Como a era um ponto qualquer, segue
−1
que f [B] é aberto.
Reciprocamente, seja a ∈ A. Dado ε > 0, seja B o intervalo aberto (f (a) − ε, f (a) + ε).
−1
Como, por hipótese, f [B] é aberto e a ∈ f −1 [B], seja δ > 0 tal que (a − δ, a + δ) ⊂ f −1 [B].
−1
Como f [B] ⊂ A, temos também (a − δ, a + δ) ⊂ A. Portanto, pelo Exercício 15 do Capítulo
1 (página 12), temos
f [(a − δ, a + δ)] ⊂ f [f −1 [B]] ⊂ B.
Logo f é contínua em a e, portanto, contínua em A porque o ponto a era um ponto qualquer.
(Para uma segunda solução do exercício acima, ver a observação feita após a solução do
exercício abaixo.)

7.10 Exercício 10
Seja F ⊂ R fechado. Uma função f : F → R é contínua se, e somente se, para todo conjunto
fechado G ⊂ R, sua imagem inversa f −1 (G) é fechada.

(Voltar para o Item (ii) da Solução do Exercício 14 do Capítulo VII: página 239.)

−1
Solução. Vamos supor f
contínua e G fechado. Para mostrar que f [G] é fechado, seja
−1
(xn )n∈N uma sequência de pontos em f [G] que converge para um ponto x; vamos mostrar que
x ∈ f −1 [G]. Como F é fechado e f −1 [G] ⊂ F , primeiro temos x ∈ F , de modo que faz sentido
falar em f (x). Como f é contínua, temos lim f (xn ) = f (x) (Teorema 4). Como f (xn ) ∈ G para
todo n e G é fechado, temos f (x) ∈ G, logo x ∈ f −1 [G], como queríamos.
Reciprocamente, supondo que as imagens inversas de conjuntos fechados são fechadas, para
mostrar que f é contínua podemos usar o Exercício 5 (página 233), vericando que, para um
236 CAPÍTULO 7. FUNÇÕES CONTÍNUAS

dado c ∈ R, ambos os conjuntos:

E[f > c] = {x ∈ F ; f (x) > c} e E[f 6 c] = {x ∈ F ; f (x) 6 c}

são fechados. Eles são porque são imagens inversas dos respectivos conjuntos fechados

G1 = [c, +∞) e G2 = (−∞, c] :

E[f > c] = f −1 [G1 ] e E[f 6 c] = f −1 [G2 ].


Segue a tese. 

Observação. Já que o Exercício 5 (página 233) foi mencionado, ele e o Exercício 4 (página
233) também podem ser usados para resolver as outras partes dos Exercícios 9 e 10, como
veremos abaixo. Para o que segue, seja a função f : X → R.
−1
Se f f −1 [(−∞, c)] e f −1 [(c, +∞)]
[B] é aberto para todo B ⊂ R aberto, então, em particular,
são abertos para quaisquer c ∈ R. Mas estes dois últimos conjuntos são os conjuntos E[f < c]
e E[f > c], respectivamente. Logo uma tal f é contínua se X é aberto.
Reciprocamente, se B é um subconjunto aberto de R, sabemos que podemos escrever B
[
como uma reunião de intervalos abertos  por exemplo, B = Iy , onde, para cada y ∈ B ,
y∈B
Iy é um intervalo aberto contido em
[ B e ao qual y pertence . Para uma coleção (Iλ )λ∈L de

intervalos abertos tal que B= Iλ , temos


λ∈L
" #
[ [ [
B= Iλ ⇒ f −1 [B] = f −1 Iλ = f −1 [Iλ ].
λ∈L λ∈L λ∈L

Dado o intervalo aberto (a, b), temos

f −1 [(a, b)] = {x ∈ X; a < f (x) < b} = E[f > a] ∩ E[f < b].

Assim, se f é contínua e X é aberto, então f −1 [Iλ ] é aberto para todo λ ∈ L, logo, f −1 [B],
sendo uma reunião de abertos, é aberto.
De modo análogo, se G ⊂ R é fechado,
[ então R −G é aberto e, para a coleção (Iλ )λ∈L de

intervalos abertos tais que R −G = Iλ , temos


λ∈L
[ \
G = R− Iλ = R −Iλ
λ∈L λ∈L

e, então, " #
\ \
−1 −1
f [G] = f R −Iλ = f −1 [R −Iλ ] .
λ∈L λ∈L

Agora, dado o intervalo aberto (a, b), temos

f −1 [R −(a, b)] = {x ∈ X; f (x) 6 a ou f (x) > b} = E[f 6 a] ∪ E[f > b].

Deste modo, se f é contínua e X é fechado, então f −1 [G] é uma interseção de conjuntos fechados,
portanto, fechado.
7.11. EXERCÍCIO 11 237

7.11 Exercício 11
SejaX = Y ∪ Z . Se f : X → R é tal que f |Y e f |Z são contínuas então f é contínua em todo
ponto a ∈ Y ∩ Z .

Solução. Dado ε > 0, fazendo Iε = (f (a) − ε, f (a) + ε), por hipótese, existem δ1 > 0 e δ2 > 0
tais que
f [Y ∩ (a − δ1 , a + δ1 )] ⊂ Iε e f [Z ∩ (a − δ2 , a + δ2 )] ⊂ Iε .
Assim, para δ = min{δ1 , δ2 }, temos:

f [X ∩ (a − δ, a + δ)] = f [Y ∩ (a − δ, a + δ)] ∪ f [Z ∩ (a − δ, a + δ)] ⊂ Iε .

Portanto, f é contínua em a. 

7.12 Exercício 12
Uma função f :X→R é descontínua no ponto a ∈ X se, e somente se, existem ε > 0 e uma
1
sequência de pontos xn ∈ X tais que |xn − a| 6 e |f (xn ) − f (a)| > ε para todo n ∈ N.
n

Solução. Se f a, então existe ε > 0 tal que, para todo n ∈ N, existe


é descontínua no ponto
1
xn ∈ X tal que |xn − a| 6 e |f (xn ) − f (a)| > ε. Isto nos dá a sequência procurada.
n
1
Reciprocamente, se (xn )n∈N é uma sequência em X tal que |xn − a| 6 e |f (xn ) − f (a)| > ε
n
1
para todo n ∈ N e para um certo ε, dado δ > 0, seja n ∈ N tal que < δ (Propriedade
n
Arquimediana); para este n ∈ N temos |xn − a| < δ e |f (xn ) − f (a)| > ε; portanto basta fazer
xn = xδ na denição de descontinuidade da página 229. 

7.13 Exercício 13
Seja F ⊂R um conjunto fechado. Dada uma função contínua f : F → R, existe ϕ:R→R
contínua tal que ϕ|F = f . (Sugestão : Dena ϕ linearmente nos intervalos componentes de
R −F de modo a coincidir com f nos extremos de cada intervalo.)

Solução. Como R −F é aberto, pelo Teorema 2 do Capítulo V (Estrutura dos abertos da


reta), escrevamos
[
R −F = (an , bn ),
n∈N

com (am , bm ) ∩ (an , bn ) = ∅ se m 6= n e denamos:

ϕ(x) = f (x) ∀x ∈ F

e
f (bn ) − f (an )
ϕ(x) = f (an ) + (x − an ) ∀x ∈ (an , bn ) ∀n ∈ N .
b n − an
Para a continuidade de ϕ em R, dado um número real a, pelo Teorema 4 vamos mostrar
que lim ϕ(xn ) = ϕ(a), onde (xn )n∈N é uma sequência de números reais que converge para a.
238 CAPÍTULO 7. FUNÇÕES CONTÍNUAS

Os pontos dessa sequência podem pertencer a F ou a R −F . Sabendo a que conjunto


cada ponto pertence, sabemos o comportamento da ϕ naquele ponto. Sejam então M = {n ∈
N; xn ∈ F } e N = {n ∈ N; xn ∈ R −F }.
Se M é innito, então a ∈ F porque F é fechado, e

lim ϕ(xn ) = lim f (xn ) = f (a) = ϕ(a),


n∈M n∈M

porque f é contínua em a
lim xn = a (lembrando que, se uma sequência converge para um
e
n∈M
ponto, então qualquer uma de suas subsequências converge para o mesmo ponto  Teorema 2
do Capítulo IV , resultado este que será usado outras vezes abaixo).
Se N é innito, para cada n ∈ N, seja m(n) ∈ N tal que xn ∈ (am(n) , bm(n) ). Podemos ter
as seguintes possibilidades consideradas em cada um dos parágrafos abaixo.
Se a pertence a algum Jn0 = (am(n0 ) , bm(n0 ) ), então, por causa da convergência lim xn =
n∈N
a, existe n1 ∈ N tal que xn ∈ Jn0 para todo n ∈ N maior do que ou igual a n1 . Então
(am(n) , bm(n) ) = Jn0 para todo n ∈ N , n > n1 (porque os intervalos (an , bn ) são dois a dois
disjuntos) e
lim ϕ(xn ) = ϕ(a),
n∈N
porque ϕ é contínua no intervalo Jn0 , por construção.
Caso contrário, isto é, se a ∈
/ (am(n) , bm(n) ) ∀n ∈ N , sejam N1 = {n ∈ N ; bm(n) 6 a} e
N2 = {n ∈ N ; am(n) > a}.
Se N1 é innito, note que lim bm(n) = a, porque xn < bm(n) 6 a para todo n ∈ N1 e
n∈N1
lim xn = a (Teorema 8 do Capítulo IV). Se também ocorrer lim am(n) = a, então seguirá
n∈N1 n∈N1
lim ϕ(xn ) = ϕ(a), porque
n∈N1

f (am(n) ) 6 ϕ(xn ) 6 f (bm(n) ) ou f (bm(n) ) 6 ϕ(xn ) 6 f (am(n) ) ∀n ∈ N1 ,


por construção, e por causa da continuidade de f em a (a existência da sequência (bm(n) )n∈N1 ,
a qual converge para a, atesta a ∈ F, uma vez que bm(n) ∈ F ∀n ∈ N1 e F é fechado).
Porém, pode acontecer de a sequência (am(n) )n∈N1 não convergir para a, que é o caso que
iremos considerar neste parágrafo.
(am(n) )n∈N1 é limitada (senão, lembrando que am(n) < bm(n) 6 a ∀n ∈ N1 e
A sequência
que a coleção {(an , bn ) ∈ P(R); n ∈ N} é de intervalos dois a dois disjuntos, obteríamos uma
subsequência de (bm(n) )n∈N1 que não convergeria para a, contrariando o fato de que lim bm(n) =
n∈N1
a, já visto acima). Então, de (am(n) )n∈N1 , podemos extrair uma subsequência convergente
( Apêndice ao Ÿ4 [do Capítulo IV], página 125 do livro). Sem perda de generalidade, vamos
supor que a própria sequência (am(n) )n∈N1 já é convergente. Seja então α seu limite; este α
é menor do que a. Aqui também vai acontecer algo que já aconteceu anteriormente: existe
n0 ∈ N1 tal que (am(n) , bm(n) ) = (α, a) para todo n ∈ N1 , n > n0 . Como lim bm(n) = a e α < a,
n∈N1
existe n ∈ N1 tal que bm(n) > α. Se, para todos os bm(n) 's tivéssemos bm(n) < a, então existiriam
n e n0 em N1 tais que α < bm(n) < bm(n0 ) < a, o que nos daria (am(n) , bm(n) ) ∩ (am(n0 ) , bm(n0 ) ) 6= ∅,
apesar de estes dois intervalos serem distintos. Portanto, existe n0 ∈ N1 tal que bm(n) = a para
todo n ∈ N1 , n > n0 . Para este n0 também ocorre am(n) = α para todo n ∈ N1 , n > n0 pelo
mesmo motivo do porquê ser bn(m) = a. Agora, lim ϕ(xn ) = ϕ(a) segue da continuidade de ϕ
n∈N1
no intervalo [α, a] = [an0 , bn0 ].
O caso em que N2 é innito é análogo ao caso em que N1 é innito.
Com o auxílio do Exercício 4 do Capítulo IV (página 118), à luz dos parágrafos anteriores,
podemos nalmente concluir que lim ϕ(xn ) = ϕ(a), como queríamos. 
n∈N
7.14. EXERCÍCIO 14 239

7.14 Exercício 14
Seja f :R→R contínua. Prove que as seguintes condições sobre f são equivalentes.

a) lim |f (x)| = lim |f (x)| = +∞;


x→+∞ x→−∞

b) Se |xn | → +∞, então |f (xn )| → +∞

c) Se K é compacto, então f −1 (K) é compacto.

Solução.

(i) [a) ⇒ b)] Seja (xn )n∈N tal que |xn | → +∞. Seja M > 0. Queremos encontrar n0 ∈ N
tal que |f (xn )| > M para todo n > n0 .
Por hipótese, seja A > 0 tal que

|x| > A ⇒ |f (x)| > M.

Como |xn | → +∞, seja n0 tal que

n > n0 ⇒ |xn | > A.

Portanto:

n > n0 ⇒ |xn | > A ⇒ |f (xn )| > M,


como queríamos.

(ii) [b) ⇒ c)] Se K é compacto, então K


é fechado e limitado (Teorema 11 do Capítulo
−1
V). Por este mesmo Teorema 11 do Capítulo V, vamos mostrar que f [K] é limitado e fechado.
−1
Pelo Exercício 10 (página 235) f [K] é fechado, porque K é fechado (e R também) e f é
contínua.
−1
Vamos mostrar que f [K] é limitado pela contrapositiva, isto é, vamos mostrar que se
f [K] não é limitado, então K não é limitado. Se f −1 [K] não é limitado, seja (xn )n∈N uma
−1
−1
sequência de pontos em f [K] tal que |xn | → +∞. Por hipótese segue então |f (xn )| → +∞,
onde f (xn ) ∈ K para todo n ∈ N, portanto K não é limitado, como queríamos.

(iii) [c) ⇒ a)] Vamos mostrar a contrapositiva desta implicação. Como f −1 [K] é fechado
se K é fechado, só por causa da continuidade de f (e independentemente de qualquer outra
hipótese), supondo a negação de a), vamos encontrar um conjunto limitado cuja imagem inversa
por f não seja limitada.
Por hipótese (pela negação do item a)), existeM > 0 tal que, para todo A > 0, existe x ∈ R
tal que |x| > A mas |f (x)| < M . Fixemos um tal M . Assim, para cada n ∈ N, seja xn ∈ R tal
que |xn | > n. Isto nos dá um conjunto X = {xn ∈ R; n ∈ N} ilimitado, com

X ⊂ f −1 [f [X]]

(Exercício 14 do Capítulo I, página 12). Portanto f −1 [f [X]] é ilimitado, mas f [X] é limitado.
Logo o conjunto procurado é f [X]. 
240 CAPÍTULO 7. FUNÇÕES CONTÍNUAS

7.15 Exercício 15
Dena uma bijeção f :R→R que seja descontínua em todos os pontos.

Solução. A denida por


x, se x∈Q e
f (x) =
x + 1, se x ∈ R−Q.

Ela é injetiva. Dados dois números reais x1 e x2 distintos, se ambos são racionais ou se
ambos são irracionais, então suas imagens são claramente distintas; se x1 ∈ Q e x2 ∈ R − Q ,
então f (x1 ) 6= f (x2 ) porque f (x1 ) = x1 ∈ Q e f (x2 ) = x2 + 1 ∈ R − Q.

Ela é sobrejetiva. Dado y ∈ R, se y ∈ Q, então y = f (y); se y ∈ R − Q, então y =


f (y − 1), onde y − 1 ∈ R − Q.

Ela é descontínua em todos os pontos. Podemos ver isso à luz do Teorema 4. Dado
a ∈ R, pela densidade de ambos os conjuntos Q e R−Q (Teorema 4 do Capítulo 3), podemos
considerar uma sequência (an )n∈N de números reais convergindo para a tal que a2n−1 ∈ Q e
a2n ∈ R − Q ∀n ∈ N, de modo que:

f (a2n−1 ) = a2n−1 → a e f (a2n ) = a2n + 1 → a + 1,

ou seja, o limite de f (x) conforme x se aproxima de a sequer existe. 

7.16 Exercício 16
Seja f :X→R monótona, tal que f (X) seja denso num intervalo limitado. Mostre que existe
uma única função contínua (monótona) ϕ : X → R, tal que ϕ|X = f .

(Voltar para a Solução do Exercício 45 do Capítulo IX: página 404.)

Solução. Conforme a prática do livro, f [X] ser denso num intervalo limitado signica que
existe um intervalo limitado I tal que f [X] ⊂ I e I ⊂ f [X].
A ideia desta solução é a mesma do Teorema 18. Denimos

ϕ(a) = lim f (x) ∀a ∈ X.


x→a

Se a ∈ X , então ϕ(a) = lim f (x) = f (a), porque f é contínua em X pelo Teorema 10, portanto
x→a
ϕ|X = f . Se a ∈ X − X, este limite existe por causa do Teorema 12 do Capítulo VI (f
é monótona e limitada), do Exercício 6 do Capítulo VI (página 208; este exercício se aplica
porque f [X] ⊂ [inf I, sup I] e I ⊂ f [X] ⇒ I = [inf I, sup I] ⊂ f [X]) e do Teorema 11 também
do Capítulo VI.
7.16. EXERCÍCIO 16 241

Continuidade. Dado a ∈ X, queremos δ>0 tal que:

x ∈ X ∩ (a − δ, a + δ) ⇒ ϕ(a) − ε < ϕ(x) < ϕ(a) + ε .


Como ϕ(a) é o limite de f (x) com x tendendo a a, existe δ1 > 0 tal que

ϕ(a) − ε < f (x) < ϕ(a) + ε ∀x ∈ X ∩ (a − δ1 , a + δ1 ).


Se x ∈ X, então f (x) = ϕ(x) e este δ1 resolve. Se x ∈ X0 (se x ∈ X, então x ∈ X ou
0
x∈X ), os trabalhos com os já referidos teoremas e exercício nos dão a dica do que fazer. Por
eles sabemos que ϕ(x) é um ínmo ou um supremo. Assim, as desigualdades acima (as que
envolvem f) implicam
ϕ(a) − ε 6 lim f (y) 6 ϕ(a) + ε,
y→x

se x ∈ X 0 ∩ (a − δ1 , a + δ1 ).
Então, só para evitar as igualdades que aí aparecem, o δ procurado (em qualquer um dos
0
casos x ∈ X ou x ∈ X ) é o mesmo δ > 0 tal que:

ε ε
ϕ(a) −
< f (y) < ϕ(a) + ∀y ∈ X ∩ (a − δ, a + δ),
2 2
o qual existe porque ϕ(a) é o limite de f (y) para y tendendo a a.
Conforme já justicamos, para este δ temos
ε ε
ϕ(a) − 6 lim f (y) 6 ϕ(a) + ∀x ∈ X ∩ (a − δ, a + δ),
2 y→x 2
ou seja,
ϕ(a) − ε < ϕ(x) < ϕ(a) + ε ∀x ∈ X ∩ (a − δ, a + δ),
como queríamos.

Monotonicidade. Armamos que o tipo de monotonicidade de ϕ é o mesmo de f , isto é,


ϕ é não decrescente se f é não decrescente e ϕ f é não crescente. Vejamos.
é não crescente se
Vamos supor f f é não crescente é análogo). Sejam a < b
não decrescente (o caso em que
pontos em X . Tomando δ = (a + b)/2, temos f (x) 6 f (y) ∀x ∈ X ∩ (a − δ, a + δ) e ∀y ∈
X ∩ (b − δ, b + δ), porque x < y se x ∈ (a − δ, a + δ) e y ∈ (b − δ, b + δ). Isto implica ϕ(a) 6 ϕ(b).
De fato, se fosse ϕ(a) > ϕ(b), tomando ε = (ϕ(a) − ϕ(b))/2, sejam δ1 > 0 e δ2 > 0 tais que

f (x) ∈ (ϕ(a) − ε, ϕ(a) + ε) ∀x ∈ X ∩ (a − δ1 , a + δ1 )


e
f (y) ∈ (ϕ(b) − ε, ϕ(b) + ε) ∀y ∈ X ∩ (b − δ2 , b + δ2 ),
os quais existem porque lim f (x) = ϕ(a) e lim f (x) = ϕ(b). Podemos supó-los tais que a + δ1 6
x→a x→b
b − δ2 . x ∈ X ∩ (a − δ1 , a + δ1 ) e y ∈ X ∩ (b − δ2 , b + δ2 ) (os quais existem
Assim, tomando
porque a, b ∈ X ), temos f (x) > f (y), apesar de ser x < y , o que contraria a hipótese de f ser
não decrescente. Logo ϕ(a) 6 ϕ(b) e ϕ é não decrescente.

Unicidade. ψ : X → R uma função contínua tal que ψ|X = f . Dado x ∈ X ,


Seja
queremos mostrar que ψ(x) = ϕ(x). Como x é ponto de aderência de X , seja (xn )n∈N uma
sequência de pontos em X que converge para x. Deste modo, respectivamente pelo Teorema 4,
pela igualdade ψ(xn ) = f (xn ) ∀n ∈ N e pela denição de ϕ, temos

ψ(x) = lim ψ(xn ) = lim f (xn ) = ϕ(x),


como queríamos. 
242 CAPÍTULO 7. FUNÇÕES CONTÍNUAS

7.17 Exercício 17
Seja K o conjunto de Cantor. Escreva A = [0, 1] − K e dena uma função monótona não-
decrescente f : A → R, constante em cada intervalo componente de A, tal que f [A] seja o
m
conjunto das frações da forma pertencentes a [0, 1]. Mostre que existe uma função monótona
2n
contínua ϕ : [0, 1] → R que coincide com f nos pontos de A. (Função de Cantor.)

(Voltar para a Solução do Exercício 26 do Capítulo IX: página 382.)

Solução. Uma vez denida uma tal função f, a existência da função ϕ será consequência do
exercício anterior, uma vez que f [A]
[0, 1] e A = [0, 1]. Esta última igualdade vale
é denso em
por causa do seguinte. Primeiro que A ⊂ [0, 1] ⇒ A ⊂ [0, 1], porque [0, 1] é fechado. Segundo
que, se [0, 1] − A 6= ∅, então existiria um intervalo aberto I 6= ∅ contido em [0, 1] e tal que
I ∩ A = ∅ (note que 0 e 1 pertencem a A), o que implicaria I ⊂ K (pois [0, 1] = A ∪ K )
contrariando o fato de o interior de K ser vazio, conforme já vimos no Exemplo 15 do Capítulo
V.
Para construir a função f, observamos que A é igual a uma reunião X1 ∪ X2 ∪ · · · X n ∪ · · · ,
onde cada Xn é a reunião dos terços médios retirados na etapa n. Assim, cada Xn é a reunião
n
de 2 − 1 intervalos abertos e dois a dois disjuntos.

I14 I24 I34 I44 I54 I64 I74 I84 I94 I10
4 4
I11 4
I12 4 4 4
I13 I14 I15
X4

1 2 3
0 0 1 1 X3
4 4 4

1
0 1 X2
2

1
X1
2
2 7
9 9
Escrevamos
     
Xn = I1n ∪ ··· ∪ I2nn−2 ∪ I2nn−2 +1 ∪ ··· ∪ I2nn−2 +(2n−1 −1) ∪ n
I3·2n−2 ∪ ··· ∪ I2nn −1 ,

para cada n > 2, onde cada Iin é um terço médio retirado na etapa n e todo ponto de Iin é
menor do que todo ponto de Ijn se i < j , e denamos f (x) = 0 se x ∈ A ∩ (0, 2/9), f (x) = 1 se
x ∈ A ∩ (7/9, 1) e
m
f (x) = , se x ∈ I2nn−2 +m
2n−1
para n>2 e m ∈ {1, . . . , 2n−1 − 1}.
Observamos que:
(i)
n+1
Iin = I
2i
n+2
=I4i = · · · = I2n+p
p ·i ,

n 2 n−2
(ii) Ii ⊂ 0, se i ∈ {1, 2, . . . , 2 },
 9
n 7
(iii) Ii ⊂ , 1 se i ∈ {3 · 2n−2 , 3 · 2n−2 + 1, . . . , 2n − 1}
9
7.18. EXERCÍCIO 18 243

e que
m n
(iv) se Ii ∩ Ij 6= ∅, então existe p ∈ N tal que Ij
n
= I2m+p
p ·i se n > m, de modo que, se
m+p
x ∈ I2mm−2 +k ∩ I2nn−2 +l , então, para p ∈ N tal n
que I2n−2 +l = I2m+p−2 +2p ·k , temos
p
2 ·k k
f (x) = m+p−1
= m−1 .
2 2
Estas observações signicam que f está denida em todo o conjunto A e está bem denida, isto
é, que não demos valores diferentes para um mesmo f (x).
Os itens (i), (ii) e (iii) podem ser demonstrados por indução. A ideia central do (i) é que
n
antes de um intervalo Ii existem i intervalos os quais terão seus terços médios perdidos na
n
etapa n + 1. O (ii) e o (iii) decorrem do (i) e, para eles, basta considerar os intervalos I2n−2 e
n n n
I3·2 n−2 respectivamente, lembrando que o intervalo Ii vem antes do Ij se i < j , por construção.

Para o item (iv) temos o seguinte. Se n > m, então existe p ∈ N tal que n = m + p e, pelo item
m+p
m n m n n n
(i), Ii = I2p ·i = I2p ·i . Daí, se Ii ∩ Ij 6= ∅, então I2p ·i ∩ Ij 6= ∅ e, portanto, Ijn = I2np ·i = I2m+p
p ·i ,

como queríamos.

3/4

1/2

1/4


(Voltar para a Solução do Exercício 45 do Capítulo IX: página 404.)

7.18 Exercício 18
Seja f : R → R uma função arbitrária. Para cada n ∈ N, consideremos o conjunto Cn , formado
pelos pontos a ∈ R com a seguinte propriedade: existe um intervalo aberto I , contendo a, tal
1
que x, y ∈ I ⇒ |f (x) − f (y)| < . Prove:
n
1. Cada Cn é um conjunto aberto;

2. f é contínua no ponto a se, e somente se, a ∈ Cn para todo n ∈ N;


Conclua que o conjunto dos pontos de continuidade de qualquer função f : R → R é uma
interseção enumerável de abertos. Em particular (veja o Exercício 55, Capítulo V) não existe
uma função f :R→R que seja contínua nos pontos racionais e descontínua nos irracionais.

(Voltar para a Solução do Exercício 19 do Capítulo IX: página 374.)

(Voltar para a Solução do Exercício 27 do Capítulo IX: página 382.)

(Voltar para a Solução do Exercício 13 do Capítulo X, para o elevador 1: página 418.)

(Voltar para a Solução do Exercício 13 do Capítulo X, para o elevador 2: página 418.)
244 CAPÍTULO 7. FUNÇÕES CONTÍNUAS

Solução. Dado a ∈ Cn , o aberto possuindo a Cn é o próprio intervalo I do


contido em
enunciado. De fato, I faz b pertencer a Cn b ∈ I , ou seja, I ⊂ Cn .
qualquer que seja o

Para demonstrar a ida do item 2, suponhamos f contínua em a ∈ R. Dado ε > 0, sabemos


que existe δ>0 tal que

x ∈ (a − δ, a + δ) ⇒ |f (x) − f (a)| < ε .

Assim, pela Desigualdade Triangular (item (i) do Teorema 2, Capítulo III), se x, y ∈ (a−δ, a+δ),
então

|f (x) − f (y)| 6 |f (x) − f (a)| + |f (y) − f (a)| < 2 ε .

Portanto, a testemunha de que a é membro de Cn para um dado n∈N é o intervalo aberto


I = (a − δ, a + δ) tal que
1
x ∈ I ⇒ |f (x) − f (a)| < .
2n
Reciprocamente, dado a ∈ R, suponhamos a ∈ Cn ∀n ∈ N. Dado ε > 0, queremos δ>0 tal
que

|f (x) − f (a)| < ε ∀x ∈ (a − δ, a + δ).


1
Pela Propriedade Arquimediana, seja n ∈ N tal que < ε. Por hipótese, temos a ∈ Cn ,
n
portanto existe um intervalo aberto I ao qual a pertence tal que

1
|f (x) − f (y)| < <ε ∀x, y ∈ I.
n

Em particular, como a ∈ I:
|f (x) − f (a)| < ε ∀x ∈ I.

É deste I que vem o δ procurado: seja δ>0 tal que (a − δ, a + δ) ⊂ I ; ele existe porque a∈I
e I é aberto. Portanto, pela desigualdade acima, temos (em particular)

|f (x) − f (a)| < ε ∀x ∈ (a − δ, a + δ),

como queríamos.
\
Segue que o conjunto de todos os pontos nos quais f é contínua é o conjunto Cn , uma
n∈N
interseção enumerável de abertos.

Se existisse uma função f : R → R contínua em Q e descontínua em R − Q, então o


conjunto de todos os pontos nos quais f é contínua seria Q e, então, pelo que vimos, Q seria
uma interseção enumerável de abertos. Por outro lado, pelo Exercício 55 do Capítulo V (página
195), Q não pode ser igual a uma interseção enumerável de conjuntos abertos. Logo uma tal
função não existe, como queríamos. 

7.19 Exercício 19
Não existe f : R → R contínua que transforme todo número racional num irracional e vice-versa.
7.20. EXERCÍCIO 20 245

Solução. A ideia central desta solução é que o conjunto dos números racionais é muito pe-
queno para dar conta de todos os irracionais, portanto vai existir um irracional que não está
na imagem de f e a conclusão vem pelo Teorema do Valor Intermediário.
Seja f : R → R uma função tal que f (x) ∈ R − Q para todo x ∈ Q e f (x) ∈ Q para todo
x ∈ R − Q. Vamos mostrar que f não é contínua.
Sejam a um número racional e b um número irracional, de modo que f (a) e f (b) são distintos
(f (a) ∈ R − Q e f (b) ∈ Q). Vamos supor a < b para podermos falar no intervalo [a, b] (essa
hipótese é sem perda de generalidade, porque tanto faz quem é menor do que quem). O
conjunto f [Q] é no máximo enumerável (Teorema 9 do Capítulo II; a restrição f | Q : Q → f [Q]
é sobrejetiva), enquanto o conjunto de todos os números irracionais compreendidos entre f (a)
e f (b) é não enumerável. Deste modo, existe um número irracional y entre f (a) e f (b) tal que
y∈/ f [Q]; este y também não pode ser imagem de um número irracional, porque f [R − Q] ⊂ Q
por hipótese. Em particular, não existe x ∈ [a, b] tal que f (x) = y . Assim, pelo Teorema do
Valor Intermediário (Teorema 12 e Observação 1 da página 236), a restrição de f ao intervalo
[a, b] não é contínua. Portanto, pelo Teorema 1, f não é contínua  pois, se fosse, então qualquer
uma de suas restrições também seria , como queríamos. 

7.20 Exercício 20
Seja f : I → R, contínua, limitada num intervalo. Para todo a ∈ I , o conjunto dos valores
de aderência de f no ponto a é um intervalo (compacto). Isto inclui a = −∞ e a = +∞, no
caso de ser ilimitado o intervalo I . Em particular, se f : [0, +∞) → R é contínua e limitada, o
conjunto dos pontos da forma c = lim f (xn ) onde xn → +∞ é um intervalo fechado e limitado.

Observação. Antes de passarmos à solução, uma observação acerca do uso do artigo inde-
nido um na primeira oração do enunciado deste exercício.
Levando em conta que no Corolário 1 e no Corolário 2, ambos do Teorema 12, o autor
também usou este artigo indenido antes do termo referente ao domínio da função em questão,
podemos suspeitar que a f considerada aqui é na verdade contínua e limitada no intervalo
I todo. Deste modo, lembrando da denição de função contínua (página 223 do livro) e da
denição de função limitada (Exercício 38 do Capítulo III, página 87), este exercício poderia
simplesmente dizer que f :I→R é contínua e limitada e que I é um intervalo.
De fato, se interpretarmos este um como algum, signicando que existe um intervalo
J ⊂ I f é contínua e f [J] é limitado, então este exercício torna-se-ia falso. Por
no qual
exemplo, para I = R a função f : I → R denida por f (x) = 0 se x < 0 e f (x) = 1 se
x > 0 é contínua e limitada em algum intervalo, por exemplo, no intervalo (1, 2). Porém:
0 ∈ I , enquanto V A(f ; 0) = {0, 1} não é um intervalo. Um outro exemplo ocorre para I = R
e f (x) = x para todo x ∈ I . Esta função é contínua (em todo seu domínio) e limitada num
intervalo, porém V A(f ; +∞) = ∅ e o conjunto vazio não é um intervalo de acordo com as
denições da página 70 e das observações da página 71.

Solução. Pelo Corolário 3 do Teorema 13 do Capítulo VI, o conjunto V A(f ; a) é compacto e


não vazio, pois se f é limitada, então, com maior razão, f é limitada em cada vizinhança de a
(a denição de função limitada numa vizinhança está na página 213 do livro). Esta armação
vale mesmo quando a é igual a +∞ ou igual a −∞, pela observação do último parágrafo da
página 215. Só falta mostrar então que V A(f ; a) é um intervalo. Lembrando da respectiva
246 CAPÍTULO 7. FUNÇÕES CONTÍNUAS

denição (página 70 do livro), já temos uma condição necessária para tanto: em R nenhum
intervalo pode ser o conjunto vazio.

Vamos mostrar que V A(f ; a) é um intervalo usando o Exercício 23 do Capítulo V (página


174). Sejam então α < β dois pontos de V A(f ; a) (se este conjunto tem apenas um ponto,
então ele já é um intervalo, um intervalo degenerado). Dado um ponto γ entre α e β , queremos
mostrar que γ ∈ V A(f ; a). Pela denição de valor de aderência de uma função, vamos obter
uma sequência de pontos (zn )n∈N em I − {a} que converge para a e tal que f (zn ) → γ . Note
como o estar entre do qual falamos agora há pouco lembra o Teorema do Valor Intermediário.

Como α e β f em a, por denição sejam as sequências (xn )n∈N


são valores de aderência de
e (yn )n∈N , ambas tendendo para a, com xn , yn ∈ I − {a} para todo n ∈ N, tais que f (xn ) → α
e f (yn ) → β . Seja n0 ∈ N tal que f (xn ) < γ < f (yn ) para todo n > n0 . Assim, pelo Teorema
do Valor Intermediário (Teorema 12, e também pela Observação 1), para cada n > n0 , seja
zn entre xn e yn tal que f (zn ) = γ (cada zn é obtido restringindo f ao conjunto [xn , yn ] ou ao
conjunto [yn , xn ]). Claramente zn ∈ I , porque xn , yn ∈ I e I é um intervalo. Se algum zn = a,
então a ∈ I e γ = f (a) ∈ V A(f ; a). Se todo zn é distinto de a, então a sequência (zn )n∈N é
de pontos em I − {a} e tende para a, por causa do Teorema 8 do Capítulo IV: um dos dois
conjuntos {n ∈ N; xn < zn < yn } ou {n ∈ N; yn < zn < xn } é innito; seja X aquele que
for innito, de modo que o referido teorema pode ser aplicado porque ambas as subsequências
(xn )n∈X e (yn )n∈X tendem para a. Segue então γ ∈ V A(f ; a) como queríamos.

Esta mesma solução funciona quando a = +∞ ou a = −∞. A armação feita após o em
particular é imediata: por denição, no caso em questão, o conjunto dos pontos da forma
c = lim f (xn ) onde xn → +∞ é justamente o conjunto V A(f ; +∞), o qual é fechado e limitado
porque todos os compactos são assim pelo Teorema 11 do Capítulo V. 

Um exemplo. A função seno é contínua e limitada; o conjunto de seus valores de aderência


no +∞, assim como no −∞, é o intervalo [−1, 1]. Em outras palavras, V A(sen; +∞) =
V A(sen; −∞) = [−1, 1].

x1 x2 x3 x4

Outra justicativa do porquê V A(f ; a) ser não vazio. Se a ∈ I , então, pela continuidade
0
de f , V A(f ; a) = {a}. Se a ∈ I , por causa da denição de V A(f ; a), consideremos as sequências
(xn )n∈N e (f (xn ))n∈N , onde xn ∈ I − {a} para todo natural n e xn → a (uma tal sequência de
xn 's existe porque estamos supondo a ponto de acumulação de I ). Pela hipótese da limitação de
f , a sequência (f (xn ))n∈N é limitada. Podemos armar então que existe um conjunto innito
M ⊂ N tal que a sequência (f (xn ))n∈M é convergente. Isto pela combinação do Exercício
15 do Capítulo IV (o do ponto destacado, o qual arma que toda sequência admite uma
subsequência monótona) com o Teorema 4 do Capítulo IV (o qual nos garante ser convergente
toda sequência monótona e limitada). Seja c o ponto para o qual esta sequência converge.
Este c pertence a V A(f ; a), porque a sequência (xn )n∈M ainda converge para a (Teorema 2 do
Capítulo IV). Logo V A(f ; a) 6= ∅, qualquer que seja o a ∈ I , como queríamos.
7.21. EXERCÍCIO 21 247

7.21 Exercício 21
Sejam T o conjunto dos números transcendentes negativos e A o conjunto dos números algébri-
2
cos > 0. Dena f : T ∪ A → [0, +∞) pondo f (x) = x . Mostre que f é uma bijeção contínua,
cuja inversa é descontínua em todos os pontos, exceto 0.

Solução.

(a) Continuidade da f . A f é contínua porque é restrição de uma função contínua


(Teorema 1; a associação x 7→ x2 é contínua pelo Teorema 5, como já foi observado no Exemplo
2).

(b) Sobrejetividade. y ∈ [0, +∞), já sabemos que existe x ∈ R tal que x2 = y


Dado
2 2 2
(Exemplo 13 do Capítulo III). Seja então x ∈ R tal que y = x . Temos y = x = (−x) . Se
y = 0, então x = 0, que é membro de A. Se y > 0, então um dos dois dentre x ou −x é
positivo e o outro é negativo. Pelo Exercício 47 do Capítulo III (página 94), o conjunto dos
números algébricos é um corpo. Assim podemos armar que x e −x são ambos algébricos ou são
ambos transcendentes (no Lema 1 abaixo temos outra justicativa deste fato). Se são ambos
algébricos, então y = f (|x|), com |x| ∈ A. Se ambos são transcendentes, então y = f (−|x|),
com −|x| ∈ T .

Lema 1. x, x é algébrico se, e somente se, −x é algébrico.


Dado um número real

Solução. Se x é algébrico, seja p(t) = an tn + · · · + a0 um polinômio não identicamente nulo


com coecientes inteiros tal que p(x) = 0. Precisamos encontrar um polinômio q(t), também
não identicamente nulo e com coecientes inteiros, tal que q(−x) = 0. Este polinômio é o
n n n−1
denido por q(t) = (−1) an t + (−1) an−1 tn−1 + · · · + (−1)a1 t + a0 . De fato:

q(−x) = (−1)n an (−x)n + (−1)n−1 an−1 (−x)n−1 + · · · + (−1)a1 (−x) + a0 = p(x) = 0.

Reciprocamente, se −x é algébrico, então, pelo que acabamos de mostrar, o número −(−x) = x


é algébrico.

(c) Injetividade. x1 e x2 pontos distintos do conjunto T ∪ A. Se x1 , x2 ∈ A ou


Sejam
se x1 , x2 ∈ T , então f (x1 ) 6= f (x2 ). No caso em que x1 ∈ T e x2 ∈ A, armamos que um
x ∈ [0, +∞) é algébrico se, e somente se, x2 também o é (a justicativa disto está no parágrafo
2 2 2
abaixo). Deste modo, o número x1 é transcendente e o número x2 é algébrico: se x1 não fosse
transcendente, então ele seria algébrico e, pela armação que acabamos de fazer, o número −x1
seria algébrico e, portanto, o x1 também seria algébrico, conforme vimos no item anterior. Logo
f (x1 ) 6= f (x2 ), como queríamos.
2
Se x é algébrico, então x = x · x é algébrico porque o conjunto dos números algébricos é um
corpo, pelo Exercício 47 do Capítulo III (página 94; o Lema 2 abaixo nos dá outra justicativa
2 n
deste fato). Se x é algébrico, seja an t + · · · + a0 um polinômio (em t) não identicamente nulo
2
com coecientes inteiros do qual x é uma raiz, de modo que

0 = an (x2 )n + an−1 (x2 )n−1 + · · · + a1 (x2 ) + a0


= an x2n + an−1 x2(n−1) + · · · + a1 x2 + a0 ,

ou seja, x é raiz do polinômioan t2n + an−1 t2(n−1) + · · · + a1 t2 + a0 , não identicamente nulo e


com coecientes inteiros. Logo x é algébrico como queríamos.
248 CAPÍTULO 7. FUNÇÕES CONTÍNUAS

Lema 2. Se um número real x é algébrico, então x2 é também algébrico.

Solução.

Nível 1. p(t) = an tn + · · · + a0 um polinômio não identicamente nulo


Como sempre, seja
com coecientes inteiros tal que p(x) = 0. Queremos encontrar um polinômio com essas mesmas
2
propriedades do qual x é uma raiz.

2
No elevador. Podemos começar conjecturando que um tal polinômio é p(t) = p(t)·p(t),
mas, de imediato, essa conjectura se revela falsa. Para formar uma outra conjectura e conseguir
enxergar melhor o que deve acontecer, vamos investigar alguns casos mais concretos, a saber,
os casos em que o grau de p é 1 ou 2.
Para n = 1, xé um número racional da forma a/b (a, b ∈ Z, b 6= 0); ele é raiz do polinômio
2 2 2 2
bt − a. Seu quadrado a /b é raiz do polinômio b t − a , isto é, b 2 x 2 − a2 = 0 . Podemos
reescrever:
0 = b2 x2 − a2 = (bx − a)(bx + a).
Isto nos dá a dica do que fazer no caso n = 2. Se ax2 + bx + c = 0, então

0 = (ax2 + bx + c) · (ax2 + c − bx) = (ax2 + c)2 − (bx)2 .


2
Este é um polinômio em x . Note que, no caso n = 1, também poderíamos ter escrito 0 =
(bx−a)(−bx−a) = −b2 x2 +a2 e, portanto, x2 é raiz do polinômio −b2 t+a2 . Assim vai surgindo
o polinômio p(t)q(t), onde q(t) é o mesmo polinômio visto na solução do lema anterior. São
essas observações que nos levam a armação do nível abaixo.

Nível 2. Sejaq(t) = (−1)n an tn + (−1)n−1 an−1 tn−1 + · · · + (−1)a1 t + a0 . Claramente,


se p(x) = 0, então p(x) · q(x) = 0. Além disso, armamos que p(t) · q(t) é um polinômio
2 2 m 2 m−1
em t . Seja ele p(t)q(t) = bm (t ) + bm−1 (t ) + · · · + b1 (t2 ) + b0 , com bi ∈ Z para todo
m m−1
i ∈ {1, . . . , m}. Então bm t + bm−1 t + · · · + b1 t + b0 é o polinômio procurado pelo Nível 1:
2
ele é não identicamente nulo, seus coecientes são inteiros e x é uma de suas raizes.

No elevador. A justicativa desta armação (e também o esboço de sua demonstração)


 
é a seguinte. O produto p(t) · q(t) é da forma a(t) + b(t) · a(t) − b(t) , onde a(t) é o somatório
k k
dos ak t para k par entre 0 e n, e b(t) é o somatório dos ak t para k ímpar entre 0 e n (de
modo que p(t) = a(t) + b(t)). Assim todos os expoentes das potências de base t do produto
2 2
p(t)q(t) = a(t) − b(t) são pares, porque o quadrado de uma soma é a soma do quadrado
de suas parcelas mais duas vezes a soma dos produtos dois a dois dessas parcelas, além de ser
par a soma de dois números pares ou a soma de dois números ímpares.

(d) Descontinuidade da inversa.


√ √ f é a função g : [0, +∞) → T ∪ A
A inversa da
denida por se x ∈ A e g(x) = − −x se x ∈ T . Dado x ∈ [0, +∞), podemos nos
g(x) = x
aproximar dele através de uma sequência (xn )n∈N de números algébricos e de uma sequência
(yn )n∈N de números transcendentes, ambas em [0, +∞). Isto é consequência dos Exercícios 44
e 46 do Capítulo III (páginas 93 e 93). Para essas sequências temos:
√ √
lim g(xn ) = x e lim g(yn ) = − x,
que são números distintos se x 6= 0. Além disto, lim g(zn ) = 0 qualquer que seja a sequência
(zn )n∈N de números em [0, +∞) convergente para 0. Portanto, pelo Teorema 4 podemos concluir
que g é contínua apenas no 0.
7.22. EXERCÍCIO 22. TEOREMA DE BROUWER EM DIMENSÃO 1 249

Observação. O Teorema 13 arma que uma função contínua injetiva cujo domínio é um
intervalo é monótona e sua inversa é contínua. Este exercício nos dá uma função contínua
injetiva (cujo domínio não é um intervalo) que não é monótona e cuja inversa não é contínua.

7.22 Exercício 22. Teorema de Brouwer em dimensão 1


Seja f : [a, b] → [a, b] contínua. Prove que f possui um ponto xo, isto é, existe x ∈ [a, b] tal
que f (x) = x. (Teorema de Brouwer em dimensão 1.) Dê exemplo de uma função contínua
f : [0, 1) → [0, 1) sem ponto xo.

Solução. Para um tal ponto x temos (f − g)(x) = 0, onde g : [a, b] → [a, b] é a função
identidade. Em outras palavras, denindo h como a diferença f − g , queremos x ∈ [a, b] tal
que h(x) = 0.
Tendo o Teorema do Valor Intermediário (Teorema 12) em mente, como f (a) ∈ [a, b] e
f (b) ∈ [a, b], temos f (a) > a e f (b) 6 b, ou seja, h(a) > 0 e h(b) 6 0. Se h(a) = 0, então x = a;
se h(b) = 0, então x = b. Se h(a) > 0 e h(b) < 0, então, pelo referido teorema, existe x ∈ [a, b]
tal que h(x) = 0, pois h é contínua (Teorema 5). De qualquer forma, sempre existe um tal x.
Para construir uma função f : [0, 1) → [0, 1) contínua sem ponto xo, levando em conta o que
zemos no parágrafo acima, observamos que f (0) > 0 e que, portanto, uma condição necessária
para f é termos f (x) > x para todo x ∈ [0, 1). Seu gráco pode ser, por exemplo, um segmento
de reta que liga os pontos (0, 1/2) e (1, 1), ou seja, a função denida por f (x) = 1/2 + x/2
∀x ∈ [0, 1). De um modo um pouco mais geral, qualquer função fα : [0, 1) → [0, 1) denida por
fα (x) = α + (1 − α)x ∀x ∈ [0, 1) para um dado α ∈ (0, 1) é contínua e não tem ponto xo. 

7.23 Exercício 23
Seja n ímpar. Prove que, para todo y ∈ R, existe um único x ∈ R, tal que xn = y e que,
√ √
escrevendo x = n y, a função y 7→ n y, assim denida é um homeomorsmo de R sobre R.

Solução. Podemos seguir a mesma ideia do Exemplo 18. A função f :R→R denida por
f (x) = xn para todo x real é contínua, injetiva e sobrejetiva.
É injetiva porque é crescente: Sex e y são números reais tais que x < y , então podemos ter:
x > 0 e y > 0; x 6 0 e y > 0; ou y 6 0. Se ambos x e y são positivos, então xn < y n por causa
n n n
do Exercício 11 do Capítulo III (página 72). Se x 6 0 e y > 0, então x < y porque x 6 0 e
250 CAPÍTULO 7. FUNÇÕES CONTÍNUAS

y n > 0. Se x < y 6 0, então 0 6 −y < −x, n n n


o que implica 0 6 (−y) = −y < (−x) = −x
n
n n
(também pelo Exercício 11, Capítulo III) e, portanto, x < y 6 0. De qualquer forma temos
f (x) < f (y), se x < y.
É sobrejetiva porque lim f (x) = +∞ e lim f (x) = −∞ (Exercício 13 do Capítulo VI,
x→+∞ x→−∞
página 217) e f [R] f [R] = R.
é um intervalo (Corolário 2 do Teorema 12). Desta forma
Por ser injetiva e sobrejetiva segue que, dado y ∈ R, existe um único x ∈ R tal que f (x) = y ,
n

ou seja, tal que x = y , como queríamos. A associação x 7→ x para todo x ∈ R é a inversa
n

de f . Ela é contínua pelo Teorema 13, portanto um homeomorsmo. 

7.24 Exercício 24
Sejam K compacto e F fechado, não-vazios. Mostre que existem x0 ∈ K , y0 ∈ F , tais que
|x0 − y0 | 6 |x − y|, para quaisquer x ∈ K , y ∈ F . Dê exemplo de dois conjuntos fechados e
disjuntos F , G tais que inf{|x − y|; x ∈ F, y ∈ G} = 0.

Solução. Já lidamos anteriormente com exercícios relacionados a este. Pelo Exercício 6 (pá-
gina 233), dado um conjunto não vazio X ⊂ R, a função f : X → R denida por f (x) = d(x, S)
∀x ∈ X para um dado conjunto S 6= ∅ de números reais, onde d(x, S) é a distância do ponto x
ao conjunto S do Exercício 29 do Capítulo V (página 177), é uniformemente contínua. Assim,
pelo Teorema de Weierstrass (Corolário do Teorema 14), se X é compacto, então f atinge um
mínimo.
Considerando então esta função f para X=K e S = F, seja x0 ∈ K
f (x0 ) 6 f (x) tal que
∀x ∈ K . Pelo item 2) do já referido Exercício 29 do Capítulo V (página 177), como F é fechado,
para este x0 , existe y0 ∈ F tal que f (x0 ) = |x0 − y0 |. Logo:

|x0 − y0 | 6 f (x) = d(x, F ) = inf{|x − z|; z ∈ F } 6 |x − y| ∀x ∈ K ∀y ∈ F,


como queríamos.
Para a segunda parte deste exercício, armamos que os conjuntos
 
1
F =N e G= n+ ; n∈N
n
possuem as propriedades desejadas. Claramente F é fechado e F ∩ G = ∅. Para ver que G é
fechado, notemos que a distância entre dois pontos distintos de G nunca é menor do que meio:
m e n, se m > n, então
dados os naturais distintos
     
1 1 n−m 1 1 1
m+ − n+ =m−n+ = (m − n) 1 − >1− >
m n mn mn mn 2
(se m > n, então, por denição, existe p∈N tal que m=n+p e, portanto, m − n > 1), uma
vez que
1 1 1 1
n > 1, m > 2 ⇒ mn > 2 ⇒ − >− ⇒1− > .
mn 2 mn 2
Assim, se existisse x ∈ G0 , então existiriam pelo menos dois pontos de G no intervalo (x −
1/4, x + 1/4) (pelo Teorema 7 do Capítulo V existiriam innitos pontos neste intervalo), e a
distância entre eles seria menor do que meio, contrariando o que acabamos de notar. Logo
G0 = ∅ e G = G ∪ G0 = G. Finalmente, dado ε > 0, pela Propriedade Arquimediana, seja
n ∈ N tal que 1/n < ε. Para x = n, que é um ponto de F, e para y = n + 1/n, que é um ponto
de G, temos |x − y| = 1/n < ε, donde segue a tese. 
(Voltar para a Solução do Exercício 39: página 266.)
7.25. EXERCÍCIO 25 251

Outra justicativa do porquê G0 = ∅. Dado um conjunto X de números reais, a existência


de um número positivo M tal que |x − y| > M , quaisquer que sejam x, y ∈ X distintos, signica
que não existem sequências de Cauchy de elementos de X dois a dois a distintos e, portanto, não
existem sequências convergentes de elementos de X dois a dois distintos (pois toda sequência
0
convergente é de Cauchy, Teorema 12 do Capítulo IV), logo X = ∅ (Teorema 7 do Capítulo
V).

7.25 Exercício 25
Seja f :R→R contínua. Se, para todo aberto A ⊂ R, sua imagem f (A) for aberta, então f é
injetiva e portanto monótona.

Solução. Uma vez mostrada a injetividade, a monotonicidade segue do Teorema 13. Na


verdade, lembrando da denição de monotonicidade (como uma função não crescente ou não
decrescente), a conclusão deste teorema é mais forte: uma função contínua e injetiva cujo
domínio é um intervalo é crescente ou decrescente.
Fixada a função contínua f : R → R, vamos mostrar a contrapositiva do resultado desejado,
isto é, que se f f não é aberta.
não é injetiva, então existe um conjunto aberto cuja imagem por
Suponhamos então que existam reais x1 e x2 distintos tais que f (x1 ) = f (x2 ). Sem perda
de generalidade, podemos supor x1 < x2 . Seja I = (x1 , x2 ) e, pelo Teorema de Weierstrass
(Corolário do Teorema 14), sejam a, b ∈ I tais que f (a) 6 f (x) 6 f (b) para todo x ∈ I . Se
a, b ∈ {x1 , x2 }, então f [I] = {f (a)} (lembrando que f (x1 ) = f (x2 )). Caso contrário, como
a, b ∈ I = I ∪ {x1 , x2 }, então a ∈ I ou b ∈ I . Se a ∈ I , então f (a) ∈ f [I] − int(f [I]), pois, se
f (a) ∈ int(f [I]), então existiriam elementos menores do que f (a) em f [I], contrariando o fato
de que f (x) > f (a) para todo x ∈ I . Analogamente, se b ∈ I , então f (b) ∈ f [I] − int(f [I]). De
qualquer forma, I é um conjunto aberto tal que f [I] não é aberto. Segue a tese. 

Observação. Ainda dentro da solução acima, sejam X = {a ∈ I; f (a) 6 f (x) ∀x ∈ I}


e Y = {b ∈ I; f (x) 6 f (b) ∀x ∈ I}. Ambos X e Y são não vazios (por Weierstrass) e
subconjuntos de I = I ∪ {x1 , x2 }. Além do que, f [I] é um intervalo (Corolário 2 do Teorema
12). Assim, temos as seguintes possibilidades:

• se X ∪ Y ⊂ {x1 , x2 }, então f [I] = {f (x1 )};


• se X ⊂ {x1 , x2 } e Y 6⊂ {x1 , x2 }, então f [I] = (f (x1 ), f (b)], para b ∈ Y ∩ I;
• se X 6⊂ {x1 , x2 } e Y 6⊂ {x1 , x2 }, então f [I] = [f (a), f (b)], para a∈X ∩I e b ∈ Y ∩ I, e;

• se X 6⊂ {x1 , x2 } e Y ⊂ {x1 , x2 }, então f [I] = [f (a), f (x1 )), para a ∈ X ∩ I.

Curiosidade. De modo geral, uma função f tal que f [A] é aberto qualquer que seja o aberto
A contido em seu domínio é chamada de função aberta.

7.26 Exercício 26
Seja p : R → R um polinômio de grau par, cujo coeciente líder é positivo. Prove que p assume
um valor mínimo em R, isto é, existe x0 ∈ R tal que p(x0 ) 6 p(x) para todo x ∈ R. Se
p(x0 ) < 0, mostre que p possui pelo menos duas raízes reais. Enuncie e demonstre resultados
análogos quando o coeciente líder de p é negativo.
252 CAPÍTULO 7. FUNÇÕES CONTÍNUAS

Solução. Os resultados que irão nos ajudar neste exercício são: o Exercício 13 do Capítulo
VI  segundo o qual lim p(x) = lim p(x) = +∞ (página 217) , o Teorema de Weierstrass
x→+∞ x→−∞
(Corolário do Teorema 14) e o Teorema do Valor Intermediário (Teorema 12).

Primeira Parte. Fixado um número a ∈ R, pelo referido Exercício 13 do Capítulo VI,


seja A > 0 tal que p(x) > p(a) ∀x ∈ (−∞, −A) ∪ (A, +∞). Pelo Teorema de Weierstrass, seja
x0 ∈ [−A, A] tal que p(x0 ) 6 p(x) para todo x ∈ [−A, A]. Deste modo temos p(x0 ) 6 p(a) se
a ∈ [−A, A], ou seja, se supormos A > |a|, o que de fato podemos fazer. Assim, se x ∈ [−A, A],
então p(x0 ) 6 p(x) e, se x ∈ R −[−A, A], então p(x) > p(a) > p(x0 ). Logo p(x0 ) 6 p(x)
qualquer que seja x ∈ R, como queríamos.
Se p(x0 ) < 0, como p(x) → +∞ se x → +∞ ou se x → −∞, sejam x1 > x0 e x2 < x0 tais
que p(x1 ) > 0 e p(x2 ) > 0. Assim, pelo Teorema do Valor Intermediário, existem c ∈ (x2 , x0 ) e
d ∈ (x0 , x1 ) tais que p(c) = p(d) = 0, com c e d claramente distintos.

Segunda Parte. Se o coeciente líder de p é negativo (e o grau de p é par), então p(x) →


−∞ se x → +∞ ou se x → −∞ (pela observação feita após o Exercício 13 do Capítulo VI,
página 218). Armamos então que existe x0 ∈ R tal que p(x) 6 p(x0 ) para todo x∈R e que,
se p(x0 ) > 0 então p possui pelo menos duas raízes reais.
A demonstração disto é análoga ao caso anterior. Fixado a ∈ R, seja A > |a| tal que
p(x) < p(a) para todo x ∈ (−∞, −A) ∪ (A, +∞). Seja x0 ∈ [−A, A] tal que p(x0 ) > p(x) para
todo x ∈ [−A, A]. Se x ∈ R −[−A, A], então p(x) < p(a) 6 p(x0 ), porque a ∈ [−A, A]. Logo
p(x) 6 p(x0 ) para todo x ∈ R. Se p(x0 ) > 0, para x1 > x0 e x2 < x0 tais que p(x1 ) e p(x2 ) são
ambos negativos (eles existem), existem c ∈ (x2 , x0 ) e d ∈ (x0 , x1 ) tais que p(c) = p(d) = 0. 

Observação. A positivo considerado acima é maior


Na verdade não precisamos supor que o
do que ou igual a |a|, isto é consequência das propriedades que A possui. De modo mais preciso,
se A > 0 é tal que p(x) > p(a) ∀x ∈ R −[−A, A], então a só pode pertencer ao intervalo [−A, A],
pois se fosse a ∈
/ [−A, A], então teríamos p(a) > p(a), o que é absurdo. E, de modo análogo, se
A > 0 é tal que p(x) < p(a) ∀x ∈ R −[−A, A], então a ∈ [−A, A].
(Voltar para a Solução do Exercício 28: página 253.)

7.27 Exercício 27
Se toda função contínua, denida num certo conjunto X, é limitada, então X é compacto.

(Compare este exercício com o Exercício 58 do Capítulo V, página 198)

Solução 1. Vamos mostrar a contrapositiva deste exercício, isto é, vamos encontrar uma
função contínua f :X→R não limitada caso X não seja compacto.
Se X não é compacto, então, pelo Teorema 11 do Capítulo V, X não é limitado ou X não é
fechado. Se X não é limitado, então a função f : X → R denida por f (x) = x ∀x ∈ X não é
limitada (e é contínua). Se X não é fechado, seja a ∈ X − X , de modo que a função f : X → R
1
denida por f (x) = ∀x ∈ X é contínua e não limitada (é contínua porque a∈
/X e não
x−a
é limitada porque, tomando uma sequência de pontos (xn )n∈N em X que converge para a, a
sequência (f (xn ))n∈N não é limitada). Segue a tese. 
7.28. EXERCÍCIO 28 253

Solução 2 (demonstração em sua forma direta). Vamos mostrar que X é limitado e


fechado. É limitado porque a associação x 7→ x, para todo x ∈ X , é contínua, portanto, por
hipótese, existe L ∈ R tal que |x| < L para todo x ∈ X . Para a outra propriedade de X , vamos
1
mostrar que R −X é aberto. Dado a ∈ R −X , a associação x 7→ , para todo x ∈ X ,
x−a
1
é contínua, portanto existe M > 0 tal que < M para todo x ∈ X , ou seja, tal que
|x − a|
1 1
|x − a| > para todo x ∈ X . Assim, se x ∈ R é tal que |x − a| < , então x ∈
/ X . Isto
M M
1
signica que, para ε = , o intervalo aberto (a − ε, a + ε) inteiro está contido em R −X . Logo,
M
R −X é aberto, como queríamos. 

Solução 3. Também podemos chegar ao resultado deste exercício através do item 4 do Teo-
rema 11 do Capítulo V. Seja (xn )n∈N uma sequência de pontos em X . Esta sequência é limitada
porque a inclusão (a associação X 3 x 7→ x ∈ R) é contínua. Seja (yn )n∈N uma subsequência
convergente de (xn )n∈N ( Apêndice ao Ÿ4 [do Capítulo IV], página 125 do livro) e seja a o nú-
mero real para o qual ela converge. Só falta agora concluirmos que a ∈ X . Se a não pertencesse
1
a X , então a função f : X → R denida por f (x) = seria ilimitada porque a sequência
x−a
(f (yn ))n∈N seria ilimitada, muito embora f seja uma função contínua. Como nenhuma função
contínua denida em X pode ser ilimitada, só pode ser a ∈ X , como queríamos. 

7.28 Exercício 28
Seja f :R→R contínua. Se lim f (x) = lim f (x) = +∞, então existe um ponto x0 ∈ R no
x→+∞ x→−∞
qual f assume seu valor mínimo.

Solução. Essas hipóteses foram justamente os fatos acerca de polinômios com grau par e
coeciente líder positivo usados na Solução do Exercício 26 (página 252). Portanto, a solução
deste Exercício 28 é exatamente igual à do 26. Pela referida solução também podemos concluir
que, se f : R → R é contínua e se lim f (x) = lim f (x) = −∞, então existe um ponto
x→+∞ x→−∞
x0 ∈ R no qual f assume seu valor máximo. 

7.29 Exercício 29
x
Mostre que f : (−1, +1) → R, denida por f (x) = , é um homeomorsmo entre o
1 − |x|
intervalo (−1, +1) e a reta.

Solução 1. Pelo Teorema 13, basta mostrar que f é contínua, injetiva e sobrejetiva.

Continuidade. Ela é contínua pelo Teorema 5, uma vez que ambas as funções denidas
por x e por 1 − |x| são contínuas no intervalo (−1, 1) e 1 − |x| =
6 0 para todo x ∈ (−1, 1).
254 CAPÍTULO 7. FUNÇÕES CONTÍNUAS

Sobrejetividade. Conforme x tende a 1, o limite de f (x) tende a +∞ (item 7 da página


210, o qual se refere ao Teorema 14 do Capítulo IV). De forma análoga, f (x) → −∞ conforme
x tende a −1. Portanto, como f [(−1, 1)] é um intervalo (Corolário 2 do Teorema 12, uma vez
que f é contínua), segue que a imagem de f é toda a reta, ou seja, f é sobrejetiva.

Injetividade. Dados x, y ∈ (−1, 1), temos:

x y
= ⇒ x − x|y| = y − y|x| ⇒ x − y = x|y| − y|x|.
1 − |x| 1 − |y|

Assim, se x, y > 0 ou se x, y 6 0, então x|y| = y|x| e, portanto, x − y = 0, donde x = y . Por


outro lado, veja que f (z) > 0, se z > 0, e f (z) < 0, se z < 0, pois, dado z ∈ (−1, 1),

z z
z>0⇒ > 0, e z<0⇒ < 0,
1 − |z| 1 − |z|

1
uma vez que z ∈ (−1, 1) ⇒ |z| < 1 ⇒ 1 − |z| > 0 ⇒ > 0. Além do que, f (0) = 0.
1 − |z|
Deste modo, se f (x) = f (y), então não existe a possibilidade de ser x > 0 e y 6 0, nem x < 0
e y > 0. Com essas considerações, podemos concluir que, de fato, f (x) = f (y) ⇒ x = y , como
queríamos. 

Solução 2 (para a bijetividade de f ). Um outro modo de demonstrar que f é bijetiva é


através dos resultados da página 22 do livro (uma função é injetiva se, e somente se, possui uma
inversa à esquerda, e é sobrejetiva se, e somente se, possui uma inversa à direita), encontrando
x
a inversa de f. Uma tal função é a g : R → (−1, 1) denida por g(x) = para todo
1 + |x|
x ∈ R (esta g pode ser encontrada resolvendo-se a equação y = f (x), equação em x, para um
dado y ∈ R). 

7.30 Exercício 30
Classique os intervalos da reta quanto a homeomorsmos, isto é, faça uma lista de tipos de
modo que dois intervalos são homeomorfos se, e somente se, têm o mesmo tipo. (Por exemplo,
dois intervalos abertos quaisquer, limitados ou não, são sempre homeomorfos. Já um intervalo
fechado, limitado, só pode ser homeomorfo a outro que seja também limitado e fechado).

Solução.
7.30. EXERCÍCIO 30 255

Tipo 1:

Tipo 2:

Tipo 3:

Vamos classicar os seguintes tipos de intervalos: o tipo 1, dos intervalos abertos; o tipo 2,
dos intervalos limitados que possuem apenas um de seus extremos ou das semirretas fechadas;
e o tipo 3, dos intervalos fechados e limitados. O exercício já deu uma boa parte da solução:
são homeomorfos dois intervalos quaisquer do tipo 1 ou dois intervalos quaisquer do tipo 3.
Para completar, armamos também que são homeomorfos dois intervalos quaisquer do tipo 2
e que intervalos de tipos diferentes (no sentido que estamos empregando aqui) não podem ser
homeomorfos (veja a tabela acima).
No exercício anterior já vimos que o intervalo (−1, 1) e a reta são homeomorfos. A par-
tir da função desse exercício, através de translações e esticamentos, os quais também são
homeomorsmos, obtemos um homeomorsmo entre os intervalos (a, b) e (c, +∞), a saber, a
função
x−a
b−a x−a
x 7→ +c= +c ∀x ∈ (a, b).
1 − x−a
b−a
b − a − |x − a|
Fazendo x ∈ [a, b), temos o homeomorsmo entre um intervalo fechado à esquerda e uma
semirreta fechada à esquerda.
O homeomorsmo entre um intervalo qualquer (a, b) e R não é obtido de forma muito
diferente, mas também podemos partir da função 
como a do
 parágrafo anterior considerando
a+b
uma extensão do respectivo homeomorsmo entre ,b e (0, +∞), a saber, a associação
2
x − a+b
x 7→ b−a
2 a+b ∀x ∈ (a, b).
2
− x − 2
Segmentos de retas e semirretas resolvem os demais casos. Dados os intervalos (a, b) e (c, d)
consideramos portanto a associação

d−c
x 7→ (x − a) + c,
b−a
256 CAPÍTULO 7. FUNÇÕES CONTÍNUAS

a qual: para todo x ∈ (a, b), é um homeomorsmo entre (a, b) e (c, d); para todo x ∈ [a, b), é
um homeomorsmo entre [a, b) e [c, d); para todo x ∈ (a, b], é um homeomorsmo entre (a, b] e
(c, d]; para todo x ∈ [a, b], é um homeomorsmo entre [a, b] e [c, d]; para todo x ∈ (a, +∞), é
um homeomorsmo entre (a, +∞) e (c, +∞), e; para todo x ∈ [a, +∞), é um homeomorsmo
entre [a, +∞) e [b, +∞). E também a associação

c−d
x 7→ (x − a) + d,
b−a

x ∈ [a, b) nos dá o homeomorsmo entre [a, b) e (c, d] e; para todo x ∈ [a, +∞),
a qual: para todo
o homeomorsmo entre [a, +∞) e (−∞, d].
O fato de a composta de homeomorsmos ser um homeomorsmo nos ajuda a concluir a
primeira parte dessa solução.
Para concluirmos este exercício, precisamos mostrar que intervalos de tipos diferentes não
podem ser homeomorfos. Pelo Teorema 14, um intervalo do tipo 3 não pode ser homeomorfo
a um intervalo do tipo 1 nem a um intervalo do tipo 2, porque, se fosse, então algum intervalo
desses dois últimos tipos seria compacto. Se um intervalo do tipo 1 fosse homeomorfo a um
intervalo do tipo 2, então existiriam intervalos[a, b) e (c, d) homeomorfos. Vamos supor então
que exista um homeomorsmo f : [a, b) → (c, d). Em particular, f é contínua e injetiva e,
portanto, monótona pelo Teorema 13. De f (a) ∈ (c, d), sejam os números reais y1 e y2 tais que
c < y1 < f (a) < y2 < d. Por f ser sobrejetiva, existem x1 , x2 ∈ [a, b) tais que f (x1 ) = y1 e
f (x2 ) = y2 . Como f é monótona e injetiva, um dos dois, x1 ou x2 é estritamente menor do que
a, o que é um absurdo. Logo um intervalo do tipo 1 não pode ser homeomorfo a um intervalo
do tipo 2, como queríamos. Segue a tese. 

7.31 Exercício 31
Se f, g : X → R são uniformemente contínuas então f + g, f ∧ g e f ∨g também o são. (Veja o
Exercício 3 acima.) Por outro lado, o produto de duas funções uniformemente contínuas pode
não ter essa propriedade, a menos que as funções sejam limitadas. A composta de duas funções
uniformemente contínuas é uniformemente contínua.

Solução.

A soma f +g . Lembrando da Desigualdade Triangular (item (i) do Teorema 2 do Capítulo


III), para x, y ∈ X temos

|(f + g)(x) − (f + g)(y)| = |f (x) − f (y) + g(x) − g(y)| 6 |f (x) − f (y)| + |g(x) − g(y)| < 2 ε,

se |f (x) − f (y)| < ε e |g(x) − g(y)| < ε. Assim, dado ε > 0, o δ > 0 escolhido que mostra que
|(f + g)(x) − (f + g)(y)| < ε se x, y ∈ X e |x − y| < δ é o tal que:
 h
 ε εi
x, y ∈ X e |x − y| < δ ⇒ |f (x) − f (y)| < e |g(x) − g(y)| < .
2 2

Este δ existe, por exemplo, fazendo δ = min{δ1 , δ2 }, onde δ1 e δ2 são as respectivas testemunhas
de que |f (x) − f (y)| < ε /2 e |g(x) − g(y)| < ε /2.
7.31. EXERCÍCIO 31 257

As funções f ∧g e f ∨ g. Dado ε > 0, seja δ>0 tal que

 
x, y ∈ X e |x − y| < δ ⇒ [|f (x) − f (y)| < ε e |g(x) − g(y)| < ε] .

Armamos que este δ também é tal que

 
x, y ∈ X e |x − y| < δ ⇒ |(f ∧ g)(x) − (f ∧ g)(y)| < ε

e
 
x, y ∈ X e |x − y| < δ ⇒ |(f ∨ g)(x) − (f ∨ g)(y)| < ε .
Vamos justicar isto. Fixados x, y ∈ X tais que |x − y| < δ , podemos ter (f ∧ g)(x) = f (x)
ou (f ∧ g)(x) = g(x) e (f ∧ g)(y) = f (y) ou (f ∧ g)(y) = f (y). Se (f ∧ g)(x) = f (x) e
(f ∧g)(y) = f (y) ou se (f ∧g)(x) = g(x) e (f ∧g)(y) = g(y), então a tese é imediata. Suponhamos
então (f ∧ g)(x) = f (x) e (f ∧ g)(y) = g(y) (o caso (f ∧ g)(x) = g(x) e (f ∧ g)(y) = g(y) é
inteiramente análogo). Queremos mostrar que |f (x) − g(y)| < ε. Se, caso contrário, fosse
|f (x) − g(y)| > ε, então g(y) > f (x) + ε ou g(y) 6 f (x) − ε. Lembrando que g(y) 6 f (y) por
hipótese, se fosse g(y) > f (x) + ε, então f (y) também seria maior do que ou igual a f (x) + ε
e, portanto, |f (x) − f (y)| > ε, contrariando o fato de que |f (x) − f (y)| < ε. De igual modo, se
fosse g(y) 6 f (x)−ε, então f (x) > g(y)+ε e, junto com g(x) > f (x), seguiria |g(x)−g(y)| > ε.
Logo, só pode ser |f (x)−g(y)| < ε, como queríamos. O raciocínio para a função f ∨g é análogo.

f (y)

g(y) − ε g(y) g(y) + ε


f (x) − ε f (x) f (x) + ε

g(y) − ε g(y) g(y) + ε


f (x) − ε f (x) f (x) + ε

g(x)

O produto. No Exemplo 23, já vimos que a função x 7→ x2 para todo x real não é
uniformemente contínua; esta associação é o produto da identidade por ela mesma, e a função
identidade é uniformemente contínua. Portanto o produto de duas funções uniformemente
contínuas pode não ser uniformemente contínuo.

Nível 1. Vamos supor as funções f e g limitadas. Queremos mostrar que f ·g é unifor-


memente contínua. Assim, xado ε > 0, queremos encontrar δ>0 tal que

x, y ∈ X, |x − y| < δ ⇒ |f (x)g(x) − f (y)g(y)| < ε .

No elevador. Por hipótese, conseguimos chegar às desigualdades

|f (x) − f (y)| < ε e |g(x) − g(y)| < ε .

Por hipótese também temos |f (x)| < A e |g(x)| < B para um certo A>0 e um certo B > 0,
qualquer que seja o x ∈ X. E precisamos fazer as letras f e g aparecerem juntas numa mesma
258 CAPÍTULO 7. FUNÇÕES CONTÍNUAS

desigualdade. Podemos, por exemplo, multiplicar a primeira desigualdade acima por |g(y)| < B
e, a segunda por |f (x)| < A, obtendo

|g(y)f (x) − g(y)f (y)| < B ε e |f (x)g(x) − f (x)g(y)| < A ε .

Agora, veja que a mesma parcela f (x)g(y) aparece em ambas as desigualdades acima, o que
nos sugere utilizar a Desigualdade Triangular (item (i) do Teorema 2, Capítulo III):

|f (x)g(x) − f (y)g(y)| 6 |f (x)g(x) − f (x)g(y)| + |f (x)g(y) − f (y)g(y)| < A ε +B ε .

Nível 2. Pela hipótese da limitação de f e de g , sejam A e B positivos tais que |f (x)| < A
e |g(x)| < B para todo x ∈ X. Da uniformidade contínua de f e de g , sejam δ1 e δ2 números
reais positivos tais que

ε
x, y ∈ X, |x − y| < δ1 ⇒ |f (x) − f (y)| <
2B
e
ε
x, y ∈ X, |x − y| < δ2 ⇒ |g(x) − g(y)| < .
2A
Tomando δ = min{δ1 , δ2 }, se x, y ∈ X e |x − y| < δ , então temos, simultaneamente

ε ε
|f (x) − f (y)| < e |g(x) − g(y)| < .
2B 2A
Assim, pela Desigualdade Triangular (item (i) do Teorema 2 do Capítulo III), se x e y são
elementos de X tais que |x − y| < δ , então:

|f (x)g(x) − f (y)g(y)| = |f (x)g(x) − f (x)g(y) + f (x)g(y) − f (y)g(y)|


6 |f (x)g(x) − f (x)g(y)| + |f (x)g(y) − f (y)g(y)|
= |f (x)| · |g(x) − g(y)| + |g(y)| · |f (x) − f (y)|
ε ε
< A· +B·
2A 2B
ε ε
= +
2 2
= ε,

como queríamos.

A composta. Dados os conjuntos não vazios X, Y de números reais, sejam as funções


uniformemente contínuas f :X →R e g : Y → R, com f [X] ⊂ Y . Queremos mostrar que a
função g ◦ f : X → R é uniformemente contínua.
Dado ε > 0, seja δ > 0 tal que

x, y ∈ Y, |x − y| < δ ⇒ |g(x) − g(y)| < ε .

Para este δ, seja η>0 tal que

x, y ∈ X, |x − y| < η ⇒ |f (x) − f (y)| < δ.

Portanto, se x, y ∈ X e |x − y| < η , como f (x), f (y) ∈ Y e |f (x) − f (y)| < δ , segue:

x, y ∈ X, |x − y| < η ⇒ |g(f (x)) − g(f (y))| < ε .

Logo g◦f é uniformemente contínua, como queríamos. 


7.32. EXERCÍCIO 32 259

Continuidade uniforme de f ∧g e de f ∨g (demonstração em sua forma direta).


Dadas as funções f, g : X → R uniformemente contínuas e dado ε > 0, seja δ>0 tal que

 
x, y ∈ X e |x − y| < δ ⇒ [|f (x) − f (y)| < ε e |g(x) − g(y)| < ε] .

Sejam x e y elementos de X tais que |x − y| < δ e consideremos todos os casos possíveis (as
duas possibilidades de valores para o módulo de um número e as duas possibilidades de valores
que cada uma das funções f ∧g e f ∨g pode assumir num dado ponto).
Quanto à função f ∧ g, temos:

• (f ∧ g)(x) − (f ∧ g)(y) = (f ∧ g)(x) − f (y) 6 f (x) − f (y) 6 |f (x) − f (y)| < ε,


se (f ∧ g)(y) = f (y);

• (f ∧ g)(x) − (f ∧ g)(y) = (f ∧ g)(x) − g(y) 6 g(x) − g(y) 6 |g(x) − g(y)| < ε,


se (f ∧ g)(y) = g(y);

• (f ∧ g)(y) − (f ∧ g)(x) = (f ∧ g)(y) − f (x) 6 f (y) − f (x) 6 |f (x) − f (y)| < ε,


se (f ∧ g)(x) = f (x), e;

• (f ∧ g)(y) − (f ∧ g)(x) = (f ∧ g)(y) − g(x) 6 g(y) − g(x) 6 |g(x) − g(y)| < ε,


se (f ∧ g)(x) = g(x).

Portanto,

|(f ∧ g)(x) − (f ∧ g)(y)| < ε .

E quanto à função f ∨ g, temos:

• (f ∨ g)(x) − (f ∨ g)(y) = f (x) − (f ∨ g)(y) 6 f (x) − f (y) 6 |f (x) − f (y)| < ε,


se (f ∨ g)(x) = f (x);

• (f ∨ g)(x) − (f ∨ g)(y) = g(x) − (f ∨ g)(y) 6 g(x) − g(y) 6 |g(x) − g(y)| < ε,


se (f ∨ g)(x) = g(x);

• (f ∨ g)(y) − (f ∨ g)(x) = f (y) − (f ∨ g)(x) 6 f (y) − f (x) 6 |f (x) − f (y)| < ε,


se (f ∨ g)(y) = f (y), e;

• (f ∨ g)(y) − (f ∨ g)(x) = g(y) − (f ∨ g)(x) 6 g(y) − g(x) 6 |g(x) − g(y)| < ε,


se (f ∨ g)(y) = g(y).

Portanto,

|(f ∨ g)(x) − (f ∨ g)(y)| < ε .

Logo, f ∧g e f ∨g são uniformemente contínuas.

7.32 Exercício 32
Seja g : R → (−1, +1) o inverso do homeomorsmo f denido no Exercício 29. Mostre que g é
uniformemente contínuo mas g −1 = f não o é.
260 CAPÍTULO 7. FUNÇÕES CONTÍNUAS

x
Solução. Esse inverso é dado por g(x) = para todo x real. Para números reais x e y,
1 + |x|
dado ε > 0, queremos δ>0 tal que

|g(x) − g(y)| < ε se |x − y| < δ.


Temos

x y |x − y + x|y| − y|x||
|g(x) − g(y)| =
− = 6 |x − y + x|y| − y|x||,
1 + |x| 1 + |y| (1 + |x|)(1 + |y|)
pois (1 + |x|)(1 + |y|) > 1. Assim

|g(x) − g(y)| 6 |x − y| + 2|xy|,


pela Desigualdade Triangular (Teorema 2 do Capítulo III, item (i)), e

|g(x) − g(y)| 6 |x − y|
se xy = 0, x, y ∈ (0, +∞) ou se x, y ∈ (−∞, 0), pois, nestes casos, x|y| = y|x|.
se
xy = 0, se x e y são ambos negativos ou se são ambos positivos, então |x − y| < ε
Assim, se
implica |g(x) − g(y)| < ε. Se um deles é positivo e se o outro é negativo, suponhamos x > 0
e y < 0. Se, além disso, tivermos |x − y| < δ para um certo δ positivo, então podemos
2
armar que −δ < y < 0 e 0 < x < δ , donde concluímos que |xy| < δ . Deste modo, teremos
|g(x) − g(y)| < δ + 2δ 2 .
2
Com base nestas considerações, dado ε > 0, seja δ0 > 0 tal que δ0 + 2δ0 < ε (ele existe) e
tomemos δ = min{ε, δ0 }. Para este δ , vamos supor |x − y| < δ . Se xy = 0 ou se x e y são ambos
negativos ou ambos positivos, então |g(x) − g(y)| 6 |x − y| < δ 6 ε. Se um deles é positivo e
2 2
o outro é negativo, então |g(x) − g(y)| < δ + 2δ 6 δ0 + 2δ0 < ε. De qualquer forma, temos o
resultado desejado:
|x − y| < δ ⇒ |g(x) − g(y)| < ε .
A f não é uniformemente contínua por causa do Corolário do Teorema 16. O número 1 é
ponto de acumulação do intervalo (−1, 1), domínio de f, enquanto o limite de f (x) conforme x
se aproxima de 1 não existe (lembrando das denições das páginas 108 e 109: o limite de uma
sequência não existe quando não existe um número real para o qual ela convirja). 

7.33 Exercício 33
A função f : R → R, denida por f (x) = sen x, é uniformemente contínua, mas g(x) = sen(x2 )
dene uma g : R → R contínua que não é uniformemente contínua.

Solução.

sen y B
B

sen y − sen x |y0 − x0 |


|y0 |
sen x A
C C A
|x0 |
AB > AB > BC

O
7.33. EXERCÍCIO 33 261

A função f é uniformemente contínua porque ela é lipschitziana, com constante c = 1, isto é,


vale | sen x−sen y| 6 |x−y| quaisquer que sejam os números reais x e y . Podemos ver isto através
de um argumento geométrico (para outros argumentos, ver os parágrafos após esta solução),
conforme sugere a gura acima: Dados os números x e y, sejam x0 e y0 números no intervalo
[−π/2, π/2] tais que sen x0 = sen x e sen y0 = sen y .
Sejam os pontos A = (cos x0 , sen x0 ) e
B = (cos y0 , sen y0 ). Na circunferência de raio 1 e com centro na origem, sendo O = (1, 0), estes
pontos A e B determinam arcos OA e OB de medidas |x0 | e |y0 |, respectivamente. Seja também
C = (cos y0 , sen x0 ). Consequentemente, o segmento BC mede | sen x0 − sen y0 | e o triângulo
ABC é retângulo em C , quando C 6= A. Por um lado, o menor arco AB é maior do que ou igual
ao segmento AB que, por sua vez, é maior do que ou é igual ao segmento BC , lembrando que
a hipotenusa é o maior lado de um triângulo retângulo. Por outro, este menor arco AB mede
|x0 −y0 |, número este menor do que ou igual a |x−y|. Logo: |x−y| > |x0 −y0 | > | sen x−sen y|,
a igualdade valendo, por exemplo, quando x e y são iguais.
Com relação à continuidade uniforme da função g , a grande observação que nos dá uma
luz sobre qual caminho seguir e que nos poupa de fazer contas demais é a seguinte: xado
y ∈ [−1, 1], dois pontos distintos do conjunto g −1 [{y}] podem ser tomados tão próximos quanto
se queira. Isto pode ser conjecturado após vermos seu gráco, por exemplo.

Sejam os conjuntos

π √ π √
r  r 
X= · 1 + 4k ∈ R; k ∈ N e Y = · 3 + 4k ∈ R; k ∈ N .
2 2

Para eles temos sen(x2 ) = 1 e sen(y 2 ) = −1 ∀x ∈ X e ∀y ∈ Y , de modo que |g(x) − g(y)| = 2


∀x ∈ X e ∀y ∈ Y . Além disto, armamos que, dado δ > 0, existem x ∈ X e y ∈ Y tais que
|x − y| < δ . Isto porque:
√ √
√ √ √ √ 3 + 4k + 1 + 4k
lim 3 + 4k − 1 + 4k = lim ( 3 + 4k − 1 + 4k) · √ √
k→+∞ k→+∞ 3 + 4k + 1 + 4k
2
= lim √ √
k→+∞ 3 + 4k + 1 + 4k
= 0.

Logo, g não é uniformemente contínua. Ela é contínua simplesmente por ser uma composta de
funções contínuas (Teorema 6). 

Sendo um pouco mais analítico... Também podemos tentar demonstrar que a f do exercí-
sen x − sen y
cio acima é lipschitziana através da derivada da função seno: dado y ∈ R, como lim =
x→y x−y
sen x − sen y
cos y , temos lim = | cos y| 6 1. Porém, a rigor, ainda não podemos fazer isso,
x→y x−y
262 CAPÍTULO 7. FUNÇÕES CONTÍNUAS

porque o assunto da derivada ainda não foi abordado no livro. Mas nada nos impede de
0
(re)visitar a demonstração de que sen x = cos x.
Da igualdade
x−y x+y
sen x − sen y = 2 sen cos ,
2 2
válida para todos x e y reais, obtemos

x−y x+y x−y


| sen x − sen y| = 2| sen | · | cos | 6 2| sen |,
2 2 2
uma vez que | cos z| 6 1 para todo z ∈ R. Lembrando que queremos mostrar que f é uniforme-
mente contínua, dado ε > 0, seja δ > 0 tal que |z| < δ ⇒ | sen z| < ε /2, o qual existe porque
lim sen z = 0. Para ele, temos:
z→0

x − y x − y ε x − y
|x − y| < 2δ ⇒ < δ ⇒ sen < ⇒ 2 sen < ε ⇒ | sen x − sen y| < ε .
2 2 2 2

Logo, f é uniformemente contínua.

Observação. Isolando os fatos acerca da função seno usados para concluir sua continuidade
uniforme, com a mesma demonstração do parágrafo acima podemos concluir que é uniforme-
mente contínua qualquer função h:R→R com
as seguintes
  propriedades: existem números
x − y
reais a e b (b não nulo) tais que |h(x) − h(y)| 6 a · h para todo x∈R e todo y ∈ R,
b
e lim h(x) = 0.
x→0

Voltando a ser mais geométrico. A partir da desigualdade

x−y
| sen x − sen y| 6 2| sen |,
2
para todo x ∈ R e todo y ∈ R, vista acima, podemos concluir que a função seno é uniformemente
contínua usando o fato de que | sen z| 6 |z| para todo z tal que |z| < π/2, cuja justicativa pode
ser feita de modo semelhante ao que zemos na solução mais acima: dado z tal que |z| < π/2,
para A = (1, 0), B = (cos z, sen z) e C = (cos z, 0), temos |z| > AB > BC = | sen z|. Deste
nπ εo
modo, dado ε > 0, para δ = min , , temos:
2 2

x − y x − y π x − y x − y x − y ε
|x − y| < 2δ ⇒ <δ⇒ < ⇒ sen 6 ⇒ sen <
2 2 2 2 2 2 2

x − y
⇒ 2 sen < ε ⇒ | sen x − sen y| < ε .
2

Logo, a função seno é uniformemente contínua.

7.34 Exercício 34
Seja X⊂R um conjunto não-fechado. Mostre que existe uma função contínua f :X→R que
não se estende continuamente a X.
7.35. EXERCÍCIO 35 263

Solução. Como X não é fechado, o conjunto X −X não é vazio. Sejam então a ∈ X −X


1
e f :X →R denida por f (x) = para todo x ∈ X. Ela é contínua e não se estende
x−a
continuamente a X, pois, xada uma função ϕ:X →R tal que ϕ|X = f , se (xn )n∈N é uma
sequência de pontos em X convergindo para a, então

1
lim |ϕ(xn )| = lim = +∞,
n→+∞ n→+∞ |xn − a|

portanto a sequência (ϕ(xn ))n∈N não converge para ϕ(a), logo ϕ não é contínua em a (Teorema
4). 

7.35 Exercício 35
Um polinômio p:R→R é uma função uniformemente contínua se, e somente se, tem grau
6 1.

Solução. O fato de que um polinômio de grau menor do que ou igual a 1 é uniformemente


contínuo já foi visto no texto (no Exemplo 22). Reciprocamente, dado o polinômio p, vamos
supor que seu grau é maior do que 1. Queremos mostrar que ele não é uniformemente contínuo.
2
A ideia é motivada pelo que foi feito no Exemplo 23 com o polinômio x .
A partir do polinômio p, para um xado δ > 0, denimos um novo polinômio q por

 
δ
q(x) = p x + − p(x).
2

Queremos x∈R tal que |q(x)| > 1. Um tal x existe porque

lim |q(x)| = +∞,


x→+∞

pelo Exercício 13 do Capítulo VI (página 217), uma vez que o grau de q é pelo menos igual a 1;
ele é uma unidade menor do que o grau de p que, por sua vez, é pelo menos 2 por hipótese. 

Observação. Essa solução também mostra que se o grau de um polinômio p é maior do que
1, sempre podemos encontrar números reais x e y próximos o suciente tais que |p(x) − p(y)| é
tão grande quanto se queria.

7.36 Exercício 36
[
f :X→R é contínua se, e somente se, para cada ε > 0, existe uma cobertura X⊂ Ix com
x∈X
Ix = (x − δx , x + δx ), tais que y, z ∈ X ∩ Ix ⇒ |f (y) − f (z)| < ε. A função f é uniformemente
contínua se, e somente se, para cada ε > 0, os intervalos Ix puderem ser escolhidos com o
mesmo comprimento.

Solução.
264 CAPÍTULO 7. FUNÇÕES CONTÍNUAS

Primeira Parte. Se f é contínua, xado ε > 0, para cada x ∈ X , seja Ix = (x−δx , x+δx ),
com δx > 0, tal que
ε
y ∈ X ∩ Ix ⇒ |f (y) − f (x)| < ,
2
[
δx este que existe porque f é contínua em x. Claramente X ⊂ Ix . Além disto, pela
x∈X
Desigualdade Triangular (Teorema 2 do Capítulo III, item (i)), dado x ∈ X, temos:

ε ε
y, z ∈ X ∩ Ix ⇒ |f (y) − f (z)| 6 |f (y) − f (x)| + |f (z) − f (x)| < + = ε.
2 2

Portanto vale a ida da primeira parte deste exercício.


[
Reciprocamente, dado ε > 0, seja X ⊂ Ix com Ix = (x − δx , x + δx ), tais que y, z ∈
x∈X
X ∩ Ix ⇒ |f (y) − f (z)| < ε. Dado x∈ X , temos x ∈ Ix ∩ X , de modo que, por hipótese,

y ∈ X ∩ Ix ⇒ |f (y) − f (x)| < ε .

Como isso pode ser feito para cada ε, segue que f é contínua em x. Portanto vale a volta da
primeira parte deste exercício.

Segunda
 Parte.
 Se f é uniformemente contínua, dado ε > 0, para cada x ∈ X , denimos
δ δ
Ix = x − ,x + , onde δ>0 é tal que:
2 2

y, z ∈ X, |y − z| < δ ⇒ |f (y) − f (z)| < ε .

Deste modo:

y, z ∈ X ∩ Ix ⇒ y, z ∈ X, |y − z| < δ ⇒ |f (y) − f (z)| < ε,

ou seja, a família (Ix )x∈X é uma cobertura de X com as propriedades desejadas.


[
Reciprocamente, dado ε > 0, seja δ >0 tal que X ⊂ (x − δ, x + δ), onde cada Ix :=
x∈X
(x − δ, x + δ) satisfaz a implicação y, z ∈ X ∩ Ix ⇒ |f (y) − f (z)| < ε. Se y, z ∈ X são tais que
|y − z| < δ , então z ∈ Iy e, portanto, |f (y) − f (z)| < ε, porque

y, z ∈ X ∩ Iy ⇒ |f (y) − f (z)| < ε

e y e z de fato pertencem à X ∩ Iy . Logo f é uniformemente contínua. 


(Voltar para a Solução do Exercício 43: página 270.)

7.37 Exercício 37
A função f (x) = xn é lipschitziana em cada conjunto limitado mas, se n > 1, não é uniforme-
mente contínua num intervalo ilimitado.

(Voltar para a Solução do Exercício 16 do Capítulo X: página 421.)


7.38. EXERCÍCIO 38 265

Solução. Pela Desigualdade Triangular (item (i) do Teorema 2 do Capítulo III) e lembrando
que
y n − xn = (y − x)(y n−1 + xy n−2 + x2 y n−3 + · · · + xn−2 y + xn−1 ),
temos

|y n − xn | 6 |y|n−1 + |x| · |y|n−2 + |x|2 · |y|n−3 + · · · + |x|n−2 · |y| + |x|n−1 · |y − x|.




Se x e y pertencem a um intervalo limitado, então existe um número real positivo M tal


que ambos os números |x| e |y| são menores do que M , de modo que, substituindo na igualdade
acima, obtemos
|y n − xn | 6 n · M n−1 · |y − x|,
donde segue que f é lipschitziana em cada conjunto limitado, com constante c = nM n−1 (a
constante c do Exemplo 23).
Para n > 1, pelo Exercício 35, o conjunto ilimitado no qual f não é uniformemente contínua
é R. Pela solução deste exercício podemos concluir mais do que isto, a saber, que f não é
uniformemente contínua em nenhum intervalo ilimitado, se n > 1. De fato, neste caso, dado
δ > 0, como  n
δ n

lim x + − x = +∞,
x→±∞ 2
 n
δ
n
seja A > 0 tal que |x| > A ⇒ − x > 1. Se I ⊂ R é um intervalo ilimitado, então
x+ 2

existe x ∈ I tal que x + δ/2 ∈ I e |x| > A, testemunha de que a armação feita é válida. 

7.38 Exercício 38

A função f : [0, +∞) → [0, +∞), denida por f (x) = n x, não é lipschitziana num intervalo da
forma [0, a], a > 0, embora seja uniformemente contínua aí. Por outro lado, f é lipschitziana,
1
com constante c = √
n
, no intervalo [a, +∞). Concluir que f é uniformemente conínua
n · an−1
em [0, +∞).

Solução. Vamos ver que esses resultados valem para qualquer real positivo a.
Dado a > 0, um raciocínio semelhante ao dos dois primeiros parágrafos da solução anterior
também mostra que f é lipschitziana em [a, +∞). De

√ √ p p p p √
n
x − y = ( n x − n y)( n y n−1 + n xy n−2 + n x2 y n−3 + · · · + n xn−2 y + xn−1 )

obtemos:
p p p p √
n
|x − y| = |f (x) − f (y)| · ( n y n−1 + n xy n−2 + n x2 y n−3 + · · · + n xn−2 y + xn−1 )
√n
> |f (x) − f (y)| · n an−1 ,
p √
n
se x, y > a (pois x, y > a ⇒ n xi y n−i−1 > an−1 , para todo i ∈ [0, n]), donde
1
|f (x) − f (y)| 6 √
n
· |x − y| ∀x, y ∈ [a, +∞),
n an−1
uma vez que a > 0. Logo f é lipschitziana em [a, +∞) qualquer que seja o número real positivo
a.
266 CAPÍTULO 7. FUNÇÕES CONTÍNUAS

Dado a > 0, f é uniformemente contínua no intervalo [0, a] como consequência do Teorema


17, pelo fato de f ser contínua e [0, a] ser compacto. Agora, dado c > 0, armamos que existe
x ∈ (0, a] tal que
|f (x) − f (0)| > c · |x − 0|,
f (x)
isto é, tal que > c. Um tal x existe porque
x

f (x) n
x 1
lim = lim = lim √
n
= +∞.
x→0 x x→0 x x→0 xn−1
Logo f é uniformemente contínua e não é lipschitziana no intervalo [0, a] qualquer que seja o
número positivo a.
Para concluir, xado a > 0, seja ε > 0. Já sabemos que f é uniformemente contínua em
ambos os intervalos [0, a] e [a, +∞) (lembrando que ser lipschitziana implica ser uniformemente
contínua). Assim, sejam os números reais positivos δ1 e δ2 tais que

ε
x, y ∈ [0, a], |x − y| < δ1 ⇒ |f (x) − f (y)| <
2
e
ε
x, y ∈ [a, +∞), |x − y| < δ2 ⇒ |f (x) − f (y)| < .
2
Armamos que o número δ = min{δ1 , δ2 } é tal que

x, y ∈ [0, +∞), |x − y| < δ ⇒ |f (x) − f (y)| < ε .

De fato: se x, y ∈ [0, a] ou se x, y ∈ [a, +∞), então o resultado é válido porque ε /2 < ε e


|x − y| < δ ⇒ |x − y| < δ1 e |x − y| < δ2 , e, se |x − y| < δ com 0 6 x 6 a 6 y , então |x − a| < δ
e |y − a| < δ , de modo que, pela Desigualdade Triangular (item (i) do Teorema 2, Capítulo III)
e por hipótese,

ε ε
|f (x) − f (y)| 6 |f (x) − f (a)| + |f (a) − f (y)| < + = ε,
2 2
como queríamos. 

7.39 Exercício 39
 
∗ 1
Seja N = n + ; n ∈ N . Escreva F = N ∪ N∗ e dena f : F → R pondo f (n) = 2 e
  n
1 1
f n+ = n + . Mostre que os conjuntos N e N∗ são fechados, que f | N e f | N∗ são
n n
uniformemente contínuas, mas f : F → R não é uniformemente contínua.

Solução. A justicativa do porquê N∗ é fechado já foi dada na Solução do Exercício 24


(página 250; por coincidência esse mesmo conjunto foi usado naquela solução). A justicativa
do porquê N é fechado é análoga: para todo x real, o intervalo (x − 1, x + 1) tem no máximo
dois números naturais, o que faz o conjunto N0 ser vazio pelo Teorema 7 do Capítulo 5, donde
N = N ∪ N0 = N.
Dado ε > 0, qualquer δ > 0 serve para mostrar que a restrição f | N é uniformemente
contínua, uma vez que |f (x) − f (y)| = 0, que é menor do que ε quaisquer que sejam os naturais
7.40. EXERCÍCIO 40 267

x e y . A restrição f | N∗ é uniformemente contínua porque é lipschitziana: |f (x) − f (y)| =


|x − y| 6 |x − y| quaisquer que sejam x, y ∈ N∗ .
Finalmente, para mostrar que f não é uniformemente contínua veja que
 
1 = n + − 2 = n − 2 + 1 > 1,
1
f n + − f (n)
n n n
se n > 2. Assim, para ε = 1, dado δ > 0, pela Propriedade Arquimediana, seja n ∈ N maior
do que 2 e tal que 1/n < δ , de modo que
 
n + 1 − n = 1 < δ, enquanto f n + 1 − f (n) > 1.

n n n

Logo, f não é uniformemente contínua, como queríamos. 

7.40 Exercício 40
Dê exemplo de dois abertos A, B e uma função contínua f : A∪B → R tal que f |A e f |B
sejam uniformemente contínuas, mas f não seja.

Solução. A = (1, 2) e B = (2, 3), a função f : A ∪ B → R denida por f (x) = 1 para


Para
todo x∈A f (x) = 2 para todo x ∈ B . Cada restrição f |A e f |B é constante e, portanto,
e
uniformemente contínua. Por outro lado, para ε = 1/2, qualquer que seja o δ > 0, tomando
x ∈ (2 − δ/2, 2) ∩ A e y ∈ (2, 2 + δ/2) ∩ B , temos |f (y) − f (x)| = 1 > ε, ou seja, f não é
uniformemente contínua. Outra forma de vermos que f não é uniformemente contínua é pelo
0
Corolário do Teorema 16, 2 ∈ (A ∪ B) , mas o limite de f (x) quando x tende a 2 não existe. 

7.41 Exercício 41
Toda função contínua monótona limitada f : I → R, denida num intervalo I , é uniformemente
contínua.

Solução. A ideia desta solução é conseguir fazer com que f (x) e f (y) pertençam a um mesmo
intervalo de comprimento menor do que ε, o que nos dará |f (x) − f (y)| < ε.
Como I é um intervalo e f é contínua, f [I] é um intervalo (Corolário 2 do Teorema 12).
Este intervalo é limitado porque f é limitada. Sejam a<b seus extremos (isto é, a = inf f [I]
268 CAPÍTULO 7. FUNÇÕES CONTÍNUAS

e b = sup f [I]; se a = b, então f é constante e, consequentemente, uniformemente contínua).


Fixado ε > 0, sejam os pontos ai = a+i·ε0 , para i ∈ {0, 1, . . . , n−1} e an = b, onde 0 < ε0 < ε /2
e n = min {m ∈ N; a + m · ε0 > b}, de modo que o comprimento de cada intervalo de extremos
ai e ai+1 não é maior do que ε0 . Se n 6 2, então b − a < ε (n = 1 ⇒ b − a 6 ε0 < ε;
n = 2 ⇒ b − a 6 2 ε0 < ε), portanto |f (x) − f (y)| < ε para todo x, y ∈ I , e temos acabado.
Supondo n > 2, para cada i ∈ {1, · · · , n − 1}, como ai ∈ f [I], seja xi ∈ I tal que f (xi ) = ai .
Vamos supor f não decrescente e considerar os intervalos {x ∈ I; x 6 x1 }, [xi , xi+1 ], para
i ∈ {1, . . . , n − 2}, e {x ∈ I; x > xn−1 }. Dos comprimentos dos intervalos limitados desta lista,
seja δ o menor deles (δ é um número real positivo).
Vamos vericar agora que este δ funciona para os nossos propósitos. Para tanto, sejam x e
y pontos de I tais que |x − y| < δ .
Se x 6 x1 , então y 6 x2 , pois y > x2 implicaria |y − x| > δ uma vez que x2 > x1 + δ . Deste
modo, como f é não decrescente, temos f (x) 6 f (x1 ) = a1 e f (y) 6 f (x2 ) = a2 , ou seja, ambos
f (x) e f (y) pertencem ao intervalo [a, a2 ] de comprimento a2 − a 6 2 ε0 < ε (por construção),
o que nos dá |f (x) − f (y)| 6 2 ε0 < ε. As outras possibilidades com relação às posições dos
pontos x e y são análogas:

• x1 6 x 6 x2 ⇒ y 6 x3 f (x), f (y) ∈ [a, a2 ] ou f (x), f (y) ∈ [a1 , a3 ] (se n = 3, o x3


e, então,
pode não fazer sentido, então notamos que x ∈ [x1 , x2 ] ⇒ |f (x) − a| < ε e |f (x) − b| < ε,
uma vez que f (x) ∈ [a1 , a2 ] implica |f (x) − a| = f (x) − a 6 a2 − a = 2 ε0 e |f (x) − b| =
b − f (x) 6 b − a1 6 2 ε0 , portanto |f (x) − f (y)| < ε para todo x ∈ [x1 , x2 ] e todo y ∈ I );

• (para 1 < i < n − 2) xi 6 x 6 xi+1 ⇒ xi−1 6 y 6 xi+2 e, então, f (x), f (y) ∈ [ai−1 , ai+1 ]
ou f (x), f (y) ∈ [ai , ai+2 ];

• xn−2 6 x 6 xn−1 ⇒ y > xn−3 (se n = 3, então caimos no primeiro tópico) e, então,
f (x), f (y) ∈ [an−3 , an−1 ] ou f (x), f (y) ∈ [an−2 , an ];

• x > xn−1 ⇒ y > xn−2 e, então, f (x), f (y) ∈ [an−2 , b].

Em qualquer um destes casos temos |f (x) − f (y)| 6 2 ε0 < ε.


Logo, x, y ∈ I , com |x − y| < δ implicam |f (x) − f (y)| < ε. O caso em que f é não crescente
é análogo. Segue a tese. 

7.42 Exercício 42
Seja f :X →R contínua. Para que f se estenda continuamente a uma função ϕ:X →R é
0
necessário e suciente que exista lim f (x) para todo a ∈ X .
x→a

Solução. Se ϕ : X → R é uma extensão contínua de f, armamos que lim f (x) = ϕ(a),


x→a
0 0
qualquer que seja o ponto a∈X . Fixados a∈X e ε > 0, queremos δ>0 tal que

x ∈ VX (a; δ) ⇒ |f (x) − ϕ(a)| < ε,

lembrando da denição de VW (w; δ) como o conjunto (W − {w}) ∩ (w − δ, w + δ).


Por causa da continuidade de ϕ em a, seja δ > 0 tal que

x ∈ VX (a; δ) ⇒ |ϕ(x) − ϕ(a)| < ε .


7.43. EXERCÍCIO 43 269

Este mesmo δ serve para mostrar o resultado desejado. De fato, se x ∈ VX (a; δ), então x∈
VX (a; δ) e f (x) = ϕ(x), porque X ⊂ X e ϕ|X = f , de modo que

|f (x) − ϕ(a)| = |ϕ(x) − ϕ(a)| < ε,

como queríamos.

Se o limite de f (x) quando x tende a a existe qualquer que seja a ∈ X 0, vamos denir

ϕ(a) = lim f (x) ∀a ∈ X 0 .


x→a

Se a ∈ X0 ∩ X, então ϕ(a) = f (a) pela continuidade de f. Esta observação nos garante não
haver ambiguidades na denição de ϕ ao fazer ϕ(x) = f (x) para todo x ∈ X . Claramente essa
0
função ϕ : X = X ∪ X → R estende f , só faltando mostrar sua continuidade. Assim, dados
ε>0 e a ∈ X, queremos δ>0 tal que

x ∈ X ∩ (a − δ, a + δ) ⇒ |ϕ(x) − ϕ(a)| < ε .

Nossas informações acerca dos valores assumidos por ϕ vem a partir de pontos em X . Para
y ∈ X, inserindo o ponto ϕ(y) = f (y) no lado esquerdo da desigualdade acima envolvendo o ε,
obtemos

|ϕ(x) − ϕ(a)| 6 |ϕ(x) − f (y)| + |f (y) − ϕ(a)|,

pela Desigualdade Triangular (item (i) do Teorema 2 do Capítulo III). Pela construção de ϕ,
este y pode ser escolhido de modo a termos cada uma das parcelas acima menor do que ε /2.
Destas considerações é que vem nossa escolha do δ abaixo.

Como ϕ(a) é o limite de f (y) quando y tende a a  o que é válido mesmo quando / X 0,
a∈
porque f é contínua  seja δ > 0 tal que

ε
y ∈ VX (a; δ) ⇒ |f (y) − ϕ(a)| < .
2

Armamos que este δ é o procurado. De fato, seja x ∈ X ∩(a−δ, a+δ). Como ϕ(x) = lim f (y),
y→x
ε
podemos escolher y ∈ X ∩ (a − δ, a + δ) tal que |f (y) − ϕ(x)| < . Para este y temos
2

|ϕ(x) − ϕ(a)| 6 |ϕ(x) − f (y)| + |f (y) − ϕ(a)| < ε,

ε
pois |f (y) − ϕ(a)| < não importando se y 6= a ou y = a: se y 6= a, então esta desigualdade
2
vale por hipótese, se y = a, então vale trivialmente. Segue a tese. 

7.43 Exercício 43
Seja f : [a, b] → R contínua. Dado ε > 0, existem a = a0 < a1 < · · · < an−1 < an = b tais que,
para cada i = 1, 2, . . . , n, x, y ∈ [ai−1 , ai ] ⇒ |f (x) − f (y)| < ε.

(Voltar para a Solução do Exercício 25 do Capítulo X: página 428.)


270 CAPÍTULO 7. FUNÇÕES CONTÍNUAS

Solução. A ideia aqui é semelhante a uma que já foi vista na Solução do Exercício 36 (prin-
cipalmente na Segunda Parte, página 264).
Como [a, b] é compacto, e f é contínua, pelo Teorema 17, f é uniformemente contínua.
Fixado ε > 0, escolhemos δ > 0 tal que

x, y ∈ [a, b], |x − y| 6 δ ⇒ |f (x) − f (y)| < ε .


Para este δ , denimos ai = a + i · δ para cada i ∈ {0, 1, . . . , n − 1}, com n = min{m ∈
N; a + mδ > b}, e an = b, de modo que
x, y ∈ [ai , ai+1 ] ⇒ x, y ∈ [a, b], |x − y| 6 δ ⇒ |f (x) − f (y)| < ε,
como queríamos. 

7.44 Exercício 44
Uma função contínua ϕ : [a, b] → R chama-se poligonal quando existem a = a0 < a1 < · · · <
an = b tais que ϕ|[ai−1 , ai ] é um polinômio de grau 6 1, para cada i = 1, . . . , n. Prove que, se
f : [a, b] → R é contínua, então, dado ε > 0, existe uma função poligonal ϕ : [a, b] → R, tal que
|f (x) − ϕ(x)| < ε para todo x ∈ [a, b].

Solução. O que acabamos de fazer no exercício anterior pode nos ajudar aqui. Fixado ε > 0,
ε
sejam os pontos a = a0 < a1 < · · · < an = b tais que x, y ∈ [ai , ai+1 ] ⇒ |f (x) − f (y)| < .
2
Escolhemos ε /2 porque este número vai dar a origem a um intervalo de comprimento ε e se
dois pontos pertencem a um tal intervalo, então a distância ente eles não ultrapassa ε.
A função ϕ procurada é a cujo gráco é formado por segmentos de retas de extremos
(ai , f (ai )) e (ai+1 , f (ai+1 )), ou seja, é a denida por
f (ai+1 ) − f (ai )
ϕ(x) = f (ai ) + (x − ai ) ∀x ∈ [ai , ai+1 ] ∀i ∈ {0, 1, . . . , n − 1}.
ai+1 − ai
Claramente ϕ é contínua e poligonal. Além disso, xado i e considerando o intervalo I =
 ε ε
f (ai ) − , f (ai ) + , temos ϕ(x) ∈ I por construção, e f (x) ∈ I por hipótese, qualquer que
2 2
seja x ∈ [ai , ai+1 ]. Portanto, como f (x) e ϕ(x) pertencem a um mesmo intervalo de comprimento
ε, segue |f (x) − ϕ(x)| < ε como queríamos. 

7.45 Exercício 45
Dado ξ : [a, b] → R, se existem a = a0 < a1 < · · · < an = b tais que ξ|(ai−1 , ai ) é constante
(= ci ) para cada i = 1, 2, . . . , n, ξ chama-se função-escada. Mostre que se f : [a, b] → R
é contínua, então, para cada ε > 0, existe uma função escada [sic ] ξ : [a, b] → R, tal que
|f (x) − ξ(x)| < ε qualquer que seja x ∈ [a, b].

Solução. Consideramos os mesmo ai 's do exercício anterior, dados pelo Exercício 43, isto
ε
é, a = a0 < a 1 < · · · < a n = b x, y ∈ [ai , ai+1 ] ⇒ |f (x) − f (y)| < . Para cada
tais que
2
ε
i ∈ {0, 1, . . . , n − 1}, denimos ξ(x) = f (ai ) + para todo x ∈ [ai , ai+1 ) e ξ(b) = f (b). Para
2  ε ε
cada i ∈ {0, . . . , n − 1} e cada x ∈ [ai , ai+1 ], como f (x) ∈ f (ai ) − , f (ai ) + , segue o
2 2
resultado desejado, |f (x) − ξ(x)| < ε. 
7.46. EXERCÍCIO 46 271

7.46 Exercício 46
Dada uma função f : X → R, suponha que para cada ε > 0 se possa obter uma função contínua
g : X → R, tal que |f (x) − g(x)| < ε qualquer que seja x ∈ X . Então f é contínua.

Solução 1. Vamos trabalhar com a contrapositiva deste enunciado. Se f não é contínua, seja
a∈X um ponto no qual f é descontínua. Assim, por denição, seja ε0 > 0 tal que:
∀δ > 0 ∃y ∈ X ∩ (a − δ, a + δ), |f (y) − f (a)| > ε0 .
Em outras palavras, denindo o intervalo I = (f (a) − ε0 , f (a) + ε0 ), podemos também dizer
que:
∀δ > 0 ∃y ∈ X ∩ (a − δ, a + δ), f (y) ∈
/ I.
Queremos encontrar ε > 0 tal que, dada uma função contínua g : X → R, exista um ponto
x∈X tal que |g(x) − f (x)| > ε.
Fixemos então uma função contínua g : X → R. No ponto a, para o ε0 xado acima, se
tivermos |f (a) − g(a)| < ε0  o que é equivalente a dizer que g(a) ∈ I  seja δ > 0 tal que
g(x) ∈ I para todo x ∈ X ∩ (a − δ, a + δ) (Corolário do Teorema 3 e observação feita depois
dele). Para este δ , existe y ∈ X ∩ (a − δ, a + δ) tal que f (y) ∈/ I , enquanto g(y) ∈ I . Quer
dizer, estes pontos f (y) e g(y) estão separados por uma certa distância da qual temos suciente
controle.
ε0
Com as considerações feitas no parágrafo acima, seja ε = . Armamos que este é o
2
ε procurado. |g(a) − f (a)| > ε, então é o ponto a quem garante esta armação. Caso
Se
contrário, seja δ > 0 tal que g(x) ∈ (f (a) − ε, f (a) + ε) para todo x ∈ X ∩ (a − δ, a + δ). Pela
descontinuidade de f em a, seja y ∈ X ∩ (a − δ, a + δ) tal que f (y) ∈ / I . O fato de termos
g(y) ∈ (f (a) − ε, f (a) + ε) implica |g(y) − f (y)| > ε, pois
ε0 ε0
f (y) > f (a) + ε0 e g(y) < f (a) + ⇒ f (y) > f (a) + ε0 e − g(y) > −f (a) −
2 2
ε0
⇒ f (y) − g(y) >
2
e
ε0 ε0
g(y) > f (a) − e f (y) 6 f (a) − ε0 ⇒ g(y) > f (a) − e − f (y) > −f (a) + ε0
2 2
ε0
⇒ g(y) − f (y) > .
2
De qualquer forma, para ε = ε0 /2, qualquer que seja a função contínua g : X → R, existe
x∈X tal que |f (x) − g(x)| > ε, como queríamos. 

Solução 2. Fixados a∈X e ε > 0, queremos δ>0 tal que

x ∈ X, |x − a| < δ ⇒ |f (x) − f (a)| < ε .


Para qualquer função g : X → R, temos

|f (x) − f (a)| 6 |f (x) − g(x)| + |g(x) − g(a)| + |g(a) − f (a)| ∀x ∈ X.


(Desigualdade Triangular, item (i) do Teorema 2 do Capítulo III). O resultado agora segue se
ε
g:X→R é uma função contínua tal que |f (x) − g(x)| < para todo x∈X e δ>0 é tal que
3
ε
x ∈ X, |x − a| < δ ⇒ |g(x) − g(a)| < ,
3
os quais existem por hipótese e porque g é contínua em a. 
272 CAPÍTULO 7. FUNÇÕES CONTÍNUAS

7.47 Exercício 47
Seja X ⊂ R. Uma função f : X → R diz-se semicontínua superiormente no ponto a ∈ X
quando, para cada ε > 0 dado, pode-se obter δ > 0, tal que x ∈ X , |x−a| < δ ⇒ f (x) < f (a)+ε.
Diz-se que f é semicontínua superiormente quando ela o é em todos os pontos de X .

a) Dena função semicontínua inferiormente e mostre que f é contínua num ponto se, e
somente se, é semicontínua superior e inferiormente naquele ponto.

b) Prove que um subconjunto A ⊂ R é aberto se, e somente se, sua função característica
ξA : R → R (denida por ξA (x) = 1 se x ∈ A e ξA (x) = 0 se x ∈
/ A) é semicontínua
inferiormente.

c) Enuncie e prove um resultado análogo ao anterior para conjuntos fechados.

d) Mostre, mais geralmente, que para todo subconjunto X ⊂ R, sua função característica
ξX : R → R é descontínua precisamente nos pontos de fronteira de X . Dado a ∈ frX ,
mostre que ξX é semicontínua superiormente no ponto a se a ∈ X e inferiormente se
a∈/ X . Conclua que a função f : R → R, denida por f (x) = 1 para x ∈ Q e f (x) = 0
para x irracional, é semicontínua superiormente nos números racionais e inferiormente
nos números irracionais.

1
e) Seja f : R → R denida por f (x) = sen se x 6= 0 e f (0) = c. Mostre que f é
x
semicontínua superiormente no ponto 0 se, e somente se, c > 1. (E inferiormente se, e
somente se, c 6 −1.) Tomando −1 < c < 1, mostre que f é semicontínua no ponto 0.
1 1
f ) As funções f, g : R → R, onde f (0) = g(0) = 0 e, para x 6= 0, f (x) = x sen , g(x) = ,
x |x|
são semicontínuas inferiormente, mas seu produto f ·g não é uma função semicontínua
no ponto 0.

g) Para que f : X → R seja semicontínua superiormente no ponto a ∈ X ∩ X0 é neces-


sário e suciente que lim sup f (x) 6 f (a). Equivalentemente: para toda sequência de
x→a
pontos xn ∈ X com lim xn = a, que seja lim sup f (xn ) 6 f (a). Vale o análogo para
n→∞
semicontinuidade inferior.

h) A soma de duas funções semicontínuas superiormente num ponto ainda goza da mesma
propriedade. Use o item e) com c = 1 e c = −1 para dar exemplo de duas funções
semicontínuas (uma superiormente e outra inferiormente) cuja soma não é semicontínua.
Mostre que se f é semicontínua superiormente, −f é inferiormente.

i) Sejam f, g : X → R semicontínuas superiormente num ponto. Se f (x) > 0 e g(x) > 0


para todo x ∈ X , então o produto f · g é uma função semicontínua superiormente no
mesmo ponto.

j) Quando X ⊂ R é compacto, toda função semicontínua superiormente f : X → R é


limitada superiormente e atinge seu valor máximo num ponto de X. Enuncie e prove um
fato análogo para semicontinuidade inferior.

Solução.
7.47. EXERCÍCIO 47 273

a) Vamos dizer que uma função f : X → R, com X ⊂ R, é semicontínua inferiormente


quando ela for semicontínua inferiormente em cada um dos pontos de seu domínio e que f é
semicontínua inferiormente no ponto a ∈ X quando, para todo ε > 0, existe δ > 0 tal que
x∈X e |x − a| < δ implicam f (x) > f (a) − ε.
Se f é contínua em a, dado ε > 0, por denição existe δ > 0 tal que:

x ∈ X, |x − a| < δ ⇒ |f (x) − f (a)| < ε ⇔ f (a) − ε < f (x) < f (a) + ε .

Logo, f é semicontínua superior e inferiormente em a.


Se f é semicontínua superior e inferiormente em a, dado ε > 0, seja δ = min{δ1 , δ2 }, sendo
δ1 > 0 e δ2 > 0 tais que

x ∈ X, |x − a| < δ1 ⇒ f (x) < f (a) + ε

e
x ∈ X, |x − a| < δ2 ⇒ f (a) − ε < f (x),
de modo que se |x − a| < δ , então |x − a| < δ1 e |x − a| < δ2 . Assim, se x∈X e |x − a| < δ ,
então
f (a) − ε < f (x) < f (a) + ε,
o que nos permite concluir que f é contínua em a, como queríamos. 

b) Vamos supor que A seja aberto. Dados a∈R e ε > 0, queremos δ>0 tal que

|x − a| < δ ⇒ ξA (x) > ξA (a) − ε .

Se ξA (a) = 0, então não há nada a se fazer; qualquer δ > 0 serve porque ξA (x) > 0 para todo
x ∈ R. Se ξA (a) = 1, então o δ > 0 que devemos escolher deve ser tal que ξA (x) = 1 para todo
x ∈ (a − δ, a + δ) porque, se ε for pequeno o suciente, então ξA (x) > ξA (a) − ε = 1 − ε ⇒
ξA (x) = 1. Se ξA (a) = 1, então, por denição, a ∈ A e, como A é aberto, escolhemos δ > 0
tal que (a − δ, a + δ) ⊂ A. Para este δ temos ξA (x) = 1 para todo x ∈ (a − δ, a + δ), como
queríamos. Logo, ξA é uma função semicontínua inferiormente. O que zemos aqui também
prepara o terreno para mostrarmos a recíproca deste resultado.
Vamos supor agora ξA semicontínua inferiormente. Dado a ∈ A, queremos δ > 0 tal que
(a − δ, a + δ) ⊂ A. Paraε = 1/2 (que é pequeno o suciente no sentido a que nos referimos no
parágrafo acima), como ξA é semicontínua inferiormente, existe δ > 0 tal que

1 1 1
ξA (x) > ξ(a) − = 1 − = ⇒ ξA (x) = 1 ∀x ∈ (a − δ, a + δ),
2 2 2
isto é, (a − δ, a + δ) ⊂ A, como queríamos. 

c) Armamos que um dado conjunto F ⊂ R é fechado se, e somente se, sua função
característica ξF é semicontínua superiormente. Uma solução semelhante a do item anterior
demonstra esta armação. Então, para acrescentarmos alguma coisa nova, vamos demonstrar
o seguinte:

Dado um conjunto X ⊂ R, ξX é semicontínua superiormente se, e somente se, ξR −X


é semicontínua inferiormente.
274 CAPÍTULO 7. FUNÇÕES CONTÍNUAS

Com este resultado e com o item anterior teremos:

F é fechado ⇔ R −F é aberto
⇔ ξR −F é semicontínua inferiormente
⇔ ξF é semicontínua superiormente.

Vamos supor ξX semicontínua superiormente. Dados a ∈ R e ε > 0, queremos δ > 0 tal que

|x − a| < δ ⇒ ξR −X (x) > ξR −X (a) − ε .

Conforme já observamos anteriormente, o caso que mais interessa mesmo é quando ξR −X (a) =
1, ou seja, quando a ∈ R −X . a temos ξX (a) = 0 e, como ξX é semicontínua supe-
Para um tal
riormente, podemos concluir que existe δ > 0 tal que ξX (x) = 0 para todo x ∈ (a − δ, a + δ), o
que é equivalente a dizer que ξR −X (x) = 1 ∀x ∈ (a − δ, a + δ). Portanto ξR −X (x) = 1 > 1 − ε =
ξR −X (a) − ε ∀x ∈ (a − δ, a + δ), como queríamos. A recíproca, isto é, que a semicontinuidade
inferior de ξR −X implica a semicontinuidade superior de ξX , é análoga. 

d) Por denição (não encontrei a denição neste livro), frX = X ∩ R −X , ou seja, um


ponto pertence à fronteira de X se, e somente se, todo intervalo ao qual ele pertence possui um
ponto de X e um ponto de R −X .

A partir da denição acima, xado a ∈ frX , para um dado δ > 0, sejam x1 ∈ X ∩(a−δ, a+δ)

e x2 ∈ R −X ∩ (a − δ, a + δ). Se a ∈ X , então |ξX (x2 ) − ξX (a)| = 1 e, se a ∈ / X , então
|ξX (x1 ) − ξX (a)| = 1. Isto mostra que ξX é descontínua em qualquer ponto do conjunto frX .
Se a ∈/ frX , então a ∈/ X ou a ∈
/ R −X . Se a ∈
/ X , então existe δ > 0 tal que (a − δ, a + δ) ⊂
R −X , de modo que ξX (x) = 0 ∀x ∈ (a − δ, a + δ). Se a ∈ / R −X , então existe δ > 0 tal que
(a − δ, a + δ) ⊂ X , de modo que ξX (x) = 1 ∀x ∈ (a − δ, a + δ). De qualquer forma, existe δ > 0
tal que

x ∈ (a − δ, a + δ) ⇒ |ξX (x) − ξX (a)| = 0 < ε,

qualquer que seja o ε>0 dado, o que mostra a continuidade de ξX em qualquer ponto fora
da fronteira de X. Com estes dois últimos parágrafos podemos concluir que ξX é descontínua
apenas em frX , como queríamos.

Para a segunda parte deste exercício, conforme já tangenciamos nos itens anteriores, o
fato de ξX ser semicontínua superiormente em a se a ∈ X e semicontínua inferiormente em
a se a ∈/ X independe de a ser ponto de fronteira. Se a ∈ X , então, ξX (a) = 1, o que faz
ξX (x) < ξX (a) + ε, para todo x ∈ R e todo ε > 0. De modo análogo, se a ∈/ X , então ξ(a) = 0,
o que implica ξX (x) > ξX (a) − ε ∀x ∈ R ∀ ε > 0. A conclusão segue disto porque a f dada é a
função característica dos racionais, f = ξQ . 

Observação. Vimos que não foi necessário supora ∈ frX para concluir que ξX é semicontínua
superiormente em a se a ∈ X e semicontínua inferiormente em a se a ∈ / X . Mas, pelos
itens a) e d), se a ∈ frX , podemos concluir que, em a, ξX é semicontínua superiormente e
não é semicontínua inferiormente se a ∈ X e que ξX é semicontínua inferiormente e não é
semicontínua superiormente se a ∈ / X . Portanto, como fr Q = R, podemos armar algo mais
forte a respeito da função ξQ , a saber, que ela é semicontínua superiormente apenas em Q e
que ela é semicontínua inferiormente apenas em R − Q.
7.47. EXERCÍCIO 47 275

e) Se c > 1, então f (x) < f (0) + ε para todo x ∈ R e todo ε > 0. Se c 6 −1, então
f (x) > f (0) − ε para todo x ∈ R e todo ε > 0. Portanto, f é semicontínua superiormente se
c > 1 e semicontínua inferiormente se c 6 −1. Se −1 < c < 1, então podemos tomar ε > 0 tal
que c + ε < 1 e c − ε > −1. Para um tal ε, já sabemos que, qualquer que seja o δ > 0, existem
x e y em (−δ, δ) tais que f (x) > c + ε e f (y) < c − ε (Exemplo 6 do Capítulo VI). Portanto,
f não é semicontínua superior nem inferiormente no 0 se −1 < c < 1. Logo, f é semicontínua
superiormente no 0 se, e somente se c > 1 e semicontínua inferiormente no 0 se, e somente se,
c 6 −1. Se −1 < c < 1, então (repetindo) f não é semicontínua superior nem inferiormente no
0. 

Observação. Talvez haja um erro de redação no enunciado deste item, f não tem nenhum
tipo de semicontinuidade em 0 se −1 < c < 1.

f) A f é semicontínua inferiormente no 0 como consequência do item a): lim f (x) = 0


x→0
(produto de uma função que tende a zero por uma limitada, Teorema 7 do Capítulo VI) implica
a continuidade de f na origem (e, portanto, f é também semicontínua superiormente no 0).
A g é semicontínua inferiormente no 0 porque g(x) > g(0) − ε para todo x ∈ R, qualquer
1
que seja ε > 0. Para o produto, temos (f · g)(x) = sen para todo x > 0 e já sabemos
x
(de novo o Exemplo 6 do Capítulo VI) que, dado δ > 0, existem x e y em (0, δ) tais que
(f · g)(x) 6 (f · g)(0) − 1/2 e (f · g)(y) > (f · g)(0) + 1/2, por exemplo. Portanto, f · g não é
semicontínua inferior nem superiormente em 0. 

g) Este item traz os símbolos lim sup f (x) e lim sup f (xn ) sem trazer as hipóteses de f
x→a n→+∞
ser limitada numa vizinhança do a (f (xn ))n∈N ser limitada. Hipóteses estas
ou de a sequência
assumidas em todo o desenvolvimento da teoria do  lim sup e do  lim inf  (sobre a ausência de
uma destas hipóteses é feito apenas um comentário no livro, na página 215; neste comentário
no lugar do numa do trecho para indicar que f é ilimitada superiormente numa vizinhança de
a, para não haver incoerência, deveria ser em toda, pois uma função pode ser ilimitada numa
vizinhança e ser limitada em outra, por exemplo, a função tan : (−π/2, π/2) → R, ilimitada
numa vizinhança do 0 e limitada numa vizinhança do 0). Assim, para podermos falar nestes
símbolos e darmos continuidade a esta solução, podemos supor f limitada numa vizinhança do
a. Uma outra alternativa é atribuirmos signicados para eles válidos mesmo nos casos em que
f não é limitada em nenhuma vizinhança do a. Vamos desenvolver esta última opção.
Fixemos então a sequência (xn )n∈N de pontos em X convergente para a e consideremos
a sequência (f (xn ))n∈N . Se f é semicontínua superiormente em a, então (f (xn ))n∈N é uma
sequência limitada superiormente (f é limitada superiormente em alguma vizinhança de a;
como xn → a, existe n0 ∈ N tal que xn pertence a esta vizinhança para todo n > n0 , portanto
(f (xn ))n>n0 é limitada superiormente, assim como o conjunto {f (x1 ), . . . , f (xn0 )}). Para uma
sequência (yn )n∈N limitada superiormente, seja bn = sup{yn , yn+1 , yn+2 , . . .} para cada n ∈ N;
denimos
lim sup yn = lim bn .
n→+∞ n→+∞

Deste modo: se a sequência (bn )n∈N é limitada inferiormente, então lim sup yn = inf bn
e a
n∈Nn→+∞
nossa denição coincide com a do livro caso (f (xn ))n∈N seja limitada, e; se a sequência (bn )n∈N
não é limitada inferiormente, então (como ela é monótona não crescente) lim sup yn = −∞.
n→+∞
Ainda supondo f semicontínua superiormente em a, seja δ1 > 0 tal que f [Vδ1 ] é limitada
276 CAPÍTULO 7. FUNÇÕES CONTÍNUAS

superiormente (lembrando da notação Vδ = (X−{a})∩(a−δ, a+δ)). Assim, como f [Vδ ] ⊂ f [Vδ1 ]


se δ ∈ (0, δ1 ], considerando a função (0, δ1 ] 3 δ 7→ Lδ := sup f [Vδ ] ∈ R, denimos

lim sup f (x) = lim Lδ .


x→a δ→0

Vamos adotar esta denição mesmo que f seja apenas limitada superiormente em alguma
vizinhança de a. Ela coincide com a antiga caso f seja limitada em alguma vizinhança de a pelo
Teorema 14 do Capítulo VI. Se a função Lδ não é limitada inferiormente, então lim sup f (x) =
x→a
−∞. No caso em que f não é limitada superiormente em nenhuma vizinhança de a, denimos

lim sup f (x) = +∞.


x→a

E se uma sequência (yn )n∈N não é limitada superiormente,

lim sup yn = +∞.


n→+∞

De forma análoga, se (yn )n∈N é limitada inferiormente, denimos

lim inf yn = lim inf yi


n→+∞ n→+∞ i>n

(se f é semicontínua inferiormente em a, então (f (xn ))n∈N é limitada inferiormente) e, se f


é limitada inferiormente em alguma vizinhança de a (que é o que ocorre se f é semicontínua
inferiormente em a),
lim inf f (x) = lim lδ ,
x→a δ→0

sendo lδ = inf f [Vδ ] para δ ∈ (0, δ2 ], δ2 > 0 tal que f [Vδ2 ] é limitada inferiormente. Se f não é
limitada inferiormente em nenhuma vizinhança de a,

lim inf f (x) = −∞.


x→a

E, se (yn )n∈N é uma sequência ilimitada inferiormente,

lim inf yn = −∞.


n→+∞

Considerações análogas às do último parágrafo continuam válidas aqui.


Com a possibilidade de lim sup ou de lim inf serem iguais aos símbolos de +∞ ou de −∞,
para que a tese continue válida, vamos denir também as seguintes desigualdades

−∞ < x < +∞ ∀x ∈ R

(+∞ não é menor do que nem igual a nenhum número real e −∞ não é maior do que nem
igual a nenhum número real). Agora sim estamos em condições de demonstrar o que se pede.
Se f é semicontínua superiormente em a, dado ε > 0, seja n1 ∈ N tal que

n ∈ N ∩[n1 , +∞) ⇒ f (xn ) < f (a) + ε

(é o n1 tal que |xn − a| < δ se n ∈ N ∩[n1 , +∞), sendo δ > 0 tal que x ∈ X, |x − a| < δ ⇒
f (x) < f (a) + ε; n1 existe porque lim xn = a). Portanto,

bn1 := sup{f (xn1 ), f (xn1 +1 ), f (xn1 +2 ), . . .} 6 f (a) + ε .


7.47. EXERCÍCIO 47 277

Como lim sup f (xn ) 6 bn1 (porque este limite superior é o ínmo dos bn 's ou porque ele é
n→+∞
igual a −∞), segue
lim sup f (xn ) 6 f (a) + ε,
n→+∞

desigualdade que vale para todo ε > 0, o que nos permite concluir que

lim sup f (xn ) 6 f (a)


n→+∞

(pois, se fosse lim sup f (xn ) > f (a), então, para 0 < ε < lim sup f (xn ) − f (a), teríamos
n→+∞ n→+∞
f (a) + ε < lim sup f (xn )).
n→+∞
Analogamente, se f é semicontínua inferiormente em a, temos

f (a) − ε 6 lim inf f (xn ),


n→+∞

para todo ε > 0:

n ∈ N ∩[n2 , +∞) ⇒ f (a) − ε < f (xn )


⇒ f (a) − ε 6 inf{f (xn2 ), f (xn2 +1 ), f (xn2 +2 ), . . .} 6 lim inf f (xn ),
n→+∞

pois lim inf f (xn ) é o supremo dos números inf f (xi ) (conforme n varia nos naturais) ou é
n→+∞ i>n
igual a +∞. Logo
f (a) 6 lim inf f (xn ).
n→+∞

Se lim sup f (x) 6 f (a), então lim sup f (x) 6= +∞ e, consequentemente, f é limitada supe-
x→a x→a
riormente numa vizinhança de a, de modo que

lim sup f (x) = lim Lδ .


x→a δ→0

Assim, dado ε > 0, como lim Lδ < f (a) + ε (o que é verdade mesmo que este limite seja
δ→0
−∞), existe δ > 0 tal que Lδ < f (a) + ε. Para este δ temos f (x) < f (a) + ε para todo
x ∈ (X − {a}) ∩ (a − δ, a + δ) pela denição de Lδ . Logo, f é semicontínua superiormente em
a.
Se lim inf f (x) > f (a), então lim inf f (x) 6= −∞ e, em decorrência, f é limitada inferior-
x→a x→a
mente numa vizinhança de a; assim temos

lim inf f (x) = lim lδ .


x→a δ→0

Dado ε > 0, de lim lδ > f (a) − ε (o que é verdade mesmo que este limite seja +∞), podemos
δ→0
concluir que existe δ > 0 tal que lδ > f (a) − ε. Para este δ , pela denição de lδ , segue
f (x) > f (a) − ε para todo x ∈ (X − {a}) ∩ (a − δ, a + δ). Logo, f é semicontínua inferiormente
em a.
Com estas novas denições, falta ainda demonstrar a parte do equivalentemente do enun-
ciado (assumindo a limitação de f em alguma vizinhança de a, não seria necessário demons-
trar isto, o que foi desenvolvido no livro já bastaria). De modo mais preciso, dada a função
f : X → R (X ⊂ R) e dado a ∈ X ∩ X 0 , vamos mostrar que

lim sup f (x) 6 f (a) ⇔ lim sup f (xn ) 6 f (a)


x→a n→+∞
278 CAPÍTULO 7. FUNÇÕES CONTÍNUAS

e que
lim inf f (x) > f (a) ⇔ lim inf f (xn ) > f (a),
x→a n→+∞

qualquer que seja a sequência (xn )n∈N de pontos emX tal que lim xn = a.
Para a ida, xemos a sequência (xn )n∈N ∈ F(N; X) convergente e tal que lim xn = a.
Se lim sup f (x) 6 f (a), então lim sup f (x) = lim sup f [Vδ ] (pois lim sup f (x) 6 f (a) ∈ R ⇒
x→a x→a δ→0 x→a
lim sup f (x) 6= +∞ ⇒ f limitada superiormente numa vizinhança de a). Como x n → a, dado
x→a
δ > 0, existe nδ ∈ N tal que n > nδ ⇒ xn ∈ Vδ ∪ {a}, então n > nδ ⇒ f (xn ) ∈ f [Vδ ] ∪ {f (a)}, o
que implica sup f (xn ) 6 sup f [Vδ ] ∪ {f (a)} (para δ tal que este supremo faça sentido) e, mais
n>nδ
do que isto, sup f (xi ) 6 sup f [Vδ ] ∪ {f (a)} para todo n > nδ . Portanto
i>n

lim sup f (xi ) 6 sup f [Vδ ] ∪ {f (a)} = max{sup f [Vδ ], f (a)}


n→+∞ i>n

para todo δ > 0, logo

ex47gCap7sol
lim sup f (xi ) 6 lim max{sup f [Vδ ], f (a)} = max{lim sup f [Vδ ], f (a)} 6 f (a)
n→+∞ i>n δ→0 δ→0
(Exercício 20 do Capítulo VI, página 224), ou seja,

lim sup f (xn ) 6 f (a),


n→+∞

como queríamos, pois lim sup f (xn ) = lim sup f (xi ) já que f limitada superiormente em
n→+∞ n→+∞ i>n
alguma vizinhança de a implica a limitação superior da sequência (f (xn ))n∈N .
Reciprocamente, suponhamos lim sup f (x) > f (a). Se lim sup f (x) = +∞,
então f não é
x→a x→a
limitada superiormente em nenhuma vizinhança de a (xn )n∈N
e podemos obter uma sequência
em X que converge para a tal (f (xn ))n∈N não é limitada superiormente. Para esta sequência
temos lim sup f (xn ) = +∞ > f (a). Se lim sup f (x) ∈ R, seja f (a) < k < lim sup f (x).
n→+∞ x→a x→a
Como lim sup f (x) é igual ao limite lim sup f [Vδ ], seja δ0 > 0 tal que sup f [Vδ ] > k para todo
x→a δ→0
1
0 < δ < δ0 . Seja n0 ∈ N tal que < δ0 , de modo que n ∈ N ∩[n0 , +∞) ⇒ sup f [V1/n ] > k .
n0
Assim, para cada natural n > n0 , podemos obter xn ∈ V1/n tal que f (xn ) > k . Esta sequência
(xn )n>n0 é tal que lim xn = a e sup f (xi ) > k ∀n ∈ N ∩[n0 , +∞) (a sequência (f (xn ))n>n0
i>n
é limitada superiormente porque f é limitada superiormente em alguma vizinhança de a).
Portanto lim sup f (xi ) > k > f (a), sendo lim sup f (xi ) = lim sup f (xn ). De qualquer
n→+∞ i>n n→+∞ i>n n→+∞
forma (f limitada superiormente ou não),

lim sup f (x) > f (a) ⇒ lim sup f (xn ) > f (a)
x→a n→+∞

para alguma sequência (xn )n∈N de pontos em X convergente para a, o que conclui esta recíproca.
A ideia para mostrar a equivalência lim inf f (x) > f (a) ⇔ lim inf f (xn ) > f (a) é basica-
x→a n→+∞
mente a mesma. 

Observação. Dada a função a ∈ X 0 , poderíamos pensar que o


f : X → R (X ⊂ R) e dado
caso em que f é limitada superiormente em alguma vizinhança do a e a função Lδ é limitada
inferiormente recai no caso em que f é limitada em alguma vizinhança de a e, portanto, não
7.47. EXERCÍCIO 47 279

há nada de novo na denição de lim sup f (x) que demos aqui para esse caso (para as denições
x→a
dadas acima não precisa ser a∈ X ), uma vez que a denição antiga se aplica a ele. Porém isto
1
não é verdade. Por exemplo, para f : R → R denida por f (x) = − se x ∈ R − Q e f (x) = 0
|x|
se x ∈ Q, para a=0 temos: f Lδ = 0 para todo δ > 0, mas f não
limitada superiormente,
é limitada inferiormente em nenhuma vizinhança do 0, lim sup f (x) = 0 e lim inf f (x) = −∞.
x→0 x→0
Algo semelhante vale se f é limitada inferiormente em alguma vizinhança de a e lδ é limitada
superiormente: isto também não implica a limitação local de f em a.

h) Sejam f, g : X → R, com X ⊂ R, duas funções semicontínuas superiormente num dado


ponto a ∈ X . Fixado ε > 0, queremos encontrar δ > 0 tal que:

(f + g)(x) < (f + g)(a) + ε ∀x ∈ X ∩ (a − δ, a + δ).

Este δ é o menor dos números reais positivos δ1 e δ2 tais que

ε
f (x) < f (a) + ∀x ∈ X ∩ (a − δ1 , a + δ1 )
2
e
ε
g(x) < g(a) + ∀x ∈ X ∩ (a − δ2 , a + δ2 ),
2
os quais existem porque ambas f e g são semicontínuas superiormente em a por hipótese. De
fato, se x ∈ X ∩ (a − δ, a + δ), então x ∈ X ∩ (a − δ1 , a + δ1 ) e x ∈ X ∩ (a − δ2 , a + δ2 ), o que
implica
ε ε
f (x) + g(x) < f (a) + + g(a) + = f (a) + g(a) + ε,
2 2
ou seja,

(f + g)(x) < (f + g)(a) + ε


como queríamos.
Seguindo a sugestão, vamos denir as funções g : R → R e h : R → R colocando
 

 0, se x<0 
 0, se x > 0
g(x) = 1, se x=0 e h(x) = −1, se x = 0
1
 sen ,  sen 1 , se x < 0
x>0
 
se
x x
280 CAPÍTULO 7. FUNÇÕES CONTÍNUAS

Na origem (bem como em qualquer outro ponto), a g é semicontínua superiormente e a h é


semicontínua inferiormente, porém a função g+h que, por sua vez, é dada por

1
(g + h)(x) = sen se x 6= 0 e (g + h)(0) = 0,
x
não é semicontínua nem inferiormente nem superiormente na origem.
Dado ε > 0, se f é semicontínua superiormente num ponto a de seu domínio, existe δ > 0
tal que f (x) < f (a) + ε para todo x ∈ (a − δ, a + δ) pertencente ao seu domínio, onde

f (x) < f (a) + ε ⇒ (−f )(x) > (−f )(a) − ε .

Logo, −f e semicontínua inferiormente em a. Segue a tese. 

i) Vamos supor f e g semicontínuas superiormente num ponto a ∈ X. Dado ε > 0,


queremos encontrar δ > 0 tal que

f (x) · g(x) < f (a) · g(a) + ε ∀x ∈ X ∩ (a − δ, a + δ).

Se f (x) < f (a) + ε0 e g(x) < g(a) + ε0 para um certo ε0 , então, como f (x) e g(x) são
números não negativos, multiplicando, obtemos

f (x) · g(x) < f (a) · g(a) + ε20 + (f (a) + g(a))ε0 .

Este ε0 pode ser tomado de modo a termos ε0 > 0 e ε20 + (f (a) + g(a))ε0 = ε. Fixando um tal
ε0 , o δ procurado é igual a min{δ1 , δ2 }, onde δ1 e δ2 são números reais positivos tais que

f (x) < f (a) + ε0 ∀x ∈ X ∩ (a − δ1 , a + δ1 )

e
g(x) < g(a) + ε0 ∀x ∈ X ∩ (a − δ2 , a + δ2 ),
os quais existem por hipótese. 

j) Fixado ε > 0, para cada a∈X seja δa > 0 tal que

f (x) < f (a) + ε ∀x ∈ X ∩ (a − δa , a + δa ).


[
Denindo Ia = (a − δa , a + δa ) ∀a ∈ X , temos X ⊂ Ia de modo que, como X é compacto,
a∈X
existe n∈N e existem elementos a1 , . . . , a n pertencentes a X tais que

X ⊂ Ia1 ∪ · · · ∪ Ian .

Seja A = max{f (ai ) + ε; i ∈ {1, . . . , n}}. Armamos que f (x) < A para todo x ∈ X. De fato:

x ∈ X ⇒ ∃i ∈ {1, . . . , n}, x ∈ X ∩ Iai ⇒ f (x) < f (ai ) + ε 6 A.

Logo, f é limitada superiormente.


Vejamos agora que f atinge seu valor máximo em algum ponto de X. Com o que vimos até
agora sabemos que o conjunto f [X] ⊂ R é limitado superiormente. Seja L uma cota superior
para f [X]. Se existe x ∈ X tal que f (x) = L, então f atinge seu valor máximo neste x.
Se um tal x não existe, para cada x ∈ X , seja Lx ∈ R tal que f (x) < Lx < L. Como f é
7.47. EXERCÍCIO 47 281

semicontínua superiormente, para cada x ∈ X , temos ηx > 0 tal que f (y) < Lx para todo
y ∈ X ∩ (x − ηx , x + ηx ). Agora, do mesmo modo como zemos no parágrafo anterior, pela
compacidade de X , podemos concluir que existe x ∈ X tal que f (y) < Lx para todo y ∈ X ,
onde Lx < L. Deste modo podemos concluir o seguinte: se o valor máximo de f não fosse
atingido, então o conjunto das cotas superiores de f [X] não teria um mínimo, o que contraria a
completude de R, segundo a qual qualquer conjunto não vazio e limitado superiormente admite
um supremo (estamos supondo X 6= ∅; se X = ∅, então o resultado é falso: o conjunto vazio é
compacto, qualquer função é semicontínua superiormente nele, mas não existe um elemento em
∅ no qual uma função possa assumir seu valor máximo, simplesmente porque não há elemento
algum em tal conjunto).
O análogo deste resultado para semicontinuidade inferior é assim: se f :X→R é semicon-
tínua inferiormente, então f é limitada inferiormente e atinge seu valor mínimo em algum ponto
de X. A demonstração também é inteiramente análoga à demonstração que demos acima. 

Observação 1. Uma outra possibilidade de caminho para uma parte deste item é a seguinte:

f semicontínua superiormente ⇒ f limitada superiormente localmente

⇒ f limitada superiormente, se X é compacto,

semelhante ao que pedia o Exercício 57 do Capítulo V (página 198).

Observação 2. Este item junto com o item a) nos dá uma outra demonstração para o Teorema
de Weierstrass (Corolário do Teorema 14), se f é contínua, então f é semicontínua superior e
inferiormente.
282 CAPÍTULO 7. FUNÇÕES CONTÍNUAS
Capítulo 8
Derivadas

8.1 Exercício 1
Sejam f, g, h : X → R tais que, para todo x ∈ X se tenha f (x) 6 g(x) 6 h(x). Se num ponto
a ∈ X ∩ X 0 tem-se f (a) = h(a) e existem f 0 (a) = h0 (a) então existe g 0 (a) e tem o mesmo valor.

Solução. Se x > a, então

f (x) − f (a) g(x) − g(a) h(x) − h(a)


6 6 ;
x−a x−a x−a
e, se x < a, então
h(x) − h(a) g(x) − g(a) f (x) − f (a)
6 6
x−a x−a x−a
(pois f (a) = h(a) e f (a) 6 g(a) 6 h(a) implicam f (a) = g(a) = h(a)). O resultado segue pelo
Teorema 4 do Capítulo VI, junto com o Teorema 2 do mesmo capítulo, mais o fato de uma
derivada existir se, e somente se, suas derivadas laterais existirem e forem iguais (no caso de
a ser um ponto de acumulação à direita e à esquerda de X; no caso de a ser de acumulação
apenas à direita de X, então consideramos só as primeiras duas desigualdades; no caso de
a ser de acumulação apenas à esquerda de X, então consideramos apenas as duas últimas
desigualdades). 

8.2 Exercício 2
Seja a ∈ Xum ponto de máximo local para a função f : X → R. Se f possui derivada à
0
direita no ponto a, então ela é 6 0. Se existir, f− (a) deve ser > 0. Dê exemplo de um ponto
de máximo local onde existam as duas derivadas laterais e sejam diferentes.

Solução. a ∈ X é um ponto de máximo


Se local para f, seja δ>0 tal que f (x) 6 f (a) para
todo x ∈ X ∩ (a − δ, a + δ), de modo que

f (x) − f (a)
60 ∀x ∈ X ∩ (a, a + δ)
x−a
e
f (x) − f (a)
>0 ∀x ∈ X ∩ (a − δ, a).
x−a

283
284 CAPÍTULO 8. DERIVADAS

Assim, se existe f+0 (a), então este é um limite de uma função não positiva e, portanto, f+0 (a) 6 0
0
pelo Corolário 2 do Teorema 5 do Capítulo VI. Análogo para f− (a): se ele existe, então ele é o
limite de uma função não negativa e a conclusão também segue pelo mesmo corolário.
Um exemplo de um ponto de máximo local no qual as duas derivadas laterais existem e são
diferentes é a função g : R → R denida por g(x) = −|x| ∀x ∈ R. Para ela: 0 é um ponto de
0 0
máximo local, g+ (0) = −1 e g− (0) = 1. 

8.3 Exercício 3
Seja p:R→R um polinômio de grau ímpar. Existe c∈R tal que p00 (c) = 0.

00
Solução. Por um lado, pelo Exemplo 4 sabemos que: se o grau de p é 1, então p é um
00
polinômio identicamente nulo, e, então, todo c ∈ R é tal que p (c) = 0, e; se o grau de p é pelo
00
menos 3, então p também é um polinômio de grau ímpar. Por outro lado, pelo Exercício 13 do
Capítulo VI, bem como pela sua observação (segundo os quais podemos armar que existem
x1 e x2 tais que q(x1 ) > 0 e q(x2 ) < 0 se q é um polinômio de grau ímpar; páginas 217 e 218),
e pelo Teorema do Valor Intermediário, podemos concluir que todo polinômio de grau ímpar
00
possui pelo menos uma raiz real. Portanto, existe c ∈ R tal que p (c) = 0, como queríamos. 

8.4 Exercício 4
Seja f :I→R denida num intervalo do qual a é ponto interior. Se f é derivável no ponto a
então
f (a + h) − f (a − h)
lim = f 0 (a).
h→0 2h
A existência do limite acima, entretanto, não implica a continuidade de f no ponto a, nem que
0
exista a derivada f (a), mesmo quando f é contínua neste ponto.

Solução. Temos
 
f (a + h) − f (a − h) 1 f (a + h) − f (a) f (a) − f (a − h)
= · + .
2h 2 h h
Se f é derivável em a, sabemos que

f (a + h) − f (a)
f 0 (a) = lim ,
h→0 h
igualdade esta obtida da igualdade

f (x) − f (a)
f 0 (a) = lim ,
x→a x−a
substituindo-se x − a por h. De modo análogo, podemos substituir a − x por h obtendo também
f (a − h) − f (a) f (a) − f (a − h)
f 0 (a) = lim = lim .
h→0 −h h→0 h
Deste modo:

f (a + h) − f (a) f (a) − f (a − h) f (a + h) − f (a − h)
2f 0 (a) = lim + lim = lim ,
h→0 h h→0 h h→0 h
8.5. EXERCÍCIO 5 285

donde
f (a + h) − f (a − h)
f 0 (a) = lim ,
h→0 2h
como queríamos.
Para ver que a existência do referido limite não implica a continuidade de f no ponto a, nem
0
que exista a derivada f (a), mesmo quando f é contínua neste ponto, consideremos as funções
f1 , f2 : R → R denidas por f1 (x) = x para todo x ∈ R −{0}, f1 (0) = 1 e f2 (x) = |x| para todo
x ∈ R. Em a = 0, para elas temos

f1 (a + h) − f1 (a − h) h − (−h)
lim = lim = 1,
h→0 2h h→0 2h
f2 (a + h) − f2 (a − h) h−h
lim+ = lim+ =0
h→0 2h h→0 2h
f2 (a + h) − f2 (a − h) −h − (−h)
lim− = lim+ = 0,
h→0 2h h→0 2h
ou seja, ambos os limites

f1 (0 + h) − f1 (0 − h) f2 (0 + h) − f2 (0 − h)
lim e lim
h→0 2h h→0 2h
existem, embora f1 não seja contínua em 0 e f2 , contínua em 0, não seja derivável em 0. 

8.5 Exercício 5
f (x) − f (a)
Seja f :X→R contínua. Dado a ∈ X ∩ X 0, dena ξ:X→R pondo ξ(x) = se
x−a
x 6= a e ξ(a) = L. Prove que ξ é contínua se, e somente se, existe f 0 (a) e f 0 (a) = L.

Solução. Se f 0 (a) existe e f 0 (a) = L, então:

f (x) − f (a)
ξ(a) = L = f 0 (a) = lim = lim ξ(x).
x→a x−a x→a

Portanto ξ é contínua em a. A ξ é contínua num ponto x 6= 0 pelo Teorema 5 do Capítulo VII.


Se ξ é contínua, então ξ é contínua em a, de modo que

f (x) − f (a)
L = ξ(a) = lim ξ(x) = lim ,
x→a x→a x−a
ou seja, f 0 (a) existe e f 0 (a) = L, como queríamos. 

8.6 Exercício 6
Seja p(x) = x3 + ax2 + bx + c um polinômio do 3.o grau com coeciente líder igual a 1. Mostre
2
que p : R → R é um homeomorsmo se, e somente se, a 6 3b. Para que o homeomorsmo
−1 2
inverso f = p seja derivável é necessário e suciente que a < 3b.

Solução 1.
286 CAPÍTULO 8. DERIVADAS

Primeira Parte: p é um homeomorsmo ⇔ a2 6 3b.

Nível 1. Já sabemos que p é uma função contínua e sobrejetiva (respectivamente, Exemplo


2 do Capítulo VII e Exercício 13 do Capítulo VI, página 217, junto com o Corolário 2 do Teorema
12 do Capítulo VII). Assim, para que p
seja um homeomorsmo, é necessário e suciente que
p seja uma função injetiva (Teorema 13 do Capítulo VII). A desigualdade a2 6 3b aparece no
0 2
estudo das raízes da derivada de p, que é dada por p (x) = 3x + 2ax + b e possui no máximo
uma raiz real se, e somente se,

(2a)2 − 4 · 3 · b = 4a2 − 12b 6 0 ⇔ a2 6 3b.


Armamos: se p não é injetiva, então existem reais α e β distintos tais que p0 (α) = p0 (β) = 0
2 0 0
(Nível 2.1) e, portanto, a > 3b, e; se existem reais α e β distintos tais que p (α) = p (β) = 0 (ou
2
seja, se a > 3b), então p não é injetiva (Nível 2.2). As justicativas de ambas estas armações
concluem a demonstração desta primeira parte, pela contrapositiva.

Nível 2.1. Se p não é injetiva, sejam os reais x1 < x2 tais que p(x1 ) = p(x2 ). Assim,
pelo Teorema de Rolle (Teorema 6), existe x3 ∈ (x1 , x2 ) tal que p0 (x3 ) = 0, e encontramos um
ponto no qual a derivada de p se anula, só faltando agora encontrar o outro. Armamos que
existe x4 6= x3 tal que p(x4 ) = p(x3 ) (Nível 3.1), de modo que, novamente pelo Teorema de
0
Rolle, existe x5 distinto de x3 tal que p (x5 ) = 0, mais precisamente x5 ∈ (x3 , x4 ) se x3 < x4 ou
x5 ∈ (x4 , x3 ) se x4 < x3 . Logo, o α e o β procurados são α = x3 e β = x5 .

Nível 2.2. Se α < β, armamos que α é um ponto de máximo local de p (e, por motivo
análogo, β
é ponto de mínimo local de p). Isto será demonstrado no Nível 3.2. Assim, podemos
tomar um δ > 0 tal que p(x) 6 p(α) para todo x ∈ [α − δ, α + δ]. Se p(α − δ) = p(α + δ),
se p(α − δ) = p(α) ou se p(α + δ) = p(α), então p não é injetiva, como queríamos. Caso
contrário, a não injetividade de p segue novamente do Teorema do Valor Intermediário: se
p(α−δ) > p(α+δ), então p(α−δ) ∈ (p(α+δ), p(α)) e existe x ∈ (α, α+δ) tal que p(x) = p(α−δ);
se p(α + δ) > p(α − δ), então p(α + δ) ∈ (p(α − δ), p(α)) e existe x ∈ (α − δ, α) tal que
p(x) = p(α + δ).

Nível 3.1. Se p(x3 ) = p(x1 ), então basta fazer x4 = x1 . Caso contrário, isto é, caso
p(x3 ) 6= p(x1 ), a existência de x4 6= x3 tal que p(x4 ) = p(x3 ) segue do Teorema do Valor
Intermediário (Teorema 12 do Capítulo VII) e do já mencionado Exercício 13 do Capítulo VI
(página 217): se p(x3 ) < p(x1 ), então existe x4 < x1 tal que p(x4 ) = p(x3 ) e, se p(x3 ) > p(x2 ) =
p(x1 ), então existex4 > x2 tal que p(x4 ) = p(x3 ). De qualquer forma x4 é distinto de x3 , como
queríamos.

Nível 3.2. A referida armação é consequência do item A) do parágrafo Aplicações da


0
fórmula de Taylor. Como α e β são raízes de p (um polinômio de segundo grau), temos
p0 (x) = 3(x − α)(x − β) = 3x2 − 3(α + β)x + 3αβ (já vimos que o coeciente líder de p0 é 3) e,
00 00
então, p (x) = 6x − 3(α + β) = 3(2x − (α + β)). Logo, p (α) = 3(2α − (α + β)) = 3(α − β) < 0,
pois α < β.

Segunda Parte: f é derivável ⇔ a2 < 3b. x ∈ R é da forma x = p(y) para


Todo ponto
algum y ∈ R, p é derivável em y e f é contínua em x. Assim, pelo Corolário do Teorema 3, f é
0 0
derivável em x = p(y) se, e somente se, p (y) 6= 0 e sabemos que p (z) 6= 0 para todo z ∈ R se,
2 2
e somente se, 4a − 12b < 0, ou seja, se, e somente se, a < 3b, como queríamos. 
8.7. EXERCÍCIO 7 287

Solução 2. Podemos também aplicar o Corolário 6 do Teorema 7. O ponto central aqui é


notar que não existe a possibilidade de p ser não decrescente sem ser crescente, isto é: p é não
decrescente se, e somente se, p é crescente (qualquer função crescente é não decrescente). Feito
isto, para completar esta primeira parte, só restará mostrar que p é injetiva se, e somente se, p
é crescente, pois p é um homeomorsmo se, e somente se, p é injetiva (ver o início do Nível 1
da Solução 1).

A função p é derivável no intervalo R. Temos

p0 (x) = 3x2 + 2ax + b > 0 ∀x ∈ R ⇔ 4a2 − 12b 6 0 ⇔ a2 6 3b

2
(4a −12b é o número que aparece dentro da raiz na fórmula que nos dá os zeros de um polinômio
do segundo grau), então, pelo Corolário 6 do Teorema 7, p é não decrescente se, e somente se,
a2 6 3b.
Se um polinômio não nulo q x1 < x2 tais que q(x1 ) > q(x2 ). Se
não é crescente, sejam
q(x1 ) > q(x2 ), então q não é não decrescente. Se q(x1 ) = q(x2 ) := k , seja x3 ∈ (x1 , x2 ) tal que
q(x3 ) 6= a (x3 existe porque o número de raízes do polinômio q(x) − k é nito). Se q(x3 ) > k ,
temos x3 < x2 com q(x3 ) > q(x2 ). Se q(x3 ) < k , temos x1 < x3 com q(x1 ) > q(x3 ). Em
qualquer um destes casos, q não é uma função não decrescente. Pela contrapositiva, como p é
não decrescente, segue então que p é crescente e, portanto, injetiva.

Reciprocamente, se p não é crescente, sejam x1 < x2 tais que p(x1 ) > p(x2 ). Se p(x1 ) =
p(x2 ), então p não é injetiva. Se p(x1 ) > p(x2 ), seja x3 > x2 tal que p(x3 ) > p(x1 ) (x3 existe
porque lim p(x) = +∞, de acordo com o Exercício 13 do Capítulo VI, página 217). Agora
x→+∞
temos p(x2 ) < p(x1 ) < p(x3 ) e, pelo Teorema do Valor Intermediário (Teorema 12 do Capítulo
VII), existe x4 ∈ (x2 , x3 ) tal que p(x4 ) = p(x1 ) (x4 6= x1 ). De qualquer forma, p não é injetiva.
2
Segue a tese: p é um homeomorsmo se, e somente se, a 6 3b. (Resumindo, o que zemos
foi o seguinte

p é um homeomorsmo ⇔p é injetiva ⇔p é crescente ⇔p é não decrescente ⇔ a2 6 3b.)

a2 < 3b, então p0 (x) > 0 para todo x ∈ R (pois a2 < 3b ⇒ (2a)2 − 4 · 3 · b < 0 ⇒
Se
3x2 + 2ax + b = p0 (x) > 0 ∀x ∈ R), portanto f é derivável em p[R] = R (novamente o Corolário
2 0
6 do Teorema 7). Se a = 3b, para x ∈ R tal que p (x) = 0 (ele existe), f não é derivável em
p(x) (Corolário do Teorema 3). Logo, f é derivável se, e somente se, a2 < 3b. .

8.7 Exercício 7
Seja f :I→R contínua no intervalo I. Se, para cada x ∈ I (exceto evidentemente a extremi-
dade superior, se ela estiver em I ), existir f+0 (x) e for > 0, então f é crescente.

Solução. Vamos trabalhar com a constrapositiva deste enunciado. Sejam a e b pontos em


I tais que a<b e f (a) > f (b). Então, com o auxílio do Teorema de Weierstrass (Corolário
do Teorema 14 do Capítulo VII), podemos armar que existe c ∈ [a, b) tal que f (c) > f (x)
∀x ∈ [a, b]. Em particular, dado δ > 0, existe x ∈ I ∩ (c, c + δ) tal que f (x) 6 f (c). Se f+0 (c)
0 0
não existe, então acabamos. Se f+ (c) existe, então, pelo Teorema 4, f+ (c) 6 0. Segue a tese.
288 CAPÍTULO 8. DERIVADAS

8.8 Exercício 8
Seja f : X → R derivável no ponto a ∈ X ∩ X 0 . Se (xn ) e (yn ) são sequências de pontos em X
tais que lim xn = lim yn = a e xn < a < yn para todo n ∈ N, prove que

f (yn ) − f (xn )
lim = f 0 (a).
n→∞ y n − xn

f (yn ) − f (xn ) f (yn ) − f (a) f (xn ) − f (a) yn − a


[ Sugestão : Note que = tn +(1−tn ) onde tn = .
y n − xn yn − a xn − a y n − xn
Observe que 0 6 tn 6 1 e use o Exercício 18 do Capítulo IV.]

(Voltar para a Solução do Exercício 11 do Capítulo IX: página 353.)

Solução. A sugestão dada já diz tudo. Pela observação da página 257, temos

f (xn ) − f (a) f (yn ) − f (a)


lim = lim = f 0 (a)
xn − a yn − a

yn − a
e, para tn = , temos 0 < tn < 1 porque yn − a > 0 e yn − xn = (yn − a) + (a − xn ) >
y n − xn
yn − a
yn − a > 0, uma vez que a − xn > 0, portanto 0< < 1. Além disto, como
y n − xn

(yn − xn ) − (yn − a) a − xn
1 − tn = = ,
y n − xn y n − xn

temos:

f (yn ) − f (a) f (xn ) − f (a) f (yn ) − f (a) a − xn f (xn ) − f (a)


tn + (1 − tn ) = + ·
yn − a xn − a y n − xn yn − xn xn − a
f (yn ) − f (a) f (a) − f (xn )
= +
y n − xn y n − xn
f (yn ) − f (xn )
= .
y n − xn

Agora o resultado segue do Exercício 18 do Capítulo IV (página 131). 

8.9 Exercício 9
Seja f :I→R contínua no ponto a interior ao intervalo I. Suponha que existe L∈R tal que

f (yn ) − f (xn )
lim =L
yn − xn

para todo par de sequências (xn ), (yn ) em I com xn < a < yn e lim xn = lim yn = a. Prove que
existef 0 (a) e é igual a L. Mostre que a hipótese de f ser contínua no ponto a é indispensável.
8.9. EXERCÍCIO 9 289

Solução. A observação central que nos conduz à solução deste exercício é a seguinte. Dado
y 6= a, se (xn )n∈N é uma sequência que converge para a, então

f (y) − f (xn ) f (y) − f (a)


−→ .
y − xn y−a
Isto pelo item 3 do Teorema 6 do Capítulo IV e do Teorema 4 do Capítulo VII, por causa da
continuidade de f em a e porque y − xn → y − a 6= 0. Essa convergência signica que, xado
y 6= a e xada a sequência (xn ) que converge para a, a diferença

f (y) − f (xn ) f (y) − f (a)



y − xn y−a
pode se tornar tão pequena quanto desejarmos.
0
Vamos mostrar que f+ (a) = L. A demonstração da igualdade f−0 (a) = L será análoga.
0
Sendo ambas essas derivadas laterais iguais a L seguirá a tese, f (a) = L.
Seja (yn ) uma sequência de pontos em I tal que yn > a para todo n e yn → a. Queremos
mostrar que
f (yn ) − f (a)
lim = L.
yn − a
Para tanto, vamos obter uma sequência (xn ) de pontos em I tal que xn < a para todo n, xn → a
e
f (yn ) − f (xn ) f (yn ) − f (a) 1

y n − xn − < ∀n ∈ N,
yn − a n
o que, pelos Teorema 8 do Capítulo IV e Exercício 6 do Capítulo IV (página 122) implica o
resultado desejado:
f (yn ) − f (a) f (yn ) − f (xn )
lim = lim = L.
yn − a y n − xn
Conforme já adiantamos no primeiro parágrafo, xada uma sequência (zm )m∈N em I tal
quezm < a e zm → a, sequência esta que existe porque a∈ int(I), para cada n ∈ N, existe
m(n) ∈ N tal que
f (yn ) − f (zm ) f (yn ) − f (a) 1

yn − zm − <
yn − a n
para todo natural m > m(n). Para cada n xamos então um natural m > max{n, m(n)} e
denimos xn = zm . A desigualdade m > n garante (xn )n∈N ser uma subsequência de (zm )m∈N
e, portanto, xn → a, pois zn → a por hipótese (Teorema 2, Capítulo IV), e m > m(n) implica

f (yn ) − f (xn ) f (yn ) − f (a) 1

y n − xn − < .
yn − a n
0
Logo, (xn )n∈N
é a sequência procurada que nos permite concluir que f+ (a) existe e é igual a L.
0
A demonstração de que f− (a) existe e é igual a L é análoga porque, dado x 6= a, se (yn )
converge para a, então

f (yn ) − f (x) f (a) − f (x) f (x) − f (a)


lim = = .
yn − x a−x x−a
Para mostrar que a hipótese de f ser contínua no ponto a é indispensável, consideremos a
função f : R → R denida por f (x) = x se x 6= 0 e f (0) = 1. O ponto 0 é interior a R, f não é
contínua em 0, mas, para quaisquer sequências (xn ) e (yn ) tais que xn < a < yn para todo n,
temos
f (yn ) − f (xn ) y n − xn
= = 1,
y n − xn y n − xn
embora f não seja derivável em 0. 
290 CAPÍTULO 8. DERIVADAS

8.10 Exercício 10
1
A função f : R → R, denida por f (x) = x2 sen , se x 6= 0, e f (0) = 0, é derivável. Obtenha
x
sequências (xn ), (yn ) tais que lim xn = lim yn = 0, xn 6= yn , mas não existe lim[f (yn ) −
f (xn )]/(yn − xn ).

Solução. Para x 6= 0, f é derivável em x pelos Teoremas 2 e 3. Para x = 0, temos

f (x) − f (0) 1
lim = lim x sen = 0
x→0 x−0 x→0 x
pelo Teorema 7 do Capítulo VI (produto de uma função que tende a zero por uma função
limitada).
1 1
Fazendo xn = , como sen = sen(2πn) = 0 para todo natural n, temos
2πn xn
f (yn ) − f (xn ) f (yn ) 2πn · f (yn )
= = .
yn − xn 1 2πn · yn − 1
yn −
2πn
1 2πm · f (ym ) 1 1
Se ym = , então =0 (para m 6= n). Para yn = π , como sen =
2πn 2πm · ym − 1 + 2πn yn
π  2
sen + 2πn = 1,
2
2πn 2πn
π 2 π
2πn · f (yn ) + 2πn + 2πn 2 2π
= 2 = 2 π =− · π ,
2πn · yn − 1 2πn − π + 2π
π −1 2 2n
+ 2πn
2
número este que tende para −2/π 6= 0 conforme
  n → +∞.
1
Assim, além da sequência (xn )n∈N = , denimos a sequência (yn )n∈N por y2n+1 =
2πn n∈N  
1 1 f (yn ) − f (xn )
e y2n = π para todo n ∈ N, de modo que a sequência =
2πn + 2π · (2n) y n − xn n∈N
 2
2πn · f (yn )
não é convergente, porque ela admite uma subsequência que converge para
2πn · yn − 1 n∈N
0 e uma subsequência que converge para −2/π (Teorema 2 do Capítulo IV). 

8.11 Exercício 11
Seja f : I → R derivável no intervalo aberto I . Um ponto crítico de f é um ponto c∈I tal
que f 0 (c) = 0. O ponto crítico c chama-se não-degenerado quando f 00 (c) existe e é diferente de
zero. Então

i) Se f C 1 , para cada intervalo compacto [a, b] ⊂ I


é de classe o conjunto dos pontos críticos
de f pertencentes a [a, b] é fechado.
8.11. EXERCÍCIO 11 291

1
ii) A função f : R → R, denida por f (x) = x2 sen + x se x 6= 0 e f (0) = 0, é derivável. O
x
conjunto dos pontos críticos de f no intervalo [0, 1] não é fechado.
iii) Os pontos de máximos e mínimos locais de f são críticos. Um ponto crítico não-
degenerado deve ser de máximo local ou de mínimo local.

iv) Existem funções C ∞ com máximos e mínimos locais isolados degenerados. Existem pontos

críticos (necessariamente degenerados) de funções C que não são máximos nem mínimos
locais.

v) Se c∈I é um ponto crítico não-degenerado para f então existe δ > 0 tal que não há outros
pontos críticos de f no intervalo (c − δ, c + δ). Ou seja: todo ponto crítico não-degenerado
é um ponto crítico isolado.

vi) Se os pontos críticos de f ∈ C1 [a, b] ⊂ I são todos não-


contidos no intervalo compacto
degenerados então há apenas um número nito deles. Conclua que f possui no máximo
uma innidade enumerável de pontos críticos não-degenerados em I .

1
vii) A função f : R → R, f (x) = x4 sen (x 6= 0), f (0) = 0, possui uma innidade de pontos
x
críticos não-degenerados no intervalo [0, 1]. Por que isto não contradiz o item (vi)?

Solução.

i) Seja X = {x ∈ [a, b]; f 0 (x) = 0} este conjunto. Vamos mostrar que seu complementar

R −X = (−∞, a) ∪ (b, +∞) ∪ {x ∈ [a, b]; f 0 (x) 6= 0}


é aberto.
c ∈ [a, b] é tal que f 0 (c) 6= 0, pela continuidade de f 0 e pelo fato de I ser aberto, sabemos
Se
0
que existe δ > 0 tal que J = (c − δ, c + δ) ⊂ I e f (x) 6= 0 para todo x ∈ J (Corolário do
Teorema 3 do Capítulo VII e observação feita abaixo dele). Este intervalo aberto J está contido
em R −X . Como (−∞, a) e (b, +∞) são abertos, segue que R −X é aberto, como queríamos.

ii) É derivável no 0 pois, para x 6= 0,


f (x) − f (0) 1
= x sen + 1 → 1,
x−0 x
se x→0 (Teorema 7 do Capítulo VI), de modo que f 0 (0) = 1 e, para x 6= 0,
1 1
f 0 (x) = 2x sen − cos + 1,
x x
cujo limite conforme x tende a 0 não existe. Portanto f não é de classe C 1; f não é contínua
em 0.
O 0 não é ponto crítico de f . Vamos então conseguir uma sequência (xn )n∈N de pontos em
[0, 1] tais que f 0 (xn ) = 0 e xn → 0, o que mostrará que o conjunto dos pontos críticos de f
pertencentes a [0, 1] não é fechado (página 170 do livro).
1 1
Para x = 1/(2π) temos sen = sen 2π = 0 e cos = cos 2π = 1. Então a sequência
x x
1
denida por xn = para todo n ∈ N serve para nossos propósitos, ela converge para 0
2π + 2πn
0
e f (xn ) = 0 para todo n, como queríamos. 
292 CAPÍTULO 8. DERIVADAS

iii) Lembrando que estamos lidando com uma função cujo domínio é aberto, um ponto de
máximo ou de mínimo local de f
é crítico pelo Corolário 2 do Teorema 4.
0 00 00
Se c é um ponto crítico não degenerado de f , então f (c) = 0 e f (c) 6= 0. Se f (c) > 0,
0 0 0
então existe δ > 0 tal que f (x) < f (c) < f (y) se c − δ < x < c < y < c + δ (Corolário 1
0 0
do Teorema 4), isto é, f (x) < 0 para todo x ∈ (c − δ, c) e f (x) > 0 para todo x ∈ (c, c + δ).
Isto implica f (c) < f (x) para todo x ∈ (c − δ, c + δ) distinto de c, ou seja, c é um ponto de
mínimo local estrito de f . De fato, se a ∈ (c − δ, c + δ) − {c} é tal que f (a) 6 f (c), então, pelo
0
Teorema 7 (o Teorema do Valor Médio): existe b ∈ (a, c) tal que f (b) > 0, se a < c, e; existe
b ∈ (c, a) tal que f 0 (b) 6 0, se a > c. Estas duas coisas contrariam os fatos de a derivada de f
ser negativa em (c − δ, c) e positiva em (c, c + δ).
00
De modo análogo, se f (c) < 0, então c é um ponto de máximo local estrito de f . Segue a
tese. 

iv) Por um lado, o conceito de ponto isolado é relativo; assim, ao se falar em ponto
isolado, uma pergunta que se impõe naturalmente é: isolado de qual conjunto? Por outro,
não há em lugar algum do livro a denição de máximo e mínimo local isolado, mas sim a de
máximo local estrito e a de mínimo local estrito (página 264 do livro). Portanto, por máximos
e mínimos locais isolados degenerados vamos entender máximos e mínimos locais estritos
degenerados.

Um exemplo de função C com mínimo local estrito degenerado é a f : R → R denida
4
por f (x) = x para todo x real. O 0 é um ponto de mínimo local (de fato global) estrito e
00 2 3
degenerado, f (0) = 12 · 0 = 0. Para a função g : R → R denida por g(x) = x , 0 é um ponto
crítico degenerado que não é nem de máximo nem de mínimo local. 

v) A demonstração disto já está embutida na solução do item iii) acima. Um ponto crítico
não degenerado é um ponto isolado do conjunto de todos os pontos críticos. 

vi) X = {x ∈ [a, b]; f 0 (x) = 0} é fechado. Limitado


Pelo item i), sabemos que o conjunto
00
e fechado (portanto compacto). Agora, pelo item anterior, como f (x) 6= 0 para todo x ∈ X ,
temos também que todo ponto de X é isolado de X . Para cada x ∈ X , seja então Ix o intervalo
aberto possuindo x tal que Ix ∩ X = {x}. Como (Ix )x∈X é uma cobertura aberta de X , seja
[
F ⊂ X um conjunto nito tal que X ⊂ Ix (Teorema 11 do Capítulo V). Disto segue X ⊂ F
x∈F
e, por conseguinte, X = F (pois já tínhamos F ⊂ X ), ou seja, X é nito, como queremos. De
fato, se x ∈ X , seja y ∈ F tal que x ∈ Iy , de modo que x ∈ X ∩ Iy = {y}, logo x = y e, então,
x ∈ F, como queríamos.
(Generalização: se todos os pontos de um conjunto compacto K são isolados de K, então
K é nito.)
No conjunto dos pontos críticos não degenerados todo ponto é isolado. Portanto tal conjunto
é enumerável (Corolário 2 do Teorema 8, Capítulo V). 

vii) Note que f é de classe C 1. O que acontece é que neste intervalo existe um ponto
crítico degenerado:
1
f 0 (0) = lim x3 sen =0
x→0 x
e
1 1
f 0 (x) = 4x3 sen − x2 cos se x 6= 0,
x x
8.11. EXERCÍCIO 11 293

o que implica
1 1
f 00 (0) = lim 4x2 sen − x cos = 0
x→0 x x
(Teorema 7 do Capítulo VI). Logo 0, que pertence ao intervalo [0,1], é um ponto crítico dege-
nerado.
Vejamos agora que existem innitos pontos críticos não degenerados de f em [0, 1]. Para
0 0
estimar onde uma raiz de f (os pontos críticos de f são as raízes de f , por denição) está,
0 0
podemos encontrar um ponto no qual f é negativa e um ponto no qual f é positiva e, então,
0
haverá uma raiz de f entre eles (por algum dos teoremas do Valor Intermediário, o para funções
0
contínuas, Teorema 12 do Capítulo VII, porque f é contínua, ou o para derivadas, Teorema
5). Não precisamos encontrar estas raízes, pois o que queremos fazer neste primeiro momento
é mostrar que existem innitas delas.
1 1 1 1 1 1
Se = 2πn e = π + 2πn, então sen = sen = 0, cos = 1 e cos = −1,
xn yn xn yn xn yn
0 2 0 2
portanto f (xn ) = −xn < 0 e f (yn ) = yn > 0. Assim, para cada n ∈ N, seja zn ∈ (yn , xn ) tal
0
que f (zn ) = 0. Deste modo, {z1 , z2 , . . .} é um conjunto innito de pontos críticos de f , todos
eles pertencentes ao intervalo [0, 1].
00
Agora devemos lidar com a f . Para x 6= 0, temos

1 1 1 1
f 00 (x) = 12x2 sen − 4x cos − 2x cos − sen
x x x x
1 1 1
= 12x2 sen − 6x cos − sen .
x x x
 
0 3 1 2 1 2 1 1 1 1
Se x 6= 0 e se f (x) = 4x sen −x cos = x 4x sen − cos = 0, então 4x sen −cos =
x x x x x x
0, parcela esta que podemos encontrar na expressão de f 00 (x) do seguinte modo:
 
00 2 1 1 1 1
f (x) = 24x sen − 6x cos − 12x2 sen − sen
x x x x
 
1 1 1 1
= 6x 4x sen − cos − 12x2 sen − sen
x x x x
1
= −(1 + 12x2 ) sen ,
x
0 00
se f (x) = 0 para x 6= 0. Portanto, dado um ponto crítico x 6= 0 de f , f (x) = 0 se, e somente
1
se, x = para algum n ∈ N. Ora, nenhum ponto do conjunto {z1 , z2 , . . .}, de pontos críticos
πn
no intervalo [0, 1], é desta forma, ou seja, todos eles são não degenerados. Mais do que isto,
todo ponto crítico de f com exceção do zero é não degenerado.
O fato de f possuir uma innidade de pontos críticos não degenerados no intervalo [0, 1]
não contradiz o item anterior porque aquele item não armava nada sobre o caso de f possuir
um ponto crítico degenerado em [0, 1]. Ele tratava apenas do caso de todos os pontos críticos
de um intervalo compacto serem não degenerados. 

Observação. Um outro modo de encontrar o número zero na expressão de f 00 (x), caso f 0 (x) =
1 1 1 1
0, é o seguinte: 4x sen − cos = 0 ⇒ 12x2 sen − 3x cos = 0, então escrevemos
x x x x
 
1 1 1 1 1 1
f 00 (x) = 12x2 sen − 3x cos − 3x cos − sen = −3x cos − sen ,
x x x x x x
mas isto tornaria o caminho um pouco mais complicado do que o escolhido acima.
294 CAPÍTULO 8. DERIVADAS

8.12 Exercício 12
Seja p : R → R um polinômio. Tem-se p(a) = p0 (a) = · · · = p(k) (a) = 0 se, e somente se,
p(x) = (x − a)k+1 · q(x), onde q é um polinômio.

Solução. Vamos começar mostrando para a = 0 e usar isto para mostrar o caso geral.
n
Seja p(x) = b0 + b1 x + · · · + bn x . Conforme já foi observado na página 280, temos

p(i) (0)
bi = ∀i ∈ {0, 1, . . . , n}.
i!

Claramente esta igualdade também é válida para todo i maior do que n (considerando bi = 0
se i > n). Assim, sendo I = {0, 1, . . . , k},

p(i) (0) = 0 ∀i ∈ I ⇔ bi = 0 ∀i ∈ I ⇔ p(x) = xk+1 · q(x),

onde q é um polinômio. Portanto a armação é verdadeira para a = 0.


De modo geral, xado a ∈ R, vamos denir um polinômio p1 tal que p(a) = 0 ⇔ p1 (0) =
0, um polinômio para o qual podemos aplicar o resultado que acabamos de mostrar. Esse
polinômio é denido por

p1 (x) = p(x + a).


(0)
Além de p1 (x) = p(0) (x + a)
(que é o que a igualdade acima está dizendo), pela Regra da
(i) (i)
Cadeia (Teorema 3) e pelo Princípio da Indução, temos p1 (x) = p (x + a) para todo natural
i, donde
(i)
p1 (0) = p(i) (a),
de modo que, para I = {0, 1, . . . , k}, pelo que zemos no segundo parágrafo,

(i)
p(i) (a) = 0 ∀i ∈ I ⇔ p1 (0) = 0 ∀i ∈ I
⇔ p1 (x) = xk+1 q1 (x)
⇔ p(x) = p1 (x − a) = (x − a)k+1 q1 (x − a)

(se p1 (x) = p(x + a) ∀x ∈ R, então, dado x ∈ R, p1 (x − a) = p((x − a) + a) = p(x), e,


reciprocamente, se p(x) = p1 (x − a) ∀x ∈ R, então, dado x ∈ R, p(x + a) = p1 ((x + a) − a) =
p1 (x); se p1 (x − a) = (x − a)k+1 q1 (x − a) ∀x ∈ R, então, dado x ∈ R, p1 (x) = p1 ((x +
a) − a) = ((x + a) − a)k+1 q1 ((x + a) − a) = xk+1 q1 (x)), onde q1 é um polinômio e, portanto,
q1 (x − a) de fato dene um polinômio (e, reciprocamente, se p(x) = (x − a)k+1 q(x), então
p(x) = (x − a)k+1 q((x − a) + a) = (x − a)k+1 q1 (x − a) se q(x + a) = q1 (x), onde q(x + a) dene
um polinômio se q é um polinômio, além do que: q(x) = q1 (x − a) ⇔ q1 (x) = q(x + a)), como
queríamos. 

8.13 Exercício 13
Seja f : I → R denida num intervalo I . Se existe α > 1 tal que |f (x) − f (y)| 6 |x − y|α
para quaisquer x, y ∈ I então f é contínua e possui derivada nula em todos os pontos de I .
Consequentemente, f é constante.
8.14. EXERCÍCIO 14 295

Solução 1. Fixado a ∈ I , vamos mostrar que f 0 (a) existe e é igual a zero. Existindo f 0 (a), a
continuidade de f em a segue pelo Teorema 1. Para β = α − 1 > 0, temos

f (x) − f (a)
0 6 6 |x − a|β
x−a

para todo x∈I diferente de a. Como lim |x − a|β = 0 (x ∈ I , 0 < |x − a| < β
ε ⇒ |x − a|β < ε
x→a
∀ε > 0), pelo Teorema 4 do Capítulo VI,

f (x) − f (a)
lim = 0,
x→a x−a
donde
f (x) − f (a)
lim = 0,
x→a x−a
0
pois ||A| − 0| = |A − 0|, qualquer que seja o número real A. Logo, f (a) existe e é igual a 0.
Se f não fosse constante, sejam x < y em I tais que f (x) 6= f (y). Como f é contínua em
[x, y] e derivável em (x, y), pelo Teorema do Valor Médio (Teorema 7), existe z ∈ (x, y) tal que
f (y) − f (x)
f 0 (z) = 0 0
; neste z ∈ I a f não é nula. Portanto, pela contrapositiva, f (z) = 0 para
y−x
0
todo z ∈ I implica f constante, como queríamos. 

Observação. Também podemos concluir que f é constante através do Corolário 1 do Teorema


7. É isto que será feito na solução do Exercício 14.

Solução 2 (para a continuidade). A desigualdade envolvendo f nos lembra a desigualdade


da denição de função lipschitziana, e vimos que estas são uniformemente contínuas. De modo
semelhante, vamos mostrar que f é uniformemente contínua. Fixado ε > 0, temos

|x − y| < α ε ⇒ |x − y|α < ε,

pois a associação x 7→ xα é crescente. Assim, para δ =
α
ε, temos
x, y ∈ I, |x − y| < δ ⇒ |f (x) − f (y)| < ε,
pois |f (x) − f (y)| 6 |x − y|α < ε. Logo, f é uniformemente contínua, como queríamos. (Aqui
bastou α ser positivo.) 

8.14 Exercício 14
Seja f :I →R derivável num intervalo arbitrário I. Se f 0 (x) = 0 para todo x∈I então f é
constante.

Solução. No último parágrafo da Solução 1 do exercício anterior, justicamos isso pela con-
trapositiva. Aqui vamos dar uma demonstração direta, aplicando o Corolário 1 do Teorema 7.
Para tanto, xemos um ponto x0 ∈ I f (x0 ) = k . Dado x ∈ I , queremos concluir que
e seja
f (x) = k . Para x = x0 não há nada a se fazer. Para x 6= x0 , sejam os pontos a = min{x, x0 }
e b = max{x, x0 }. A restrição f |[a, b] : [a, b] → R é uma função contínua e é derivável no
0
intervalo (a, b), com f (y) = 0 para todo y ∈ (a, b) (Teorema 1 do Capítulo VII e observação da
página 259), de modo que, pelo Corolário 1 do Teorema 7, f (y) = k para todo y ∈ [a, b]. Em
particular, f (x) = k , como queríamos. 
296 CAPÍTULO 8. DERIVADAS

8.15 Exercício 15
Seja f :I →R de classe C1 a ∈ I , considere sequências (xn ) e (yn )
num intervalo I. Dado
f (xn ) − f (yn )
em I , com xn 6= yn e lim xn = lim yn = a. Prove que lim = f 0 (a). Note que a
xn − y n
1
hipótese f ∈ C dispensa a exigência de se ter xn < a < yn como no Exercício 8 acima.

Solução. Pelo Teorema 7, do Valor Médio, para cada n ∈ N, podemos escolher zn ∈ R entre
f (xn ) − f (yn )
xn e yn tal que f 0 (zn ) = . Pelo Teorema 8 do Capítulo IV (com as adaptações
xn − y n
0
evidentes), a sequência (zn )n∈N também converge para a. Agora, pela continuidade de f , segue
0 0
lim f (zn ) = f (a) (Teorema 4, Capítulo VII), donde segue a tese. 

8.16 Exercício 16
Seja f :I→R derivável no intervalo I . Para um certo a ∈ I , suponha que lim xn = lim yn = a
f (yn ) − f (xn )
com xn 6= yn em I implique lim = f 0 (a). Prove que f 0 é contínua no ponto a.
y n − xn

Solução. Dada uma sequência (zn )n∈N em I cujo limite é a, pelo Teorema 4 do Capítulo VII,
mostremos que
lim[f 0 (zn ) − f 0 (a)] = 0.
Para podermos fazer algo, introduzamos aqui um número do qual o exercício nos traz alguma
informação e escrever
   
0 0 0 f (xn ) − f (yn ) f (xn ) − f (yn ) 0
f (zn ) − f (a) = f (zn ) − + − f (a) ,
xn − y n xn − y n
onde (xn )n∈N e (yn )n∈N são sequências em I, com xn 6= yn para todo n. Se ambas essas
sequências convergirem para a, então, por hipótese, a segunda parcela do lado direito desta
igualdade converge para zero. Se também tivermos

f (xn ) − f (yn )
lim = f 0 (zn ),
xn − y n
então seguirá o nosso resultado desejado. Vamos então obter tais sequências.
Observando como o último limite acima se assemelha com a denição de f 0 (zn ), comecemos
denindo yn = zn para todo n ∈ N, só faltando agora obtermos (xn )n∈N .
f (x) − f (zn )
De lim = f 0 (zn ), para cada n ∈ N, podemos encontrar um x próximo o
x→zn x − zn
suciente de zn de modo a tornar o número

f (x) − f (zn ) 0


x − zn − f (zn )

tão pequeno quanto desejarmos. Para este pequeno, escolhamos o número 1/n  o fato de
a sequência (1/n)n∈N convergir para 0 vai forçar o módulo acima também convergir para 0 .
Assim, pela denição de limite, seja δn > 0 tal que

f (x) − f (zn ) 0
1
x ∈ I ∩ (zn − δn , zn + δn ), x 6= zn =⇒
− f (zn ) < .
x − zn n
8.17. EXERCÍCIO 17 297

Precisamos garantir que a sequência dos xn 's tenda


  para cada n ∈ N
para a. Então,
δn δn
escolhamos xn ∈ I tal que xn pertença ao intervalo zn − , zn + e seja distinto de zn .
n n
Além disso, vamos supor também 0 < δn < 1. Deste modo de fato temos lim xn = a, porque

δn δn
zn − < xn < zn + ∀n ∈ N
n n
e    
δn δn
lim zn − = lim zn + = a,
n n
teoremas 5, 6 e 8 do Capítulo IV. Este teorema, o 8, também nos permite concluir que
 
f (xn ) − f (zn ) 0
lim − f (zn ) = 0,
xn − zn

f (xn ) − f (zn ) 0
1
porque
xn − zn − f (zn ) < para todo n por construção, uma vez que xn ∈ I , x n ∈
n
(zn − δn , zn + δn ), pois δn /n 6 δn , e xn 6= zn . Segue a tese. 

8.17 Exercício 17
Seja f : (c, +∞) → R derivável. Se existem lim f (x) = a e lim f 0 (x) = b com a ∈ R, então
x→+∞ x→+∞
b = 0.
[ Sugestão : f (n + 1) − f (n) = f 0 (xn ) onde xn → ∞].
(Voltar para a Solução 2 do Exercício 28: página 307.)

Solução. Seguindo a sugestão, pelo Teorema do Valor Médio (Teorema 7), para cada n ∈ N
seja xn ∈ (n, n + 1) tal que f 0 (xn ) = f (n + 1) − f (n)  a rigor, para cada n sucientemente
grande, o que pode ser feito pela Propriedade Arquimediana; existe n0 ∈ N tal que n>c para
todo n > n0 . Portanto:

b = lim f 0 (x) = lim f 0 (xn ) = lim[f (n + 1) − f (n)] = 0


x→+∞

(página 208 do livro junto com o Teorema 6 do Capítulo VI), uma vez que lim f (n + 1) =
lim f (n) = a (a ∈ R). 

8.18 Exercício 18
Seja f : [a, b] → R contínua, derivável em (a, b). Suponha f (a) = f (b) = 0. Então, dado
arbitrariamente k ∈ R, existe c ∈ (a, b) tal que f 0 (c) = k · f (c). [Sugestão : Tome p(x) =
f (x) · e−kx e aplique o Teorema de Rolle].

Solução. A função p : [a, b] → R da sugestão, denida por p(x) = f (x) · e−kx para todo
x ∈ [a, b], é contínua e é derivável em (a, b) e, junto com p(a) = p(b), podemos aplicar o
0
Teorema de Rolle: existe c ∈ (a, b) tal que p (c) = 0, onde

p0 (x) = f 0 (x)e−kx − kf (x)e−kx ,


298 CAPÍTULO 8. DERIVADAS

donde, uma vez que e−kx 6= 0 para todo x ∈ R,

f 0 (c)e−kc − kf (c)e−kc = 0 ⇒ f 0 (c) − kf (c) = 0 ⇒ f 0 (c) = kf (c),

como queríamos. 

8.19 Exercício 19
Seja f : I → R derivável num intervalo. Uma raiz
de f é um ponto c ∈ I tal que f (c) = 0.
0
Entre duas raízes consecutivas de f existe no máximo uma raiz de f . Use este fato para mostrar
3 2
que o polinômio p(x) = x − 6x + 9x − 1 possui exatamente uma raiz no intervalo (1, 3).

Solução.

Primeira Parte. Se f não tem raiz ou se f tem apenas uma raiz, então não há nada a se
fazer. Se f
tem duas raízes, sejam elas a e b. Então, como f (a) = f (b), pelo Teorema de Rolle
0
(Teorema 6), existe uma raiz de f no intervalo (a, b). E é disto que segue o resultado desejado:
0
se α e β são duas raízes de f e se a e b são duas raízes de f entre α e β , então a existência do
c ∈ (a, b) tal que f 0 (c) = 0 faz com que α e β não sejam raízes consecutivas, o que conclui esta
demonstração pela contrapositiva.

Segunda Parte. Temos

p0 (x) = 3x2 − 12x + 9 = 3(x2 − 4x + 3) = 3(x − 1)(x − 3).

Portanto, 1 e 3 são duas raízes consecutivas de p0 , donde, pela Primeira Parte, podemos concluir
que existe no máximo uma raiz de p entre 1 e 3. Existe pelo menos uma, pelo Teorema do
Valor Intermediário, porque

p(1) = 1 − 6 + 9 − 1 = 3 > 0 e p(3) = 27 − 54 + 27 − 1 = −1 < 0.

Logo, existe exatamente uma raiz de p no intervalo (1, 3). 

8.20 Exercício 20
Se f : [0, +∞) → R é derivável e lim f 0 (x) = L então, para cada c > 0, lim [f (x+c)−f (x)] =
x→+∞ x→+∞
f (x)
c·L e lim = L.
x→+∞ x

Solução. Pelo Teorema 6 do Capítulo VI (junto com uma observação feita na página 208 do
livro), xado c>0 e xada uma sequência xn → +∞, vamos mostrar que

lim[f (xn + c) − f (xn )] = c · L.

A ideia aqui é a mesma da Solução do Exercício 17. Para cada n ∈ N, pelo Teorema do
0 f (x n + c) − f (xn )
Valor Médio (Teorema 7), seja yn ∈ (xn , xn +c) tal que f (yn ) = . Claramente
c
yn → +∞ de modo que
lim f 0 (yn ) = L,
8.21. EXERCÍCIO 21 299

novamente pela junção do Teorema 6 do Capítulo VI com a referida observação da página 208
0
do livro, uma vez que lim f (x) = L por hipótese. Portanto
x→+∞

f (xn + c) − f (xn )
lim = L,
c
o que implica
lim[f (xn + c) − f (xn )] = c · L,
como queríamos.
f (x)
A conclusão de que lim = L segue do Exercício 22 do Capítulo VI (página 225);
x→+∞ x
f é uma função limitada em cada intervalo limitado pelo Teorema 1 (deste capítulo) e pelo
Teorema 14 do Capítulo VII (f é limitada num intervalo (a, b) porque o é no intervalo [a, b]) e,
para c = 1, lim [f (x + 1) − f (x)] = L. 
x→+∞

8.21 Exercício 21
f (c) − f (a)
Seja f : [a, b] → R derivável. Se f 0 (a) = f 0 (b), mostre que existe c ∈ (a, b) tal que =
c−a
f 0 (c). Interprete este fato geometricamente. [Sugestão : Considere primeiro o caso em que
f (x) − f (a)
f 0 (a) = f 0 (b) = 0. Mostre que a função ξ : [a, b] → R, dada por ξ(x) = se x 6= a e
x−a
ξ(a) = 0, atinge seu máximo ou seu mínimo num ponto interior c ∈ (a, b). Para passar ao caso
geral, considere a função auxiliar g(x) = f (x) − x · f 0 (a).]

0 0
Solução. Seguindo a sugestão vamos começar supondo f (a) = f (b) = 0. Se c ∈ (a, b) é um
0
ponto de máximo ou de mínimo de ξ , então ξ (c) = 0 (Corolário 2 do Teorema 4; ξ é derivável
em c pelo Teorema 2). Para x 6= a temos

f 0 (x)(x − a) + f (a) − f (x)


ξ 0 (x) = ,
(x − a)2

de modo que

f (c) − f (a)
ξ 0 (c) = 0 ⇒ f 0 (c)(c − a) + f (a) − f (c) = 0 ⇒ f 0 (c) =
c−a
(poisc 6= a), como queremos. Vejamos então que um tal ponto c existe.
A ξ de fato atinge seus mínimo e máximo em [a, b] por causa do Teorema de Weierstrass
(Corolário do Teorema 14 do Capítulo VII); ξ é contínua em (a, b] porque é derivável aí (Teorema
1 + Teorema 2) e é contínua em a porque

lim ξ(x) = f 0 (a) = 0 = ξ(a).


x→a

Mas só isso não basta, queremos esses mínimo ou máximo no interior de [a, b].
Como acontecia na demonstração do Teorema 6, se ξ(b) = ξ(a) e o máximo e o mínimo de
ξ são ambos atingidos nos extremos do intervalo [a, b], então ξ é constante. De ξ(x) = α para
todo x ∈ [a, b] e para algum α∈R segue

f (x) = α · (x − a) + f (a) ∀x ∈ (a, b],


300 CAPÍTULO 8. DERIVADAS

donde vem f 0 (x) = α x ∈ (a, b] e, daí, α = 0, porque f 0 (b) = 0 por hipótese (outro
para todo
caminho que nos leva a concluir que α = 0 começa com ξ(b) = ξ(a) ⇒ ξ(b) = 0 ⇒ f (b) = f (a)).
Portanto, neste caso, f (x) = f (a) para todo x ∈ [a, b] e qualquer c ∈ (a, b) serve para os nossos
própositos. Caso contrário, isto é, caso um dos dois (o máximo ou o mínimo) não é atingido
nos extremos do intervalo [a, b], então ele é atingido no interior de (a, b), como queríamos.
Se ξ(b) 6= ξ(a), então ξ(b) > 0 ou ξ(b) < 0 (lembrando que ξ(a) = 0). Lembrando também
que
f (b) − f (a)
ξ(b) =
b−a
e

0 f 0 (b)(b − a) + f (a) − f (b) f (a) − f (b)


ξ (b) = 2
= ,
(b − a) (b − a)2
notamos que ξ(b) e ξ 0 (b) têm sinais opostos. Deste modo, pelo Teorema 4 (mais precisamente,
pela sua observação), se ξ(b) > 0, então existe x ∈ (a, b) tal que ξ(x) > ξ(b) e, se ξ(b) < 0,
então existe x ∈ (a, b) tal que ξ(x) < ξ(b). De qualquer forma, se ξ(b) 6= ξ(a), então o mínimo
ou o máximo de ξ é atingido no intervalo (a, b), como queríamos.
No caso geral, ainda seguindo a sugestão dada, consideramos a função denida por g(x) =
f (x) − xf 0 (a) para todo x ∈ [a, b]. Sua derivada é dada por g 0 (x) = f 0 (x) − f 0 (a), o que
0 0
implica g (a) = 0 e g (b) = 0, de modo que podemos aplicar para a função g o que acabamos

0 g(c) − g(a)
de demonstrar acima. Para c ∈ (a, b) tal que g (c) = , temos
c−a

f 0 (c) − f 0 (a) = g 0 (c)


f (c) − cf 0 (a) − (f (a) − af 0 (a))
=
c−a
f (c) − f (a) f 0 (a)(a − c)
= +
c−a c−a
f (c) − f (a)
= − f 0 (a),
c−a

ou seja,
f (c) − f (a)
f 0 (c) = ,
c−a
como queríamos.
f (c) − f (a)
Geometricamente, se c ∈ (a, b) é tal que f 0 (c) =, então a reta determinada pelos
c−a
pontos (a, f (a)) e (c, f (c)) coincide com a tangente ao gráco de f pelo ponto (c, f (c)), elas são
paralelas por hipótese e ambas possuem o ponto (c, f (c)), logo, de fato, são coincidentes. 

8.22 Exercício 22
Seja f : [0, +∞) → R duas vezes derivável. Se f 00 é limitada e existe lim f (x), então
x→+∞
lim f 0 (x) = 0.
x→+∞

Solução.
8.22. EXERCÍCIO 22 301

No elevador. Como uma primeira tentativa, podemos procurar relacionar todas as fun-
ções f , f 0 e f 00 dentro de uma mesma fórmula, pois, assim, talvez possamos tirar conclusões
0 00
sobre f a partir das informações acerca de f e f . Uma forma de se fazer isso é aplicando
o Teorema do Valor Médio duas vezes, considerando-se uma sequência (xn )n∈N em [0, +∞)
estritamente crescente.
Para cada n ∈ N, sejam yn ∈ (xn , xn+1 ) e zn ∈ (xn , yn ) tais que

f (xn+1 ) − f (xn ) f 0 (yn ) − f 0 (xn )


f 0 (yn ) = e f 00 (zn ) = .
xn+1 − xn y n − xn

Comparando a primeira igualdade com f 0 (yn ) = f 00 (zn )(yn − xn ) + f 0 (xn ), obtida da segunda,
vem
f (xn+1 ) − f (xn )
f 0 (xn ) = − f 00 (zn )(yn − xn ).
xn+1 − xn
Poderíamos, em vez de xn+1 e xn , considerar xn e xn + 1 , obtendo

f 0 (xn ) = [f (xn + 1) − f (xn )] − f 00 (zn )(yn − xn ),

para yn ∈ (xn , xn + 1) e zn ∈ (xn , yn ). Nesta igualdade, conforme n → +∞, até sabemos que
f (xn +1)−f (xn ) converge para zero, por causa da existência do limite lim f (x), mas e quanto
x→+∞
a f 00 (zn )(yn − xn )? 00
A sequência (f (zn ))n∈N é limitada, de modo que, se yn − xn → 0, então
00
poderíamos concluir que f (zn )(yn − xn ) também tende a zero. Podemos então nos perguntar:
conforme n yn escolhido de forma a satisfazer o Teorema do Valor Médio  para f
cresce, o
aplicada em xn e em xn + 1  se aproxima cada vez mais de xn ? Explorando a função denida
por 1/x (ou, como queira, 1/(x + 1), para termos ela denida no zero), para a qual podemos
encontrar explicitamente quem é yn , vemos que a resposta para esta pergunta é negativa, para
ela temos yn − xn → 1/2 (!).
0 00
Uma outra forma de relacionarmos as funções f , f e f dentro de uma equação só é através
da Fórmula de Taylor com Resto de Lagrange. Por tal teorema, para cada a > 0, xado h > 0,
temos
f (a + h) − f (a) f 00 (a + θh)
f 0 (a) = − h
h 2
para algum θ ∈ (0, 1). Aqui também nos esbarramos num problema: xado h, a primeira parcela
acima até converge a zero conforme a cresce, porém não conseguimos dizer se a segunda assim
também o faz; ela tem este comportamento se h tende a zero, mas, se h → 0, então já não
f (a + h) − f (a)
sabemos se continua tendendo a zero.
h
Numa terceira tentativa, notamos que o fato de f 00 ser limitada implica f0 lipschitziana, pelo
mesmo motivo da validade do Corolário 3 do Teorema 7. Lipschitziana, portanto uniformemente
contínua. Podemos então nos perguntar: o resultado do Corolário do Teorema 16 do Capítulo
VII (o qual arma a existência de limites de funções uniformemente contínuas quando x tende
para pontos de acumulação) continua válido para x → +∞? Assim, com a existência de
0 0
ambos os limites lim f (x) e lim f (x), teremos lim f (x) = 0 pelo Exercício 17. Porém a
x→+∞ x→+∞ x→+∞
demonstração deste corolário passa longe de ser verdade para +∞; uma sequência que tende
para +∞ não pode ser de Cauchy...
Envolvendo este fato de f 0 ser uniformemente contínua temos a demonstração do Teorema
8; pela primeira parte dela podemos concluir o seguinte

f0 uniformemente contínua =⇒ f uniformemente derivável


302 CAPÍTULO 8. DERIVADAS

sem a necessidade de o domínio de f ser compacto (no Teorema 8 foi a hipótese de esse domínio
0
ser compacto que induziu f ser uniformemente contínua). Finalmente é esta observação que
nos leva à solução deste exercício!

(Voltar para a Observação 5 do Exercício 51 do Capítulo X: página 456.)

Nível 1. Dado ε > 0, queremos encontrar A>0 tal que

x > A ⇒ |f 0 (x)| < ε .

Nível 2. Dado ε > 0, existe uma função g : [0, +∞) → R tal que |f 0 (x) − g(x)| < ε
∀x ∈ [0, +∞) e lim g(x) = 0 (a justicativa desta existência está no Nível 3). Assim, com o
x→+∞
auxílio destas funções, escrevemos f 0 (x) = f 0 (x) − g(x) + g(x), donde

|f 0 (x)| 6 |f 0 (x) − g(x)| + |g(x)| ∀x ∈ [0, +∞).


Duas parcelas cuja soma queremos limitar. Deste modo, xado o ε > 0 do Nível 1, escolhemos
a funçãog tal que |f 0 (x) − g(x)| < ε /2, para todo x, e um número A > 0 tal que x > A ⇒
|g(x)| < ε /2. Para esta g temos

ε
|f 0 (x)| 6 |f 0 (x) − g(x)| + |g(x)| < + |g(x)|
2
e x>A implica
ε ε
|f 0 (x)| < + = ε,
2 2
como queríamos!

Nível 3. Conforme já observamos anteriormente, no elevador, f é uniformemente deri-


00 0 0
vável (f limitada ⇒ f lipschitziana, pela mesma justicativa do Corolário 3 do Teorema 7; f
0
lipschitziana ⇒ f uniformemente contínua ⇒ f uniformemente derivável, pela primeira parte
da demonstração do Teorema 8). Assim, dado ε > 0, existe h>0 tal que

f (a + h) − f (a) 0

− f (a) <ε ∀a ∈ [0, +∞].
h

Para este h denimos a função g : [0, +∞) → R colocando

f (x + h) − f (x)
g(x) = ∀x ∈ [0, +∞).
h
Esta é a função procurada: lim g(x) = 0, pois lim f (x + h) = lim f (x), já que h é um
x→+∞ x→+∞ x→+∞
número xado a priori. 

Observação. Num certo sentido, o Nível 2 acima nos diz que o conjunto das funções g :
[0, +∞) → R lim g(x) = 0 é fechado; fechado entre aspas porque a denição de
tais que
x→+∞
conjunto fechado vista foi feita para conjuntos de números reais, não de funções.

8.23 Exercício 23
Sejam
 b] → R contínuas e deriváveis
f, g : [a, no intervalo (a, b). Então existe c ∈ (a, b), tal que
f (b) − f (a) g 0 (c) = g(b) − g(a) f 0 (c).

8.24. EXERCÍCIO 24 303

Solução. c ∈ (a, b) tal que


Queremos

f (b) − f (a) g 0 (c) − g(b) − g(a) f 0 (c) = 0.


   
  
Consideremos então a função h : [a, b] → R denida por h(x) = f (b) − f (a) g(x) − g(b) −
g(a) f (x) para todo x ∈ [a, b]. Queremos c ∈ (a, b) tal que h0 (c) = 0. Para a função h temos

   
h(b) = f (b) − f (a) g(b) − g(b) − g(a) f (b) = −f (a)g(b) + g(a)f (b)
e
   
h(a) = f (b) − f (a) g(a) − g(b) − g(a) f (a) = f (b)g(a) − g(b)f (a).
Portanto h(b) = h(a). Assim, pelo Teorema de Rolle (Teorema 6), existe c ∈ (a, b) tal que
0
h (c) = 0, como queríamos. 

8.24 Exercício 24
Uma função f : I → R, derivável num intervalo I , satisfaz à condição de Lipschitz |f (x) −
f (y)| 6 c|x − y|, para x, y ∈ I quaisquer, se, e somente se, |f 0 (x)| 6 c para todo x ∈ I .
(Voltar para a Solução do Exercício 30: página 308.)

(Voltar para a Observação do Exercício 9 do Capítulo IX: página 351.)

(Voltar para a Solução do Exercício 11 do Capítulo IX, para o Nível 3.2: página 357.)

(Voltar para a Solução 1 do Exercício 25 do Capítulo IX: página 379.)

(Voltar para a Solução do Exercício 30 do Capítulo IX, item 5: página 388.)

(Voltar para o Exercício 40 do Capítulo IX, para a Solução de seu Lema: página 400.)

Solução. Como na justicativa do Corolário 3 do Teorema 7, dados x, y ∈ I , existe z entre


x e y tal quef (x) − f (y) = f 0 (z)(x − y). Isto pelo Teorema 7 do Valor Médio, porque f é
contínua no intervalo fechado de extremidades xey e é derivável em seu interior. Se a derivada
de f é limitada pelo número real c, então

|f (x) − f (y)| = |f 0 (z)| · |x − y| 6 c|x − y|,


o que conclui a volta deste exercício.
f (x) − f (y)
Com relação à ida, xado x ∈ I, se −c 6 6c para todo y∈I diferente de x,
x−y
então o limite
f (x) − f (y)
f 0 (x) = lim ,
y→x x−y
que existe porque f é derivável em x, também está entre −c e c, como queríamos. 

Observação. O Corolário 3 do Teorema 7 traz a hipótese de I ser aberto, porém, pelo menos
aparentemente, tal hipótese não foi usada em sua demonstração. Podemos nos perguntar então
f : [a, b] → R é suposta
por que ela foi colocada lá. No teorema do qual este corolário é oriundo,
contínua (em todo o seu domínio) e derivável em (a, b). O mesmo é feito no Teorema 6, no qual
0 0
Elon diz que assim ele fez porque as derivadas f (a) e f (b) não intervêm na demonstração
(página 271). A minha conclusão então é que colocar aquela referida hipótese de I ser aberto foi
uma opção do Elon, como uma forma de enunciar um resultado mais geral, mais geral também
por causa do Teorema 1 e do que indica a Observação feita após o corolário em questão: uma
função derivável num intervalo I é contínua em I e derivável em int(I).
304 CAPÍTULO 8. DERIVADAS

8.25 Exercício 25. Teorema do Ponto Fixo de Banach


Seja f : I → R derivável no intervalo fechado I (limitado ou não). Suponha que |f 0 (x)| 6 c
para todo x ∈ I , onde c é uma constante tal que 0 6 c < 1. Além disso, admita que f (I) ⊂ I .
Então, dado qualquer x0 ∈ I , tem sentido formar a sequência x1 = f (x0 ), x2 = f (x1 ), . . . , xn =
f (xn−1 ), . . .. Mostre que, seja qual for o ponto inicial x0 escolhido, existe lim xn = a e que a ∈ I
é o único ponto de I tal que f (a) = a. (Veja o Exemplo 19, Capítulo IV.)

(Voltar para as Curiosidades do Exercício 17 do Capítulo II: página 51.)

(Voltar para a Solução do Exercício 30: página 308.)

Solução. Pelo referido exemplo  o método das aproximações sucessivas  para vermos a
convergência desta sequência basta vericarmos que

|xn+2 − xn+1 | 6 c|xn+1 − xn | ∀n ∈ N .


Isto acontece pelo que acabamos de ver no exercício anterior:

|xn+2 − xn+1 | = |f (xn+1 ) − f (xn )| 6 c|xn+1 − xn |,


uma vez que |f 0 (x)| 6 c implica |f (x) − f (y)| 6 c|x − y| para todo x, y ∈ I . O limite lim xn
pertence a I porque I é fechado.
Seja a = lim xn . Pela continuidade de f em a temos

a = lim xn = lim xn+1 = lim f (xn ) = f (a)


(Teorema 4 do Capítulo VII). Logo o limite de qualquer sequência da forma (x, f (x), f (f (x)), . . .)
é um ponto xo de f.
Se a ∈ I é tal que a = f (a), então a é limite da sequência (xn )n∈N denida por xn = f (xn−1 ),
com x0 = a.
Finalmente, vejamos que qualquer sequência construída da forma proposta converge para
o mesmo número. Sejam x0 ∈ I e y0 ∈ I e as sequências (xn )n∈N e (yn )n∈N denidas por
xn = f (xn−1 ) e yn = f (xn−1 ) para todo n ∈ N. Conforme já vimos, ambas estas sequências
convergem. Para ver agora que elas convergem para o mesmo número, basta mostrarmos a
igualdade
lim(xn − yn ) = 0
(Exercício 6 do Capítulo IV, página 122). Novamente podemos aplicar o exercício anterior:

|x1 − y1 | = |f (x0 ) − f (y0 )| 6 c|x0 − y0 |


e
|xn − yn | 6 cn |x0 − y0 |,
pois

|xn − yn | 6 cn |x0 − y0 | ⇒ |xn+1 − yn+1 | 6 c|xn − yn | 6 c · cn |x0 − y0 | = cn+1 |x0 − y0 |.


Portanto
0 6 |xn − yn | 6 cn |xn − yn | ∀n ∈ N .
Agora |xn − yn | → 0, pelo Teorema 8 do Capítulo IV, porque cn |xn − yn | → 0 (por resultados
do Capítulo IV, a saber, seu Exemplo 6 e seus Teoremas 3, 5 e 6). Logo xn − yn → 0, como
queríamos. 
(Voltar para a Solução do Exercício 30: página 308.)
8.26. EXERCÍCIO 26 305

Observação. Em seu livro Espaços Métricos, Elon chama de contração a uma função que
satisfaz |f (x)−f (y)| 6 c|x−y| para todos x e y de seu domínio e algum 0 6 c < 1. Neste mesmo
livro, uma generalização do exercício acima aparece com o nome Teorema de Banach, sobre
pontos xos de contrações. Na realidade, as funções das quais tratam tal teorema não precisam
ser necessariamente deriváveis. Por outro outro lado, no exercício acima, o que importou acerca
de f :I→R foi que f é uma contração, I é um subconjunto fechado de R e f [I] ⊂ I . Foi por
isso que intitulei este exercício de Teorema do Ponto Fixo de Banach.

8.26 Exercício 26
Obtenha uma função f : R → R, de classe C ∞, tal que |f 0 (x)| < 1 e f (x) 6= x para todo x ∈ R.

Solução. Uma primeira observação é que, para termos f (x) 6= x para todo x ∈ R, pela
continuidade de f (mais precisamente pelo Teorema do Valor Intermediário), o gráco de f
deve estar todo acima ou todo abaixo da diagonal, gráco da função identidade.
Um exemplo é a função cujo gráco é um ramo de hipérbole que tem as retas y = x e

y = −x como assíntotas; f : R → R denida por f (x) = 1 + x2 para todo x ∈ R satisfaz
estas condições. Ela é derivável, com

x
f 0 (x) = ∀x ∈ R,
f (x)
|f 0 (x)| < 1, porque f (x) > x para x > 0, f (x) > −x para x < 0 e f (x) > 0 para todo x
x x x
implicam 0 6 < 1, se x > 0, e −1 < < 0, se x < 0, logo −1 < = f 0 (x) < 1
f (x) f (x) f (x)
para todo x.
0 x 0 0 2 (2)
De f (x) = , temos f (x) · f (x) = x. Disto vem: [f (x)] + f (x) · f (x) = 1,
f (x)
3f 0 (x)f (2) (x) + f (x)f (3) (x) = 0, etc, f 0 e f (2) deriváveis porque f (x) 6= 0 para todo x. De
modo geral, para cada n ∈ N, existe uma função derivável gn : R → R tal que

gn (x) + f (x) · f (n) (x) = 0;


g1 (x) = −x e gn+1 = gn0 + f 0 · f (n) , f (n) derivável de novo porque f (x) 6= 0 para todo x. Logo f
é innitamente derivável, o que implica a continuidade das derivadas de todas as ordens.

8.27 Exercício 27
Seja p ∈ N. Dada f : I → R derivável no intervalo fechado I , com f (I) ⊂ I , suponha que
g = f ◦ f ◦ · · · ◦ f = f p cumpre |g 0 (x)| 6 c < 1 para todo x ∈ I , onde c é constante. Prove que
306 CAPÍTULO 8. DERIVADAS

existe um único a∈I tal que f (a) = a e que, para todo x∈I tem-se lim f n (x) = a (onde
n→∞
n
f = f ◦ f ◦ · ◦ f, n vezes).

Solução. A função g = fp possui todas as propriedades daquela considerada no Exercício


25 acima. Assim, sabemos que o limite lim g n (x) existe para todo x ∈ I e que o conjunto
n o n→+∞

lim g n (x) ∈ R; x ∈ I é únitário. Seja {a} este conjunto. Então, xado x ∈ I, todas as
n→+∞
sequências

x, f p (x), f 2p (x), . . . ,

f (x), f p+1 (x), f 2p+1 (x), . . . ,

f 2 (x), f p+2 (x), f 2p+2 (x), . . . ,


.
.
.

f p−1 (x), f 2p−1 (x), f 3p−1 (x), . . .


convergem para a. Logo, pelo Exercício 4 do Capítulo IV (página 118), a sequência (f n−1 (x))n∈N
também converge para a. Como x era um ponto qualquer de I, podemos concluir que

lim f n (x) = a ∀x ∈ I.
n→+∞

Este a é um ponto xo de f pelo mesmo motivo já visto no Exercício 25: xado x ∈ I,
temos

a = lim f n (x) = lim f n+1 (x) = lim f (f n (x)) = f (a),


n→+∞ n→+∞ n→+∞

por causa da continuidade de f (é contínua porque é derivável).


Com relação à unicidade do ponto xo, se b ∈ Iné tal que f (b) = b,oentão g n (b) = b para
todo n ∈ N, de modo que b = lim g n (b) e, então, b ∈ lim g n (x); x ∈ I = {a}, donde b = a,
n→+∞
como queríamos. 

8.28 Exercício 28
Dada f : R → R derivável, com derivada limitada, prove que existe uma constante c ∈ R tal
que a função ϕ : R → R, denida por ϕ(x) = x + c · f (x), é uma bijeção cuja inversa é derivável.

Solução 1.

No elevador. Na procura de quais propriedades c precisa possuir para fazer ϕ ser injetiva,
sobrejetiva e derivável, algo de bastante revelador surge na investigação acerca da sobrejetivi-
dade de ϕ. Dado y ∈ R, queremos um x∈R tal que x + cf (x) = y . Isto pode ser parafraseado
de um modo bastante interessante assim: xado o y ∈ R, queremos que x seja um ponto xo
da função y − cf (!). Sendo que há logo ali em cima um exercício que trata de pontos xos,
em especial os de contrações, a saber, o 25. Por ele, sabemos que este ponto xo é único se
|(y − cf )0 (x)| = |cf 0 (x)| for menor do que ou igual a um número menor do que 1 para todo
x ∈ R. Isto nos dará de uma só vez todas as propriedades requeridas para ϕ (!).
8.28. EXERCÍCIO 28 307

Por hipótese, seja A um número real tal que |f 0 (x)| 6 A para todo x ∈ R. Seja B um
número positivo maior do que A. Para ele temos
0
f (x) A
B 6 B <1
∀x ∈ R .

1
Armamos que, para c= , a função ϕ, denida por ϕ(x) = x + cf (x), é uma bijeção cuja
B
inversa é derivável. É uma bijeção por causa do seguinte: xado y ∈ R, seja fy : R → R uma
função denida por fy (x) = y − cf (x); ela é tal que
0
f (x) A
|fy0 (x)| 0
= |cf (x)| = 6 <1
B B

para todo x ∈ R, portanto, pelo Exercício 25 (ponto xo de uma contração), existe um único
x∈R tal que fy (x) = x, ou seja, existe um único x ∈ R tal que ϕ(x) = y , logo ϕ é bijetiva. É
derivável por causa do Corolário do Teorema 3: ϕ é derivável, sua inversa é contínua (Teorema
13 do Capítulo VII) e

f 0 (x)
ϕ0 (x) = 1 + cf 0 (x) = 0 ⇔ = −1 ⇒ ϕ0 (x) 6= 0 ∀x ∈ R,
B
0
f (x)
pois
B <1 para todo x real, logo a inversa de ϕ é derivável. 

Solução 2. Na solução acima zemos referência ao Corolário do Teorema 3, o qual também


está relacionado ao Corolário 6 do Teorema 7. Usemos este corolário.
Não importa quem seja c, ϕ será derivável. Daí, pelo Corolário 6 do Teorema 7, se tivermos
ϕ0 (x) > 0 para todo x ∈ R, então ϕ possuirá uma inversa derivável e denida no intervalo
J := ϕ[R]. Só restando ter J = R.
0 0
Como f é limitada, seja A um número real positivo tal que −A < f (x) < A para todo
x ∈ R. Assim, para c > 0, temos sucessivamente:

−cA < cf 0 (x) < cA e 1 − cA < 1 + cf 0 (x) < 1 + cA,

ou seja, 1 − cA < ϕ0 (x) < 1 + cA x ∈ R. Portanto, duas condições que c deve


para todo
1 1
satisfazer são c > 0 e 1 − cA > 0, isto é, 0 < c < . Fixemos então 0 < c < (o que também
A A
faz ϕ ser crescente) e vejamos que sua imagem J é igual a R. Como J é um intervalo, vamos
mostrar que J não é limitado superior nem inferiormente. A ideia será a mesma do Exercício
0
17 (página 297): o resultado desejado vai seguir do fato de ϕ (x) estar sempre longe o suciente
0
do 0 (de fato ϕ (x) > 1 − cA > 0 para todo x ∈ R).
Se ϕ fosse limitada superiormente, como ϕ é crescente, existiria L ∈ R tal que lim ϕ(x) =
x→+∞
L. Assim, para cada n ∈ N, tomando xn ∈ (n, n + 1) tal que ϕ0 (xn ) = ϕ(n + 1) − ϕ(n) (Teorema
7, do Valor Médio), teríamos

0 = lim ϕ(n + 1) − ϕ(n) = lim ϕ0 (xn ),


n→+∞ n→+∞

0
isto é, existiria uma sequência (xn )n∈N de números reais tal que ϕ (xn ) → 0, em particular,
0 0
existiria x ∈ R tal que ϕ (x) < 1 − cA, o que contradiz o fato de ϕ (x) ser sempre maior do que
1 − cA. Portanto, ϕ não é limitada superiormente.
308 CAPÍTULO 8. DERIVADAS

De modo análogo, se ϕ fosse limitada inferiormente, então existiria x ∈ R tal que ϕ0 (x) <
1 − cA, pois: existiria M ∈ R tal que lim ϕ(x) = M e, então, tomando xn ∈ (−n − 1, −n)
x→−∞
tal que ϕ0 (xn ) = ϕ(−n) − ϕ(−n − 1), teríamos lim ϕ0 (xn ) = 0. Portanto, ϕ também não é
n→+∞
limitada inferiormente.
Logo, ϕ[R] = R e estamos feitos. 

8.29 Exercício 29
A função f : [0, π] → R, denida por f (x) = cos x, não cumpre a condição |f 0 (x)| 6 c < 1 para
todo x ∈ [0, π], com c ∈ R constante, mas f 2 = f ◦ f cumpre.

Solução. De fato, para x = π/2, temos

0
π
|f (x)| = − sen = 1.

2

f 2 , notamos que, na realidade, ela não está bem denida, f (x) sequer pertence
Com relação à
ao domínio de f se x ∈ (π/2, π]. Vamos então corrigir o domínio da f para [0, π/2]. Agora sim
  2
temos f [0, π/2] ⊂ [0, π/2] e faz sentido considerar a composta f . Além disso essa correção
não altera o que zemos no páragrafo acima.
2
Seja g a derivada de f . Queremos mostrar que existe |g(x)| 6 c para
c < 1 tal que
todo x ∈ [0, π/2]. Pela Regra da Cadeia (Teorema 3), g é dada por g(x) = sen x · sen(cos x)
para todo x ∈ [0, π/2]. Se x ∈ [0, π/2], então 0 6 cos x 6 1, 0 6 sen(cos x) 6 sen 1 e
0 6 g(x) 6 sen x · sen 1, pois a função seno é crescente no intervalo [0, 1] ⊂ [0, π/2] e sen x > 0
∀x ∈ [0, π/2]. Como sen x · sen 1 6 sen 1 para todo x ∈ R, segue

|g(x)| 6 c ∀x ∈ [0, π/2],

onde c = sen 1 é estritamente menor do que 1, como queríamos. 

Observação. Com apenas umas poucas modicações, podemos concluir que os resultados
acima são válidos mesmo quando o domínio de f é R. Este exercício junto com o Exercício
27 nos dão um modo de encontrarmos uma solução cada vez mais aproximada para a equação
cos x = x, a qual existe e é única de acordo com eles.

8.30 Exercício 30
Seja f : [a − δ, a + δ] → R derivável, com |f 0 (x)| 6 c < 1 para todo x ∈ [a − δ, a + δ]. Se
|f (a) − a| 6 (1 − c)δ , então existe um único x0 ∈ [a − δ, a + δ] tal que f (x0 ) = x0 .

Solução. Vamos denotar por I o intervalo [a−δ, a+δ], domínio de f . Se pudermos considerar
n−1
a sequência (f (a))n∈N , então, do mesmo modo como zemos na Solução do Exercício 25
(página 304), poderemos concluir que ela converge para um número x0 ∈ I , que ele é um ponto
xo de f e que ele é o único ponto xo de f (se α e β são pontos xos de f , sejam as sequências
n n
denidas por xn = f (α) e yn = f (β) na notação do quarto parágrafo daquela solução). Para
n
considerarmos tal sequência basta mostrarmos que f (a) ∈ I para todo n ∈ N.
8.31. EXERCÍCIO 31 309

Para n = 1, |f (a)−a| 6 (1−c)δ < δ ⇒ f (a) ∈ I . Portanto f (a) ∈ I , pois |f (a)−a| 6 (1−c)δ
por hipótese. Para n = 2,

|f 2 (a) − a| 6 |f 2 (a) − f (a)| + |f (a) − a|


6 c|f (a) − a| + |f (a) − a|
= (1 + c)|f (a) − a|
6 (1 + c)(1 − c)δ
= (1 − c2 )δ
< δ,

lembrando da Desigualdade Triangular (item (i) do Teorema 2, Capítulo III) e que |f (x) −
f (y)| 6 c|x − y| (Exercício 24, página 303). E, de modo geral, se f (a), f 2 (a), . . . , f n−1 (a)
pertencem a I, então:

|f n (a) − a| 6 |f n (a) − f n−1 (a)| + |f n−1 (a) − f n−2 (a)| + · · · + |f 2 (a) − f (a)| + |f (a) − a|
6 cn−1 |f (a) − a| + cn−2 |f (a) − a| + · · · + c|f (a) − a| + |f (a) − a|
6 (cn−1 + cn−2 + · · · + c + 1)(1 − c)δ
= (1 − cn )δ
< δ,

portanto f n (a) ∈ I . Logo, f n (a) ∈ I ∀n ∈ N, como queríamos. 

Observação. A partir de |f n (a) − a| 6 (1 − cn )δ ∀n ∈ N, podemos também fazer o seguinte:


n
lim f (a) = a, pois lim(1 − cn )δ = 0 e, portanto,

a = lim f n (a) = lim f n+1 (a) = lim f (f n (a)) = f (a),

pela continuidade de f em a.

8.31 Exercício 31
f (x) − f (a)
Seja f : [a, b] → R contínua, derivável em (a, b). Se lim+ f 0 (x) = +∞ então lim+ =
x→a x→a x−a
+∞.

f (x) − f (a)
Solução. Dado x ∈ (a, b], seja y ∈ (a, x) tal que f 0 (y) = , o qual existe pelo
x−a
Teorema 7, uma vez que f é contínua em [a, x] e derivável em (a, x) por hipótese. Claramente
lim f 0 (y) = +∞, donde segue a tese. 
x→a+

8.32 Exercício 32
Dada f : [a, b] → R derivável e dado c ∈ [a, b], dena uma função ξ : [a, b] → R pondo
f (x) − f (c) 0
ξ(x) = se x 6= c e ξ(c) = f (c). Prove que se f é duas vezes derivável no ponto
x−c
f 00 (c)
c então existe ξ 0 (c) e vale ξ 0 (c) = 0
. Dê um exemplo em que existe ξ (c) mas não existe
2
f 00 (c).
310 CAPÍTULO 8. DERIVADAS

Solução.

Primeira Parte. Vamos mostrar que

ξ(c + h) − ξ(c) f 00 (c)


 
lim − = 0.
h→0 h 2

Para h 6= 0, temos

f (c + h) − f (c)
ξ(c + h) − ξ(c) f 00 (c) − f 0 (c) f 00 (c)
− = h −
h 2 h 2
f 00 (c) 2
 
0
f (c + h) − f (c) + f (c)h + h
2
= ,
h2
que tende para 0 conforme h tende para zero pelo Teorema 9, como queríamos.

Segunda Parte.

No elevador. A foragida f é derivável e não é duas vezes derivável. Começamos então


com aquelas funções que sabemos não serem deriváveis e, em seguida, tomamos as funções que
0 0

tem elas como derivadas. Casos mais a mão, como quando f (x) = |x| ou f (x) = x, e c = 0
0
para ambos, não funcionam. Em qualquer um deles f e ξ tem um comportamento semelhante
0
num certo sentido; existe α ∈ R tal que f = α · ξ . Folheamos o livro para experimentar outras
2
funções mencionadas por ele, até que chegamos a f dada por f (x) = x sen 1/x se x 6= 0 e
f (0) = 0; para c = 0 temos ξ(x) = x sen 1/x e ξ(0) = 0. Esta não é derivável no 0 e, portanto,
0
esta f também não serve. Porém o que elas têm de interessante é que agora f e ξ possuem
0
comportamentos já mais distintos, f (x) = 2x sen 1/x − cos 1/x, o que pode ser um indício de
que estamos próximos da função procurada. Para corrigir a falta da derivabilidade de ξ na
3
origem, subimos um grau e consideramos a f dada por f (x) = x sen 1/x. Encontrada!

Seja f : [0, 1] → R denida por

1
f (x) = x3 sen se x 6= 0 e f (0) = 0.
x
f (x) − f (0) 1
Ela é derivável no 0: lim = lim x2 sen = 0 (produto de uma função limitada por
x→0 x−0 x→0 x
0
uma que tende a zero), ou seja, f (0) = 0. Para x 6= 0

1 1
f 0 (x) = 3x2 sen − x cos ;
x x
f não é duas vezes derivável no 0: se x 6= 0, então

f 0 (x) − f 0 (0) 1 1
= 3x sen − cos ,
x−0 x x
cujo limite quando x → 0 não existe (para ver isto, considere as sequências (1/(2πn))n∈N e
(1/(π + 2πn))n∈N ).
8.33. EXERCÍCIO 33 311

Para c=0 a função ξ : [0, 1] → R é dada por

f (x) − f (0) 1
ξ(x) = = x2 sen , se x 6= 0
x−0 x

e ξ(0) = f 0 (0) = 0. Ela sim é derivável no 0:

ξ(x) − ξ(0) 1
lim = lim x sen = 0.
x→0 x−0 x→0 x

Logo, para c = 0, ξ 0 (c) existe e f 00 (c) não existe, como queríamos. Isto mostra a importância
de f ser duas vezes derivável em c para que o resultado da primeira parte deste exercício seja
válido. 

8.33 Exercício 33
Se f :I→R e g:J →R são funções de classe Cn tais que f (I) ⊂ J então g◦f :I →R é de
n
classe C .

(Voltar para a Solução do Exercício 35, Nível 1: página 314.)

(Voltar para a Solução do Exercício 39, Nível 2.1: página 320.)

Solução. Em primeiro lugar, como a denição do conceito de função de classe C n foi feita
apenas para quando o domínio de uma função é um intervalo, podemos supor que I e J são
intervalos.

Nível 1. Vamos proceder por indução em n.


Para o passo inicial, sejam g, f ∈ C 1 . Queremos ver que g◦f
é continuamente derivável.
0
Primeiro, g ◦ f é derivável porque ambas f e g o são (Regra da Cadeia, o Teorema 3, f (a) ∈ J
0 0 0
para todo a ∈ I pois J é um intervalo). Segundo, (g ◦ f ) = (g ◦ f ) · f é contínua porque
são contínuas as compostas de funções contínuas e os produtos de funções contínuas (Teoremas
0 0
5 e 6 do Capítulo VII); g e f são contínuas por hipótese e f é contínua porque é derivável
1
(Teorema 1). Logo g ◦ f ∈ C , como queríamos.
Para o passo de indução, vamos supor a veracidade deste exercício para um xado número
n+1 n+1
natural n. Dadas f, g ∈ C , vamos mostrar, no Nível 2, que g ◦ f ∈ C .

No elevador. Há diversos modos de cairmos no caso n. Dentre eles, a nossa inspiração
surge no trabalho com o nosso objeto-alvo, a saber, a n + 1-ésima derivada de g ◦ f (mesmo
sem ainda sabermos se ela existe), escrevendo

(n) (n)
(g ◦ f )(n+1) = (g ◦ f )0 = (g 0 ◦ f ) · f 0 .

Nível 2. Se f e g são de classe C n+1 , então f, f 0 e g0 são de classe C n. Também é de


classe C n
a composta g0 ◦ f .
Isto pela hipótese de indução. Armamos agora que o produto de
n n 0 0
funções de classe C é de classe C (Nível 3). Com isto, a função (g ◦ f ) · f = (g ◦ f ) é de
n n+1
classe C , logo g ◦ f é de classe C , como queríamos.
312 CAPÍTULO 8. DERIVADAS

Nível 3: Lema. Se f : I → R e g : I → R são funções de classe Cn (onde I é um


n
intervalo), então o produto f · g : I → R é de classe C .

(Voltar para a Solução do Exercício 43, Segunda Parte: página 325.)

Solução.Esta solução é semelhante aos Níveis 1 e 2 acima. Para n = 1, f · g é derivável


0 0 0
pelo Teorema 2 (produto de funções deriváveis é derivável) e (f · g) = f · g + f · g é contínua
0 0
pelo Teorema 5 do Capítulo VII (todas as funções f , g, f e g são contínuas). Para um número
n+1 0 0 n
natural n, se f e g são de classe C , então f, f , g e g são de classe C e, se este lema é válido
0 0 n
para n, então f · g e f · g também são de classe C . Descendo mais um nível, armamos que
n n
a soma de funções de classe C é de classe C (Nível 4). Desta forma,

f 0 · g + f · g 0 = (f · g)0

é de classe C n, o que implica f · g ∈ C n+1 , como queríamos.

Nível 4: Lema. Se f : I → R e g : I → R são funções de classe Cn (onde I é um


n
intervalo), então a soma f +g :I →R é de classe C .

(Voltar para a Solução do Exercício 39, Nível 2.2: página 320.)

Solução. A função f +g é n vezes derivável em I porque f (i) + g (i) = (f + g)(i) para todo
(n)
i ∈ {0, 1, . . . , n},
pelo Teorema 2, além das hipóteses do lema. E (f + g) = f (n) + g (n)
(n) (n)
é contínua porque f e g são contínuas por hipótese e uma soma de funções contínuas é
contínua (Teorema 5 do Capítulo VII). 

Observação. No uso que demos à Regra da Cadeia mais acima foi importante a hipótese de
0
J ser um intervalo para termos f (a) ∈ J para todo a ∈ I . Pois, tal teorema, do modo como
enunciado no livro, nos garante que g ◦ f é derivável num ponto a de acumulação do domínio de
f pertencente a este domínio caso f (a) (além de pertencer ao domínio de g ) seja de acumulação
do domínio de g . Podemos então nos perguntar o seguinte: e se f (a) não for um ponto de
acumulação do domínio de g , isto é, e se f (a) for um ponto isolado do domínio de g , g ◦ f
continuará sendo derivável em a?
0
Fixemos as funções f : X → R e g : Y → R, com f [X] ⊂ Y , e seja a ∈ X ∩ X um ponto
no qual f é derivável e tal que f (a) é isolado em Y . Começamos observando o seguinte: para
uma sequência (xn )n∈N de pontos de X , dois a dois distintos e convergente para a, a partir de
um certo momento, todo f (xn ) deve ser igual a f (a), pois, caso contrário, como f (xn ) → f (a)
(f derivável em a implica f contínua em a), o ponto f (a) seria de acumulação de Y . Vamos
então mostrar a

Proposição. (Composta de funções deriváveis.) Dadas as funções f : X → R e


0
g : Y → R, f [X] ⊂ Y , seja a ∈ X ∩ X e seja b = f (a). Se f é derivável
com em a e se b é um
ponto isolado de Y , então g ◦ f é derivável em a, e (g ◦ f )0 (a) = 0.
Solução. Fixado ε > 0, queremos encontrar δ > 0 tal que

g(f (x)) − g(f (a)) g(f (x)) − g(b)
x ∈ X, 0 < |x − a| < δ ⇒
=
< ε.
x−a x−a

Como b é isolado em Y, seja δ0 > 0 tal que (b − δ0 , b + δ0 ) ∩ Y = {b}. Como f é derivável


em a, f é contínua em a (Teorema 1), ou seja, lim f (x) = b. Assim, seja δ1 > 0 tal que
x→a

x ∈ X, 0 < |x − a| < δ1 ⇒ |f (x) − b| < δ0 .


8.34. EXERCÍCIO 34 313

Deste modo, como f (x) ∈ Y ∀x ∈ X , temos:

x ∈ X, |x − a| < δ1 ⇒ f (x) ∈ Y ∩ (b − δ0 , b + δ0 ) = {b},


portanto
x ∈ X, |x − a| < δ1 ⇒ f (x) = b.
Finalmente:

g(f (x)) − g(b) g(b) − g(b)


x ∈ X, 0 < |x − a| < δ1 ⇒ = = 0 < ε.
x−a x−a
Logo, g◦f é derivável em a, e (g ◦ f )0 (a) = 0, como queríamos. 

Conclusão. Combinando o resultado acima com o Teorema 3 do livro (a Regra da Cadeia),


podemos concluir que a composta de duas funções deriváveis sempre é derivável. De fato, por
0
denição, para vericar que g ◦ f : X → R é derivável, devemos vericar que, dado a ∈ X ∩ X ,
g ◦ f é derivável em a. Como f (a) ∈ Y (por hipótese), temos dois casos a considerar: ou
f (a) ∈ Y 0 e caímos na Regra da Cadeia, ou f (a) é isolado em Y e caímos na proposição acima.

8.34 Exercício 34
Sejaf :I →R derivável, com f 0 (x) 6= 0 para todo x ∈ I. Se f 00 (a) existe, calcule (f −1 )00 no
ponto b = f (a).

Solução. Se I é um intervalo, podemos provar que f é invertível. Se I não é um intervalo,


f pode não admitir uma inversa (ver a Observação mais abaixo). Por outro lado, assim como
zemos no exercício anterior, podemos entender que a hipótese de I ser um intervalo já está
00
contida implicitamente na hipótese de f (a) existir, uma vez que a denição de função n vezes
derivável num ponto (e, no caso, n = 2) foi feita apenas dentro desse contexto (ver página 278
do livro).
A f admite uma inversa pelo Corolário 6 do Teorema 7 (e sua observação) e pelo Teorema
do Valor Intermediário para a Derivada (Teorema 5): se existissem x1 e x2 em I , com x1 < x2
0 0
(spg), tais que f (x1 ) > 0 e f (x2 ) < 0 (ou vice-versa), então existiria c ∈ (x1 , x2 ), e portanto
c ∈ I tal que f 0 (c) = 0, uma vez que f é derivável em [x1 , x2 ] e f 0 (x2 ) < 0 < f 0 (x1 ) (ou
f 0 (x1 ) < 0 < f 0 (x2 )). Portanto, como f 0 (x) 6= 0 para todo x ∈ I por hipótese, ou f 0 (x) > 0
0
para todo x ∈ I ou f (x) < 0 para todo x ∈ I . De qualquer forma, pelo Corolário 6 do Teorema
−1
7, f admite uma inversa f denida no intervalo J := f [I]; ela é derivável em seu domínio e
sua derivada é dada por
1
(f −1 )0 (x) = ∀x ∈ J.
f 0 (f −1 (x))
−1 0
O que justica a derivabilidade de (f ) em b é o Teorema 2, porque f 0 é derivável em
f −1 (b) = a e f 0 (f −1 (b)) = f 0 (a) 6= 0 por hipótese. Finalmente, pelos Teoremas 2 e 3 (existem
(f −1 )0 (b) e f 00 (f −1 (b)) = f 00 (a)), calculamos (f −1 )00 em b:
(f 0 ◦ f −1 )0 (b)
(f −1 )00 (b) = −
[f 0 (f −1 (b))]2
(f −1 )0 (b) · f 00 (f −1 (b))
= −
[f 0 (a)]2
f 00 (a)
= − 0 .
[f (a)]3
314 CAPÍTULO 8. DERIVADAS

f 00 (a)
Logo (f −1 )00 (b) = − . 
[f 0 (a)]3

Observação. Existe uma função f : I → R (com I ⊂ R) derivável e tal que f 0 (x) 6= 0 ∀x ∈ I


que não é invertível. Vejamos alguns exemplos.

Exemplo 1. Para I = [−1, 0) ∪ (0, 1], a função f :I→R denida por f (x) = |x| ∀x ∈ I
é derivável e sua derivada nunca se anula; f não é invertível, mas, ainda sim, podemos fazer
o que o exercício pede, considerando as inversas locais de f; não importa de quem o b seja
imagem, teremos sempre um mesmo valor para (f −1 )00 (b).
Mas isto pode nem sempre ser verdade, existem casos em que de fato o valor de (f −1 )00 (esta
inversa entendida como inversa local) em b = f (a) depende do valor de a escolhido:

Exemplo 2. Ainda para I = [−1, 0) ∪ (0, 1], seja agora a função f : I → R denida por
2 −1 00
f (x) = x ∀x ∈ I . Para ela podemos ter duas possibilidades para o número (f ) (b), uma
simétrica da outra.
No exemplo acima, os possíveis valores para o número (f −1 )00 (b) apresentaram um bom
comportamento num certo sentido, um era o simétrico do outro. Porém, os candidatos para
(f −1 )00 (b) podem se tornar cada vez mais complicados, como indica o exemplo abaixo.

[
Exemplo 3. Sejam I = (n − 1, n) e a função f : I → R denida por f (x) = (x −
n∈N
(n − 1))n , se x ∈ (n − 1, n). Esta função ainda é derivável e sua derivada nunca se anula.
Fixado b ∈ (0, 1), para todo n ∈ N, existe an ∈ (n − 1, n) tal que f (an ) = b. Cada restrição
f |(n − 1, n) : (n − 1, n) → (0, 1) é invertível, sendo √ sua inversa gn : (0, 1) → (n − 1, n)
1−n √
n
√ x1−n n
denida por gn (x) =
n
x + (n − 1). Portanto, gn0 (x) = 00
e gn (x) =
2
· x1−2n , logo
n n
00 1−n √ n
1−2n
gn (b) = · b para todo n ∈ N.
n2
−1
Nestes exemplos, apesar de o símbolo f já não fazer mais sentido, ainda sim foi possível
−1 00
calcular o valor de ((f |X) ) (b), sendo X ⊂ I um intervalo tal que a ∈ X , para a xado a
priori.

8.35 Exercício 35
Dados a < b, dena ϕ : R → R pondo ϕ(x) = e1/(x−a)(x−b) se a < x < b e ϕ(x) = 0 se x ∈
/ (a, b).

Mostre que ϕ é uma função C com um único ponto de máximo.

(Voltar para a Solução do Exercício 45, Segunda Parte: página 329.)

(Voltar para a Solução do Exercício 14a do Capítulo IX, para o elevador de seu Lema:
página 362.)

Solução.

Nível 1. Em x∈
/ {a, b}, ϕ é innitamente derivável (Exercício 33, página 311, por exem-
plo). A questão é vericar que ϕ é innitamente derivável em a e em b. Aqui notamos a
8.35. EXERCÍCIO 35 315

semelhança deste exercício com os Exemplos 20 e 24 do livro. Motivados por estes exemplos,
armamos que

lim ϕ(n) (x) = lim− ϕ(n) (x) = 0 ∀n ∈ N


x→a+ x→b

(Nível 2). Estas igualdades também valem para n = 0, o que implica a continuidade de ϕ
em a e em b. Disto, junto com o fato de que as igualdades acima valem para n = 1, temos
ϕ (a) = ϕ0 (b) = 0 pelo Corolário 5 do Teorema 7.
0
De forma análoga, se ϕ
(n)
(a) = ϕ(n) (b) = 0,
então, novamente pelo Corolário 5 do Teorema 7,

lim ϕ(n) (x) = lim− ϕ(n) (x) = 0


x→a+ x→b

lim ϕ(n+1) (x) = lim− ϕ(n+1) (x) = 0.


x→a+ x→b

implicam ϕ(n+1) (a) = ϕ(n+1) (b) = 0. Logo, por indução em n, podemos concluir que

ϕ(n) (a) = ϕ(n) (b) = 0 ∀n ∈ N .

(Voltar para a Solução do Exercício 43, Nível 1: página 323.)

No elevador. Para n=1 e para x ∈ (a, b) temos

a + b − 2x
ϕ(n) (x) = 2 2
e1/(x−a)(x−b)
(x − a) (x − b)

e queremos mostrar que este número, quando x tende para a pela direita e para b pela esquerda,
1
tende para zero. Uma mudança de variável como y = não ajuda muito, ela
(x − a)(x − b)
mantém a função exponencial em cima, quando seria bom que ela fosse para baixo. Na
verdade, um tal y é sempre negativo para todo x ∈ (a, b), então a grande sacada é fazer

1
y= ;
(x − a)(b − x)

para ele temos

lim y = lim− y = +∞
x→a+ x→b
e
y2
ϕ0 (x) = (a + b − 2x) · .
ey
No caso mais genérico poderíamos conjecturar que ϕ(n) (x) sempre é desta forma, isto é, que
sempre existem polinômios qn e pn tais que

ϕ(n) (x) = qn (x) · pn (f (x))ef (x) ,

1
para f (x) = , mas, na tentativa de demonstrar esta igualdade por indução, vemos
(x − a)(x − b)
(n)
a possibilidade de ela ser falsa. Isto não é problema, ϕ não precisa ser assim, ser uma soma
de funções deste tipo já basta...
316 CAPÍTULO 8. DERIVADAS

Nível 2. Dados n ∈ N e x ∈ (a, b), armamos que ϕ(n) (x) pode ser escrito como uma
k f (x)
soma na qual cada parcela é do tipo p(x) · [f (x)] e , onde p é um polinômio não nulo,
1
f (x) = e k é um número natural maior do que ou igual a 2 (Nível 3).
(x − a)(x − b)
1
Conforme já sugerimos no elevador que nos trouxe para este nível, para y= ,
(x − a)(b − x)
temos lim y = lim− y = +∞. Se k é par, então
x→a+ y→b

yk
p(x)[f (x)]k ef (x) = p(x) ·
ey
e, se k é ímpar, então
yk
p(x)[f (x)]k ef (x) = −p(x) · .
ey
yk
De qualquer forma, como lim p(x) = p(a), lim p(x) = p(b) e lim =0 (Exemplo 19), temos
x→a x→b y→+∞ ey

lim+ p(x)[f (x)]k ef (x) = lim− p(x)[f (x)]k ef (x) = 0,


x→a x→b
o que faz
lim ϕ(n) (x) = lim− ϕ(n) (x) = 0,
x→a+ x→b
como queríamos.

(n)
Nível 3. Vamos demonstrar a referida armação por indução em n. Para n = 1, ϕ (x) é
k f (x)
igual a p(x)[f (x)] e , uma soma de uma única parcela, com p(x) = a + b − 2x e k = 2. Com
k f
relação ao passo de indução, derivamos a parcela pf e :

(pf k ef )0 = p0 f k ef + pkf k−1 f 0 ef + pf k f 0 ef ,


onde f 0 = qf 2 , com q(x) = a + b − 2x. Fazendo esta substituição, obtemos

(pf k ef )0 = p0 f k ef + kpqf k+1 ef + pqf k+2 ef .


Deste modo, se ϕ(n) (x) é uma soma de parcelas do tipo pf k ef , onde p é um polinômio não nulo
(n+1)
e k é um natural > 2, então ϕ (x) continua sendo uma soma do mesmo tipo (mesmo que a
0 k f
parcela p f e tenha sumido). Segue a tese.
a+b
Com relação ao ponto de máximo de ϕ, ele ocorre apenas em , o ponto médio do
2
segmento do (a, b). Para ver isto, pelo fato da função exponencial ser crescente e de ϕ ser
constante fora do intervalo (a, b), basta estudar os pontos de máximo da f : (a, b) → R denida
1
por f (x) = . Para ela, conforme já vimos, denindo q(x) = a + b − 2x, temos
(x − a)(x − b)
a+b
f 0 (x) = q(x) · (f (x))2 , que é igual a zero somente quando x = 00 2
, e, de f (x) = 2(f (x)) ·
2
(f (x)(q(x))2 −1), temos f 00 ((a+b)/2) = 2(f ((a+b)/2))2 ·(−1) < 0. Além do que ϕ((a+b)/2) > 0.
Logo ϕ((a + b)/2) > ϕ(x) para todo x ∈ R distinto de (a + b)/2. 

8.36 Exercício 36
Obtenha funções f, g : R → R de classe C∞ com as seguintes propriedades:

i) f (x) = 0 ⇔ 0 6 x 6 1;
ii) g(x) = x se |x| 6 1, e |g(x)| < |x| se |x| > 1.
8.36. EXERCÍCIO 36 317

Solução. O exercício anterior e os exemplos referentes à função exponencial nos dão um bom
ponto de partida para encontrarmos as funções f e g. Denimos

e1/x , para x<0
f (x) = 0, para 06x61
 1/(1−x)
e , para x > 1.
Somar x com f quase nos dá g. Para corrigir esse quase, denimos primeiro a função h
fazendo 
e1/(x+1) ,
 para x < −1
h(x) = 0, para −16x61
1/(1−x)
−e , para x>1

e nalmente denimos
g(x) = x + h(x) ∀x ∈ R .
Para todo n ∈ N, existe um polinômio pn tal que
 
(n) 1 1/x
f (x) = pn − e se x<0
x
e um polinômio qn tal que
 
(n) 1
f (x) = qn − e1/(1−x) se x > 1.
1−x
Assim, fazendo y = −1/x e z = −1/(1 − x), temos

pn (y) qn (z)
f (n) (x) = se x < 0 e f (n) (x) = se x > 1,
ey ez

onde lim− y = lim+ z = +∞, de modo que a justicativa de que f é de classe C é semelhante
x→0 x→1
à do exercício anterior. Analogamente, h também é de classe C ∞ . Além disso, notamos que
0 < h(x) < 1 para todo x < −1 e −1 < h(x) < 0 para todo x > 1 e, portanto,

x < g(x) < 1 + x ∀x ∈ (−∞, −1)


e
−1 + x < g(x) < x ∀x ∈ (1, +∞),
donde |g(x)| < |x| se |x| > 1. 
318 CAPÍTULO 8. DERIVADAS

(Voltar para a Solução do Exercício 45, Segunda Parte: página 329.)

8.37 Exercício 37
Seja f :I→R duas vezes derivável no ponto a ∈ int(I). Então

f (a + h) + f (a − h) − 2f (a)
f 00 (a) = lim .
h→0 h2
Dê um exemplo em que existe o limite acima mas f 0 (a) não existe.

Solução. Vamos mostrar que

f (a + h) + f (a − h) − 2f (a) − f 00 (a)h2
lim = 0.
h→0 h2
Quem lida com limites semelhantes a este é o Teorema 9 da Fórmula de Taylor Innitesimal.
De acordo com ele, denindo

0 f 00 (a) 2
r(h) = f (a + h) − f (a) − f (a)h − h
2
(para todo h∈R tal que a + h ∈ I ), temos

r(h)
lim = 0.
h→0 h2

O sinal de menos em f (a − h) da igualdade da tese nos sugere considerar também r(−h) (o


que só faz sentido caso a − h ∈ I ). Claramente também temos

r(h) r(−h)
0 = lim 2
= lim .
h→0 h h→0 h2

Por outro lado, a soma r(h)+r(−h) é justamente igual a f (a+h)+f (a−h)−2f (a)−f 00 (a)h2 .
Porém, um cuidado que devemos tomar aqui é que, para ser lícito considerarmos o número
r(h) + r(−h), devemos ter simultaneamente a + h ∈ I e a − h ∈ I , o que é possível graças a
hipótese a ∈ int(I): existe δ > 0 tal que a + h ∈ I e a − h ∈ I para todo h cujo módulo é menor
do que δ . Podemos concluir então que

 
r(h) r(−h) r(h) + r(−h)
0 = lim 2
+ 2
= lim ,
h→0 h h h→0 h2

donde segue o resultado desejado.


Um exemplo para o que foi pedido é f : R → R, denida por f (0) = 0, f (x) = 1 se x>0 e
0
f (x) = −1 se x < 0, para a = 0: f (a) não existe e

f (h) + f (−h) − 2f (0)


lim = 0,
h→0 h2
f (h) + f (−h) − 2f (0) 0
pois
2
= 2 = 0, para todo h 6= 0. 
h h
8.38. EXERCÍCIO 38 319

8.38 Exercício 38
Seja f :I→R duas vezes derivável no ponto a ∈ int(I). Prove que

f (a + 2h) − 2f (a + h) + f (a)
f 00 (a) = lim .
h→0 h2

Solução. O mesmo procedimento do exercício anterior funciona aqui, onde, agora

f (a + 2h) − 2f (a + h) + f (a) − f 00 (a)h2 r(2h) − 2r(h) r(2h) r(h)


2
= 2
=4· 2
−2· 2 ,
h h (2h) h

se h 6= 0, a + h ∈ I e a + 2h ∈ I , para a mesma função r daquele exercício, e o δ > 0 tal que


|h| < δ implica ambos os pertencimentos a + h ∈ I e a + 2h ∈ I é δ = δ0 /2, sendo δ0 aquele
número positivo tal que (a − δ0 , a + δ0 ) ⊂ I , o qual existe pelo fato de a ser ponto interior de
I. 

8.39 Exercício 39
Sejam f, g : R → R n vezes deriváveis. Prove que se, para algum a∈R vale f 0 (a) = · · · =
(n)
f (a) = 0 então (g ◦ f )(i) (a) = 0 para i = 1, . . . , n.

Solução.

No elevador. Uma possível estratégia para resolver este exercício é a conjectura de que
(n)
(g ◦ f ) (a) tem o seguinte aspecto

(g ◦ f )(n) (a) = f 0 (a)h1 (a) + f (2) (a)h2 (a) + · · · + f (n) (a)hn (a).

De fato esta igualdade implica o resultado desejado. O problema com ela é que, ao tentarmos
demonstrá-la por indução em n, veríamos a necessidade de mais uma hipótese acerca dos hk 's
que nela aparecem, a saber, a de que eles devem ser deriváveis. A estratégia adotada na solução
abaixo foi adaptar esta conjectura.

Nível 1. Armamos que, para cada i ∈ {1, . . . , n}, existem funções h1,i , h2,i , . . . , hi,i de R
em R, cada uma pelo menos n−i vezes derivável (lembrando que a noção de 0 vez derivável
também foi denida no livro), tais que

(g ◦ f )(i) (a) = f 0 (a)h1,i (a) + f (2) (a)h2,i (a) + · · · + f (i) (a)hi,i (a).

Vamos demonstrar esta armação no Nível 2 (e desdobramentos). Desta igualdade, dado i ∈


{1, . . . , n}, como f (k) (a) = 0 para cada k ∈ {1, . . . , i} por hipótese, segue (g ◦ f )(i) (a) = 0, como
queríamos.

Nível 2. Vamos mostrar a armação acima para i = 1 (Nível 2.1) e que o fato de valer
para i implica valer para i + 1, para cada i ∈ {1, . . . , n − 1} (Nível 2.2). Assim, a armação
segue do lema abaixo, um resultado semelhante ao do Princípio da Indução.
320 CAPÍTULO 8. DERIVADAS

Nível 2.1. Para i=1 temos

(g ◦ f )(i) (a) = f 0 (a) · (g 0 ◦ f )(a).

Então denimos a função h1,1 como sendo a função g 0 ◦ f , função esta que é n − 1 = n − i vezes
derivável, porque ambas g 0 e f o são (Exercício 33, página 311).

Nível 2.2. Fixado i ∈ {1, . . . , n − 1} vamos supor que

(g ◦ f )(i) (a) = f 0 (a)h1,i (a) + f (2) (a)h2,i (a) + · · · + f (i) (a)hi,i (a),

onde hk,i é n − i vezes derivável para cada k ∈ {1, . . . , i}. Em particular, cada hk,i , assim como
cada f (k) , é derivável e podemos fazer

(g ◦ f )(i+1) (a) = f 0 (a)h01,i (a) + f (2) (a)h1,i (a) + f (2) (a)h02,i (a) + f (3) (a)h2,i (a) + · · · +
 

+ f (i−1) (a)h0i−1,i (a) + f (i) (a)hi−1,i (a) + f (i) (a)h0i,i (a) + f (i+1) (a)hi,i (a) .
 

Assim denimos h1,i+1 = h01,i , h2,i+1 = h1,i + h02,i , . . . , hi,i+1 = hi−1,i + h0i,i e hi+1,i+1 = hi,i . A
função hi+1,i+1 é n − i vezes derivável e, portanto, n − i − 1 = n − (i + 1) vezes derivável e cada
uma das outras funções hk,i+1 's também é n − (i + 1) vezes derivável, porque cada uma delas é
uma soma de funções n − 1 − i vezes deriváveis (Nível 4 do Exercício 33, página 312).

Lema. Dado um número natural n, seja X um conjunto de números naturais com as duas
propriedades seguintes: (a) 1 ∈ X , e; (b) i ∈ X ⇒ i + 1 ∈ X , para todo i ∈ {1, . . . , n − 1}.
Então {1, . . . , n} ⊂ X .
(Voltar para a Solução do Exercício 43, Segunda Parte: página 325.)

Solução. Podemos ver isso por indução em n. Para n = 1, a inclusão {1} ⊂ X é imediata
do item (a). Fixado n ∈ N, i ∈ X ⇒ i + 1 ∈ X , para todo i ∈ {1, . . . , n}
o item (b) diz que
(colocando n + 1 no lugar de n). Em particular, esta implicação vale para todo i ∈ {1, . . . , n −
1}, de modo que, se este lema vale para n, então {1, . . . , n} ⊂ X . Agora concluímos que o
número n + 1 também pertence a X aplicando o item (b) para i = n (pois n ∈ X ). Portanto
{1, . . . , n + 1} ⊂ X . Logo, por indução em n, segue a tese. 

8.40 Exercício 40
0 0 0
Sejam f, g : R → R 4 vezes deriváveis.
Escreva a regra da cadeia sob a forma (g◦f ) = (g ◦f )·f
00 00 0 2 0 00
onde o ponto signica multiplicação de funções. Conclua que (g◦f ) = (g ◦f )·(f ) +(g ◦f )·f .
000 000 0 3 00 00 0 0 000
Mostre também que (g ◦ f ) = (g ◦ f ) · (f ) + 3(g ◦ f ) · f · f + (g ◦ f ) · f . Calcule ainda
(g ◦ f )(4) .

Solução. Temos, pelos Teoremas 2 e 3,

(g ◦ f )00 = [(g 0 ◦ f ) · f 0 ]0
= (g 0 ◦ f )0 · f 0 + (g 0 ◦ f ) · f 00
= ((g 00 ◦ f ) · f 0 ) · f 0 + (g 0 ◦ f ) · f 00
= (g 00 ◦ f ) · (f 0 )2 + (g 0 ◦ f ) · f 00 ,

como queríamos.
8.41. EXERCÍCIO 41 321

Também

(g ◦ f )000 = [(g 00 ◦ f ) · (f 0 )2 + (g 0 ◦ f ) · f 00 ]0
= ((g 000 ◦ f ) · f 0 ) · (f 0 )2 + (g 00 ◦ f ) · 2(f 0 ) · f 00 + ((g 00 ◦ f ) · f 0 ) · f 00 + (g 0 ◦ f ) · f 000
= (g 000 ◦ f ) · (f 0 )3 + 3(g 00 ◦ f ) · f 0 · f 00 + (g 0 ◦ f ) · f 000

(o produto de funções é comutativo), como queríamos.


Ainda

(g ◦ f )(4) = [(g 000 ◦ f ) · (f 0 )3 + 3(g 00 ◦ f ) · f 00 · f 0 + (g 0 ◦ f ) · f 000 ]0


= (g (4) ◦ f ) · (f 0 )4 + 3(g 000 ◦ f ) · (f 0 )2 · f 00 + 3(g 000 ◦ f ) · (f 0 )2 · f 00 + 3(g 00 ◦ f ) · f 000 · f 0 +
3(g 00 ◦ f ) · (f 00 )2 + (g 00 ◦ f ) · f 0 · f 000 + (g 0 ◦ f ) · f (4)
= (g (4) ◦ f ) · (f 0 )4 + 6(g 000 ◦ f ) · (f 0 )2 · f 00 + 4(g 00 ◦ f ) · f 0 · f 000 + 3(g 00 ◦ f ) · (f 00 )2 +
(g 0 ◦ f ) · f (4) .

Logo

(g ◦ f )(4) = (g (4) ◦ f ) · (f 0 )4 + 6(g 000 ◦ f ) · (f 0 )2 · f 00 + 4(g 00 ◦ f ) · f 0 · f 000 + 3(g 00 ◦ f ) · (f 00 )2 + (g 0 ◦ f ) · f (4) .

8.41 Exercício 41
Sejam f, g : R → R n vezes deriváveis. Para cada partição n = n1 + · · · + nk de n como soma
de k números naturais, existe um inteiro α = α(n1 , . . . , nk ) tal que

n
X
(g ◦ f )(n) = α(n1 , . . . , nk ) · (g (k) ◦ f ) · f (n1 ) · f (n2 ) · · · f (nk ) ,
k=1

onde, para cada k = 1, 2, . . . , n, vale n1 + n2 + · · · + nk = n.

Solução.

No elevador. Por exemplo, conforme vimos no exercício anterior, para n = 4, α(1, 1, 1, 1)


= 1 = α(4), α(1, 1, 2) = 6, α(1, 3) = 4 e α(2, 2) = 3.

Nível 1. Vamos proceder por indução em n. No Nível 2.1 vamos mostrar que esta ar-
mação é verdadeira para n = 1 e, no Nível 2.2, que ela é verdadeira para n+1 supondo-a
verdadeira para n.

Nível 2.1. Para n=1 existe uma única partição de n; para ela temos α(1) = 1.

No elevador. Dada uma partição n + 1 = n1 + n2 + · · · + nk ,


queremos denir o número
(n)
α(n1 , · · · , nk ) a partir dos α's que aparecem no desenvolvimento de (g ◦ f ) na forma em
questão. Esta partição de n+1 só pode ter vindo de uma partição da forma n = n1 + · · · +
322 CAPÍTULO 8. DERIVADAS

ni−1 + (ni − 1) + ni+1 + · · · + nk , com i ∈ {1, 2, . . . , k}, para n. Então poderíamos começar
conjecturando que

k
X
α(n1 , . . . , nk ) = α(n1 , . . . , ni−1 , ni − 1, ni+1 , . . . , nk ),
i=1

interpretando o número α(n1 , . . . , ni−1 , 0, ni+1 . . . , nk ) como sendo o número α(n1 , . . . , ni−1 , ni+1 ,
. . . , nk ). Mas há dois problemas com a igualdade acima. Um é que um mesmo número pode
estar sendo contado mais de uma vez de forma inapropriada. Por exemplo, 7 = 2 + 2 + 3, que,
pelo raciocínio acima, veio das partições 6 = 1 + 2 + 3, 6 = 2 + 1 + 3 e 6 = 2 + 2 + 2. Mas
o número α(1, 2, 3) foi contado duas vezes de modo indevido, porque α(1, 2, 3) = α(2, 1, 3). O
outro é que pode acontecer também o oposto, um número pode ter sido contado menos vezes
do que deveria. O mesmo exemplo anterior serve, nele, o número α(2, 2, 2) foi contado apenas
uma vez, quando deveria ser contado 3 vezes. Então, neste caso, na verdade, temos

1 1
α(2, 2, 3) = · α(1, 2, 3) + · α(2, 1, 3) + 3 · α(2, 2, 2).
2 2
A divisão por 2 nas duas primeiras parcelas acima foi feita porque, sendo n1 = n2 = 2, em
7 = 2 + 2 + 3, há duas parcelas iguais a n1 e 2 parcelas iguais a n2 ; e a multiplicação por 3,
porque, em 6 = 2 + 2 + 2, há 3 parcelas iguais a n3 − 1, onde n3 = 3.

Nível 2.2. Dada a partição n + 1 = n1 + · · · + nk para n + 1, para cada i ∈ {1, 2, . . . , k},


denimos
αi = α(n1 , . . . , ni−1 , ni − 1, ni+1 , . . . , nk )
se ni 6= 1,
αi = α(n1 , . . . , ni−1 , ni+1 , . . . , nk )
se ni = 1, e

card {j ∈ {1, . . . , k}; nj = ni − 1} + 1
xi =  .
card {j ∈ {1, . . . , k}; nj = ni }

k
X
Desta forma, xi α i é o número α(n1 , . . . , nk ) procurado. Este número é inteiro porque
i=1
nj = ni ⇒ αj = αi , assim podemos agrupar as parcelas do somatório anterior de modo a
fazer com que o denominador de cada xi desapareça. Mais precisamente, denindo I =
{1, 2, · · · , k}, {n1 , · · · , nk } = {m1 , · · · , ml } (isto é, mi 6= mj se i 6= j ), Ip = {i ∈ I; ni = mp } e
{αi ∈ R; i ∈ Ip } = {ap } (estas duas últimas denições para cada p ∈ {1, · · · , l}), temos

k
X l X
X l
X  

xi α i = xi αi = ap · card {j ∈ I; nj = mp − 1} + 1 ,
i=1 p=1 i∈Ip p=1

sendo cada ap inteiro pela hipótese de indução. 

8.42 Exercício 42
A função f (x) = |x|2n+1 é de classe C 2n na reta inteira mas não é 2n + 1 vezes derivável.
8.43. EXERCÍCIO 43 323

Solução. Para x 6= 0, f admite derivadas de todas as ordens dadas por

(2n + 1)!
f (k) (x) = |x|2n+1−k ∀x 6= 0 ∀k ∈ N .
(2n + 1 − k)!
Em especial,

(2n) (2n+1) (2n + 1)! se x>0
f (x) = (2n + 1)! · |x| ∀x 6= 0 e f (x) =
−(2n + 1)! se x < 0.

Disto já podemos ver que f (2n)


não pode ser derivável no zero, porque, caso f
(2n+1)
(0) estivesse
(2n+1)
denido, a descontinuidade de f em 0 seria de primeira espécie, contrariando o Corolário
do Teorema 5. Portanto 2n + 1 vezes derivável f não é.
Com relação à origem, podemos proceder como no Exemplo 24; f é contínua no 0 e
lim f 0 (x) = 0, portanto, pelo Corolário 5 do Teorema 7, f é derivável no 0 e f 0 (0) = 0. Para
x→0
k ∈ {0, 1, . . . , 2n−1}, se f (k) é contínua no 0, então, novamente por este corolário, f (k+1) (0) = 0,
(k+1)
pois lim f (x) = 0. Assim, por indução em k , segue que f (k) está denida na reta inteira e
x→0
2n
é contínua, para todo k ∈ {0, 1, . . . , 2n}, logo, f é de classe C como queríamos. 

8.43 Exercício 43
Seja f : R → R de classe C ∞ . Se f se anula, juntamente com todas as suas derivadas, num
k
ponto a ∈ R, então, para cada k ∈ N, podemos escrever f (x) = (x − a) · ϕ(x), onde ϕ : R → R
∞ n 0 (n)
é de classe C . Se f é de classe C e f (a) = f (a) = · · · = f (a) = 0 então, para k 6 n temos
k
f (x) = (x − a) ϕ(x), com ϕ : R → R de classe n − k .

Solução.

Primeira Parte.

No elevador. O ponto central nesta solução é a Fórmula de Taylor Innitesimal. Para r


tal que
f (x) = f (a) + f 0 (a)(x − a) + · · · + f (k) (a)(x − a)k + r(x − a)
para todo x ∈ R, ou seja, tal que r(x − a) = f (x) (pois f (a) = f 0 (a) = · · · = f (k) (x) = 0 por
hipótese) para todo x ∈ R, temos

r(x − a) f (x)
lim k
= lim = 0.
x→a (x − a) x→a (x − a)k

É aí que aparece a função ϕ,


que será contínua denindo ϕ(a) = 0. Nada nos impede de
(m)
aplicarmos novamente este teorema para f no lugar de f e desenvolvermos esta fórmula até
a ordem que quisermos.

Nível 1. Denimos ϕ:R→R fazendo



f (x)
, se x 6= a

ϕ(x) = (x − a)k
0, se x = a.

324 CAPÍTULO 8. DERIVADAS

Em x 6= a ela é innitamente derivável. Para ver que ϕ é innitamente derivável em x = a,


assim como no Exemplo 24 do livro e também como na Solução do Exercício 35 acima (página
314), vamos mostrar no Nível 2 (e desdobramentos) que

lim ϕ(n) (x) = 0 ∀n ∈ N ∪{0}.


x→a

Disto vem ϕ(n) (a) = 0 para todo n ∈ N,


de acordo com o Corolário 5 do Teorema 7 (junto com

o fato de derivabilidade implicar continuidade). Logo, ϕ é de classe C .

No elevador. Os cálculos de ϕ0 (x) e de ϕ00 (x) para x 6= a nos permitem fazer uma
conjectura da qual vem a armação do nível abaixo.

Nível 2. Dado n ∈ N ∪{0}, armamos que existem números reais A0,n , A1,n , . . . , An,n tais
que
n
(n)
X f (n−i) (x)
ϕ (x) = Ai,n ∀x ∈ R −{a}.
i=0
(x − a)k+i

A demonstração disto está no Nível 3.1. Armamos também que

f (m) (x)
lim =0 ∀m ∈ N ∪{0} ∀l ∈ N .
x→a (x − a)l

Nível 3.2. Destas armações segue

lim ϕ(n) (x) = 0,


x→a

como desejado.

Nível 3.1. Vamos demonstrar a referida armação por indução em n. Para n = 0, temos

f (x)
ϕ(n) (x) = ϕ(x) = ∀x ∈ R −{a}.
(x − a)k

Portanto A0,0 = 1. Supondo a armação verdadeira para um dado n ∈ N ∪{0}, derivando uma
parcela genérica da soma na qual ϕ(n) (x) foi escrito, temos
0
f (n−i) (x) f (n−i+1) (x)(x − a)k+i − (k + i)(x − a)k+i−1 f (n−i) (x)

Ai,n = Ai,n
(x − a)k+i (x − a)2k+2i
f (n−(i−1)) (x) f (n−i) (x)
= Ai,n − (k + i)A i,n .
(x − a)k+i (x − a)k+(i+1)

A partir daí podemos concluir que:

A0,n+1 = A0,n ,
Ai,n+1 = −(k + (i − 1))Ai−1,n + Ai,n para i ∈ {1, . . . , n}
e

An+1,n+1 = −(k + n)An,n .


Segue a tese.
8.43. EXERCÍCIO 43 325

Nível 3.2. Dados m ∈ N ∪{0} e l ∈ N, pela Fórmula de Taylor Innitesimal (Teorema 9)


escrevemos

f (m+l) (a)
f (m) (x) = f (m) (a) + f (m+1) (a)(x − a) + · · · + (x − a)l + r(x − a)
l!
= r(x − a),

para todo x ∈ R; igual a r(x − a) por causa da hipótese de f, juntamente com todas suas
r(x − a)
derivadas, se anularem em a; r tal que lim = 0, ou seja
x→a (x − a)l

f (m) (x)
lim = 0,
x→a (x − a)l

como queríamos.

Segunda Parte. Algumas adaptações na primeira parte nos dão a solução desta segunda
f (x)
parte. Denimos ϕ da mesma forma como lá, isto é, fazendo (k ∈ N ∩[1, n]
ϕ(x) =
(x − a)k
n−k
xado) se x 6= a e ϕ(a) = 0. Em (a, +∞) e em (−∞, a), ϕ é de classe C (Nível 3 do
Exercício 33, página 312). Com relação a a temos o seguinte. Para cada m ∈ {0, 1, . . . , n − k},
existem números reais A0,m , A1,m , . . . , Am,m tais que

m
(m)
X f (m−i) (x)
ϕ (x) = Ai,m ∀x ∈ R −{a},
i=0
(x − a)k+i

onde, pelo Fórmula de Taylor Innitesimal (Teorema 9), como cada f (m−i) é k+i vezes derivável
em a (por hipótese), existe uma função r tal que

f (m+k) (a)
f (m−i) (x) = f (m−i) (a) + f (m−i+1) (a)(x − a) + · · · + (x − a)k+i + r(x − a) ∀x ∈ R
(k + i)!

r(x − a)
e lim = 0. Deste modo, como f (a) = f 0 (a) = · · · = f (n) (a) = 0 por hipótese, segue
x→a (x − a)k+i

r(x − a) f (m−i) (x)


0 = lim = lim .
x→a (x − a)k+i x→a (x − a)k+i

Essas considerações implicamlim ϕ(m) (x) = 0 para todo m ∈ {0, 1, . . . , n − k}. Assim,
x→a
aplicando sucessivamente o Corolário 5 do Teorema 7 em conjunto com o Teorema 1, concluímos
(por indução em m nos moldes do lema enunciado na Solução do Exercício 39, página 320) que
ϕ(m) (a) = 0 para todo m ∈ {0, 1, . . . , n − k}. A ϕ(n−k) é contínua em a porque

lim ϕ(n−k) (x) = 0 = ϕ(n−k) (a).


x→a

Logo, ϕ é de classe C n−k . 


326 CAPÍTULO 8. DERIVADAS

8.44 Exercício 44
Seja f :I→R de classe C n+1 , com a ∈ I. Temos

f (n−1) (a) n−1 f (n) (a + θn · h) n


f (a + h) = f (a) + f 0 (a) · h + · · · + h + ·h .
(n − 1)! n!

Mais precisamente, para todo h tal que a + h ∈ I, podemos encontrar θn = θn (h), com
0 < θn < 1, tal que a fórmula acima vale. Mostre que, se f (n+1) (a) 6= 0, seja qual for a função
θn , denida da maneira acima, tem-se

1
lim θn (h) = .
h→0 n+1
[ Sugestão : Desenvolva f (a+h) segundo Taylor-Lagrange até a ordem n+1, compare o resultado
obtido com a expressão acima e use o Teorema do Valor Médio].

Solução. Em primeiro lugar, podemos supor que I é um intervalo porque f é uma função
n+1
de classe C e o livro deniu o conceito de função m vezes derivável apenas no caso em
que seu domínio é um intervalo. A existência de cada um desses θn (h)'s pode ser justicada
através do Teorema da Fórmula de Taylor com Resto de Lagrange (Teorema 10); para h positivo
consideramos a restrição f |[a, a + h] e, para h negativo, a função g : [a + h, a] → R denida por
g(x) = f (2a + h − x) para todo x ∈ [a + h, h].
Vamos seguir a sugestão dada. Do mesmo modo como feito no parágrafo acima (o Teo-
rema 10 pode ser aplicado da forma sugerida porque f também é de classe C n , uma vez que
derivabilidade implica continuidade), para cada h∈R tal que a + h ∈ I, seja θ ∈ (0, 1) tal que

f (n−1) (a) n−1 f (n) (a) n f (n+1) (a + θ · h) n+1


f (a + h) = f (a) + f 0 (a) · h + · · · + h + h + ·h .
(n − 1)! n! (n + 1)!
Comparando esta igualdade com a dada no enunciado, obtemos

f (n) (a) n f (n+1) (a + θ · h) n+1 f (n) (a + θn (h)h) n


h + ·h = ·h .
n! (n + 1)! n!

Multiplicando ambos os membros da igualdade acima por n!/hn (para h 6= 0, já que nosso
objetivo é fazer esse h tender a zero por valores distintos de zero) e a reescrevendo de modo
conveniente para aplicarmos o Teorema do Valor Médio, vem

f (n+1) (a + θ · h)
· h = f (n) (a + θn (h)h) − f (n) (a).
n+1
Por este referido Teorema (que é o 7), existe b pertencente ao interior do intervalo de
extremos a e a + θn (h)h tal que

f (n+1) (a + θ · h)
· h = f (n+1) (b) · θn (h)h
n+1
(n)
(f é derivável neste intervalo). Conforme h se aproxima de zero, o número a + θn (h)h se
aproxima de a (produto de uma função limitada por uma que tende a zero, Teorema 7 do
Capítulo VI), de modo que b também se aproxima de a (Teorema 4, Capítulo VI). Assim,
f (n+1) , como f (n+1) (a) 6= 0 por hipótese, temos f (n+1) (b) 6= 0 para todo
pela continuidade de
8.45. EXERCÍCIO 45 327

b próximo o suciente de a (Corolário, com suas observações, do Teorema 3 do Capítulo VII).


Obtemos então um δ>0 tal que 0 < |h| < δ e a + h ∈ I implicam f (n+1) (b) 6= 0 e, assim, da
igualdade acima, vem
f (n+1) (a + θ · h)
θn (h) = .
(n + 1)f (n+1) (b)
1
Finalmente podemos concluir o resultado desejado,lim θn (h) = , pelos Teoremas 5 e
h→0 n+1
n+1
6 do Capítulo VII e pelo Teorema 7 do Capítulo VI (f é contínua em a e lim a + θ · h = a,
h→0
porque θ é limitada). 

8.45 Exercício 45
Sejam f, g analíticas no intervalo aberto I . Se existe a ∈ I tal que f e g coincidem, juntamente
com todas suas derivadas, no ponto a, então f (x) = g(x) para todo x ∈ I . Mostre que isto
seria falso se supuséssemos apenas f e g de classe C ∞ .

Solução.

Primeira Parte.

Nível 1. Vamos mostrar que f (n) (a) = 0 para todo n ∈ N ∪{0} implica f (x) = 0 para
todo x ∈ I. Vamos fazê-lo no Nível 2.1 (e sub-níveis). Com isto:

f (n) (a) = g (n) (a) ∀n ∈ N ∪{0} ⇒ (f (n) − g (n) )(a) = (f − g)(n) (a) = 0 ∀n ∈ N ∪{0}
⇒ (f − g)(x) = 0 ∀x ∈ I
⇒ f (x) = g(x) ∀x ∈ I,
como queremos, desde que f −g seja uma função analítica, como é o caso (Nível 2.2).


X f (n) (a)
No elevador. Em af é analítica, então existe ε > 0 tal que f (x) = (x−a)n = 0
n=0
n!
para todo x ∈ (a − ε, a + ε) = I0 . Por causa da continuidade de f e de todas suas derivadas,
tudo aquilo que f tem no intervalo não degenerado I0 é transportado para o ponto a + ε, quer
(n)
dizer, f (a + ε) = 0 para todo n ∈ N ∪{0} e podemos repetir com o ponto a + ε = a1 o mesmo
que zemos com o ponto a, obtendo ε1 > 0 tal que f (x) = 0 para todo x ∈ (a1 − ε1 , a1 + ε1 ).
Agora temos f identicamente nula num intervalo um pouco maior do que antes. Continuando
assim, obtemos uma sequência crescente (an )n∈N tal que f (x) = 0 para todo x ∈ (a − ε, an )
e para todo n ∈ N. Se essa sequência não for limitada, então estamos feitos. Mas e se ela
for limitada? Se limitada, então convergente, porque monotóna. Sendo α seu limite podemos
concluir que f também se anula no intervalo (a − ε, α), pois se x ∈ (a − ε, α), então existe
n ∈ N tal que x ∈ (a − ε, an ). Certo... mas e daí? Isto que aconteceu com o ponto α também
nos permite dizer que o conjunto de todo ponto x tal que f se anula no intervalo (a − ε, x) se
comporta como um conjunto fechado. Claro que (a − ε, x) precisa estar contido em I para que
isto faça sentido. Um tal conjunto se comportando como fechado e sem elemento máximo não
pode ser limitado, pois, caso contrário, ele admitiria um supremo e este supremo pertenceria
a ele, portanto, um elemento máximo... Aparar as pontas deste raciocínio nos dá a solução
pretendida.
328 CAPÍTULO 8. DERIVADAS

Nível 2.1. Seja X = {x ∈ I ∩ [a, +∞); f (y) = 0 ∀y ∈ [a, x]}. Armamos que X tem as
duas propriedades seguintes

1. X não tem elemento máximo (Nível 3.1), e ;

2. se (an )n∈N é uma sequência de pontos em X que converge para um ponto α ∈ I, então
α ∈ X (Nível 3.2).

Nestas condições armamos que X = I ∩ [a, +∞) (Nível 3.3). Analogamente, denimos o
conjunto Y = {x ∈ I ∩ (−∞, a]; f (y) = 0 ∀y ∈ [x, a]} e concluímos que Y = I ∩ (−∞, a], ou
seja, I = X ∪ Y , donde segue a tese: f se anula em todo o intervalo I .

Nível 2.2: Lema. Se f e g são analíticas no intervalo aberto I , então, dado λ ∈ R, f + λg


é analítica em I.
Solução. Dado α ∈ I, existem δ1 > 0 e δ2 > 0 tais que

+∞ (n)
X f (α)
f (x) = (x − α)n ∀x ∈ (α − δ1 , α + δ1 )
n=0
n!

e
+∞ (n)
X g (α)
g(x) = (x − α)n ∀x ∈ (α − δ2 , α + δ2 ).
n=0
n!

Deste modo, denindo δ = min{δ1 , δ2 }, lembrando da soma de séries e do produto de uma série
por um número real (Exemplo 25 do Capítulo IV), para todo x ∈ (α − δ, α + δ), temos

+∞  (n)
g (n) (α)

X f (α) n
(f + λg)(x) = (x − α) + λ · (x − α)n
n=0
n! n!
+∞
X (f + λg)(n) (α)
= (x − α)n ,
n=0
n!

onde esta última série é a de Taylor de f + λg em torno de α. Logo f + λg é analítica em α e


portanto em I, uma vez que α era um ponto qualquer de I, como queríamos.

Nível 3.1. O não tem elemento máximo por causa da analiticidade de f . Se x ∈ X ,


X
(n)
então f (y) = 0 para todo y ∈ [a, x], o que implica f (x) = 0 para todo n ∈ N ∪{0}, do fato
de f ser innitamente derivável em x (se x > a; por hipótese se x = a). Como f é analítica em
x, obtemos ε > 0 tal que f (y) = 0 para todo y ∈ (x − ε, x + ε). Daí, qualquer x0 ∈ (x, x + ε) é
tal que f (y) = 0 para todo y ∈ [a, x0 ], o que faz x0 pertencer a X , sendo x0 > x. Ou seja, X
não tem elemento máximo.

Nível 3.2. x ∈ [a, α] distinto de α, por causa da convergência an → α, existe an tal


Dado
que x ∈ [a, an ], portanto f (x) = 0, uma vez que an ∈ X por hipótese. Além disso, f (α) = 0
por causa da continuidade de f ; f (α) = lim f (an ) = 0. Logo α ∈ X .
8.46. EXERCÍCIO 46 329

Nível 3.3. Primeiro que X não é um conjunto vazio; a ∈ X . Se I ∩ [a, +∞) = [a, β) para
algum β ∈ R, então X é um conjunto limitado superiormente, de modo que ele admite um
supremo sup X . Queremos mostrar, neste caso, que sup X = β . Como X ⊂ I , se sup X 6= β ,
então sup X < β , sup X ∈ I e teríamos sup X ∈ X pela propriedade 2, portanto, sup X seria
o elemento máximo de X , contrariando a propriedade 1. Logo sup X = β como queríamos.
Por motivo semelhante, se I não é limitado superiormente, então X também não é limitado
superiormente. De qualquer forma (I sendo limitado superiormente ou não) segue X = I ∩
[a, +∞), como queríamos.

Segunda Parte. Para ver que apenas supor essas funções C∞ não basta, retomamos a
função ϕ do Exercício 35 (página 314) e as funções f e g da Solução do Exercício 36 (página
(n)
317); R é um intervalo aberto, ϕ (a) = f (n) (0) = 0 e g (n) (0) = id(n) (0) (onde id é a função
identidade) para todo n ∈ N ∪{0}, mas ϕ e f não são a mesma função (e fazemos a = 0),
também nenhuma delas é identicamente nula, e g não coincide com a função identidade em
toda a reta. O Exemplo 29 do livro também é um contra-exemplo para o caso das funções
serem apenas C ∞. 
(Ver também o Teorema 17 do Capítulo X.)

8.46 Exercício 46
Dadas f e g X ⊂ I um conjunto que possui um ponto de
analíticas no intervalo aberto I, seja
acumulação a ∈ I . Se f (x) = g(x) para todo x ∈ X então f (x) = g(x) para todo x ∈ I . Em
particular, se f (x) = 0 para todo x ∈ X então f (x) = 0 para todo x ∈ I .

Solução. Assim como na solução do exercício anterior, e por motivo semelhante, basta consi-
(n)
derarmos o caso em que f (x) = 0 para todo x ∈ X . Neste caso, vamos mostrar que f (a) = 0
para todo n ∈ N ∪{0} e, assim, pelo referido exercício, teremos f (x) = 0 para todo x ∈ I, uma
vez que f é analítica. Vamos fazer isto por indução em n, começando com n = 0. Como a é
ponto de acumulação de X , xemos desde já uma sequência de pontos (xm )m∈N em X tal que
xm 6= a para todo m∈N e lim xm = a.
Em a, f vale 0 por causa de sua continuidade: f (a) = lim f (xm ) = 0. Fixado n ∈ N ∪{0},
pelo Teorema 9, como f é n+1 vezes derivável em a, temos
f (n) (a) f (n+1) (a)
f (x) = f (a) + f 0 (a)(x − a) + · · · + (x − a)n + (x − a)n+1 + r(x − a)
n! (n + 1)!
r(x − a)
para todo x ∈ I , onde r é tal que lim = 0. Da igualdade acima, supondo f (k) (a) = 0
x→a (x − a)n+1
para todo k ∈ {0, 1, . . . , n}, e lembrando que f (xm ) = 0 para todo m ∈ N, obtemos

f (n+1) (a)
r(xm − a) = − (xm − a)n+1 ∀m ∈ N,
(n + 1)!
donde
r(xm − a) f (n+1) (a)
0 = lim = lim −
m→+∞ (xm − a)n+1 m→+∞ (n + 1)!
(n+1)
e portanto só pode ser f (a) = 0, o que completa esta demonstração por indução, isto é,
(n)
f (a) = 0 para todo n ∈ N ∪{0}. 
(Ver também o Teorema 18 do Capítulo X.)
330 CAPÍTULO 8. DERIVADAS

8.47 Exercício 47
Seja I = (a − δ, a + δ). Dada f : I → R, C ∞ , suponha que existam constantes
de classe
X∞
an (x − a)n . Prove que
P
a0 , a1 , . . . , an , . . . tais que, para todo x ∈ I se tenha f (x) = an (x −
n=0
(n)
f (a)
a)n é a série de Taylor de f em torno de a, isto é, que an = para todo n = 0, 1, 2, . . ..
n!

Solução.

Nível 1. Para tanto, vamos utilizar a Fórmula de Taylor Innitesimal (Teorema 9). Fixado
n ∈ N, de

X
n
f (x) = a0 + a1 (x − a) + · · · + an (x − a) + ak (x − a)k ,
k=n+1

X
denimos r(x − a) = ak (x − a)k e vamos mostrar no Nível 2 (e níveis subsequentes) que
k=n+1

r(x − a)
lim = 0.
x→a (x − a)n

Disto, como f é n vezes derivável em a, pelo Teorema 9, segue

f (n) (a)
a0 + a1 (x − a) + · · · + an (x − a)n = f (a) + f 0 (a)(x − a) + · · · + (x − a)n
n!
f (k) (a)
para todo x ∈ I , donde ak = para cada k ∈ {0, 1, . . . , n}. Como isto pode ser feito para
k!
f (n) (a)
qualquer número natural n, concluímos que an = para todo n ∈ N ∪{0}.
n!

Nível 2. Para x 6= a (x ∈ I ), temos


r(x − a) X
= ak (x − a)k−n
(x − a)n k=n+1

= (x − a) · an+1 + an+2 (x − a) + an+3 (x − a)2 + · · ·




(lembrando dos Exemplos 25 e 28 do Capítulo IV). Agora, para concluir o resultado desejado,
X∞
denindo g(x) = an+k (x − a)k−1 para todo x ∈ I − {a} e g(a) = an+1 (de fato g(x) ∈ R
k=1
para todo x ∈ I − {a} novamente pelo Exemplo 25 do Capítulo IV), vamos mostrar no Nível 3
que g é limitada numa vizinhança de a e, portanto,

r(x − a)
lim =0
x→a (x − a)n

pelos Teoremas 2 e 7 do Capítulo VI (convergência numa vizinhança e produto de uma função


que tende a zero por uma limitada, respectivamente).
8.47. EXERCÍCIO 47 331

No elevador. Vamos lidar com um número da forma |g(x)|. Se |x − a| < |y − a|, então
|an+k (x−a)k−1 | 6 |an+k (y −a)k−1 | para todo k ∈ N. Se as convergências de g(x) e de g(y) forem

X ∞
X
k−1
absolutas, então |an+k (x − a) | 6 |an+k (y − a)k−1 |. Se também valer uma espécie de
k=1 k=1
Desigualdade Triangular para uma soma com innitas parcelas, então teremos


X ∞
X
k−1
|g(x)| 6 |an+k (x − a) |6 |an+k (y − a)k−1 |,
k=1 k=1

desigualdades estas que nos ajudarão a mostrar o resultado desejado.

Nível 3. x0 ∈ I distinto de a, armamos que a série g(x) é absolutamente conver-


Fixado
gente qualquer que seja x ∈ I tal que |x − a| < |x0 − a| (Nível 4.1). Assim, xamos um número
X∞
y0 ∈ I diferente de a tal que |y0 −a| < |x0 −a| e consideramos o número A = |an+k (y0 −a)k−1 |.
k=1
A primeira desigualdade envolvendo séries nas contas acima (no elevador) vale por causa do
Corolário 1 do Teorema 7, Capítulo IV. Além disso, armamos também que vale a Desigualdade
Triangular para Séries (Nível 4.2). Logo, conforme as contas do parágrafo anterior indicam,
temos |g(x)| 6 A para todo x ∈ (a − |y0 − a|, a + |y0 − a|) como desejado.


X
No elevador. Vamos comparar uma série da forma |αn (x−a)n | com uma série geomé-
n=0

x − a n

x−a X
trica. Se |x − a| < |x0 − a|, então
x0 − a < 1, de sorte que a série x0 − a é convergente.

n=0
n n
Então, a partir do número |αn (x − a) |, vamos fazer aparecer um (x0 − a) no denominador.
Isto pode ser feito do seguinte modo

|(x0 − a)n | x − a n

n n n
|αn (x − a) | = |αn (x − a) | · = |αn (x0 − a) | · .
|(x0 − a)n | x0 − a

X
Agora, se tivermos a convergência de αn (x0 − a)n , então poderemos controlar o fator
n=0
|αn (x0 − a)n | que aparece na última igualdade acima.


X
Nível 4.1: Lema. Se a série αn (x0 − a)n é convergente, sendo x0 um número real
n=0

X
diferente de a, então a série αn (x − a)n é absolutamente convergente qualquer que seja o x
n=0
real tal que |x − a| < |x0 − a|.

Solução.
X
A convergência da série αn (x0 − a)n implica a existência de um número c>0
n=0
tal que |αn (x0 − a)n | 6 c para todo n ∈ N. Isto porque o limite do termo geral de uma
série convergente é zero (Teorema 15 do Capítulo IV) e sequências convergentes são limitadas
(Teorema 3 do Capítulo IV). Portanto, de acordo com as contas acima, para este c temos

x − a n

n
|αn (x − a) | 6 c ·
∀n ∈ N,
x0 − a
332 CAPÍTULO 8. DERIVADAS

∞ ∞
x − a n

X X
onde a série
x0 − a é convergente. Logo, a série |αn (x − a)n | também é convergente,
n=0 n=0
pelo Critério de Comparação (Corolário do Teorema 16, Capítulo IV), como queríamos.


X
Nível 4.2: Lema. Se uma série αn é absolutamente convergente, então
n=1

X∞ X ∞
αn 6 |αn |.



n=1 n=1

Solução. Sabemos que |α1 + · · · + αn | 6 |α1 | + · · · + |αn | para todo n ∈ N. Então, por um
lado,
lim |α1 + · · · + αn | 6 lim |α1 | + · · · + |αn |,
n→∞ n→∞
n
!
X
pelo Corolário 1 do Teorema 7 do Capítulo IV, pois ambas as sequências |αk | e

! k=1 n∈N
Xn ∞
X
αk de fato convergem. A primeira, por causa da convergência absoluta de αn e


n=1
k=1 n∈N
a segunda, por causa do Exercício 1 do Capítulo IV (página 117), segundo o qual


X ∞
lim |α1 + · · · + αn | = lim α1 + · · · + αn = αn ,

n→∞ n→∞
n=1

lembrando que séries absolutamente convergentes são convergentes (Teorema 18, Capítulo IV).
Segue a tese. 
(Ver também o Corolário do Teorema 13 do Capítulo X.)

8.48 Exercício 48
x5
Seja f (x) = . Calcule as derivadas de ordem 2001 e 2003 da função f :R→R no ponto
1 + x6
0.

Solução. A partir do Exemplo 28 podemos escrever

6n
1 1 6 12 n−1 6n−6 n x
= = 1 − x + x + · · · + (−1) x + (−1)
1 + x6 1 + (x3 )2 1 + x6
e, então,
x6n+5
f (x) = x5 − x11 + x17 + · · · + (−1)n−1 x6n−1 + (−1)n .
1 + x6
Sejam
p(x) = x5 − x11 + x17 + · · · + (−1)n−1 x6n−1
e
x6n+5
n
r(x) = (−1) .
1 + x6
8.49. EXERCÍCIO 49 333

r(x)
Como lim = 0 (produto de uma limitada por uma que tende a zero), pelo Teorema 9
x→0 x6n−1
podemos concluir que p é o polinômio de Taylor de ordem 6n − 1 de f no ponto 0. Podemos ir
um pouco mais além e concluir que p também é o polinômio de Taylor de ordem 6n + 4 de f no
r(x) x
ponto 0, porque o limite de
6n+4
= (−1)n também tende a zero para x → 0. Ambas as
x 1 + x6
derivadas de ordem 2001 e 2003 de f em 0 aparecem nos coecientes de p para n = 334; para
ele temos
p(x) = x5 − x11 + x17 + · · · + x1997 − x2003 .
Portanto
f (2001) (0) f (2003) (0)
=0 e = −1.
2001! 2003!
Logo f (2001) (0) = 0 e f (2003) (0) = −2003!. 

8.49 Exercício 49
Seja f : I → R denida num intervalo. Prove que f é convexa se, e somente se, para quaisquer
a, b em I e 0 6 t 6 1 vale f ((1 − t)a + tb) 6 (1 − t)f (a) + tf (b).
(Voltar para a Solução do Exercício 51: página 335.)

Solução. Com respeito ao número (1 − t)a + tb = t(b − a) + a, começamos notando que a


igualdade g(t) = (1−t)a+tb dene uma função sobrejetiva g : [0, 1] → J , onde J é um intervalo
fechado cujos extremos são a e b (e, portanto, J ⊂ I ), tal que g(t) ∈ int(J) se a 6= b e t ∈ (0, 1).
Vamos supor f convexa. Se a = b, então f (g(t)) = f (a) e (1 − t)f (a) + tf (b) = f (a) para
todo t ∈ [0, 1]. Para t = 0 ou t = 1, podemos ver que as desigualdades são verdadeiras por
vercação, não importando quem sejam a e b; de fato valem as igualdades. Se a < b, então,
por denição (de função convexa), como a < g(t) < b para 0 < t < 1, temos

f (b) − f (a)
f (g(t)) 6 · (g(t) − a) + f (a)
b−a
f (b) − f (a)
= · (t(b − a)) + f (a)
b−a
= t[f (b) − f (a)] + f (a)
= (1 − t)f (a) + tf (b).

para todo t ∈ (0, 1). De modo análogo, se b < a, então b < g(t) < a para todo t ∈ (0, 1), de
modo que

f (a) − f (b)
f (g(t)) 6 · (g(t) − b) + f (b)
a−b
f (a) − f (b)
= · (t(b − a) + a − b) + f (b)
a−b
= [f (a) − f (b)](1 − t) + f (b)
= (1 − t)f (a) + tf (b),

para todo t ∈ (0, 1). De qualquer forma vale f (g(t)) 6 (1 − t)f (a) + tf (b) para todo t ∈ [0, 1]
quaisquer que sejam a, b ∈ I , como queríamos.
334 CAPÍTULO 8. DERIVADAS

Reciprocamente, por denição, dados a<x<b em I, queremos mostrar que

f (b) − f (a)
f (x) 6 (x − a) + f (a).
b−a
Pela sobrejetividade de g, seja t ∈ [0, 1] tal que g(t) = x. Por hipótese sabemos que

f (g(t)) 6 (1 − t)f (a) + tf (b),

onde

f (b) − f (a) f (b) − f (a)


(1 − t)f (a) + tf (b) = (g(t) − a) + f (a) = (x − a) + f (a),
b−a b−a
donde segue a tese. 

8.50 Exercício 50
Verique que f : R → R, dada por f (x) = ex , é convexa e conclua que, para 0 6 t 6 1 e x, y ∈ R
(1−t)x+ty
quaisquer vale e 6 (1 − t)ex + t · ey . Deduza daí a desigualdade aα · bβ 6 α · a + β · b,
para α, β, a, b não-negativos, com α + β = 1.

Solução. A convexidade de f é imediata do Teorema 11: f é duas vezes derivável em R (de


∞ 00 x
fato é C ) e f (x) = e > 0 ∀x ∈ R; R é um intervalo aberto. Portanto, pelo exercício anterior,
vale
e(1−t)x+ty 6 (1 − t)ex + tey ,
para t ∈ [0, 1] e x, y ∈ R.
α β
Se a = 0 e α > 0 ou se b = 0 e β > 0, então a · b = 0, de modo que vale a desigualdade
α β
a · b 6 α · a + β · b porque α, β, a, b não negativos implicam α · a + β · b não negativo também.
0
Se a = α = 0 ou se b = β = 0, então surge o símbolo 0 na desigualdade a ser deduzida; ela se
0 0
torna 0 · b 6 b ou a · 0 6 a. Um modo de evitar o surgimento de um tal símbolo em nossas
contas é acrescentar as hipóteses a + α 6= 0 e b + β 6= 0.
x y
Se a e b são ambos positivos, sejam x, y ∈ R tais que a = e e b = e (eles existem), então,
de α = 1 − β , temos

aα · bβ = e(1−β)x · eβy = e(1−β)x+βy 6 (1 − β)ex + βey = α · a + β · b,

pelo que acabamos de ver mais acima, porque α, β > 0 e α+β = 1 implicam 0 6 β 6 1. Logo

aα · b β 6 α · a + β · b

se α, β, a, b são não negativos, a + α 6= 0, b + β 6= 0 e α + β = 1. 


(Voltar para a Solução do Exercício 52: página 336.)

8.51 Exercício 51
Seja f : I → R convexa no intervalo! I . Se a1 , . . . , a n pertencem a I , t1 , . . . , tn pertencem a [0, 1]
X n Xn Xn
e ti = 1, prove que f ti ai 6 ti f (ai ).
i=1 i=1 i=1
8.51. EXERCÍCIO 51 335

Solução. No Exercício 49 acima (página 333) já vimos isto ser verdade para n = 2; se
t ∈ [0, 1], então 1 − t ∈ [0, 1] também, além do que t + (1 − t) = 1. Para n = 1, a tese é
trivialmente verdadeira. Para demonstrar a veracidade dessa desigualdade para n = 3, usando
o fato de que ela é verdadeira para n = 2, escrevemos

 
t1 t2
t1 a1 + t2 a2 + t3 a3 = (t1 + t2 ) a1 + a2 + t 3 a3 .
t1 + t2 t1 + t2

Isto se, claro,t1 + t2 for não nulo. Na igualdade acima observamos que as somas (t1 + t2 ) + t3 e
t1 t2 t1 t2 t1 t2
+ ainda são ambas iguais a 1, com , ∈ [0, 1] e a1 + a2 ∈
t1 + t2 t1 + t2 t1 + t2 t1 + t2 t1 + t2 t1 + t2
I , porque, conforme já sabemos, este é um número entre a1 e a2 (Solução do Exercício 18 do
Capítulo IV, página 131) e I é um intervalo (Exercício 23 do Capítulo V, página 174). E é esta
a ideia central desta solução por indução.
Vamos supor que a tese seja verdadeira para um xado número natural n (incluindo tam-
bém a armação implícita de que aquele número no qual f foi aplicada do lado esquerdo da
desigualdade pertence a I ). Sejam a1 , . . . , an , an+1 elementos de I e t1 , . . . , tn , tn+1 ∈ [0, 1] tais
n+1
X Xn
que ti = 1. Se ti = 0, então t1 = t2 = · · · = tn = 0 e tn+1 = 1, de modo que
i=1 i=1

n+1
! n+1
X X
f ti ai = f (an+1 ) 6 f (an+1 ) = ti f (ai ).
i=1 i=1

n
X
Se ti 6= 0, escrevamos
i=1

n+1 n
! n
!
X X X tk
t i ai = ti · Pn ak + tn+1 an+1 .
i=1 i=1 k=1 i=1 ti

n n+1 n
X X X t
Como tn+1 + ti = 1, ti ai é um número entre Pnk ak e an+1 e, portanto, pertencente
i=1 i=1 k=1 i=1 ti
n
X tk t
a I, pois Pn ak ∈ I (pela hipótese de indução, porque Pnk ∈ [0, 1] para todo
k=1 i=1 ti i=1 ti
n
X t
k ∈ {1, . . . , n}, Pnk =1 e a1 , . . . , an ∈ I ), an+1 ∈ I e I é um intervalo  os mesmos
k=1 i=1 ti
argumentos do primeiro parágrafo . Deste modo, pelo já mencionado Exercício 49, temos

n+1
! n
! n
!
X X X tk
f ti ai 6 ti ·f Pn ak + tn+1 · f (an+1 ),
i=1 i=1 k=1 i=1 ti

onde, pela hipótese de indução,

n
! n
X tk X t
f Pn ak 6 Pn k f (ak ).
k=1 i=1 ti k=1 i=1 ti
336 CAPÍTULO 8. DERIVADAS

n
X
Portanto, já que ti > 0,
i=1

n+1
! n
! n
!
X X X tk
f ti ai 6 ti · Pn f (ak ) + tn+1 · f (an+1 )
i=1 i=1 k=1 i=1 ti
n
X
= tk f (ak ) + tn+1 · f (an+1 )
k=1
n+1
X
= tk f (ak ),
k=1

o que completa esta demonstração. 

8.52 Exercício 52
Sejam x1 , x2 , . . . , xn e t1 , . . . , tn números não-negativos, com t1 + · · · + tn = 1. Prove que
xt11 · xt22 · · · xtnn 6 t1 x1 + t2 x2 + · · · + tn xn . Conclua, em particular a desigualdade entre as médias
aritmética e geométrica. (Cfr. Exercício 54, Capítulo III.)

Solução. A ideia aqui é a mesma vista na Solução do Exercício 50 (página 334). Assim como
lá, vamos supor também que não existe i ∈ N ∩[0, n] tal que xi e ti sejam simultaneamente
iguais a zero. Se algum dos números x1 , x2 , . . . , xn é igual a zero, então a desigualdade desejada
é válida porque t1 x2 + · · · + tn xn nunca é um número negativo. No caso em que todos os xi 's
x
são positivos, considerando a função exponencial f : R → R denida por f (x) = e para todo
x real, a qual já sabemos ser convexa, pelo exercício anterior, temos

xt11 · xt22 · · · xtnn = et1 ln x1 · · · etn ln xn


= f (t1 ln x1 + · · · + tn ln xn )
6 t1 f (ln x1 ) + · · · + tn f (ln xn )
= t1 x1 + · · · + tn xn ,

como queríamos.
A desigualdade entre as médias aritmética e geométrica é imediata do que acabamos de
1
demonstrar fazendo-se ti = para todo i ∈ {1, . . . , n}. 
n

8.53 Exercício 53
Seja ϕ : [a, b] → R duas vezes derivável, com ϕ(a) = ϕ(b) = 0 e ϕ00 (x) < 0 para todo x ∈ [a, b].
Prove que ϕ(x) > 0 para todo x ∈ (a, b). [Sugestão : todo ponto onde ϕ se anule deve ser de
0

máximo. Logo, o mínimo de ϕ é atingido nos extremos do intervalo.] Conclua que, se f : I → R


00
é duas vezes derivável e f (x) > 0 para todo x ∈ I , então f é estritamente convexa no intervalo
I . [Dados a < b em I , seja g a função linear que coincide com f nos pontos a e b. Aplique o
resultado anterior à função ϕ = g − f .]
8.54. EXERCÍCIO 54 337

Solução. Por Weierstrass (Corolário do Teorema 14, Capítulo VII), ϕ atinge seus valores de
máximo e de mínimo em [a, b] (ϕ é contínua porque derivável, Teorema 1, e [a, b] é compacto).
O mínimo de ϕ não pode ser atingido no interior do intervalo [a, b], porque se c ∈ (a, b) fosse
um ponto de mínimo local de ϕ, então ϕ0 (c) = 0 (Corolário 2 do Teorema 4), o que faria
o
c também ser um ponto de máximo local de ϕ (item 1 da seção Aplicações da Fórmula de
00
Taylor) e, então, haveria uma vizinhança de c na qual ϕ seria constante, portanto ϕ (c) = 0,
uma contradição. Logo ϕ(x) > 0 para todo x ∈ (a, b), como queríamos para esta primeira
parte.
Para a segunda parte, dados a<b em I, consideremos a função g : [a, b] → R denida por

f (b) − f (a)
g(x) = (x − a) + f (a) ∀x ∈ [a, b],
b−a
de modo que a função ϕ = g − f |[a, b] : [a, b] → R satisfaz todas as condições da primeira
00 00
parte deste exercício: é duas vezes derivável, ϕ(a) = ϕ(b) = 0 e ϕ (x) = −f (x) < 0 para todo
x ∈ [a, b]. Logo ϕ(x) > 0 para todo x ∈ (a, b), ou seja, f (x) < g(x) para todo x ∈ (a, b), o que
signica que f é estritamente convexa, como queríamos. 

8.54 Exercício 54
Seja f contínua num ponto. Prove que se f é derivável nesse ponto então existe no máximo
uma reta que coincide com o gráco de f uma innidade de vezes em qualquer vizinhança do
ponto.

Solução. Seja a este ponto no qual f é contínua e derivável. Se esta reta não existe, então
não há nada a se fazer. Se ela existe, a chamemos de g. Claramente, g não pode ser uma reta
vertical, simplesmente pelo fato de f ser uma função; uma reta vertical tem no máximo uma
interseção com o gráco de uma função. Seja (an )n∈N uma sequência de pontos no domínio de
f que converge para a por valores distintos de a (ie, an 6= a ∀n ∈ N) tal que g(an ) = f (an ) para
todo n. Esta sequência existe em decorrência da hipótese de g coincidir com f uma innidade
de vezes em qualquer vizinhança de a. Então, por causa das continuidades de f e de g , podemos
fazer a primeira e a terceira das igualdades abaixo

f (a) = lim f (an ) = lim g(an ) = g(a).

Agora, pela derivabilidade de ambas f e g em a, temos também

f (an ) − f (a) g(an ) − g(a)


f 0 (a) = lim = lim = g 0 (a).
an − a an − a
Com estas considerações, temos uma caracterização completa de g : ela passa pelo ponto
(a, f (a)) e tem inclinação igual a f 0 (a). Logo g só pode ser a tangente ao gráco de f pelo
0
ponto (a, f (a)), ou seja, só pode ser o gráco da função denida por f (a)(x − a) + f (a) para
todo x real. 

8.55 Exercício 55
Seja f : [a, +∞) → R duas vezes derivável. Se lim f (x) = f (a) então existe x ∈ (a, +∞) tal
x→+∞
00
que f (x) = 0.
338 CAPÍTULO 8. DERIVADAS

Solução.

Nível 1. Se f (x) = f (a) para todo x > a, então não há nada a se fazer; f 00 (x) = 0 para
todo x > a.Caso contrário, isto é, caso exista x > a tal que f (x) 6= f (a), vamos obter (no
00
Nível 2) dois pontos distintos y e z tais que f (y) e f 00 (z) têm sinais opostos. E então, pelo
0
Valor Intermediário para a Derivada (Teorema 5), pelo fato de f ser derivável por hipótese,
00
existe um ponto entre eles no qual f se anula.

Nível 2. x > a tal que f (x) > f (a) (o caso em que existe x > a
Vamos supor que exista
tal que f (x) < f (a) será análogo). Seja x0 > a tal que f (x0 ) > f (a). Começamos obtendo
x1 ∈ (a, x0 ) e x2 > x0 tais que f (x1 ) = f (x2 ) e f (x0 ) > f (x1 ) (Nível 3.1). Desta forma,
00
podemos concluir que existe um ponto y ∈ (x1 , x2 ) tal que f (y) < 0 (Nível 3.2). Em seguida,
obtemos um ponto x3 > x2 tal que o ponto (x2 , f (x2 )) esteja estritamente abaixo da secante
que liga os pontos (x0 , f (x0 )) e (x3 , f (x3 )) (Nível 3.3). Isto implica a existência de um ponto
z ∈ (x0 , x3 ) tal que f 00 (z) > 0 (Nível 3.4). Claramente y e z não podem ser iguais, porque f 00
é uma função (ela não pode ser ao mesmo tempo positiva e negativa num ponto). E temos
00 00
encontrado os pontos y 6= z tais que f (y) e f (z) têm sinais opostos, como queríamos.

Nível 3.1. Estes pontos existem por causa da continuidade de f e do fato do limite de
f (x) quando x tende a +∞ ser f (a). Temos f (x0 ) > f (a) e f contínua em [a, x0 ]. Então, pelo
Teorema do Valor Intermediário (Teorema 12 do Capítulo VII), tomando-se y1 ∈ (f (a), f (x0 )),
existe x1 ∈ (a, x0 ) tal que f (x1 ) = y1 .
Para o y1 do parágrafo acima, que é maior do que f (a) por construção, como lim f (x) =
x→+∞
f (a), existe w > x0 tal que f (w) < y1 e, novamente pelo Teorema do Valor Intermediário, de
y1 ∈ (f (w), f (x0 )), existe x2 ∈ (x0 , w) tal que f (x2 ) = y1 , como queríamos.

Nível 3.2. Aqui temos o intervalo I = (x1 , x2 ), com x0 ∈ I e f (x0 ) > f (x1 ) = f (x2 ). Disto
segue quef não é convexa em I, pois podemos obter pontos α e β em I tais que f (α) = f (β),
α < x0 < β e f (x0 ) > f (α). Estes pontos podem ser obtidos aplicando-se o Teorema do Valor
Intermediário, do mesmo modo como feito no nível acima. Portanto, pelo Teorema 11, existe
y ∈ I tal que f 00 (y) < 0.

Nível 3.3. Queremos obter x3 > x2 tal que

f (x3 ) − f (x0 )
f (x2 ) < (x2 − x0 ) + f (x0 ).
x3 − x0
Esta inequação é equivalente a cada uma das inequações abaixo

f (x2 ) − f (x0 ) f (x3 ) − f (x0 )


<
x2 − x0 x3 − x0
e
f (x2 ) − f (x0 )
f (x3 ) > (x3 − x0 ) + f (x0 ),
x2 − x0
lembrando que x2 − x0 e x3 − x0 (se x3 > x2 ) são ambos números positivos. O lado direito
desta última inequação é o valor que a reta determinada pelos pontos (x0 , f (x0 )) e (x2 , f (x2 ))
assume no ponto x3 . Então, denindo

f (x2 ) − f (x0 )
r(x) = (x − x0 ) + f (x0 ) ∀x ∈ R,
x2 − x0
8.55. EXERCÍCIO 55 339

queremos x3 > x 2 tal que f (x3 ) > r(x3 ). É esta interpretação geométrica do problema que
vai nos guiar em nossa solução. A ideia central dela, o que vai permitir a ocorrência desta
desigualdade, é o fato de, conforme o tempo passa, o gráco de r afastar-se cada vez mais da
reta horizontal cujas segundas coordenadas de seus pontos são todas iguais a f (a), enquanto o
gráco de f não, o gráco de f se aproxima cada vez mais desta reta horizontal com a passagem
do tempo. No parágrafo abaixo vamos formalizar isto.
Como lim r(x) = −∞ (lembrando que f (x2 ) < f (x0 ) e x2 > x0 ), seja y0 ∈ R tal que
x→+∞
f (a) − r(y0 ) > 0. Tomando-se ε = f (a) − r(y0 ), pela convergência lim f (x) = f (a), seja
x→+∞
A∈R x > A ⇒ f (x) > f (a) − ε = r(y0 ). Além disso, se x > y0 , então r(y0 ) > r(x), de
tal que
modo que x > max{A, y0 } implica f (x) > r(x). Logo, qualquer x3 que seja simultaneamente
maior do que x2 , A e y0 serve para os nossos propósitos.

Nível 3.4. Se (x2 , f (x2 )) está estritamente abaixo da secante que liga os pontos (x0 , f (x0 ))
e (x3 , f (x3 )), então (x2 , −f (x2 )) está estritamente acima da secante que liga os pontos (x0 , −f (x0 ))
e (x3 , −f (x3 )), ou seja, a função −f não é convexa no intervalo (x0 , x3 ). Logo, pelo Teorema
00 00 00
11, existe z ∈ (x0 , x3 ) tal que (−f ) (z) < 0, onde (−f ) (z) = −f (z), portanto z ∈ (x0 , x3 ) tal
00
que f (z) > 0, como queríamos. 
340 CAPÍTULO 8. DERIVADAS
Capítulo 9
Integral de Riemann

9.1 Exercício 1
Z b Z b

Seja f : [a, b] → R limitada. Prove que f (x) dx 6 |f (x)| dx. Dê um exemplo mostrando

a a
que uma desigualdade análoga não vale para integrais inferiores.

Solução 1. Claramente a função |f | também é limitada. Vamos mostrar que

Z b Z b Z b
− |f (x)| dx 6 f (x) dx 6 |f (x)| dx
a a a

(Teorema 1 do Capítulo III).


De −|f (x)| 6 f (x) 6 |f (x)| para todo x ∈ [a, b] (página 72 do livro), pelo item 3 do Teorema
3, vem
Z b Z b Z b
− |f (x)| dx 6 f (x) dx 6 |f (x)| dx,
a a a

onde
Z b Z b Z b
− |f (x)| dx = − |f (x)| dx > − |f (x)| dx,
a a a

Z b Z b
pelo item 2 do Teorema 3 e porque g(x) dx 6 g(x) dx qualquer que seja a função limitada
a a
g : [a, b] → R, conforme já observado na página 307 do livro. A tese segue das duas últimas
linhas centralizadas acima.
Um exemplo mostrando que uma desigualdade análoga não vale para integrais inferiores é
o da função f : [a, b] → R denida por f (x) = 0 se x ∈ [a, b] − Q e f (x) = −1 se x ∈ [a, b] ∩ Q.
Para ela temos Z
b
f (x) dx = | − (b − a)| = b − a,

a
enquanto

Z b
|f (x)| dx = 0. 
a

341
342 CAPÍTULO 9. INTEGRAL DE RIEMANN

Solução 2 (para a primeira parte do exercício). Também podemos usar a denição


|A| = max{A, −A}, A ∈ R (página 72 do livro), e vericar que valem ambas as desigualdades

Z b Z b Z b Z b
|f (x)| dx > f (x) dx e |f (x)| dx > − f (x) dx.
a a a a

De fato:
Z b Z b
|f (x)| > f (x) ∀x ∈ [a, b] ⇒ |f (x)| dx > f (x) dx
a a

(item 3 do Teorema 3) e

Z b Z b Z b
|f (x)| > −f (x) ∀x ∈ [a, b] ⇒ |f (x)| dx > − f (x) dx = − f (x) dx
a a a
Z b Z b
⇒ |f (x)| dx > − f (x) dx
a a

Z b Z b
(itens 3 e 2 do Teorema 3), pois |f (x)| dx > |f (x)| dx (página 307 do livro). Valendo
a a
as desigualdades |f (x)| > f (x) e |f (x)| > −f (x), ambas para todo x ∈ [a, b], novamente pela
denição: |f (x)| = max{f (x), −f (x)} ∀x ∈ [a, b]. Segue a tese. 

Observação. Na solução acima poderíamos ter feito

Z b Z b Z b Z b
|f (x)| > −f (x) ∀x ∈ [a, b] ⇒ |f (x)| dx > − f (x) dx = − f (x) dx > − f (x) dx
a a a a

e na Solução 1,

Z b Z b Z b Z b
f (x) > −|f (x)| ∀x ∈ [a, b] ⇒ f (x) dx > f (x) dx > − |f (x)| dx = − |f (x)| dx.
a a a a

9.2 Exercício 2
Seja f : [a, b] → R integrável. As seguintes armações são equivalentes:

Z b
1. |f (x)| dx = 0;
a

2. Se f é contínua no ponto c então f (c) = 0;

3. X = {x ∈ [a, b]; f (x) 6= 0} tem interior vazio.

(Voltar para a Solução do Exercício 16, para o seu Nível 2: página 370.)

(Voltar para a Solução do Exercício 23: página 378.)

Solução. Vamos mostrar que 1 ⇒ 2, 2 ⇒ 3 e 3 ⇒ 1.


9.3. EXERCÍCIO 3 343

[1 ⇒ 2] De forma equivalente, supondo a negação de 2 vamos mostrar a negação de 1. Se


α < β são pontos em [a, b], então, pelo item 1 do Teorema 5, podemos escrever
Z b Z α Z β Z b
|f (x)| dx = |f (x)| dx + |f (x)| dx + |f (x)| dx,
a a α β

(x 7→ |f (x)| é integrável pelo item 5 do Teorema 5) soma esta cujas três parcelas são não
Z β
negativas pelo item 4 do Teorema 5. Vamos escolher αeβ de modo a termos |f (x)| dx > 0,
α
de onde seguirá a tese.
c ∈ [a, b] um ponto no qual f é contínua e f (c) 6= 0. Assim, xado um
Por hipótese, seja
número real positivo A menor do que |f (c)|, seja [α, β] ⊂ [a, b], com α < β , um intervalo ao
qual c pertence e tal que |f (x)| > A para todo x pertencente a ele. Um tal intervalo existe
porque f é contínua em c (Teorema 3 do Capítulo VII). Pelo item 4 do Teorema 5 temos

Z β
|f (x)| dx > A(β − α) > 0
α

e estes são o α e o β escolhidos.

[2 ⇒ 3] O item 2 nos diz que f (x) = 0 em todo ponto x ∈ [a, b] no qual f é contínua.
Em consequência, X só pode estar contido no conjunto D dos pontos de descontinuidade de f.
Então, m(X) = 0, uma vez que m(D) = 0 (item (1) da página 343 e Teorema 20  podemos
supor, implicitamente, que f é limitada porque o conceito de função integrável foi denido
apenas para funções limitadas). Logo, o interior de X é vazio, pois, se I ⊂ X, então m(I) = 0
e nenhum intervalo aberto tem medida nula, ou seja, X não contém um intervalo aberto.
Nenhum intervalo aberto tem medida nula pelo seguinte motivo. Se I fosse um intervalo
aberto com medida nula, seja [a, b] ⊂ I , com a < b. Então a medida do compacto [a, b] seria
nula (por ser um subconjunto de um conjunto de medida nula) e seu conteúdo seria nulo, pois
conjuntos compactos com medida nula têm conteúdo nulo (item (2) da página 343). Portanto,
o conteúdo do conjunto Q ∩[a, b] também seria nulo (item 2 da página 338), contrariando o
exemplo 12. Logo, a medida de I não é nula.

(Voltar para a Solução do Exercício 21: página 377.)

(Voltar para a Solução do Exercício 22: página 378.)

[3 ⇒ 1] Dada uma partição P = {t0 , t1 , . . . , tn } de [a, b], o ínmo do conjunto {|f (x)|; x ∈
[ti−1 , ti ]} é igual a zero para cada i ∈ {1, . . . , n}. Isto porque X não contém nenhum intervalo
aberto, por hipótese, ou seja, dado i ∈ {1, . . . , n}, existe x ∈ [ti−1 , ti ] tal que f (x) = 0. Logo,
como f é integrável,
Z bZ b
|f (x)| dx = |f (x)| dx = 0
a a

(item 5 do Teorema 5 + denição de função integrável), como queríamos. 

9.3 Exercício 3
Z b
Seja f : [a, b] → R contínua. Se f não é identicamente nula, então |f (x)| dx > 0.
a
344 CAPÍTULO 9. INTEGRAL DE RIEMANN

Solução. Isto decorre do exercício anterior, em particular da implicação 1 ⇒ 2: f é integrável


porque é contínua (Teorema 6) e existe um ponto c ∈ [a, b] no qual f é contínua e tal que
f (c) 6= 0 (f é contínua no ponto c ∈ [a, b] tal que f (c) 6= 0, o qual existe porque f não
é identicamente nula, porque f é contínua), portanto, pela forma contrapositva da referida
Z b Z b
implicação, |f (x)| dx 6= 0 (|f (x)| é integrável pelo item 5 do Teorema 5), logo, |f (x)| dx >
a a
Z b
0, já que |f (x)| dx > 0 (item 4 do Teorema 5). 
a

9.4 Exercício 4
Dê exemplo de uma função integrável que seja descontínua num conjunto innito.

Solução. Além do Exemplo 7 (deste capítulo), o Exemplo 9 do Capítulo VII também nos dá
uma função integrável descontínua num conjunto innito. Ela é a função ϕ : [0, 1] → R denida
por ϕ(x) = 0 se x∈K e ϕ(x) = 1 x ∈ [0, 1] − K , sendo K o conjunto de Cantor. Lá vimos
se
que o conjunto dos pontos de descontinuidade de ϕ é K  um conjunto não enumerável, de
acordo com o Corolário 2 do Teorema 9 do Capítulo V , o que foi feito usando os fatos de que
K é fechado e seu interior é vazio (conforme o Exemplo 15 do Capítulo V). Pelo Exemplo 13,
c(K) = 0 e, portanto, m(K) = 0. Logo, ϕ é integrável (Teorema 20). 

9.5 Exercício 5
Sejam f, g : [a, b] → R contínuas, com f (x) 6 g(x) para todo x ∈ [a, b]. Dena ϕ : [a, b] → R,
pondo ϕ(x) = f (x) se x for racional e ϕ(x) = g(x) para x irracional. Prove que

Z b Z b Z b Z b
ϕ(x) dx = f (x) dx e ϕ(x) dx = g(x) dx.
a a a a

Conclua que ϕ é integrável se, e somente se, f = g.

Solução.

Primeira Parte.

Nível 1. Dado um intervalo I ⊂ [a, b] não unitário, armamos que

inf ϕ(x) = inf f (x) e sup ϕ(x) = sup g(x)


x∈I x∈I x∈I x∈I

(Nível 2). Disto seguem

s(ϕ, P ) = s(f, P ) e S(ϕ, P ) = S(g, P ),

qualquer que seja a partição P de [a, b]. Portanto

Z b Z b Z b
ϕ(x) dx = f (x) dx = f (x) dx
a a a
9.6. EXERCÍCIO 6 345

e
Z b Z b Z b
ϕ(x) dx = g(x) dx = g(x) dx,
a a a
já que f e g são integráveis (Teorema 6).

Nível 2. Vamos mostrar que os números A = inf f (x) e B = sup g(x) são, respectiva-
x∈I x∈I
mente, o ínmo e o supremo do conjunto {ϕ(x); x ∈ I}.
Claramente f (x) 6 ϕ(x) e g(x) > ϕ(x) para todo x ∈ [a, b], porque ϕ(x) = f (x) ou
ϕ(x) = g(x) e f (x) 6 g(x), portanto A 6 ϕ(x) e B > ϕ(x) para todo x ∈ I . Dado ε > 0,
existem x1 e x2 em I tais que f (x1 ) < A + ε e g(x2 ) > B − ε. Pelas continuidades de f
e de g , existem intervalos abertos não vazios I1 e I2 contidos em I tais que f (x) < A + ε
para todo x ∈ I1 e g(x) > B − ε para todo x ∈ I2 (Corolário do Teorema 3 do Capítulo
VII). Para um y1 racional em I1 e um irracional y2 em I2 (Teorema 4 do Capítulo III), temos
ϕ(y1 ) = f (y1 ) < A + ε e ϕ(y2 ) = g(y2 ) > B − ε. Seguem as teses.
Z b Z b
Segunda Parte. Se f = g, então f (x) dx = g(x) dx. Portanto, pela Primeira
a a
Parte,
Z b Z b
ϕ(x) dx = ϕ(x) dx.
a a

Logo, por denição, ϕ é integrável, notando que ϕ é limitada.


Z b Z b
Reciprocamente, se ϕ é integrável, então, também da Primeira Parte, f (x) dx = g(x) dx,
a a
donde Z b
g(x) − f (x) dx = 0
a
pelos itens 2 e 3 do Teorema 5. Queremos concluir que a função g−f é identicamente nula.
Por um lado, se g−f não é identicamente nula, então, pelo Exercício 3,
Z b
|g(x) − f (x)| dx > 0,
a

pois g − f é contínua. Por outro lado, de f (x) 6 g(x) para todo x ∈ [a, b], segue |g(x) − f (x)| =
g(x) − f (x) para todo x ∈ [a, b], de modo que
Z b Z b
|g(x) − f (x)| dx = g(x) − f (x) dx > 0,
a a
Z b Z b
o que implica g(x) dx 6=
f (x) dx e, portanto, ϕ não é integrável (lembrando novamente
a a
que limitada ela é). Isto demonstra o resultado desejado (o de que ϕ integrável implica g−f ≡ 0)
pela contrapositiva. 

9.6 Exercício 6
Z b Z b
Seja f : [a, b] → R não-negativa, limitada. Prove que f (x) dx = sup ξ(x) dx, onde ξ
a ξ a
percorre o conjunto das funções-escada tais que ξ(x) 6 f (x) para todo x ∈ [a, b]. Mostre
346 CAPÍTULO 9. INTEGRAL DE RIEMANN

que um resultado análogo vale se tomarmos ξ contínua ou ξ integrável (mantendo-se sempre a


hipótese ξ(x) 6 f (x) para todo x em [a, b]).

Solução. Pelo item 3 do Teorema 3, se ξ é uma função integrável  em particular, se ξ é uma


função-escada ou uma função contínua (página 310 e Teorema 6) , então valem as implicações

Z b Z b
ξ(x) 6 f (x) ∀x ∈ [a, b] ⇒ ξ(x) dx 6 f (x) dx
a a
Z b Z b
⇒ ξ(x) dx 6 f (x) dx.
a a

Com a denição de supremo em mente, dado ε > 0, seja P = {t0 , t1 , . . . , tn } uma partição
de [a, b] tal que
Z b
f (x) dx − ε < s(f ; P ).
a
O número s(f ; P ) é a integral de uma função-escada ξ : [a, b] → R tal que ξ(x) 6 f (x)
para todo x ∈ [a, b], a saber, a denida por ξ(x) = mi para todo x ∈ [ti−1 , ti ) (para cada
i ∈ {1, . . . , n}) e ξ(b) = mn , com

mi = inf{f (x); x ∈ [ti−1 , ti ]} ∀i ∈ {1, . . . , n}.

Logo

Z b Z b 
f (x) dx = sup ξ(x) dx; ξ(x) 6 f (x) ∀x ∈ [a, b] e ξ é função-escada ,
a a

o que demonstra a primeira parte deste exercício. Como funções-escada são integráveis, também
podemos concluir a igualdade

Z b Z b 
f (x) dx = sup ξ(x) dx; ξ(x) 6 f (x) ∀x ∈ [a, b] e ξ é integrável .
a a

Vamos obter agora uma função real contínua ξ denida em [a, b] tal que ξ(x) 6 f (x) para
todo x ∈ [a, b] e
Z b Z b
f (x) dx − ε < ξ(x) dx.
a a

Ainda com o mesmo s(f ; P ) e os mesmos mi 's de acima, podemos denir esta função de
Z b
modo a tornar o número ξ(x) dx tão próximo de s(f ; P ) quanto se queria. Isto pode ser feito
a
do seguinte modo (as guras abaixo ilustram essa construção). Fixado i ∈ {1, 2, . . . , n − 1},
ao denir ξ no i-ésimo intervalo de P , olhamos para o mi+1 . Se mi+1 < mi , então denimos
ξ(x) = mi para todo x ∈ [ti−1 + δ, ti − δ], ligamos o par (ti − δ, mi ) até o par (ti , mi+1 ) por
um segmento de reta e, em seguida, continuamos com ξ(x) = mi+1 para todo x ∈ [ti , ti + δ].
Se mi+1 > mi , então denimos ξ(x) = mi para todo x ∈ [ti−1 + δ, ti ] e ligamos o par (ti , mi )
até o par (ti + δ, mi+1 ) por um segmento de reta. Se mi+1 = mi , então fazemos ξ(x) = mi
para todo x ∈ [ti−1 + δ, ti + δ]. Em qualquer um destes três casos, denimos ξ(x) = m1 para
todo x ∈ [t0 , t0 + δ] e ξ(x) = mn para todo x ∈ [tn−1 + δ, tn ]. Fazemos isto para 0 < δ <
 
ti − ti−1
min ; i ∈ {1, 2, . . . , n} .
2
9.7. EXERCÍCIO 7 347

mi+1
mi
δ(mi − mi+1 ) δ(mi+1 − mi )
2 2
mi

mi+1

ti − δ ti ti ti + δ
A ξ assim construída é contínua, satisfaz ξ(x) 6 f (x) para todo x ∈ [a, b] e o módulo da
diferença de sua integral com o número s(f ; P ) é
Z b n−1
δ X
s(f ; P ) − ξ(x) dx = · |mi+1 − mi |,
a 2 i=1

uma soma de áreas de triângulos.


! Para nalizar, queremos que esta diferença seja menor do
Z b
que s(f ; P ) − f (x) dx − ε , então tomamos
a
h R i
b
2 · s(f ; P ) − f (x) dx − ε
a
δ< Pn−1 .
i=1 |mi+1 − mi |

n−1
X Z b
Isto se, claro, a soma |mi+1 −mi | for diferente de zero. Se for zero, então temos ξ(x) dx =
i=1 a
s(f ; P ). De qualquer forma, para este δ, temos

Z b Z b
f (x) dx − ε < ξ(x) dx,
a a

como queríamos. Logo também vale

Z b Z b 
f (x) dx = sup ξ(x) dx; ξ(x) 6 f (x) ∀x ∈ [a, b] e ξ é contínua . 
a a

(Voltar para a Solução do Exercício 14a, para o elevador de seu Nível 2: página 361.)

Observação. Nesta solução não foi usada a hipótese de que f deve ser não negativa.

9.7 Exercício 7
Sejam f : [a, b] → R contínua, ϕ : [t0 − ε, t0 + ε] → [a, b] derivável e c ∈ [a, b]. Prove que as
armações a seguir são equivalentes:

A. ϕ0 (t) = f (ϕ(t)) para todo t ∈ [a, b] [sic ] e ϕ(t0 ) = c.


Z t
B. ϕ(t) = c + f (ϕ(s)) ds para todo t ∈ [a, b] [sic ].
t0
348 CAPÍTULO 9. INTEGRAL DE RIEMANN

Solução. t ∈ [t0 −ε, t0 +ε], que é onde ϕ está denida.


Os itens A e B fazem sentido para todo
⇒ B vale por causa do Teorema Fundamental do Cálculo (Teorema 9).
A implicação A
Fixado t ∈ [t0 − ε, t0 + ε], seja I o intervalo [t0 , t] se t > t0 ou o intervalo [t, t0 ] se t < t0 . A
armação A signica que ϕ|I é uma primitiva de (f ◦ ϕ)|I , esta integrável porque contínua
(Teorema 6  funções deriváveis são contínuas e compostas de funções contínuas são contínuas,
Teoremas 1 do Capítulo VIII e 6 do Capítulo VII, respectivamente). Então, pelo Teorema
Fundamental do Cálculo, supondo A, temos
Z t
f (ϕ(s)) ds = ϕ(t) − ϕ(t0 ) = ϕ(t) − c,
t0

se t > t0 , e
Z t Z t0
f (ϕ(s)) ds = − f (ϕ(s)) ds = −(ϕ(t0 ) − ϕ(t)) = ϕ(t) − c,
t0 t

se t < t0 , lembrando das convenções feitas nas páginas 318 e 319. De qualquer forma
Z t
ϕ(t) = c + f (ϕ(s)) ds,
t0

qualquer que seja t ∈ [t0 − ε, t0 + ε], como queríamos.


A implicação B ⇒ A vale por causa do Teorema 8. Supondo a armação B, denimos
Z x
F (x) = f (ϕ(s)) ds = ϕ(x) − ϕ(t0 − ε) ∀x ∈ [t0 − ε, t0 + ε].
t0 −ε

Fixado t ∈ [t0 − ε, t0 + ε], pelo referido teorema, como f ◦ϕ é contínua em t, temos

ϕ0 (t) = F 0 (t) = f (ϕ(t)),

além de Z t0
ϕ(t0 ) = c + f (ϕ(s)) ds = c.
t0

Logo ϕ0 (t) = f (ϕ(t)) para todo t ∈ [t0 − ε, t0 + ε] e ϕ(t0 ) = c, como queríamos. 

9.8 Exercício 8
Seja f : R → R derivável, tal que f (0) = 0 e, para todo x ∈ R, vale f 0 (x) = [f (x)]2 . Mostre
que f (x) = 0 para todo x ∈ R.

Solução. Vamos imitar a demonstração do Teorema 24. Para tanto, precisamos de uma
função g tal que ϕ = f · (exp ◦ g) seja constante. De

ϕ0 (x) = f 0 (x) · eg(x) − f (x) · g 0 (x) · eg(x) = [f (x)]2 · eg(x) − f (x)g 0 (x) · eg(x) ,

a g a ser considerada é a primitiva de f.


Para aplicarmos os teoremas abaixo de forma mais apropriada, seja a∈R um número real
não nulo e consideremos o intervalo

[0, a], se a > 0 ou
I=
[a, 0], se a < 0.
9.9. EXERCÍCIO 9 349

A restrição f |I é contínua, então ela admite uma primitiva g:I →R contínua, contínua
porque derivável (Corolário do Teorema 8 e Teorema 1 do Capítulo VIII). Portanto ϕ=f·
(exp ◦ g) : I → R é contínua (Teoremas 5 e 6 do Capítulo VII e Teorema 22). Disto, e de
ϕ0 (x) = 0 para todo x ∈ I , segue que ϕ é constante (Corolário 1 do Teorema 7 do Capítulo
VIII), logo ϕ(x) = 0 para todo x ∈ I , pois ϕ(0) = 0. Uma vez que exp(x) 6= 0 para todo x, só
pode ser f (x) = 0 para todo x ∈ I .
Deste modo, dado x ∈ R, para concluimos que f (x) = 0 basta tomar a ∈ R tal que x ∈ I
no raciocínio acima. Logo f (x) = 0 para todo x ∈ R, como queríamos. 

9.9 Exercício 9
Dê exemplo de uma função não-integrável que possua primitiva [ Sugestão : Ache uma função
f , derivável em [−1, +1], com f 0 ilimitada.]

Solução.

Exemplo 1. Um exemplo de uma função com as propriedades dadas na sugestão podemos


encontrar no livro Counterexamples in Analysis (em sua página 37): a f : [−1, 1] → R denida
por
1
f (x) = x2 sen se x 6= 0
x2
e f (0) = 0. Ela é derivável e sua derivada não é limitada; ela é dada por

1 2 1
f 0 (x) = 2x sen − cos 2 se x 6= 0
x x x
e f 0 (0) = 0. Por denição, f é a primitiva de f 0.

Feito isto, cabe a pergunta: por que uma função ilimitada não é integrável? O exemplo acima
é retomado na página 43 do Counterexamples também como o de uma função não integrável
que possui primitiva. Porém, lá está explícito: A function f whose domain is a closed interval
[a, b] is Riemann-integrable there i it is bounded and continuous almost everywhere (página
42; em tradução livre: Uma função f cujo domínio é um intervalo fechado [a, b] é Riemann-
integrável aí sse ela é limitada e contínua quase sempre; sse signica se e somente se e contínua
quase sempre signica que ela é contínua em todo ponto, exceto nos pontos de um conjunto
de medida nula). Já no livro do Elon, o conceito de função integrável foi denido apenas no
caso em que se supõe a priori que esta função é limitada, não havendo a denição de quando
uma função não limitada é integrável ou não. Dentro deste contexto, podemos dizer então que
sequer faz sentido classicar uma função não limitada como integrável ou como não integrável.
Tendo em vista a sugestão feita, o que caberia no Curso de Análise seria simplesmente denir:
é não integrável qualquer função não limitada. Mas, caso queiramos car no contexto do livro,
ainda sim podemos encontrar uma função limitada (para a qual, portanto, faz sentido aplicar
a denição de função integrável) que não seja integrável e que possua primitiva. Uma função
assim pode ser encontrada na página 107 do Counterexamples, da onde extraímos o

Exemplo 2. Vamos construir uma função derivável f : [0, 1] → R tal que sua derivada
0
f seja limitada (em todo seu domínio) e descontínua num conjunto cuja medida não é nula.
Assim f 0 não será integrável (Teorema 20).
350 CAPÍTULO 9. INTEGRAL DE RIEMANN

Para tanto, vamos reciclar uma função vista nos Exemplos 12 e 14 do Capítulo VIII, a
1
saber, a função g : R → R denida por g(x) = x2 sen para x 6= 0 e g(0) = 0. Restrita
x
0 1 1
ao intervalo [0, 1], sua derivada, dada por g (x) = 2x sen − cos para x 6= 0 e g 0 (0) = 0, é
x x
limitada,
1 1
|g 0 (x)| 6 |2x| · sen + cos 6 |2x| + 1 6 3,

x x
eg 0 não é contínua na origem (a sequência (1/(2πn))n∈N se aproxima do zero e g 0 (1/(2πn)) = −1
para todo todo n, Teorema 4 do Capítulo VII).
0
Podemos aproveitar essa descontinuidade da g para criar uma nova função com muitas
outras descontinuidades, através de translações e reexões em torno do eixo das ordenadas (e
combinações desses movimentos). Por exemplo, considerando o conjunto K de Cantor, podemos
denir uma função do seguinte modo. Fixado um intervalo (a, b) removido em alguma etapa
da construção de K, partindo do ponto a, vamos tomando os valores g(x − a) até chegar à
abscissa de algum pico ou de algum vale que esteja antes da metade do caminho, mantemos
o valor assumido neste ponto até a metade do caminho e depois retornamos de forma espelhada
até alcançar o ponto b (ver a imagem abaixo). E associamos cada ponto de K ao zero. Em
cada intervalo retirado na construção de K, a função assim denida é derivável e tem derivada
limitada. Se em K ela também for derivável e tiver derivada nula, então essa derivada será
descontínua em K, pois próximo o suciente de cada ponto de K há um extremo de algum
intervalo removido (Exemplo 16 do Capítulo V) e próximo o suciente deste extremo há algum
ponto no qual essa derivada vale 1 ou -1 (por causa da sequência mencionada mais acima).
O problema é que K tem conteúdo nulo (Exemplo 13) e, portanto, medida nula (consequen-
temente, uma tal derivada seria integrável). Por outro lado, em vez de K, podemos considerar
um conjunto parecido com ele (parecido no que importa pelo menos), porém com medida
positiva: o conjunto de Cantor com medida positiva. Para obter este conjunto, começamos
com o intervalo [0, 1] e, na etapa n de sua construção, retiramos o intervalo aberto médio de
n n−1
comprimento α /2 de cada intervalo remanescente da etapa anterior, sendo α um número
pré-xado do intervalo (0, 1/2). Este conjunto também é mencionado no Exercício 45 (página
404). No segundo parágrafo de sua solução veremos que a medida do conjunto de Cantor com
medida positiva é de fato positiva (então este conjunto faz jus ao seu nome).
Mais precisamente, xados um conjunto X de Cantor com medida positiva e um intervalo
 0
(a, b) retirado em alguma etapa de sua construção, sendo xa = max x ∈ 0, b−a

2
; g (x) = 0
(1/xa é também o menor y > 2/(b − a) tal que 2 tan y = y ), a função em questão é a denida
assim: 
 g(x − a), se a 6 x 6 a + xa
f (x) = g(xa ), se a + xa 6 x 6 b − xa
g(b − x), se b − xa 6 x 6 b

e f (x) = 0 para todo x ∈ X.

Gráco de f restrito ao intervalo (a, b), removido durante a construção do conjunto de Cantor com
medida positiva.

A mesma demonstração feita no Exemplo 16 do Capítulo V (sobre a densidade em K do


conjunto dos extremos dos intervalos retirados) também vale para X: o comprimento de cada
9.10. EXERCÍCIO 10 351

intervalo remanescente após cada etapa da construção de X também tende a zero. Assim, só
resta mostrar que f 0 é derivável em X e que f 0 (p) = 0 para todo p ∈ X .
Fixemos então um ponto
p de X e seja x um ponto do intervalo [0, 1] distinto de p. Vamos
f (x) − f (p) 0
mostrar que
x − p 6 |x − p|, o que implicará f (p) = 0 pelo teorema do confronto


f (x) − f (p)
(Teorema 4 do Capítulo VI). Se x ∈ X , então é imediato:
x − p = 0 6 |x − p|. Se

x∈ / X , consideremos o intervalo (a, b) ao qual x pertence e que foi removido em alguma etapa
da construção de X . Seja c ∈ {a, b} tal que |x − c| = min{x − a, b − x} (isto é, c é aquele ponto
dentre a e b que está mais próximo de x) e notemos que |x − p| > |x − c| (pois p ∈ / (a, b)) e que
2
|f (x)| 6 |x − c| . Deste modo:

f (x) − f (p) f (x) f (x) |x − c|2



x − p = x − p 6 x − c 6 |x − c| = |x − c| 6 |x − p|.


f (x) − f (p)
Logo,
x − p 6 |x − p| para todo p∈X e todo x ∈ [0, 1], como queríamos. 

Observação. O Exemplo 1 também é de uma função uniformemente contínua que não é de


Lipschitz (isto é, não vale a recíproca daquele resultado que diz que uma função lipschitzi-
ana é uniformemente contínua, visto no Exemplo 23 do Capítulo VII). Derivabilidade implica
continuidade (Teorema 1 do Capítulo VIII); continuidade num conjunto compacto implica con-
tinuidade uniforme (Teorema 17 do Capítulo VII), e; se uma função tem derivada ilimitada,
então ela não é de Lipschitz (Exercício 24 do Capítulo VIII, página 303).

9.10 Exercício 10
Sejam f, g : [a, b] → R funções integráveis. Pontilhemos cada partição P de duas maneiras
diferentes, escolhendo em todo intervalo [ti−1 , ti ] um ponto ξi e um ponto ηi . Mostre que

n
X Z b
lim f (ξi )g(ηi )(ti − ti−1 ) = f (x)g(x) dx.
|P |→0 a
i=1

Solução.

Nível 1. Pelo item 6 do Teorema 5, o produto f ·g é integrável, então, pelo Teorema 15,
sabemos que
n
X Z b
lim f (ξi )g(ξi )(ti − ti−1 ) = f (x)g(x) dx.
|P |→0 a
i=1

Dado ε > 0, vamos mostrar que existe δ>0 tal que


Xn n
X
f (ξi )g(ξi )(ti − ti−1 ) − f (ξi )g(ηi )(ti − ti−1 ) < ε,



i=1 i=1

se |P | < δ . Vamos mostrar a existência deste δ no Nível 2.


352 CAPÍTULO 9. INTEGRAL DE RIEMANN

Deste modo, dado |P | < min{δ1 , δ2 }, então


ε > 0, se

Z Z
b Xn b X n
f (x)g(x) dx − f (ξi )g(ηi )(ti − ti−1 ) 6 f (x)g(x) dx − f (ξi )g(ξi )(ti − ti−1 ) +


a a
i=1 i=1

X n Xn
f (ξi )g(ξi )(ti − ti−1 ) − f (ξi )g(ηi )(ti − ti−1 )



i=1 i=1
ε ε
< +
2 2
= ε,

se δ1 e δ2 são números positivos tais que


Z
b Xn ε
|P | < δ1 ⇒ f (x)g(x) dx − f (ξi )g(ξi )(ti − ti−1 ) <

a 2
i=1

e
X n Xn ε
|P | < δ2 ⇒ f (ξi )g(ξi )(ti − ti−1 ) − f (ξi )g(ηi )(ti − ti−1 ) < .

2
i=1 i=1

Logo
n
X Z b
lim f (ξi )g(ηi )(ti − ti−1 ) = f (x)g(x) dx,
|P |→0 a
i=1

como queríamos.

No elevador. Temos
n n
n
X X X
f (ξi )g(ξi )(ti − ti−1 ) − f (ξi )g(ηi )(ti − ti−1 ) = f (ξi )(g(ξi ) − g(ηi ))(ti − ti−1 ) .



i=1 i=1 i=1

Cada fator g(ξi ) − g(ηi ) que aparece na igualdade acima nos remete à oscilação de g no
intervalo [ti−1 , ti ]. Com respeito a isso, pelo item (4) do Teorema 4, sabemos que existe uma
partição P = {t0 , t1 , . . . , tn } de [a, b] tal que
n
X
ωi · (ti − ti−1 ) < ε,
i=1

onde ωi = ω(g; [ti−1 , ti ]). Como não é ωi |g(ξi ) − g(ηi )| 6 ωi


que queremos, notamos que
ε
(Corolário do Lema 5). E, como vamos multiplicar por f (ξi ), em vez de ε, tomamos , onde
K
K é um número positivo que limita f . Estas considerações nos dão a limitação por ε, mas para
uma partição P , enquanto queremos que ela valha para qualquer partição P com norma menor
do que δ , para algum δ . Dentro deste contexto é que enunciamos o lema abaixo.

Lema. Sejaϕ : [a, b] → R uma função integrável, onde a<b são números reais. Dado
ε > 0, existe δ > 0 tal que
n
X
ω(ϕ; [ti−1 , ti ]) · (ti − ti−1 ) < ε,
i=1
9.11. EXERCÍCIO 11. MUDANÇA DE VARIÁVEL 353

qualquer que seja a partição P = {t0 , t1 , . . . , tn } de [a, b] com norma menor do que δ.
Solução. Pelo Teorema 14 e seu corolário, sabemos que

Z b Z b
lim S(ϕ; P ) = ϕ(x) dx = ϕ(x) dx = lim s(ϕ; P ),
|P |→0 a |P |→0
a

ou seja, lim S(ϕ; P ) = lim s(ϕ; P ). Assim, dado ε > 0, existe δ>0 tal que
|P |→0 |P |→0

|P | < δ ⇒ S(ϕ; P ) − s(ϕ; P ) < ε,


n
X
onde a diferença S(ϕ; P )−s(ϕ; P ) é justamente igual ao somatório ω(ϕ; [ti−1 , ti ])·(ti −ti−1 ),
i=1
se P = {t0 , . . . , tn }, conforme já foi observado na demonstração do Teorema 4. Segue a tese.

Nível 2. Como f é limitada por hipótese (por ser integrável), seja K > 0 tal que |f (x)| <
K para todo x ∈ [a, b]. Dado ε > 0, seja δ > 0 tal que
n
X ε
P = {t0 , . . . , tn }, |P | < δ ⇒ ω(g; [ti−1 , ti ]) · (ti − ti−1 ) <
i=1
K
n
X ε
⇒ |g(ξi ) − g(ηi )| · (ti − ti−1 ) <
i=1
K
n
X
⇒ K · |g(ξi ) − g(ηi )| · (ti − ti−1 ) < ε
i=1
n
X
⇒ |f (ξi )| · |g(ξi ) − g(ηi )| · (ti − ti−1 ) < ε
i=1
X n
⇒ f (ξi ) · (g(ξi ) − g(ηi )) · (ti − ti−1 ) < ε .


i=1

A primeira implicação acima vem do lema anterior; a segunda, do Corolário do Lema 5; a


quarta, de K > |f (ξi )| para cadai, e; a quinta, da Desigualdade Triangular. Logo

X n n
X
P = {t0 , . . . , tn }, |P | < δ ⇒ f (ξi )g(ξi )(ti − ti−1 ) − f (ξi )g(ηi )(ti − ti−1 ) < ε,


i=1 i=1

como queríamos. 

9.11 Exercício 11. Mudança de variável


Seja f : [a, b] → R integrável e g : [c, d] → R monótona com derivada g0 integrável. Se

Z g(d) Z d
g([c, d]) ⊂ [a, b], então f (x) dx = f (g(t))g 0 (t) dt.
g(c) c

Solução.
354 CAPÍTULO 9. INTEGRAL DE RIEMANN

No elevador. Sequer sabemos ainda, de modo imediato, se (f ◦ g)g 0 é integrável. Come-


çamos, então, mexendo com esta função. A partir dela, o Teorema do Valor Médio (para deri-
vadas) nos dá um modo de chegar às somas de Riemann de f . Dada a partição P = {t0 , . . . , tn }
∗ 0
de [c, d], seja P o pontilhamento dos ξi 's tais que ξi ∈ [ti−1 , ti ] e g (ξi )(ti −ti−1 ) = g(ti )−g(ti−1 );
assim

X n
X
0 ∗
((f ◦ g)g ; P ) = f (g(ξi ))g 0 (ξi )(ti − ti−1 )
i=1
n
X
= f (g(ξi ))(g(ti ) − g(ti−1 ))
i=1
X
= (f ; g[P ∗ ]),

se g é não decrescente, pois, neste caso, g[P ] é uma partição para [g(c), g(d)], cujos intervalos
são da forma [g(ti−1 ), g(ti )]; e, se g é não crescente, então

n
X n
X X
f (g(ξi ))(g(ti ) − g(ti−1 )) = − f (g(ξi ))(g(ti−1 ) − g(ti )) = − (f ; g[P ∗ ]),
i=1 i=1

pois, desta vez, g[P ] particiona [g(d), g(c)] e seus intervalos são da forma [g(ti ), g(ti−1 )]. De
qualquer modo, os g(ξi )'s pontilham g[P ] por causa da monotonicidade de g . Note não haver
problema caso aconteça g(ti ) = g(ti−1 ) dentro da partição g[P ]; ela continua sendo uma partição
0
e g (ξi ) é igual a zero, o que anula as parcelas a mais das somas acima. Além disto, ao lidar
com a norma da partição g[P ], observamos que g é de Lipschitz porque sua derivada é limitada
e, daí, |g[P ]| 6 k|P |, onde k é a testemunha de que g é lipschitziana. Então podemos concluir
que
X Z g(d)
0 ∗
lim ((f ◦ g)g ; P ) = f (x) dx,
|P |→0 g(c)
se g é não decrescente e

X Z g(c) Z g(d)
0 ∗
lim ((f ◦ g)g ; P ) = − f (x) dx = f (x) dx,
|P |→0 g(d) g(c)

se g é não crescente. Mas isto para um pontilhamento especíco de cada P , e não para qualquer
pontilhamento, o que nos permitiria concluir este exercício.
∗∗
Indo para um pontilhamento P qualquer de P podemos tentar fazer com que seus pontos
ηi 's quem pelo menos próximos de satisfazerem a propriedade boa que os pontos de P ∗ cons-
0
truído acima possuem, a saber, a igualdade g (ξi )(ti − ti−1 ) = g(ti ) − g(ti−1 ). De modo mais
preciso, dado ε > 0, queremos δ > 0 tal que |P | < δ implique

g(ti ) − g(ti−1 ) 0


ti − ti−1 − g (ηi ) < ε.

Lembrando do Exercício 8 do Capítulo VIII (página 288), para cada ηi até é possível encon-
trar um δ tal que isto seja verdade caso ti − ti−1 < δ. O problema é que cada ηi poderia pedir
um δ diferente e, para termos um único δ que resolva o problema de todos os pontilhamentos
∗∗
P , seria bom se g fosse uniformemente derivável. Mas isto é pedir muito: uma função denida
1
num intervalo compacto é uniformemente derivável se, e somente se, é de classe C (Teorema 8
0
do Capítulo VII), e não temos a continuidade de g ; ela é integrável mas pode não ser contínua.
Explorando um pouco mais a desigualdade centralizada acima, acabamos percebendo que a
0
integrabilidade de g e, portanto, o fato de podermos controlar suas oscilações, pode ajudar.
9.11. EXERCÍCIO 11. MUDANÇA DE VARIÁVEL 355

Z g(d)
Nível 1. Seja I= f (x) dx. Podemos considerar este I porque f é integrável em [a, b]
g(c)
(item 1 do Teorema 5; se g é não decrescente, f |[a, g(d)] integrável ⇒ f |[g(c), g(d)] integrável;
0
se g é não crescente, f |[a, g(c)] integrável ⇒ f |[g(d), g(c)] integrável). Escrevamos h = (f ◦ g)g .
Dado ε > 0, nos níveis abaixo vamos encontrar δ > 0 tal que
X

|P | < δ ⇒ (h; P ) − I < ε,

qualquer que seja o pontilhamento P∗ de P. Encontrado este δ, o resultado desejado

Z d Z g(d)
h(x) dx = f (x) dx,
c g(c)

segue pelo Teorema 15 (h é limitada).

Nível 2. Vamos começar supondo g não decrescente. Fixada a partição P de [c, d] e


∗ ∗
um pontilhamento P seu (um pontilhamento qualquer), g[P ] particiona [g(c), g(d)] e g[P ]
pontilha g[P ]. Conforme a conversa no elevador acima já sugeriu, se a norma de P for
(f ; g[P ∗ ]) estará próximo o sucente de ambos os números
P
pequena o suciente, o número
(h; P ∗ ) e I (lembrando: h = (f ◦ g)g 0 ). É por isto que o introduziremos abaixo, escrevendo,
P
pela Desigualdade Triangular,
X X X X
∗ ∗ ∗ ∗
(h; P ) − I 6 (h; P ) − (f ; g[P ]) + (f ; g[P ]) − I .

No Nível 3.1, vamos obter δ1 > 0 tal que


X X ε
|P | < δ1 ⇒ (h; P ∗ ) − (f ; g[P ∗ ]) <

2
e, no Nível 3.2, δ2 > 0 tal que
X ε

|P | < δ2 ⇒ (f ; g[P ]) − I < .

2
Para estes números, denimos δ = min{δ1 , δ2 }, o delta procurado do Nível 1.

No elevador. Retomemos a desigualdade



g(ti ) − g(ti−1 ) 0


ti − ti−1 − g (ηi ) < ε.

Vimos mais acima não ser possível chegar até ela para todos os ηi 's através da derivabilidade
g (se g 0 não é contínua, então g não é uniformemente derivável). Porém, podemos
uniforme de
desenvolvê-la com o auxílio daqueles ξi 's do Teorema do Valor Médio para derivadas:

g(ti ) − g(ti−1 )
− g (ηi ) = |g 0 (ξi ) − g 0 (ηi )| < ε
0


ti − ti−1
⇒ |g 0 (ξi ) − g 0 (ηi )|(ti − ti−i ) < ε(ti − ti−i )
Xn
⇒ |g 0 (ξi ) − g 0 (ηi )|(ti − ti−i ) < ε(b − a)
i=1
356 CAPÍTULO 9. INTEGRAL DE RIEMANN

e aqui podemos perceber que a integrabilidade de g0 pode nos dar outro caminho para chegar
onde queremos. Lembrando da nossa motivação inicial de deixar g 0 (ηi )(ti − ti−1 ) próximo de
g(ti ) − g(ti−1 ), voltamos para

n
X n
X
0 0
|g (ξi ) − g (ηi )|(ti − ti−i ) = |[g(ti ) − g(ti−i )] − g 0 (ηi )(ti − ti−i )| < ε(b − a)
i=1 i=1

e lembrando também que queremos chegar à soma de Riemann de (f ◦g)g 0 calculada na partição
dos ηi 's, multiplicamos cada parcela do último somatório por |f (g(ηi ))|; com isto, usando sua
limitação por um certo K, obtemos

n
X
|f (g(ηi ))(g(ti ) − g(ti−i )) − f (g(ηi ))g 0 (ηi )(ti − ti−i )| < ε(b − a)K
i=1

e, pela Desigualdade Triangular,



Xn
f (g(ηi ))(g(ti ) − g(ti−i )) − f (g(ηi ))g 0 (ηi )(ti − ti−i ) < ε(b − a)K.



i=1

Daí, denotando por P∗ o pontilhamento dos ηi 's,


X X
∗ 0 ∗
(f ; g[P ]) − ((f ◦ g)g ; P ) < ε(b − a)K,

se g é não decrescente, e
X X
− (f ; g[P ∗ ]) − ((f ◦ g)g 0 ; P ∗ ) < ε(b − a)K,

se g é não crescente. Agora é só organizar essas ideias (lembrando que queremos a limitação
por ε /2).

Nível 3.1. Seja K > 0 tal que |f (g(x))| < K para todo x ∈ [c, d]. Dada uma partição
P = {t0 , . . . , tn } de [c, d], da integrabilidade de g 0 , seja δ1 > 0 tal que
n
X ε
|P | < δ1 ⇒ ω(g 0 ; [ti−1 , ti ]) · (ti − ti−1 ) <
i=1
2K

(lema da solução do exercício anterior). Assim, para ηi , ξi ∈ [ti−1 , ti ], ηi qualquer e ξi tal que
g 0 (ξ)(ti − ti−1 ) = g(ti ) − g(ti−1 ) (Teorema 7 do Capítulo VIII, o do Valor Médio para derivadas):
n
X ε
|P | < δ1 ⇒ |g 0 (ηi ) − g 0 (ξi )| · (ti − ti−1 )| <
i=1
2K
n
X ε
⇒ |g 0 (ηi )(ti − ti−1 ) − [g(ti ) − g(ti−1 )]| <
i=1
2K
n
X ε
⇒ |f (g(ηi ))g 0 (ηi )(ti − ti−1 ) − f (g(ηi ))(g(ti ) − g(ti−1 ))| <
2
i=1 ε
X 0 ∗
X

⇒ ((f ◦ g)g ; P ) − (f ; g[P ]) < ,
2
9.11. EXERCÍCIO 11. MUDANÇA DE VARIÁVEL 357

denotando por P∗ o pontilhamento dos ηi 's (isto é, P∗ é um pontilhamento qualquer de P ). A


primeira implicação acima vale pelo Corolário do Lema 5 e, a última, pela suposição de que gé
não decrescente, o que faz cada [g(ti−1 ), g(ti ))] ser um intervalo da partição g[P ] de [g(c), g(d)]

e g[P ] ser um pontilhamento de g[P ]: ηi ∈ [ti−1 , ti ] ⇒ g(ηi ) ∈ [g(ti−1 ), g(ti )]. Logo,
X X ε
∗ ∗
|P | < δ1 ⇒ (h; P ) − (f ; g[P ]) <

2
0
(lembrando que h = (f ◦ g)g ), como queríamos.

Nível 3.2. Como a integral da restrição f |[g(c), g(d)] (ainda supondo g não decrescente)
é I , seja δ0 tal que X ε
|Q| < δ0 ⇒ (f ; Q∗ ) − I < ,

2

onde Q é uma partição de [g(c), g(d)] e Q é um pontilhamento de Q. Em particular, dada a

partição P de [c, d] e dado seu pontilhamento P ,
X ε

|g[P ]| < δ0 ⇒ (f ; g[P ]) − I < ,

2

pois g[P ] é uma partição de [g(c), g(d)] e g[P ] é um pontilhamento de g[P ]. Mas não queremos
a norma |g[P ]| na hipótese da implicação acima, e, sim, a norma |P |. Então notamos que
|g[P ]| 6 k|P |, onde k é um número real positivo tal que |g(x) − g(y)| 6 k|x − y| para todo
x, y ∈ [c, d], o qual existe porque g é de Lipschitz, uma vez que sua derivada é limitada (Exercício
24 do Capítulo VIII, página 303). Deste modo, denimos δ2 = δ0 /k . Para ele, temos

δ0 X

ε
|P | < δ2 = ⇒ k|P | < δ0 ⇒ |g[P ]| < δ0 ⇒ (f ; g[P ]) − I < ,

k 2
como queríamos.

Nível 4. Adaptações em tudo que zemos acima, levando-se em conta os comentários


feitos nos elevadores, nos dão a solução deste exercício para o caso em que g é não crescente.
Dado ε > 0, sejam δ1 e δ2 números positivos tais que
X X ε
0 ∗ ∗
P é partição de [c, d], |P | < δ1 ⇒ ((f ◦ g)g ; P ) + (f ; g[P ]) <

2
e X ε
P é partição de [c, d], |P | < δ2 ⇒ (f ; g[P ∗ ]) + I < .

2
Assim, se P é uma partição de [c, d] com norma menor do que min{δ1 , δ2 }, então
X X X X
((f ◦ g)g 0 ; P ∗ ) − I = ((f ◦ g)g 0 ; P ∗ ) + (f ; g[P ∗ ]) − (f ; g[P ∗ ]) − I

X X X
0 ∗ ∗ ∗
6 ((f ◦ g)g ; P ) + (f ; g[P ]) + − (f ; g[P ]) − I

X X X
0 ∗ ∗ ∗
= ((f ◦ g)g ; P ) + (f ; g[P ]) + (f ; g[P ]) + I

ε ε
< +
2 2
= ε.
Logo, pelo Teorema 15, a função (f ◦ g)g 0 : [c, d] → R é integrável e
Z d Z g(d)
0
f (g(t))g (t) dt = I = f (x) dx,
c g(c)

como queríamos. 
358 CAPÍTULO 9. INTEGRAL DE RIEMANN

9.12 Exercício 12
Se f : [0, 2] → R e g : [−1, 1] → R são integráveis então

Z 2 Z π
2
(x − 1) · f [(x − 1) ] dx = 0 = g(sen x) · cos x dx.
0 0

Solução. Aqui podemos aplicar o exercício anterior, tomando cuidado principalmente com os
intervalos nos quais as funções de dentro são monótonas. Sejam ϕ : [0, 2] → R e ψ : [0, π] → R
2
dadas por ϕ(x) = (x − 1) e ψ(x) = sen x. Ambas são deriváveis, com derivadas  denidas por
0 0
ϕ (x) = 2(x − 1) e ψ (x) = cos x  integráveis; ϕ monótona em [0, 1] e em [1, 2], e ψ monótona
em [0, π/2] e em [π/2, π]. Além do que, ϕ[[0, 2]] ⊂ [0, 2] e ψ[[0, π]] ⊂ [−1, 1]. Então, com o item
1 do Teorema 5 em mente, fazemos

2
1 2
Z Z
(x − 1) · f ((x − 1) ) dx = 2
f (ϕ(x)) · ϕ0 (x) dx
0 2 0
Z 1 Z 2 
1 0 0
= f (ϕ(x))ϕ (x) dx + f (ϕ(x))ϕ (x) dx
2 0 1
"Z #
ϕ(1) Z ϕ(2)
1
= f (x) dx + f (x) dx
2 ϕ(0) ϕ(1)
Z 0 Z 1 
1
= f (x) dx + f (x) dx
2 1 0
= 0

e
Z π Z π
g(sen x) · cos x dx = g(ψ(x)) · ψ 0 (x) dx
0 0
Z π/2 Z π
0
= g(ψ(x))ψ (x) dx + g(ψ(x))ψ 0 (x) dx
0 π/2
Z ψ(π/2) Z ψ(π)
= g(x) dx + g(x) dx
ψ(0) ψ(π/2)
Z 1 Z 0
= g(x) dx + g(x) dx,
0 1
= 0
Z b Z a
lembrando da segunda convenção feita na página 319, a de que h(x) dx = − h(x) dx se
a b
h : [a, b] → R é uma função integrável. .

9.13 Exercício 13
a+b
Se f : [a, b] → R possui derivada integrável, ponha m= e prove que
2
Z b
2
f (a) + f (b) = [f (x) + (x − m)f 0 (x)] dx.
b−a a
9.13. EXERCÍCIO 13 359

Solução 1.

No elevador. Algumas tentativas... Do lado esquerdo da igualdade acima temos uma


soma, mas, para extrair uma integral dali, seria desejável vermos uma diferença. Deste racio-
cínio surgem as duas tentativas abaixo.
b−a
Tentativa 1. O número é igual ao número m − a, então temos
2
b−a
(f (a) + f (b)) = (m − a)(f (a) + f (b))
2
= m(f (a) + f (b)) − a(f (a) + f (b))
Z m
= f (a) + f (b) dx.
a

Nada...
Tentativa 2. Temos

Z b
f 0 (x) dx = f (b) − f (a) = f (b) + f (a) − 2f (a),
a

então
Z b Z b
0 2
f (b) + f (a) = f (x) dx + 2f (a) = f 0 (x) + f (a) dx.
a a b−a
Esta já pareceu um pouco melhor do que a primeira. Nela, ambas as funções f e f0 aparecem,
2
além do número .
b−a
0
Envolvendo as funções f e f , também podemos tentar a função xf (x). Sua derivada é
f (x) + xf 0 (x), número este que de fato aparece em nossa tese.

Seja a função g : [a, b] → R denida por g(x) = xf (x) para todo x ∈ [a, b]. Pela Regra do
Produto, sua derivada é dada por g (x) = f (x)+xf 0 (x). Derivada esta que é integrável, porque
0
0
soma de funções integráveis é integrável; xf (x) é integrável porque é um produto de funções
integráveis e f é integrável porque, admitindo derivada, é contínua (Teorema 1 do Capítulo
VIII) e funções contínuas são integráveis (Teorema 6). Então temos

Z b Z b Z b
0 0
[f (x) + (x − m)f (x)] dx = f (x) + xf (x) dx − m f 0 (x) dx
a a a
= [g(b) − g(a)] − m(f (b) − f (a))
= [bf (b) − af (a)] − mf (b) + mf (a)
= (b − m)f (b) + (m − a)f (a)
b−a b−a
= f (b) + f (a)
2 2
b−a
= (f (a) + f (b)),
2
isto é,
Z b
2
f (a) + f (b) = [f (x) + (x − m)f 0 (x)] dx,
b−a a

como queríamos. 
360 CAPÍTULO 9. INTEGRAL DE RIEMANN

Solução 2. Um outro modo de desenvolver as contas acima é notar que a função h denida
por h(x) = (x − m)f (x) é uma primitiva da função (integrável) dada por f (x) + (x − m)f 0 (x),
então, pelo Teorema Fundamental do Cálculo (Teorema 9):

Z b
[f (x) − (x − m)f 0 (x)] dx = (b − m)f (b) − (a − m)f (a)
a
= (b − m)f (b) + (m − a)f (a)
b−a
= (f (a) + f (b)),
2
Z b
2
logo, f (a) + f (b) = [f (x) + (x − m)f 0 (x)] dx. 
b−a a

9.14 Exercício 14
Z d
Sejam f, g : [a, +∞] [sic ] → R contínuas e K > 0 tal que

f (x) dx 6 K para quais-

c
1
quer c, d ∈ [a, +∞). Se g ∈ C é decrescente, com lim g(x) = 0, prove que existe o limite
Z ∞ Z x x→∞

f (x)g(x) dx = lim f (t)g(t) dt (integral imprópria). [Sugestão : Critério de Cauchy.


a x→∞ a
Z d

Mostre que ∀ ε > 0 ∃A > a tal que A < c < d ⇒ f (x)g(x) dx < ε. Para estimar esta

c
integral, use o item C do Teorema 12.]

Solução. A sugestão dada é o Critério de Cauchy para Funções, o Teorema 8 do Capítulo


VI junto com a observação feita
Z xna página 208, a de que este resultado também vale quando
x → +∞; a igualdade F (x) = f (t)g(t) dt dene uma função real no intervalo [a, +∞) pelos
a
Teoremas 5 do Capítulo VII (f g é contínua) e 6 (funções contínuas são integráveis), e, para
Z d
esta F, f (x)g(x) dx = F (d) − F (c).
c
Para aplicar o item C do Teorema 12, notamos que g decrescente e lim g(x) = 0 implicam
x→+∞
0
g positiva, e g é integrável porque é contínua (novamente o Teorema 6). Assim, dados c<d
Z d Z α
em [a, ∞), para α ∈ [c, d] tal que f (x)g(x) dx = g(c) · f (x) dx, temos
c c

d α
Z Z


f (x)g(x) dx = |g(c)| · f (x) dx 6 |g(c)| · K.
c c

Portanto, xado ε > 0, o resultado segue tomando-se A>a tal que

ε
x > A ⇒ |g(x)| < ,
K
o qual existe pelo fato de g(x) tender a zero conforme x tende a +∞. 

9.14.1 Exercício 14a

Considerando funções escada [ sic ] convenientes, mostre que o Critério de Dirichlet (Teorema
21, Capítulo IV) é consequência do Exercício 14.
9.14. EXERCÍCIO 14 361

Solução.

P
Nível 1. Sejam an e (bn )n∈N!uma série e uma sequência que satisfazem as hipóteses do
Xn
Critério de Dirichlet, isto é, ai é uma sequência limitada e a sequência dos bn 's é não
i=1 n∈N
crescente, de números positivos e tende a zero. Pelo Critério de Cauchy para séries (Teorema
17 do Capítulo IV), dado ε > 0, vamos encontrar (no Nível 2) um n0 ∈ N tal que
n+p
X
n ∈ N, n > n0 ⇒ ai b i < ε ∀p ∈ N .


i=n+1

P
No elevador. Para aplicar o Exercício 14, vamos aproximar a série an b n por uma
integral de uma função do tipo f g. Quem nos dá a ideia de como fazer esta aproximação é
a Solução do Exercício 6 (página 346), observando que aquela hipótese de f ser não negativa
P
pode ser dispensada; passando por funções-escada, an
f e (bn ), a g . Para
vai dar origem a
tanto, observamos que a hipótese de g ser decrescente pode ser trocada pelas hipóteses de g ser
não crescente e positiva. Isto porque essa hipótese foi usada apenas para aplicar o item C do
Teorema 12 (a positividade de g decorreu de ela ser decrescente e de lim g(x) = 0; por isso,
x→+∞
ao tirar o decrescente, precisamos colocar o positiva), mas, para aplicá-lo, podemos fazer
essas hipóteses mais fracas. Além do que, exatamente a mesma demonstração do referido item
funciona supondo-se p não crescente em vez de supor p decrescente. Logo, não iremos aplicar
o Exercício 14 ipsis litteris, mas sim a sua adaptação resultante dessas observações.
n+p
X
Nível 2. Começamos observando que o somatório ai b i pode ser intepretado como
i=n+1
uma integral cujo integrando é a função que associa cada x ∈ [n, n + 1) ao número an b n , para
cada n ∈ N, e cujo intervalo de integração é [n + 1, n + p + 1]. Escrevendo
n+p n+p Z n+p+1 Z n+p+1
X X
ai b i = ai b i − f (x)g(x) dx + f (x)g(x) dx,
i=n+1 i=n+1 n+1 n+1

vamos obter, no Nível 3 (e sub-níveis), f, g : [1, +∞) → R contínuas tais que:


Z d 
f (x) dx; c, d ∈ [1, +∞)
c

é limitado; g é positiva, não crescente, derivável, com derivada integrável e tal que lim g(x) = 0,
x→∞
e; n+p
X Z n+p+1 ε
(∗) ai b i − f (x)g(x) dx < ∀n, p ∈ N .

2


i=n+1 n+1
Z ∞
Assim, pelo Critério de Cauchy para funções, como f (x)g(x) dx é convergente (pela
1
adaptação do exercício anterior, conforme as observações feitas no elevador acima), o n0 ∈ N
procurado é aquele que satisfaz a implicação

n+p+1
Z
ε
n > n0 ⇒ f (x)g(x) dx < .
n+1 2
362 CAPÍTULO 9. INTEGRAL DE RIEMANN

Para ele, se n ∈ N é maior do que n0 (e se vale a desigualdade (∗)), então


n+p Z n+p+1 Z
X n+p+1
ε ε
ai b i − f (x)g(x) dx + f (x)g(x) dx < + = ε .

2 2


i=n+1 n+1 n+1

Disto, e da desigualdade

n+p n+p Z n+p+1 Z


X X n+p+1

ai b i 6 ai b i − f (x)g(x) dx + f (x)g(x) dx ,



i=n+1

i=n+1n+1 n+1

válida para todo n e para todo p naturais, segue o resultado desejado

n+p
X
n > n0 ⇒ ai bi < ε,


i=n+1

para todo p ∈ N.

No elevador. É neste momento de aproximar nossos objetos em questão por funções


é que se torna conveniente olhar para eles como funções-escada. Conforme já mencionamos
anteriormente, na Solução do Exercício 6 já vimos como aproximar uma função-escada por
uma função contínua. O problema é que esta construção serve apenas para se obter f, mas
não g; a função obtida daquele modo sequer é derivável. Felizmente, também já vimos num
exercício anterior como ligar trechos constantes de modo mais suave, a saber, no Exercício
35 do Capítulo VIII (página 314). A partir dele, fazemos as devidas correções para fazer
coincidir uma ponta desse trecho com um dos extremos daquele intervalo e a outra ponta
com o ponto de máximo daquela função. Estas correções são feitas através de uma dilatação e
de uma translação. De modo mais preciso, enunciamos o lema abaixo.

Lema. Sejam a 6 x1 < x2 6 b e y1 > y2 . A função ϕ : [a, b] → R denida por ϕ(x) = y1


para todo x ∈ [a, x1 ], ϕ(x) = y2 para todo x ∈ [x2 , b] e

y1 − y2 1
ϕ(x) =   · exp + y2 ∀x ∈ (x1 , x2 )
1 (x − (2x1 − x2 ))(x − x2 )
exp −
(x2 − x1 )2

é de classe C 1. Além disto, no contexto em que este lema se insere, também é útil observar que

 Z x2 
lim y1 (x2 − x1 ) − ϕ(x) dx = 0
x1 →x2 x1

(bem entendido: a denição de ϕ também varia conforme x1 varia, então talvez devéssemos
escrever ϕ(x) = ϕ(x1 )(x); ϕ(x1 ) : [a, b] → R sendo a função construída no lema, dependente de
x1 ).
Em outras palavras, considerando a função-escada ξ : [a, b] → R denida por ξ(x) = y1
para todo x ∈ [a, x2 ) e ξ(x) = y2 para todo x ∈ [x2 , b], se x1 estiver próximo o suciente de
Z b Z b
x2 , então a diferença ξ(x) dx − ϕ(x) dx entre as integrais de ξ e ϕ pode se tornar tão
a a
pequena quanto se queira.
9.14. EXERCÍCIO 14 363

Solução. O fato de ϕ ser de classe C 1 segue do Exercício 35 do Capítulo VIII e de sua


solução, observando que x1 é o ponto médio do intervalo (2x1 − x2 , x2 ), ponto em que ocorre o
valor de máximo da função considerada naquele exercício (conforme já vimos em sua solução),
0
portanto, ϕ (x1 ) = 0. Com relação a última armação feita no lema, pelo item A do Teorema
12, para x0 ∈ (x1 , x2 ) tal que
Z x2
ϕ(x) dx = ϕ(x0 )(x2 − x1 ),
x1

temos Z x2
y1 (x2 − x1 ) − ϕ(x) dx = (y1 − ϕ(x0 ))(x2 − x1 ).
x1

Assim, conforme x1 se aproxima de x2 , o número x2 − x1 se aproxima de zero, donde lim (y1 −


x1 →x2
ϕ(x0 ))(x2 − x1 ) = 0 em consonância com o Teorema 7 do Capítulo VI, porque y2 6 ϕ(x0 ) 6
y1 ⇒ y2 − y1 6 ϕ(x0 ) − y1 6 0.

No elevador. A m de enxergar mais claramente o comportamento do produto fg (o


comportamento que interessa para nossos propósitos), as alterações que faremos nas funções-
escada convenientes para as obtenções de f e de g serão feitas em intervalos que não possuem
interseção.

Nível 3. Sejam (γn )n∈N e (δn )n∈N sequências de números reais tais que 0 < γn < 1/2 e
0 < δn < 1/2 para todo n. E denamos as funções f, g : [1, +∞) → R fazendo: f (x) = a1 para
todo x ∈ [1, 1 + γ1 ], f (x) = an ∀x ∈ [n + γn , n + 1], g(x) = bn ∀x ∈ [n, n + 1 − δn+1 ],

an+1 − an
f (x) = (x − (n + 1)) + an ∀x ∈ [n + 1, n + 1 + γn+1 ]
γn+1
e

bn − bn+1 1
g(x) =   ·exp +bn+1 ∀x ∈ (n+1−δn+1 , n+1),
1 (x − (n + 1 − 2δn+1 ))(x − (n + 1))
exp −
(δn+1 )2
para cada n ∈ N.
Com esta construção: f é contínua, g ∈ C 1, g é positiva, g é não crescente e lim g(x) = 0.
x→+∞
No Nível 3.1, vamos encontrar sequências (γn )n∈N
(δn )n∈N e que façam valer a desigualdade
n+p Z n+p+1
X ε
ai b i − f (x)g(x) dx < ∀n, p ∈ N

2


i=n+1 n+1
364 CAPÍTULO 9. INTEGRAL DE RIEMANN

d
Z

e, no Nível 3.2, vamos encontrar K tal que
f (x) dx 6 K ∀c, d ∈ [1, +∞).
c

Nível 3.1. Devido ao apelo geométrico da questão, considerar as funções-escada ξ, χ :


[1, ∞), denidas por
P ξ(x) = an
χ(x) = b n e
para todo x ∈ [n, n + 1), para todo n ∈ N, nos
lugares da série an e da sequência (bn ) nos auxiliou a obter f e g . De modoZ similiar, a função- n+1
escada ξ · χ, que nos permite interpretar o número an b n como a integral ξ(x) · χ(x) dx,
n
também nos auxilia a armar o seguinte: se

n+1
Z

an − f (x) dx < An ∀n ∈ N

n
e
n+1
Z

bn − g(x) dx < Bn ∀n ∈ N,

n
então
n+1
Z

an b n − f (x)g(x) dx 6 bn · An + |an | · Bn ∀n ∈ N .

n
Deste modo, dados os naturais n e p, temos
n+p Z n+p+1 n+p  Z i+1 
X X
ai b i − f (x)g(x) dx = ai b i − f (x)g(x) dx



i=n+1 n+1
i=n+1 i
n+p Z i+1
X
6 ai b i − f (x)g(x) dx

i=n+1 i
n+p
X
6 bi Ai + |ai |Bi .
i=n+1

Ambas as sequências (|an |)n e (bn )n são limitadas (para a limitação da primeira, ver o m
do Nível 3.2; a segunda é limitada porque é convergente). Sendo X tal que|an | + bn < X para
ε
todo n, escolhemos as sequências (γn )n e (δn )n de tal modo a termos An = Bn = n ; assim
2 ·X
continuamos a conta acima:
n+p n+p
X X ε
bi Ai + |ai |Bi = (|ai | + bi ) ·
i=n+1 i=n+1
2i ·X
n+p
X ε
<
i=n+1
2i
ε
< ,
2
como queríamos.

Nível 3.2. A limitação desejada é consequência do fato de a função f estar próxima


da função-escada ξ denida por ξ(x) = an para todo x ∈ [n, n + 1) e todo n ∈ N. Fixados
c, d ∈ [1, +∞), temos

d d d d
Z Z Z Z


f (x) dx 6 ξ(x) dx − f (x) dx + ξ(x) dx ,
c c c c
9.15. EXERCÍCIO 15 365

onde (de modo semelhante ao que zemos no Nível 3.1, para c = n+1 e d = n + p + 1, e
n
lembrando que An = ε /(2 · X))

d d
Z Z
ε
ξ(x) dx − f (x) dx <

c c X
e Z d Z n2
ξ(x) dx = an1 −1 (n1 − c) + ξ(x) dx + an2 (d − an2 ),
c n1

para n1 = min{n ∈ N; n1 > c} (denindo assim o n1 é pelo menos 2 e, portanto, o número


an1 −1 está denido) e n2 = max{n ∈ N; n 6 d}, donde
Z d Z n2

ξ(x) dx 6 |an1 −1 |(n1 − c) + ξ(x) dx + |an2 |(d − an2 )

c n1
Z n2

6 |an1 −1 | + ξ(x) dx + |an2 |
n1
6 6A,
n
X
para A > 0 tal que ai 6 A para todo n ∈ N, pois isto implica


i=1
n+p n+p n+p
X X n
X X X n
a = a − a 6 a + a i 6 2A ∀n, p ∈ N .

i i i i

i=n+1 i=1 i=1 i=1 i=1

ε
Logo, o K procurado é K = 6A + . 
X

9.15 Exercício 15
Z ∞
A integral imprópria (sen x/x) dx é convergente mas não converge absolutamente, isto é,
0
Z x
pondo F (x) = | sen x/x| dx [sic ], temos lim F (x) = ∞.
0 x→∞

Solução.

Primeira Parte. Sendo f a função seno e g a função denida por 1/x, temos

d
Z


f (x) dx = | cos c − cos d| 6 2 ∀c, d ∈ [0, +∞),
c

g decrescente e tal quelim g(x) = 0. O problema é que g não é de classe C1 no intervalo


x→∞
[0, +∞), mas ela o é, por exemplo, no intervalo [1, +∞). Então escrevemos
Z x Z x Z 1
sen t sen t sen t
dt = dt + dt ∀x ∈ [0, +∞)
0 t 1 t 0 t
sen t
(lembrando que lim = 1), onde o limite
t→0 t
Z x
sen t
lim dt
x→+∞ 1 t
366 CAPÍTULO 9. INTEGRAL DE RIEMANN

existe pelo Exercício 14. Portanto, também existe o limite

Z x
sen t
lim dt,
x→+∞ 0 t

como queríamos.

Segunda Parte.

d
Z

No elevador. Começamos notando que não existe K > 0 tal que
| sen x| dx 6 K
c
para quaisquer c, d ∈ [0, +∞), pois

Z (m+k)π
| sen x| dx = 2k ∀m, k ∈ N .

Assim como no Exercício 14, aqui também podemos usar o Teorema 12 para estimar o valor
de F. Pelo seu item C (o mesmo usado lá), dados 0 < a < b, existe c ∈ [a, b] tal que

Z c
1
F (b) − F (a) = | sen x| dx.
a a

Mas, apesar de sabermos calcular a integral do módulo do seno em certos intervalos  como
vimos no primeiro parágrafo  não sabemos como este número c se comporta, se ele estiver
muito próximo de a, a integral de | sen x| no intervalo de a a c pode se tornar muito pequena.
Outra tentativa que podemos fazer é utilizar o item A [do Teorema 12]. Por ele, para todo
x>0 existe y ∈ (0, x) tal que

| sen y|
F (x) = · x.
y
Novamente, não sabemos como este y se comporta, ele pode ser tal que o número | sen y|/y se
torne pequeno demais; o bom seria se y fosse tal que | sen y|/y fosse sempre maior do que um
mesmo número positivo.
Com relação ao item B, para todo x > 1, existe y ∈ [1, x] tal que

F (x) − F (1) = | sen y| · ln x.

Mesmo problema: ln x → +∞, mas não sabemos como o | sen y| afeta o ln x.


Outras tentativas são: pensar em séries  por exemplo, por um raciocínio análogo ao do
Exercício 47 do Capítulo IV (segundo o qual o conjunto dos valores de aderência da sequência
(cos n)n∈N é [−1, 1]), talvez possamos armar que existe um conjunto innito I tal que | sen n| >
1/2 para todo n ∈ I e, portanto, | sen n|/n > 1/(2n) ∀n ∈ I , e usar a divergência da série
0
harmônica  ou aplicar o Exercício 17 do Capítulo VIII  se o limite de F (x) = | sen x|/x
conforme x → +∞ não fosse 0, então, por tal exercício, o limite de F (x) para x → +∞ não
existiria, mas não é o caso . No entanto, há também um outro modo de aplicarmos o item B,
e é esta a ideia que iremos desenvolver abaixo, retomando a conta com a qual iniciamos esta
introdução.
9.15. EXERCÍCIO 15 367

Nível 1. A função F é não decrescente (Nível 2.1). Assim, dado M > 0, basta obter
x0 > 0 tal que F (x0 ) > M .
Com o Critério de Cauchy em mente, armamos que existe ε > 0 tal que, dado A > 0,
existem números c<d maiores do que A tais que
Z d
| sen x|
dx > ε
c x

(Nível 2.2). Com isto, para n ∈ N tal que n · ε > M (Propriedade Arquimediana), conseguimos
obter números 0 = a0 < a1 < · · · < an tais que F (ai ) − F (ai−1 ) > ε para todo i ∈ {1, . . . , n}
(Nível 2.3). Para eles, temos

n
X
F (an ) = F (ai ) − F (ai−1 ) > n ε > M
i=1

(pois F (a0 ) = 0) e an é o número x0 procurado.

Nível 2.1. Se 0 6 a < b, então

b
| sen x|
Z
F (b) = F (a) + dx > F (a),
a x
b
| sen x|
Z
pois dx > 0 pelo item 4 do Teorema 5.
a x

Nível 2.2. Por um lado, pelo item B do Teorema 12, dados os números naturais m e k,
existe c0 ∈ [mπ, (m + k)π] tal que

Z (m+k)π
1
F ((m + k)π) − F (mπ) = | sen x| dx,
c0 mπ

onde Z (m+k)π
| sen x| dx = 2k

(Nível 3), portanto, como c0 6 (m + k)π ,


2k 2k
F ((m + k)π) − F (mπ) = > .
c0 (m + k)π
Por outro, xado m, temos
2k 2
lim = .
k→+∞ (m + k)π π
 
2
Assim, xamos ε ∈ 0, . Pelas cosiderações acima, para um tal ε, podemos concluir o
π
seguinte. Para todo m ∈ N, existe k ∈ N tal que

F ((m + k)π) − F (mπ) > ε .

Então, dado A > 0, para m ∈ N tal que mπ > A e para o k como no parágrafo acima, o
resultado desejado segue fazendo-se c = mπ e d = (m + k)π .
368 CAPÍTULO 9. INTEGRAL DE RIEMANN

Nível 2.3. Começamos com a0 = 0 e, xado A > 0, obtemos

A < a < b < a1 < b1 < a2 < b2 < · · · < an−1 < bn−1 = an
tais que
F (b) − F (a) > ε e F (bi ) − F (ai ) > ε ∀i ∈ {1, · · · , n − 1}
(obtemos a e b aplicando a armação que levou ao Nível 2.2, depois obtemos a1 e b1 do mesmo
modo, agora para b no lugar de A, e assim por diante).
Temos
F (ai ) − F (ai−1 ) > ε ∀i ∈ {1, . . . , n}
porque
F (a1 ) − F (a0 ) = F (a) + (F (b) − F (a)) + (F (a1 ) − F (b)),
F (ai ) − F (ai−1 ) = (F (bi−1 ) − F (ai−1 )) + (F (ai ) − F (bi−1 )) ∀i ∈ {2, 3 . . . , n}
e F (a), F (a1 ) − F (b) e F (ai ) − F (bi−1 ) para i ∈ {2, 3 . . . , n} são todos números não negativos
pelo item 4 do Teorema 5.

Nível 3. Fixado m ∈ N, vamos mostrar que


Z (m+k)π
| sen x| dx = 2k ∀k ∈ N

por indução em k.
Para k = 1, temos

sen x ∀x ∈ [mπ, (m + 1)π], se m é par, e
| sen x| =
− sen x ∀x ∈ [mπ, (m + 1)π], se m é ímpar.

Daí, se m é par, então


Z (m+1)π Z (m+1)π
| sen x| dx = sen x dx = cos(mπ) − cos((m + 1)π) = 2
mπ mπ
e, se m é ímpar, então
Z (m+1)π Z (m+1)π
| sen x| dx = − sen x dx = −(cos(mπ) − cos((m + 1)π)) = 2.
mπ mπ
De qualquer forma,
Z (m+1)π
| sen x| dx = 2 · 1,

como queríamos.
Z (m+1)π
Usando este fato que acabamos de demonstrar, isto é, que | sen x| dx = 2 para todo

m ∈ N, se
Z (m+k)π
| sen x| dx = 2k,

então
Z (m+k+1)π Z (m+k)π Z (m+k+1)π
| sen x| dx = | sen x| dx + | sen x| dx
mπ mπ (m+k)π
= 2k + 2
= 2(k + 1).
Segue a tese. 
9.16. EXERCÍCIO 16 369

9.16 Exercício 16
Para cada inteiro p, com 0 6 p 6 n, escreva (1 − t)n = (1 − t)n−p (1 − t)p na expressão do resto
Rn+1 da fórmula de Taylor como integral (veja o Teorema 13). Obtenha então

(1 − θ)n−p f (n+1) (a + θh) n+1


Rn+1 = ·h , 0 < θ < 1.
n!(p + 1)
Tomando p = n, reobtenha o resto de Lagrange e, fazendo p = 0, ache o resto de Cauchy:

(1 − θ)n f (n+1) (a + θh) n+1


Rn+1 = ·h .
n!
Pondo a+h=b e a + θh = ξ , o resto de Cauchy se torna

f (n+1) (ξ)
Rn+1 = (b − ξ)n (b − a), onde a < ξ < b.
n!

Solução.

Nível 1. Dada a função f : [a, a + h] → R, n + 1 vezes derivável e com derivada de ordem


n+1 integrável, queremos θ ∈ (0, 1) tal que
1
(1 − t)n−p · (1 − t)p (n+1) (1 − θ)n−p f (n+1) (a + θh)
Z
·f (a + th) dt = .
0 n! n!(p + 1)
Vamos obtê-lo no Nível 2. As outras armações são imediatas. Porém, convém ressaltar que,
para a obtenção do resto de Lagrange através do caminho sugerido neste exercício, é preciso
(n+1)
acrescentar, no Teorema 10 do Capítulo VIII (o respectivo teorema), a hipótese de que f
é integrável. Com relação ao Resto de Cauchy, substituindo a = b − h em ξ = a + θh, obtemos
n n+1
ξ = (b − h) + θh, o que implica h − θh = b − ξ, de modo que (1 − θ) h = (h − θh)n h =
(b − ξ)n (b − a).

No elevador. O objetivo pretendido se assemelha a alguma das fórmulas de valor médio


para integrais. Dos itens do Teorema 12 (o teorema destas fórmulas), o que mais se encaixa
nesta situação é o B. Isto é até reforçado levando-se em conta a primeira observação da página
(1 − t)n−p (n+1)
329. Mais precisamente, denindo g(t) = ·f (a + th), queremos θ ∈ (0, 1) tal que
n!
Z 1 Z 1
p 1
g(t) · (1 − t) dt = g(θ) · (1 − t)p dt = g(θ) · .
0 0 p+1
A função denida por (1 − t)p é integrável e não muda de sinal, mas ainda não podemos
aplicar o resultado desse item, pois não temos a continuidade de g. Revisitando sua demons-
tração, podemos ver que o que foi importante da continuidade foram a limitação e o Teorema
(n+1)
do Valor Intermediário. E a g possui estas propriedades: ela é limitada  f é limitada
porque é integrável (tacitamente supomos que as funções integráveis são limitadas, porque a
denição de integrabilidade foi feita apenas para funções limitadas, de modo que falar numa
função ilimitada integrável seria algo desprovido de sentido)  e satisfaz o Teorema do Valor
Intermediário, porque ela é uma derivada, a saber, a derivada da função G dada por

t
f (n) (a + th) (1 − t)n−p f (n) (a + xh) (1 − x)n−p−1 (n − p)
Z
G(t) = · + · dx,
h n! 0 h n!
370 CAPÍTULO 9. INTEGRAL DE RIEMANN

(n)
onde o integrando que aí aparece é uma função contínua, f contínua por admitir a derivada
(n+1) 0
f , então podemos aplicar o Teorema 8 (página 322) para nos ajudar a concluir que G (t) =
g(t).
Deste modo, podemos seguir os mesmos passos da demonstração do item B do Teorema 12
(tomando o cuidado de não pensar que g atinge seus valores de máximo e de mínimo). Isto nos
dará θ ∈ [0, 1]. Num segundo momento, concluímos que este θ pode ser tomado no intervalo
(0, 1), o que pode indicar também uma direção a ser tomada para a justicativa da observação
feita no livro.

Nível 2. Seja g : [0, 1] → R a função denida por

(1 − t)n−p (n+1)
g(t) = ·f (a + th) ∀t ∈ [0, 1].
n!
Conforme já mencionamos, ela é integrável (portanto limitada) e admite uma primitiva (por-
tanto g satisfaz o Teorema do Valor Intermediário para Derivadas, o Teorema 5 do Capítulo
VIII). Sejam m = inf{g(x); x ∈ [0, 1]} e M = sup{g(x); x ∈ [0, 1]}. Seja também q : [0, 1] → R
p
a função denida por q(t) = (1 − t) para todo t ∈ [0, 1]; q(t) > 0 ∀t ∈ [0, 1]. Deste modo,
temos:

m 6 g(t) 6 M ∀t ∈ [0, 1] ⇒ mq(t) 6 g(t)q(t) 6 M q(t) ∀t ∈ [0, 1]


Z 1 Z 1 Z 1
⇒ m q(t) dt 6 g(t)q(t) dt 6 M q(t) dt.
0 0 0
Z 1 Z 1
O d ∈ [m, M ] tal que g(t)q(t) dt = d q(t) dt existe por causa da continuidade da
0 0Z
1
função ϕ : [m, M ] → R denida porϕ(x) = x · q(t) dt para x ∈ [m, M ]; com esta notação,
Z 1 0

a desigualdade acima ca ϕ(m) 6 g(t)q(t) dt 6 ϕ(M ), e, então, existe d ∈ [m, M ] tal
0
Z 1
que g(t)q(t) = ϕ(d). Para nalizar, isto é, para encontrar θ ∈ (0, 1) tal que g(θ) = d,
0
consideremos as possibilidades com respeito ao lugar onde d está.
Se d ∈ (m, M ), x1 , x2 ∈ [0, 1] tais que g(x1 ) < d < g(x2 ), os quais existem porque,
tomamos
do conjunto dos valores que g assume em [0, 1], m é o ínmo e M é o supremo. Então, pelo
Teorema do Valor Intermediário para Derivadas (Teorema 5 do Capítulo VIII), seja d0 ∈ (x1 , x2 )
ou d0 ∈ (x2 , x1 ) (dependendo de quais dos dois for menor do que o outro, x1 < x2 ou x2 < x1 ;
o referido teorema vale para qualquer um destes dois casos) tal que d = g(d0 ); d0 ∈ (0, 1) e é
este o θ procurado.
Se d = m, então
Z 1 Z 1 Z 1 Z 1

g(t)q(t) dt = m q(t) dt ⇒ [g(t) − m]q(t) dt = [g(t) − m]q(t) dt = 0,
0 0 0 0

pois g(t) − m > 0 e q(t) > 0 para todo t ∈ [0, 1]. Assim, pelo Exercício 2 (página 342)  em
particular, pelo fato de seu item 1 implicar seu item 3 , seja θ ∈ (0, 1) tal que [g(θ)−m]q(θ) = 0.
Então, de q(θ) 6= 0 q(t) > 0 ∀t ∈ [0, 1)), só pode ser
(pois g(θ) = m, como queríamos.
Analogamente para d = M:
Z 1 Z 1 Z 1 Z 1
g(t)q(t) dt = M q(t) dt ⇒ [M − g(t)]q(t) dt = [M − g(t)]q(t) dt = 0,
0 0 0 0
9.17. EXERCÍCIO 17 371

o que implica g(θ) = M paraθ ∈ (0, 1) tal que [M − g(θ)]q(θ) = 0.


De qualquer forma, encontramos θ ∈ (0, 1) tal que
1 Z 1
(1 − t)n (n+1)
Z
·f (a + th) dt = g(t) · (1 − t)p dt
0 n! 0
Z 1
= g(θ) · (1 − t)p dt
0
1
= g(θ) ·
p+1
(1 − θ)n−p f (n+1) (a + θh)
= ,
n!(p + 1)
como queríamos. 

9.17 Exercício 17
Use o resto de Cauchy no desenvolvimento de Taylor de f (x) = (1 + x)α , α ∈
/ N, para mostrar
que a série de Taylor de f em torno do ponto 0, converge para f (x) desde que −1 < x < 1.
(Série do Binômio.)

Solução.

Nível 1. Para x pertencente ao intervalo aberto (−1, 1), sejaI o intervalo [0, x] ou o
intervalo [x, 0], dependendo se x>0 ou x<0 respectivamente. Em I , f é de classe C ∞ e sua
derivada de qualquer ordem é integrável; qualquer que seja o natural n,

f (n) (x) = α(α − 1) · · · (α − (n − 1))(1 + x)α−n .


Fixado n ∈ N, pelo Resto de Cauchy,

f (n) (0) n
f (x) = f (0) + f 0 (0) · x + · · · + · x + Rn+1 (x),
n!
onde

(1 − θ)n f (n+1) (θx) n+1


Rn+1 (x) = ·x
n!
(1 − θ)n · α(α − 1) · · · (α − n)(1 + θx)α−n−1 n+1
= ·x ,
n!
para θ no intervalo (0, 1).
P f (n) (0) n
Conforme já foi observado na página 289, para que a série ·x convirja para f (x)
n!
é suciente que
lim Rn+1 (x) = 0.
n→+∞

É isto então que iremos mostrar abaixo.

No elevador. Dos diversos modos com os quais podemos abordar o número Rn+1 (x),
vem a calhar o Exercício 38 do Capítulo IV, cuja aplicação nos leva também ao Exemplo 21 do
Capítulo IV.
372 CAPÍTULO 9. INTEGRAL DE RIEMANN

Nível 2. Com a notação do Exercício 38 do Capítulo IV (página 150), de modo bastante


conveniente (levando-se em conta o comentário feito acima, no elevador) escrevemos

     n 
α n+1 1−θ
Rn+1 (x) = ·x · n· · (1 + θx)α−1 .
n+1 1 + θx

Pelo referido exercício, como −1 < x < 1, temos

 
α
lim · xn+1 = 0,
n→+∞ n+1

O fator (1+θx)α−1 é um número real (o x está xado). Agora, pelo Exemplo 21 do Capítulo
IV  pelo o qual podemos concluir que lim nan = 0 se |a| < 1  para concluirmos que
n→+∞

 n
1−θ
lim n · = 0,
n→+∞ 1 + θx

basta vericarmos a desigualdade



1−θ
1 + θx < 1.

De fato,

[0 < θ < 1 e − 1 < x] ⇒ −θ < θx ⇒ 0 < 1 − θ < 1 + θx,

donde

1−θ 1−θ
1 + θx = 1 + θx < 1.

Logo

lim Rn+1 (x) = 0,


n→+∞

um número real multiplicado por fatores que tendem a zero, como queríamos. 

Observação. Notemos como o Resto de Cauchy foi útil para a conclusão deste exercício. O
resto de Lagrange de f é

f (n) (θx) n
 
α n 1
rn (x) = ·x = x · · (1 + θx)α ,
n! n (1 + θx)n

1
onde → +∞ se 1 + θx < 1, o que ocorre quando x < 0.
(1 + θx)n

9.18 Exercício 18
Seja f : [a, b] → R limitada. Prove que f é integrável se, e somente se, existe um número real
IP
com a seguinte propriedade: para todo ε > 0, pode-se obter uma partição Pε de [a, b] tal que
| (f ; Q∗ ) − I| < ε seja qual for a partição pontilhada Q∗ com Q ⊃ Pε .
9.18. EXERCÍCIO 18 373

Z b
Solução. Se f é integrável, vejamos que I=
f (x) dx possui a propriedade desejada.
a
Se Pε ⊂ Q, então, pelo Teorema 1 e, conforme já foi observado na página 333, temos
X
s(f ; Pε ) 6 s(f ; Q) 6 (f ; Q∗ ) 6 S(f ; Q) 6 S(f, Pε ),

de modo que, pela Desigualdade Triangular,


X X
∗ ∗
(f ; Q ) − I 6 (f ; Q ) − s(f ; Pε ) + |s(f ; Pε ) − I|

6 |S(f ; Pε ) − s(f ; Pε )| + |s(f ; Pε ) − I|.

Então, xado ε > 0, escolhemos uma partição Pε de [a, b] tal que tenhamos simultaneamente

ε ε
|S(f ; Pε ) − s(f ; Pε )| < e |s(f ; Pε ) − I| < .
2 2
Uma tal partição existe pelo Teorema 14 e seu corolário, porque, deles, podemos concluir que

lim S(f ; P ) − s(f ; P ) = 0 e lim s(f ; P ) = I,


|P |→0 |P |→0

já que estamos supondo f integrável.


Reciprocamente, dado ε > 0, pelo Teorema 4, vamos encontrar uma partição P de [a, b] tal
que
|S(f ; P ) − s(f ; P )| < ε .
Precisamos fazer o número I aparecer nesta história. Para uma partição P qualquer, temos

X X
∗∗ ∗∗
|S(f ; P ) − s(f ; P )| 6 S(f ; P ) − (f ; P ) + (f ; P ) − I +

X X
+ I − (f ; P ∗ ) + (f ; P ∗ ) − s(f ; P ) ,

onde P∗ eP ∗∗ são pontilhamentos de P. Agora, vamos tornar cada uma das parcelas acima
menor do que ε /4.
X
∗∗
Primeiro, escolhemos a partição P tal que ambas as parcelas do meio, isto é, (f ; P ) − I

X
e I − (f ; P ∗ ) , são menores do que ε /4, não importam quais sejam os pontilhamentos P ∗

∗∗
e P de P . Esta partição P existe pela hipótese desta recíproca. Com relação às primeira
e última parcelas, fazemos o seguinte. Se P = {t0 , . . . , tn }, para cada i ∈ {1, . . . , n}, se-
jam mi = inf{f (x); x ∈ [ti−1 , ti ]} e Mi = sup{f (x); x ∈ [ti−1 , ti ]}. Dado δ > 0, para cada
i ∈ {1, . . . , n}, escolhemos ξi e ηi em [ti−1 , ti ] tais que f (ξi ) − mi < δ e Mi − f (ηi ) < δ . Deste
∗ ∗∗
modo, sendo P o pontilhamento dos ξi 's e P o pontilhamento dos ηi 's, temos

X n
X
∗∗
S(f ; P ) − (f ; P ) = (Mi − f (ηi ))(ti − ti−1 ) < δ · (b − a)
i=1

e
X n
X

(f ; P ) − s(f ; P ) = (f (ξi ) − mi )(ti − ti−1 ) < δ · (b − a).
i=1
ε
Portanto, o resultado desejado segue-se para δ= . 
4(b − a)
374 CAPÍTULO 9. INTEGRAL DE RIEMANN

9.19 Exercício 19
Diz-se que f : R → R é localmente limitada x quando existe ε > 0 tal que f |(x−ε, x+ε)
no ponto
é limitada. Prove que o conjunto dos pontos x ∈ R onde f é localmente limitada é aberto.
Mostre que se pode denir a oscilação de f nos pontos onde esta função é localmente limitada.
Prove que o conjunto dos pontos onde uma função arbitrária f : R → R é contínua é uma
interseção enumerável de abertos. Usando o Teorema de Baire (Exercício 54, Capítulo V),
conclua que nenhuma função f : R → R pode ter Q como o conjunto dos seus pontos de
continuidade.

Solução. A parte nal deste exercício já foi vista no Exercício 18 do Capítulo VII (página
243). O que há de novidade aqui é que podemos provar novamente que o conjunto dos pontos
de continuidade de f é uma interseção enumerável de abertos usando a parte inicial deste
exercício e resultados deste capítulo. Em particular, adaptações do Teorema 16 e do Corolário
do Teorema 17. A conclusão sobre nenhuma função poder ter Q como seu conjunto de pontos
de continuidade é exatamente igual a daquele exercício; isto é consequência do Exercício 55 do
Capítulo V (página 195).
Seja X o conjunto dos números reais nos quais f é localmente limitada. Se x ∈ X , então
(x − ε, x + ε) ⊂ X , onde ε > 0 é tal que f |(x − ε, x + ε) é limitada. Isto porque se f |(x − ε, x + ε)
é limitada, então f |I é limitada qualquer que seja o conjunto I contido em (x − ε, x + ε). Em
particular f |(y−δ, y+δ) é limitada se y ∈ (x−ε, x+ε) e δ > 0 é tal que (y−δ, y+δ) ⊂ (x−ε, x+ε).
Portanto, f é localmente limitada em y qualquer que seja o y no intervalo (x − ε, x + ε). Logo,
X é aberto.
Se f é localmente limitada em x, então existe ε > 0 tal que f |[x − ε, x + ε] é limitada.
Então podemos denir a oscilação ω(x) de f em x exatamente do mesmo modo como no livro,
a saber, pela igualdade

ω(x) = lim ω(f |[x − ε, x + ε]; (x − δ, x + δ) ∩ [x − ε, x + ε]).


δ→0

f , sabemos que uma condição necessária para f ser contínua


Com relação à continuidade de
num ponto x é f ser localmente limitada em x (Teorema 3 do Capítulo VI). Neste sentido, com
pequenas alterações no Teorema 16 e no Corolário do Teorema 17, enunciamos:

Seja x0 ∈ X um ponto no qual f :R→R é localmente limitada. Para ele, temos

• ω(x0 ) = 0 ⇔ f é contínua em x0 , e;
• se ω(x0 ) < α, então existe δ > 0 tal que ω(x) < α para todo x tal que
|x − x0 | < δ .
O segundo dos itens acima faz sentido porque X é aberto. Ele também nos diz que o
conjunto dos pontos x∈X tais que ω(x) < α, para um dado α ∈ R, é aberto. Em particular,
é aberto o conjunto  
1
Cn = x ∈ X; ω(x) < .
n
Além disto, com relação ao primeiro item, notamos que ω(x) = 0 ⇔ ω(x) < 1/n para todo
n ∈ N. Podemos então concluir que f é contínua num número real x se, e somente se, x ∈ Cn
∀n ∈ N. Logo, o conjunto dos pontos nos quais f é contínua é o conjunto
\ \ 1

Cn = x ∈ X; ω(x) < ,
n∈N n∈N
n
uma interseção enumerável de abertos, como queríamos. 
9.20. EXERCÍCIO 20 375

9.20 Exercício 20
Seja f : [a, b] → R uma função limitada. Indiquemos com ω(x) a oscilação de f no ponto
x ∈ [a, b]. Prove que
Z b Z b Z b
f (x) dx − f (x) dx = ω(x) dx.
a a a

Solução. Temos
Z b Z b
f (x) dx − f (x) dx = inf S(f ; P ) − sup s(f ; P )
a P P
a

(Lema 3) = inf S(f ; P ) + inf − s(f ; P )
P P

(Lema 2) = inf S(f ; P ) − s(f ; Q) ,
P,Q

pois o conjunto das somas superiores e o conjunto das somas inferiores de f são ambos limitados
de acordo com o Corolário do Teorema 1 (além do que s(f ; {a, b}) é a menor soma inferior e
S(f ; {a, b}) é a maior soma superior). Neste ínmo, em vez de considerarmos as duas variáveis
P e Q, podemos considerar apenas uma. De modo mais preciso, vale a igualdade
 
inf S(f ; P ) − s(f ; Q) = inf S(f ; P ) − s(f ; P ) ,
P,Q P

pois (ainda com o Corolário do Teorema 1 em mente)



inf S(f ; X) − s(f ; X) 6 S(f ; P ∪ Q) − s(f ; P ∪ Q) 6 S(f ; P ) − s(f ; Q) ∀P ∀Q
X
 
⇒ inf S(f ; X) − s(f ; X) 6 inf S(f ; X) − s(f ; Y )
X X,Y

e, pelo Exercício 33 do Capítulo III (página 84; as vericações de suas hipóteses são imediatas),

S(f ; P ) − s(f ; P ) ∈ {S(f ; X) − s(f ; Y ); X e Y partições} ∀P


 
⇒ inf S(f ; X) − s(f ; X) > inf S(f ; X) − s(f ; Y ) .
X X,Y

(Voltar para a Solução do Exercício 32, item 1: página 390.)

A tese do enunciado transforma-se, então, na igualdade



inf S(f ; P ) − s(f ; P ) = inf S(ω; P ).
P P

Do Teorema 18 podemos extrair um resultado útil para este contexto. Dada uma partição
P = {t0 , . . . , tn } de [a, b], para cada i ∈ {1, . . . , n}, seja Mi = sup{ω(x); x ∈ [ti−1 , ti ]}. Dado
ε > 0, temos ω(x) < Mi + ε para todo x ∈ [ti−1 , ti ]. Então existe uma partição Pi de [ti−1 , ti ]
tal que
S(f |[ti−1 , ti ]; Pi ) − s(f |[ti−1 , ti ]; Pi ) < (Mi + ε) · (ti − ti−1 ).
Portanto, para a partição Q = P1 ∪ P2 ∪ · · · ∪ P n de [a, b], temos
n
X
S(f ; Q) − s(f ; Q) = S(f |[ti−1 , ti ]; Pi ) − s(f |[ti−1 , ti ]; Pi )
i=1
Xn
< Mi · (ti − ti−1 ) + ε ·(ti − ti−1 )
i=1
= S(ω; P ) + ε ·(b − a).
376 CAPÍTULO 9. INTEGRAL DE RIEMANN

Repetindo este raciocínio, começando agora com ω(x) < Mi + ε /(b − a), podemos concluir
o seguinte:

Para toda partição P de [a, b] e para todo ε > 0, existe uma partição Q de [a, b]
tal que

S(f ; Q) − s(f ; Q) < S(ω; P ) + ε.

Em outras palavras, chamando de X o conjunto de todos os números da forma S(f ; P ) −


s(f ; P ) e, de Y o conjunto dos números da forma S(ω; P ),

∀y ∈ Y ∀ ε > 0 ∃x ∈ X; x < y + ε .

Pelo Exercício 31 do Capítulo III (página 83), isto equivale à desigualdade

y > inf X ∀y ∈ Y.

De fato:

y > inf X ⇔ −y 6 − inf X


(Lema 3) ⇔ −y 6 sup(−X)
(Exercício 31 do Capítulo III) ⇔ ∀ ε > 0 ∃x ∈ X; −y − ε < −x
⇔ ∀ ε > 0 ∃x ∈ X; y + ε > x.

Logo, de y > inf X ∀y ∈ Y , obtemos

inf Y > inf X.

Por outro lado,


ω(x) 6 ω(f ; [ti−1 , ti ]) ∀x ∈ (ti−1 , ti ),
pois, se x ∈ (ti−1 , ti ), para δ>0 tal que (x − δ, x + δ) ⊂ [ti−1 , ti ], temos

ω(x) 6 ω(f ; (x − δ, x + δ)) 6 ω(f ; [ti−1 , ti ]),

pela observação da página 339. Então

sup{ω(x); x ∈ (ti−1 , ti )} 6 ω(f ; [ti−1 , ti ]) ∀i ∈ {1, 2, . . . , n}.

Assim, se não fossem pelos números da forma ω(ti ) (i ∈ {0, 1, · · · , n}), os quais não foram
contemplados no supremo acima, teríamos S(ω; P ) 6 S(f ; P )−s(f ; P ). No entanto, o Corolário
do Lema 7 nos dá a ideia de como controlar esta situação. De modo mais preciso, dado ε > 0,
0 00 0 00 0 00 00 0
sejam os pontos qi < ti < qi (∀i ∈ {1, . . . , n − 1}), q0 = t0 < q0 e qn < tn = qn , com qi−1 < qi
X n
∀i ∈ {0, 1, · · · , n}, tais que |qi00 − qi0 | < ε. Fixando L tal que L > ω(x) para todo x ∈ [a, b] e
i=0
denindo Q = {q00 , q000 , q10 , q100 , . . . , qn0 , qn00 }, temos

S(ω; Q) < L · ε +[S(f ; P ) − s(f ; P )].


n
X
Daí, de modo semelhante ao que zemos mais acima, requerendo |qi00 − qi0 | < ε /L,
i=0
podemos concluir:
9.21. EXERCÍCIO 21 377

Para toda partição P de [a, b] e para todo ε > 0, existe uma partição Q de [a, b]
tal que

S(ω; Q) < [S(f ; P ) − s(f ; P )] + ε,

ou, em outras palavras,


∀x ∈ X ∀ ε > 0 ∃y ∈ Y ; y < x + ε,
donde, novamente pelo Exercício 31 do Capítulo III, x > inf Y ∀x ∈ X , o que acarreta inf X >
inf Y .
Logo, de inf Y > inf X e de inf X > inf Y , segue inf X = inf Y , como queríamos. 

9.21 Exercício 21
Se um intervalo I tem medida nula, então I reduz-se a um ponto.

(Voltar para a Solução do Exercício 26: página 382.)

Solução. Vamos trabalhar com a contrapositiva deste enunciado. No Exercício 2, ao demons-


trarmos que seu item 2 implica seu item 3 (página 343), já vimos um modo de se fazer isto, a
saber, tomando [a, b] ⊂ I , com a < b, pelo Exemplo 12:

c(Q ∩[a, b]) 6= 0 ⇒ c([a, b]) 6= 0 ⇒ m([a, b]) 6= 0 ⇒ m(I) 6= 0.

Vejamos outra justicativa. Se I é um intervalo que não se reduz a um ponto, então I


contém um intervalo J = (a, b), onde a<b são números reais. Vamos mostrar que J não tem
medida nula, donde podemos concluir que I também não tem medida nula, pelo item (1) da
página 343 (todo subconjunto de um conjunto de medida nula tem medida nula).
Lembremos da Proposição 3 do Capítulo V. Se I1 = (a1 , b1 ), I2 = (a2 , b2 ), . . . , In = (an , bn ), . . .
X∞
é uma coleção enumerável de intervalos abertos e |In | < b − a, então
n=1

∞ ∞
!
[ [
J− In = [a, b] − In − {a, b}
n=1 n=1

é um conjunto não enumerável. Em particular,


[
J 6⊂ In .
n=1

Logo, J não tem medida nula, como queríamos. 

9.22 Exercício 22
Todo conjunto de medida nula tem interior vazio.

(Voltar para a Solução 1 do Exercício 41: página 401.)


378 CAPÍTULO 9. INTEGRAL DE RIEMANN

Solução. Isto já foi visto na demonstração da implicação 2⇒3 do Exercício 2 (página 343).
Mas também podemos cair no exercício anterior, que é o que veremos abaixo.
Se o interior de um conjunto X não é vazio, então X contém um intervalo aberto I; um
intervalo aberto não se reduz a um ponto, então, pelo exercício anterior, I não tem medida nula
e, portanto, X também não tem medida nula, pelo item (1) da página 343, o que demonstra a
contrapositiva deste exercício. 

9.23 Exercício 23
Dadas, f, g : [a, b] → R integráveis, seja X = {x ∈ [a, b]; f (x) 6= g(x)}. Se X tem medida nula
então Z b Z b
f (x) dx = g(x)dx.
a a

Solução. Nos remetemos ao exercício anterior e ao Exercício 2 (página 342).


Pelo exercício anterior, se m(X) = 0, então int(X) = ∅. Agora, para podermos aplicar o
Exercício 2, trabalhamos com a função f − g (integrável pelos itens 2 e 3 do Teorema 5) e
reescrevemos o conjunto X:

X = {x ∈ [a, b]; (f − g)(x) 6= 0}.

Deste modo, como int(X) = ∅, pelo Exercício 2,

Z b
|f (x) − g(x)| dx = 0.
a

Portanto Z b


f (x) − g(x) dx 60

a

(item 5 do Teorema 5), o que implica

Z b Z b Z b
0= f (x) − g(x) dx = f (x) dx − g(x) dx.
a a a

(novamente os itens 2 e 3 do Teorema 5). Logo,

Z b Z b
f (x) dx = g(x) dx,
a a

como queríamos. 

9.24 Exercício 24
Dê exemplo de duas funções limitadas f, g : [a, b] → R, tais que o conjunto X = {x ∈
[a, b]; f (x) 6= g(x)} tenha medida nula, f seja integrável e g não seja.

Solução. Ambas f ≡ 0 e g , denida por g(x) = 0 se x ∈ [a, b] − Q e g(x) = 1 se x ∈ [a, b] ∩ Q,


são limitadas; f é integrável, g não é integrável e X = [a, b] ∩ Q tem medida nula porque é
enumerável. 
9.25. EXERCÍCIO 25 379

9.25 Exercício 25
Se f : [a, b] → R é lipschitziana (em particular, se f ∈ C 1) e X ⊂ [a, b] tem medida nula, então
f (X) tem medida nula.

(Voltar para a Solução do Exercício 40: página 400.)

Solução 1.

No elevador. De modo ingênuo


"∞ # ∞
[ [ [
X⊂ Ii ⇒ f [X] ⊂ f Ii = f [Ii ].
i=1 i=1 i=1

Ingênuo porque f [Ii ] não está denido se Ii 6⊂ [a, b], um detalhe a ser arrumado. Se cada
Ii é um intervalo, então f [Ii ] também é um intervalo, porque f é contínua (Teorema do Valor
Intermediário). Não sabemos se f [Ii ] é aberto, mas podemos dilatá-lo um pouquinho fazendo
 
δ δ
f [Ii ] ⊂ αi − i+1 , βi + i+1 ,
2 2

onde αi = inf f [Ii ], βi = sup f [Ii ] e δ é um número real positivo a ser escolhido de modo
conveniente. A escolha da fração cujo denominador é uma potência de dois deve-se porque
2
conhecemos a soma 1/2 + 1/2 + · · · ; ela é igual a 1. Este número 1 é grande demais para os
nossos propósitos, por isso colocamos um fator δ. Por ser lipschitziana, armamos que existe
c>0 tal que βi − αi 6 c|Ii |. Portanto

∞   ∞
X
αi − δ δ X δ
i+i
, βi + i+1
= βi − α i + i
i=1
2 2 2
"i=1∞ #
X
= βi − αi + δ
i=1
" ∞
#
X
6 c· |Ii | + δ.
i=1

São estas considerações que nos permitem fazer a escolha adequada da cobertura do conjunto
f [X].

Lema. Dada uma função lipschitziana f : [a, b] → R, se I ⊂ [a, b] é não vazio, então existe
c>0 tal que

sup f [I] − inf f [I] 6 c · (sup I − inf I).


Em particular, se I é um intervalo, então

|f [I]| 6 c · |I|

(f [I] também é um intervalo  Corolário 2 do Teorema 12, do Valor Intermediário, Capítulo


VII , portanto |f [I]| está denido).
380 CAPÍTULO 9. INTEGRAL DE RIEMANN

Solução. Começamos notando que funções lipschitzianas são limitadas: f lipschitziana


implica f contínua e f contínua implica f limitada (Teorema de Weierstrass), uma vez que
[a, b] é compacto. Pela denição de oscilação num conjunto e pelo Corolário do Lema 5, temos

sup f [I] − inf f [I] = ω(f ; I) = sup{|f (x) − f (y)|; x, y ∈ I}.

Como f é lipschitziana, seja c > 0 tal que |f (x) − f (y)| 6 c|x − y| para todo x, y ∈ [a, b].
Se x, y ∈ I , então |x − y| 6 sup I − inf I , de modo que

|f (x) − f (y)| 6 c · (sup I − inf I) ∀x, y ∈ I

e, portanto,
sup{|f (x) − f (y)|; x, y ∈ I} 6 c · (sup I − inf I),
donde segue a tese.

Seja c>0 a testemunha de que f é lipschitziana (e também a testemunha do lema acima).


ε −δ
Dado ε > 0, xemos δ positivo e menor do que ε. Para ε0 = (que é positivo), como X
c
tem medida nula, sejam os intervalos abertos I1 , I2 , . . . , In , . . . tais que

[ ∞
X
X⊂ Ii e |Ii | < ε0 .
i=1 i=1

Vamos supor também Ii ∩ X 6= ∅ para todo i ∈ N, de modo que cada Ji = [a, b] ∩ Ii é


um subconjunto não vazio de [a, b]. Armamos que os intervalos abertos K1 , K2 , . . . , Kn , . . . da
forma  
δ δ
Ki = αi − i+1 , βi + i+1 ∀i ∈ N,
2 2
onde αi = inf f [Ji ] e βi = sup f [Ji ] para cada i ∈ N, são tais que


[ ∞
X
f [X] ⊂ Ki e |Ki | < ε .
i=1 i=1

De fato, de f [Ji ] ⊂ Ki para todo i ∈ N, temos

∞ ∞
"∞ #
[ [ [
Ki ⊃ f [Ji ] = f Ji ⊃ f [X],
i=1 i=1 i=1

pois

[ ∞
[ ∞
[
X = [a, b] ∩ X ⊂ [a, b] ∩ Ii = [a, b] ∩ Ii = Ji
i=1 i=1 i=1

(lembrando que X ⊂ [a, b]), e

δ δ
|Ki | = βi − αi + = sup f [J i ] − inf f [J i ] +
2i 2i
δ
6 c · (sup Ji − inf Ji ) + i
2
δ
6 c · (sup Ii − inf Ii ) + i
2
δ
= c · |Ii | + i
2
9.26. EXERCÍCIO 26 381

para todo i ∈ N (sup Ji − inf Ji 6 sup Ii − inf Ii porque Ji ⊂ Ii ) implica

∞ ∞
X X δ
|Ki | 6 c · |Ii | +
i=1 i=1
2i
"∞ #
X
= c· |Ii | + δ
i=1
< c · ε0 +δ
ε −δ
= c· +δ
c
= ε.

Logo, f [X]
tem medida nula. A justicativa da frase entre parênteses do enunciado deste
1 0 0
exercício é a seguinte: se f é de classe C , então f é contínua; f contínua num conjunto
0 0
compacto implica f limitada (Weierstrass, Corolário do Teorema 14 do Capítulo VII), e f
limitada implica f lipschitziana, pelo Exercício 24 do Capítulo VIII (página 303). 

Solução 2. Levando em conta o que foi dito na página 344 do livro, podemos cobrir f [X]
com intervalos fechados em vez de intervalos abertos, que é o que faremos nesta solução.
Seja c > 0 tal que |f (x) − f (y)| 6 c|x − y| para todo x, y ∈ I . Dado ε > 0, sejam os
intervalos abertos I1 , I2 , . . . , In , . . . tais que
∞ ∞
[ X ε
X⊂ Ii , |Ii | <
i=1 i=1
c

e Ii ∩ [a, b] 6= ∅, para cada i ∈ N.  


Assim, denindo Ji = Ii ∩ [a, b] e Ki = inf f [Ji ], sup f [Ji ] (pelo Teorema de Weierstrass,
cada f [Ji ] é limitado pois f é contínua num compacto) para cada i ∈ N, temos

[
f [X] ⊂ Ki
i∈N

com

|Ki | = sup f [Ji ] − inf f [Ji ] 6 c(sup Ji − inf Ji ) 6 c(sup Ii − inf Ii ) = c|Ii | ∀i ∈ N

(a primeira desigualdade vale pelo Lema da Solução 1 e a segunda, porque Ji ⊂ Ii ), portanto,


X ∞
X
|Ki | 6 c |Ii | < ε .
i=1 i=1

Logo, f [X] tem medida nula, como queríamos. 

9.26 Exercício 26
Seja K o conjunto de Cantor. Dê exemplo de uma função monótona contínua f : [0, 1] → [0, 1]
tal que f (K) não tenha medida nula. (Veja o Exercício 17, Capítulo VII).
382 CAPÍTULO 9. INTEGRAL DE RIEMANN

Solução. A função ϕ : [0, 1] → R construída no Exercício 17 do Capítulo VII (página 242)


possui todas as propriedade pedidas. Para ela, temos

[0, 1] = ϕ[([0, 1] − K) ∪ K] = ϕ[[0, 1] − K] ∪ ϕ[K],

onde ϕ[[0, 1] − K] tem medida nula porque é enumerável, o conjunto de todas as frações da
forma m/2n pertencentes ao intervalo [0,1], com m e n naturais. Portanto, ϕ[K] não tem
medida nula. Pois, se tivesse, então [0, 1] também teria (uma reunião nita de conjuntos de
medida nula tem medida nula), o que é um absurdo de acordo com o Exercício 21 (página 377),
um intervalo não degenerado não tem medida nula. 

9.27 Exercício 27
Verique que, a partir do item 4 do Teorema 4, todas as armações do texto, no sentido de que
uma certa função é integrável, podem ser obtidas como consequências do Teorema 20.

Solução.

Teorema 4; (4) ⇒ (1). A inspiração para a demonstração desta implicação vem das
demonstrações dos Teoremas 19 e 20; com pequenas alterações nestas podemos fazê-la usando
apenas resultados anteriores.
Seja f : [a, b] → R uma função limitada satisfazendo o item 4 do Teorema 4. Denotando
por D o conjunto dos pontos de descontinuidade de f, queremos mostrar que m(D) = 0.
Através de uma adaptação do Exercício 18 do Capítulo VII (página 243), temos
\ [
D ∩ (a, b) = (a, b) − Cn = (a, b) − Cn ,
n∈N n∈N

cada Cn sendo agora denido da seguinte forma: x0 ∈ Cn se, e somente se, x0 ∈ (a, b) e existe
um intervalo aberto I ⊂ (a, b) possuindo x0 tal que x, y ∈ I ⇒ |f (x) − f (y)| < 1/n. De modo
equivalente, x0 ∈ (a, b) − Cn se, e somente se, existem x, y ∈ I tais que |f (x) − f (y)| > 1/n
qualquer que seja o intervalo aberto I ⊂ (a, b) possuindo x0 . Consequentemente, no que
nos interessa, ω(f ; I) > 1/n se o intervalo aberto I ⊂ (a, b) possui algum ponto de (a, b) − Cn
(Corolário do Lema 5). Feita estas observações, dado m ∈ N, vamos mostrar que c((a, b)−Cm ) =
0. n
X ε
Fixado o ε > 0, seja a partição P = {t0 , t1 , . . . , tn } de [a, b] tal que ωi (ti − ti−1 ) < . Se
i=1
m
nenhum intervalo da forma (ti−1 , ti ) tem interseção com (a, b) − Cm , então (a, b) − Cm ⊂ P . Se
(ti−1 , ti ) possui algum ponto de (a, b) − Cm , então ωi > 1/m. Assim, denotando por (t0i−1 , t0i ) os
0
intervalos abertos da partição P que possuem algum ponto de (a, b) − Cm e por ωi a variação
de f neles, temos

n
1 X 0 0
X
0 0 0
X ε
· (ti − ti−1 ) 6 ωi (ti − ti−1 ) 6 ωi (ti − ti−1 ) < ,
m i=1
m

(t0i − t0i−1 ) < ε, onde os intervalos abertos da forma (t0i−1 , t0i ), que são em número
P
portanto
nito, cobrem (a, b) − Cm a não ser, possivelmente, por um número nito de elementos (os
pontos de P ). Logo, c((a, b) − Cm ) = 0 pela propriedade 4 dos conjuntos de conteúdo nulo,
como queríamos. 
9.27. EXERCÍCIO 27 383

Exemplo 5. f, g : [a, b] → R limitadas e suponhamos que X = {x ∈ [a, b]; f (x) 6=


Sejam
g(x)} seja nito. Supondo g integrável, queremos mostrar que f é integrável. Sejam D1 e D2
os respectivos conjuntos dos pontos de descontinuidade de f e de g . O conjunto dos pontos de
descontinuidade de f − g é X . Assim, escrevendo f = g + (f − g), pelo Teorema 5 do Capítulo
VII, vemos que se x ∈ D1 , então x ∈ D2 ou x ∈ X , portanto, D1 ⊂ D2 ∪ X , donde m(D1 ) = 0,
uma vez que m(D2 ∪ X) = 0. 

Teorema 5. f, g : [a, b] → R. Sejam D1 e D2 os respectivos pontos de


Sejam as funções
descontinuidade de f e de g . Para Y ⊂ [a, b] não vazio e c ∈ R, seja h ∈ {f |Y, c·f, f +g, |f |, f ·g};
h é limitada. Seja D3 o conjunto dos pontos de descontinuidade de h. Pelos Teoremas 1, 5 e 6
do Capítulo VII (pelas suas contrapositivas), podemos concluir que D3 ⊂ D1 ∪ D2 , lembrando
que a função módulo é contínua. Deste modo, se f e g são integráveis, então m(D3 ) = 0. 

Teorema 6. Se f : [a, b] → R é contínua, então o conjunto dos seus pontos de desconti-


nuidade é vazio e este tem medida nula. Além do que, f é limitada ([a, b] é compacto; Teorema
14 do Capítulo VII). Logo, f é integrável. 

Teorema 7. f : [a, b] → R limitada. Seja (cn )n∈N uma sequência de pontos em [a, b)
Seja
que converge para b. Sejam D e Dn os respectivos conjuntos dos pontos de descontinuidade de
f e de f |[a, cn ], para cada n ∈ N. Então D ⊂ {b} ∪ D1 ∪ D2 ∪ · · · Dn ∪ · · · : se f é descontínua
em x0 ∈ [a, b), então f |[a, cn ] também é descontínua em x0 , sendo n ∈ N tal que cn > x0 (o
qual existe por causa da convergência cn → b). Assim, se f |[a, cn ] é integrável para todo n ∈ N,
então m({b} ∪ D1 ∪ D2 ∪ · · · ∪ Dn ∪ · · · ) = 0, logo, m(D) = 0. 

Corolário 1 (do Teorema 7). f : [a, b] → R, agora consideramos


Dada a função limitada
as sequências (cn )n∈N e (dn )n∈N em(a, b). A primeira a e a segunda, para
convergente para
b, e tais que cn < d n
para todo n ∈ N. Assim como
no teorema acima, se D e, para cada
n ∈ N, Dn são os respectivos conjuntos dos pontos de descontinuidade de f e de f |[cn , dn ],
então D ⊂ {a, b} ∪ D1 ∪ D2 ∪ · · · ∪ Dn ∪ · · · . Portanto, se f |[cn , dn ] é integrável para todo n ∈ N,
então f também e integrável. 

Corolário 2 (do Teorema 7). Se o conjunto dos pontos nos quais uma função limitada
f : [a, b] → R não é contínua é nito, então f é integrável porque esse conjunto tem medida
nula. 

Exemplo 6. O conjunto no qual a função limitada f : [−1, 1] → R, denida porf (x) =


sen 1/x se x 6= 0 e f (0) = 0, é descontínua é um conjunto unitário, portanto de medida nula.

Exemplo 7. O conjunto Q ∩[a, b] é enumerável e portanto tem medida nula. Ele é o


conjunto dos pontos de descontinuidade da função limitada f : [a, b] → R denida por f (x) = 0
se x ∈ {0} ∪ [a, b] − Q e f (p/q) = 1/q se p/q ∈ [a, b] é uma fração irredutível com p 6= 0. 

X Seja f : [a, b] → R limitada e suponhamos a existência de um I ∈ R tal


Teorema 15.
que I = lim (f ; P ∗ ), qualquer que seja o pontilhamento P ∗ de P . Seguindo o roteiro da
|P |→0
demonstração do Teorema 15 (do reciprocamente em diante), chegamos a uma partição P tal
384 CAPÍTULO 9. INTEGRAL DE RIEMANN

n
X
que S(f ; P ) − s(f ; P ) < ε. Como S(f ; P ) − s(f ; P ) = ωi (ti − ti−1 ) (se P = {t0 , . . . , tn }),
i=1
isto signica que f satisfaz o item 4 do Teorema 4. A partir daí seguimos o que já zemos mais
acima, no início da solução deste exercício, para chegar ao resultado desejado: m(D) = 0. 

Teorema 19. f : [a, b] → R e D o conjunto de seus pontos de


Sejam a função limitada
descontinuidade. Supondo Eδ = {x ∈ [a, b]; ω(f ; x) > δ} um conjunto de conteúdo nulo para
cada δ > 0, queremos ver que m(D) = 0.
[
Do mesmo modo como na demonstração do Teorema 20, temos D ⊂ E1/n , donde segue a
n∈N
tese. Esta inclusão vale porque ω(f ; x0 ) = 0 ⇒ x0 ∈
/ D, e ω(f ; x) > 0 para todo x ∈ [a, b]. Estas
duas coisas seguem da denição de oscilação de uma função num ponto (vide demonstração do
Teorema 16). Portanto, x0 ∈ D ⇒ ω(f ; x0 ) > 0, isto é, se x0 ∈ D , então existe n∈N tal que
ω(f ; x0 ) > 1/n, ou seja, x0 ∈ E1/n . 

Corolário 1 do Teorema 20. f, g : [a, b] → R integráveis. Que f · g é integrável


Sejam
já foi visto acima, no item sobre o Teorema 5. Se f (x) 6= 0 para todo x ∈ [a, b] e 1/f é limitada,
então 1/f é integrável porque, neste caso, o conjunto de seus pontos de descontinuidade é o
mesmo de f , de acordo com o Teorema 5 do Capítulo VII. 

Corolário 2 do Teorema 20. Conjuntos enumeráveis têm medida nula. Seja f : [a, b] →
R uma função limitada. Se existem os limites laterais de f em cada ponto de [a, b] ou se f
é monótona, então suas descontinuidades (caso existam) são todas de primeira espécie (por
denição e pelo Teorema 9 do Capítulo VII, respectivamente). Então, de qualquer forma, o
conjunto dos pontos de descontinuidade de f é enumerável, conforme o Teorema 11 do Capítulo
VII. 

9.28 Exercício 28
Dadas f, g : [a, b] → R dena as funções f ∧ g e f ∨ g pondo (f ∧ g)(x) = min{f (x), g(x)} e
(f ∨ g)(x) = max{f (x), g(x)} para todo x ∈ [a, b]. Prove que se f e g são integráveis então
f ∧ g e f ∨ g são integráveis. Conclua que uma função é integrável se, e somente se, sua parte
positiva e sua parte negativa são integráveis. (Veja Capítulo IV, Ÿ7.)

Solução. Sejam D1 , D2 , D3 e D4 os respectivos conjuntos dos pontos de descontinuidade de


f, g, f ∧ g e f ∨ g. Temos
Di ⊂ D1 ∪ D2 para i ∈ {3, 4},
pelo Exercício 3 do Capítulo VII (página 232). Além disto, as limitações de f e de g implicam
as limitações de f ∧g e de f ∨ g. Logo, as integrabilidades de f ∧g e de f ∨ g seguem do
Teorema 20.
Com respeito à última parte deste exercício, sejamf : [a, b] → R uma função e g : [a, b] → R
a função identicamente nula, de sorte que a parte positiva de f é a função f ∨g e a parte negativa
de f é a função −(f ∧ g) (assim como foi feito para séries, estamos denindo a parte negativa de
uma função como uma função não negativa). Conforme acabamos de ver no parágrafo acima
(em conjunto com o item 2 do Teorema 5), se f é integrável, então f ∨ g e −(f ∧ g) são
integráveis. Reciprocamente, como f = (f ∨ g) − (−(f ∧ g)), se f ∨ g e −(f ∧ g) são integráveis,
9.29. EXERCÍCIO 29 385

então f é integrável pelos itens 2 e 3 do Teorema 5. Logo, f é integrável se, e somente se, suas
partes positiva e negativa o são. 

9.29 Exercício 29
Z b Z b
Seja g>0 integrável. Se g(x) dx = 0 então f (x)g(x) dx = 0, seja qual for f integrável.
a a

Solução. O item B do Teorema 12, com g desempenhando o papel da p, cairia bem aqui, mas
ele não pode ser aplicado porque não temos a continuidade de f como hipótese. No entanto
podemos imitar sua demonstração, que é o que faremos abaixo.
Da limitação de f, a qual é pressuposta por causa da integrabilidade de f, sejam m e M
números reais tais que
m 6 f (x) 6 M ∀x ∈ [a, b].
Multiplicando por g(x), como g(x) > 0 para todo x ∈ [a, b], vem

m · g(x) 6 f (x)g(x) 6 M · g(x) ∀x ∈ [a, b]

e Z b Z b Z b
m· g(x) dx 6 f (x)g(x) dx 6 M · g(x) dx,
a a a
do item 4 do Teorema 5, ou seja,

Z b
06 f (x)g(x) dx 6 0.
a

Logo,
Z b
f (x)g(x) dx = 0,
a
como queríamos. 

9.30 Exercício 30
n
X
Dada f : [a, b] → R, para cada partição P = {t0 , . . . , tn } de [a, b], seja V (f ; P ) = |f (ti ) −
i=1
f (ti−1 )|. Quando o conjunto {V (f ; P ); P = partição de [a, b]} for limitado, diz-se que f é uma
função de variação limitada b b
e escreve-se Va (f ) = sup V (f ; P ). O número Va (f ) chama-se a
P
variação total b
de f no intervalo [a, b]. O signicado intuitivo de Va (f ) é o comprimento total
do caminho percorrido por f (t) quando t cresce de a até b. Prove:

1. Toda função lipschitziana é de variação limitada.

2. Toda função monótona é de variação limitada.

3. Se f e g são de variação limitada, o mesmo ocorre com f + g, f · g e |f |;

4. Seja f contínua de variação limitada. Para todo ε > 0, existe δ>0 tal que |P | < δ ⇒
|V (f ; P ) − Vab (f )| < ε;
386 CAPÍTULO 9. INTEGRAL DE RIEMANN

Z b
5. Se f possui derivada contínua, então f b
é de variação limitada e Va (f ) = |f 0 (t)| dt.
a

(Voltar para a Solução do Exercício 32, item 4: página 393.)

Solução.

item 1. c > 0 tal que |f (x) − f (y)| 6 c|x − y|


Seja para todo x, y ∈ [a, b]. Dada uma
partição P = {t0 , t1 , . . . , tn } de [a, b], temos
n
X n
X n
X
V (f ; P ) = |f (ti ) − f (ti−1 )| 6 c|ti − ti−1 | = c (ti − ti−1 ) = c(b − a).
i=1 i=1 i=1

Logo, 0 6 V (f ; P ) 6 c(b − a) para toda partição P de [a, b]. 

item 2. Se f é não decrescente, então

n
X
V (f ; P ) = |f (ti ) − f (ti−1 )|
i=1
n
X 
= f (ti ) − f (ti−1 )
i=1
   
= f (t1 ) − f (t0 ) + f (t2 ) − f (t1 ) + · · · + f (tn−1 ) − f (tn−2 ) + f (tn ) − f (tn−1 )
= f (tn ) − f (t0 )
= f (b) − f (a).

Se f é não crescente, então, observando que −f é não decrescente (o que nos permite
aproveitar a mesma conta acima),

n
X n
X 
V (f ; P ) = |f (ti ) − f (ti−1 )| = − f (ti ) − f (ti−1 ) = f (a) − f (b).
i=1 i=1

Logo, se f é monótona, então 0 6 V (f ; P ) 6 |f (b) − f (a)| qualquer que seja a partição P .

item 3. Para a soma, de

|(f + g)(x) − (f + g)(y)| = |f (x) − f (y) + g(x) − g(y)| 6 |f (x) − f (y)| + |g(x) − g(y)|

para todos x e y pertencentes a [a, b], vem

V (f + g; P ) 6 V (f ; P ) + V (g; P ).

Para o produto, escrevendo

|(f g)(x) − (f g)(y)| = |f (x)g(x) − f (x)g(y) + f (x)g(y) − f (y)g(y)|


6 |f (x)g(x) − f (x)g(y)| + |f (x)g(y) − f (y)g(y)|
= |f (x)| · |g(x) − g(y)| + |g(y)| · |f (x) − f (y)|,
9.30. EXERCÍCIO 30 387

percebemos a conveniência de vericar se funções de variação limitada são limitadas. Vejamos.


Seja K tal que V (f ; P ) 6 K para toda partição P de [a, b]. Dado x ∈ [a, b], em particular,
para a partição P = {a, x, b}, temos

|f (x) − f (a)| + |f (b) − f (x)| 6 K,

donde

2|f (x)| 6 |f (x) − f (a)| + |f (x) − f (b)| + |f (a) + f (b)| 6 K + |f (a) + f (b)|

e, portanto, de fato, f e g são funções limitadas. Assim, sendo A e B números tais que
|f (x)| 6 A e |g(x)| 6 B para todo x ∈ [a, b], podemos concluir que

V (f · g; P ) 6 A · V (g; P ) + B · V (f ; P ),

qualquer que seja a partição P.


Para o módulo, como ||f (x)| − |f (y)|| 6 |f (x) − f (y)| para todos x e y em [a, b] (item (iii)
do Teorema 2 do Capítulo III),

V (|f |; P ) 6 V (f ; P ) para toda partição P.

Logo, se f e g são funções de variação limitada, então f + g, f · g e |f | também são todas


funções de variação limitada, como queríamos. 
(Voltar para a Solução do Exercício 32, item 5: página 393.)

item 4. Este item é semelhante ao Teorema 14. Assim como em sua demonstração,
ε
xemos a partição P0 = {t0 , t1 , . . . , tn } de [a, b] V (f ; P0 ) > Vab (f ) −
, ou seja, tal que
tal que
2
ε ε
V (f ; P0 )− > Vab (f )−ε, e vamos obter δ > 0 tal que |P | < δ implique V (f ; P ) > V (f ; P0 )− ,
2 2
de tal modo a termos

ε
|P | < δ ⇒ V (f ; P ) > V (f ; P0 ) − > Vab (f ) − ε,
2
isto é, se |P | < δ , então

|Vab (f ) − V (f ; P )| = Vab (f ) − V (f ; P ) < ε,

como desejado.
Pela continuidade de f , dado ε0 > 0, para cada i ∈ {1, 2, . . . , n − 1}, existe δi > 0 tal que
|f (x) − f (ti )| < ε0 |x − ti | < δi (pela continuidade uniforme de f , decorrente do Teorema 17
se
do Capítulo VII, podemos supor estes δi 's todos iguais entre si, mas isto não será necessário).
Para usar este fato vamos impor que a norma de uma dada partição P seja tão pequena a ponto
de fazer com que exista, para cada i, um p ∈ P tal que |p − ti | < δi . Façamos isto denindo
δ = min{δ1 , δ2 , . . . , δn−1 }, pois

(ti − δi , ti + δi ) ∩ P = ∅ ⇒ |P | > 2δi > δi > δ.

Para o δ denido acima, seja P uma partição de [a, b] com norma menor do que δ e, para
cada i ∈ {1, . . . , n − 1}, seja pi ∈ P tal que |ti − pi | < δi . Para não correr o risco de tomar
um mesmo ponto para dois i's distintos, vamos supor os δi 's tais que os intervalos da forma
(ti − δi , ti + δi ) sejam dois a dois disjuntos. Denindo também p0 = a e pn = b, observamos
agora que
388 CAPÍTULO 9. INTEGRAL DE RIEMANN

(i) |f (ti ) − f (ti−1 )| 6 |f (ti ) − f (pi )| + |f (ti−1 ) − f (pi−1 )| + V (f |[pi−1 , pi ]; P ∩ [pi−1 , pi ]), para
cada i ∈ {1, . . . , n}, e;

n
X
(ii) V (f ; P ) = V (f |[pi−1 , pi ]; P ∩ [pi−1 , pi ]),
i=1

portanto (somando as n desigualdades do item (i), usando (ii) e o fato de quê |f (ti )−f (pi )| < ε0 ),

V (f ; P0 ) 6 V (f ; P ) + 2(n − 1) ε0 ,

ou seja,
V (f ; P ) > V (f ; P0 ) − 2(n − 1) ε0 .
ε0 .
Este raciocínio pode ser feito para qualquer Em particular, lembrando do nosso objetivo,
ε
para ε0 tal que 2(n−1) ε0 = ε /2, isto é, para ε0 = (onde n+1 é o número de elementos
4(n − 1)
de P0 , o qual, por sua vez, está atrelado a ε; se n = 1, este ε0 não faz sentido, mas, neste caso,
temos V (f ; P ) > V (f ; P0 ) > V (f ; P0 ) − ε /2 se |P | < |P0 |; outra opção para contornar o
problema com o n = 1 é mostrar que renamentos nunca fazem o número V (f ; P ) diminuir, o
que nos permite supor n > 1), temos

ε
V (f ; P ) > V (f ; P0 ) − 2(n − 1) ε0 = V (f ; P0 ) − ,
2
como queríamos. 

item 5. As implicações

f0 contínua ⇒ f0 limitada (Weierstrass, Corolário do Teorema 14 do Capítulo VII), e;

f0 limitada ⇒f de Lipschitz (Exercício 24 do Capítulo VIII, página 303)

já são conhecidas. Então, f é de variação limitada por ser lipschitziana (item 1 acima). Se
f é derivável, então f é contínua e podemos aplicar o Teorema do Valor Médio (Teorema 7
do Capítulo VIII). Dada a partição P = {t0 , t1 , . . . , tn }, para cada cada i ∈ [1, n] ∩ N, seja
ξi ∈ (ti−1 , ti ) tal que f (ti ) − f (ti−1 ) = f 0 (ξi )(ti − ti−1 ). Assim, sendo P ∗ o pontilhamento desses
ξi 's, temos X
V (f ; P ) = (|f 0 |; P ∗ ).
Z b
Seja I = |f 0 (t)| dt. Por um lado, dado ε > 0, existe δ1 > 0 tal que
a
X
0 ∗
|P | < δ1 ⇒ ε > |V (f ; P ) − Vab (f )| b
= (|f |; P ) − Va (f ) ,

existe segundo o item anterior. Por outro, pelo Teorema 15, existe δ2 > 0 tal que
X
|P | < δ2 ⇒ (|f 0 |; P ∗ ) − I < ε .

Daí, com a mesma justicativa do porquê o limite de uma sequência é único (Teorema 1 do
Capítulo IV, por exemplo), podemos concluir a tese:

Z b
|f 0 (t)| dt = I = Vab (f ).
a
9.31. EXERCÍCIO 31 389

b
Um outro argumento para a igualdade Vab (f ) = I é o de que o número Va (f ) − I pode se
tornar tão pequeno quanto se queira:
X X
0 ∗ 0 ∗
|Vab (f )
b
06 − I| 6 Va (f ) − (|f |; P ) + (|f |; P ) − I < ε,

X
para um ε>0 dado, onde (|f 0 |; P ∗ ) é um número que faz ambas as parcelas acima carem

menores do que ε /2. 

9.31 Exercício 31
1
Seja f : [0, 1] → R denida por f (x) = x · sen se x 6= 0 e f (0) = 0. Mostre que f embora
x
sendo contínua, não possui variação limitada. [ Sugestão: use o fato de que as reduzidas da série
X1
não formam um conjunto limitado.]
n
π −1 2
Solução. Para a sequência (xn )n∈N denida por xn = + nπ = para todo
2 (2n + 1)π
n ∈ N, temos

xn se n é par, e;
f (xn ) =
−xn se n é ímpar.

De qualquer forma,

2 2 8 n+1
|f (xn+1 ) − f (xn )| = |xn+1 + xn | = + = · ∀n ∈ N .
(2n + 3)π (2n + 1)π π (2n + 3)(2n + 1)
n+1 1
Não temos > para todo n ∈ N, porém podemos corrigir isto multi-
(2n + 3)(2n + 1) n+1
plicando o denominador da fração da direita por 4:

n+1 1
> ⇔ 4n2 + 8n + 4 > 4n2 + 8n + 3 ⇔ 4 > 3.
(2n + 3)(2n + 1) 4(n + 1)

Portanto, de acordo com o Critério de Comparação (Corolário do Teorema 16 do Ca-


pítulo IV), pela divergência da série harmônica (Exemplo 23 do Capítulo IV), a sequên-
n
!
X
cia |f (xi+1 ) − f (xi )| não é limitada superiormente. Assim, denindo a partição
i=1 n∈N
Pn = {0, xn+1 , xn . . . , x2 , x1 , 1} para cada n ∈ N, o número

n
X
V (f ; Pn ) = |f (x1 ) − f (1)| + |f (0) − f (xn )| + |f (xi+1 ) − f (xi )|
i=1

pode se tornar tão grande quanto se queira, logo, f não é de variação limitada. 

9.32 Exercício 32
Sejaf : [a, b] → R. Para cada partição P de [a, b], indiquemos com (t0i−1 , t0i ) os intervalos de
P nos quais f (t0i−1 ) 6 f (t0i ) com [sic ] (t00i−1 , t00i ) os intervalos onde f (t00i−1 ) > f (t00i ). Escrevamos
390 CAPÍTULO 9. INTEGRAL DE RIEMANN

V + (f ; P ) = [f (t0i ) − f (t0i−1 )] V − (f ; P ) = [f (t00i−1 ) − f (t00i )], Vab (f )+ = sup V + (f ; P )


P P
e e
P
Vab (f )− = sup V − (f ; P ). Estes números chamam-se a variação positiva e variação negativa de
P
f em [a, b]. Prove

1. Vab (f ) = Vab (f )+ + Vab (f )− ;

2. As funções ϕ, ξ : [a, b] → R, denidas por ϕ(x) = Vax (f )+ e ξ(x) = Vax (f )− são não-
decrescentes;

3. f (x) − f (a) = Vax (f )+ − Vax (f )− ;

4. Uma função é de variação limitada se, e somente se, é diferença entre duas funções mo-
nótonas;

5. Toda função de variação limitada é integrável.

Solução.

item 1. Já que a notação Vab (f ) está sendo usada, subentendemos f suposta de vari-
ação limitada, uma vez que este símbolo foi denido só para este caso. Se f é de varia-
+
ção limitada, é imediato que ambos os conjuntos {V (f ; P ) ∈ R; P é partição de [a, b]} e

{V (f ; P ) ∈ R; P é partição de [a, b]} são limitados, pois

Vab (f ) > V (f ; P ) > V + (f ; P ) ∀P

e
Vab (f ) > V (f ; P ) > V − (f ; P ) ∀P.
Além disto, para que as outras notações façam sentido, esses conjuntos devem ser não vazios.
Para serem não vazios, e até para que os somandos dos somatórios não se degenerem, vamos
+
completar a denição do enunciado colocando V (f ; P ) = 0 caso não existam intervalos da
0 0 − 00 00
forma (ti−1 , ti ) e V (f ; P ) = 0 caso não existam intervalos da forma (ti−1 , ti ).
A solução deste item é semelhante ao que já vimos no primeiro parágrafo da Solução do
Exercício 20 (página 375). De

V (f ; P ) = V + (f ; P ) + V − (f ; P )
∈ {V + (f ; X) + V − (f ; Y ); X e Y são partições de [a, b]},

qualquer que seja a partição P de [a, b], temos, primeiro pelo Exercício 33 do Capítulo III
(página 84) e, em seguida, pelo Lema 2,

Vab (f ) 6 sup(V + (f ; X) + V − (f ; Y ))
X,Y

= sup V + (f ; X) + sup V − (f ; Y )
X Y

= Vab (f )+ + b
Va (f ) .

Renamentos em P não diminuem o número V + (f ; P ), nem o número V − (f ; P ), isto é, se


P e Q são partições de [a, b] e se P ⊂ Q, então

V + (f ; P ) 6 V + (f ; Q) e V − (f ; P ) 6 V − (f ; Q).
9.32. EXERCÍCIO 32 391

A justicativa disto está mais abaixo. Assim, dadas partições P e Q, temos

V + (f ; P ) + V − (f ; Q) 6 V + (f ; P ∪ Q) + V − (f ; P ∪ Q)
= V (f ; P ∪ Q)
6 Vab (f ),

donde

Vab (f ) > sup(V + (f ; X) + V − (f ; Y )) = Vab (f )+ + Vab (f )− .


X,Y

De Vab (f ) 6 Vab (f )+ + Vab (f )− e de Vab (f ) > Vab (f )+ + Vab (f )− segue o resultado desejado:

Vab (f ) = Vab (f )+ + Vab (f )− .


+
Para vericar a armação feita acima a respeito das monotonicidades dos números V (f ; P )

e V (f ; P ), seguimos o mesmo espírito de seu correspondente no livro, a soma inferior s(f ; P ),
acrescentando um ponto na partição P. Em seguida continuamos analisando todos os casos
possíveis.
Dada a partição P = {t0 , t1 , . . . , tn } de [a, b], xado i ∈ {1, . . . , n}, seja Q = P ∪ {r}, com
ti−1 < r < ti . Se f (ti ) > f (ti−1 ), as possibilidades são

• f (ti−1 ) 6 f (r) 6 f (ti ), e nada muda com V + (f ; P ), pois

[f (ti ) − f (r)] + [f (r) − f (ti−1 )] = f (ti ) − f (ti−1 ),

e nem com V − (f ; P );

• f (ti−1 ) 6 f (ti ) < f (r), a parcela f (ti ) − f (ti−1 ) de V + (f ; P ) é trocada pela parcela

f (r) − f (ti−1 ), que é estritamente maior do que f (ti ) − f (ti−1 ), e, em V (f ; P ) surge a
parcela f (r) − f (ti ), isto é,

V + (f ; Q) = [V + (f ; P ) − (f (ti ) − f (ti−1 ))] + f (r) − f (ti−1 )


= V + (f ; P ) + (f (r) − f (ti ))
> V + (f ; P )

V − (f ; Q) = V − (f ; P ) + [f (r) − f (ti )] > V − (f ; P );

• f (r) < f (ti−1 ) 6 f (ti ),

V + (f ; Q) = [V + (f ; P ) − (f (ti ) − f (ti−1 ))] + f (ti ) − f (r)


= V + (f ; P ) + (f (ti−1 ) − f (r))
> V + (f ; P )

V − (f ; Q) = V − (f ; P ) + [f (ti−1 ) − f (r)] > V − (f ; P ).

De qualquer forma, V + (f ; Q) > V + (f ; P ) e V − (f ; Q) > V − (f ; P ). Se f (ti ) < f (ti−1 ) as


análises são análogas. Seguem as teses. 
392 CAPÍTULO 9. INTEGRAL DE RIEMANN

item 2. Vamos continuar supondo f uma função tal que ambos os conjuntos dos números
das formas V + (f ; P ) e V − (f ; P ) são limitados (devido a abertura que este exercício dá, a outra
x +
opção seria demonstrar, por exemplo, que se faz sentido falar em Va (f ) para todo x ∈ [a, b],
b +
então faz sentido falar em Va (f ) ). Além disto, está implícito que, para cada x, a f das
denições de ϕ e de ξ é, na realidade, a restrição f |[a, x]. Nas contas abaixo, denotemos por fx
esta restrição. Dados x < y , vamos mostrar que ϕ(y) > ϕ(x) e ξ(y) > ξ(x).
Dada uma partição P de [a, x], podemos denir a partição Py = P ∪ {y} de [a, y]. Se
f (y) < f (x), então
V + (fy ; Py ) = V + (fx ; P )
e
V − (fy ; Py ) = V − (fx ; P ) + (f (x) − f (y)) > V − (fx ; P ).
Se f (y) > f (x), então

V + (fy ; Py ) = V + (fx ; P ) + (f (y) − f (x)) > V + (fx ; P )


e
V − (fy ; Py ) = V − (fx ; P ).
Em qualquer um destes dois casos, lembrando que ϕ(y) e ξ(y) são supremos e que as contas
acima não dependem de uma escolha particular da partição P de [a, x], temos

ϕ(y) > V + (fy ; Py ) > V + (fx ; P ) e ξ(y) > V − (fy ; Py ) > V − (fx ; P ) ∀P,
+
ou seja, é uma cota superior para o conjunto {V (fx ; P ); P é partição de [a, x]} e ξ(y)
ϕ(y)

é uma cota superior para o conjunto {V (fx ; P ); P é partição de [a, x]}. Como, por denição
(de supremo), ϕ(x) e ξ(x) são, respectivamente, as menores dessas cotas superiores, segue
ϕ(y) > ϕ(x) e ξ(y) > ξ(x), como queríamos. 

item 3. Queremos que os números f (x) − f (a) + Vax (f )− e f (a) − f (x) + Vax (f )+ sejam
+ −
os respectivos supremos dos conjuntos dos V (f ; P )'s e dos V (f ; P )'s. Então, em primeiro
lugar, devemos ter
f (x) − f (a) + Vax (f )− > V + (f ; P ) ∀P
e
f (a) − f (x) + Vax (f )+ > V − (f ; P ) ∀P,
donde vem
Vax (f )− > V + (f ; P ) − f (x) + f (a) ∀P
e
Vax (f )+ > V − (f ; P ) + f (x) − f (a) ∀P.
Para termos estas duas desigualdades, como Vax (f )− e Vax (f )+ também são supremos, dada
a partição P, basta que existam partições Q1 e Q2 tais que
V − (f ; Q1 ) > V + (f ; P ) − f (x) + f (a)
e
V + (f ; Q2 ) > V − (f ; P ) + f (x) − f (a).
Bem entendido, em todo este item, f refere-se à restrição f |[a, x] e todos essas partições são
do intervalo [a, x]. Em outras palavras, dada P, queremos Q1 e Q2 tais que
V + (f ; Q2 ) − V − (f ; P ) > f (x) − f (a) > V + (f ; P ) − V − (f ; Q1 ).
9.33. EXERCÍCIO 33 393

Estas duas desigualdades implicam a tese. Ambos os conjuntos Q1 e Q2 podem ser tomados
como o próprio P. Para ver isto, é suciente observar que

V + (f ; P ) − V − (f ; P ) = f (x) − f (a). 

item 4. Se f é de variação limitada, então, pelos dois itens anteriores, f (x) = (f (a) +
ϕ(x)) − ξ(x) para todo x ∈ [a, b], onde f (a) + ϕ e ξ são função monótonas. Se f é uma diferença
entre duas funções monótonas, então ela é de variação limitada como consequência dos itens
2 e 3 do Exercício 30 (página 385): f é uma soma de duas funções monótonas (pois se g é
monótona, então −g também é) e funções monótonas são de variação limitada. 

item 5. Por um lado uma função de variação limitada é limitada, conforme já observamos
na solução do item 3 do Exercício 30 (página 386). Por outro, pelo item anterior, uma função
de variação limitada também é uma diferença de funções monótonas. Funções monótonas
(denidas em intervalos compactos, como é o caso, uma vez que o domínio de uma função de
variação limitada é um intervalo desta forma) são integráveis (Corolário 2 do Teorema 20) e
uma diferença de funções integráveis é integrável (itens 2 e 3 do Teorema 5). Logo, toda função
de variação limitada é integrável, como queríamos. 

9.33 Exercício 33
Seja f : R+ → R contínua, tal que f (x · y) = f (x) + f (y) para x, y ∈ R+ quaisquer. Prove que
existe c ∈ R tal que f (x) = c · log x.

Solução. Assim como no Corolário 1 do Teorema 21, repetindo aquelas mesmas contas, a
propriedade f (x · y) = f (x) + f (y) ∀x, y ∈ R+ implica f (xr ) = r · f (x) ∀x ∈ R+ ∀r ∈ Q
(f (1) = 0 pois f (1) = f (1 · 1) = f (1) + f (1)). Por outro lado, se existe c ∈ R tal que
f (x) = c · log x, então este c deve ser o número f (e), pois log e = 1. Podemos suspeitar então
que há algo a mais a ser explorado neste número e. Indo nesta direção, de acordo com as contas
que acabamos de fazer, em particular temos

f (er ) = r · f (e) = r · f (e) · log e = f (e) · log er ∀r ∈ Q .

Isto, com a continuidade da composta f ◦ exp (ambas f e exp são contínuas; Teorema 6 do
x
Capítulo VII e Teorema 22) e com a igualdade x = log e , nos dá

f (ex ) = x · f (e) = f (e) · log ex ∀x ∈ R .

A conclusão agora vem da sobrejetividade da função exp : R → R+ , ou seja, do Teorema


+ y
22. Dado x ∈ R , para y ∈ R tal que e = x, temos

f (x) = f (ey ) = f (e) · y = f (e) · log ey = f (e) · log x.

Logo

f (x) = c · log x ∀x ∈ R,
para c = f (e). 
394 CAPÍTULO 9. INTEGRAL DE RIEMANN

9.34 Exercício 34
 a x
Mostre que lim 1 + = ea .
x→∞ x
(Voltar para a Solução do Exercício 38: página 399.)

Solução 1. Podemos imitar a demonstração do caso a = 1 feita no livro. Assim como lá,
com a mudança de variável y = 1/x em mente, vamos trabalhar com a função denida por
g(x) = (1 + ax)1/x , para x diferente de 0 e próximo o suciente dele (de modo a fazer o número
1 + ax ser positivo). Vamos começar mostrando que o limite de g(x) quando x tende a 0 é ea .
Temos

(1 + ax)1/x = exp(log(1 + ax)1/x ),

onde
log(1 + ax) log(1 + ax)
log(1 + ax)1/x = =a· ,
x ax
log(1 + ax)
cujo limite quando x tende a 0 é a, pois lim =1 do fato de a derivada de log em
x→0 ax
x=1 ser igual a 1. Portanto,

lim (1 + ax)1/x = exp(a) = ea ,


x→0

pelo Teorema 9 do Capítulo VI, uma vez que lim exp(x) = exp(a).
x→a
A partir disto concluímos o resultado desejado, ou seja, a igualdade

 
1
lim g = ea ,
x→∞ x

pois, dado ε > 0, seja δ>0 tal que

0 < |x| < δ ⇒ |g(x) − ea | < ε;

para A∈R tal que



1
x > A ⇒ < δ,
x

o qual existe uma vez que lim 1/x = 0, temos


x→∞

 
1 1
a
x > A ⇒ 0 < < δ ⇒ g
− e < ε,
x x

logo,
 
1
lim g = ea ,
x→∞ x
como queríamos. 
9.34. EXERCÍCIO 34 395

Solução 2. Fazendo y = ax, temos

 a x
lim 1 + = lim (1 + ax)1/x
x→∞ x x→0

= lim [(1 + y)1/y ]a


y→0
a
= e .

A última das igualdades acima vale pelo Teorema 9 do Capítulo VI. A penúltima, pelo
 a x
Exercício 18 do Capítulo VI (caso a 6= 0, página 221; se a = 0, então lim 1+ = lim 1x =
  x→∞ x x→∞
1
1 = e0 ). E, a primeira, porque se lim f (x) = L, então lim f = L. 
x→0 x→∞ x

Solução 3. A mudança de variável x = ay nos dá

 y a
 a x 1
1+ = 1+ .
x y

Se a > 0, então lim y = +∞, de modo que


x→∞
 y a
 a x 1
lim 1 + = lim 1+ = ea
x→∞ x y→∞ y

(para a última igualdade: se lim f (x) = b e lim g(y) = g(b), então lim g(f (x)) = g(b), situação
x→∞ y→b x→∞
parecida com a do Teorema 9 do Capítulo VI; para a penúltima igualdade: se lim f (x) = +∞
x→∞
e lim g(y) = c, então lim g(f (x)) = c).
y→∞ x→∞
Para concluir o mesmo quando a < 0, precisamos também ter

 x
1
lim 1+ = e,
x→−∞ x

pois lim y = −∞ se a < 0. De fato é o que ocorre: dado ε > 0, seja δ>0 tal que
x→∞

0 < |x| < δ ⇒ (1 + x)1/x − e < ε .

como lim 1/x = 0, seja A∈R tal que


x→−∞

1
x < A ⇒ < δ,
x

então  x
1 1
x < A ⇒ 0 < < δ ⇒ 1 + − e < ε,
x x
logo,
 x
1
lim 1 + = e.
x→−∞ x
Para a=0 é imediato: lim (1 + 0/x)x = lim 1x = 1 = e0 . Segue a tese. 
x→∞ x→+∞
396 CAPÍTULO 9. INTEGRAL DE RIEMANN

9.35 Exercício 35
Prove que lim xx = 1, lim xlog(x+1) = 1 e lim [log x · log(x + 1)] = 0.
x→0 x→0 x→0

Solução. Com relação ao primeiro limite, temos (para x positivo)

xx = exp(log(xx )) = exp(x log x).

Como lim x log x = 0 (Corolário do Teorema 23) e lim exp(x) = exp(0) = 1,


x→0 x→0

lim exp(x log x) = 1


x→0

(Teorema 9 do Capítulo VI). Portanto,

lim xx = 1.
x→0

Com relação ao segundo limite, podemos cair no primeiro multiplicando e dividindo o ex-
poente da respectiva função por x, obtendo

log(x+1)
xlog(x+1) = xx· x
log(x+1)
= (xx ) x
 
log(x + 1) x
= exp · log(x ) ,
x

log(x + 1)
onde lim = log0 1 = 1 e lim log(xx ) = 0, pelo Teorema 9 do Capítulo VI, uma vez
x→0 x x→0
que lim xx = 1 e lim log x = log 1. Deste modo
x→0 x→1
 
log(x + 1) x
lim · log(x ) = 0
x→0 x
e
lim xlog(x+1) = exp(0) = 1,
x→0

novamente pelo Teorema 9 do Capítulo VI.


Este segundo limite também pode ser usado para se calcular o terceiro. Podemos perceber
isto escrevendo
log x · log(x + 1) = log xlog(x+1) .
Mais uma aplicação do Teorema 9 do Capítulo VI nos dá

lim [log x · log(x + 1)] = lim log xlog(x+1) = log 1 = 0,


x→0 x→0

como queríamos. 

Observação. Se o domínio da funçãof : X → R, denida por f (x) = xx ∀x ∈ X , é


X = {−1, −1/3, −1/5, −1/7, . . .} ∪ {x ∈ R; x > 0}, então
√ o limite de f (x) quando x tende
a zero não existe: para n ∈ N ímpar, f (−1/n) = − n, que tende a -1 conforme n cresce
n

(Exemplo 14 do Capítulo IV).


9.36. EXERCÍCIO 36 397

9.36 Exercício 36
Seja f : R → R contínua tal que f (x + y) = f (x) · f (y). Prove que f (x) = ax (a ∈ R+ ) ou
f (x) = 0 para todo x ∈ R.

Solução. Vamos começar calculando no zero:

f (0) = f (0 + 0) = f (0) · f (0) ⇒ f (0) · f (0) − f (0) = f (0) · (f (0) − 1) = 0.

Portanto f (0) = 0 ou f (0) = 1.


Se f (0) = 0, então

f (x) = f (x + 0) = f (x) · f (0) = 0 ∀x ∈ R .

Vamos supor agora f (0) = 1. Temos

f (2) = f (1 + 1) = f (1) · f (1) = [f (1)]2 , f (3) = f (1 + 2) = f (1) · f (2) = [f (1)]3 , . . .

e, por indução, podemos concluir que

f (n) = [f (1)]n ∀n ∈ N .

Também, para n ∈ N:

1 = f (0) = f (n − n) = f (n) · f (−n) ⇒ f (−n) = [f (n)]−1 .

Portanto,
f (n) = [f (1)]n ∀n ∈ Z .
m
Dados m ∈ Z e n ∈ N, vamos concluir também que f = [f (1)]m/n . Lembrando
n
da denição de raiz enésima, vamos mostrar que os números [f (1)]m = f (m) e f (m/n) são
n
positivos e [f (m/n)] = f (m). De um modo geral, temos

x x  h  x i2
f (x) = f + = f >0 ∀x ∈ R .
2 2 2
Se f (x) = 0 para algum x ∈ R, então, repetindo sucessivamente a conta acima, teríamos
0 = f (x/2) = f (x/4) = f (x/8) = · · · . A sequência (x/2n )n∈N converge para zero, portanto,
pela continuidade de f , viria f (0) = 0. Porém, como estamos supondo f (0) = 1, segue f (x) > 0
n
para todo x ∈ R. Além disto, por indução, também podemos ver que [f (x)] = f (nx) para
todo n ∈ N e para todo x ∈ R. Em particular,
h  m in  m
f =f n· = f (m),
n n
como queríamos. Logo
f (r) = ar ∀r ∈ Q,
com a = f (1), positivo, pois já vimos ser f (x) um número positivo para todo x. Finalmente
f (x) = ax para todo x ∈ R pelas continuidades de f e da função denida por ax : dado x ∈ R,
xando uma sequência (rn )n∈N de números racionais que converge para x, temos

f (x) = lim f (rn ) = lim arn = ax

(Ver Exercício 7 do Capítulo VII, página 234). Segue a tese. 


398 CAPÍTULO 9. INTEGRAL DE RIEMANN

9.37 Exercício 37

Prove que lim n( n a − 1) = log a para todo a > 0.
n→∞

Solução. Esta igualdade, do modo como está escrita, é falsa. Por exemplo, para a = 2,
√ √
n
n( n a − 1) = n 1 = n ∀n ∈ N

n
e, de um modo mais geral, se a > 1, então lim a − 1 = 1, conforme já vimos no Exemplo 13
do Capítulo IV; portanto

lim n( n a − 1) = +∞ se a > 1.

O que é verdade é o seguinte


lim n( n a − 1) = log a.

Vamos mostrar que


ax − 1
lim = log a.
x→0 x
A demonstração se desenrola quando conseguimos identicar a derivada da função denida
x
por a :

ax − 1 ax − a0
= .
x x−0
Portanto
ax − 1
lim = a0 · log a = log a.
x→0 x
Logo

lim x( x a − 1) = log a
x→∞
x
a − 1
(dado ε > 0, para δ > 0 tal que 0 < |x| < δ ⇒
− log a < ε, tome A ∈ R tal que
x
x > A ⇒ 1/x < δ ). Em particular,


lim n( n a − 1) = log a,
n→∞

como queríamos. 

9.38 Exercício 38
Mostre que
n
nn+1 + (n + 1)n

lim = ee .
n→∞ nn+1
9.38. EXERCÍCIO 38 399

Solução. Vamos começar reescrevendo o termo geral desta sequência de modo a obter uma
expressão mais familiar:
n n
nn+1 + (n + 1)n (n + 1)n
 
= 1+
nn+1 nn+1
   n n
1 n+1
= 1+ ·
n n
   n n
1 1
= 1+ · 1+
n n
n n
" #
1 + n1

= 1+
n
h an in
= 1+ ,
n
para
 n
1
an = 1 + ∀n ∈ N .
n
Por um lado, dado ε0 > 0, sabemos que existe n0 tal que

n > n0 ⇒ e − ε0 < an < e + ε0 .


Dividindo por n, somando 1 e elevando a n, obtemos
 n  n
e − ε0  an  n e + ε0
n > n0 ⇒ 1+ < 1+ < 1+ .
n n n

 Por outro,
n pelo
 Exercício 34 (página 394), sabemos que, conforme n cresce, os números
n
e − ε0 e + ε0
1+ e 1+ se aproximam cada vez mais dos números ee−ε0 e ee+ε0 , res-
n n
pectivamente. Estas observações nos dão o norte de como proceder para demonstrar o resultado
desejado.
Dado ε > 0, seja ε0 > 0 tal que

ee − ε < ee−ε0 < ee+ε0 < ee + ε .


Este ε0 existe porque a função exponencial é contínua e crescente. Pelo referido exercício, sejam
n1 e n2 números naturais tais que
 n
e e − ε0
n > n1 ⇒ e − ε < 1 +
n
e  n
e + ε0
n > n2 ⇒ 1 + < ee + ε .
n
Seja também n0 ∈ N tal que
 n  n
e − ε0  an  n e + ε0
n > n0 ⇒ 1+ < 1+ < 1+ .
n n n
Então, para n3 = max{n0 , n1 , n2 }, temos
 n   n
e e − ε0 an n e + ε0
n > n3 ⇒ e − ε < 1 + < 1+ < 1+ < ee + ε .
n n n
400 CAPÍTULO 9. INTEGRAL DE RIEMANN

Logo,
 an  n
n > n 3 ⇒ ee − ε < 1 + < ee + ε,
n
o que demonstra a convergência desejada. 

9.39 Exercício 39
Se g : [c, d] → R é contínua e f : [a, b] → [c, d] é integrável então g ◦ f : [a, b] → R é integrável.

Solução. g ◦ f não é contínua num ponto x ∈ [a, b], então f não é contínua em x ou g
Se
não é contínua em f (x). Isto é a contrapositiva do Teorema 6 do Capítulo VII, o qual trata da
continuidade de funções compostas. Como g é contínua, se g ◦ f não é contínua em x, então
f não é contínua em x. Isto signica que o conjunto dos pontos de descontinuidade de g ◦ f é
subconjunto do conjunto dos pontos de descontinuidade de f . Este tem medida nula, porque f
é integrável (Teorema 20) e, portanto, aquele também tem medida nula. Logo, g ◦ f é integrável
de acordo com o Teorema 20. 

9.40 Exercício 40
Se f : [a, b] → [c, d] é de classe C 1 , com f 0 (x) 6= 0 para todo x ∈ [a, b], e g : [c, d] → R é
integrável, então g ◦ f é integrável.

(Voltar para a Observação 2 do Exercício 45: página 405.)

Solução. Seguindo o mesmo raciocínio da solução anterior  aplicando o Teorema 6 do Ca-


pítulo VII , como f é contínua, se g◦f não é contínua num ponto x ∈ [a, b], então g não é
contínua no ponto f (x), ou seja,
D1 ⊂ f −1 [D2 ],
onde D1 e D2 denotam os respectivos conjuntos dos pontos de descontinuidade de g ◦ f e de
−1
g. Pelo Teorema 20, D2 tem medida nula e, pela inclusão acima, se a medida de f [D2 ]
for nula, então a medida de D1 também será, donde poderemos concluir o resultado desejado
novamente pelo Teorema 20 (g◦f é limitada porque g é pressuposta limitada, pois ela é assumida
−1
integrável). Para tanto, vamos mostrar que a função f : f [[a, b]] → [a, b] é lipschitziana e,
então, aplicar o Exercício 25 (página 379). Porém, para que faça sentido falarmos na imagem
−1
de um conjunto por uma função (e possamos interpretar f [D2 ] não como a imagem inversa
−1
de f , mas como a imagem de f ), este conjunto deve primeiramente estar contido no domínio
desta função. Deste modo, observamos que vale mais do que a inclusão destacada mais acima,
a saber, que
D1 ⊂ f −1 D2 ∩ f [a, b] ,
  

sendo D2 ∩ f [[a, b]] (subconjunto de um conjunto de medida nula) também um conjunto de


medida nula. Neste contexto, enunciamos o seguinte

Lema. f : [a, b] → [c, d] uma função de classe C 1 com f 0 (x) 6= 0 para todo x ∈ [a, b].
Seja
−1
Se X ⊂ [c, d] ∩ f [[a, b]] tem medida nula, então f [X], sua imagem inversa por f , também tem
medida nula.

Solução. Pelo Teorema 5 do Capítulo VIII (o Teorema do Valor Intermediário para a


Derivada), se f 0 (x) 6= 0 para todo x ∈ [a, b], 0 0
então f (x) > 0 ∀x ∈ [a, b] ou f (x) < 0 ∀x ∈ [a, b].
9.41. EXERCÍCIO 41 401

Assim, pelo Corolário 6 do Teorema 7 do Capítulo VIII (e também pela observação que o
−1
segue), f admite uma inversa f : f [[a, b]] → [a, b] (sendo f [[a, b]] um intervalo) derivável, cuja
derivada é dada por
1
(f −1 )0 (f (x)) = ∀x ∈ [a, b].
f 0 (x)
Por Weierstrass (Corolário do Teorema 14 do Capítulo VII), como f0 é contínua, sejam x1
e x2 pontos em [a, b] tais que

f 0 (x1 ) 6 f 0 (x) 6 f 0 (x2 ) ∀x ∈ [a, b].

Disto obtemos
1 1
6 (f −1 )0 (f (x)) 6 ∀x ∈ [a, b],
f 0 (x 2) f 0 (x 1)

o que signica que a derivada de f −1 é limitada, portanto f −1 é lipschitziana (Exercício 24 do


Capítulo VIII, página 303). Deste modo, pelo Exercício 25 (página 379; f [[a, b]], além de ser
−1
um intervalo, é também compacto  Teorema 14 do Capítulo VII), a imagem de X por f tem
−1
medida nula. Neste caso, como X está contido na imagem de f , a imagem de X pela função f
−1
(denida na imagem de f ) coincide com a imagem inversa de X por f . Logo, f [X] (imagem
−1
inversa de X por f ou imagem de X por f , tanto faz) tem medida nula, como queríamos.

Logo g◦f é integrável. 

9.41 Exercício 41
Se X ⊂R tem contéudo nulo, o mesmo ocorre com seu fecho. Em particular, X tem interior
vazio. E se X tem medida nula?

(Voltar para a Solução do Exercício 43: página 403.)

Solução 1. Sejam I1 = (a1 , b1 ), . . ., In = (an , bn ) intervalos abertos. Se X ⊂ I1 ∪ · · · ∪ In ,


então

X ⊂ I1 ∪ · · · ∪ In = I1 ∪ I2 ∪ · · · ∪ In
(X ⊂ Y ⇒ X ⊂ Y, conforme a observação feita na página 170, e indução no Exercício 20 do
Capítulo V, página 173). Agora, com o item 4 das propriedades dos conjuntos de conteúdo
nulo em mente, para F = {a1 , b1 , . . . , an , bn }  um conjunto nito  temos

X − F ⊂ I1 ∪ · · · In .

Dado ε > 0, esses intervalos podem ser tomados de modo a termos |I1 | + |I2 | + · · · + |In | < ε,
uma vez que c(X) = 0. Logo c(X) = 0, pela referida propriedade 4, enunciada na página 338
do livro. Neste caso, o interior de X é vazio porque conjuntos de conteúdo nulo têm medida
nula (observação feita na página 343) e conjuntos de medida nula têm interior vazio (Exercício
22, página 377).
Para a última parte deste exercício, consideremos o conjunto dos racionais. Ele tem medida
nula, seu fecho é R e R não tem medida nula. Portanto, ao contrário do que ocorre com o
conteúdo, m(X) = 0 não implica m(X) = 0. 
402 CAPÍTULO 9. INTEGRAL DE RIEMANN

Solução 2 (para a primeira parte do exercício). Vamos levar em conta agora a observação
feita na página 338 do livro, a de que, na denição de conjunto de conteúdo nulo, podemos
considerar intervalos fechados em vez de intervalos abertos. O resultado seguirá então porque
os mesmos intervalos fechados que cobrem X também cobrem X: se I1 , · · · , In são intervalos
fechados tais que X ⊂ I1 ∪ · · · ∪ In , então, do mesmo modo como zemos no primeiro parágrafo
da solução acima, temos

X ⊂ I1 ∪ · · · ∪ In = I1 ∪ I2 ∪ · · · ∪ In = I1 ∪ I2 ∪ · · · ∪ In ,

pois, para cada i, Ii = Ii , uma vez que Ii é fechado. 

9.42 Exercício 42
Seja Q = {r1 , r2 , . . . , rn , . . .} uma enumeração dos racionais. Para n, i ∈
[N arbitrários,
\ seja
n+i
Jni o intervalo aberto de centro ri e comprimento 1/2 . Pondo An = Jni e A = An ,
i∈N n∈N
mostre que A tem medida nula e R = A ∪ (R −A) é uma decomposição da reta como reunião
disjunta de um conjunto de medida nula com um conjunto magro (isto é, reunião enumerável
de fechados com interior vazio).

(Voltar para a Solução do Exercício 13 do Capítulo X, para o elevador 1: página 418.)

Solução. Temos
[
A⊂ Jni ∀n ∈ N,
i∈N

onde, xado n ∈ N,
∞ ∞ ∞
X X 1 1 X 1 1
|Jni | = n+i
= n· i
= n.
i=1 i=1
2 2 i=1 2 2
Assim, dado ε > 0, tomando n∈N tal que 1/2n < ε, temos

[ ∞
X
A⊂ Jni e |Jni | < ε,
i∈N i=1

isto é, A tem medida nula.


Com relação ao conjunto R −A, temos

\ [
R −A = R − An = R −An ,
n∈N n∈N

onde cada R −An é fechado porque cada An é aberto, por ser uma reunião de abertos, e
int(R −An ) = ∅ por R −An não possuir nenhum número racional:

ri ∈ Jni ∀n ∈ N ⇒ ri ∈ An ∀n ∈ N ⇒ ri ∈
/ R −An ∀n ∈ N .

R −A é uma reunião de conjuntos fechados, cada um deles com interior vazio. Em


Logo,
consequência, R −A também tem interior vazio (Teorema de Baire - Exercício 54 do Capítulo
V, página 194). Em particular, temos também dois conjuntos, cada um com interior vazio, cuja
reunião é toda a reta. 
9.43. EXERCÍCIO 43 403

9.43 Exercício 43
O conjunto dos pontos de descontinuidade de uma função integrável tem medida nula porque é
reunião enumerável de conjuntos de conteúdo nulo. Caberia indagar se todo conjunto X ⊂ R de
medida nula é dessa forma. Mostre que o conjunto A do Exercício 42 não é reunião enumerável
de conjuntos de conteúdo nulo.

Solução. Ao supor A igual a uma reunião enumerável de conjuntos de conteúdo nulo, tenta-
mos derivar uma contradição ou um absurdo. Fazer isto olhando apenas para A mostra-se algo
infrutífero (pelo menos a princípio). Porém, o exercício anterior também nos sugere olhar para
fora de A, o que nos remete ao Teorema de Baire. Conjuntos de conteúdo nulo têm interior
vazio, mas não sabemos se eles são fechados, o que não é problema porque podemos considerar
seus fechos, os quais também têm interior vazio (Exercício 41, página 401). Este caminho nos
leva a escrever R como um conjunto magro, o que é um absurdo. Abaixo vamos escrever esta
demonstração em sua forma direta.
SejamX1 ,[
X2 , . . . , Xn , . . . conjuntos de números reais, cada um de conteúdo nulo, e escreva-
mos R −A = Fn , onde cada Fn é fechado e tem interior vazio (a possibilidade de se fazer isto
n∈N
é garantida pelo exercício anterior). Conforme vimos no Exercício 41, cada Xn também é um
conjunto fechado com interior vazior. Então, denindo G2n = Fn ∀n ∈ [
G2n−1 = Xn
N, pelo e

Teorema de Baire (Exercício 54 do Capítulo V, página 194), o interior do conjunto G = Gn é


n∈N
G não pode[
vazio, em particular ser igual a R. Seja y ∈ R −G. Este y não pertence à diferença

R −A e não pertence à reunião Xn . Por outro lado, de y ∈ R = A ∪ (R −A) e de y ∈/ R −A,


[n∈N [ [
segue y ∈ A. Logo, y ∈ A− Xn ⊂ A − Xn e A não é igual a Xn , como queríamos.
n∈N n∈N n∈N

9.44 Exercício 44
P
Dada uma sequência de intervalos abertos In , contidos em [0, 1], se |In | < 1 então o conjunto
fechado F = [0, 1] − ∪In não tem medida nula.


X
Solução. Sejam os intervalos abertos J1 , J2 , . . . , Jn , . . . e vamos supor que |Jn | seja um
n=1
número real. Se [ [
F = [0, 1] − In ⊂ Jn ,
n∈N n∈N

então
! !
[ [
[0, 1] ⊂ Jn ∪ In
n∈N n∈N
= J1 ∪ I1 ∪ J2 ∪ I2 ∪ J3 ∪ I3 ∪ · · · .

Daí, pela Proposição 2 do Capítulo V,


! ∞
!
X X
1 < |J1 | + |I1 | + |J2 | + |I2 | + · · · = |Jn | + |In | .
n=1 n=1
404 CAPÍTULO 9. INTEGRAL DE RIEMANN

Esta igualdade valendo pelo Exemplo 39 do Capítulo IV (dissociatividade). Portanto


! ∞
!
X X
|Jn | >1− |In |
n=1 n=1

e F não pode ter medida nula. 

9.45 Exercício 45. Conjunto de Cantor com medida posi-


tiva
Veja, na página 356 do volume 2, a construção de X , o conjunto de Cantor com medida
positiva. Dena um homeomorsmo h : [0, 1] → [0, 1] tal que h(X) = K (verdadeiro conjunto
de Cantor). Em seguida considere g : [0, 1] → R, a função característica de K . Mostre que g ◦ h
não é integrável, embora g seja integrável e h seja contínua.

(Voltar para o Exemplo 2 do Exercício 9: página 349.)

Solução. A única diferença na referida construção com relação à construção de


é que, em K
n
sua n-ésima etapa, em vez de retirarmos intervalos abertos de comprimentos iguais a (1/3) ,
n n−1
retiramos intervalos abertos de comprimentos iguais a α /2 , onde α é um número positivo
menor do que 1/2 xado a priori. Intervalos estes cujos centros coincidem com os centros dos
intervalos dos quais eles são retirados (do mesmo modo como acontece com K ).
Na referência feita ao volume 2 também é mostrado que X não tem medida nula e que
int(X) = ∅. A medida de X
não é nula como consequência do exercício anterior: a soma dos
n−1 αn
comprimentos dos intervalos retirados na etapa n é 2 · n−1 = αn (há 2n−1 intervalos que
2
sofrem este processo de retirada na etapa n), portanto o somatório dos comprimentos de todos

X X∞
os intervalos retirados é αn < (1/2)n = 1. E o interior de X é vazio pelo mesmo motivo
n=1 n=1
de o interior de K o ser.
A função g é integrável porque o conjunto de seus pontos de descontinuidade é K (a jus-
ticativa disto é semelhante à do Exemplo 9 do Capítulo VII), um conjunto de medida nula.
Além disto, se h : [0, 1] → [0, 1] é uma função injetiva tal que h[X] = K , então g ◦ h é a
função característica de X . Isto, junto com o fato de o interior de X ser vazio, implica X ⊂ D ,
denotando por D o conjunto dos pontos de descontinuidade de g ◦ h (novamente por uma das
ideias contidas no Exemplo 9 do Capítulo VII); m(X) 6= 0 ⇒ m(D) 6= 0 (m(S) 6= 0 signicando
que a medida de S não é nula), logo g ◦ h não é integrável. A única coisa que falta agora é
construir o homeomorsmo h. Pelo Teorema 13 do Capítulo VII, podemos nos preocupar mais
em fazer com que h : [0, 1] → [0, 1] seja uma bijeção contínua.
Vamos começar denindo h em [0, 1] − X . Em seguida, pela densidade, vamos conseguir
estendê-la para todo o intervalo [0, 1]. Usando notações análogas às da Solução do Exercício
m m m
17 do Capítulo VII (página 242), dado m ∈ N, sejam I1 , I2 , . . . , I2m −1 os intervalos retirados
m m m
na etapa m da construção de X e J1 , J2 , . . . , J2m −1 os da etapa m da construção de K , onde
x < y para todo x ∈ Ikm e para todo y ∈ Ilm se k < l e analogamente para os Jkm 's: x < y
∀x ∈ Jkm ∀y ∈ Jlm se k < l. Então denimos h linearmente em cada Ikm , fazendo

|Jkm |
h(x) = · (x − inf Ikm ) + inf Jkm ∀x ∈ Ikm .
|Ikm |
9.45. EXERCÍCIO 45. CONJUNTO DE CANTOR COM MEDIDA POSITIVA 405

Para vermos que não há ambiguidades nesta denição, notamos o seguinte (observações
m m
análogas as que seguem já foram feitas no referido Exercício 17). Dado m ∈ N, Ik ∩ Il = ∅
m m+1 m+2 m+p
se k 6= l e Ik = I2k = I4k = · · · = I2p k ; se x ∈ Ikm ∩ Iln , com n > m (sem perda de
generalidade), então, para p ∈ N tal que n = m + p, temos

Ikm ∩ Iln = Ikm ∩ Ilm+p = I2m+p


pk ∩ Ilm+p 6= ∅
e, portanto, l = 2p k , de modo que

|Jln | n n |J2m+p
pk | m+p m+p
n
· (x − inf Il ) + inf Jl = m+p · (x − inf I2p k ) + inf J2p k
|Il | |I2p k |
|Jkm |
= m
· (x − inf Ikm ) + inf Jkm ,
|Ik |
pois também vale J2m+p
pk = Jkm .
m n
Esta função é crescente: para x1 < x2 , se x1 ∈ Ik e x2 ∈ Il (se x1 e x2 pertencem a um
m m n m m+p
mesmo Ik , então é imediato), então h(x1 ) ∈ Jk e h(x2 ) ∈ Jl ; se n = m + p, então Ik = I2p k
n m+p p
e Il = Il , de sorte que x1 < x2 implica 2 k < l , portanto, h(x1 ) < h(x2 ) e analogamente
m −1
[ 2[
caso m = n + p. Além disto, a imagem de Ikm = [0, 1] − X por h é o conjunto
m∈N k=1
m −1
[ 2[
Jkm = [0, 1] − K , denso em [0, 1]. Portanto, pelo Exercício 16 do Capítulo VII (página
m∈N k=1
240), existe um único modo de estender h continuamente para [0, 1] − X = [0, 1], e este modo
se faz denindo
h(x) = lim
y→x
h(y) ∀x ∈ [0, 1].
y∈[0,1]−X

Ela é crescente: dados a < b em [0, 1], sejam x0 < y0 pontos em [0, 1] − X e sejam as sequências
(xn )n∈N e (yn )n∈N em [0, 1] − X tais que a 6 xn 6 x0 , y0 6 yn 6 b para todo n, xn → a e
y n → b, de sorte que

h(a) = inf{h(xn ); n ∈ N} 6 h(x0 ) < h(y0 ) 6 sup{h(yn ); n ∈ N} = h(b);


e sua imagem é o intervalo [0, 1]. Logo, pelo Teorema 13 do Capítulo VII, h : [0, 1] → [0, 1] é
um homeomorsmo; h[X] = K , pois x ∈ X ⇔ x ∈ / [0, 1] − X ⇔ h(x) ∈
/ [0, 1] − K ⇔ h(x) ∈ K
(a segunda dessas equivalências vale por causa da injetividade de h e porque h(y) ∈ [0, 1] − K
se y ∈ [0, 1] − X , pelo modo como começamos denindo h). 

a
Observação 1. Na 12 edição do volume 2 do Curso de Análise, a construção do conjunto
de Cantor com medida positiva começa na página 353.

Observação 2. Comparando com o Exercício 40 (página 400), podemos enunciar o seguinte:

seh : [0, 1] → [0, 1] é uma função injetiva tal que h[X] = K , então / C1
h ∈ ou
h ∈ C 1 e h0 (x) = 0 para algum x ∈ [0, 1],
lembrando que para g ◦ h (onde g : [0, 1] → R é a função característica de K ) não ser integrável
bastou ser h injetiva e h[X] = K .

Observação 3. Este exercício também nos diz que a propriedade de um conjunto ter medida
nula não é uma propriedade topológica, pois existe um homeomorsmo que não a preserva.
406 CAPÍTULO 9. INTEGRAL DE RIEMANN
Capítulo 10

Sequências e Séries de Funções

10.1 Exercício 1

Sejam fn (x) = x2n , gn (x) = x2n+1 e hn (x) = (1 − x2 )n . Determine os limites destas sequências
de funções no intervalo [−1, 1]. Examine a uniformidade da convergência e trace, em cada caso,
os grácos das três primeiras funções da sequência e da função limite.

Solução. Temos: fn (1) = gn (1) = fn (−1) = hn (0) = 1, gn (−1) = −1. Se x ∈ (−1, 1),
então as sequências (fn (x)) e (gn (x)) convergem para 0 (Exemplo 6 do Capítulo IV). E, pelo
mesmo motivo, se x ∈ [−1, 1] − {0}, então (hn (x)) também converge para zero. As sequências
(fn ), (gn ) e (hn ) convergem simplesmente, então, para f , g e h (respectivamente) denidas por
f (1) = f (−1) = g(1) = 1, g(−1) = −1, g(x) = f (x) = 0 para todo x ∈ (−1, 1), h(0) = 1 e
h(x) = 0 para todo x ∈ [−1, 1] − {0}.

Pelo Teorema 4, nenhuma destas convergências pode ser uniforme, porque todas as fn 's,
todas as gn 's e todas as hn 's f , g e h não. Por este mesmo teorema e
são contínuas, enquanto
pelo Corolário do Teorema 1, as convergências fn → f e gn → g não podem ser uniformes em
nenhum conjunto que possua o 1 ou o -1 em seu derivado (conjunto dos pontos de acumulação),
e a convergência hn → h não é uniforme em nenhum conjunto que possua o 0 em seu derivado.
Pelo Teorema de Dini (Teorema 5), porque todas estas convergências são monótonas, as con-
vergências fn |C → f |C , gn |C → g|C e hn |D → h|D são uniformes se C e D são subconjuntos
compactos de / C 0 , −1 ∈
[−1, 1], 1 ∈ / C0 e 0 ∈
/ D0 .

407
408 CAPÍTULO 10. SEQUÊNCIAS E SÉRIES DE FUNÇÕES

···

f1 f2 f3 f

···

g1 g2 g3 g

···

h1 h2 h3 h 

10.2 Exercício 2
Prove que fn (x) = nx(1 − x)n converge simplesmente, porém não uniformemente, em [0, 1],
para a função identicamente nula.

Solução. Converge simplesmente principalmente por causa do Exemplo 21 e do item 3 do


Teorema 14, ambos do Capítulo IV:

1 (1/(1 − x))n
= → +∞,
n(1 − x)n n

se x ∈ (0, 1), pois, neste caso, 1/(1 − x) > 1; e fn (0) = fn (1) = 0 para todo n.
O motivo pelo qual essa convergência não é uniforme é semelhante ao do Exemplo 4. De

fn0 (x) = n(1 − x)n − n2 x(1 − x)n−1 = n(1 − x)n−1 [−x(n + 1) + 1],

1
podemos ver que o valor máximo de fn em [0, 1] é atingido em x= , e este valor é
n+1
   n
1 n 1
fn = 1−
n+1 n+1 n+1
 n
n n
=
n+1 n+1
n 1
= · ,
n + 1 (1 + 1/n)n

número este que converge para 1/e, um número positivo. Então, xando ε positivo e menor
 
1
do que 1/e, para qualquer n0 ∈ N sempre podemos encontrar n > n0 tal que fn > ε.
n+1
Logo, a convergência fn → 0 não é uniforme. 
10.3. EXERCÍCIO 3 409

10.3 Exercício 3
 
1
Seja lim an = a, com an 6= a para todo n. Prove que a sequência de funções fn (x) = an x +
  x
1
converge simplesmente para f (x) = a x + em R −{0}. A convergência é uniforme num
x
conjunto X ⊂ R −{0} se, e somente se, X é limitado e 0 ∈ / X 0.

Solução. O que justica a convergência simples é o Exemplo 1. Com respeito à convergência


uniforme, vamos começar supondo X ⊂ R −{0} limitado e tal que 0 ∈/ X 0 . Isto signica que
existem δ e K positivos tais que δ 6 |x| < K para todo x ∈ X. Dado ε > 0, queremos n0 ∈ N
tal que

1
n > n0 ⇒ |an − a| · x + <ε ∀x ∈ X,
x
ou, de forma equivalente, e até para aproveitar a informação sobre o limite dos an 's ser a,
n0 ∈ N tal que

x
n > n0 ⇒ |an − a| < ε · 2 ∀x ∈ X.
x + 1
As hipóteses acerca de X nos permitem limitar os números
|x|/(x2 + 1) por baixo por um
x
número positivo. Sendo K0 > 0 tal que K0 6 2 para todo x ∈ X, o n0 ∈ N testemunha
x + 1
de que fn → f uniformemente é aquele tal que


x
n > n0 ⇒ |an − a| < ε ·K0 6 ε 2
∀x ∈ X
x + 1
2
x + 1
⇒ |an − a| ·
= |fn (x) − f (x)| < ε ∀x ∈ X,
x

o qual existe porque lim an = a. K0 a partir da implicação


Denimos este


|x| > δ |x| δ
2 2 ⇒ 2 > 2
x +1 < K +1 x +1 K +1

δ
como K0 = . Logo, dado X ⊂ R −{0}, se X é limitado e se / X 0,
0 ∈ então fn → f
K2 +1
uniformemente.
410 CAPÍTULO 10. SEQUÊNCIAS E SÉRIES DE FUNÇÕES

Reciprocamente, dado X ⊂ R −{0}, vamos supor X ilimitado ou 0 ∈ X 0. Queremos


encontrar ε0 > 0 tal que, dado n0 ∈ N, exista n > n0 tal que

1
|an − a| · x + > ε0

x
para algum x ∈ X. Isto é possível porque
 
1 1
x + > max |x|, .
x |x|
Assim tomamos ε0 = 1. Se X não é limitado, dado n ∈ N, existe x ∈ X tal que |x| >
1 0 1 1
. Se 0 ∈ X , dado n ∈ N, existe x ∈ X tal que > . Lembremos que
|an − a| |x| |an − a|
|an − a| =
6 0 para todo n ∈ N. De qualquer forma, se X é ilimitado
ou se admite o 0 como um
1 1
ponto de acumulação, dado n ∈ N, existe x ∈ X tal que x + > , ou seja, tal que

x |an − a|
1
|an − a| · x + > 1, como queríamos. 
x

Observação. Este Exercício pode ser comparado com o Exemplo 6. Considerando a função
1
g denida por g(x) = x + para todo x real diferente de zero, com as contas feitas acima,
x
podemos concluir que, dado ∅ =
6 X ⊂ R −{0}, a restrição g|X é limitada se, e somente se, X é
/ X 0.
limitado e 0 ∈
Outra forma de concluir que
g|X é limitada, caso X seja limitado e 0 ∈ / X 0 , é através da
1 1
Desigualdade Triangular: x + 6 |x| + (o fato de X ser limitado limita os números da
x |x|
1
forma |x|, com x ∈ X, e o fato de 0 ∈ / X 0 limita os números da forma , com x ∈ X ).
|x|

10.4 Exercício 4
Se lim an = a, lim bn = b, . . . , lim jn = j, lim ln = l então a sequência de polinômios pn (x) =
an + bn x + · · · + jn x9 + ln x10 converge para o polinômio p(x) = a + bx + · · · + jx9 + lx10 , unifor-
memente em cada intervalo compacto [a, b]. (Para armações mais completas, veja Elementos
de Topologia Geral, do autor, p. 211.)

Solução 1. Existe K > max{1, |x|, |x|2 , . . . , |x|10 } para todo x ∈ [a, b], de tal modo que, para
este K,

|pn (x) − p(x)| 6 |an − a| + |bn − b| · |x| + · · · + |jn − j| · |x|9 + |ln − l| · |x|10

6 |an − a| + |bn − b| + · · · + |jn − j| + |ln − l| · K,

para todo x ∈ [a, b]. Assim, dado ε > 0, para n0 tal que cada um dos números |an − a|,
ε
|bn − b|, . . . , |ln − l| é menor do que se n > n0 , temos
11K
ε
|pn (x) − p(x)| < 11 · · K = ε,
11K
logo, pn → p uniformemente em [a, b], como queríamos. 
10.5. EXERCÍCIO 5 411

Solução 2. Pelo Exemplo 6, como, para cada k ∈ {1, · · · , 10}, a função x 7→ xk é limitada
em [a, b], temos as convergências: an → a, bn x → bx, . . . , jn x9 → jx9 e ln x10 → lx10 , todas
uniformes.
(fn ) e (gn ) (todas com o mesmo
Agora a tese segue do seguinte resultado: se as sequências
domínio X 6= ∅ de números reais) convergem uniformemente para f e para g (respectivamente),
então a sequência (fn + gn ) converge uniformemente para f + g .
De fato, dado ε > 0, para n0 = max{n1 , n2 }, sendo n1 e n2 naturais tais que

ε
n > n1 ⇒ |fn (x) − f (x)| < ∀x ∈ X
2
e
ε
n > n2 ⇒ |gn (x) − g(x)| < ∀x ∈ X,
2
temos

n > n0 ⇒ |(fn + gn )(x) − (f + g)(x)| 6 |fn (x) − f (x)| + |gn (x) − g(x)| < ε ∀x ∈ X
(Desigualdade Triangular). 

Observação. Nas soluções acima, de [a, b], só foi usada sua limitação. O exercício abaixo
vai nos dizer que, neste contexto, não há problemas em termos usado apenas esta propriedade
dentre todas as propriedades que o intervalo compacto [a, b] possui.

10.5 Exercício 5
As seguintes armações a respeito de uma sequência de funções fn : R → R são equivalentes:

(a) a sequência converge uniformemente em toda parte limitada de R;


(b) idem em toda parte compacta;

(c) idem em todo intervalo compacto.

Solução. Que (a) implica (b) e (b) implica (c) é imediato, pois uma parte compacta é uma
parte limitada e um intervalo compacto é uma parte compacta. Se X ⊂R é limitado, sejam
a e b números reais, com a < b, tais que X ⊂ [a, b]. Supondo o item (c), (fn ) converge
uniformemente em [a, b], isto é, dado ε > 0, existe n0 ∈ N tal que

m, n > n0 ⇒ |fm (x) − fn (x)| < ε ∀x ∈ [a, b]


(Teorema 1). Em particular,

m, n > n0 ⇒ |fm (x) − fn (x)| < ε ∀x ∈ X,


portanto (fn ) converge uniformemente em X (Teorema 1), ou seja, o item (c) implica o item
(a). 

10.6 Exercício 6
Se fn → f e gn → g simplesmente em X, então fn + gn → f + g e f n · gn → f · g simplesmente
1 1
em X. Além disso, → simplesmente em X, desde que f (x) 6= 0 para todo x ∈ X.
fn f
412 CAPÍTULO 10. SEQUÊNCIAS E SÉRIES DE FUNÇÕES

Solução. Isto é consequência imediata do Teorema 6 do Capítulo IV. Por ele, dado x ∈ X,
temos
lim(fn + gn )(x) = lim(fn (x) + gn (x)) = f (x) + g(x) = (f + g)(x),
lim(fn · gn )(x) = lim(fn (x) · gn (x)) = f (x) · g(x) = (f · g)(x)
e, se f (x) 6= 0, então

1 1
lim = . 
fn (x) f (x)

10.7 Exercício 7
Se fn → f e gn → g uniformemente em X , então fn + gn → f + g uniformemente em X . Se
existir c > 0 tal que |fn (x)| 6 c e |gn (x)| 6 c para todo n ∈ N e todo x ∈ X então fn · gn → f · g
1 1
uniformemente em X . Também → uniformemente em X desde que exista a > 0 tal
fn f
que |fn (x)| > a para todo n ∈ N e todo x ∈ X . Dê exemplos mostrando que as duas últimas
conclusões seriam falsas sem as hipóteses adicionais.

(Voltar para a Solução 2 do Exercício 26: página 429.)

Solução. Quanto à soma, de

|(fn + gn )(x) − (f + g)(x)| 6 |fn (x) − f (x)| + |gn (x) − g(x)| ∀x ∈ X ∀n ∈ N,

dado ε > 0, vem

ε ε
n > max{n1 , n2 } ⇒ |(fn + gn )(x) − (f + g)(x)| < + =ε ∀x ∈ X
2 2
se os naturais n1 e n2 são tais que

ε
n > n1 ⇒ |fn (x) − f (x)| < ∀x ∈ X
2
e
ε
n > n2 ⇒ |gn (x) − g(x)| < ∀x ∈ X,
2
logo, fn + gn → f + g uniformemente.
Quanto ao produto, escrevendo


|fn (x) · gn (x) − f (x) · g(x)| = |fn (x) · gn (x) + fn (x)g(x) − fn (x)g(x) − f (x) · g(x)|
 
= | fn (x) · gn (x) − fn (x)g(x) + fn (x)g(x) − f (x) · g(x) |
6 |fn (x) · gn (x) − fn (x)g(x)| + |fn (x)g(x) − f (x) · g(x)|
= |fn (x)| · |gn (x) − g(x)| + |g(x)| · |fn (x) − f (x)|,

notamos que |gn (x)| 6 c ∀n ∈ N ∀x ∈ X e lim gn (x) = g(x) implicam |g(x)| 6 c ∀x ∈ X


(Corolário 1 do Teorema 7, Capítulo IV, pois |α| 6 c ⇔ −c 6 α 6 c), de modo que

|fn (x) · gn (x) − f (x) · g(x)| 6 c · |gn (x) − g(x)| + c · |fn (x) − f (x)| ∀x ∈ X

e, dado ε > 0,
ε ε
n > max{n1 , n2 } ⇒ |fn (x) · gn (x) − f (x) · g(x)| < c · +c· =ε ∀x ∈ X
2c 2c
10.8. EXERCÍCIO 8 413

se n1 e n2 são números naturais tais que

ε
n > n1 ⇒ |fn (x) − f (x)| < ∀x ∈ X
2c
e
ε
n > n2 ⇒ |gn (x) − g(x)| < ∀x ∈ X,
2c
o que mostra a convergência uniforme de f n gn para f g.
Com respeito aos inversos multiplicativos, começamos notando que |fn (x)| > a > 0 e
fn (x) → f (x) implicam |f (x)| > a (Corolário 1 do Teorema 7, Capítulo IV, junto com a
igualdade lim |xn | = | lim xn | se (xn ) é uma sequência convergente), em particular, fn (x) 6= 0 e
f (x) 6= 0 ∀n ∈ N ∀x ∈ X . Fora isto,

1 1 f (x) − fn (x)
fn (x) − f (x) = f (x)fn (x)

1
= · |f (x) − fn (x)|
|f (x)fn (x)|
1
6 2 · |f (x) − fn (x)| ∀x ∈ X,
a
portanto, dado ε > 0,

1 1 1
n > n0 ⇒
− < 2 · ε a2 = ε ∀x ∈ X,
fn (x) f (x) a

se n0 ∈ N é tal que
n > n0 ⇒ |f (x) − fn (x)| < ε a2 ∀x ∈ X,
1 1
logo → uniformemente.
fn f
Quanto aos exemplos, para cada n ∈ N, denimos fn (x) = 1/n e gn (x) = x para todo x ∈ R.
As sequências
  (fn )n∈N e (gn )n∈N são uniformemente convergentes em R, enquanto (fn gn )n∈N e
1
não são uniformemente convergentes em R. 
fn n∈N

10.8 Exercício 8
Seja lim fn = f uniformemente em X . Se a 6 f (x) 6 b para todo x ∈ X então, dado ε > 0,
existe n0 ∈ N tal que n > n0 ⇒ a − ε < fn (x) < b + ε para todo n > n0 e todo x ∈ X . Conclua
que, no Exercício 7, as hipóteses sobre gn e fn podem ser substituídas por hipóteses sobre g e
f respectivamente.
(Voltar para a Solução do Exercício 11: página 416.)

Solução. A desigualdade |f (x)−fn (x)| < ε equivale a f (x)−ε < fn (x) < f (x)+ε (Observação
após o Teorema 1 do Capítulo III), então, tomando n0 tal que n > n0 ⇒ |f (x) − fn (x)| < ε
∀x ∈ X , temos

a − ε 6 f (x) − ε < fn (x) < f (x) + ε 6 b + ε ∀x ∈ X

se n > n0 .
414 CAPÍTULO 10. SEQUÊNCIAS E SÉRIES DE FUNÇÕES

Em particular, se c>0 é tal que −c/2 6 f (x) 6 c/2 ∀x ∈ X e se fn → f uniformemente,


então existe n3 tal que

c c c c
n > n3 ⇒ − − = −c < fn (x) < + = c ∀x ∈ X ⇔ |fn (x)| < c ∀x ∈ X.
2 2 2 2
E, seguindo esta mesma linha de raciocínio, se a>0 é tal que |f (x)| > 3a/2 ∀x ∈ X , então
existe n4 tal que
n > n4 ⇒ |fn (x)| > a ∀x ∈ X
(fn (x) < −3a/2 + a/2 = −a se f (x) < −3a/2
fn (x) > 3a/2 − a/2 = a se f (x) > 3a/2).
e
Deste modo, supondo ambas as convergências fn → f e gn → g uniformes num conjunto
X , para concluir que fn gn → f g uniformemente, podemos supor a existência de um c > 0
tal que |f (x)| 6 c e |g(x)| 6 c ∀x ∈ X e, para termos 1/fn → 1/f uniformemente, podemos
supor a existência de um a > 0 tal que |f (x)| > a ∀x ∈ X . Com isto, nas contas do exercício
anterior, no primeiro caso tomamos n > max{n1 , n2 , n3 } e, no segundo caso, n > max{n0 , n4 }
(onde n0 , n1 , n2 , n3 e n4 são números naturais convenientes que nos permitem tirar as devidas
conclusões). 

10.9 Exercício 9
Dada f : X → R limitada, seja ||f || = sup{|f (x)|; x ∈ X}. O número ||f || chama-se a norma
da função f . Prove as seguintes relações:

(1) ||f || 6 0 [sic ]; ||f || = 0 ⇔ f (x) = 0 para todo x ∈ X;

(2) ||f + g|| 6 ||f || + ||g||;

(3) ||c · f || = |c| · ||f || se c ∈ R;

(4) | ||f || − ||g|| | 6 ||f − g||;

(5) ||f · g|| 6 ||f || · ||g||.

(Voltar para a Solução do Exercício 11: página 416.)

Solução.

item (1) No livro está escrito ||f || 6 0, mas deveria ser ||f || > 0: ||f || > |f (x)| > 0 para
um dado x ∈ X . Se f (x) = 0 para todo x ∈ X , então ||f || = sup{0} = 0. Reciprocamente, se
||f || = 0, então |f (x)| 6 0 ∀x ∈ X , o que implica |f (x)| = 0 ∀x ∈ X , uma vez que |f (x)| > 0
∀x ∈ X . 

item (2) Temos

||f || + ||g|| = sup{|f (x)|; x ∈ X} + sup{|g(x)|; x ∈ X}


(Lema 2 do Capítulo IX) = sup{|f (x)| + |g(y)|; x, y ∈ X}
> sup{|f (x)| + |g(x)|; x ∈ X}
> |f (a)| + |g(a)| ∀a ∈ X
(Desigualdade Triangular) > |f (a) + g(a)| ∀a ∈ X
10.10. EXERCÍCIO 10 415

(na primeira desigualdade que aparece acima: se A ⊂ B e B é limitado superiormente, então


A é limitado superiormente e sup A 6 sup B ). Portanto, ||f || + ||g|| é uma cota superior para
{|f (x) + g(x)|; x ∈ X} = {|(f + g)(x)|; x ∈ X}. Assim,
||f || + ||g|| > sup{|(f + g)(x)|, x ∈ X} = ||f + g||,
como queríamos. 

item (3) Isto é consequência do Lema 3 do Capítulo IX:

||c · f || = sup{|cf (x)|; x ∈ X}


= sup{|c| · |f (x)|; x ∈ X}
= |c| · sup{|f (x)|; x ∈ X}
= |c| · ||f ||
(o que é verdade mesmo que c seja igual a zero). 

item (4) Seguindo a mesma ideia da demonstração do item (iii) do Teorema 2 do Capítulo
III (um Teorema análogo a este exercício), vamos mostrar que ||f −g|| > ||f ||−||g|| e ||f −g|| >
||g|| − ||f || (e então o resultado vai seguir do Teorema 1 do Capítulo III). Pelo item (2) acima,
temos
||f || = ||(f − g) + g|| 6 ||f − g|| + ||g||
e
||g|| = ||(g − f ) + f || 6 ||g − f || + ||f ||,
portanto,
||f − g|| > ||f || − ||g|| e ||g − f || > ||g|| − ||f ||,
onde, pelo item acima,

||g − f || = || − (f − g)|| = | − 1| · ||f − g|| = ||f − g||.


Segue a tese. 

item (5) Temos

|f (x)g(x)| = |f (x)| · |g(x)| 6 ||f || · ||g|| ∀x ∈ X,


portanto,
||f || · ||g|| > sup{|f (x)g(x)|; x ∈ X} = ||f g||,
como queríamos. 

Observação. Os três primeiros itens deste exercício nos dão um modo de medir um com-
primento de uma dada função limitada f :X →R e ||f − g|| nos dá uma distância entre as
funções limitadas f, g : X → R.

10.10 Exercício 10
Prove que fn → f uniformemente em X se, e somente se, fn − f é limitada para todo n
sucientemente grande e lim ||fn − f || = 0.
n→∞
416 CAPÍTULO 10. SEQUÊNCIAS E SÉRIES DE FUNÇÕES

Solução. Se fn → f uniformemente, dado ε > 0, existe n0 tal que |fn (x) − f (x)| < ε para
todo x ∈ X , qualquer que seja o natural n maior do que n0 . Em particular fn − f é limitada
para todo n > n0 . Se |fn (x) − f (x)| < ε para todo x ∈ X , então

||fn − f || = sup{|fn (x) − f (x)|; x ∈ X} 6 ε .

Logo lim ||fn − f || = 0 (convergência de uma sequência de números reais) se fn → f uniforme-


mente.
Reciprocamente, sefn − f é limitada para todo n sucientemente grande, então faz sentido
considerarmos o limite dos ||fn −f ||'s (possivelmente desconsiderando os primeiros termos desta
sequência, aqueles tais que fn − f não é limitada). Dado ε > 0, para n0 ∈ N tal que fn − f é
limitada para todo n > n0 e


n > n0 ⇒ ||fn − f || = ||fn − f || < ε,

como ||fn − f || > |fn (x) − f (x)| para todo x ∈ X, temos

n > n0 ⇒ |fn (x) − f (x)| < ε ∀x ∈ X.

Logo, fn → f uniformemente, como queríamos. 

10.11 Exercício 11
Sejalim fn = f uniformemente em X . Então f é limitada se, e somente, existe n0 ∈ N tal que
n > n0 ⇒ fn limitada. No caso armativo, tem-se lim ||fn || = ||f ||.

Solução. Se f é limitada, então existe n0 tal que n > n 0 ⇒ fn limitada, pelo Exercício 8
acima (página 413). Reciprocamente, temos

|f (x)| 6 |f (x) − fn (x)| + |fn (x)| ∀x ∈ X ∀n ∈ N .

Em particular, tomando n∈N tal que |f (x) − fn (x)| < 1 para todo x∈X e fn seja limitada,
temos
|f (x)| 6 1 + K ∀x ∈ X,
onde K∈R é tal que K > |fn (x)| ∀x ∈ X , logo, f é limitada.
No caso armativo, podemos considerar as normas ||f || e ||fn || para n sucientemente
grande. Para além disto, pelo item (4) do Exercício 9 (página 414) e pelo exercício anterior
temos

0 6 ||fn || − ||f || 6 ||fn − f || → 0,

portanto, ||fn || − ||f || → 0, segundo o Teorema 8 do Capítulo IV, ou seja, lim ||fn || = ||f ||,
como queríamos. 

10.12 Exercício 12
Seja f : I → R uma função que é contínua em todos os pontos do intervalo I , salvo um
único ponto c. Obtenha uma sequência de funções contínuas fn : I → R tal que lim fn =
f simplesmente. Generalize para uma função f com um número nito de descontinuidades
c1 , . . . , c n .
10.13. EXERCÍCIO 13 417

  
1 1
Solução. n ∈ N, denimos fn ligando o ponto
Para cada c − ,f c −
ao ponto
   n n
1 1
(c, f (c)) e este ao ponto c + , f c + por segmentos de retas. Isto se c for ponto
n n
interior. Se c for um dos extremos de I , consideramos apenas o segmento de reta conveninente.
Nos demais pontos de I fazemos fn coincidir com f . Em qualquer um destes casos, para n
grande o suciente, fn estará bem denida. De modo mais preciso:

• se c ∈ int(I), seja n0 ∈ N [c − 1/n0 , c + 1/n0 ] ⊂ I . Dado n > n0 , denimos


tal que

 
1 1
fn (x) = f (x) ∀x ∈ I − c − , c + ,
n n
        
1 1 1 1
fn (x) = n · f (c) − f c − · x−c+ +f c− ∀x ∈ c − , c
n n n n
e      
1 1
fn (x) = n · f c + − f (c) · (x − c) + f (c) ∀x ∈ c, c + ;
n n

• se c é um extremo à esquerda de I, seja n0 ∈ N


[c, c + 1/n0 ] ⊂ I . Dado n > n0 ,
tal que
denimos  
1
fn (x) = f (x) ∀x ∈ I − c, c + ,
n
e      
1 1
fn (x) = n · f c + − f (c) · (x − c) + f (c) ∀x ∈ c, c + ;
n n

• se c é um extremo à direita de I, seja n0 ∈ N [c − 1/n0 , c] ⊂ I .


tal que Dado n > n0 ,
denimos  
1
fn (x) = f (x) ∀x ∈ I − c − , c ,
n
e
        
1 1 1 1
fn (x) = n · f (c) − f c − · x−c+ +f c− ∀x ∈ c − ,c .
n n n n

Cada fn fn → f simplesmente porque: fn (c) = f (c) para todo n > n0 e se x


é contínua e
é um ponto de I c, então existe n1 ∈ N tal que x ∈
diferente de / (c − 1/n, c + 1/n) para todo
n > n1 , o que faz fn (x) = f (x) para todo n > max{n0 , n1 }. A ideia do caso mais geral é a
mesma, tomando agora o cuidado de fazer com que o n0 da construção acima também satisfaça
1/n0 6 min{|cj − ci |/2; i, j ∈ {1, . . . , n}, i 6= j}, para não haver ambiguidades na construção
de cada termo da sequência. 

10.13 Exercício 13
Mostre que não existe uma sequência de funções contínuas fn : [0, 1] → R, convergindo sim-
plesmente para a função f : [0, 1] → R tal que f (x) = 0 para x racional e f (x) = 1 quando x é
irracional.

Solução.
418 CAPÍTULO 10. SEQUÊNCIAS E SÉRIES DE FUNÇÕES

No elevador 1. Por fazer uma armação acerca dos racionais e dos irracionais, lembramos
dos Exercícios 55 do Capítulo V (Q não é uma interseção enumerável de abertos e R−Q não
é uma reunião enumerável de fechados, página 195) e 18 do Capítulo VII (uma função denida
em R não pode ser contínua nos racionais e descontínua nos irracionais, página 243), este
decorrente daquele e ambos relacionados com o Teorema de Baire (Exercício 54 do Capítulo
V, página 194). Uma primeira pergunta que podemos nos fazer então é: o atual exercício
tem a ver com estes dois? Uma outra coisa que chama a atenção aqui é o fato de f ser
descontínua em todos os pontos de seu domínio. Podemos então colocar também a seguinte
pergunta: o limite simples de uma sequência de funções contínuas, denidas em [0, 1], caso
exista, é contínuo em pelo menos um ponto de [0, 1]? A resposta, positiva neste caso, é dada
por um resultado conhecido como Teorema de Baire-Kuratowski, segundo o qual o conjunto
dos pontos de descontinuidade do limite simples de uma sequência de funções contínuas é um
conjunto magro (conforme a denição do Exercício 42 do Capítulo IX, página 402). Resultado
este que reúne as duas perguntas feitas anteriormente. A solução que iremos dar abaixo é uma
adaptação da demonstração deste teorema apresentada no livro Aplicações da Topologia à
Análise de Chaim Samuel Hönig, o qual, por sua vez, se baseou no artigo On set points of
discontinuity de Ofélia Teresa Alas.

Nível 1. Seja (fn )n∈N uma sequência de funções contínuas denidas no intervalo [0, 1]. Se
ela não converge pontualmente, então não há nada a se fazer; com maior razão ela não converge
paraf . Vamos supor então que ela converge simplesmente para uma função g : [0, 1] → R.
Sendo D o conjunto dos pontos nos quais g é descontínua, armamos que D é um conjunto
magro (Nível 2 e subsequentes). Assim, pelo Teorema de Baire (Exercício 54 do Capítulo V,
página 194), o interior de D é vazio e, em particular, D [0, 1], uma vez que
não pode ser igual a
o interior deste é não vazio. Portanto, existe um ponto em [0, 1] no qual g é contínua, donde g
não pode ser igual a f , uma função descontínua em todos os pontos de [0, 1].

No elevador 2. No Exercício 18 do Capítulo VII (página 243) já vimos como escrever


o conjunto dos pontos de continuidade de uma função denida em R como uma interseção
enumerável de abertos. Passando para o complementar, podemos escrever o conjunto dos
pontos de descontinuidade de uma função denida em R como uma reunião enumerável de
fechados. Para seguir esta ideia, um cuidado que precisamos tomar é o de atentar para o fato
de que g não está denida em toda a reta, mas apenas no intervalo [0, 1].

Nível 2. Fixado ε > 0, seja Dε o conjunto denido da seguinte forma: dadoa ∈ R, a ∈ Dε


se, e somente se, dado um intervalo aberto I possuindo a, existem x e y em I ∩ [0, 1] tais que
|g(x) − g(y)| > ε. Isto implica Dε ⊂ [0, 1] (se a ∈
/[[0, 1], considere o intervalo aberto I possuindo
a tal que I ∩ [0, 1] = ∅). Armamos que D = D1/n (Nível 3.1). Assim, para concluirmos
n∈N
que D é um conjunto magro, é suciente mostrar que Dε é magro (Nível 3.2, e subsequentes).

Nível 3.1. a ∈ D, seja ε > 0 tal que, dado o intervalo aberto I possuindo a, |g(x) −
Se
g(a)| > ε x ∈ I ∩ [0, 1] (lembrando que D ⊂ [0, 1]). Portanto, a ∈ D1/n se n > 1/ ε
para algum
(Propriedade Arquimediana). Reciprocamente, seja a ∈ / D. Dado n ∈ N, se a ∈ / [0, 1], então
a ∈
/ D1/n , uma vez que D1/n ⊂ [0, 1], e; se a ∈ [0, 1], então, tomando o intervalo aberto I
possuindo a tal que |g(x) − g(a)| < 1/(2n) ∀x ∈ I ∩ [0, 1] (o qual existe porque g é contínua em
10.13. EXERCÍCIO 13 419

a), temos |g(x) − g(y)| 6 |g(x) − g(a)| + |g(a) − g(y)| < 1/(2n) + 1/(2n) = 1/n ∀x, y ∈ I ∩ [0, 1],
logo, a ∈
/ D1/n .

Nível 3.2. Pelo fato de (fn (x)) ser uma sequência convergente qualquer que seja o x∈
[0, 1], xado um ε0 > 0, podemos armar o seguinte: dado x ∈ [0, 1], existe k ∈ N tal que
|fn (x) − fk (x)| 6 ε0 para todo n>k (Critério de Cauchy). Em outras palavras, denindo

Fk = {x ∈ [0, 1]; |fn (x) − fk (x)| 6 ε0 para todo n > k},

temos [
[0, 1] = Fk .
k∈N

Assim,
" #
[ [
Dε = Dε ∩ [0, 1] = Dε ∩ Fk = Dε ∩ Fk .
k∈N k∈N

A partir disto armamos: Dε Fk são ambos fechados (Níveis 4.1 e 4.2 respectivamente),
e
portanto Dε ∩ F k é fechado, e; int(Dε ∩ Fk ) = ∅ para uma escolha apropriada do ε0 (Nível 4.3).
Logo Dε é, de fato, um conjunto magro, como queríamos.

Nível 4.1. Sea∈ / Dε , para o intervalo aberto I possuindo a tal que |g(x) − g(y)| < ε para
todo x, y ∈ I ∩ [0, 1], temos I ⊂ R −Dε , pois, se x ∈ I , então I é um intervalo aberto possuindo
x tal que |g(y) − g(z)| < ε para todo y, z ∈ I ∩ [0, 1], o que faz x não pertencer a Dε . Logo,
R −Dε é aberto, ou seja, Dε é fechado.

Nível 4.2. Que cada Fk é um conjunto fechado é consequência da combinação do Teorema


5 com os Exercícios 3 (página 232) e 5 (página 233), todos, do Capítulo VII:

\
Fk = E[|fn − fk | 6 ε0 ],
n>k

onde cada conjunto E[|fn − fk | 6 ε0 ] := {x ∈ [0, 1]; |fn (x) − fk (x)| 6 ε0 } é fechado porque a
função x 7→ |fn (x) − fk (x)| = max{fn (x) − fk (x), fk (x) − fn (x)} é contínua e [0, 1] é fechado.

Nível 4.3. De int(Dε ∩ Fk ) = int(Dε ) ∩ int(Fk ) (Exercício 5 do Capítulo V, página 161)


e intDε ⊂ Dε , vamos mostrar que o ε0 da denição de Fk pode ser escolhido de modo a termos
a implicação
a ∈ int(Fk ) ⇒ a ∈
/ Dε ,
isto é, xado a ∈ int(Fk ), queremos δ>0 tal que

x, y ∈ (a − δ, a + δ) ∩ [0, 1] ⇒ |g(x) − g(y)| < ε .

Para usar a hipótese a ∈ int(Fk ), escrevemos

|g(x) − g(y)| 6 |g(x) − fk (x)| + |fk (x) − fk (y)| + |fk (y) − g(y)|.

Então seja I = (a − δ, a + δ), com δ > 0, tal que I ⊂ Fk e |fk (x) − fk (y)| < ε /3 para
todo x, y ∈ I ∩ [0, 1]. Este I existe porque a pertence ao interior de Fk e fk é contínua. Dado
420 CAPÍTULO 10. SEQUÊNCIAS E SÉRIES DE FUNÇÕES

z ∈ I ∩ [0, 1], de fn (z) → g(z) e |fn (z) − fk (z)| 6 ε0 para todo n > k, temos |g(z) − fk (z)| 6 ε0
(Exercício 1 do Capítulo IV, página 117). Portanto:
ε
x, y ∈ I ∩ [0, 1] ⇒ |g(x) − g(y)| < 2 ε0 + .
3
Logo, escolhendo ε0 = ε /3, temos
x, y ∈ I ∩ [0, 1] ⇒ |g(x) − g(y)| < ε,
como queríamos. 

Observação. Um resultado semelhante ao citado Teorema de Baire-Kuratowski também apa-


rece no já referido livro Elementos de Topologia Geral, do Elon; Proposição 15 do Capítulo 6,
página 166.

10.14 Exercício 14
Dada f : R → R, tem-se lim f (x) = a se, e somente se, a sequência de funções fn (x) = f (x+n)
x→+∞
converge uniformemente em [0, +∞) para a função constante a.

Solução. Supondo lim f (x) = a, dado ε > 0, seja n0 ∈ N tal que |f (x) − a| < ε para todo
x→+∞
x > n0 ; de |fn (x) − a| = |f (x + n) − a|, temos |fn (x) − a| < ε se x ∈ [0, +∞) e se n > n0 , pois
x + n > n > n0 se x > 0 e se n > n0 ; logo, fn → f uniformemente.
Reciprocamente, dado ε > 0, seja n0 tal que |fn (x) − a| < ε para todo x ∈ [0, +∞) se
n > n0 ; de |f (x) − a| = |f ((x − n0 ) + n0 ) − a| = |fn0 (x − n0 ) − a|, temos |f (x) − a| < ε se
x > n0 , pois x − n0 > 0 se x > n0 ; logo lim f (x) = a. 
x→+∞

10.15 Exercício 15
Se cada função fn : X → R é uniformemente contínua em X e fn → f uniformemente em X
então f é uniformemente contínua em X.

Solução. Seja ε > 0. Queremos encontrar δ>0 tal que

x, y ∈ X, |x − y| < δ ⇒ |f (x) − f (y)| < ε .


O modo como iremos usar as hipóteses dadas se torna bastante claro ao escrevermos, pela
Desigualdade Triangular,

|f (x) − f (y)| 6 |f (x) − fn (x)| + |fn (x) − fn (y)| + |fn (y) − f (y)|,
o que pode ser feito para todo x ∈ X e para todo n ∈ N. Pela convergência uniforme de (fn )
para f , para n ∈ N tal que |f (z) − fn (z)| < ε /3 para todo z ∈ X , escolhemos δ > 0 tal que
|fn (x) − fn (y)| < ε /3 se x, y ∈ X e |x − y| < δ pela continuidade uniforme de fn . Para esta
função fn , temos

|f (x) − f (y)| 6 + |fn (x) − fn (y)|
3
e, se x, y ∈ X são tais que |x − y| < δ , então

2ε ε
|f (x) − f (y)| 6 + = ε,
3 3
como queríamos. 
10.16. EXERCÍCIO 16 421

10.16 Exercício 16
1
Nenhuma sequência de polinômios pode convergir uniformemente para a função ou para
x
1
função sen no intervalo aberto (0, 1).
x

Solução. O exercício anterior pode ser aplicado aqui. Pela mesma justicativa do Exemplo
24 do Capítulo VII, as funções dadas por 1/x e sen(1/x) não são uniformemente contínuas
no intervalo (0, 1) e, através do Exercício 37 do Capítulo VII (página 264), podemos concluir
também que um polinômio dene uma função lipschitziana em (0, 1), portanto, uniformemente
contínua neste intervalo (Exemplo 23 do Capítulo VII), só faltando vericar que uma função
resultante de uma soma de funções uniformemente contínuas ou de um produto de uma função
uniformemente contínua por um número real é uniformemente contínua:

ε ε
|(f + g)(x) − (f + g)(y)| 6 |f (x) − f (y)| + |g(x) − g(y)| < + =ε
2 2
e
ε
|af (x) − af (y)| = |a||f (x) − f (y)| < |a| · =ε
|a|
(se a 6= 0; se a = 0, então af ≡ 0 é uniformemente contínua) se x e y pertencem aos domínios de
f e deg e são tais que |x − y| < min{δ1 , δ2 , δ3 }, onde δ1 , δ2 e δ3 são números reais positivos tais
que |f (x)−f (y)| < ε /2 se |x−y| < δ1 , |g(x)−g(y)| < ε /2 se |x−y| < δ2 e |f (x)−f (y)| < ε /|a|
se |x − y| < δ3 , para um real a não nulo xado a priori. Seguem as teses. 

10.17 Exercício 17
Dada uma sequência simplesmente convergente de funções fn : X → R, seja U o conjunto das
partes U ⊂X tais que fn → f uniformemente em U. Prove:

(a) U 6 = ∅;

(b) Se U ∈U e V ⊂U então V ∈ U;

(c) Se U1 , . . . , Uk ∈ U , então U1 ∪ · · · ∪ Uk ∈ U .

(Voltar para a Solução do Exercício 38: página 439.)

Solução.

item (a) {x} ∈ U ∀x ∈ X ;

item (b) |fn (x) − f (x)| < ε


Se para todo x∈U e se V ⊂ U, então, com maior razão,
|fn (x) − f (x)| < ε para todo x ∈ V ;
(Voltar para a Solução do Exercício 35: página 436.)
422 CAPÍTULO 10. SEQUÊNCIAS E SÉRIES DE FUNÇÕES

item (c) Dado ε > 0, para cada i ∈ {1, . . . , k}, seja ni ∈ N tal que

n > ni ⇒ |fn (x) − f (x)| < ε ∀x ∈ Ui .

Para n0 = max{n1 , . . . , nk }:

n > n0 ⇒ n > ni ∀i ∈ {1, . . . , k}


⇒ |fn (x) − f (x)| < ε ∀x ∈ Ui ∀i ∈ {1, . . . , k}
⇒ |fn (x) − f (x)| < ε ∀x ∈ U1 ∪ · · · ∪ Uk ,

logo U1 ∪ · · · Uk ∈ U . 
(Voltar para a Solução 2 do Exercício 53: página 459.)

Curiosidade/Observações Seja o item (a')  ∅ ∈U e X ∈


/ U . Dado o conjunto não vazio
X , a coleção U ⊂ P(X) com as propriedades (a'), (b) e (c) é chamada de um ideal sobre X . No
exercício acima, de fato, por vacuidade, o conjunto vazio pertence a U , mas pode acontecer de
X também pertencer. Portanto a coleção U do exercício anterior é um ideal sobre X se X 6= ∅
e a convergência fn → f não é uniforme, e U = P(X) se fn → f uniformemente, pelo item (b).

10.18 Exercício 18
Enuncie uma armação equivalente a: a sequência de funções fn : X → R não converge
uniformemente para a função f : X → R, sem usar a palavra não.

Solução. Existe um ε0 positivo tal que, dado um natural n, existem um natural m > n e um
x∈X tais que |fm (x) − f (x)| > ε0 . 

10.19 Exercício 19
Exiba uma sequência de funções fn : [0, 1] → R que convirja uniformemente em (0, 1) mas não
em [0, 1].

Solução. n ∈ N, fn (x) = 0 para todo x ∈ [0, 1) e fn (1) = n; (fn )n∈N converge


Para cada
uniformemente em (0, 1) (para a função identicamente nula), mas (fn )n∈N não converge uni-
formemente em [0, 1], porque a sequência (fn (1))n∈N = (n)n∈N não é convergente (convergência
uniforme implica convergência simples). 

10.20 Exercício 20
Se fn → f X , com fn (X) ⊂ Y para todo n e f (X) ⊂ Y então g ◦ fn → g ◦ f
uniformemente em
uniformemente em X desde que g : Y → R seja uniformemente contínua. Enuncie e prove um
resultado análogo para gn ◦ f → g ◦ f . Foi preciso supor algo sobre f ?
10.21. EXERCÍCIO 21 423

Solução. Dado ε > 0, queremos n0 ∈ N tal que

n > n0 ⇒ |g(fn (x)) − g(f (x))| < ε ∀x ∈ X.

Com respeito a este ε, pela continuidade uniforme de g, existe δ>0 tal que

x, y ∈ Y, |x − y| < δ ⇒ |g(x) − g(y)| < ε

e, com relação a este δ > 0, pela convergência uniforme fn → f , existe n0 ∈ N tal que

n > n0 ⇒ |fn (x) − f (x)| < δ ∀x ∈ X.

Portanto (como fn (x), f (x) ∈ Y ∀n ∈ N ∀x ∈ X ):

n > n0 ⇒ |fn (x) − f (x)| < δ ∀x ∈ X


⇒ |g(fn (x)) − g(f (x))| < ε ∀x ∈ X,

como queríamos.
Sobre a convergência gn ◦ f → g ◦ f , queremos investigar quando vale uma desigualdade do
tipo
|gn (f (x)) − g(f (x))| < ε
qualquer que seja o x. Sendo Y o domínio das gn 's e da g , se a convergência gn → g for
uniforme, então |gn (x) − g(x)| < ε para todo x ∈ Y , a partir de um certo n0 , . Em particular,
se f (x) ∈ Y para todo x de seu domínio, então |gn (f (x)) − g(f (x))| < ε para todo x no domínio
de f , a partir desse mesmo n0 . Deste modo, enunciamos:

Dado Y ⊂ R não vazio, seja g : Y → R e, para cada n ∈ N, sejam as funções


gn : Y → R. Dado X ⊂ R não vazio, seja também a função f : X → R tal que
f [X] ⊂ Y . Se gn → g uniformemente, então gn ◦ f → g ◦ f uniformemente.

Sobre f foi preciso supor que sua imagem estava contida no domínio das gn 's e da g. 

10.21 Exercício 21
Se fn → f uniformemente em X , gn → g uniformemente em Y , f (X) ⊂ Y , fn (X) ⊂ Y para
todo n e g é uniformemente contínua em Y , então gn ◦ gn → g ◦ f [sic ] uniformemente em X .

Solução. Esta tese independente de qualquer uma das fn 's, as quais fazem parte das hipóteses,
causam estranheza. De fato, que gn ◦ gn converge para g ◦ f é falso. Por exemplo: a sequência
das gn 's, gn (x) = x − 1/n ∀x ∈ R ∀n ∈ N, converge uniformemente para a função
denidas por
uniformemente contínua g , denida por g(x) = x ∀x ∈ R, assim como a sequência das compostas
gn ◦ gn 's, (gn ◦ gn )(x) = x − 2/n, de modo que, se f é qualquer função diferente da função
identidade (e uma função diferente da identidade pode ser limite uniforme de uma sequência),
a sequência (gn ◦ gn ) não converge para g ◦ f . De modo mais geral, após a demonstração abaixo,
poderemos concluir que gn ◦ gn → g ◦ g uniformemente (se gn [Y ] ⊂ Y e g[Y ] ⊂ Y ).
Vamos mostrar que gn ◦ fn → g ◦ f uniformemente, isto é, que, dado ε > 0, existe n0 ∈ N
tal que
n > n0 ⇒ |gn (fn (x)) − g(f (x))| < ε ∀x ∈ X.
424 CAPÍTULO 10. SEQUÊNCIAS E SÉRIES DE FUNÇÕES

Escrevendo

|gn (fn (x)) − g(f (x))| 6 |gn (fn (x)) − g(fn (x))| + |g(fn (x)) − g(f (x))|,

podemos ver um modo de aplicar o exercício anterior aqui (bem como a ideia vista em sua
solução). Pela convergência uniforme gn → g , seja n1 ∈ N tal que

ε
|gn (x) − g(x)| < ∀x ∈ Y
2
se n > n1 . Como fn (x) ∈ Y ∀x ∈ X ∀n ∈ N, também podemos escrever

ε
|gn (fn (x)) − g(fn (x))| < ∀x ∈ X
2
se n > n1 . E, porque fn → f uniformemente e g é uniformemente contínua, seja n2 ∈ N tal que
ε
n > n2 ⇒ |g(fn (x)) − g(f (x))| < ∀x ∈ X
2
(exercício anterior). Logo, denindo n0 = max{n1 , n2 },
ε ε
n > n0 ⇒ |gn (fn (x)) − g(f (x))| < + =ε ∀x ∈ X,
2 2
como queríamos. 

10.22 Exercício 22
Uma sequência monótona de funções é uniformemente convergente desde que possua uma sub-
sequência com esta propriedade.

Solução 1. No livro não há a denição de sequência monótona de funções. O que há é a


denição de convergência monótona de funções (página 375). A expressão monotonicidade
da sequência (fn ) até foi usada, mas de modo informal (ou seja, sem denição) dentro da
demonstração do Teorema 5.
Por sequência monótona de funções vamos entender uma sequência (fn )n∈N de funções fn :
X → R tal que (fn (x))n∈N é monótona para todo x ∈ X (uma outra opção seria denir
(fn )n∈N como, por exemplo, não decrescente caso m 6 n implique fm (x) 6 fn (x) ∀x ∈ X ;
a monotonicidade neste sentido implica a monotonicidade de (fn (x))n∈N ∀x ∈ X , então esta
opção ca contemplada pela escolha feita anteriormente).
(fn )n∈N uma sequência monótona de funções, todas com um mesmo domínio X ⊂
Seja
R (então (fn (x))n∈N é monótona para todo x ∈ X ), e seja (fn )n∈M (M ⊂ N innito) sua
subsequência que converge uniformemente para f : X → R.
Fixado ε > 0, seja n0 ∈ M tal que n ∈ M ∩ [n0 , +∞) ⇒ |fn (x) − f (x)| < ε ∀x ∈ X . Em
outras palavras, denindo Ix = (f (x) − ε, f (x) + ε) para cada x ∈ X , temos

n ∈ M ∩ [n0 , +∞) ⇒ fn (x) ∈ Ix ∀x ∈ X.

Dado n ∈ N ∩[n0 , +∞) e dado x ∈ X , seja m ∈ M maior do que ou igual a n e teremos:


fn0 (x) 6 fn (x) 6 fm (x) ou fm (x) 6 fn (x) 6 fn0 (x), onde fn0 (x) e fm (x) são elementos do
intervalo Ix . Portanto, fn (x) também é elemento de Ix (Exercício 23 do Capítulo V, página 174).
Logo, n ∈ N ∩[n0 , +∞) ⇒ fn (x) ∈ Ix ∀x ∈ X , ou seja: n ∈ N ∩[n0 , +∞) ⇒ |fn (x) − f (x)| < ε
∀x ∈ X . 
10.23. EXERCÍCIO 23 425

Solução 2. (fn )n∈N uma sequência de funções de X ⊂ R em R. Vamos supor (fn (x))n∈N
Seja
monótona para todo x ∈ X e a existência de um M ⊂ N innito tal que a subsequência (fn )n∈M
convirja uniformemente. Seja M um tal conjunto.
Dado ε > 0, seja m0 ∈ M tal que

m, n ∈ M ∩ [m0 , +∞) ⇒ |fm (x) − fn (x)| < ε ∀x ∈ X

(Teorema 1). Se m0 < m < n, seja n0 ∈ M maior do que n. Então, dado x ∈ X, temos

fm0 6 fm (x) 6 fn (x) 6 fn0 (x) ou fn0 (x) 6 fn (x) 6 fm (x) 6 fm0 (x).

De qualquer forma,
|fm (x) − fn (x)| 6 |fm0 (x) − fn0 (x)| < ε .
Logo, (fn )n∈N converge uniformemente (Teorema 1). 

10.23 Exercício 23
Se uma sequência de funções contínuas fn : X → R converge uniformemente para f :X→R
e x n → a, onde a∈X e cada xn ∈ X , então lim fn (xn ) = f (a).

Solução. Dado ε > 0, queremos n0 ∈ N tal que |fn (xn ) − f (a)| < ε para todo natural n > n0 .
Para podermos usar a convergência uniforme de (fn ) para f , escrevemos

|fn (xn ) − f (a)| 6 |fn (xn ) − f (xn )| + |f (xn ) − f (a)| ∀n ∈ N .

Nesta desigualdade podemos ver que a continuidade de f também seria útil para os nossos
propósitos. E, de fato, o Teorema 4 nos garante isto. Então, por este teorema (o 4 deste
capítulo) e também pelo Teorema 4 do Capítulo VII, seja n1 ∈ N tal que

ε
n > n1 ⇒ |f (xn ) − f (a)| <
2
e seja n2 ∈ N tal que
ε
n > n2 ⇒ |fn (x) − f (x)| < ∀x ∈ X,
2
em particular,
ε
n > n2 ⇒ |fn (xn ) − f (xn )| < .
2
Assim, para n0 = max{n1 , n2 } temos

n > n0 ⇒ n > n1 e n > n2


ε ε
⇒ |f (xn ) − f (a)| < e |fn (xn ) − f (xn )| <
2 2
ε ε
⇒ |fn (xn ) − f (a)| < + = ε,
2 2
como queríamos. 

10.24 Exercício 24
Se uma sequência de funções contínuas fn : X → R é tal que xn ∈ X , lim xn = a ∈ X implica
lim fn (xn ) = f (a) então fn → f uniformemente em cada parte compacta de X.
426 CAPÍTULO 10. SEQUÊNCIAS E SÉRIES DE FUNÇÕES

Solução.

Nível 1. Armamos que a sequência (fn )n∈N f (Nível 2.1)


converge simplesmente para
1
e é equicontínua (Nível 2.2). Assim, pelo Teorema 20 (X é denso em X ) , (fn )n∈N converge
uniformemente em cada parte compacta de X , convergência esta que só pode ser para f , uma
vez que fn → f simplesmente.

No elevador. Vamos trabalhar com um número do tipo |fn (a) − f (a)|. Para usar uma
das informações dadas, escrevemos |fn (a) − f (a)| 6 |fn (a) − fn (xn )| + |fn (xn ) − f (a)|, onde
x n → a. Desigualdade esta cuja segunda parcela de seu lado direito pode ser tornada tão
pequena quanto se queira para n sucientemente grande, o problema é com a primeira parcela:
para cada n até temos lim fn (xm ) = fn (a) por causa da continuidade de fn , mas não sabemos
m→+∞
se o m que torna|fn (xm ) − fn (a)| pequeno pode ser tomado igual a n. Isto pode ser corrigido
obtendo-se, a partir de (xn ), uma sequência (yn ) tal que |fn (yn ) − fn (a)| < ε /2 para todo n.

Nível 2.1. Seja a ∈ X. Dado ε > 0, queremos um natural n0 tal que

n > n0 ⇒ |fn (a) − f (a)| < ε .


Seja(xn ) uma sequência em X convergente para a e sejam m1 < m2 < · · · < mi < · · · tais
que |fi (xmi ) − fi (a)| < ε /2 para todo i ∈ N (por exemplo, m1 = min{m ∈ N; |f1 (xm ) − f1 (a)| <
ε /2} e mi+1 = min{m ∈ N; m > mi e |fi+1 (xm ) − fi+1 (a)| < ε /2} ∀i ∈ N). Estes mi 's existem
por causa da continuidade de cada fi (em combinação com o Teorema 4 do Capítulo VII). Dado
i ∈ N, seja yi = xmi . Isto nos dá a sequência (yn ), uma subsequência de (xn )  e, portanto,
yn → a, o que, por sua vez, implica fn (yn ) → a  satisfazendo |fn (yn ) − fn (a)| < ε /2 para todo
n. De
ε
|fn (a) − f (a)| 6 |fn (a) − fn (yn )| + |fn (yn ) − f (a)| < + |fn (yn ) − f (a)|,
2
para todo n, e de fn (yn ) → f (a), seja n0 ∈ N tal que |fn (yn ) − f (a)| < ε /2 se n > n0 , de modo
que
ε ε ε
n > n0 ⇒ |fn (a) − f (a)| < + |fn (yn ) − f (a)| < + = ε,
2 2 2
como queríamos.

Observação. (yn ) aplicando o Exercício 24 do Capítulo


Também podemos obter a sequência
IV (página 139), denindo, para cada n ∈ N, yn = xm , sendo m > n tal que |fn (xm ) − f (a)| <
ε /2. Então temos m = ϕ(n), onde ϕ(n) > n implica lim ϕ(n) = +∞, portanto uma tal ϕ
n→+∞
está dentro das condições do Exercício 23 do Capítulo IV. Pode ser que uma tal (yn ) não seja
subsequência de (xn ), mas, pelo Exercício 24 do Capítulo IV, ainda temos yn → a, com yn ∈ X
para todo n ∈ N.

Nível 2.2. Vamos trabalhar com a contrapositiva da armação em questão. Se (fn ) não é
equicontínua em X , seja a ∈ X um ponto no qual (fn ) não é equicontínua. Assim, seja ε0 > 0
e, para cada m ∈ N, sejam xm ∈ X e n(m) ∈ N tais que
1
|xm − a| < e |fn(m) (xm ) − fn(m) (a)| > ε0 .
m
1 Cuidado com a expressão X é denso em si mesmo, ela pode signicar X ⊂ X 0, o que é falso se X possuir
um ponto isolado.
10.25. EXERCÍCIO 25 427

A sequência (xm ) assim obtida converge para a. Agora, considerando o conjunto n[N] =
{n(m) ∈ N; m ∈ N}, temos duas possibilidades: ou n[N] é innito, ou n[N] é nito. No Nível
3.1, vamos trabalhar com o caso em que n[N] é innito e, no Nível 3.2, com o caso em que n[N]
é nito. No primeiro deles, com uma hipótese adicional, vamos obter uma sequência (ym ) em
X tal que ym → a mas fm (ym ) 6→ f (a). Sem essa hipótese adicional, cairemos na negação do
Nível 2.1. No segundo deles, vamos obter m∈N tal que fm não é contínua. Destas armações
segue a tese.

No elevador. A sequência desejada vem da própria (xm ); a ideia é que (fn(m) (xm ))m∈N
não converge para f (a), se fn(m) (a) → f (a). O primeiro cuidado que devemos ter é o de
organizar o conjunto n[N].

Nível 3.1. Sejam m1 < m2 < · · · < mi < · · · tais que n(m1 ) < n(m2 ) < · · · < n(mi ) <
··· (por exemplo, m1 = 1 e mi+1 = min{m ∈ N; m > mi e n(m) > n(mi )} para cada i ∈ N).
De

|fn(mi ) (xmi ) − f (a)| > |fn(mi ) (xmi ) − fn(mi ) (a)| − |fn(mi ) (a) − f (a)| ∀i ∈ N

(item (iii) do Teorema 2 do Capítulo III), vamos supor lim fn(mi ) (a) = f (a) (se lim fn(mi ) (a) 6=
i→+∞ i→+∞
f (a), então (fn ) não converge simplesmente para f e o resultado segue pela contrapositiva do
Nível 2.1); daí
ε0
|fn(mi ) (xmi ) − f (a)| > ,
2
se i > i0 , sendo i0 ∈ N tal que

ε0
i > i0 ⇒ |fn(mi ) (a) − f (a)| < ,
2

pois |fn(mi ) (xmi ) − fn(mi ) (a)| > ε0 para todo i ∈ N, por construção.
Logo, a sequência (fn(mi ) (xmi ))i∈N não converge para f (a), enquanto lim xmi = a. Deste
i→+∞
modo, denimos yn(mi ) = xmi , yp = yn(mi ) se p ∈ N ∩[n(mi ), n(mi+1 )) para todo i ∈ N e
yp = xn(m1 ) se p ∈ N ∩[1, n(m1 )]. A sequência (ym )m∈N possui as propriedades desejadas:
lim ym = a e (fm (ym ))m∈N não converge para f (a), como queríamos.

Nível 3.2. n[N] é nito, seja n[N] = {a1 , . . . , ak }. Para cada i ∈ {1, . . . , k}, seja
Se
Ni = {m ∈ N; n(m) = ai }. Da igualdade N = N1 ∪ · · · ∪ Nk , seja i ∈ {1, . . . , k} tal que Ni é
innito. Então o m tal que fm não é contínua é m = ai , pois de Ni podemos extrair os números
m1 < m2 < · · · mj < · · · , de tal modo que (xmj )j∈N é uma sequência que converge para a, mas
|fai (xmj ) − fai (a)| > ε0 para todo j ∈ N, ou seja, (fai (xmj ))j∈N não converge para fai (a), logo
fai não é contínua (Teorema 4 do Capítulo VII). 

10.25 Exercício 25
Se uma sequência de funções monótonas converge simplesmente para uma função contínua num
intervalo I, então a convergência é uniforme em cada parte compacta de I.
428 CAPÍTULO 10. SEQUÊNCIAS E SÉRIES DE FUNÇÕES

Solução. Como I é um intervalo, notamos que é suciente mostrar a convergência uniforme


em conjuntos da forma [a, b], com a, b ∈ I , pois, se K ⊂ I é compacto e I é um intervalo,
então K ⊂ [inf K, sup K] ⊂ I e, se fn → f uniformemente em [inf K, sup K], então fn → f
uniformemente em K .
Fixado então o intervalo compacto [a, b] ⊂ I , pela continuidade de f , sabemos obter pontos
a = a0 < a1 < · · · < ak−1 < ak = b tais que |f (x) − f (y)| seja tão pequeno quanto se queira se
x, y ∈ [ai−1 , ai ] para i ∈ N ∩[1, k] (Exercício 43 do Capítulo VII; página 269). Vamos usar estes
pontos para nos auxiliar em nossas contas; as desigualdades abaixo vão nos dizer de que modo
eles devem ser escolhidos:

|fn (x) − f (x)| 6 |fn (x) − fn (ai )| + |fn (ai ) − f (x)|,


onde
|fn (ai ) − f (x)| 6 |fn (ai ) − f (ai )| + |f (ai ) − f (x)|
e, se x ∈ [ai , ai+1 ] (lembrando da monotonicidade de fn ),
|fn (x) − fn (ai )| 6 |fn (ai ) − fn (ai+1 )|
6 |fn (ai ) − f (ai )| + |f (ai ) − f (ai+1 )| + |f (ai+1 ) − fn (ai+1 )|.
Portanto, se x ∈ [ai , ai+1 ],
|fn (x) − f (x)| 6 |fn (ai ) − f (ai )| + |f (ai ) − f (ai+1 )| + |f (ai+1 ) − fn (ai+1 )|+
+ |fn (ai ) − f (ai )| + |f (ai ) − f (x)|.
Assim, dado ε > 0, escolhemos a = a0 < a1 < . . . < ak−1 < ak = b tais que, para cada
i ∈ {0, 1, . . . , k − 1},
ε
x, y ∈ [ai , ai+1 ] ⇒ |f (x) − f (y)| < .
5
Tendo esses ai 's, para cada i ∈ {0, 1, . . . , k}, denimos ni como aquele natural tal que
ε
n > ni ⇒ |fn (ai ) − f (ai )| < .
5
Se x ∈ [a, b], então existe i ∈ {0, 1, . . . , k − 1} tal que x ∈ [ai , ai+1 ] e, se n > max{n0 , n1 , . . . ,
nk }, então
ε
|fn (x) − f (x)| 6 5 · = ε .
5
Logo, fn → f uniformemente em [a, b], como queríamos. 

10.26 Exercício 26
P
Se a série fn converge uniformemente em X então fn → 0 uniformemente no mesmo conjunto.
(Voltar para a Solução 1 do Exercício 39: página 439.)

Solução 1. Podemos seguir a mesma ideia da demonstração de seu análogo com números reais,
a saber, a do Teorema 15 do Capítulo IV. Indo por este caminho, por denição, a sequência
de funções (sn )n∈N , dada por sn = f1 + · · · + fn ∀n ∈ N, é uniformemente convergente; e ela
satisfaz sn+1 − sn = fn+1 . Pelo Teorema 1, dado ε > 0, existe n0 ∈ N tal que
n > n0 ⇒ |sn+1 (x) − sn (x)| < ε ∀x ∈ X,
ou seja,
n > n0 ⇒ |fn+1 (x)| < ε ∀x ∈ X,
logo, fn → 0 uniformemente. 
10.27. EXERCÍCIO 27 429

Solução 2 (esta sim está mais parecida com a Demonstração do Teorema 15 do


Capítulo IV). Assim como na solução anterior, seja (sn )n∈N , denida por sn = f 1 + · · · + f n
∀n ∈ N. Vamos denotar por lim gn o limite uniforme da sequência uniformemente convergente
(gn )n∈N . Seja s = lim sn . A função s é também s = lim sn−1 , assim

0 = s − s = lim sn − lim sn−1 = lim(sn − sn−1 ) = lim fn ,

pelo Exercício 7 (página 412) e porque lim(−gn ) = −g se g = lim gn (pois |−gn (x)−(−g(x))| =
|gn (x) − g(x)|). 

10.27 Exercício 27
fn : I → R, todas denidas no mesmo intervalo I , é tal que,
Suponha que a sequência de funções
para cada x ∈ I , apenas um número nito de valores fn (x), n ∈ N, é diferente de zero. Prove
P
que
P fn é absolutamente convergente em I . Dê um exemplo em que cada fn seja contínua
mas fn não convirja uniformemente.

Solução. Dado x ∈ I, seja {n ∈ N; fn (x) 6= 0} = {n1 , n2 , . . . , nk }; então

X
|fn (x)| = |fn1 (x)| + |fn2 (x)| + · · · + |fnk (x)|,
n

X
portanto, a série |fn (x)| converge qualquer que seja x ∈ I .
n
Como exemplo de uma sequência (fn ) de funções contínuas (tal que {n ∈ N; fn (x) 6= 0} é
nito para todo x de seus domínios) cuja soma não converge uniformemente, denimos, para
cada n ∈ N, fn : R → R, fazendo fn (x) = x − n se x > n e fn (x) = 0 se x < n. Cada P fn é
contínua e, dado x ∈ R, fn (x) = 0 para todo n > x. Pelo exercício anterior, a série fn não
converge uniformemente porque (fn ) não converge uniformemente para zero.
Seguindo a mesma ideia do parágrafo anterior, denindo essas funções num conjunto com-
pacto, pelo Corolário 1 do Teorema 5, podemos dizer mais acerca de sua soma, a saber, que
ela não é contínua. Por exemplo, denindo no intervalo [a, b] a função fn como aquela cujo
gráco é a junção do segmento de reta cujos extremos são os pares ordenados (a, 0) e (bn , 0)
com o segmento de reta cujos extremos são os pares ordenados (bn , 0) e (b, 1), onde (bn ) é uma
sequência estritamente crescente em [a, b] que converge para b. 

10.28 Exercício 28
P
Se |gn | converge P
uniformemente em X e existe k > 0 tal que |fn (x)| 6 k para todo n∈N e
todo x ∈ X , então fn gn converge absoluta e uniformemente em X .

P
Solução. Vamos mostrar que
P |fn gn | converge uniformemente (donde também segue a con-
vergência uniforme de fn gn ). Com o Critério de Cauchy (Teorema 1) em mente, escrevemos


|fn (x)gn (x)| + · · · + |fn+p (x)gn+p (x)| = |fn (x)| · |gn (x)| + · · · + |fn+p (x)| · |gn+p (x)|

6 k · |gn (x)| + · · · + |gn+p (x)| .
430 CAPÍTULO 10. SEQUÊNCIAS E SÉRIES DE FUNÇÕES

ε > 0, tomamos n0 ∈ N tal que n > n0 implique |gn (x)| + · · · + |gn+p (x)| < ε /k
Assim, dado
para todo p ∈ N, qualquer P que seja x ∈ X . Este n0 existe pelo Critério de Cauchy e pela
convergência uniforme de |gn |. Deste modo
ε
n > n0 ⇒ |fn (x)gn (x)| + · · · + |fn+p (x)gn+p (x)| < k · =ε ∀p ∈ N ∀x ∈ X,
k
P
logo, |fn gn | converge uniformemente. 

10.29 Exercício 29
São dadas as sequências de funções fn , gn : X → R com as seguintes propriedades: existe uma
constante k > 0 tal que as reduzidas sn = f1 + · · · + fn satisfazem |sn (x)| 6 k para todo
n ∈ N e todo x ∈ X . Além disso, as funções gn decrescem uniformemente para zero, isto é,
gn (x) 6 gn+1 (x) [sic ] para todo
Pn ∈ N e todo x ∈ X e lim gn = 0 uniformemente em X . Nestas
condições, prove que a série fn · gn é uniformemente convergente em X . [Sugestão : use o
método de demonstração do Teorema 21, Capítulo IV, e o exercício anterior.]

Solução. Entendemos que o exercício quis dizer gn (x) > gn+1 (x) ∀n ∈ N e ∀x ∈ X . Seguindo
a sugestão, escrevemos

n
X
f1 (x)g1 (x) + · · · + fn (x)gn (x) = si−1 (x) · (gi−1 (x) − gi (x)) + sn (x)gn (x),
i=2

n
!
X P
onde: si−1 (gi−1 − gi ) converge uniformemente, pelo exercício anterior (se |hn | con-
i=2 n>2
P
verge uniformemente, então hn também converge uniformemente), pois, além de existir k tal
Xn Xn
que |sn (x)| < k para todox ∈X e todo n ∈ N, |gi−1 − gi | = gi−1 − gi = g1 − gn converge
i=2 i=2
uniformemente para g1 ; sn gn converge uniformemente para zero, pois |sn (x)gn (x)| 6 k · |gn (x)|
para todo x ∈ X, e; soma de sequências uniformemente convergentes é uniformemente conver-
P
gente. Logo, a sequência das reduzidas da série
P f n · gn é uniformemente convergente, isto é,
f n · gn é uniformemente convergente. 

10.30 Exercício 30
P sen(nx) P cos(nx)
Para todo ε > 0, as séries e convergem uniformemente no intervalo
n n
[ε, 2π − ε].

Solução. Isto segue do exercício anterior e do Exemplo 37 do Capítulo IV: as funções gn 's,
dadas por gn (x) = 1/n para todo x e para todo n, decrescem uniformemente para zero, e; se
x ∈ [ε, 2π − ε], então x não é múltiplo de 2π , de modo que existem k e l positivos tais que
| sen(x) + sen(2x) + · · · sen(nx)| 6 k e | cos(x) + cos(2x) + · · · + cos(nx)| 6 l para todo x e para
todo n. 
10.31. EXERCÍCIO 31 431

10.31 Exercício 31

X
Se a série de Fourier f (x) = a0 +
(an cos nx+bn sen nx) converge uniformemente no intervalo
n=1
[−π, π], prove que valem as relações de Euler:
Z π
1 π
Z
1
a0 = f (x) dx, am = f (x) cos mx dx,
2πZ −π π −π
1 π
bm = f (x) sen mx dx, m > 1.
π −π

Solução. m, vamos integrar no intervalo [−π, π] cada uma das três


Fixado o número natural
funções dadas por f (x), f (x) cos mx e f (x) sen mx usando o Corolário do Teorema 6.
Para tanto, observemos que também são uniformemente convergentes em [−π, π] as séries


X
f (x) cos mx = a0 cos mx + (an cos mx cos nx + bn cos mx sen nx)
n=1

e

X
f (x) sen mx = a0 sen mx + (an sen mx cos nx + bn sen mx sen nx)
n=1

(estas multiplicações pelo Exemplo 25 do Capítulo IV), pois, considerando o resto

n
X
rn (x) = f (x) − (ak cos kx + bk sen kx),
k=1

temos |rn (x) cos mx| = |rn (x)| · | cos mx| 6 |rn (x)| e |rn (x) sen mx| = |rn (x)| · | sen mx| 6 |rn (x)|
(observação sobre convergências uniformes de séries da página 369).
Usando (quando necessário) as igualdades

1 
cos a · cos b = cos(a + b) + cos(a − b) ,
2
1 
cos a · sen b = sen(a + b) + sen(b − a)
2
e
1 
sen a · sen b = cos(a − b) − cos(a + b) ,
2
temos:
Z π Z π Z π Z π
a0 dx = 2πa0 , a0 cos mx dx = a0 sen mx dx = an cos nx + bn sen nx dx = 0,
−π −π −π −π

Z π Z π Z π 
an
an cos mx cos nx + bn cos mx sen nx dx = cos(m + n)x dx + cos(m − n)x dx
−π 2 −π −π
Z π Z π 
bn
+ sen(m + n)x dx + sen(n − m)x dx
2 −π −π
= 0
432 CAPÍTULO 10. SEQUÊNCIAS E SÉRIES DE FUNÇÕES

(se m 6= n),
Z π Z π Z π 
an
an sen mx cos nx + bn sen mx sen nx dx = sen(m + n)x dx + sen(m − n)x dx
−π 2 −π −π
Z π Z π 
bn
+ cos(m − n)x dx − cos(m + n)x dx
2 −π −π
= 0

(se m 6= n),
Z π Z π
am
am cos mx cos mx + bm cos mx sen mx dx = 1 dx = am π
−π 2 −π

e Z π Z π
bm
am sen mx cos mx + bm sen mx sen mx dx = 1 dx = bm π.
−π 2 −π

Portanto,

Z π Z π Z π
f (x) dx = 2πa0 , f (x) cos mx dx = am π e f (x) sen mx dx = bm π,
−π −π −π

ou seja,

Z π Z π Z π
1 1 1
a0 = f (x) dx, am = f (x) cos mx dx e bm = f (x) sen mx dx,
2π −π π −π π −π

como queríamos. 

10.32 Exercício 32
Para cada n ∈ N, seja fn : [0, 1] → R denida por fn (x) = xn+1 − xn . Prove que lim fn (x) = 0
x→1
uniformemente em relação a n.

Solução 1. Dado ε > 0, queremos δ>0 tal que

x ∈ [0, 1], 0 < |x − 1| < δ ⇒ |fn (x)| < ε ∀n ∈ N .

Temos
|fn (x)| = |xn+1 − xn | = xn · |x − 1| 6 |x − 1|,
pois 0 6 xn 6 1 para todo x ∈ [0, 1] e todo n ∈ N. Logo, para ter o que queremos, basta fazer
δ = ε. 

Solução 2. Essa sequência tem uma espécie de monotonicidade uniforme. Mais precisa-
mente, vale fn+1 (x) > fn (x) para todo x ∈ [0, 1] e todo n ∈ N. Em isso sendo verdade, como
n+1
x − x = xn (x − 1) 6 0 para todo x ∈ [0, 1] e todo n ∈ N, o delta que mostra o resultado
n

desejado é o mesmo delta que mostra que lim f1 (x) = 0. Com efeito, dado ε > 0, se 0 < δ < 1
x→1
é tal que
1 − δ < x < 1 ⇒ − ε < f1 (x) < ε
10.33. EXERCÍCIO 33 433

e se f1 (x) 6 fn (x) 6 0 para todo x ∈ [0, 1], então

1 − δ < x < 1 ⇒ − ε < f1 (x) 6 fn (x) 6 0 < ε .

Então só falta vericarmos a monotonicidade mencionada:

xn+2 − xn+1 > xn+1 − xn ,

xn+2 − 2xn+1 + xn > 0,


xn (x2 − 2x + 1) > 0,
xn (x − 1)2 > 0
e de fato xn (x − 1)2 > 0 para todo x ∈ [0, 1] e todo n ∈ N. 

Solução 3. Como fn (1) = 0 ∀n ∈ N 1 ∈ [0, 1]),


a expressão  lim fn (x) = 0 uniforme-
(e
x→1
mente em relação a n signica que (fn )n∈N é equicontínua em 1. Essa sequência converge
uniformemente para zero, porque fn (x) 6 0 ∀x ∈ [0, 1] ∀n ∈ N e o valor mínimo

   n    n
n n n 1 n
fn = · −1 =− ,
n+1 n+1 n+1 n+1 n+1

de cada fn , converge para zero (porque a sequência ((n/(n+1))n )n∈N é limitada e −1/(n+1) →
0).
Assim, pelo Exemplo 24, como cada fn é contínua, o conjunto {f, f1 , . . . , fn , . . .} (para
f ≡ 0) é equicontínuo. Em particular, é também equicontínua a sequência (fn )n∈N . Portanto,
podemos concluir algo mais forte do que a tese do exercício, a saber, que

lim fn (x) = an+1 − an uniformemente em relação a n ∀a ∈ [0, 1]


x→a

(donde o resultado segue fazendo a = 1). 

10.33 Exercício 33

X sen(nx) R π/2
Seja f :R→R denida por f (x) = . Determine
0
f (x) dx e f 0 (π).
n=1
n2

Solução. Podemos integrar e derivar termo a termo (respectivamente pelo Corolário do Te-
orema 6 junto com o Exemplo 10 e pelo Corolário 1 do Teorema 7 junto com o Exercício 30
acima, por exemplo, para ε = π/2):
Z π/2 ∞ Z π/2
X sen(nx)
f (x) dx = dx,
0 n=1 0 n2

onde
π/2 π/2
− cos(nx)
Z
sen(nx) cos 0 cos(nπ/2) 1 cos(nπ/2)
2
dx = = − = 3−
0 n n3 0 n3 n 3 n n3
434 CAPÍTULO 10. SEQUÊNCIAS E SÉRIES DE FUNÇÕES

então,

Z π/2    
1 1 1 1 1
f (x) dx = 1+ + + 3+ − + ···
0 23 23 3 43 43
X 1 X 1 X 1
= + −
n3 (4n − 2)3 (4n)3
X 1 1X 1 1 X 1
= + −
n3 8 (2n − 1)3 64 n3
63 X 1 1X 1
= 3
+
64 n 8 (2n − 1)3
 X 1 
63 X 1 1 X 1
= + −
64 n3 8 n3 (2n)3
 
63 X 1 1 X 1 1X 1
= + −
64 n3 8 n3 8 n3
63 X 1 1 7X 1
= + ·
64 n3 8 8 n3
70 X 1
=
64 n3
(Exemplos 25, 29 e 39, Critério da Comparação, associatividade das séries convergentes,
página 148, todos do Capítulo IV), e;

∞  0 ∞
0
X sen(nx) X cos(nx)
f (x) = = ,
n=1
n2 n=1
n

donde
∞ ∞ ∞
0
X cos(nπ) X
n1
X 1
f (π) = = (−1) =− (−1)n−1 = − log 2,
n=1
n n=1
n n=1
n

pelo Exemplo 10 do Capítulo IX (junto com o Exemplo 25 do Capítulo IV). 

Observações.
R 2π
• Integrando termo a termo, encontramos
0
f (x) dx = 0 (a mesma f do exercício acima).
Outra forma de fazer essa conta é usando o Exercício 31, através de mudança de variáveis:

Z 2π Z π
f (x) dx = f (x + π) dx
0 −π

que é o produto de 2π pelo coeciente livre da série de Fourier f (x + π), que é zero,
pois seu termo geral é

sen(nx + nπ) sen(nx) cos(nπ) + sen(nπ) cos(nx) (−1)n


= = · sen(nx).
n2 n2 n2
Disso também podemos concluir que

π π
(−1)n π
Z Z
f (x + π) cos nx dx = 0 e f (x + π) sen nx dx = ∀n ∈ N;
−π −π n2
10.34. EXERCÍCIO 34 435

• Seguindo os mesmos passos da solução do exercício acima, podemos calcular também


Z π X∞ ∞
Z π/2 X
sen(nx) sen(nx)
dx (Exercício 30) e 3
dx (Teste de Weierstrass). Sabendo
π/2 n=1 n 0 n=1
n
∞ ∞
X 1 π2 X 1 π4
que
2
= e que
4
= , aquela solução nos dará racionais A e B tais que
n=1
n 6 n=1
n 90

π ∞
π2
Z X sen(nx)
dx = A ·
π/2 n=1 n 6
e

π/2 X
π4
Z
sen(nx)
dx = B · ;
0 n=1
n3 90

X 1 π2
• O valor = pode ser encontrado através de resultados acerca das séries de
n=1
n2 6
Fourier. Começamos considerando a função g dada por g(x) = x2 e calculamos a sua
série de Fourier (uma série cujos coecientes são dados pelo Exercício 31), o que vai dar

π2 X (−1)n
+4 · cos(nx).
3 n2
π2 X 1
Essa série, calculada em x = π , dá +4 . Se a série de Fourier acima for a própria
3 n2
π2 X 1
g (e, no intervalo [−π, π], de fato ela é), então +4 = π 2 , donde vem o valor
3 n2
X 1 π2
= ;
n2 6

1
função zeta de Riemann.
X
• A função ζ, dada por ζ(x) = , é conhecida como
n=1
nx

10.34 Exercício 34
Dê exemplo de uma sequência de funções deriváveis fn : [a, b] → R tais que (fn0 ) seja uniforme-
mente convergente, mas (fn ) não convirja num único ponto de [a, b].

Solução. n ∈ N, fn : [a, b] → R denida por fn (x) = (−1)n para todo x ∈ [a, b];
Para cada
fn0≡ 0 para todo n ∈ N, e; (fn (x))n∈N = ((−1)n )n∈N não converge, não importa quem seja
x ∈ [a, b]. (Esta sequência (fn )n∈N também não converge uniformemente.) 

Observação. Então, este Exercício mostra a necessidade da hipótese da existência de um


c ∈ [a, b] tal que (fn (c))n∈N convirja, no Teorema 7.

10.35 Exercício 35
Se existe lim fn (c) = L e (fn0 ) converge uniformemente para zero em I , então fn → L, unifor-
n x
memente em cada compacto K ⊂ I. Aplique este fato a fn (x) = sen .
n
436 CAPÍTULO 10. SEQUÊNCIAS E SÉRIES DE FUNÇÕES

Solução. Entendemos que este I é um intervalo, pois, se não for, este resultado é falso, por
exemplo: para I = [1, 2] ∪ [3, 4] e para as fn 's denidas porfn [[1, 2]] = {1} e fn [[3, 4]] = {2}
para todo n, temos fn0 ≡ 0 para todo n, lim fn (1) = 1, mas fn não converge uniformemente
para 1.
Dito isto, a armação feita é consequência do Teorema 7 e do que já observamos no item (b)
do Exercício 17 (página 421), considerando-se um intervalo compacto [a, b] tal que K ∪ {c} ⊂
[a, b] ⊂ I  por exemplo, [a, b] = [inf K ∪ {c}, sup K ∪ {c}], uma vez que ambos inf K ∪ {c} e
sup K ∪ {c} pertencem a K ∪ {c} (que é compacto), K ∪ {c} ⊂ I e I é um intervalo . Num
tal intervalo, pelo Teorema 7, fn converge uniformemente para uma função cuja derivada é
identicamente nula, ou seja, podemos dizer que fn converge uniformemente para um certo A
real (Corolário 1 do Teorema 7 junto com o Teorema 1, ambos do Capítulo VIII) neste intervalo,
em particular lim fn (c) = A, portanto A=L pela unicidade do limite de uma sequência de
números reais (Teorema 1 do Capítulo IV). Então, fn → L uniformemente em [a, b], conjunto
este que contém K, logo, fn → L uniformemente em K , como queríamos.
x 1 x
Para fn (x) = sen , temos fn (0) = sen 0 = 0 para todo n∈N e fn0 (x) = cos →0
0
n n n
uniformemente (|fn (x)| 6 1/n para todo n). Então, pelo que acabamos de ver, podemos
concluir que fn → 0 uniformemente em cada compacto K ⊂ R. 

10.36 Exercício 36
A sequência de funções
Z 1 fn : [0, 1] → R, onde fn (x) = xn (1 − xn ), converge simplesmente para

zero. Tem-se lim fn (x) dx = 0. Existe lim fn0 (x) = g(x) para todo x ∈ [0, 1] [sic ] mas g não
0 n n
é derivada de função alguma em [0, 1]. Note que a sequência (fn ) cumpre lim(fn0 ) = (lim fn )0
em [0, 1), embora a convergência não seja uniforme aí.

Solução.

1. Converge simplesmente para zero porque fn (1) = fn (0) = 0 para todo n, e

x ∈ (0, 1) ⇒ lim xn (1 − xn ) = 0

(Exemplo 6 e Teorema 6, ambos do Capítulo IV);

2.
Z 1 Z 1
lim fn (x) dx = lim xn − x2n dx
n 0 n 0
1
xn+1 x2n+1
= lim −
n n+1 2n + 1 0
1 1
= lim −
n n+1 2n + 1
= 0;

3. Temos fn0 (x) = nxn−1 − 2nx2n−1 , então fn0 (1) = −n ∀n ∈ N e aqui vemos um problema
0
no enunciado: lim fn (x) = g(x) ∀x ∈ [0, 1] só se aceitarmos que g não é uma função real
n
(g(1) = −∞?). Fora isso, fn0 (0) = 0 e lim fn0 (x) = 0 se x ∈ (0, 1) por motivo semelhante a
n
10.36. EXERCÍCIO 36 437

um visto na Solução do Exercício 2 (um raciocínio alternativo é usar o Teorema 13, para
an = 1 , ou o Exemplo 32a do Capítulo IV, ambos juntos com o Teorema 15 co Capítulo
IV):
1
lim nxn−1 = lim · nxn = 0,
n n x
n
1 (1/x) 1
pois
n
= → +∞, uma vez que >1 (Exemplo 21, item 3. do Teorema 14 e
nx n x
item 2 do Teorema 6, todos estes resultados do Capítulo IV), donde

lim 2nx2n−1 = 0
n

2n−1
(Teorema 2 do Capítulo IV, porque (2nx )n∈N é subsequência de (nxn−1 )n∈N ), logo,
0
lim fn (x) = 0 se x ∈ (0, 1) (item 1 do Teorema 6, Capítulo IV);
n

4. Se g estivesse denida em 1, e se fosse g(1) 6= 0, poderíamos usar o Corolário do Teorema


5 do Capítulo VIII para concluir que ela não é derivada de nenhuma função, pois o 1 seria
um ponto de descontinuidade de primeira espécie de g;

5. A convergência lim(fn0 ) = (lim fn )0 0


não é uniforme em [0, 1), porque fn (1) = −n para
0
todo n e cada fn é contínua em 1, então sempre podemos encontrar x ∈ (0, 1) tal que
fn0 (x) esteja próximo o suciente de −n, em particular, para cada n ∈ N, existe x ∈ (0, 1)
0
tal que fn (x) 6 −1/2.

Uma outra forma de justicar que a sequência (fn0 )n∈N


não é uniformemente convergente
0
em [0, 1) é através do Corolário do Teorema 1: como cada fn é contínua em [0, 1], se
(fn0 )n∈N convergisse uniformemente em [0, 1), então (fn0 )n∈N convergeria uniformemente
0
em [0, 1], e, em particular, a sequência (fn (1))n∈N = (−n)n∈N convergeria (mas ela não
converge, de acordo com a denição de convegência da página 108).

Por causa dos itens 3 e 4 acima, acredito que tenha havido um erro de tipograa na redação
n n
do enunciado do Exercício, talvez tenha faltado dividir x (1 − x ) por n: para a sequência
n n
de funções gn : [0, 1] → R, com gn (x) = x (1 − x )/n, sim, valem todas as propriedades
enunciadas, as quais podem ser mostradas com contas semelhantes às já feitas acima. Quanto
0
a não convergência uniforme de (gn )n∈N em [0, 1) também podemos proceder como no Exemplo
0
4 encontrando o ponto de máximo de cada gn ; assim, considerando, para cada n ∈ N, o número
r
n n−1
xn = , temos (para n > 1)
4n − 2
√ √
r r
2n − 1 n(n − 1)
2n − 1 1 − 1/n 1
gn0 (xn )
n n n n
= 2· · 2
= 2·
· 2

n − 1 2(2n − 1)
n − 1 8(1 − 1/(2n)) 8

n
p
(resultados do Capítulo IV: Exemplos 13 e 14, Teoremas 6 e 8, pois
p √ 2 < n
(2n − 1)/(n − 1) =
n
n
2 + 1/(n − 1) 6 3).
Outro exemplo de sequência que também possui todas as propriedades deste Exercício é
aquela cujon-ésimo termo é denido por xn /n ∀x ∈ [0, 1]. A sequência de suas derivadas não
é uniformemente convergente no intervalo [0, 1) pelo Exemplo 5.
Em ambos estes exemplos podemos justicar que as respectivas sequências de suas derivadas
não são uniformemente convergentes em [0, 1) pelo Corolário do Teorema 1 e pelo Teorema 4:
0 0
se gn fosse uniformemente convergente em [0, 1), então existiria uma função ϕ tal que gn → ϕ
uniformemente em [0, 1], portanto ϕ seria contínua em [0, 1], o que não é verdade. 
438 CAPÍTULO 10. SEQUÊNCIAS E SÉRIES DE FUNÇÕES

Observação. Este Exercício mostra a importância da hipótese da uniformidade da conver-


gência das derivadas no Teorema 7.

10.37 Exercício 37
an x n , |an cn | 6 M
P
Dada uma série de potências sejam c>0 e M >0 tais que para todo
n ∈ N. Prove que (−c, c) está contido no intervalo de convergência da série considerada.

an x n
P
Solução 1. Dado x ∈ (−c, c), vamos mostrar que converge. Na Demonstração do
Teorema de Abel encontramos a inspiração para escrever
 x n
an x n = an c n · .
c
Agora podemos aplicar o Critério da Comparação (Corolário do Teorema 16, Capítulo IV)
escrevendo x n x n
|an xn | = |an cn | · 6 M · ,

c c
|x/c|n
P
onde é uma série convergente  ela é uma série geométrica com |x/c| < 1 , logo
n
P
an x é absolutamente convergente, mais do que queríamos. 

Solução 2. Seja r o raio de convergência dessa série. Queremos mostrar que r > c, se r ∈ R,
ou que r = ∞.
1 p
Sabemos que r= p (r = 0 se (
n
|an |)n∈N for ilimitada, o que, como veremos,
lim sup |ap
n
n|
não é o caso, e r = ∞ se lim sup n |an | = 0). Então vamos mostrar que
p
n 1
lim sup |an | 6 .
c
Temos: √
n
n
p M
|an c | 6 M ∀n ∈ N ⇒ n |an | 6 ∀n ∈ N,
c
√n
M 1 √
n
sendo lim = (pois lim M = 1, pelo Exemplo 13 do Capítulo IV). Em particular, a
n pc c n
sequência (
n
|an |)n∈N (de termos não negativos) é limitada. Agora o resultado desejado segue
1
do Corolário 2 do Teorema 11 do Capítulo IV: dado d > , existe n0 ∈ N tal que
c
√n
M
n > n0 ⇒ < d,
c
portanto, p
n
n > n0 ⇒ |an | < d,
√ 1
logo lim sup n an 6 , como queríamos. 
c

10.38 Exercício 38
Se uma série de potências converge em todos os pontos de um intervalo compacto, a convergência
é uniforme nesse intervalo (mesmo que ele não esteja contido no intervalo de convergência).
10.39. EXERCÍCIO 39 439

Solução. Seja [a, b] este intervalo compacto. Se [a, b] está contido no intervalo de convergência
da série considerada, a convergência é uniforme pelo Teorema 10. Se não está, então o raio de
convergência dela é um número real r igual a b ou igual a −a (b > r ou a < −r implicariam
a existência de um ponto fora do intervalo de convergência no qual acontece a convergência,
contrariando o Teorema 9). Se b = r, então a convergência é uniforme em [a, r] com 0 6 a 6 r e
em [a, r] = [a, 0] ∪ [0, r] com −r < a < 0. Isto por causa do Teorema de Abel, do Teorema 10 e
do seguinte fato (já visto no item (c) do Exercício 17, página 421): se uma sequência de funções
converge uniformemente em V , então esta sequência converge uniformemente em U ∪V .
U e em
Analogamente, se r = −a, então a convergência é uniforme em [−r, b] com −r 6 b 6 0 e em
[−r, b] = [−r, 0] ∪ [0, b] com 0 < b < r (incluindo também, na lista de resultados utilizados, a
primeira observação que segue o Teorema de Abel). Se r = b e r = −a, então a convergência é
uniforme em [−r, r] = [−r, 0] ∪ [0, r]. 

Observação. Que [a, b] está contido em [−r, r], caso a série convirja em todos os pontos de
[a, b] e r seja seu raio de convergência, já foi visto na demonstração do Teorema 12.

10.39 Exercício 39

X xn
Prove que a série não converge uniformemente em toda a reta.
n=0
n!

Solução 1. Isto é consequência do Exercício 26 (página 428). Denindo, para cada n ∈ N,


fn (x) = xn−1 /(n − 1)! 0
(convencionando-se x = 1) para todo x ∈ R, a sequência (fn )n∈N não
p
converge uniformemente para zero; por exemplo, fn ( n−1 (n − 1)!) = 1 para todo n. 

Solução 2. Queremos mostrar que o resto dessa série não converge uniformemente para
zero em toda a reta.

X xn
Sabemos que = ex ∀x ∈ R (Exemplo 31 do Capítulo VIII). Assim, denindo os
n=0
n!
x xn
polinômos pn (x) = 1 + + · · · + ∀n ∈ N ∀x ∈ R e p0 (x) = 1 ∀x ∈ R, esses restos são
1! n!
dados por
rn (x) = ex − pn (x) ∀n ∈ N ∪{0} ∀x ∈ R .
p(x)
Sabemos também que lim = 0 qualquer que seja o polinômio p (Exemplo 19 do
x→+∞ ex
Capítulo VIII). Lembrando do nosso objetivo, a ideia é mostrar que cada polinômio pn está
muito longe da exponencial. Por exemplo, para todo n ∈ N, seja An ∈ R tal que

ex

pn (x) 1
x > An ⇒ x < ⇒ |pn (x)| < ,
e 2 2

então
ex ex
x > An ⇒ |rn (x)| > |ex | − |pn (x)| > ex − =
2 2
(item (iii) do Teorema 2 do Capítulo III).
1
Portanto, dado n ∈ N, para x > max{An , 0}, temos |rn (x)| > . Logo, a sequência (rn )n∈N
2
não converge uniformemente para zero, como queríamos. 
440 CAPÍTULO 10. SEQUÊNCIAS E SÉRIES DE FUNÇÕES

10.40 Exercício 40
(−1)n x2n (1 + x2 ) · xn
P P
Mostre que as séries de potências (1 − x) · e têm ambas raio de
convergência 1.

Solução 1. Pelo Teorema 9, basta vericar que elas convergem se |x| < 1 e divergem se
|x| > 1.
n 2n 2n n
Se |x| > 1, então ambas as sequências (|(−1) x |)n∈N = (|x |)n∈N e (|x |)n∈N tendem
n
ao innito (por exemplo: |x| > M ⇔ n log |x| > log M ⇔ n > log M/ log |x|, se |x| > 1,
(−1)n x2n e
P P n
para um dado M > 0), então x divergem pelo TeoremaP15 do Capítulo
P n 2n
P n 2 n
P n IV.
Se |x| < 1, então (−1) x e x convergem absolutamente, porque |x | e |x| são
séries geométricas neste caso. 

Solução 2. Pelo Teorema 14 o maior intervalo no qual podemos armar que (1 − x) ·


(−1) x e (1 + x2 ) ·
n 2n
P P n
x são séries de potências é o intervalo (−1, 1), porque
P as séries
1P− x e 1 + x2 convergem em toda a reta e os raios de convergência das séries (−1)n x2n e
xn são ambos iguais a 1 (o que pode ser justicado como na solução acima ou pela forma de
se calcular raios de convergência dada pelo Teorema 9).
Em (−1, 1), podemos calcular que séries são essas pelo Teorema 14 ou por resultados já
conhecidos sobre séries numéricas:
X X X
(1 − x) · (−1)n x2n = [(−1)n x2n + (−1)n+1 x2n+1 ] = an x n ,

com a2n = (−1)n e a2n+1 = (−1)n+1 para todo n ∈ N ∪{0}, e


X X X X
(1 + x2 ) · xn = xn + xn+2 = bn x n ,

com b0 = b1 = 1 e bn = 2 para todo n ∈ N ∩[2, +∞). p


Agora calculamos os raios de convergência dessas séries pelo Teorema 9:

n
|anp
| = 1 e
p √
n n
n
|bn | = 2 para todo n ∈ N ∩[2, +∞). Portanto (lembrando que lim 2 = 1), lim n |an | =
p
n
P n
Pn n n
lim |bn | = 1. Logo, os raios de convergência das séries an x e bn x são iguais a 1. 
n

10.41 Exercício 41
an xn . Pondo
P
Seja (−r, r) o intervalo de convergência da série
P n
P sn n= a0 + a1 + · · · + an prove
que, para todo x ∈ (−1, 1) ∩ (−r, r), vale an x = (1 − x) · sn x .

Solução. Fixado x ∈ (−1, 1) ∩ (−r, r), lembrando da série geométrica (Exemplo 24 do Capí-
tulo IV), temos
1 X X  X 
· an x n = xn · an x n
1−x
s n xn
P
e isto é igual a pelo Teorema 14. Segue a tese. 

10.42 Exercício 42
an x n
P
Suponha que an > 0 para todo n, que f (x) = no intervalo (−r, r) e que lim− f (x) = L.
x→r
n
P
Prove que an r = L.
10.43. EXERCÍCIO 43 441

Solução. Está implícito que este r é um número real positivo, por causa da convenção feita
na página 71 (a de que, ao se considerar um intervalo da forma (a, b), é pressuposto que a < b)
an rn não faria sentido se r = ∞. Quanto ao L, podemos ter L ∈ R ou L = ∞.
P
e porque
Vamos começar supondo L ∈ R.
an r n ,
P
Se tivermos a convergência da série poderemos aplicar o Teorema de Abel:

X X
an rn = lim− an xn = lim− f (x) = L.
x→r x→r

an r n
P
Vejamos então que converge.
Para cada n ∈ N, seja o polinômio sn denido por

sn (x) = a0 + a1 x + · · · + an xn ∀x ∈ R .

Como an > 0 para todo n∈N e r > 0, temos

a0 6 a0 + a1 r 6 a0 + a1 r + a2 r2 6 · · · ,

ou seja, a sequência (sn (r))n∈N é monótona não decrescente. Então, para concluirmos a conver-
gência desejada, vamos mostrar que a sequência (sn (r))n∈N é limitada (Teorema 4 do Capítulo
IV).
DadoM > L, seja δ ∈ (0, r) tal que f (x) < M ∀x ∈ (r − δ, r) (o qual existe porque
lim− f (x) = L). Isto implica sn (x) < M para todo x ∈ (r − δ, r) e todo n ∈ N, porque
x→r
lim sn (x) = f (x) e (sn (x))n∈N também é monótona não decrescente para cada x ∈ (0, r).
n→+∞
Portanto sn (r) 6 M ∀n ∈ N, já que sn (r) = lim sn (x) ∀n ∈ N. Isto é, (sn (r))n∈N é limitada,
x→r
como queríamos.
L = ∞, dado M ∈ R, seja δ ∈ (0, r) tal que f (x) > M ∀x ∈ [r − δ, r) e seja n0 ∈ N tal
Se
que sn0 (r − δ) > M , de modo que sn (x) > M para todo n ∈ N ∩[n0 , +∞) e todo x ∈ [r − δ, r)
(observando que cada sn é uma função monótona não decrescente no intervalo (0, r)), portanto,
X
sn (r) > M para todo n ∈ N ∩[n0 , +∞). Logo, an rn = lim sn (r) = +∞. 
n→+∞

an rn =
P
Observação. Com as mesmas contas feitas acima também poderíamos mostrar que
L (no caso em que L ∈ R) por denição, sem aplicar o Teorema de Abel.

10.43 Exercício 43
X X
Sejam ai x i e bj x j séries de potências com raios de convergência r>0 e s>0 respecti-
i>1 j>0
X
vamente. Prove que existe uma série de potências cn x n , com raio de convergência t>0 tal
n>0
X X X
n
que, para todo x ∈ (−t, t), vale cn x = bj ( ai xi )j . Conclua que a composta de duas
n>0 j>0 i>1
funções analíticas é analítica.

Solução.
442 CAPÍTULO 10. SEQUÊNCIAS E SÉRIES DE FUNÇÕES

Primeira Parte. Vamos imitar a Demonstração do Teorema 15. A sugestão de se fazer


isso é dada no primeiro parágrafo do texto Nota sobre funções complexas.
X
Sejam f : (−r, r) → R e g : (−s, s) → R as funções denidas por f (x) = ai xi para todo
i>1
X
j
x ∈ (−r, r) e g(x) = bj x para todo x ∈ (−s, s). Pelo Teorema 14, para cada j > 0, seja
j>0

j X
f (x) = cji xi ∀x ∈ (−r, r)
i>0

(convencionando daqui em diante a igualdade 00 = 1).


Para podermos fazer

X X j X X XX XX XhX i
bj ai x i = bj · cji xi = bj cji xi = bj cji xi = bj cji xi ,
j>0 i>1 j>0 i>0 j>0 i>0 i>0 j>0 i>0 j>0

precisamos:

1. x ∈ (−r, r);

2. f (x) ∈ (−s, s);


X
3. |bj cji xi | convergente, para cada j>0 (Teorema 8), e;
i>0

XX
4. |bj cji xi | convergente (Teorema 8).
j>0 i>0

Para 1 e 2: f (0) = 0 ∈ (−s, s) e f é contínua em 0 (Corolário do Teorema 10), então existe


t1 > 0 tal que (−t1 , t1 ) ⊂ (−r, r) e f (x) ∈ (−s, s) para todo x ∈ (−t1 , t1 ), e; para 3, dado j > 0,
temos
X X
|bj cji xi | = |bj | · |cji xi |,
i>0 i>0
X
onde cji xi é uma série absolutamente convergente se x ∈ (−r, r), uma vez que (−r, r) está
i>0
contido em seu intervalo de convergência (Teorema 9). Com relação a 4 também procedemos
do mesmo modo como na Demonstração do Teorema 15. Assim, denindo

X
ϕ(x) = |ai |xi ,
i>1

cujo raio de convergência é o mesmo de f (pela denição de raio de convergência) e

j X
ϕ(x) = dji xi ∀x ∈ (−r, r),
i>0

para cada j > 0, temos |cji | 6 dji para todo i > 0 X j > 0 (cuja demonstração está
e todo

na página 398 do livro). Isso nos dá a convergência de |cji xi | para cada j > 0 e cada
i>0
10.43. EXERCÍCIO 43 443

i i
x ∈ (−r, r) pelo Critério da Comparação,
X pois |cji x | 6 djij|x | para todo i > 0, para todo j > 0,
todo x ∈ (−r, r) e, para cada j > 0, dji |x|i = ϕ(|x|) converge se |x| < r, além do que
i>0

X X
|cji xi | 6 dji |x|i ∀j > 0 ∀x ∈ (−r, r)
i>0 i>0

(Corolário 1 do Teorema 7, Capítulo IV), donde

X X
|bj cji xi | 6 |bj | dji |x|i ∀j > 0 ∀x ∈ (−r, r).
i>0 i>0

XX X X
Agora teremos a desejada convergência de |bj cji xi | se |bj | · dji |x|i , que é igual
j>0 i>0 j>0 i>0
X j
a |bj | ϕ(|x|) , convergir. Isso nos motiva a denir
j>0

X
ψ(x) = |bj |xj ∀x ∈ (−s, s)
j>0

X X
dji |x|i = ψ ϕ(|x|)

(cujo raio de convergência é o mesmo de g ), de tal modo que |bj | ·
j>0 i>0
XX
i
converge se ϕ(|x|) ∈ (−s, s). Portanto, |bj cji x | converge qualquer que seja o x ∈
j>0 i>0
(−t2 , t2 ), t2 é um número real positivo menor do que ou igual a r tal que ϕ(x) ∈ (−s, s)
onde
para todo x ∈ (−t2 , t2 ). Este t2 existe porque ϕ(0) ∈ (−s, s) e ϕ é contínua em 0.
X
Logo, com as considerações acima, podemos concluir que existem uma série ci xi e um
i>0
X X
i i
t0 ∈ (0, r] tais que: ci x é convergente ∀x ∈ (−t0 , t0 ), ai x ∈ (−s, s) para todo x ∈
i>0 i>1
(−t0 , t0 ) e

X X X j
ci x i = bj ai x i ∀x ∈ (−t0 , t0 ).
i>0 j>0 i>1

No elevador. A questão agora é: sendo t o raio de convergência dessa série de potências,


o qual é não nulo pelo Teorema 9 (pois existe x 6= 0 no qual ela converge, de acordo com as
contas feitas),
X X X j
ci x i = bj ai x i ∀x ∈ (−t, t)?
i>0 j>0 i>1

No livro foi enfatizado que as operações de adição e de multiplicação de séries de potências


podem resultar em séries de potências com raios de convergência maior do que os raios de suas
parcelas ou de seus fatores. Isso nos leva a desconar do que foi pedido para provarmos. Pode
acontecer, por exemplo, t > r ? Investigando contra-exemplos, procuramos por séries f e g tais
−1
que g = f , de tal modo que g ◦ f é a função identidade, uma série cujo raio de convergência
é +∞.
444 CAPÍTULO 10. SEQUÊNCIAS E SÉRIES DE FUNÇÕES

X
Segunda Parte. É falso que existe uma série cn x n com raio de convergência t > 0 tal
n>0
que
X X X j
ci x i = bj ai x i ∀x ∈ (−t, t).
i>0 j>0 i>1

Por exemplo, as séries geométricas


X X X X
ai xi = xi e bj x j = (−1)j xj+1 .
i>1 i>1 j>0 j>0

Seus raios de convergência são r = s = 1 e de


X x
xi = ∀x ∈ (−1, 1)
i>1
1−x
e X x
(−1)j xj+1 = ∀x ∈ (−1, 1),
j>0
1+x
temos
X X j X  X j+1 x
j 1−x
bj ai x i = (−1) xi = x =x
j>0 i>1 j>0 i>1
1 + 1−x
X x X
se x ∈ (−1/2, 1/2), pois |x| < 1/2 implica xi = ∈ (−1, 1). Se cn x n é uma série
i>1
1−x n>0
tal que
X X  X j+1
n j
cn x = (−1) xi ∀x ∈ (−t0 , t0 ),
n>0 j>0 i>1
X
para algum t0 positivo, então cn xn = x para todo x com módulo menor do que min{1/2, t0 },
n>0
X
n
logo cn x = x para todo x∈R (Corolário 2 do Teorema 18), e o raio de convergência de
n>0
X
cn x n é t = +∞. Por outro lado, não é verdadeira a armação:
n>0
X X j+1
j i
(−1) x = x ∀x ∈ R .
j>0 i>1

Terceira parte. Sejam I e J R e a função analítica f : I → R


intervalos abertos em
satisfazendo f [I] ⊂ J . Seja também a função analítica g : J → R e um ponto a ∈ I . Queremos
X
um número t > 0, com (a − t, a + t) ⊂ I , e uma série de potências cn (x − a)n tais que
n>0
 X
g f (x) = cn (x − a)n ∀x ∈ (a − t, a + t).
n>0

Como f é analítica em a e g é analítica em f (a), sejam


X r > 0, comX (a − r, a + r) ⊂ I , s > 0,
 i
j
com f (a) − s, f (a) + s ⊂ J , e as séries de potências ai (x − a) e bj x − f (a) tais que
i>0 j>0
X
f (x) = ai (x − a)i ∀x ∈ (a − r, a + r)
i>0
10.43. EXERCÍCIO 43 445

e X j 
g(x) = bj x − f (a) ∀x ∈ f (a) − s, f (a) + s .
j>0

Pela continuidade de f em a, como f (a) ∈ f (a) − s, f (a) + s , se o t procurado pertence
ao intervalo (0, r] e é tal que f (x) ∈ f (a) − s, f (a) + s para todo x ∈ (a − t, a + t), então
X X j
ai (x − a)i − f (a)

g f (x) = bj
j>0 i>0
X X j
= bj ai (x − a)i ∀x ∈ (a − t, a + t),
j>0 i>1

poisa0 = f (a). Esta observação é quem nos dá o norte de como usar o que zemos na Primeira
Parte. X
A série bj x j converge se x ∈ (−s, s), pois
j>0

X X  j
j

bj x = bj x + f (a) − f (a)
j>0 j>0


converge se x + f (a) ∈ f (a) − s, f (a) + s , o que de fato ocorre se x ∈ (−s, s); x ∈ (−s, s) ⇒
X
ai xi converge se x ∈ (−r, r), o que implica

x + f (a) ∈ f (a) − s, f (a) + s . Analogamente,
i>0
X
i
a convergência de ai x se x ∈ (−r, r). Assim, pela Primeira Parte desta solução (mesmo
i>1
X
que r e s aqui não sejam raios de convergência), sejam t ∈ (0, r] e cn x n tais que, para todo
n>0
X X X X j
i n
x ∈ (−t, t): ai x ∈ (−s, s) e cn x = bj ai x i . Abaixo vamos vericar que
i>1 n>0 j>0 i>1

X
cn (x − a)n = g f (x)

∀x ∈ (a − t, a + t).
n>0

X
Se x ∈ (a − t, a + t), então x − a ∈ (−t, t), de modo que ai (x − a)i ∈ (−s, s) e
i>1

X X X j
cn (x − a)n = bj ai (x − a)i
n>0 j>0 i>1
X X j
= bj ai (x − a)i − f (a)
j>0 i>0
X j
= bj f (x) − f (a)
j>0

= g f (x) ,
X X
pois x ∈ (a − t, a + t) ⇒ x ∈ (a − r, a + r), e ai (x − a)i ∈ (−s, s) ⇒ f (x) = f (a) + ai (x −
i>1 i>1
a)i ∈ f (a) − s, f (a) + s . Logo, g ◦ f é analítica em a.


446 CAPÍTULO 10. SEQUÊNCIAS E SÉRIES DE FUNÇÕES

10.44 Exercício 44
A sequência de funções fn (x) = nx2 possui derivadas limitadas no ponto 0 mas não é equicon-
tínua neste ponto.

Solução. Esta sequência não ser equicontínua no 0 signica que existe ε > 0 tal que, dado
δ > 0, existem x ∈ R e n ∈ N tais que |x| < δ e |nx2 | > ε. Isto acontece porque lim nx2 = +∞
n
qualquer que seja o x real não nulo. 

Observação. Este exercício dialoga com o Exemplo 27.

10.45 Exercício 45
Um conjunto de polinômios de grau 6 k, uniformemente limitado num intervalo compacto, é
equicontínuo nesse intervalo.

Solução.

Nível 1. Seja E um tal conjunto, um conjunto de polinômios de grau menor do que ou


igual a k uniformemente limitado no intervalo compacto [a, b]. Pelo Teorema 23, xada uma
sequência (pn )n∈N de elementos em E, vamos mostrar que (pn )n∈N admite uma subsequência
uniformemente convergente.

No elevador. Um polinômio de grau 6k ca inteiramente caracterizado por seus valo-


res em k + 1 pontos distintos. De modo mais preciso, dado os pares ordenados (t1 , x1 ), . . .,
(tk+1 , xk+1 ), onde os tm 's são dois a dois distintos, existe um único polinômio de grau 6 k cujo
gráco os possui (o que pode ser justicado através da resolução de um sistema de equações).
Isto nos sugere usar k+1 pontos distintos de [a, b] para apoiar nossas contas. A inspiração
para resolver este exercício desta forma veio do Exercício 21 da página 211 do livro Elementos
de Topologia Geral, cuja leitura foi recomendada no Exercício 4, em conexão com ideias da
Álgebra Linear, em particular com as relacionadas ao conceito de base.

Nível 2. Sejam os k+1 pontos t1 , t2 , . . . , tk+1 em [a, b] dois a dois distintos. De modo
semelhante ao feito na Demonstração do Teorema de Cantor-Tychonov (Teorema 21), obtemos
uma subsequência (pni )i∈N tal que a sequência (pni (tm ))i∈N é convergente ∀m ∈ {1, 2, . . . , k + 1}
(usando a limitação de (pn (tm ))n∈N para cada m, primeiro obtemos uma subsequência con-
vergente em t1 ; depois, desta subsequência, obtemos uma subsequência convergente em t2 , e
assim por diante). Armamos que (pni )i∈N é a sequência uniformemente convergente em [a, b]
procurada.

No elevador. Conforme já indicamos no outro elevador, podemos obter k + 1 polinômios


de grau 6k de tal modo que qualquer outro polinômio de grau 6k possa ser escrito em função
deles, de forma conveniente. O fato de esses polinômios serem em número nito vai nos ajudar
em nossas contas.
10.45. EXERCÍCIO 45 447

Nível 3. Pelo Critério de Cauchy (Teorema 1), dado ε > 0, vamos encontrar um i0 ∈ N
tal que

i, j > i0 ⇒ |pni (x) − pnj (x)| < ε ∀x ∈ [a, b].


Sejam q1 , q2 , . . . , qk+1 polinômios de grau 6k tais que

p(x) = p(t1 ) · q1 (x) + p(t2 ) · q2 (x) + · · · + p(tk+1 ) · qk+1 (x) ∀x ∈ R,

qualquer que seja o polinômio p de grau 6 k (as denições destes polinômios estão no Nível 4).
Assim, pela Desigualdade Triangular,

|pni (x) − pnj (x)| 6 |pni (t1 ) − pnj (t1 )| · |q1 (x)| + |pni (t2 ) − pnj (t2 )| · |q2 (x)| + · · · +
+ |pni (tk+1 ) − pnj (tk+1 )| · |qk+1 (x)| ∀x ∈ R .

Disto, para M > 0 tal que |qm (x)| < M para todo x ∈ [a, b] e para todo m ∈ {1, 2, . . . , k +1}
(o qual existe uma vez que cada qm é uma função contínua e [a, b] é compacto, portanto, cada
qm é limitada, Teorema de Weierstrass do Capítulo VII, seção 4), vem

|pni (x) − pnj (x)| 6 |pni (t1 ) − pnj (t1 )| + |pni (t2 ) − pnj (t2 )| + · · · +

+ |pni (tk+1 ) − pnj (tk+1 )| · M ∀x ∈ [a, b].

Deste modo, denimos i0 como o maior dentre os números i1 , i2 , . . . , ik+1 , sendo, para cada
m ∈ {1, 2, . . . , k + 1}, im tal que

ε
i, j > im ⇒ |pni (tm ) − pnj (tm )| < ,
(k + 1)M

cuja existência é garantida pela convergência de (pni (tm ))i∈N . De fato

ε
i, j > i0 ⇒ |pni (x) − pnj (x)| < (k + 1) · · M = ε,
(k + 1)M

para todo x ∈ [a, b], como queríamos.

Nível 4. Para cada m ∈ {1, 2, . . . , k + 1} denimos qm fazendo qm (tm ) = 1 e qm (tl ) = 0


se l ∈ {1, 2, . . . , k + 1} − {m}; mais explicitamente,

(x − t1 )(x − t2 ) · · · (x − tm−1 )(x − tm+1 ) · · · (x − tk+1 )


qm (x) = ,
(tm − t1 )(tm − t2 ) · · · (tm − tm−1 )(tm − tm+1 ) · · · (tm − tk+1 )

para todo x ∈ R. Assim, se p é um polinômio de grau 6 k, então

p(x) = p(t1 ) · q1 (x) + p(t2 ) · q2 (x) + · · · + p(tk+1 ) · qk+1 (x) ∀x ∈ R,

pois

p(t1 ) · q1 (tm ) + p(t2 ) · q2 (tm ) + · · · + p(tm ) · qm (tm ) + · · · + p(tk+1 ) · qk+1 (tm ) = p(tm )

para todo m ∈ {1, 2, . . . , k + 1}. 


448 CAPÍTULO 10. SEQUÊNCIAS E SÉRIES DE FUNÇÕES

Observações.

• Em vista da solução acima, podemos enfraquecer a hipótese de E ser uniformemente


limitado em [a, b] para: existe um subconjunto F ⊂ [a, b] com k + 1 elementos no qual E
é simplesmente limitado;

• Usando a linguagem e resultados da Álgebra Linear, uma outra forma de obter os polinô-
mios q1 , · · · , qk+1 acima é a seguinte. Sendo Pk o espaço vetorial de todos os polinômios
de grau 6 k (polinômios reais de uma variável real), e xados os k + 1 pontos distintos
t1 , · · · , tk+1 , a função ξ : Pk → Rk+1 denida por ξ(p) = (p(t1 ), . . . , p(tk+1 )) ∀p ∈ Pk é
−1
um isomorsmo de espaços vetoriais. Então sua inversa ξ também é um isomorsmo
de espaços vetoriais. O conjunto {e1 , . . . , ek+1 }, onde em é um vetor unitário cuja sua
m-ésima coordenada é 1, é uma base para Rk+1 . Portanto {ξ −1 (e1 ), . . . , ξ −1 (ek+1 )} é uma
−1
base para Pk . Por construção, cada polinômio qm = ξ (em ) é tal que qm (tm ) = 1 e
qm (tl ) = 0 se l 6= m;

• Ao se investigar as derivadas dos polinômios de E (se elas forem uniformemente limita-


das, então E é equicontínuo) podem surgir algumas questões interessantes, como: Uma
inclinação da tangente ao gráco de um polinômio de E em algum ponto de [a, b] pode
se tornar tão grande quanto se queira? Se não, dados n ∈ N e M > 0, dentre todos os
polinômios p de grau n tais que |p(x)| < M ∀x ∈ [a, b], como obter aquele polinômio
no qual podemos encontrar a maior inclinação possível que uma tangente ao seu gráco
(restrito ao intervalo [a, b]) pode assumir (supondo que isso aconteça)?

10.46 Exercício 46
Diz-se que uma sequência de funções fn : X → R converge fracamente para uma função
f : X → R quando lim fn (x) = f (x) para cada ponto x ∈ X no qual f é contínua. Seja D ⊂ R
n
denso. Prove que se uma sequência de funções monótonas fn : R → R converge simplesmente
em D para uma função f : R → R então (fn ) converge fracamente para f em R.

(Voltar para a Solução do Exercício 48, Nível 1: página 449.)

Solução. Seja x0 ∈ R um ponto no qual f é contínua. Dado ε > 0, queremos um natural n0


tal que |fn (x0 ) − f (x0 )| < ε para todo n > n0 .
Pela continuidade de f em x0 , seja I = (a, b) um intervalo aberto ao qual x0 pertence
tal que f (x) ∈ J := (f (x0 ) − ε, f (x0 ) + ε) para todo x ∈ I e, pela densidade de D em I ,
sejam x1 ∈ D ∩ (a, x0 ) e x2 ∈ D ∩ (x0 , b), de modo que fn (x0 ) pertence ao intervalo fechado
Jn de extremos fn (x1 ) e fn (x2 ), pela monotonicidade de cada fn . De fn (x1 ) → f (x1 ) ∈ J
e fn (x2 ) → f (x2 ) ∈ J , sejam os naturais n1 e n2 tais que fn (x1 ) ∈ J para todo n > n1 e
fn (x2 ) ∈ J para todo n > n2 , isto é, Jn ⊂ J e, consequentemente, fn (x0 ) ∈ J para todo
n > max{n1 , n2 }. Logo, max{n1 , n2 } é um n0 desejado. 

10.47 Exercício 47
|x|
Seja f (x) = x+ se x 6= 0 e f (0) = 0. Obtenha uma sequência de funções contínuas crescentes
x
fn : R → R que converge para f em R −{0}, mas (fn (0)) não converge.
10.48. EXERCÍCIO 48 449

Solução. Vamos denir f2n−1 como a função que liga os pares ordenados (0, −1) e (1/n, 1 +
1/n) com um segmento de reta e que coincide com f fora do intervalo [0, 1/n] e a f2n como a
que liga os pares ordenados (−1/n, −1 − 1/n) e (0, 1) com um segmento de reta e que coincide
com f fora do intervalo [−1/n, 0], isto é,


 x−1 se x < 0, 
 1
f2n−1 (x) = (2n + 1)x − 1 se x ∈ 0, ,

 n
x+1 se x > 1/n

e 

   x−1 se x < −1/n, 
 1 1 1
f2n (x) = (2n + 1) x + −1− se x ∈ − ,0 ,

 n n n
x+1 se x > 1/n,

para todo n ∈ N. Todos os termos desta sequência são crescentes e contínuos, fn → f sim-
plesmente em R −{0} e (fn (0))n∈N não converge, porque f2n−1 (0) = −1 e f2n (0) = 1 para todo
n.
Assim, dialogando com o exercício anterior, temos uma sequência de funções monótonas
que converge simplesmente em D = R −{0}, denso em R, para uma função f e que converge
fracamente, mas não simplesmente em R, para f . 

10.48 Exercício 48
Uma sequência simplesmente limitada de funções monótonas fn : R → R possui uma sub-
sequência que converge fracamente para uma função monótona f : R → R, a qual podemos
tomar contínua à direita. [ Sugestão : use Cantor-Tychonov e o Exercicio 46.]

Solução.

Nível 1. D de números reais


Para seguir a sugestão, começamos xando um conjunto
enumerável e denso em R (por exemplo, D = Q). Como a sequência (fn |D)n∈N é simplesmente
limitada e D é enumerável, por Cantor-Tychonov, seja N ⊂ N innito tal que (fn |D)n∈N é
simplesmente convergente. Isto nos dá uma função g : D → R denida por g(x) = lim fn (x)
n∈N
para todo x ∈ D. Pelo Exercício 46 (página 448), a subsequência (fn )n∈N converge fracamente
para qualquer extensão de g com domínio igual a R. Primeiro vamos estendê-la para uma função
monótona f :R→R (Nível 2.1) e, em seguida, vamos fazer as modicações necessárias em f
para que ela se torne contínua à direita, preservando sua monotonicidade e a convergência
fraca de (fn )n∈N para f (Nível 2.2).

Nível 2.1. Para estender g a uma função monótona, armamos que g já é uma função
monótona (Nível 3.1). Deste modo, semelhante ao que acontece no Teorema 12 do Capítulo
VI, ambos os limites laterais lim g(x) e lim− g(x) existem qualquer que seja o número real a
x→a+ x→a
(Nível 3.2) e podemos considerar qualquer um deles para denir f . Façamos

f (a) = g(a) ∀a ∈ D e f (a) = lim+ g(x) ∀a ∈ R −D.


x→a

A justicativa do porquê f é monótona está no Nível 3.3.


450 CAPÍTULO 10. SEQUÊNCIAS E SÉRIES DE FUNÇÕES

No elevador. Agora precisamos corrigir f para alcançar uma função monótona h con-
tínua à direita e limite fraco da subsequência (fn )n∈N . Nos pontos onde f é contínua não há
problema  se ela é contínua num ponto, então, em particular, ela é contínua à direita neste
ponto  ; o problema está nos pontos nos quais ela não é contínua. Pela monotonicidade, o
perigo de transformar um ponto de descontinuidade num ponto de continuidade alterando o
valor de f nele não existe. Então vamos poder mexer nestes pontos sem medo de mudar algo
na convergência fraca.

Nível 2.2. Seja C o conjunto dos pontos nos quais a função monótona f : R → R é
contínua. Armamos que se uma função h:R→R coincide com f em C, então o conjunto
dos pontos nos quais h é contínua está contido em C (Nível 3.4). Consequentemente, uma
sequência que converge fracamente para f também converge fracamente para h se h|C = f |C .
Então denimos h fazendo

h(a) = f (a) ∀x ∈ C e h(a) = lim+ f (x) ∀a ∈ R −C


x→a

(Nível 3.5). Esta h é monótona (Nível 3.6) e contínua à direita (Nível 3.7).

Nível 3.1. O tipo de monotonicidade da função g : D → R, denida por g(x) =


lim fn (x), é o mesmo predominante na subsequência (fn )n∈N . De modo mais preciso, sejam
n∈N
N1 = {n ∈ N ; fn é não decrescente} e N2 = {n ∈ N ; fn é não crescente}. Como N1 ∪ N2 = N
e N é innito, um dos dois, N1 ou N2 , é innito. Para xar as ideias, suponhamos N1 innito.
Assim, se x < y são pontos em D , então g(x) 6 g(y) porque:

fn (x) 6 fn (y) ∀n ∈ N1 ⇒ lim fn (x) 6 lim fn (y),


n∈N1 n∈N1

onde lim fn (x) = g(x) e lim fn (y) = g(y) (Corolário 1 do Teorema 7 e Teorema 2, ambos, do
n∈N1 n∈N1
Capítulo IV). Logo, g é não decrescente se N1 é innito e, analogamente, g é não crescente se
N2 é innito (se N1 e N2 são innitos, então g é constante).

Nível 3.2. Se g : D → R é monótona, então ambos os limites laterais lim+ g(x) e lim− g(x)
x→a x→a
existem para qualquer a ∈ R, não pelo Teorema 12 do Capítulo VI em si, porque não temos
a limitação de g, mas pela ideia de sua demonstração usando o fato de que D é um conjunto
ilimitado inferior e superiormente: se g é não decrescente, então o conjunto {g(x); x ∈ D, x > a}
é limitado por baixo por g(b), onde b é um elemento de D menor do que a, e {g(x); x ∈ D, x < a}
é limitado por cima por g(c), com c ∈ D , c > a; se g é não crescente, então os conjuntos
{g(x); x ∈ D, x > a} e {g(x); x ∈ D, x < a} são, respectivamente, limitado por cima por g(b) e
limitado por baixo por g(c), com b, c ∈ D , b < a < c; de qualquer modo podemos considerar ou
o ínmo ou o supremo (de acordo com a conveniência) desses conjuntos e proceder do mesmo
modo como na Demonstração do Teorema 12 do Capítulo VI.

Nível 3.3. a e b, com a < b. Para a f denida no Nível 2.1, vamos mostrar
Sejam os reais
que f (a) 6 f (b) se g é não decrescente e f (a) > f (b) se g é não crescente (g é monótona pelo
Nível 3.1). Pela denição de f , vamos fazer isto considerando as possibilidades quanto aos
pontos a e b pertencerem ou não a D .
Se a, b ∈ D , então f (a) = g(a) e f (b) = g(b). Daí f (a) 6 f (b) se g é não decrescente e
f (a) > f (b) se g é não crescente.
10.48. EXERCÍCIO 48 451

Se a ∈ D e b ∈/ D, então f (a) = g(a). Se g é não decrescente, então f (b) = inf{g(x); x ∈


D, x > b} e g(a) 6 f (b), porque g(a) 6 g(x) ∀x ∈ D ∩ (b, +∞). Se g é não crescente, então
f (b) = sup{g(x); x ∈ D, x > b} e g(a) > f (b), porque g(a) > g(x) ∀x ∈ D ∩ (b, +∞).
Se a ∈/ D e b ∈ D, então f (b) = g(b). Se g é não decrescente, então f (a) = inf{g(x); x ∈
D, x > a} e g(b) > f (a), porque g(b) ∈ {g(x); x ∈ D, x > a}. Se g é não crescente, então
f (a) = sup{g(x); x ∈ D, x > a} e g(b) 6 f (a), porque g(b) ∈ {g(x); x ∈ D, x > a}.
Se a ∈
/Deb∈ / D, então, tomando c ∈ D ∩ (a, b), temos

f (a) = inf{g(x); x ∈ D, x > a} 6 g(c) 6 inf{g(x); x ∈ D, x > b} = f (b)

se g é não decrescente e

f (a) = sup{g(x); x ∈ D, x > a} > g(c) > sup{g(x); x ∈ D, x > b} = f (b)

se g é não crescente. Segue a tese.

Nível 3.4. Aqui temos uma função monótona f : R → R e o conjunto C de números reais
nos quais f é contínua. Sendo h : R → R uma função tal que h|C = f |C e C1 o conjunto
dos pontos nos quais h é contínua, vamos mostrar que R −C ⊂ R −C1 (o que é equivalente a
C1 ⊂ C pela propriedade C2 da operação de tomar complementares da página 10 do livro).
Vamos fazer isto pelo Teorema 6 do Capítulo VI, encontrando duas sequências (xn ) e (yn ) que
convergem para a e tais que lim h(xn ) 6= lim h(yn ).
f ser monótona, armamos que C é denso em R (Nível 4.1). Assim, dado
Pelo fato de
a ∈ R −C , sejam as sequências (xn ) e (yn ) em C tais que xn < a < yn para todo n e lim xn =
lim yn = a, de modo que

lim f (x) = lim f (xn ) = lim h(xn )


x→a− n→+∞ n→+∞

e
lim f (x) = lim f (yn ) = lim h(yn )
x→a+ n→+∞ n→+∞

(o que nos permite considerar os limites laterais lim f (x) e lim+ f (x) é o Teorema 9 do Capítulo
x→a− x→a
VII; as igualdades lim− f (x) = lim f (xn ) e lim+ f (x) = lim f (yn ),
análogas ao Teorema 6
x→a n→+∞ x→a n→+∞
do Capítulo VI, são legitimadas por uma observação feita após o Teorema 10 do Capítulo VI),
onde
lim f (x) 6= lim+ f (x)
x→a− x→a

(Nível 4.2). Portanto, o limite de h(x) conforme x se aproxima de a não existe (Teorema 6 do
Capítulo VI), logo, h não é contínua em a, como queríamos.

Nível 3.5. O argumento do Nível 3.2 também justica a existência dos limites laterais de
f, f é monótona pelo Nível 3.3 e C é um conjunto ilimitado inferior e superiormente por ser
denso em R (Nível 4.1). Em particular, como estamos lidando apenas com os limites laterais
à direita, bastava a ilimitação inferior de C. Outra forma de justicar a existência dos limites
laterais de f em R −C é aplicando o Teorema 9 do Capítulo VII.

Nível 3.6. Pela densidade de C em R (Nível 4.1), o motivo de h ser monótona é o mesmo
de f, ou seja, mutatis mutandis, o Nível 3.3 também justica a monotonicidade de h.
452 CAPÍTULO 10. SEQUÊNCIAS E SÉRIES DE FUNÇÕES

Nível 3.7. Queremos mostrar que a h denida no Nível 2.2 é contínua à direita. Vamos
aplicar o Exercício 8 do Capítulo VI (página 210;h é monótona pelo Nível 3.6). Por causa da
densidade de C em R (Nível 4.1), dado a ∈ R, seja (xn )n∈N uma sequência de pontos em C tal
que xn > a para todo n e lim xn = a. Se a ∈/ C,
n→+∞

h(a) = lim+ f (x) = lim f (xn ) = lim h(xn )


x→a n→+∞ n→+∞

(respectivamente: pela denição de h(a); por uma observação feita após o Teorema 10 do
Capítulo VI aplicada ao Teorema 6 do mesmo capítulo, e; porque f (xn ) = h(xn ) para todo
n ∈ N, uma vez que xn ∈ C ). Se a ∈ C,

h(a) = f (a) = lim f (xn ) = lim h(xn )


n→+∞ n→+∞

(a segunda igualdade pelo Teorema 4 do Capítulo VII). Logo,

h(a) = lim+ h(x) ∀a ∈ R,


x→a

pelo Exercício 8 do Capítulo VI.

Nível 4.1. Dada uma função monótona f : R → R, o conjunto C dos pontos nos quais
f é contínua é denso em R porque R −C é enumerável (Corolário do Teorema 11 do Capítulo
VII), enquanto qualquer intervalo aberto de R é não enumerável (Corolário 1 do Teorema 6 do
Capítulo III; um intervalo aberto é não degenerado).

Nível 4.2. Dada a função monótona f : R → R, podemos fortalecer o Teorema 9 do


Capítulo VII com a seguinte armação:

Se um número real a é um ponto de descontinuidade de f , então os limites laterais


lim f (x) e lim− f (x), além de existirem, são distintos.
x→a+ x→a

Solução. Sejam M = lim− f (x) e L = lim+ f (x). Se f é não decrescente, sabemos que
x→a x→a
M = sup{f (x); x < a} e L = inf{f (x); x > a}, de modo que M 6 f (a) 6 L. Portanto, M = L
implica M = f (a) = L, ou seja, a continuidade de f em a. Analogamente, se f é não crescente,
então

L = sup{f (x); x > a} 6 f (a) 6 inf{f (x); x < a} = M

e M = L ⇒ L = f (a) = M . Portanto, se f não é contínua em a, então M 6= L, como


queríamos. 

10.49 Exercício 49
Seja (fn ) uma sequência equicontínua e simplesmente limitada num compacto X ⊂ R. Se toda
subsequência uniformemente convergente em X tem o mesmo limite f : X → R, então fn → f
uniformemente em X.
10.50. EXERCÍCIO 50 453

Solução. Vamos trabalhar com a contrapositiva deste resultado. (fn )n∈N não converge Se
uniformemente para f em X, M de números
então podemos obter um subconjunto innito
naturais com a seguinte propriedade: existe ε > 0 tal que, para todo n ∈ M , existe x ∈ X tal
que |fn (x) − f (x)| > ε. Disto decorre que nenhuma subsequência de (fn )n∈M converge unifor-
memente para f em X . Por outro lado, (fn )n∈M é uma sequência equicontínua e simplesmente
limitada em X , um conjunto compacto, portanto, por Ascoli-Arzelá, existe N ⊂ M innito tal
que (fn )n∈N converge uniformemente em X . Esta é uma subsequência de (fn )n∈N que converge
uniformemente em X , mas não para f . Segue a tese. 

10.50 Exercício 50
Dê exemplo de uma sequência equicontínua de funções fn : (0, 1) → (0, 1) que não possua
subsequência uniformemente convergente em (0, 1).

1
Solução. Numa primeira abordagem, podemos começar com as funções dadas por . O
nx
problema é que, como funções de (0, 1) em (0, 1), elas não estão bem denidas (suas imagens
saem do intervalo (0, 1)). Podemos corrigir isso as tornando constantes nos intervalos que
começam no 0 e que terminam num ponto um pouco mais à direita do 1/n. Assim, dado n∈N
(n > 2), denimos

n − 1/2 1
, 0<x< ,

 se e
n − 1/2

fn (x) = n
1 1

 , se 6 x < 1,
nx n − 1/2

(e f1 (x) = 1/2 para todo x ∈ (0, 1)). Essa sequência é equicontínua em a ∈ (0, 1) porque

1 1 1 1 1 1 1
|fn (x) − fn (a)| 6 − = · − 6 − ∀x ∈ (0, 1);
nx na n a x a x
assim, pela continuidade da função dada por 1/x em a, dado ε > 0, o δ > 0 que atesta a
equicontinuidade de (fn ) em a é o δ>0 tal que

1 1
x ∈ (0, 1), |x − a| < δ ⇒ − < ε .
a x
Qualquer subsequência de (fn ) converge simplesmente para a função identicamente nula,
1 1
porque, xado x ∈ (0, 1), lim = 0 e existe n0 ∈ N tal que 6 x se n > n0 .
n→+∞ nx n − 1/2
Portanto, se uma subsequência convergisse uniformemente, essa convergência só poderia ser
para 0, mas não é o que acontece, pois, para todo n ∈ N, existe x ∈ (0, 1) tal que fn (x) =
n − 1/2 1
, número este que sempre é maior do que ou igual a .
n 2
Então esta sequência é equicontínua e uniformemente limitada, ela mostra a necessidade da
compacidade do domínio das funções no teorema de Ascoli-Arzelá. De quebra, esta sequência
também converge simplesmente. 

10.51 Exercício 51
Dada uma sequência de funções duas vezes deriváveis fn : I → R, suponha que fn → f
0
simplesmente em I , que (fn (a)) é limitada para um certo a ∈ I e que (fn00 ) é uniformemente
454 CAPÍTULO 10. SEQUÊNCIAS E SÉRIES DE FUNÇÕES

limitada em I. Prove que F [sic ] ∈ C 1 .

Solução. Vamos supor que este I é um intervalo, por causa da denição de derivabilidade
contínua (páginas 268 e 269)  na qual, para dizer se uma tal função é de classe C 1 , é assumido
a priori que seu domínio é um intervalo  e também porque este resultado é falso caso I não
seja um intervalo (se estendermos a denição de derivabilidade contínua mesmo para quando o
domínio da respectiva função não é um intervalo)  exemplo: I = R −{1/n; n ∈ N}, fn (x) =
min{1, nx} se x ∈ I ∩ [0, +∞) e fn (x) = max{−1, nx} se x ∈ I ∩ (−∞, 0).

Nível 1. Vamos mostrar que f é derivável (Nível 2.1) e que f0 é contínua (Nível 2.2).

No elevador. Note a semelhança deste exercício com o Teorema 7. Ao contrário do que


0
tínhamos lá, aqui não temos a convergência uniforme da sequência (fn )n∈N , porém ainda sim
podemos usar o resultado do Teorema 3 (um dos pontos centrais da Segunda Demonstração do
Teorema 7), por causa de sua Observação 2 (sobre o limite uniforme em relação a n).

Nível 2.1. Fixado x0 ∈ I , vamos mostrar que existe o limite lim fn0 (x0 ) e que
n→+∞

f (x) − f (x0 )
lim fn0 (x0 ) = lim .
n→+∞ x→x0 x − x0
Armamos que
fn (x) − fn (x0 )
lim = fn0 (x0 )
x→x0 x − x0
uniformemente em relação a n (Nível 3). Além disto,

fn (x) − fn (x0 ) f (x) − f (x0 )


lim =
n→+∞ x − x0 x − x0

para todo x ∈ I − {x0 }, porque fn → f simplesmente. Deste modo, pela Observação 2 do


Teorema 3 (x0 é ponto de acumulação de I − {x0 }), existe o limite lim fn0 (x0 ) e
n→+∞

fn (x) − fn (x0 )
lim fn0 (x0 ) = lim lim
n→+∞ n→+∞ x→x0 x − x0
fn (x) − fn (x0 )
= lim lim
x→x0 n→+∞ x − x0
f (x) − f (x0 )
= lim ,
x→x0 x − x0
como queríamos.

Nível 2.2. Além de f0 existir e ser limite simples da sequência (fn0 )n∈N (Nível 2.1), temos

lim fn0 (x) = fn0 (x0 )


x→x0

00
uniformemente em relação a qualquer que seja o x0 ∈ I , por causa da limitação de (fn )n∈N
n
00 0
(Exemplo 27; a limitação uniforme de (fn ) implica a equicontinuidade de (fn )). Assim, dado
10.51. EXERCÍCIO 51 455

x0 ∈ I , pela Observação 2 do Teorema 3,

lim f 0 (x) = lim lim fn0 (x)


x→x0 x→x0 n→+∞
= lim lim fn0 (x)
n→+∞ x→x0
= lim fn0 (x0 )
n→+∞
= f 0 (x0 ),

ou seja, f0 é contínua em x0 .

Nível 3. Seja x0 ∈ I . Dado ε > 0,


queremos δ > 0 tal que

fn (x) − fn (x0 ) 0

x ∈ I, 0 < |x − x0 | < δ ⇒ − fn (x0 ) < ε ∀n ∈ N .
x − x0

Pelo Teorema do Valor Médio, para cada x ∈ I diferente de x0 e para cada n ∈ N, seja bx,n
pertencente ao intervalo aberto de extremos x0 e x tal que

fn (x) − fn (x0 )
= fn0 (bx,n ).
x − x0
Para δ>0 tal que

x ∈ I, |x − x0 | < δ ⇒ |fn0 (x) − fn0 (x0 )| < ε ∀n ∈ N


0
((fn ) é equicontínua pelo Exemplo 27 uma vez que (fn00 ) é uniformemente limitada), temos

x ∈ I, 0 < |x − x0 | < δ ⇒ |bx,n − x0 | < δ ∀n ∈ N



0 0
fn (x) − fn (x0 ) 0

⇒ ε > |fn (bx,n ) − fn (x0 )| =
− fn (x0 ) ∀n ∈ N,
x − x0

como queríamos. 

Observações.

0
1. Na solução acima não foi usada a limitação de (fn (a))n∈N . Nos Níveis 2.1 e 3 cou
00
demonstrado que fn → f simplesmente e (fn ) uniformemente limitada implicam a con-
0 0 0
vergência simples de (fn ). Se (fn ) é simplesmente convergente, em particular, (fn (x)) é
0
limitada para todo x ∈ I , e, mais particularmente ainda, existe a ∈ I tal que (fn (a)) é
0
limitada. Sendo assim, a hipótese da existência de um a tal que (fn (a)) é limitada pode
ser dispensada;

2. Se também tivéssemos a compacidade de I (além de I ser um intervalo), então, com


0 00
essa compacidade, com a limitação de (fn (a)) e com a limitação uniforme de (fn ) (com
0
o auxílio do Teorema do Valor Médio), poderíamos concluir que (fn ) é uniformemente
0 0 0 0 00 0
limitada (|fn (x)| 6 |fn (x) − fn (a)| + |fn (a)| = |fn (b)| · |x − a| + |fn (a)|, sendo o b dado
0
pelo TVM). Com isso, com a equicontinuidade de (fn ) (provinda da limitação uniforme
00
de (fn )) e com a compacidade de I , poderíamos aplicar o Teorema de Ascoli-Arzelá, e o
0
resultado desejado seguiria pelos Teoremas 7 e 4 (cada fn é contínua porque é derivável,
Teorema 1 do Capítulo VIII);
456 CAPÍTULO 10. SEQUÊNCIAS E SÉRIES DE FUNÇÕES

3. Com a compacidade de I (I ainda um intervalo), e com a convergência pontual de (fn0 ),


o resultado também segue pelo Exemplo 29 (página 411) e pelos Teoremas 7 e 4 (e 1 do
Capítulo VIII);

4. Talvez não seja muito seguro quebrar I em (eventualmente innitos) compactos, por causa
da observação feita após o Corolário do Teorema 7 do Capítulo VII (página 228);

5. Pela observação feita após a denição de equicontinuidade uniforme, a limitação uniforme


00 0 0
da sequência (fn ) implica mais do que a equicontinuidade de (fn ). De fato, (fn ) é uni-
formemente equicontínua. E aqui voltamos para algo que já foi comentado na Solução do
0
Exercício 22 do Capítulo VIII (página 301). Se fn é uniformemente contínua, então fn é
uniformemente derivável. Então será que essa derivabilidade uniforme é uniforme para
toda a sequência (fn )? Ou, para usar outra expressão, será que a sequência (fn ) é equi-
uniformemente derivável? Revisitando a Demonstração do Teorema 8 do Capítulo VIII
e levando em conta a respectiva observação feita na Solução do Exercício 22 do Capítulo
VIII (no último parágrafo do elevador), vemos que a resposta para essa pergunta é sim.
De modo mais preciso, podemos armar que, dado ε > 0, existe δ>0 tal que


fn (x + h) − fn (x) 0

(∗) 0 < |h| < δ ⇒ − fn (x) < ε, se x ∈ I, x + h ∈ I e n∈N
h

(o delta que mostra isso é o mesmo delta que mostra a equicontinuidade uniforme de
(fn0 )). Resumindo, temos

(fn00 )n∈N uniformemente limitada =⇒ (fn0 )n∈N uniformemente equicontínua

=⇒ (fn )n∈N equi-uniformemente derivável.

Uma implicação dessa uniformidade na derivabilidade uniforme está expressa na armação

fn (x) − fn (x0 )
existe lim = fn0 (x0 ) uniformemente em relação a n, para todo
x→x0 x − x0
x0 ∈ I ,

utilizada na solução do exercício acima.

Fazendo n tender a innito na implicação (∗)


(o limite entra no módulo, Exercício 1 do
Capítulo IV, página 117), e usando as convergências pontuais fn → f e fn0 → f 0 , podemos
concluir que f também é uniformemente derivável, e, então, pela mesma demonstração
1
do Teorema 8 do Capítulo VIII, f é de classe C ;

6. Portanto (levando também em conta aquela observação feita na Solução do Exercício 22


do Capítulo VIII, mencionada mais acima), podemos trocar o enunciado do Teorema 8
do Capítulo VIII para:

Uma função f : I → R (sendo I um intervalo) é uniformemente derivável se, e


somente se, f 0 : I → R é uniformemente contínua.

Talvez o autor do livro tenha optado em trazer esse teorema com [a, b] no lugar de I para
1
substituir o trecho sublinhado pela expressão é de classe C , o que torna o enunciado
mais conciso.
10.52. EXERCÍCIO 52 457

10.52 Exercício 52
Dada uma sequência de funções k + 1 vezes deriváveis fn : I → R, suponha que existam
(k)
a0 , . . . , ak ∈ I e c > 0, tais que |fn (a0 )| 6 c, |fn0 (a1 )| 6 c, . . . , |fn (ak )| 6 c para todo n ∈
(k+1)
N e que a sequência (fn )n∈N seja uniformemente limitada em I . Prove que existe uma
subsequência (fni ) que converge, juntamente com suas k primeiras derivadas, uniformemente
em cada parte compacta de I .

Solução.

(k)
Conversando. Num primeiro momento extraímos a equicontinuidade de (fn ), por causa
(k+1) (i)
da limitação uniforme de (fn ), e um conjunto innito N ⊂ N tal que (fn (ai ))n∈N seja
convergente ∀i ∈ {0, 1, . . . , k}. Uma eventual angústia de não termos usado toda a hipótese de
(i)
as sequências (fn (ai ))n∈N serem limitadas por um mesmo c (para obter N importou apenas a
limitação de cada uma dessas sequências) pode começar a ser diluída quando observamos que,
neste caso, serem limitadas e serem limitadas por um mesmo c acabam sendo coisas equivalentes.
Isso de termos a convergência num número nito de pontos também aconteceu na nossa so-
lução do Exercício 45. Inspirados por ela, tentamos escrever cada função como uma combinação
de um número nito de funções contínuas. Porém não há muitas esperanças de conseguirmos
alcançar esse objetivo do mesmo modo como lá, porque aqui não temos a nitude da dimen-
são da base em questão, um fato importante naquela solução. Por outro lado, a Fórmula
de Taylor com resto de Lagrange (Teorema 10 do Capítulo VIII) até nos dá uma maneira de
relacionar numa equação só uma função com suas k + 1 derivadas, mas, para tanto, esta função
k
deve ser de classe C e todas as funções nesta fórmula são aplicadas num mesmo ponto. Neste
momento estamos no terreno do Teorema do Valor Médio.
(k)
Indo para uma outra abordagem, trabalhemos com a sequência (fn ). Como não precisamos
nos preocupar com o conjunto N
obtido no primeiro parágrafo, o esqueçamos por ora, supondo
(k)
N= N. Tendo a convergência de (fn (ak )), podemos tentar seguir o mesmo espírito da Segunda
Demonstração do Teorema 7, relacionando qualquer outro ponto x de I com o ak através do
Teorema do Valor Médio; assim escrevemos

fn(k) (x) = fn(k) (ak ) + (x − ak )fn(k+1) (y),


(k)
onde y é um ponto entre x
e ak . Conforme n cresce, a parcela fn (ak ) converge, mas não
(k+1)
sabemos como a parcela fn (y) se comporta, sabemos apenas que a sequência formada por
ela é limitada. Usando essa limitação, se permitirmos que o x se aproxime de ak , teremos a
(k)
convergência de fn (xn ), onde (xn ) é uma sequência em I convergente para ak , e isso pode nos
levar a querer aplicar o Exercício 24. O problema é que, para aplicá-lo, devemos vericar que
a mesma coisa acontece não importa para qual número o x se aproxime.
Uma outra coisa desejável, ainda na igualdade acima, seria termos a convergência de
(k+1)
(fn (y)). Não a temos a priori, mas o Teorema de Cantor-Tychonov nos diz ser possível
obter uma subsequência sua simplesmente convergente num subconjunto enumerável. Aqui
(k)
surge o Teorema 20  o qual nos permite vericar a convergência simples de fn apenas num
conjunto denso , pois sabemos da existência de um enumerável e denso. Para aplicar este
(k)
teorema precisaremos também da equicontinuidade de fn , e isso nós temos. Essa ideia vai
nos dar a convergência uniforme em cada parte compacta de I de alguma subsequência de
(k)
(fn ), a qual vai nos levar à convergência uniforme em cada parte compacta de I da respectiva
(i)
subsequência de cada uma das (fn ) através de sucessivas aplicações do Teorema 7.
458 CAPÍTULO 10. SEQUÊNCIAS E SÉRIES DE FUNÇÕES

Outras tentativas que caram pelo caminho:

(k)
• (fn ) é simplesmente limitada? Se sim, pelo Corolário do Teorema 22 (se I for aberto!),
(k)
(fn ) admite uma subsequência que converge uniformemente em cada parte compacta de
(k)
I . (fn ) é uniformemente limitada? Se sim, repita o processo;
(k+1)
• (fn )n∈N admite uma subsequência uniformemente convergente em cada compacto?
Porque, daí, dá para usar o Teorema 7 também.

(k)
• Com o N acima, (fn )n∈N
converge simplesmente? Se sim, então converge uniformemente
(k−1)
em cada parte compacta de I (Exemplo 29). Daí, como (fn (ak−1 ))n∈N converge, então,
(k−1)
(fn )n∈N converge uniformemente em cada parte compacta (Teorema 7). E assim por
diante.

Obtendo a subsequência. N um conjunto innito de números naturais tal que


Seja
(i)
∀i ∈ {0, 1, . . . , k} (o qual pode ser obtido com uma parte do argu-
(fn (ai ))n∈N é convergente
mento usado na Demonstração do Teorema de Cantor-Tychonov). Seja D = {xp ∈ I; p ∈ N}
um conjunto enumerável e denso em I . Para cada xp em D , p ∈ N, distinto de ak e cada n ∈ N ,
seja yp,n ∈ I tal que
fn(k) (xp ) = fn(k) (ak ) + (xp − ak )fn(k+1) (yp,n ).
A coleção Y yp,n 's é um conjunto enumerável, portanto, pelo Teorema de Cantor-
de todos os
(k+1)
Tychonov, seja N1 ⊂ N innito tal que (fn )n∈N1 converge simplesmente em Y . Veremos
abaixo que N1 nos dá a subsequência procurada, isto é, que: para cada i ∈ {0, 1, . . . , k},
(i)
(fn )n∈N1 converge uniformemente em cada parte compacta de I .

(k)
Vericando. Pela igualdade acima, (fn )n∈N1converge simplesmente em D . Sendo esta
(k)
sequência equicontínua (Exemplo 27) e D denso em I , (fn )n∈N1 converge uniformemente em
cada parte compacta de I (Teorema 20). Dado um compacto K contido em I , seja [a, b] ⊂ I
(k)
tal que {a0 , a1 , . . . , ak−1 } ∪ K ⊂ [a, b]; (fn )n∈N1 converge uniformemente em [a, b] e ak−1 é
(k−1) (k−1)
um ponto em [a, b] tal que (fn (ak−1 ))n∈N1 converge, então, pelo Teorema 7, (fn )n∈N1
(k−2)
converge uniformemente em [a, b], em particular, em K . Pelo mesmo motivo (fn )n∈N1
(k−1)
converge uniformemente em [a, b] e em K : (fn )n∈N1 converge uniformemente em [a, b] e
(k−2)
(fn (ak−2 ))n∈N1 converge, onde ak−2 ∈ [a, b]. E assim sucessivamente. Segue a tese. 

10.53 Exercício 53
Demonstre o Corolário do Teorema 22 para intervalos arbitrários (abertos ou não) I ⊂ R.

Solução 1. Vamos combinar o Teorema de Cantor-Tychonov com o Teorema 20  de modo


semelhante ao que zemos na solução do exercício anterior . O Teorema 6 do Capítulo V
(um conjunto X de números reais admite um subconjunto enumerável e denso em X) vai nos
permitir demonstrar o resultado desejado para qualquer subconjunto X de R.
Seja X (fn ) uma sequência equicontínua e simplesmente
um conjunto de números reais. Seja
limitada de funções denidas em X . Seja D ⊂ X enumerável e denso em X ; (fn ) é simples-
mente limitada em D , então (fn ) admite uma subsequência (fni )i∈N tal que, para todo x ∈ D ,
(fni (x))i∈N é uma sequência convergente (Cantor-Tychonov). Esta (fni )i∈N é uma sequência
10.53. EXERCÍCIO 53 459

equicontínua de funções denidas em X que converge simplesmente no subconjunto D, denso


em X. Logo (fni )i∈N converge uniformemente em cada parte compacta de X (Teorema 20). 

Solução 2. A demonstração do respectivo corolário e o Teorema 22 dão conta de intervalos


das formas[a, b], (a, b), (a, +∞) e (−∞, b). Falta mostrar então que este corolário também vale
se I [a, b), [a, +∞), (a, b] ou (−∞, b].
é da forma
Dentre essas quatro últimas formas, vamos começar supondo que I é fechado à esquerda e
vamos denotar por a o seu ínmo.
Um modo de cair no que já foi demonstrado deste corolário é considerar o intervalo aberto
I0 = I − {a}. Então, uma vez que(fn |I0 )n∈N é equicontínua e simplesmente limitada, podemos
primeiro tomar o conjunto innito M ⊂ N tal que (fn )n∈M converge uniformemente em cada
compacto contido em I0 e, em seguida, tomar N ⊂ M innito tal que (fn (a))n∈N converge (pelo
Teorema 22, porque {a} é compacto, ou pela limitação da sequência (fn (a))n∈N ). O problema
de se fazer isso é que, dado K ⊂ I , não poderemos concluir imediatamente que (fn )n∈N converge
uniformemente em K − {a}, uma vez que, se a ∈ K , existe a possibilidade de K − {a} não ser
compacto.
Vamos remediar esse problema escrevendo I = [a, b] ∪ I0 , onde I0 é um intervalo aberto e
b ∈ I0 . Imitando o que foi feito no parágrafo acima, obtemos a sequência (fni )i∈N que converge
uniformemente em [a, b] e em cada compacto contido em I0 . Fixado um conjunto compacto
K ⊂ I , vamos mostrar que essa subsequência converge uniformemente em K .
Se K ⊂ I0 , então o resultado segue por construção. Se K 6⊂ I0 , seu ínmo α pertence a [a, b].
Se o supremo β de K também não pertence a I0 , então [α, β] ⊂ [a, b] e o resultado segue porque
(fni )i∈N é uniformemente convergente em [a, b] e porque K ⊂ [α, β]. Se β ∈ I0 , então tomamos
γ ∈ I0 ∩ [a, b] tal que γ 6 β e teremos [α, β] = [α, γ] ∪ [γ, β] tal que [α, γ] ⊂ [a, b] e [γ, β] é um
compacto contido em I0 , portanto (fni )i∈N converge uniformemente em [α, β] (usando o critério
de Cauchy, o Teorema 1, a justicativa disso é semelhante à do item (c) do Exercício 17, página
422).
Logo, (fni )i∈N converge uniformemente em cada compacto contido em I, como queríamos.
A demonstração no caso em que I não é aberto e é fechado à direita é análoga. Segue a
tese. 
460 CAPÍTULO 10. SEQUÊNCIAS E SÉRIES DE FUNÇÕES
Referências Bibliográcas
• Ávila, Geraldo Severo de Souza. Análise Matemática para Licenciatura, 3
a
ed. rev. e
ampl. São Paulo: Blucher, 2006

• Ávila, Geraldo Severo de Souza. Introdução à Análise Matemática, 2a ed. rev. São Paulo:
Blucher, 1999

• Engelking, Ryszard. General Topology, revised and completed ed. Berlin: Heldermann,
1989 (Sigma series in pure mathematics; vol. 6)

• Gelbaum, Bernard Russell e Olmsted, John Meigs Hubbell. Counterexamples in Analysis.


Mineola, New York: Dover Publications, Inc.

• Gleason, Andrew Mattei. Fundamentals of Abstract Analysis. Addison-Wesley Publishing


Company, 1966 (Addison-Wesley series in mathematics)

• Guidorizzi, Hamilton Luiz. Um Curso de Cálculo, vol. 4, 5a ed. [reimpr.]. Rio de Janeiro:
LTC, 2015

• Halmos, Paul Richard. Naive Set Theory. Springer (Undergraduate texts in mathematics)

• Herstein, Israel Nathan. Topics in Algebra. Ginn and Company

• Hönig, Chaim Samuel. Aplicações da Topologia à Análise. São Paulo: Editora Livraria
da Física, 2011 (Coleção textos universitários do IME-USP; v. 4)

• Leron, Uri. Structuring Mathematical Proofs. The American Mathematical Monthly, vol.
90, no. 3 (mar., 1983), p. 174-185. Disponível em: https://www.jstor.org/stable/2975544

• Lima, Elon Lages. Curso de Análise, vol. 1, 14a ed. Rio de Janeiro: Associação Instituto
Nacional de Matemática Pura e Aplicada, 2012 (Projeto Euclides)

• Lima, Elon Lages. Curso de Análise, vol. 2, 11a ed. Rio de Janeiro: IMPA, 2015 (Projeto
Euclides)

• Lima, Elon Lages. Elementos de Topologia Geral, 3a ed. 2


a
impr. Rio de Janeiro: Editora
SBM, 2014 (Textos universitários)

• Lima, Elon Lages. Espaços Métricos, 5


a
ed. Rio de Janeiro: IMPA, 2013 (Projeto
Euclides)

• Monteiro, Luiz Henrique Jacy. Elementos de Álgebra. Ao Livro Técnico S. A. e Editora


da Universidade de São Paulo, 1969 (IMPA: Elementos de matemática)

• Roque, Tatiana. História da Matemática : uma visão crítica, desfazendo mitos e lendas.
Rio de Janeiro: Zahar, 2012

461

Você também pode gostar